Você está na página 1de 495

Bidtd

A. Biological:Molecul€s
Cti6'p:'l/f I.
2.
Arnino Acids & P,foteifls
Carbohydrates & Lipids
3. Nucleic Acids
8., EukarJrotic rCelld
ffi.hdtioh in l. Cellular ReproductioR ': , ,, ;, 1 , : ;

C [ili ,',,' 'r'':Vi $eS a. Eukaryotic Chromosomes


b. The Cell Cycte, Mitosis, & Meiosis
2. Cellular & Moiecular Organization
a. Biomembranes & Memhrane Transport
I
lrr
b. Nucleus, Nucleolus, & Ribosomes "

c. Endoplasmicr Reticulurn,,,Col$i Atrrpaf atu s.


Lyqosomes.Fq{o:-liisr}mep,.&vtiioChondlia
d. Microtubules, Microfilaments,
& Tntermedia te Fila ments
C. Prok4ry.atie Cetl$,: ,r ,

l. Cellular Reproduction
a. Prokaryotic Chromosomes
b. The Cell Cycle
2. Cellular & Molecular Organization
a. Biomembranes & Membrane Transport
b. Nucleoid & Ribosomes
D. Viruses
l. Architecture & Cenomes
2. Infection & Cenomic Expression
5. Assembly & Release of Progeny Virons
Practice Passageo &i Angwers

RE -{AffiY
l)n.n v.r.'"8.\ I'
Specializing in MCAT Preparation
Structure & Function in Cells & Viruses
Top lO Section Goals

o? Be familiar with the basic biotoqical molecules of cells and viruses.


The 4 general classes of bioJogical,molecules are_a.mino acids,nucleic acids, carbohydrates, and Iipids.
Be able to recognize their ba-sic differences and know their functional significanie.

av lQotqllg2q
B".bl" t"
standard amino acids that
"r-.,.t" rrnt"" ".
are used to synthesize proteins.

Arg, & His) that can have charged polar side chains.

Understand the difference between mitosis and meiosis at the cellular level.
@v Know the different stages of the cell cycle and how they relate to one another. Be able to describe
the chromosome nirmber and ptoiay level ai any stage of mitosis or meiosis.

Be familiar with the different tvoes of membrane transoort processes.


@w Have a feel for the differences between simple diffusion, facilitated diffusion, active transport, and
bulk transport. Be aware of symports, antiports, uniports, and membrane charge'balance.

Be familiar with the different organelles in a eukaryotic cell.


"? Be aware of how they relate to one another wiihin their cellular environment. Know their general
structural characteiistics (e.g., single membrane, double membrane, contains DNA", etc.).

Be familiar with the functions of eukaryotie oryanelles. --


"? For each of the major organelles in a eukaryotic cell, be ablc to define its [unction in general terms.
Are they involved in metabolism? Cellular packaging and sorting? Replication? Protein synthesis?

ov Know where the different metabolic processes occur in a cell.


The general metabolic reactions of glycoiysis, Krebs cycle, electron transport, and oxidativc
phosphorylation are common to eukaryotes and prokaryotes. Where do they occur in each cell?

oB Know the general differences between Gram-negative and Gram-positive bacteria.


Bacl.erial cells, for the most part, can be divided into Cram- negative and Cram-positive cells. It is
important to have a feel f or the structural differences between the- two cell types.

t? Be familiar with the reproductive cycle of a virus.


The biological diversity within viruses is enormous._ Just have a general idea of ihe strategy that an
enveloped and a non-enveloped virus uses when taking over a host cell.

Know the general differences between eukaryotes, prokaryotes, and viruses.


@v Just as it is important to know which galaxy our solar system is located in, it also important to know
the basic differences between eukaryotic celis, prokarvotic ce11s, and viruses.
Biology Structure & Function in Cells & Viruses Biological Molecules

ffi ffi fi fi llffi Gilff $i i :iiffi iiiiFrot#fi ffi s


Amino Acids
Even though there are more than 400 naturally occurring amino acids, there are a
standqrd set of 20 amino acids that comprise the proteins of all living species. As
we will see, proteins are simply individual amino acid residues linked together
in a head-to-tail fashion. All of the standard 20 amino acids are referred to as cr-
amino acids (i.e., a 2-amino acid), except for proline which is referred to as an o(-
imino acid.

An amino acid is composed of a basic amino group (-NHz), an acidic carboxyl


group (-COOH), a hydrogen (H) atom, and a side chain which is characteristic to o
each amino acid. If no one particular amino acid is being discussed, the side coo
@l
chain is usually designated as -R. The amino group of an amino acid is covalently H,N_ C- H
lc
attached to the cr-carbon atom of the amino acid--hence the name cr-amino acid. R
The a-carbon atom (Figure 6-1) is the carbon atom next to the carboxyl carbon
atom. Figure 6.1
An u,-amino acid.
Each of the 20 standard amino acids has a particular three letter and one letter
abbreviation. For example, proline can be abbreviated either as Pro or as P. Each
amino acid also has a defined molecular weight (M.) and can be placed,
according to its R group (at pH 7.0), into one of three main families of amino
acids. The first family (Figure 6-2) consists of those amino acids with nonpolar R
groups. These amino acids are hydrophobic.

Nonpolar R Groups
HO HO HO HO HO
@tilo orilo orilo @tlO OrilO
-l C- C- O
HrN- HrN- C- C-
I
O H3N- C- C- O H3N- Cs C- O H3N- C- C- O
H cHs H1C- CH CH, HlC_ CH
t- -l
Glycine (Gly)[G] CH: H3C- CH CH,
M. = 75 Alanine (AIa)[A]
t-
CH: CH:
M'= 89
Valine (ValXVl Leucine (Leu)[L] Isoleucine (Ile)[I]
Mr= 117 M.= 131 M,= 131

HO HO HO HO
@rrO
H2N- q- C-
orro @rilO
H3N- C- C-
@rilo
H,N_ C_ C- H3N- C- C> O

U
O O O
'l
CHz
I

d
CH"
l'
S
Proline (Pro)[P]
I
M,. = 115
CH:

Methionine (Met)[M] Phenylalanine (Phe)lFl (Trp)twl


Tryptophan
M'= 149 M,= 165 M,=204

Figure 6-2
Amino acids with nonpolar side chains

Copyright O by The Berkeley Review 3 The Berkeley Review


Specializing in MCAT Preparation
BiOlOgy Structure & Function in Cells & Viruses Biological Molecules

The second family consists of those amino acids with polar uncharged R groups
(Figure 6-3). These amino acids are hydrophilic because they contain side chain
functional groups (e.g., -oH, -sH, or -NHz) that can hydrogen bond with water.

Polar Uncharged R Groups


HO
OrrO HO HO
@rrO @rilO
H3N- C- C- O H3N- C- C- O H3N- C- C- O
I I I

CHz HO_ CH CH,


I I t-
OH CH: SH

Serine (Ser)[S] Threonine (Th)tTl Cysteine (Cys)tcl


M.= l05 M.= ll9 M'= 121

HO HO HO
@rilO @rriO @ruO
HrN- C- C- O 'l C- C-
HrN- O HlN-
-l c- C- O

t'
CH" CHz CH,
I t-
O= C- NHz CH,
t-

a OH
Tyrosine (Tyr)tYl
Asparagine (Asn)[N]
M. = 132
O= C-
Glutamine (Gln)tQl
M' =
NH2

146

M.= l8l

Figure 6-5
Amino acids with uncharged polar side chains.

The third family (Figure 6-4) consists of those amino acids with charged R
groups. Note that two of the amino acids (Asp and Glu) have side chains which
are carboxylic acids and three of the amino acids (Lys, Arg, and His) have side
chains which are amine bases. The side chains of Asp, Glu, Lys, and Arg are
highly ionized at neutral pH. The side chain of His is weakly ionized at neutral
pH. we will see that the degree of ionization of an amino acid depends on its
acidic and basic properties.

Charged R Groups H o HO
@l ilo orilo
H3N- C- C- O
''l C- C_ O
H,N-
HO
CH. @rilo
H O
@T?o
HrN-C-C-O
CHu
I

CH"
l'
CH,
HrN-
-l C- C- O

otilol t' t' CHz


H3N- C- C- O CHz CH, CH,

/,'_\o
tl t- t- I

CH,'O CH, CH, NH


r r-O 6l r@ H-N\/,N-H
O= C-O O= C-O NH: H2N- C= NH2

Aspartate(AspXDl Glutamate (Clu)[E] Lysine (Lys)[K] Arginine (Arg)[R] Histidine (HisXHl


M. = 133 M,= 147 M' = 146 M,.= 174 M' = 155

Figure 6-4
Amino acids with charged side chains.

Copyright @ by The Berkeley Review 4 The Berkeley Review


Specializing in MCAT Preparation
Biology Structure & Function in Cells & Viruses Biological Molecules

Although the standard 20 amino acids can exist in either the D or the L
configuration (with the exception of glycine because it is non-chiral), only the L-
amino acids are found in proteins. It is not known why evolution chose to incor- o o
coo coo
porate L-amino acids into proteins instead of D-amino acids. As an example, the oi
H3N>.C< H>. C<
tO
two non-superimposable mirror images of the o-amino acid alanine are shown in H NHj

Figure 6-5. Note that the o-carbon of alanine is chiral (there are 4 different cHl CHr
substituents attached to that u-carbon). The D isomer and the L isomer of alanine Mirror
are called enantiomers. The D and L isomers of the different amino acids are L-alanine D-alanine
based on the absolute configuration of D and L-glyceraldehyde. [All amino acids
mentioned in these readings will be in the L-configuration, unless otherwise Figure 6-5
stated.l Non-superimposable minor images.

Even though D-amino acids are never found in proteins, they do exist in many
organisms. For example, D-alanine and D-glutamate are found within the rigid
cell walls of some bacteria. The presence of these D-amino acids helps to prevent
the degradation of the bacterial cell wall by specific enzymes called proteases
(more on this later). D-Valine is found in the antibiotic valinomycin, a carrier of
K+ ions across membranes.

Amino acids found within proteins can be modified as shown by the examples
given in Figure 6-6. Two important amino acid modifications can be found in
collagen, the most abundant protein in mammals. Some of the proline and lysine
amino acid residues found within nascent molecules of collagen are
hydroxylated to give -hydroxyproline and S-hydroxylysine, respectively. These
modified amino acids help to give collagen its high tensile strength. Histamine, a
powerful vasodilator, is formed from the decarboxylation of the amino acid
histidine. S-Adenosylmethionine is an activated form of the amino acid
methionine and can methylate protein or nucleic acid substrates.

HO
Orll O H
HrN- C- C- O @r
I

CHz H3N- C-H


I

CHz CHz
o I I

ll o @t
H-C- OH
c-o H1N- CH2
H-N./N
HO OH

4-Hydroxyproline 5-Hydroxylysine Histamine S-Adenosylmethionine

Figure 6-6
Modified amino acids.

Acid-Base Properties
Body fluids in biological systems generally have a pH range of 6.5 to 8.0. This is
considered to be physiological pH. At these pH ranges in the cell the amino and
carboxyl groups of the standard amino acids are ionized. In other words, the cr-
amino group bears a positive charge while the a-carboxyl group bears a negative
charge. The dipolar or zwitterionic nature of the amino acids is due to the fact
that the pKu value for the s-amino terminal is about 9.4 while the pKu for the s-
carboxyl terminal is about 2.2. Since amino acids can act as either an acid or a
base they are referred to as ampholytes. Table 6-1 shows the pKu values for the

Copyright @ by The Berkeley Review D The Berkeley Review


Specializing in MCAT Preparation
BiOlOgy Structure 6r Function in Ceils & Viruses Biological Molecules

o-carboxyl and s-amino groups of a selection of the 20 standard amino acids and
their ionizable side chains. Small pKu values refer to a strong acid while large pKu
values refer to a weak acid.

Amino Acid pKu (u-COOH) pKu (a-NH3+) pKu (side chain groups)
Glycine 2.4 9.8
Alanine 2.3 9.9
Aspartic Acid 2.0 9.9 3.e (p-cooH)
Glutamic Acid 2.2 9.1 4.2 (y-cOOH)
Histidine 1.8 9.2 6.0 Imidazole)
Cysteine 1.8 10.8 8.3 (Sulftydryl)
Tyrosine 2.2 9.1 10.1 (Phenol)
Lysine 2.2 9.2 10.8 (e-amino)
Arginine 1.8 9.0 12.5 (Guanidino)
Serine 2.2 9.2 13.0 (Hydroxyl)
Table 6-l
The pK values for the ionizing groups of a few of the standard 20 amino acids.

The tlenderson-flasselbalch Equation


The Henderson-Hasselbalch equation (6-1) can be used to determine the fraction
of either the g-amino, G-carboxyl, or side chain groups that are ionized at a
particular pH. In this equation [Ae] is the concentration of conjugate base while
[HA] is the concentration of conjugate acid.

(6-1) pH=pKu+rosffi fl enderson-llasselbalch Equation

Let's consider the ratio of the protonated to the unprotonated cr-amino group of a
general amino acid at a pH of 7.0. The pKu for a typical s-amino gtonp is ibout
9.4. using the Henderson-Hasselbalch equation, we find that the ratio of the
protonated cx-amino group to the unprotonated o-amino group is about 102.4 or
251:1. This tells us that the predominant form of the u-amino group at a pH of 7.0
pH - PKa lAl/HA is the protonated form. The same type of analysis for the cr-carboxyl group tells
J 0.001 us that the ratio of the o-carboxylate anion to the protonated cr-carboxyl group is
2 0.01 about 63,000:1.
1 0.1
0 1

I 10 (6-2) 7 .o = 9.4* ,on" [R-NHz] 102.4 - tR-NHil _2sr (6-3)


2 100 tR-NHil lR-NH2l 1

-)
1000
similarly, at a neutral pH of 7.0 the ratio of the protonated to non-protonated
Table 6-2 side chain of histidine is about 1:10.
Difference between pH and
pKa and the relationship to the If we know the difference between the pH and the pKu (i.e., pH - pKu), then we
ratio of conjugate base to can establish a ratio (Table 6-2) between the concentration of conjugate base to
conjugate acid. conjugate acid. If a particular amino acid is required at the catalytic site 6f an
enzyrne, knowing the ratio between conjugate base and conjugate acid will help
determine the maximum potential activity of the enzyme.

Copyright @ by The Berkeley Review 6 The Berkeley Review


Specializing in MCAT Preparation
Biology Structure & Function in Cells & Viruses Biological Molecules

Isoelectric Point (pI)


The isoelectric point (pI)'is that p H at which an amino acid (or a molecule)
carries no net electric charge. By manipulating the Henderson-Hasselbalch
equation we can obtain an expression (6-a) that states that the isoelectric point is
simply the arithmetic mean of two pKu values.

pI= lpKar + pKa2]


(6-4) 2

For example, the isoelectric point for the amino acid glycine is 6.1 while the
isoelectric point for the amino acid lysine is 1.0.0. If we were to place these two
amino acids in an electric field, we would find that at any pH above their
isoelectric points they would migrate toward the anode (positive electrode) while
at any pH below their isoelectric points they would migrate toward the cathode
(negative electrode). This characteristic allows for separation of amino acids by a
process called electrophoresis. Paper electrophoresis is generally used for
separating mixtures that contain charged molecules which are small while gel
electrophoresis is used for separating proteins and nucleic acids.

Titration Curves
At physiological pH glycine exists in solution as the dipolar ion. The net charge
on this amino acid would be 0. However, if we were to add a strong acid to the
dipolar solution, the carboxylate group would become protonated and the
overall charge of the amino acid would be +1. Similarly, addition of a strong base
to the dipolar solution would remove a proton from the protonated cr-amino
group, giving the amino acid a net charge of -L. As shown in Figure 6-7,the
ionization of glycine depends on the pH of the solution.

pH=1 pH = 6.1 pIJ=14

HO
oT3c- oH
s,N-
-H
@ HO
@rrio -H
@
rilo
?- I

HPK"H a^
PKo H
9.8
[Net charge is +1] [Net charge is 0] [Net charge is -1]

Figure 6-7
Ionization of glycine.

The ionization of glycine can be followed by using a titration curve (Figure 6-8).
We can start our titration by adding equivalents of base (such as NaOH) to a
solution of glycine which is fully protonated. The initial pH of this solution might
be in the neighborhood of 1 and the overall net charge on glycine is roughly +1.0.

The first midpoint of the titration curve comes when [NH3@CH2COOH] =


[NH3oCH2COOO].The Henderson-Hasselbalch equation tells us that at this first
midpoint the pH = pKa. In other words, the pH at this point is 2.4 (which is the
pKu of the first ionizable proton), and half of the solution is composed of
NH3@CH2COOH while the other half of the solution is composed of
NH3oCH2COOo. The overall net charge on glycine in solution at this point is
about +0.5.

Copyright @ by The Berkeley Review The Berkeley Review


Specializing in MCAT Preparation
BiOlOgy Structure & Function in Cetts & Viruses Biological Molecules

Addition of more base will eventually convert all of the NH3@CH2COOH to


NH3@CH2Cooo. At this point the first equivalence point of the solution has
been reached. The overall net charge on glycine is 0. The isoelectric point has
been reached and the pH is about 6.1.

In order to reach the second midpoint in the titration we add more base. At the
second midpoint we find that [NH3@CH2COOO] = [NH2CH2COOO].Again, the
Henderson-Hasselbalch equation tells us that at the second midpoint the pH =
pKu. The pH at this point is 9.8 and the overall net charge on glycine in solution
is about -0.5.

Second Midpoint
INH3+CH2COO-] = tNH2CH2COO-l

8
PKa = 9'8

lNH2CH2
+
coo-l
First Midpoirtt EquivaLence Point

pH6 lNH3+CH2COOHI tNH3+CH2COO-I


il PKa = 6'1
lNH3+CH2COO-I
4
Y Buffer Region
2 PKa = 2'4

+0E-

tN H3+cH2cooHl
0.5 1.0

Equivalence of NaOH Added


1.5

Figure 6.8
Titration of glycine.

By the time the second equivalence point is reached enough base has been added
to completely titrate all the ionizable protons on glycine. The dominant species in
solution is now NH2CH2COOO. The overall net charge on the amino acid is -1.

Buffers
The ability of a solution to resist a change in its pH when either an acid or a base
is added is the principle behind the buffer capacity of a solution. In Figure 6-8
note that the slope of the titration curve is much less near the first and second
midpoints than near the equivalence point. For each increment of base added in
the vicinity of the pKa's of glycine the pH changes very little. This means that
the numerical value of the log ([Ae]/[HA]) in the Henderson-Hasselbalch
equation does not change that much. In other words, [Ae] approximately equals
[HA] in the region of the pKa. If a weak acid is within 1pH unit of its pK3 value,
it resides within a good buffering range.

Copyright @ by The Berkeley Review B The Berkeley Review


Specializing in MCAT Preparation
Biology Structure & Function in Cells & Viruses Biological Molecules

Proteins
The 20 standard amino acids that we have mentioned can be linked together to
form long polypeptide chains. Amino acids can be joined to one another by a
special type of amide bond called a peptide linkage. For example, in Figure 6-9
two amino acids are joined together to form a dipeptide.

As we will see in a later discussion, this reaction requires an input of free energy.
This means that the hydrolysis of the dipeptide would be more favorable than its
synthesis. All hydrolysis reactions are favorable reactions.

Peptide Bond

0
H o Ho Hzo HO
@l ilo o I il o / @ "[o-l
I lil I rilo
H3N- C- C- O HrN_c_c_o C-C-O
_;:tj,__i,l.-ir
I

R1 R2

Figure 6-9
Formation of a dipeptide from two amino acids.

A polypeptide is a convenient term for any length of a polymer of amino acids. If H O Cysteine
Iil
we put two amino acids together, we have a dipeptide. If we put three amino C_N-C_C-N-C
acids together, we have a tripeptide. We might run into the word oligopeptide. An
tt I
HCH,H
oligopeptide is simply 10 or so amino acids linked together. A protein is t-
SH
generally described by 100 or more amino acids linked together.
SH
I

H CH" H
Amino terminal Carboxyl terminal I t- I

end end C_ N- C- C_N - C


I il
H o
HOHOHOHOHO
oriltilrtttttlllo Cysteine
'rtt C- C- N- C- C- N-
HlN- C- C- N- C- C- N- C- C- O
tl ll ll
RTHRzHRjHR+HR5
reduction 1l
"*,o"u.r
ll
Figure 6-lO
Modified amino acids. HO
Iil
C-N-C_C-N-C
tt
Amino acid units are often referred to as residues. Consider the pentapeptide HCH^H
I

shown in Figure 6-10. This peptide is composed of 5 amino acid residues and is t'
S
synthesized (and written) from the amino terminal residue to the carboxyl I

terminal residue, left to right, respectively. In other words, if we had the S


I

pentapeptide Tyr-Gly-Gly-Phe-Met, then Tyr would be the amino terminal and HCH"H
tt"l
Met would be the carboxyl terminal. C-N-C_C_N-C
lll
HO
X-ray crystallographic studies performed by Linus Pauling and Robert Corey in Cystine
the late 1930s showed that the peptide unit (i.e., the O-C-N-H bonding) is planar
and rigid due to the partial double bond character of the C-N bond (i.e., the Figure 6-l I
peptide bond). However, there can be rotation about the bonds between the a- Formation of a disulfide
carbon and the carbonyl carbon and the s-carbon and the nitrogen. linkage between two cysteine
residues.
Some proteins that contain cysteine residues can formdisulfide bonds as shown
in Figure 6-11. For example, within the amino acid sequence of bovine insulin
there are 6 cysteine residues. These residues have the ability to form three

Copyright O by The Berkeley Review I The Berkeley Review


Specializing in MCAT Preparation
BiOIOgy Structure & Function in Cetls & Viruses Biological Molecules

(a)
distinct sets of disulfide cross-links. When two cysteine residues are oxidized they
form a disulfide called cystine. Be careful of the difference between cvsteine and
cystine.

t-9st Pauling and Corey suggested that polypeptide chains have the ability to
tn
fold into a-helices and B-pleated sheets. the cr-netix is stabilized by hydrogen
bonding between the Co and NH groups as shown in Figure 6-12a (that are f6ur
residues apart). There are about 3.6 amino acids per turn of the cr-helix. They are
separated from one another by a translational distance of 1.5 A and a rotation of
100 degrees. The side chains of the amino acid residues extend outward and
away from the helical axis (Figure 6-12b).

Hydrogen bonds between the


In the B-pleated sheet the hydrogen bonding between the co and NH groups
C=O and N-H groups help to occurs between different polypeptide chains as shown in Figure 6-13. These
stabilize the cr-helix. polypeptide chains can either be parallel (running in the same direction) or
antiparallel (running in opposite directions). In the situation of the antiparallel
B-
pleated sheet the polypeptide chain is almost fully extended and the distance
ft) between amino acid residues is about s.s A. As a polypeptide chain folds back to
itr the opposite direction, it reverses direction by making a
_tnt B-Turn. Hydrogen
bonding at the B-Turn occurs between the Co group of one amino acid residue
and the NH group of an amino acid residue which is three residues away.

ROHRO
l ll H l t ilH
/ .c. c. r-N_^_c_---c_'i-N. rro
F i-1l1--tc c r N C' c
Looking down the o-helix Ht l ilHt t \
we see the side chains HROHN\
pointing outward.
Hydrogenbond+: : ! n_C_R
Figure 6- l2 + \ Hfl
I THP )"./
N-. w
Two views of an s-helix. (a) Side t-a-
^,f-
ttHilttHil
c. !.c. N,?-
view. (b) Top view. RHORHO
".
Figure 6-13
Hydrogen bonding in antiparallel B-sheet.

In 1957 John Kendrew worked out the three-dimensional structure of the protein
myoglobin (Figure 6-14). Human myoglobin consists of 153 amino acid reiidues.
Roughly 75"/, of this polypeptide chain is in an a,-helical conformation. There are
8 major helices called helix A, B, c, D, E, F, G, and H. within a crevice of the
protein is a heme group which can bind oxygen. The interior of the myoglobin
protein is composed almost entirely of hydrophobic non-polar residues.

Not all proteins have such a high percentage of o-helices within their framework.
For example, ribonuclease S (an enzyme secreted from the pancreas that hydro-
lyzes RNA) is a protein that is composed of 124 amino acid residues which are
arranged in a number of B-pleated sheets.

Proteins with a single polypeptide chain can be defined by their primnry,


secondary, and tertiary structure. The primary structure represents the sequence of
Figure 6- l4 amino acids in a protein and includes the location of disulfide bonds. The
Three dimensional structure of secondary structure represents the spatial arrangement of amino acids that are
myoglobin.
close to one another while the tertiary structure represents the spatial
arrangement of amino acids that are far from one another. A protein with tertiary

Copyright @ by The Berkeley Review to The Berkeley Keview


Specializing in MCAT Preparation
Biology Structure & Function in Cells & Viruses Biological Molecules

structure can be referred to as a subunit. If various subunits associate with one


another, then the protein is said to have quaternary structure. For example,
myoglobin is a protein with tertiary structure but hemoglobin, because it cr-helix
contains four polypeptide subunits, is a protein with quaternary structure.

The level of structure in a protein can best be understood by considering


hemoglobin. The four polypeptide (two a and two p) subunits of hemoglobin
form a tetramer which is spheroidal in shape. Associations between these
subunits are formed through hydrophobic, ionic, hydrogen bonding, or polar
interactions. It is the interactions between these four polypeptide subunits that
gives hemoglobin its quaternary structure.
Secondary
However, if we were to examine just one of the polypeptide subunits, we would
find that it would primarily contain a number of cr-helical domains (given the
letters A-H). The complete spatial arrangement of all the amino acids to one
another within this polypeptide subunit defines the tertiary structure. The
Primary
9
secondary structure is simply the spatial arrangement of amino acids adjacent to
one another within the polypeptide chain. The primary structure includes the
primary sequence of the polypeptide chain. These four levels of protein structure
are depicted in Figure 6-15. In particular, we have focused in on a set of amino
acids in the E helix of one of the B subunits of hemoglobin.

It tums out that it is the primary structure that determines the way in which a
protein will fold up on itself. Insight into this phenomenon came from the work
that Christian Anfinsen did with bovine ribonuclease. He found that in the
presence of p-mercaptoethanol (which cleaves disulfide bonds) and 8 M urea, the
ribonuclease enzyme was denatured and lost its enzymatic activity.

Enzymatic activity was restored to this protein after the sulfhydryl groups on the
cysteine residues were oxidized and dialysis of the p-mercaptoethanol and urea
was complete (Figure 6-16). This experiment confirmed that it is the primary
structure of a polypeptide that determines the three-dimensional tertiary
structure of a protein.

Quaternary
Chemical treatment
Figure 6- l5
SH The four levels of protein structure
Removal of chemical
in the molecule hemoglobin.
treatment SH

Ribonuclease Denatured Ribonuclease


(active) (inactive)

Figure 6- l6
Chemical denaturation of ribonuclease using B-mercaptoethanol.

Out of the many possible ways in which the ribonuclease protein can fold into its
tertiary structure, only one will lead to the correct enzymatic activity. It turns out
that protein folding is not a random search. Why? Because it would take too long
to complete the process. In fact, if a protein had 100 amino acid residues, each
having the possibility of assuming 3 different positions,then there would 6" 3100
possible stiuctures. This would take about 'L.6 x L027 yearsl The universe,
however, is only about 1.5 x 1010 years old (give or take a few billion years).

Copyright @ by The Berkeley Review ll The Berkeley Review


Specializing in MCAT Preparation
Biology structure & Function in ceils & viruses Biological Molecules

How is it thai polypeptides fold into a tertiary structure? one theory says that
sections of the primary structure fluctuate between being in the natlve linear
(primary) arrangement and either an s-helix or a
B-pleated'sheet and that when
two such forms of secondary structure interact wiih one another by diffusion
they tend to stabilize each other. The exact mechanism as to how this happens is
not known.

The idea of two helices


Filg side by side is rather interesting, especially in
proteins that bind to DNA. If two cx-helices, each containing af leasl 4 leucine
residues and located on separate proteins, come together throirgh interdigitation
of those leucine resid'es, tfen a leucine zipper wilt form bnainjthe two
tgsetfel Even though the leucine zipper does not directly inteict with the iroteins
DNA
double helix, it does allow for the binding of some DNA regulatory proteins.
we
will examine this in a little more detail when we discuss thJexpressi-on of genetic
information.

Essentially all reactions in a cell are cataryzed by proteins called enzymes. when
a substrate comes into contact with the active rit" or catalytic site of an enzqe,
it will be bound at that site in such a way that the enzyme will be able to convert
that substrate into a product molecule. There ur" oth", enzymes that, besides
having a catalytic site, have-a regulatory site or allosteric site.'The regulatory site
is a site that is different from the catalytic site. Enzymes of thiJ naturi are
generally referred to as allosteric enzymes (from the ireek allo for',other" and
stereos for "space" or "site").

We will find that many different proteins have these allosteric regulatory sites
and that these sites can control the binding of various substrate or modulator
molecules. A good example of an allosteric protein is hemoglobin. Hemoglobin
has four polypeptide subunits (it has quatemary structure), -each having u"h"-"
gro-up that has the capability of binding an oxygen molecule. when or,J o*yg"r,
molecule binds to the heme group of one of the polypeptide subunits, tlat
binding information is somehow transmitted to the otn". tnr"" polypeptide
subunits, thus facilitating their ability to bind oxygen.

Copyright @ by The Berkeley Review t2 The Berkeley Review


Specializing in MCAT Preparation
Biology Structure & Function in Cells & Viruses Biological Molecules

Carbohydrates E(i,Lipids
Carbohydrates
The name carbohydrate is misleading. It was coined at a time when the formulae
of carbohydrates was just becoming known. Almost all carbohydrates had a oH
similar empirical formula which looked like Cn(H2O)tr' This formula leads us to -\/
C
believe that there is a water molecule attached to each carbon atom that apPears I

in the structure. However, this is not the case. A better empirical formula to use H- C- OH
I

is (CH2O)n. cH2-oH
Glyceraldehyde
In order for a molecule to be classified as a carbohydrate, it must have an
aldehyde or a ketone functional gloup and two or more alcohol functional cH2-oH
gto.rpi. Carbohydrates are simply polyhydroxylated aldehydes or ketones. If we I

iollow this rule, we find that the simplest carbohydrate is obtained when n = 3 in C=O
I

the empirical formula (CH2O)1. Glyceraldehyde and dihydroryacetone are two cH2-oH
simpleit carbohydrates that can be formed (Figure 6-17). Glyceraldehyde has an Dihydroxyacetone
aldehyde functional group while dihydroxyacetone has a ketone functional
group. Each has two alcohol groups. If the carbohydrate contains an aldehyde, it Figure 6-17
is referred to as an aldose. If it contains a ketone, it is referred to as a ketose. Molecules with the molecular
formula (CHZO)n.
As a group carbohydrates are the most abundant biological molecules on earth.
Theyian be divided into three major classes: monosaccharides, oligosaccha-
rides, and polysaccharides.

Monosaccharides
A monosaccharide, besides being a polyhydroxylated aldehyde or ketone,
contains a backbone of unbranched carbon atoms that are covalently linked to
one another. Glyceraldehyde and dihydroxyacetone are the two simplest
monosaccharides. If the carbohydrate has a 3-carbon backbone, it is called a
triose. Besides the triose sugars, we will also find sugars which are tetroses (4-C),
pentoses (5-C), hexoses (6-C), and heptoses (7-C).

At the turn of the twentieth century Emil Fisher realized that he could devise a Mirror
system for naming carbohydrates (and amino acids) based on the structure of H. ,f oat ,H
glyceraldehyde. He chose glyceraldehyde because it has just one chiral center (a C C

iuiUon atom to which four different substituents are attached). If a molecule has
I I

n chiral centers, it will have 2n stereoisomers. Glyceraldehyde has two stereo- "o-!-, n7!\ou
cH2-oH cH2-oH
isomers (Figure 6-L8). Fisher said that if the hydroxyl group on glyceraldehyde's
chiral carbon is to the right, the molecule is in the form of the D-isomer. If the L D
hydroxyl group is to the left, the molecule is in the form of the L-isomer. Glyceraldehyde

In order to tel| if a monosaccharide drawn in the Fisher projection is in the D or L Figure 6-lB
isomeric form, we only need to look at that chiral carbon which is most distant D-Glyceraldehyde and its
trom the carbonyl carbon. That particular chiral carbon atom is often referred to enantiomer.

as the reference carbon. If the hydroxyl group attached to that reference carbon
is to the right, the molecule is the D isomer. If the hydroxyl group is to the left,
the molecule is the L isomer. Most of the naturally occurring sugars are found in
their D form (while most of the naturally occurring amino acids are found in
their L form).

\lonosaccharides can either be found in the linear form or in the cyclic form.
\fany sugars that contain five or more carbon atoms in their backbone prefer to
re rn the cyclic form. For example, the aldohexose D-glucose (Figure 6-19) can

Copyright @ by The BerkeleY Review r5 The BerkeleY Review


Specializing in MCAT PreParation
Biology structure & Function in celrs & viruses Biological Molecutes

form a six-membered ring referred to as a pyranose. In the formation of the ring


o.H
r\z
-c
(Figure 5-20), the oxygen atom of the C-5 hydroxyl group reacts with the
C-l carbonyl carbon atom to form two different diastereomeir, a-D-gl.r.opyranose
C.2 H_C-OH
I
and p-D-glucopyranose. Because these two molecules diffe-r or,ly in th"
I configuration about the C-1 carbon atom, they are referred. to as anomers, and
c-3 Ho- c-H the C-1 carbon atom itself is referred to as the anomeric carbon. L:r the o( anomer
I
C-4 H-C-OH note that the oH group at the anomeric carbon is on the opposite side of the ring
C-5 H-c-oH
I
f1o*_!" CH2OH group that is attached to the referet .iibon. In the B anome"r
I tle oH group at the anomeric carbon is on the same "" side of the ring as that
c-6 cH2oH
CH2OH group.
D-Glucose

Figure 6-lg
The straight chain form of D- .H
o
Glucose. t
n-H
Ht \ o
-
HOH HOH
u,-D-Glucopyranose
C-l cH"oH
l"
B-D-Glucopyranose

C-2 C= o Figure 6.2O


c-3
I

Ho- c* H Formation of the two anomers of D-Glucopyranose.


I

C-4 H-C_OH
I Carbohydrates can also exist in a five membered ring. ln this case the cyclic sugar
C-5 H-c-oH would be called a furanose. The ketohexose D-fructose (Figure 6-21) can.y"i""
I
c-6 cH2oH to form a five membered ring (Figure 6-22). There are two diastereomers, o-D-
fructofu ranose and p-D-fructofuranose.
D-Fructose

Figure 6-21
*"W:
l''"[?{::]'ry'".
The straight chain form of D-
Fructose.

OHH OH OHH
d,-D-Fructofuranose
B-D-Fructofuranose

Figure 6-22
Formation of the two anomers of D-Fructofuranose.

Formation of the pyranose ring in D-glucopyranose results from a general


reaction involving an alcohol and an aldehyde. The resulting derivative is
referred to as a hemiacetal. similarly, formation of the furanose ring in D-
fructofuranose results from the general reaction involving an alcohoi and a
ketone. In this case a hemiketal is formed. Formation of these derivatives is
shown in Figure 6-23.

Since D-glucose and D-fructose readily interconvert between the linear and cyclic
forms, they can undergo reactions which are typical to aldehydes and ketones,
respectively. Two oxidizing agents used to identify the functional groups of
carbohydrates is the Tollens'reagent (containing an Ag@ complex) and the
Benedict's reagent (containing a Cu2@ complex). Ir tne rouens' reagent is used to

Copyright @ by The Berkeley Review t4 The Berkeley Review


Specializing in MCAT Preparation
Biology Structure & Function in CeIIs & Viruses Biological Molecules

identify an aldose or a ketose, the sugar is oxidized and the Ag@ ion is reduced to
silver metal (which precipitates as a siluer mirror on the sides of the reaction
vessel. If the Benedict's reagent is used, the sugar is oxidized and the Cu2@ ion is
reduced to give a brick-red precipitate. If an aldose or a ketose is capable of
reducing these ions, those sugars are referred to as reducing sugars.
Carbohydrates that contain a hemiacetal or a hemiketal group give positive tests
with Tollens' and Benedict's reagents.

o R2_ O\ H o R2_ O\ H
il /O_ il /o_
Rr- C- HO- =* Rl- C- HO- R.- -i*
H + R2
,C,
R.3 +
/\
C
RrH Rr R:
Aldehyde Alcohol Hemiacetal Ketone Alcohol Hemiketal

Figure 6-23
Hemiacetai and hemiketal formation.

Oligosaccharides
Oligosaccharides are relatively short chains of monosaccharides linked to one
another by a glycosidic bond. Disaccharides are the most common oligo-
saccharides. The disaccharide sucrose (table sugar) is formed by the linkage of a-
D-glucopyranose and B-D-fructofuranose. The bond connecting the two mono-
saccharides is termed an O-glycosidic bond, and it is the carbohydrate analog of
the peptide bond found in proteins.

The systematic naming of disaccharides (and larger sugars) follows a few simple
rules. Let's consider sucrose as an example (Figure 6-24). First, notice how the
two monosaccharides in sucrose are joined together. They are linked through an
oxygen atom, which means that the bond is an O-glycosidic bond. If we were to
hydrolyze that bond with H2O, we would get the two individual mono-
saccharides (glucose and fructose).

CH,OH
o o

OH
CI

HOH OHH HOHOHH


G-o-D-glucopyranosyl-( 1
-r2)-B-D-fructofuranoside cr-D-Glucopyranose B-D-Fructofuranose
(Sucrose)

Figure 6-24
Sucrose is also systematically called O-a-D-glucopyranosyl-(1-+2)-$D-fructofuranoside.

Locate the anomeric carbon for glucose. Is the OH group attached to that
anomeric carbon in the cr or B conformation? It is s because the hydroxyl group
at the anomeric carbon is on the opposite side of the ring from the CH2OH group
that is attached to the reference carbon. Is the OH group attached to the anomeric
carbon of fructose in the cr or B conformation? It is p because the hydroxyl group
at the anomeric carbon is on the same side of the ring from the CH2OH group that
is attached to the reference carbon. This is an unusual linkage in that the O-
glycosidic bond is formed between two anomeric carbons. If we view this linkage

Copyright @ by The Berkeley Review l5 The Berkeley Keview


Specializing in MCAT Preparation
Biology structure & Function in cells & viruses Biological Molecules

from the point of view of the glucose molecule, we will see that the bond is
formed between the C-1 atom of glucose and the C-2 atom of fructose. This is
written as (1-+2) in the naming of the molecule. putting what we have learned so
far together, we could name sucrose as o -a-n-glucopyranosyl-(L-+2)-B-D-
fructofuranoside' The O in front of the name is to remind us that the linkage is
an o-glycosidic linkage. The -ide at the end of the name tells us that sucroie is
not a reducing sugar.

We know that a sugar is a non-reducing sugar if its hemiacetal or hemiketal


group has been converted to an acetal or ketal group, respectively. we can form
an acetal or a ketal as outlined in Figure 6-2s.If a carbohydrate contains only an
acetal or a ketal group, it will not react with either the Tollens' ,"ug"rri o,
Benedict's reagent.

R.-O O-H R2- O\ R.-O O-H


P- R3 R2- O,
,O- Rc

,"t + HO- R: c
/-\ C-
,/\ HO-R4 : A
RrH RrH Rr R3 Rr Rs

Hemiacetal Alcohol Acetal Hemiketal Alcohol Ketal

Figure 6-25
Acetal and Ketal formation.

Lactose (milk sugar) is a disaccharide that is found only in milk. It is composed


-D-galactopyranose and p-D-glucopyranose linked together through a
B
9f
B(l-+a) O-glycosidic linkage (Figure 5-26). Notice that the anorieric cu.bon o-f the
galactose residue is tied up in an acetal linkage with glucose. However, the
anomeric carbon of the glucose residue is just part of a hemiacetal group, and
since it can be oxidized,lactose is a reducing sugar. [The structure oflactose has
been given on the MCAT a number of times.l

cH2oH

H
al-
OH H'I
2

HOH HOH HOH


o-B-D-galactopyranosyl-( I -;4)-p-D-glucopyranose B-D-Galactopyranose B-D-Glucopyranose
(Lactose)

Figure 6.26
The disaccharide lactose and its two monosaccharide components, D-galactose and D-glucose.

Polysaccharides
Polysaccharides (glycans) are large molecules which contain many mono-
saccharide residues linked together through glycosidic linkages. Polysaccharides
can consist of just one type of monosaccharide, in which case they are called
homopolysaccharides, or they can consist of different types of monosaccharid.es,
in which case they are called heteropolysaccharides. Each type can be either
unbranched or branched. Two important (storage) polysaccharides are starch
and glycogen.

Copyright @ by The Berkeley Review l6 The Berkeley Review


Specializing in MCAT Preparation
Biology Structure & Function in Cells & Viruses Biological Molecules

Starch is a food reserve in plants. It can be found as cr-amylose, a linear polymer


of unbranched D-glucose residues linked together in o(1+4) linkages, or as
amyldpectin, a branched polymer of D-glucose residues linked together
primarily in a(1+4) linkages but at the branch points in o(1-+5) linkages.
Examples of both polymers are shown in Figure 6-27. Tll.e branch points in
amylopectin occur roughly every 24 to 30 glucose residues.

CH"OH

QH
H
OHH

HOH HOHO
I

cH2oH cH2oH 6 cH2


H \r
fLoJ\
23
HOH HOH
c-amylose amylopectin

Figure 6-27
The two forms of starch:. cx-amylose and amylopectin.

Starch can be found in wheat, rice, corn, and potatoes and is the major
carbohydrate source in the human diet. Many of the starches contain about 15%
cr-amylose and 85% amylopectin. Digestion of starches begins in the mouth with
the enzyme saliaary cr-amylase. This enzyme hydrolyzes many of the o(1-+a)
linkages in starch and degrades large polysaccharides into smaller oligo-
saccharides. Once the oligosaccharides pass through the stomach and into the
duodenum of the small intestine, they are degraded even further by pancreatic u-
amylase to disaccharides, trisaccharides, and small branched oligosaccharides
referred to as dextrins. Enzymes within the intestinal system degrade these
oiigosaccharides into individual monosaccharides, which are then absorbed by
the intestinal epithelial cells and passed to the blood'

Glycogen is the storage polysaccharide common to all animals and is located


primarily in skeletal muscle and liver tissue. Glycogen has a structure similar to
that of amylopectin, except that the branch points in glycogen come about every
8 to 12 glucose residues.

Lipids
Biological lipids are molecules that can readily dissolve in nonpolar solvents but
are relatively insoluble in water. These compounds are chemically and
lunctionally quite diverse. Some of the classes of lipids that we will examine are
the fatty acids and triacylglycerols, glycerophospholipids, sphingolipids, and
cholesterol.

Copyright @ by The Berkeley Review The Berkeley Review


Specializing in MCAT Preparation
Biotogy structure & Function in cells & viruses Biological Molecules

Fatty Acids & Triacylgtycerots


Ho. Ho. Fatty acids are carboxylic acids with a hydrocarbon side chain. The majority of
c'-o C/-o fatty acids have side chains that are unbranched and contain an even number of
H"C H"C garbon atoms' In plants and animals the more common fatty acids contain either
16 carbons or 18 carbons. If a fatty acid is saturated, ali of its carbon
CH, CH. atoms
t' (except the carbonyl carbon) will have a full complement of hydrogen atoms.
H,C H,C we
will not find any double bonds between carbon uio*s. An unsaturfted fatty acid,
CH. CH"
t' however, will contain double bonds between specific carbon atoms. Two
H"C H,C common fatty acids are shown in Figure 6-2g.
/'CH, CH,
H"C H"C
'\^_H In nature, fatty acids are rarely free. Rather, they are esterified to a glycerol
CH, L backbone to form a triacylglycerol (Figure 6-29). In animals these compo,inds are
t' il
synthesized and stored in adipocytes (fat cells). Triacylglycerols arc-neutral
H-C c- fats
/H and serve as storage depots for fuel used in metabolism. Triacylglycerols ihat
CH" H.C
contain a high degree of saturated fatty acids (e.g., butter) pac{ t*ofether quite
H"C CH" well and form crystalline structures that havefairty high meltiig pointJ. This is due
CH.
/' H"C to the large van der waals attractions between ttre rnethyle"J (-cuz-) groups of
H.C CH" the hydrocarbon portion of the fatty acid. As the molecular weight of a iaturated
/" CH. HNC fatty acid incteases, so does the melting point. The fluidity of tIi" tlpia decreases
H:C CH, and at room temperature tend to become solids referrei to as fats. A double
Palmitic acid H"C
bond in an unsaturated fatty acid places a kink in the hydrocarbon chain, thereby
(Cro) decreasing the van der waals interactions and lowering the melting poini.
CH:
Triacylglycerols that contain a high degree of unsaturated or polyunsiturated
Oleic acid fatty acids (e.g., olive oil) tend to be oils at room temperature.
(Cra)

Figure 6-28
Two common fatty acids.
H,CF OH
"oAAz\o H,c- oA-'\'/\ s
ln
"l- "/--..-,,^---
H2C- OH
Glycerol Fatty Acids
",Lo-[^^o
Triacylglycerol

Figure 6'29
The formation of a triacylglycerol.

Glycerophospholipids
The glycerophospholipids (or phosphoglycerides) are the predominant lipid
component of all biological membranes. Glycerophospholipids (rigure 6-30)
have two fatty acid residues esterified to the C-1 and e-2 cirbons ol glycerol.
Esterified to the c-3 carbon of glycerol is a phosphate group which-bears a
negative charge at pH 7.0. Three of the more common molecules that can be
attached to the phosphoryl hpad group are ethanolamine (HocH2cHzNHg@),
choline (HoCH2CH2N(CH3)3o), and serine. Glycerophospholipids u.e1*pni-
philic in that they have nonpolar tails (the hydrocarbon chiins of the fatty acids)
and a polar heads(the phosphate group and its associated attachment). Beca1,s"
glycerophospholipids contain a phosphate group, they are also referred to as
phospholipids.

Copyright @ by The Berkeley Review l8 The Berkeley Review


Specializing in MCAT preparation
Biology Structure & Function in CeIIs & Viruses Biological Molecules

Sphingolipids
The sphingolipids are not based on a glycerol backbone. Instead, they are
derivatives of amino alcohols. One common derivative is sphingosine. Attached il
to the C-2 carbon is an amino group that can be linked to a fatty acid through an s,c-o€n
'l
amide linkage. This molecule is called a ceramide (Figure 6-31,a), and is the n
structural residue that is common to all sphingolipids.

Ifa phosphoethanolamine or a phosphocholine group is attached to the C-1


,L-"A^^.-
CHr
carbon of the ceramide, the sphingolipid becomes a sphingomyelin (Figure 6- o 6\l
31b), and sphingomyelins are abundant in the myelin sheaths that surrounds the H"C- O- 'r-n-VT-tn'
axons of nerve cells in the nervous system. I cHl
oo

(a) (b)
Figure 6-3O
H H
I I
Phosphatidylcholine.
C-3 HO- CH- (CH2)r2- CHi HO- CH- C- (CHz)rz- CHr
t= "- f=
HO HO

c-2
H
I
"L-*A--I

H
o
il @
c-l Hz(F OH H,C-
'l O- P- O- CH, - CH2 - NH.r

Ceramide
o^v
Sphingomyelin

Figure 6-3 I
Different types of sphingolipids.

\Vhen a single monosaccharide like glucose or galactose is attached to the C-1


carbon of the ceramide, the sphingolipid becomes a cerebroside. Cerebrosides
are the simplest of the sphingoglycolipids. The more complex sphingoglyco-
lipids are the gangliosides. In this case several sugar residues (i.e., an
oligosaccharide) are attached to the C-1 carbon of the ceramide (Figure 6-32).
The extended oligosaccharide residues of a ganglioside are quite important as
they are thought to be involved in recognition of other molecules at the surface of Figure 6-52
the cell. For example, specific gangliosides act as the antigenic determinants in A typicai ganglioside.
the human ABO blood group system. Note that cerebrosides and gangliosides do
,:of contain a phosphate group.

Cholesterol
Cholesterol is synthesized in the cytosol and generally exists as the cholesterol
ester. This lipid is a major constituent of eukaryotic animal plasma membranes
and is an intermediate in the biosynthesis of all human steroids. It has a fairly Steroid Hormones
rigid set of four fused nonpolar rings and a polar hydroxyl group that gives it a 1. Progesterone
:Li ghtly amphiphilic attribute. 2. Glucocorticoids
3. Mineralocorticoids
There are five major classes of steroids (Table 6-3), all of which are synthesized 4. Androgens
tn the mitochondrion, Synthesis of a steroid hormone begins with the hydrolysis of 5. Estrogens
:holesterol esters in the cytosol and the subsequent transport of cholesterol into
Table 6-5
ihe mitochondrion. Adrenocorticotropic hormone (ACTH) stimulates the Steroid classes.
conversion of cholesterol to pregnenolone, an intermediate in the pathway of
steroid synthesis (Figure 6-33).

Copyright @ by The Berkeley Review l9 The BerkeleY Review


Specializing in MCAT Preparation
Biology Structure & Function in Cells & Viruses Biotogical Molecules

Later, when we discuss basic physiological processes, we will return to these


steroids and discuss them in more detail. But for the moment, let's consider their
general function.

Progesterone has a primary function in women where it prepares the uterine


lining for implantation of an ovum. If implantation occurs, this steroid is
necessary to maintain the endometrial lining of the uterus and hence maintain
pregnancy. During pregnancy progesterone stimulates mammary tissue growth
in preparation for parturition. Progesterone is also synthesized in low levels in
the testes of the male and in the adrenal cortex of the adrenal glands (located on
top of the kidneys) of both sexes.
Cholesterol
Cortisol is synthesized and secreted from cells in the cortex of the adrenal glands.
I
In the liver cortisol acts to increase both glycogen sy'rthesis and gluconeogenesis.
f
Pregnenolone
In skeletal muscle this glucocorticoid acts to decrease both glucose uptake and
protein synthesis, and increases protein catabolism. In adipose tissue cortisol
increases lipid mobilization and decreases glucose uptake. Cortisol is also known
I

i as hydrocortisone.
Progesterone
Aldosterone is also synthesized and released from cells in the adrenal cortex. It

tl
"4\
Testosterone I Cortisol
acts to increase the reabsorption of sodium (Na@) at the level of the kidney,
intestines, salivary glands, and sweat glands. The net effect is to cause retention
of Nao in the extracellular fluid (ECF) thereby increasing ECF volume. Not only
will this lead to an increase in blood volume, but it will also lead to an increase in
I nto.,.t
"".,r,"
blood pressure and blood flow.
I

Testosterone is synthesized in the male in the Leydig cells of the testes and aids in
I sperm maturation. This androgen can also reach the blood and circulates
Estradiol throughout the body, promoting a variety of biological effects, including
development of the secondary sex characteristics
Figure 6-33
Synthesis of the major steroids Bstradiol is the primary estrogen in women and is synthesized in the theca cells
from cholesterol. of ihe ovarian follicles. Estrogens have a variety of biological effects, including
development of secondary sex characteristics, regulation of the ovarian cycle,
and control of certain metabolic processes.

Copyright @ by The Berkeley Review 20 The Berkeley Review


Specializing in MCAT Preparation
Biology Structure & Function in Cells & Viruses Biological Molecules

Nucleic,,,Acids
Nucleic Acids
Genetic information in organisms is carried in the form of nucleic acids. Nucleic
acids occur as either deoxyribonucleic acid (DNA) or ribonucleic acid (RNA).
Both DNA and RNA are made from molecular units called nucleotides, and each
nucleotide consists of (1) a nitrogenous base, (2) a pentose sugar, and (3) phos-
phoric acid. DNA and RNA are polynucleotides of high molecular weight.

Nitrogenous Bases
DNA contains four different nitrogenous bases. Adenine (A) and guanine (G) are
referred to as purines while thymine (T) and cytosine (C) are referred to as
pyrimidines. These bases are essentially planar and relatively insoluble in water.
RNA also contains four different nitrogenous bases. The only difference between
the bases of DNA and RNA is that in RNA thymine is replaced by uracil (U). The
structures of these bases are shown in Figure 6-34.

NHz o o NHz O

*4.-.-*. ,*4. cHr -4


'it ;il 1)
\* A'*,
i H
HzN
t.\p o-*,,u N
H "^.) HH
"*A
./-^j,l
Adenine Guanine Thymine Cytosine Uracil

Figure 6-54
The nitrogenous bases of DNA and RNA.

Pentose Sugar
The pentose sugar found in DNA is 2'-deoxy-D-ribose while in RNA it is simply
D-ribose (Figure 6-35). In order to avoid ambiguity between the numbering
system in the nitrogenous bases and the numbering system in the ribose ring, the
carbon atoms of the furanose ring are given a number immediately followed by a
prime (') marking. Note that the C-1'hydroxyl group in both 2'-deoxy-D-ribose
and D-ribose is in the B configuration.

5'
HOCH" OH
4' l'
HH
H H
3' z',

HOH HO

2'-deoxy-D-ribose D-ribose

Figure 6-55
The p-furanose rings of DNA and RNA.

Phosphorus
The pH of a cell's cytoplasm is approxim ately 7.2. Phosphoric acid (H3Po4) is an
:mportant molecule in the cell and helps in the buffering of the cytoplasm. There
ut" three hydrogen atoms which are capable of dissociating, each at a different
:K, value (Table 6-4).

Copyright @ by The Berkeley Review 2l The BerkeleY Keview


Specializing in MCAT Preparation
Biology Structure & Function in Cells & Viruses Biologicat Molecutes

H3Po4 : Hrpooo * H@ pKa = z.l

HrPOoo --i- HpO42o + H@ pKr = j.2


o
Ol
o-P=O HPO42o -i+ PO43O + H@ pKu =
rO 12.7
o
Table 6,4
I The three ionizations of phosphoric acid.
5'
K"Jr9Hr

V---y
rt
-o

11
\

n--Glycosidic
\A/hat is the relative concentration of free phosphoric acid in the
In order to determine this we need to use the Henderson-Hasselbalch
cytoplasm?

\ / tinkage equation. We know ,T, t" p-H of the_cytoplasrr-isT.2,so we set the pH in


OH our equation at 7.2.
ol {1toatlo know which pKu value to use. It is thai pKu
o-
f=
o T",
value which is closest the pH value *e ut" given. we then solve the
Henderson-Hasselbalch equation (6-1) for the rela"tive concentrations
of the
conjugate base (HPo42e) and weak acid (H2poae) in solution.
we find
i",^ft, that the ratios of the two species are equal (6-5).

t .2 = t.2* ro, -[!!o?l roo={gle?l 1= trrPo?l (6-s)


tH2PoJl lH2poa) tH2PoJl
why is it important to have an understanding of phosphoric acid? First, the
molecule has an excellent buffering capaciiy. since ptro"phoric acid.
has
three pKu values, it means there are lhtel .egio's wheie the conjugate
base
(A) is going to equal the weak acid (HA). Iismall amounts of acid or base
are added to any one of these three regions, the pH of the solution will
essentially remain constant. This is what defines the buffering capacity
of a
solution.

0 second, phosphoric acid is a component of the nucleotide unit in both


.9o
r00
DNA and RNA. Each nucleotide is linked to the next by a phosphodiester
bond. The backbone of DNA and RNA consists of itternating pentose
to sugars and phosphate groups. Since the hydrogen on the hydroiyi group
of each phosphate residue has a pK3 close-to 3, we find that each
phosphate in DNA and RNA has a negative charge associated with it,
making the backbone quite polar. The nitrogenous bises that are attached
to the sugar residues are hydrophobic and. direct them-selves awav from the
backbone. )

3'Ho H
DNA
The short segment of single-stranded DNA shown in Figure 6,36 contains
Figure 6-36 four nucleotide units and three phosphodiester linkages b'etween the ribose
A polynucleotide sequence of DNA sugars. The nitrogenous bases are attached to the sugar residues by an N-
showing the bases, phosphodiester glycosidic linkage. Both of these rinkages, the phosphodiester and the N-
bonds, and polarity of the polymer. glycosidic, were formed by a pariicular dehydration reaction, and this
means that those b""9:. can be hydrolyzed. Note that the N-glycosidic
linkage is also an acetal linkage.

Copyright @ by The Berkeley Review 22 The Berkeley Review


Specializing in MCAT preparation
Biology Structure & Function in Cells & Viruses Biological Molecules

The synthesis of a polynucleotide chain has direction or polarity. Each strand of


nucleic acid will have a 5' end and a 3' end. Single strands of nucleic acids are (by
definition) written with the 5' end at the left and the 3' end at the right. This is
classically expressed in the notation 5'+3'. If we are discussing a double-stranded
polymer
-of
of DNA (i.e., a DNA double helix or duplex DNA), then the top strand
the duplex is written with the 5' end at the left and the 3' end at the right while
the bottom strand takes on the opposite orientation, 3' end on the left and 5' end
on the right. This can be represented by segment of DNA shown in Figure 6-37.

3.4 A

Hffi*rjfly'm
;:x,l'#o'I
! ,f [],r*:", Daes

""4,{,4,{l
Figure 6-37
Duplex DNA showing antiparallel arrangement.

Each strand of a DNA double helix is joined to the other through interactions of
the nitrogenous bases. Base pairing rules established by Watson and Crick allow
for adenine and thymine to be joined by two hydrogen bonds and guanine and
cytosine to be joined by three hydrogen bonds (Figure 6-38). As the two nucleic
acld polymers wrap around each other in a helical fashion, the bases begin to
stack on top of one another, adding to the stability of the duplex'

H
I Hydrogen f",
(\(n-,.,,,,,,,? "ono /yo"',,,,,,,,
N""",,,,, - *y *- *

o"" 'T^ir\
Ribose
*,ri:Y rL _. .,
""'T3\
",,,,,,,,,,1X
rr'v""'i"'
arron'inn"""-"4-,,ll-) -,.1*,-^ t*^*'
i Ribose *,Jo..
Guanine Adenine

Figure 6-38
Hi.og.n bonding between adenine and thymine and guanine and cytosine.

i* r-o)'right @ by The Berkeley Review 25 The BerkeleY Review


Specializing in MCAT PreParation
Biology Structure & Function in Cells & Viruses Biological Molecules

The structures of nucleic acids can become quite a pain to write out, especially if
the nucleotide sequence is hundreds or thousandi of bases long. A number of
different nomenclatures are used to solve this problem. we can let the bases be
symbolized by their respective single letter abbreviations (e.g., G, c, A, & T). Each
pentose sugar unit can be s)rmbolized by a aerticnl line and each phosphate by a p
within a circle. Finally, we can place the 3' and 5' numerals on the vertical lines to
indicate where the phosphate attachment takes place. This type of abbreviated
nomenclature is shown in Figure 6-39.

5'

Figure 6-39
Abbreviated nomenclature for writing poiynucleotide sequences.

It is understood that when we write a nucleotide sequence from the 5' to the 3'
end, there will be an unreacted phosphate group at the 5' end and an rrnreacted
hydroxyl group at the 3' end. we can simplify our notation even more as shown
below. Both sequences are identical to the nomenclature given in Figure 6-36. ht
the sequence on the left it is understood that there is an unreacted hydroxyl at
the 3' end. If we want to just consider the bases in a polynucleotide, the sequlnce
on the right is the easiest and simplest to write. Based on this nomenclaiure, a
DNA duplex should not be difficult to imagine.

pCpCpApT GCAT
RNA
As we have mentioned, RNA contains all of the same components as DNA with
the exception that uracil is found in RNA instead of thymine and the ribose ring
of RNA is hydroxylated at the 2'position whereas in DNA it is not.

There are three types of RNA, and all are slmthesizedby a process called DNA
transcription. Each RNA transcript that is made carries out a specific function in
the cell. RNA polymers that allow for the synthesis of proteinJare referred. to as
transcripts of messenger RNA (mRNA). The building blocks of proteins are
amino acids. Amino acids are brought to the site of protein synthesis by an RNA
polymer called transfer RNA (tRNA). The site of protein synthesis is the
ribosome, which itself is composed of protein (i.e., amino acids) and RNA. In
particular, the RNA polymers that help define the ribosome is ribosomal RNA
(rRNA).

We will come back to both DNA and RNA in a later discussion and consider the
reguiatory aspects that allow for DNA replication, transcription, and translation.

Copyright @ by The Berkeley Review 24 The Berkeley Review


Specializing in MCAT Preparation
Biology Structure & Function in Cells & Viruses Eukaryotic Cells

Cel!ffi,ffii1i fddfibti;uil
Eukar5rotic Chromosomes
lvithin the membrane-bound nucleus of eukaryotic cells resides the majority of
the genetic information needed for cellular growth and division. This genetic
information is found in the nondividing (interphase) cell in the form of
chromatin, a complex of linear, double-stranded DNA, and protein (histones). As
the cell prepares for division (mitosis and cytokinesis) the chromatin becomes
highly condensed into chromosomes.

Haploid & Diptoid


In humans the characteristic number of chromosomes in the nuclei of each
gamete (eggs and sperm) is 23. There are22 autosomes and l sex chromosome
either an X or a Y). Since each gamete has just one set of 23 chromosomes it is
said to be haploid (from the'Greekhaplous, meaning single), or n, where n is the
:.umber of chromosome types. During fertilization the fusion of a male and
:emale gamete produces a fertilized egg, or zygote, which now has a chromo-
Sister
-rorre Coffrplement of 46. Cells bearing this number of chromosomes are diploid
::om the Greek diplous, meaning double), or 2n. Human somatic cells (i.e., cells Chromatids
',"'-nich are not gametes) are diploid in that
they have 23 pairs of chromosomes.

:ach pair of chromosomes is referred to as a homologous pair of


ft-
chromosomes. " \L/
a"nttor"r"tf\
-:.er- generally have similar shapes and sizes and carry the same hereditary traits
::'ntro11ed by segments of DNA called genes. Individual members of each pair
i:- referred to as homologs. lt
Metacentric
^::omosomes
in the nuclei of non-dividing cells are not visible under the light
::_::oscope. However, when cells do begin to divide, either through mitosis or
: =lrrsis, the chromosomes become highly condensed and can easily take up a s7
:::-etr. of stains, making them quite visible under the light microscope. As we
, -.- see in a few moments, the best phase of the cell cycle to view chromosomes

-" ;-::hg metaphase.


t\
,V
-, . rmportant to remember that the chromosomes in metaphase have previously Acrocentric
.: ':.:::.ted during the synthetic phase (S phase) of the cell cycle. These chromo,
: ::.es have a characteristic size and shape that define the karyotype of the
. r-:sm. In the human karyotype the chromosomes are numbered, ihe largest
,:-: :i homologous chromosomes being assigned the number 1 and the smallest
-,-: 'leing assigned the number 22. These are the autosomes. The number 23 is /\
,-..,a;red to the sex chromosome.

Chromatids & Centromeres ,\J


:. r-. :rromosome arrested at metaphase consists of two sister chromatids, each Telocentric
-.-1 i..gether through a constricted region called the centromere. Individual
Figure 6-4O
-,---::::atids will become individual chromosomes when the centromere joining General classification of
'--. :.sler chromatids divides and the chromatids are allowed to separate. By eukaryotic chromosomes based
;.--:,:ron chromosomes containing just one chromatid do not exist. This is an on centromere position.
-: ::::ant point as it will help us understand exactly how much DNA there is
,-- i . r'.r- many chromosomes there are at any given moment in the life of a cell.

r:ht O by The Berkeley Review 25 The Berkeley Review


Specializing in MCAT Preparation
Biology Structure & Function in CeIIs & Viruses Eukaryotic Cens

The position of the centromere along the length of the chromosome allows for a
general classification of eukaryotic chromosomes (Figure 5-40) as either being
metacentric, acrocentric, or telocentric.

llistones & Nonhistones


If all the DNA of all the chromosomes within a nucleus of a single human cell
were placed end-to-end, it would stretch to about 6 feet. How is it that all of this
DNA, roughly 6 x 109 base pairs, can be so successfully confined to the diploid
nucleus of a cell and still retain its functionality?

The two major types of proteins associated with the structure of DNA are
histones and nonhistones. The most abundant are the histones, which are basic
proteins consisting of a high percentage of lysine (Lys) and arginine (Arg)
residues. At physiotogical pH the side chain amino groups (-NHSO) of these
amino acids bear a positive charge. An electrostatic relationship can be easily
formed with the negatively charged DNA polymer. Nonhistones are proteins that
associate with DNA, but are not histone proteins. Rather than being basic, these
proteins are acidic and bear a net negative charge. A classic nonhistone protein is
RNA polymerase. This enzyme synthesizes an RNA transcript from a DNA
\ template.
K There are five types of histones, designated as H1, HZA,IIZB, H3, and H4. The
association of these histones with a specific length of DNA defines a structure
referred to as a nucleosome. Each nucleosome repeats itself, on average, about

B
DNA
every 200 base pairs (Figure 6-41). The region that separates each nucleosome is
called the linker. There are about 30 million nucleosomes associated with the
DNA in each nucleus.

Many
s
packing
steps
vtl A string of nucleosomes

Figure 6.41
Nucleosomes containing DNA and histones has the appearance of beads on a srring.

The core histones (H2A,F{ZB,I{3, and H4) bind roughly one-and-three-quarters


A replicated and turns of DNA, or about 146 base pairs. This association allows the length of the
condensed chromosome DNA to decrease by about a factor of six.
Figure 6.42
There are many different levels The H1 histone is though to play an important role in pulling the individual
of packing that are presumed to nucleosomes together to create a 30-nm chromatin fiber. This fiber, which is about
give rise to the final, highly
L mm in length, is still much larger than the diameter of the cell's nucleus.
condensed chromosome
Further packing of the DNA is needed, and this is where current speculation
begins. It could be that there is an ever increasing number of coils until a
particular higher-order structure of chromatin is established (Figure 6-42).

Copyright @ by The Berkeley Review 26 The Berkeley Review


Specializing in MCAT Preparation
Biotogy Structure & Function in CeIIs & Viruses Eukaryotic Cells

The Cell Cycle, Mitosis, & Meiosis

The Cell Cycle


The cells of a eukaryotic organism complete a cell cycle for either reproductive,
growth, or replacement purposes (Figure 6-43). The cycle occurs in both haploid
and diploid cells and is defined as the sum of all events that occur between the
completion of one cell division and the next. The rate of cell division varies
between different cells, and some, like striated muscle cells and nerve cells, never
divide. A typical eukaryotic cell cycle lasts anywhere from 16 hours to 24 hours.
Let's briefly consider the events of a cell cycle.

The Phases of the Cell Cycle


The three major stages of the ce1l cycle are interphase, mitosis, and cytokinesis.
Interphase can be divided into the G1, S, and G2 phases. Mitosis (M) is divided
urto prophase, metaphase, anaphase, and telophase. Mitosis is usually followed Figure 6'43
:r' cytokinesis, that portion of the cell cycle that allows for partitioning of the A generalized eukaryotic cell cycle.
:ontents of the cytoplasm into two new daughter cells.

tnterphase
During the first growth phase (Gi), which lasts about 10 hours, RNA and
:roteins are actively being synthesized. The centriole pair separates in
:reparation for the synthesis of daughter centrioles in the S phase. The cell and
::s nucleus begins to increase in volume. If a cell is destined to never divide
.gain, it will remain arrested in G1.

:ach of the 46 strands of chromatin, with the exception of the centromeres, are
:eplicated by the end of the synthetic phase (S). This event lasts between 6 to 8
r.crurs. If we could visualize the chromatin at this stage, we would find that it
', ould be extended and loosely coiled. Even though the DNA has doubled to

. -rm 92 sister chromatids, the chromosome number has remained the same at 46.

l,aughter centrioles are synthesized at right angles to the parental centrioles. This
::ccess is completed by mitosis.

-:e second growth phase (C2) lasts about 2 to 6 hours. During this phase the
-- :omatin is beginning to condense and become more tightly coiled. Protein
i .:.ihesis is quite active and the cel1 prepares for mitosis.

![itosis
l, titosis is simply nuclear division, and involves the equal partitioning of genetic
: a:erial to two new daughter cells. The phases of mitosis are outlined below.

Prophase
Microtubules
-,.:he beginning of prophase the two centriole pairs begin to move apart.
:-:iotubules begin to radiate from each pair in all directions, forming a star-like
,::-:.ture called an aster. The region from which the microtubules extend
, -.:',.''ard is called the centrosome, or microtubule-organizing center (MTOC). Kinetoc
.1.::otubules leaving the centrosome will eventually form the mitotic spindle, a
:r:--::ture involved in the separation of the chromosomes during anaphase. By Metaphase chromosome
-= :nd of prophase the condensation of the chromatin is complete and the
Figure 6-44
,.-:-i:rlosomes, each containing a pair of sister chromatids held together at the Microtubules attaching to the
:r irorrr€r€, can be visualized using different staining techniques. Many of the kinetochores.
- ,::rtubules leaving the centrosome attach at the kinetochore (Figure 6-44), a
,:-:-alized area closely associated with the centromere. Other microtubules

: . rrght @ by The Berkeley Review 27 The Berkeley Review


Specializing in MCAT Preparation
Biology Structure & Function in Cells & Viruses Eukaryotic Cells

extend from the centrosomes to the equatorial region of the nucleus without
contacting the kinetochores. The nucleolus disappears and the nuclear membrane
begins to break down.

Metaphase
The fully condensed chromosomes align themselves along the equator of the cell,
a region also referred to as the metaphase plate. By this time the nuclear
membrane has completely disappeared.

Centromere

\';?-* ]tr
XN\
5
(h

Sister

I-ate Interphase (2N) Prophase (2N) Metaphase (2N)

^^.
I Cytokinesis is complete and two new
daughter cells are formed Daughter chromosomes
t

if _,

Nuclear envelope
begins to form
Early Interphase (2N) Telophase (4N) Anaphase (4N)

Figure 6-45
Mitosis showing a nucleus containing four nonhomologous chromosomes. In this diploid cell, N
refers to the number of chromosomes.

Anaphase
The centromeres of each chromosome aligned on the metaphase plate diaide and
the two sister chromatids, which can now officially be called daughter
chromosomes, move towards opposite poles. At this moment in the cell cycle we
should have a total of 92 chromosomes; 46 are moving towards one pole and 46
are moving towards the opposite pole. Movement occurs because of microtubule
depolymerization at the region of the kinetochore. Cytokinesis, the cleavage of
one cell into two daughter cells, usually begins late in anaphase.

Copyright @ by The Berkeley Review 2A The Berkeley Review


Specializing in MCAT Preparation
Biology Structure & Function in Cells & Viruses Eukaryotic Cells

Telophase
,"':lr the start of telophase all daughter chromosomes have reached their
:=.:e:tive poles. Each chromosome begins to uncoil and extend itself. The
::-::olubules of the spindle apparatus begin to disappear as a nuclear membrane
- :::r-s around each of the two daughter nuclei. The nucleolus (or nucleoli)
:::::ears. The cleavage furrow of cytokinesis continues to deepen.
Cltokinesis
--"'::'srnesis simply involves the cytoplasmic division of a cell into two daughter
:
=-
Ti,e process generally begins during late anaphase and when it is complete,
i. ::.a j the end of mitosis (or meiosis). Mitosis is outlined in Figure 5-45.

!!eiosis
:- -:::r.: diploid cells (2N) which are destined to become haploid gametes (N)
--:::;r DNA replication and then two successive nuclear divisions. This
::::-:s rs called meiosis and the formation of the gametes is called
;.;:-.etoqenesis (discussed in the section onReproduction and Deaelopment). In the
: r l-: --i.3 gametes are called eggs (ova) and are produced in the ovaries. The
:".i.: .::-:e:es are called sperm (spermatozoa) and are produced in the testes.
" : I . successive nuclear divisions in meiosis are called meiosis I and meiosis
: , --r :,erotic divisions follow the same four phases in mitosis: prophase,
* : r:-.e. alaphase, and telophase. In meiosis we distinguish those phases by
r.i r
' l--: i --:.e Roman numeral I or II after the phase name (e.g., prophase I or
:-;. IIi. Meiosis I and meiosis II are each preceded by an interphase. The
',r-:-, --: -r,terphase before meiosis I proceeds through the familiar G1, S, and
- ;.':'- -';. During the S period the DNA replicates and during the G2 period the
:'-:, ==i'.: to meiosis takes place. After the first nuclear division one cell with
.": :.::orLes becomes two cells, each with 23 chromosomes. This is
' r' : i-i-:i :alied a reductive division. Each of these two cells will enter into the
:-r -::.: ::-.at precedes meiosis II. During this second interphase the S period
'" - .: : '-.: and so the DNA cannot be replicated again. After these cells leave
r:1-- - '-nev enter into meiosis II. At the end of the second nuclear division
:--r r=rr.iln, each with 23 chromosomes. What we would like to do is
: :: j ::,e events of meiosis at a basic level and then, during a later

Prophase I
s::ge of meiosis (Figure 6-46), prophase I is quite long and
::r,pared to prophase of mitosis. Prophase I can be divided into
-:lc:ene, zygotene, pachytene, diplotene, and diakinesis. Each of
: -:.:ains terminology which is important to meiosis and important
""r : : troughout prophase I we will find 46 chromosomes or 92
- - r- -^tl
- - -e11.

-,sptotene. The replicated chromosomes have already started to


:::*... a:-Li nolv become visible. If we wanted to, we could assign 23 of
'-
-' ,:: - i - rr,-rSorrl€S (i.e., 46 chromatids) as being maternal and 23 as being
I- : -l:, -:. --,ilglrl.

t]]ilrni fl
":"' l'-: Berkelev Review 29 The Berkeley Review
Specializing in MCAT Preparation
Biology Structure & Function in Cells & Viruses Eukaryotic Cells

Zygotene. Homologous chromosomes begin to pair up longitudinally


in preparation for crossing-over (genetic recombination). This is referred
to as synapsis. A specialized protein and RNA scaffold called the
synaptonemal complex (Figure 6-47a) appears between the pairing

t chromosomes and facilitates their union. The homologous chromosomes


which have undergone synapsis are referred to as a bivalent, and
because there are four chromatids in a bivalent the paired homologous
Chromatin chromosomes are also referred to as tetrads. How many tetrads are there
hemeiotic Synapsis at this stage? Only 23. How many centromeres are there in each tetrad?
Interphase Prophase (2N) Just two. How many chromosomes would we find? Based on the number
(2N) of centromeres, we would fl::.d 46. How many pairs of chromosomes are
centromere + there? We could count 23 pairs at this stage.

D;. ads are (a) (b)

,,:*ffir.r\wl;,:r:
p'.rlled
r: poles

TzlS
*P, -fP=A.> |
c"nro^rK ,Kn ol
K.w-="::l:'*..
-\\;!--,
Microtubules-1l-
chiasma

\TT Figure 6,47


Formation of the synaptonemal complex between a pair of homologous chromosomes.
F,r\ l/,+
t\\ri,v? Pachytene. The chromosomes continue to condense and become much
more distinct. Genetic recombination occurs through a process called
I

il

Anaphase crossing-over. {

(2N)
Nuclear envelope
Diplotene. As the homologous chromosomes begin to separate, the I
(
begins to fonn events of crossing-over become visible at structures called chiasmata
Cleavage furrow (singular: chiasma). At each chiasma any one of the four chromatids may I
deepens be involved in a cross-over event. For each bivalent, there is an average
of about 10 cross-overs, and this leads to a very thorough mixing of the /
maternal and paternal chromatids. I
{
In the section on Reproduction snd Deuelopmenl we learned that the
developing eggs (oocytes) in a female are arrested in diplotene of
prophase I by about the seventh month of gestation (fetal development)-
Those eggs wilt not be able to continue their meiotic adventure until the
onset of puberty. Even then just one egg per month will have the
Two new daughter capacity to develop and be fertilized. The rest will be left in diplotene,
cells are formed
some residing in that state for more than 50 years.

Diakinesis. While the nuclear envelope begins to break down and the
Meiotic Interphase nucleoli disappear, the chiasmata move along the lengths of the
(N) chromatids until they reach the ends. As the homologous chromosomes
begin to separate they appear to be joined to each other at their ends.

Figure 6-46 Metaphase I


Meiosis I showing the different The homologous chromosomes align on the metaphase plate. The centromeres of
stages that lead to the formation
each pair appear to be quite distant from each other. It seems as if the only parts
of two haploid cells.
of the chromosomes touching one another are the ends. Microtubules from the
spindle apparatus are attached at the centromeres and the nuclear membrane has
disappeared.

Copyright @ by The BerkeleY Review 30 The BerkeleY Review


Specializing in MCAT Preparation
Biology Structure & Function in Cells & Viruses Eukaryotic Cells

Anaphase I
As the microtubules puII the homologous chromosomes apart, the
centromeres do not divide. The pull on each chromosome is from one
direction only. Each chromosome that migrates toward a pole is still
composed of sister chromatids and this pair is referred to as a dyad
@*;*@
+ + (N)
(Figure 6-47b). Cytokinesis beings.

Telophase I
'rZAr\- /z=A\\
The migrating chromosomes (dyads) complete their movement to '/d"*\\\ G+N
-rpposite poles and the nuclear membrane reforms. Cytokinesis is more \lt \*'ll'll
:ronounced.
\\\tr"b/1
\\\\ /-r',/ N\\="94
\\\\ /-.7 /
\\)i$:j-l-, Prophase
\'\iFZl
Cytokinesis
-fter cytokinesis is complete we find that a dipioid cell with 46
I (N)
I
,fl, N\
:lromosomes has divided into two haploid cells, each with tZ)
7N\i
23
-jlromosomes. Since there is a reduction in the number of chromosomes,
-:-.rs is aiso called a reductive division. \ \\\
,17,
Interphase II
,:.e second (meiotic) interphase is usually brief. DNA repiication
..,ie place during the S phase of this interphase.
does not \\4
',trifl*)r,

I
Metaphase
(N)
I
{4,t
'leiosis II: Prophase II
, - - stages of meiosis II (Figure
:.'-:.'rsis. As
6-48) are quite similar to the stages of
in prophase of mitosis, the chromosomes in prophase II of Zzf;N
-,osis II begin to condense. The microtubules of the spindle apparatus
,: :--h to the kinetochores of each chromosome. #N rfi \li
\ifl A#
trfetaphase II
'.,.mosomes line up along the metaphase plate as the haploid cells
Anaphase
\\#2
: :::3re for division.
+ (2N)
I
{naphase II
.. ,-:',e centromeres begin to divide the chromatids of each of the 23
-: ,:rlosomes are pulled to opposite poles of the dividing cell.
: -<rnesis is in progress. The chromatids which separate from one
- :-'.er can officially be called chromosomes. There are now 46
r ::'.osomes in the dividing cell. An interesting technicality arises
-,--:: anaphase IL The chromatids of each chromosome that are
: , :::tng cannot be referred to as sister chromatids. Why? Think about
, -i-:ierence in DNA between the replication of the chromatin during
' : rSi interphase and the events of crossing-over during pachytene of (:W,t
' ::.:.se I of the first meiosis. The DNA is no longer the same as there r\W2,,
, ir::r1 genetic recombination. Telophase

-:lophase II I (2N)
I
--=ar membrane forms around the decondensing chromosomes. The

@@@@
..
, : ,r t€ furrow created by cytokinesis deepens.

- 't,rkinesis Haploid Gametes (N)


:,- :.-.e completion of cytokinesis four haploid cells, each having 23
' . - somes, will have been derived from one diploid cell. Each of Figure 6-48
-: , -ir ce11s enters into interphase where they are arrested at the G1 Meiosis II.
- - lhese cells will remain quiescent until fertilization takes place.

O by The Berkeley Review The Berkeley Review


Specializing in MCAT Preparation
Bionogy Structure & Function in Cells & Viruses Eukaryotic Cells

Asexual & Sexual Reproduction


In asexual reproduction, a new organism develops without a sexual process,
either from a single cell or a group of cells. It can occur in either haploid or
diploid organisms. The offspring produced are identical to the parents. \Atrhat is
an example of a eukaryote that produces asexually (Figure 6-49)? Yeast.

Animals that reproduce sexually have two types of cells: germ cells and somatic
cells. As we have seen, haploid germ cells are produced by the process of meiosis
while diploid somatic cells are produced by the process of mitosis. \Atrhen haploid
gametes from a male and a female fuse together, a diploid zygote is formed. This
process, termed sexual reproduction, involves the alternation of diploid and
haploid phases of an organism's life cycle. It occurs only in diploid organisms.
The offspring that rcsult are nof identical to the parents due to extensive genetic
recombination. What is an example of a eukaryote.that produces sexually (Figure
6-50X Humans.

0
n Sperm ftil fA)
0

fure 6-a9
Srmrmm"! oi asexual reproduction.
W1 f ;*,3tU t_UJ

ilQ..,.*wrr,/^t8 Tt-*'
fl[--1[*$\ chromosomes \a/B
c11 o
ffi
ffi'
r0r Meiosis

Figure 6-5O
Summary of sexual reproduction.

Copyright @ by The Berkeley Review 32 The Berkeley Review


Specializing in MCAT Preparation
Biotogy Structure & Function in Cells & Viruses Eukaryotic Cells

fldtr,ebu[ iii:OfghiliZatidil
Biomembranes & Membrane Transport
The major components of both eukaryotic and prokaryotic membranes are lipids,
proteins, and carbohydrates. Even though there will be differences in the specific
molecular components of eukaryotic and prokaryotic membranes, we will find
that their structural organization is similar. in both cases we will see membranes
that are dynamic.

Biomembranes H,c-oA.'v- R

Glycerophospholipids, sphingolipids, and cholesterol are the three major lipid lo


components of eukaryotic biological membranes. Amphiphilic molecules like the
glycerophospholipids and sphingolipids present a shape similar to that of a
rectangle. At one end of the rectangle the molecule is polar (hydrophilic) while at "L- "A--^,^ -
the other end it is nonpolar (hydrophobic). In biological membranes the two
most abundant glycerophospholipids are phosphatidylcholine (Figure 6-30) and
lhosphatidylethanolamine (Figure 6-51). We can let these amphiphilic molecules
H2C- O-
?_o{.",
oo
be represented by the drawing shown in Figure 6-52. The hydrophobic tails of
:he phospholipids represent the fatty acyl side chains containing the methylene
*roups while the polar head is represented by the phosphate group and its Figure 6.5 I
associated molecular attachments. Phosphatidylethanolamine.

Lipid Structures
The three structures that can be formed by phospholipids are micelles, lipid
:Llayers, and liposomes. Micelles are spherical structures that are formed when
=;rough phospholipids congregate together such that the polar heads interact
,",-ith water while the hydrophobic tails exclude water (Figure 6-53a). In our
:iscussion on gastrointestinal physiology, we will see how bile acids form
Polar
:rrcelles and act to solublize fats during digestion.
Head

gl Hydrophobic
Tail

Figure 6-52
General representation of a
phospholipid.
Figure 6'55
l:oss sections of a (a) micelle, (b) lipid bilayer, and (c) liposome.

Lipid bilayers are formed when the hydrocarbon tails of two phospholipid
.:.eets interact with one another to exclude water (Figure 6-53b). These structures
::e stabilized through hydrogen bonding and electrostatic interactions of the
::ad groups with water and by the hydrophobic van der Waals interactions
--:trveen the hydrocarbon side chains. The average thickness of a lipid bilayer is
:out 60A (or 6 nanometers). if a lipid bilayer folds back on itself, a hollow
::ueous-filled structure will form called a liposome (Figure 6-53c).
Lipid Mobility
-:.e individual phospholipids in the plane of a bilayer have a great deal of
:.obi1ity. Neighboring phospholipids can easily exchange places by a process
-: led lateral diffusion. However, movement of a phospholipid from one lipid
: -ane to the next, a process called transverse diffusion or flip-flop, is a very rare
= " ent (Figure 6-54).

- pr right O by The Berkeley Review .'J The Berkeley Review


Specializing in MCAT Preparation
Biology S,ttnncture Er Function in Cetls & Viruses Eukaryotic Cells

Phospholipids that have shorlcharn fatty acids and fatty acids with increasing
sites of cis unsaturation (i.e., double bonds) tend to increase their mobility (i.el
mf'' fluidity) in the membrane. The greater the motion of the fatty acid side chains,
lr the more fluid the membrane. Addition of cholesterol to a'membrane acts to
decrease fluiditr-. The planar.steroid ring inserts between neighboring fatty
acid
1
side charns and interferes with the movement of those chains."Memfrine Rliaity
-:.;
-::-s:,rn is also dependent on temperature. Cooler temperatures act to decrease
- :"St] membrane iluidity while higher temperatures act to increase it.
ats Membrane Proteins
,itllll A r-arietv of different proteins can be associated with the lipid bilayer. However,
each of these proteins will have an operational classification. For example,
_,_rJluuu integral membrane proteins are embedded in the lipid bilayer. These proteins
can either span the bilayer, in which case they aie called trans-membrane
proteins, or they can be embedded in the membrane and exposed at just the
exterior surface or interior surface of the bilayer. Integral membrane proteins are
tightly bound to the lipid bilayer by hydrophobic forces and harsh uru
mffifif\ffin needed to remove them. once removed the membrane is usually disrupted."o.,ditions

[uuruus Peripheral membrane proteins are weakly attached to the surface of the lipid
bilayer, either through hydrogen bonding or electrostatic associations. Removal
of these proteins occurs under mild conditions and the membrane is usuallv left
intact. In Figure 6-55 we see a schematic diagram of these general classls of
proteins.

Glycolipid
Integral
Figure 6-54
Diffusion of phospholipids in a
lipid bilayer.

Figure 6.55
A schematic diagram showing the relationship between integral and peripheral proteins.
WU
Glycolipids & Glycoproteins
If a membrane lipid is attached to a carbohydrate, it is referred to as a glycolipid.
The majority of glycolipids are found o.r ih" exterior surface of a lip"id bilayer. fiq
Carbohydrates that are attached to membrane proteins are called glycoproteins. ti
All sugar residues are found attached to proteins that face the exterior of the l:ir,'gM

membrane. one important class of glycolipids and glycoproteins that we will be wl,..,r[1

discussing in a later section are the human blood group antigens that make up
the A, B, and O blood groups. lm
,41
Membrane Transpotr lIWH
The cell membrane acts as a permeability barrier that selectively allows mole- M
cules to enter and exit the celi. There are five processes by which this can occur: dhl*
(1) simple diffusion, (2) facilitated diffusion, (3) active transport, (4) group "!F}tm

3 by The Berkeley Review 34 The Berkeley Review


Specializing in MCAT preparation
Biology Structure & Function in Cells & Viruses Eukar5rotic Cells

-:"nslocation, and (5) bulk transport. Let's take a brief look at each of these types
-: rembrane transport.

Simple Difftrsion
--:.-:irer name for simple diffusion is passive diffusion. Simple diffusion
-- , ,:-r-es the spontaneous movement of solute molecules through a lipid bilayer, Solute/Diffusion Rate
r: l. an area of high concentration to an area of low concentration, down a
:,:.:entration gradient. The relative rates of equilibration of a few molecules Hydrophobic
r :r-:s a lipid bilayer is shown in Table 6-5. o,2,N2,coz
Diffuse very fast
Fa,cilitated Difftrsion
Polar Uncharged (small)
ln;rliiated diffusion (or carrier-mediated diffusion) is similar to simple diffusion
Water, Urea, Ethanol
,- --:.1: rt also takes place down a concentration gradient. However, in this case
',,-:::',olecules cannot simply pass through a lipid bilayer. Instead, diffusion Diffuse fast
-=:=::s on the interaction of the solute molecules with integral membrane Polar Uncharged (large)
r.i r. r;-rLS :hat are embedded in the lipid bilayer. If one type of solute molecule Glucose, Galactose, Fructose
; lr::r -:r:ough a transmembrane protein in one direction, the transporter is Diffuse s1ow1y
r-,:: : uniport. If two different types of solute moiecules pass through a Ions
r':rrrrrr-rc rn the same direction, the transporter is called a symport. If a yu@, g@, gu2@
:,--.:: -::er al1orvs for the passage of two different solute molecules through a
Diffuse very slowly
-r :_ri;i,e in opposite directions, it is referred to as an antiport (Figure 6-56).
:-
. --.-- ::ar*<porters that allow for the movement of a solute across a membrane Table 6-5
; r.-: - . --;rili'€lr- called permeases. Symports and antiports are also referred to as Lipid solubility of select solutes.
:;a::sporters. Both facilitated diffusion and simple diffusion are examples of a
r ,1rli. "{ e t:ansport system.

Antiport

$il$flmq rl" 56
, . rli |: " ,:..it , -::'::,:::i:t.lr oi three different types of transport systems.

4|nnthrw'e 1flr-alrsport
"': -: --=--.:: ::ansported against its concentration gradient, energy will be
. --., : -: -:: :--:n or another. We will divide active transport into primnry
,rrriiil 'rlirrr' rr'r-lr .': "': ::. j ::--iidnry ACtiae trAnSpOrt.

"' i-llrm.;l*l a:C.-e transport the classic energy source is adenosine triphosphate
-":i :l .r :-*= :-:-.sr. erample involves the Na@-K@ pump.This pump is also
1il,l ,rl'i ri i:i -*. \a3-K3 ATPase and its primary responsibility is to set and
Illr ;-l -'"i: --it::-l1.ilar concentrations of Na@ and KO. The concentrations of
ilrLiii;iui:

ttlrilrl : - - :.-.---:. -.ns are quite different between the intracellular and
'r l:*iiL i I r.-i,: :,.; - :.-. c,: ne ce1l (Table 6-6). The intracellular K@ concentration is
.

,, tllllllil t' nr"l .'5 The Berkeley Review


Specializing in MCAT Preparation
Biology Structure & Function in Cells & Viruses Eukaryotic Cells

about 140 mM while that of Na@ is about 5 mM to 15 mM. The extracellular K@

concentration is about 5 mM while that of Na@ is about L45 mM.


llnsidel IOutside]
The imbalance of these two ions across the plasma membrane of a cell is
lon (mM) (mM)
maintained by the Na@-K@ ATPase. This_pump is an antiport that transports two
K@ ions into the cell for every three Na@ ions out of the cell (Figure 6-57). The
Na@ 5-15 145
energy required for this transport comes from the hydrolysis of ATP to ADP and
6@ t4o 5 li (iiorganic phosphate). It has been calculated that in nerve cells about 70% of
ca2@ l-2 3-5 the cell's energy is used to maintain these pumps.

cro 4 110 Note that as three Na@ ions move out of the cell and two K@ ions move into the
Table 6-6 cell, a net separation of charge (an electrical potential) is established across the
Common ionic concentrations inside plasma *"*btut". The inside of the cell becomes negatiue with respect to the
and outside the cell. Lutside. The membrane potential that is established is generally in the range of
-50 millivolts (mV) to -70 mV. We will come back to this point during a later
discussion.

Another important membrane pump is the Ca2o ATPase' This protein ensules
that the Ca2b concentration within the cell is always at a low level by pumping
two Ca2@ ions out of the cytosol for every ATP hydrolyzed.

Ionic gradients that have been established by primary active transport systems
can piovide a driving force that allows for the cotransport of other molecules
against their concentration gradients. This process is called secondary active
tiansport and a classic e*arttple involves the cotransport of Nao with glucose
into specialized intestinal or kidney cells (Figure 6-53). The symport protein that
facilitates this process has two binding sites; one for NaO and one for glucose'
Figure 6-57 Both need to be occupied in order for the translocation to occur. The transport of
The Na@-Ko ATPase. Na@ into the cell o."irc because of (1) the concentration difference of Nao across
the cell's membrane (high outside and low inside) and (2) the attractive net
negative charge inside the cell. Once Na@ and glucose are brought into the cell,
Nab is pnttlp"a out through the Na@-K@ ATPase. This helps to reestablish the
electroclhemical gradient across the plasma membrane. Other secondary active
transport systems cotransport Nao and amino acids into the cell'
Glucose Na@

Group TYanslocation
Examples of group translocation can be found in certain bacteria. In this process
u ,rrgu, residire liie glucose is phosphorylated as it is being transported through
the ilasma membrane. This type of transport is coupled to cellular metabolism.
We will return to it when we examine the regulation and metabolism of lactose.

Bulk TransPort
Many animil cells will show an invagination of a portion of their plasma
*"rrrbrunu that will eventually pinch off to form an internalized vesicle' This is
Figure 6-58 called endocytosis and the vesicles that are formed are called endosomes or
An example of secondarY active endocytotic vesicles. Within these vesicles is a portion of the extracellular
transport. environment. If the invagination resulted in a vesicle that contains the liquid
portion of the extracellutai environment, the process is referred to as pinocytosis
ior cell drinking). F{owever, if the vesicle contains some type of particulate
matter, the process is referred to as phagocytosis (or cell eating). As these
vesicles move toward the interior of the cell some will fuse with others to form
larger structures. Material can also be released from the cell through a plocess
.utLa exocytosis. Endocytosis and exocytosis are both examples of bulk
transport.

36 The BerkeleY Review


Copyright @ by The BerkeleY Review
Specializing in MCAT PreParation
Biology Structure & Function in Cells & Viruses Dukaryotic Cells

l\ucleus, Nucleolus, & Kibosomes


SucIeus
.:.e most prominent structure in a eukaryotic cell is a double membrane-bound
::sanelle called the nucleus (Figure 6-59). The inner membrane of this nuclear
=:.-,-elope surrounds the majority of the cell's genetic material (DNA). The
:enLaining genetic material in an animal cell can be found in the mitochondria.
l:.e outer membrane becomes part of an extensive membranous system found in
re cytoplasm called the rough endoplasmic reticulum (RER). Between the inner
::.d outer membrane is the perinuclear space.

).rrrng interphase of the cell cycle the DNA is referred to as chromatin, and the
:-irture of chromatin and the aqueous phase of the nucleus is referred to as the
nucleoplasm. During mitosis the extended chromatin begins to condense into
:ghtly coiled structures called chromosomes. We will see that within the nucleus Figure 6-59
fNA is involved in two important events. The first is DNA replication while the The nucleus.
.econd is the transcription of DNA into messenger RNA (mRNA). Translation of
nRNA into protein (i.e., protein symthesis) occurs in the cytoplasm of the cell.

Scattered throughout the nuclear envelope are thousands of nuclear pores with
:hannel diameters that range from 10 nm to 20 nm. These pores are areas where
:he inner and outer nuclear membranes come together to form passage ways that
allow for the two-way flow of selected materials between the cytoplasm of the
cell and the nucleoplasm of the nucleus. Since protein synthesis cannot take place
-n the nucleus, all nuclear proteins found within the nucleoplasm must be
imported from the cytosol. The exact mechanism that allows these proteins to be
transported into the nucleus us unknown, but it has been postulated that the
transport process is energy-dependent and involves protein signal sequences.

The major type of protein associated with nuclear DNA are histones. These
proteins are rich in the amino acids lysine and arginine. At physiological pH
these basic amino acids bear a positiae charge on their side chain and can associate
rvith the negatiuely charged DNA through electrostatic linkages. Histones are
thought to be involved in DNA folding and the highty condensed chromosomal
structures observed during metaphase of mitosis.

Nucleolus
Within the nucleus is a highly organized region called the nucleolus. The
nucleolus, which is not a membrane-bound organelle, is centered around certain
chromosomes that contain nucleolus organizer regions and is involved in the
synthesis of ribosomal RNA (rRNA). The rRNA which is transcribed from
specific genes in the DNA of the nucleolus organizer regions associates with
ribosomal proteins in the nucleoplasm. Together the ribosomal proteins and
rRNA form two different ribosomal subunits, one small and one large, that are
then transported out of the nucleus through the nuclear pores and assembled in
the cytoplasm into complete and functional ribosomes that carry out translation.
If a cell is quite actively involved in protein synthesis, one would expect the
nucleolus to be larger than if a cell were not as actively involved in protein
synthesis.

Ribosomes
Eukaryotic ribosomes are the sites of protein synthesis. They are composed of
two subunits, each differing in size and content of RNA and protein. The size of a

Copyright @ by The Berkeley Review J/ The Berkeley Review


Specializing in MCAT Preparation
Biology Structure & Function in Cells & Viruses Eukar5rotic Cells

ribosome and each of its component parts is based on a sedimentation coefficient,


s, that is expressed in svedberg units (s), where one s = 10-13 s. The rate at
which a molecule sediments in an ultracentrifuge tells us something about its
mass (i.e., molecular weight). The sedimentation coefficient of a complete
eukaryotic ribosome is expressed as 80S.

If we separate this ribosome into its two component parts (Figure 6-60), we find
that the large subunil sediments at 605 while the small subunil sediments at 40S.
These values are not additive. In other words, 605 + 40s does not equal \00s.
Sedimentation coefficients are not linearly related to molecular weight as they
depend on the size and the shape of the molecule.
80s Ribosome

The overall dimensions of a complete ribosome is about 20 nm by 30 nm and


u contains roughly 60% rRNA and 40h protein. The small subunit is about 9 nm in
diameter and contains roughly half rRNA and half protein. The large subunit is
{, roughly 25 nm in diameter and contains about 65% rRNA and 35% protein.

The point here is not to memorize values but rather to think about dimensions
and content. We just learned that ribosomal subunits are transported from the
nucleus to the cytoplasm where they are assembled into complete and functional
ribosomes. Based on what we have discussed we would e*pect the small subunit
605 Subunit to pass through a nuclear pore with relative ease. The large subunit seems to be
at least as large as the diameter of a nuclear pore. what would need to happen to
rRNA allow for its passage into the cytoplasm?
Protein
What are the two major types of biological molecules contained in a ribosome?
40S Subunit
Nucleic acids and amino acids. \zVhich nucleic acids are directly associated with
Figure 6-6O ribosomes? In the section on Expression of Genetic Information we will learn that
The components of a during protein synthesis ribosomes are associated with IRNA, mRNA and
eukaryotic ribosome. transfer RNA (IRNA).

In any eukaryotic cell that is involved in protein synthesis there are hundreds of
thousands if not millions of ribosomes within the cell. In eukaryotic animal cells
ribosomes are not only found floating free in the cytoplasm, but they are also
found associated with a membranous structure called the endoplasmic
reticulum. what determines if a ribosome remains free in the cytoplaim or is
bound to the endoplasmic reticulum? It depends on what happens during the
initial stages of protein synthesis. All ribosomes involved in the translation of
nuclear genetic information begin polypeptide synthesis in the cytoplasm. If a
particular ribosome is destined to be bound to the endoplasmic reticulum, then
during the initial stages of protein synthesis a short signal peptide is made
which directs the ribosome to the endoplasmic reticulum. This model, called the
signal hypothesis, will be discussed shortly.

Ribosomes are also found in the matrix of the mitochondria. Ribosomes found in
the mitochondrial matrix not only differ in RNA and protein content from those
found in the cytoplasm, but they are also smaller and sediment at about 55S. This
smaller sedimentation coefficient is close to the sedimentation coefficient of
prokaryotic ribosomes, which is about 70S.

fr

Copyright @ by The Berkeley Review 3a The Berkeley Review


Specializing in MCAT Preparation
Biology Structure & Function in Cells & Viruses Eukaryotic Cells

Endoplasmic Keticulum & Golgi Apparatus

Endoplasmic Reticulum
The endoplasmic reticulum (ER) is a network of membranes that is found within
the cytoplasm of most eukaryotic cells and is continuous with the outer nuclear
membrane. The space enclosed by this membranous system is referred to as the
lumen. The ER can either be smooth, in which case it is called the smooth endo-
plasmic reticulum (SER), or it can be studded with ribosomes, in which case it is
called the rough endoplasmic reticulum (RER). The ER is the largest membrane
system in a eukaryotic cel1.

Smooth Endoplasmic Reticulum


The SER has a membrane system that lacks ribosomes and appears more tubular
in shape. The SER is involved in the synthesis of a majority of the cell's
membrane lipids, including the neutral fats, phospholipids, prostaglandins, and
steroid hormones. Specific integral membrane proteins embedded in the SER act
as enzymes that help catalyze these reactions.

The SER, especially of hepatocytes (liver cells), is involved in hydroxylation


reactions that aid in the detoxification of drugs. Hydroxylating a compound
makes it more water soluble, thereby making it easier to be eliminated from the
body.

In hepatocytes the SER also plays an important role in the catabolism of liver
glycogen. Recall that glycogen is a storage polysaccharide consisting of many
;lucose residues linked together in o(1-+a) and u(1+6) linkages. If blood glucose
-evels fall below normal values, a hormonally mediated set of events occur that
allow for the breakdown of glycogen and the release of glucose. One of those
:i-ents involves the enzyme glucose-6-phosphatase. As glycogen is being broken
iown, a molecule of glucose-6-phosphate will eventually be produced. The
:1zyme glucose-6-phosphatase is embedded in the membrane of the SER and
:atalyzes the removal of the phosphate group at the C-6 position of glucose-6-
::.osphate. The desired product is glucose, as shown in (6-6). Because glucose is a
eutral compound (i.e., it no longer has any negative charges due to the
:rosphate group) it can readily pass from the cytoplasm of the hepatocyte,
--:ough a permease in the plasma membrane, and into the blood.

G luc o s e - 6 -p ho s p hatas e
Glucose-6-phosphate + Hzo Glucose + P; (6-6)

-- :ertain cell types the SER regulates Ca2@ levels. For example, in muscle cells
: = SER is referred to as the sarcoplasmic reticulum (SR). The SR sequesters Ca2@
.-.s and when stimulated by a nerve impulse releases those ions into the
: .:plasm of the muscle cell. The result is a contraction of actin and myosin
' -:rents in the muscle cell.

Kough Endoplasmic Reticulum


-- = RER has a membrane system that is generally flat and sheet-like. Ribosomes
,-': line the cytoplasmic face of the RER are bound to the membrane by their
::;e (605) subunit. The ribosomes synthesize membrane and secretory proteins
r':: are then passed through the membrane of the RER and into the lumen where

:reht @ by The Berkeley Review 39 The Berkeley Keview


Specializing in MCAT Preparation
Biology Structure Er Function in CeIIs & Viruses Eukaryotic Cells

post-translational modification begins. In other words, after synthesis of the


polypeptide chain by the ribosome, certain amino acids are modified by
hydroxylation and glycosylation events. After a brief period of time in the lumen
of the RER, these proteins are shuttled to the Golgi apparatus (see below) where
post-translational modification continues. These modified proteins are directed
to their final destinations by the Golgi apparatus.

Signal llypothesis
What is the general mechanism that accounts for the passage of proteins into the
lumen of the RER? Recall that proteins encoded by nuclear genes begin the initial
stages of their synthesis in the cytoplasm of the cell. Some of these proteins have
at their N-terminus a sequence of amino acids that act as a signal sequence
which binds to a signal recognition particle (SRP) in the cytoplasm, Translation
is temporarily halted as the SRP directs the large subunit of the ribosome to a
signal sequence receptor embedded in the membrane of the RER. Once the
ribosome docks at the membrane receptor protein, polypeptide synthesis
continues and the signal sequence is inserted through the membrane of the RER.
The released SRP is free to bind to another developing signal sequence in the
cytoplasm. Meanwhile, the signal sequence that was passed into the lumen of the
RER is cleaved and degraded by the enzyme signal peptidase. Modification of
the growing polypeptide chain begins. After completion of protein synthesis, the
ribosome dissociates from the RER (Figure 6-61). Non-membrane proteins
synthesized by the ribosome are released into the lumen of the RER. These
proteins are usually destined for secretion or incorporation into lysosomes. Other
proteins may span the membrane of the RER, becoming integral membrane
proteins that are destined for the plasma membrane.

Signal
sequence
7--.\ ,' 6os

-)
OL--J
5'
SRP.-
-----=
\ 4os
Ribosome
a
@

o*(, 'r**ffi'oo"
J,,, potypeptide
,.qi.n.. tlW O
Modified ?^
RER Lumen protein - ---

Figure 6-61
Synthesis of secretory proteins in the lumen of the RER. O Ribosomal subunits associate on
mRNA. @ Signal sequence leaves large subunit. O SRP binds signal sequence and ribosome.
Protein synthesis stops. @ Ribosome-SRP complex binds SRP receptor on RER. SRP dissociates
and protein synthesis starts. Signal sequence guides growing polypeptide into RER lumen. €l Signal
peptidase cleaves signal sequence. @ Polypeptide continues to grow. @ After protein is formed
ribosome dissociates and is recycled. @ Protein modification continues.

Copyright @ by The Berkeley Review 40 The Berkeley Review


Speciatizing in MCAT Preparation
Biology Structure & Function in Cells & Viruses Eukaryotic Cells

Golgi Apparatus
\lost proteins synthesized in the RER are transported in small vesicles to the
Golgi apparatus, a complex of flattened membranous sacs called cisternae. The
cisternae of the Golgi complex (Figure 6-62) is divided into three distinct regions,
clsFace
each containing an environment that has a relatively neutral pH. The cis cisterna
tace the nucleus and endoplasmic reticulum; the trans cisterna face the plasma
U @

membrane; the medial cisterna are located between the cis and trans cisternae.

As a protein passes from the cis cisterna to the trans cisterna, different chemical
modifications occur along the way. The principal chemical modification involves
:he addition of carbohydrates (glycosylation) to the maturing protein in a
.equential fashion. Other modifications involve sulfation (addition of inorganic
.ulfate) and proteolysis (reducing the size of the protein).
@
Once the proteins reach the trans face of the Golgi complex they appear to be
:orted and concentrated into vesicles destined for different regions of the cell. Trans Face

Lysosomes nigure 6-62


Specific glycosylated proteins found in the lumen of the RER are transported to The Golgi complex. The cis
:he cis Golgi where individual (mannose) sugar residues are phosphorylated. In lace is olten called the lorming
:he lumen of the trans Golgi these phosphorylated glycoproteins bind to a face while the trans face is
often called the maturing face.
mannose-6-phosphate) membrane-bound receptor that directs them to sorting
,.-esicles. Due to a low pH within the sorting vesicles the phosphorylated
ilvcoproteins are released from their membrane-bound receptors. Enzymes
:alled phosphatases remove the phosphate groups from the sugar residues. The
:lr-coproteins are transported to lysosomes while the membrane-bound receptors
: j the sorting vesicles are recycled back to the trans Golgi. Lysosomes number in
.re hundreds in most cells.

-1,e glycoproteins that reach the spherically-shaped, single membrane-bound


. sosomes are actually inactive precursors of lysosome enzymes. These inactive

:recursors are called proenzymes and they undergo proteolytic cleavage in an


'.idic environment (pH = 5) to form the mature and active enzyme.
-,.'sosomes are unique in their composition in that they contain more that 50
:'"pes of hydrolytic enzymes. Since all of these enzymes function at acidic pH
,:lues, they are often referred to as acid hydrolases. The general reaction that
-.ese enzymes catalyze is A-B + H2O -+ A-H + B-OH. In other words, hydrolytic
::.z\:mes degrade polymers into individual monomeric subunits. Table 6-7 lists
.:,rre acid hydrolases common to lysosomes.

E:zyme Substrate Bond Hydrolyzed


FROTEASES
Peptidase Peptides Peptide
r-\-COSIDASES
j-herosandnidase Glycolipids Glycoside
,.P \SES
Flnsph.oli.pase Phospholipids Carboxylic ester
].;L'CLEASES
.t, c i tl de o xy r ib o nuc l,e as e DNA Phosphoric diester
::IOSPHATASES
\cid phosphatase Phosphomonoesters Phosphoric monoester

Table 6-7
: :-:rted lysosomal enzymes.

- l'right O by The Berkeley Review 4t The Berkeley Review


Specializing in MCAT Preparation
Eukaryotic Cells
Biology Structure & Function in Celts & Viruses

Lysosomal enzymes are inactive at neutral pH' Therefll":


tl" environment
withinthelysosomeismaintainedatacidpHvalues.Thisisaccomplished in the lysosomal
Cytoplasm
,nr""gn,n.'action of an ATPase-driven proton pump located pump that
of the protein
ADP membrane (Figure 6,63). ATP binds to u po.iiot-, to- enter
two protons
extends into the cytoplasm and as it is hydrolyzed
+Pi allows
across the membrane'
proton ir,,o ,f," lysosome. In Lrder to maintain electrical neutrality
pump
2H@ achloridetranspolterworksinconjunctionwiththeprotonpumptoallow
Lumen anions into the organelle'
of
Lysosome
Whathappenstothehydrolysisproductscatalyzedbythedifferentenzymatic
and are utilized in a variety of
reactions? They simply hif.rr" out of the organeile
would expect to see lysis of this
*",uUofi. pro""rr"r. Ii this were not the .ur*u, -" the solute
Figure 6.65 ;rg^";ii;: why? rne proa.rcts. of hydrolysisthe osmoticincrease
would
entry of water' The
concentration in the tyro'ro*" and wouli leadto
Lysosomal Proton PumP.
this is not observed' it means
would sweil and eventually lyse. Since
"rg;rr"
inlt tn" hydrolysis products are removed from the organelle'

an inborn error of
If the activity of a particular hydrolase enzyme is absent, then
discussed lysosomal storage
metabolism can result. One oi the more commonly
disorder is due to the absence
abnormalities is Tay-sachs disease. This recessive
accumulation of a
,i tn" enzyme hexosaminidase A, andresults in the excessive
of the cerebral cortex'
of ganglion cells
;;i;;r'glycolipid in theandlysosomes
motor impaiime|t result' Death usually occurs by
Severe mental retard.ation
the age of 5'

Peroxisomes cellular
peroxisomes are single membrane-bound organelles found within the
being catalase' This
cvtoolasm that contain a variety of enzymes, tf,e most notable (6-7).
;i;;#;r"i", r,yar"gen peroxide (il2o2) according to equation

vorqruov
Catalase
> 2H2o + 02
(6-7) zE2oz

a potentially damaging oxidant,


The peroxisome generates hydrogen peroxide, acid
fatty
iypr.auct o-f both amino aJid oxidation u.d (very) iong chain
", "
oxidation.Peroxisomes,whichnumberinthehundredsinmostcells,arealso
heat'
thought to be involved in the dissipation of

Mitochondria
Inanimals,mitochondria(Figure6-64)arefoundinallcellsexcept"'ytllo.-y-T:
ur" oti" of the iargest organelles in the cel1, roughly
1500
i;;;;i;'.ells). They occupying as much s
nm long by 500 n* *idu, and can numbei in thJthousands'
25oh ofa cell's cytoplasm. This organelle
is characterizedby a double membrane
space
,yri"r" two distinct compar"tments, the matrix and intermembrane
(Figure ""a
6-65).
Figure 6-64
The mitochondrion. Theoutermembraneiscomposedofroughly50%proteinand50%lipid.The
a trarismembrane protein which
major protein in the outer *"^*brun" is pJrin,
forms channels that allows small molecules
with molecular weights of less than
10,000 to freelY Pass'

nd2S"hlipidandis
TheinnermembraneiscomposedofroughlyT5To-p.rotein
essentially i*p"r*"uUt"" t"lif-*tf"tt't"''"lvtu'ly.
of ine p.i:j:t::::::i:t"*,:t::
il""H;##ur"""lt" involved in an electrontransport process that is coupled
42 The BerkeleY Review
Copyright @ bY The BerkeleY Review Specializing in MCAT PreParation
Biology Structure & Function in Cells Er Viruses Eukaryotic Cells

to the generation of ATP from ADP and Pi. Other proteins act as carriers or
channels that allow for the transport of certain molecules across this highly
impermeable membrane. The number of reactions that occur along the inner
membrane is greatly increased by the presence of numerous convoluted foldings
called cristae. These foldings extend into the matrix of the mitochondrion.

Within the matrix some of the cell's most important biochemical reactions occur.
The majority of the cell's ATP is produced in the matrix. The Krebs (tricarboxylic
acid) cycle, B-oxidation of fatty acids, ketone body metabolism, gluconeogenesis,
and some of the reactions of the urea cvcle also occur in the matrix.

t Outer

I Membrane

aao
s re
a'c
lJ
o
t Inner

I Membrane

Malrix

Figure 6-65
.\ section of the membrane system and compartments of the mitochondrion

Er.en though most of a mitochondrion's proteins are encoded by nuciear genes,


',r'edo find that mitochondria have their own circular, double-stranded DNA
referred to as mtDNA) and their own ribosomes (sedimenting at 55S) and
:ransfer RNA molecules that allow for the synthesis of certain mitochondrial
':roteins. Within the small mtDNA genome are 37 genes, and almost all of the
-6,569 base pairs are transcribed and translated, making the presence of introns a
:emote possibility. Later, when we consider the endosymbiont hypothesis for
-.rganelle evolution, we will see that the mitochondrial genome and ribosomes
:esembles that of a prokaryote rather than that of a eukaryote.

llicrotubules, Microfi laments, Ee Intermediate Filaments


:rlending throughout the cytoplasm of a eukaryotic cell is the cytoskeleton, an
-:',tricate protein network of microtubules, microfilaments, and intermediate
--,aments that help define the shape of the cell and allow for ceil movement and
-:il division.
llicrotubules
I.licrotubules are composed of 13 protofilaments arranged side-by side to create a
:llow tube-like structure that is about 25 nm in diameter. Each protofilament is
- -,nposed of alternating cx-tubulin and B-tubulin proteins, linked together as
r.-lers and arranged in the same direction. This gives polarity to the molecule.

- :'e growth of microtubules occurs from regions cailed microtubule organizing


:enters (MTOCs). Three common centers are the centrosome (cell center),

, rr li-sht O by The Berkeley Review 43 The Berkeley Keview


Specializing in MCAT Preparation
Biology Structure & Function in Cells & Viruses Eukaryotic Cells

kinetochores (spindle attachment sites on chromosomes), and centrioles. As the


microtubules being to polymerize, tubulin dimers add to the fast growing plus
(+) end and extend toward the periphery of the cell. The opposite end is
the minus (-) end. "ultea

Microfilaments
Microfilaments are about 7 nm in diameter and consist of the protein G-actin
(where "G" refers to globular). As the individual G-actin *onb*ers begin to
polymetize, they form a long, double-helical, thread-like structure called F-actin
(where "F" refers to filamentous). Each G-actin monomer in the polymer is
arranged in the same direction, giving the microfilament polarity. Polymerization
occurs faster at the plus (+) end of the filament while depolymerization occurs
faster at the minus (-) end.

Intermediate Filaments
Intermediate filaments are about 8 to 12 nm in diameter and differ in their
composition, depending upon what tissue they are located in. For example, the
intermediate filaments in epithelial cells are composed of keratins while in
muscle cells they are composed of desmin.

Copyright @ by The Berkeley Review 44 The Berkeley Review


Specializing in MCAT Preparation
Biology Structure & Function in Cells & Viruses Prokaryrotic Cells

Bdctffiffiiilrchi Gtffi$ticc
Size and Shape
Bacteria come in all shapes and size. The two most frequently encountered
bacteria are the cocci (singular, coccus) and the rods (sometimes called a
bacillus). Cocci are essentially spherical in shape while rods generally resemble
that of a tube. Bacteria that have a rigid twist to their rod-like structure are called
spirilla. If their twisted structure is more flexible, they are called spirochetes.
Bacteria not only differ in shape, but they also vary in size. They can be as small
as the largest virus or as large as an erythrocyte.

Cell Structure
Prokaryotic cells have a variety of structures important for survival. They all
have a plasma membrane which acts as a selectively permeable barrier to
metabolites entering and leaving the cell. Ribosomes are found within the
cytoplasm and are important for protein synthesis. Inclusion bodies aid in the
storage of a wide variety of substances. The genetic material is found in an
amorphous region of the cell called a nucleoid. Almost all bacteria contain a cell
rvall which helps to prevent cell lysis. Some bacteria have layers external to their
cell wall called capsules; others have slime layers. Bacterial movement is brought
about by the use of flagella (singular, flagellum).

Plasma Membrane
Prokaryotic cells are fairly consistent in their cellular structure. They all have a
plasma membrane which bounds the cytoplasm. Within this plasma membrane
are both proteins and lipids. Even though bacterial membranes do not contain
cholesterol, many do contain a sterol-like molecule that probably functions in
much the same manner as cholesterol. Invaginations of bacterial cell membranes
are called mesosomes. The function of these structures is not known.

Cytoplasm & Ribosomes


The cytoplasm of bacterial cells lack membrane-bound organelles (e.g., nucleus,
nitochondria, Golgi, etc.) that we often associate with eukaryotic cells. Within
*re cytoplasm are structures called inclusion bodies. These structures can
rontain organic molecules like glycogen or inorganic molecules like phosphate
*ranules.

lhe ribosomes found in the cytoplasm of prokaryotic cells are much small than
ie ribosomes of eukaryotic cells. Complete prokaryotic ribosomes are referred to
rs being 70S ribosomes. These ribosomes are composed of a large 50S subunit
'nd a small 30S subunit.

l{ucleoid of DNA
?rokaryotic cells do not have a nucleus. Instead, their circular, double-stranded
fNA chromosome is confined to a region in the cell called a nucleoid. These
::gions are often associated with either the plasma membrane or mesosomes.
:ome bacteria also have other chromosomes which are circular and double-
.=anded called plasmids. These additional DNA molecules allow bacteria to be
::sistant to some drugs, and give the bacteria a selective advantage.

J,rpyright @ by The Berkeley Review 45 The Berkeley Review


Specializing in MCAT Preparation
Biology Structure & Function in Cells & Viruses Dukaryotic Cells

The Cell Wall


In 1884 Christian Gram developed a stain that allowed bacteria to be classified
into two groups. Bacteria that stained purple were called Gram positive (+)
bacteria while those that stained red or pink were called Gram negative (-)
outside cen ile bacteria. The difference between the two types of bacteria has to do with their
^^^^^{ ;-\-, ? cell walls. structures outside of the plasmf membrane are usually referred to
ifittrlfll?l'ffixlffi'tff t 9-
Xo
as the cell wall or envelope.

uuuuuuiiffijuu I (D Gram positive bacteria have a rather thick homogeneous peptidoglycan or


murein layer (20 nm to 80 nm) just outside their plasma membrane. They also
M M-M M-.-M o contain a large amount of teichoic acids that extend to the surface of the cell
it d d d d 1 from either the plasma membrane or the peptidoglycan layer.
eGi i-ri i- ri 0c

,( ,c ,( ,( ,( I o Gram negative bacteria (Figure 6-66) have a much thinner peptidoglycan layer
(about 1 nm to 3 nm) surround their plasma membrane. Flowever, surrounding
?
o tu the peptidoglycan layer is an outer membrane that contains
lipopolysaccharides and porins. Not only does the polysaccharide help to
stabilize the membrane, but it also acts as an endotoxin and provides a defense
mechanism for the cell. The porins allow for passage of materials smaller than
700 daltons. Larger materials must be transported across the outer membrane.

Figure 6-66 of two acetylated amino sugars, N-


The peptidoglycan itself layer is composed
Gram negative bacterial membrane. acetylglucosamine (G) and N-acetylmuramic acid (M), linked together in a
B(1,a) [nkage, and a small number of amino acids, including D-glutamic acid
and D-alanine. Attached to each N-acetylmuramic acid residue is a
tetrapeptide side chain.

Capsules & Slime Layers


In general, both capsules and slime layers are composed of polysaccharides
that extend from the surface of the cell. This polysaccharide layer is sometimes
called a glycocalyx. The distinguishing features between the a capsule and a
slime layer is that slime layers are easier to wash off. Capsules are much more
resistant. In fact, they help protect the bacterium from foreign invaders. We
will come back to this point when we consider the Griffith experiment
involving the bacterium Streptococcus pneumoniae.

Flagella
Many bacteria use flagella for movement. These protein structures extend from
the plasma membrane and cell wall and provide a propeller-like movement in
a counterclockwise direction that propels the bacterium through its
environment. The major protein component of the flagellar structure is
flagellin.

Why do bacteria need to move in their environment? Nutrients. Bacteria are


attracted by chemical nutrients like sugars and amino acids. Attraction towards
or repulsion away from certain chemicals is referred to as chernotaxis.

Life Cycle
The process of cell division in bacteria is rather simple. The DNA, which is
attached at some point to the inside of the plasma membrane, undergoes
replication at a site called the replication origin. Once the DNA has been
replicated a new plasma membrane and cell wall begin to enclose the newly
synthesized chromosome. The two bacterial cells will eventually separate by a
process called binary fission.

Copyright @ by The Berkeley Review 46 The Berkeley Review


Specializing in MCAT Preparation
Biology Structure E( Function in CeIIs & Viruses Prokaryotic Cells

DNA Transfer
Genetic material can be passed from one bacterial cell to the next by the simple
process of binary fission. However, genetic information can also be transferred to
a bacterial cell by either bacterial conjugation, transformation, or transduction.

In bacterial conjugation the transfer of genetic information occurs by cell-cell


.ontact. Donor strains of bacteria are referred to as being F+ (male) while the
:ecipient bacteria are F- (female). The "F" refers to the fertility plasmid. During
:onjugation the F factor replicates by a process called the rolling circle
mechanism of replication and the newly synthesized DNA is passed to the
recipient bacterium.

Transformation involves the uptake of genetic material from the surrounding


:-Ledium. Usually this new genetic material is incorporated into the host
;hromosome.

Transduction is simply the transfer of bacterial genes by viruses. How do viruses


:et bacterial genes in the first place? In general, when a virus infects a host cell it
ncorporates its own DNA into the DNA genome of the host. When it is time for
ie virus to leave, it removes its own DNA from the host genome. It is during
'jris process that errors can occur. Sometimes the virus removes a few host genes
as well. Once the virus has replicated and the progeny phage have been
:ssembled and released, they are able to infect other bacteria. Those progeny
:hage will bring to those new bacteria a few genes from previously infected host
:eils.

,\-ewill be returning to these different types of DNA transfer when we begin our
liscussion on Genetic lnformation.

- rpvright @ by The Berkeley Review 47 The Berkeley Keview


Specializing in MCAT Preparation
Biology Structure & Function in Cells & Viruses Viruses

Architecture
\-iruses show a wide range of biological diversity and are quite successful at
:arasitizing other organisms. Quite simply, at the genetic level, viruses are
.-bligate intracellular parasites that cause infected host cells to produce viral gene
:roducts rather than host gene products. Lipid membrane

/ tu,,,*
lrre architecture of a virus (Latin for poison) is usually based upon one of two / protein
.ructural motifs (Figure 6-67); those which are isometric (usually in the form of
:,.. icosahedron) or those which are helical. In its simplest form a virus is
--lmposed of a nucleic acid that is surrounded by a protein coat. The protein coat
-. iormed from capsomers, which are building blocks composed of a specific
:,'-rmber of individual proteins. If there is no nucleic acid within the protein coat
:r shell, then the empty shell is referred to as a capsid. Flowever, if there is
:-ucleic acid within the protein shell, the complex is called a nucleocapsid.
L'N
-: -]re nucleocapsid of the virus is not surrounded by a lipid membrane, the virus
;o referred to as a non-enveloped (naked) virus. However, if the nucleocapsid is Nucleocapsid Capsomer
.;rror-rnded by a lipid membrane, the virus is referred to as an enveloped virus.
li.e membrane of an enveloped virus is derived from the host cell that the virus
::ected and is attached to the nucleocapsid by matrix proteins. Transmembrane (b)
::oteins which have been glycosylated (i.e., glycoproteins) act as antigens and
.'-on. the virus to communicate with its environment. These complexes are
>:,=retirnes called spikes.

:"::re viruses, like the bacteriophage T4 that infects the bacterium Escherichia coli
Z :':li), are quite complex. This phage not only has an elongate iscoshedral head,
::: it also has a helical tail section with tail fibers that can attach to the
';".polysaccharide portion of the host's membrane.

Genome
l:e genetic information within the genome of a virus may be encoded in either Figure 6,67
:-e nanguage of DNA or RNA. The nucleic acid can either be linear or circular, Viruses with (a) enveloped
;::rl.e-stranded or double stranded, and it can even be segmented. However, no isometric and (b) non-
:'Jer what type of nucleic acid is found in a viral genome, the translational enveloped helical geometry.
:f :..ess involved in the expression of that genome uses mRNA as a template.
-}.=:efore, by convention, we define that nRNA as being a positive (+) strand
- ::leic acid. DNA and RNA polymers that have a base sequence identical to this
;,:s:ti\re mRNA strand are also designated as being positive (+) strands'
i,:::ember, in DNA we find that the base thymine is used instead of the base
,*.:-1. DNA and RNA polymers with a base sequence that is complementary to the
:,-srdve mRNA strand are referred to as being negative (J strands. In order to
, :.fr.esize a positive mRNA strand, the nucleic acid template must either be a
ri:":aLive DNA or RNA strand.

l:= relationship between the positive mRNA strand and the different nucleic
.r:- j:'
allows us to organize viruses into six classes based on a scheme proposed
:,, lar-id Baltimore. The Baltimore classification for the six viral classes and some
',*::esentative viral families are shown in Table 6-8.

, :r "i-isht @ by The Berkeley Review 49 The Berkeley Review


Specializing in MCAT Preparation
Biotogy Structure & Function in Cells & Viruses Viruses

Virus Host Genome Class Size (kb) Mo


T4 phage Bacteria Linear (+/-) ds DNA t70 No Elongated icosahedral head
Helical tail; Tail fibers
), phage Bacteria Linear (+/-) ds DNA I 46 No Icosahedral head
Helical tail
Herpes simplex Animal Linear (+/-) ds DNA I 150 Yes Icosahedral
M13 phage Bacreria Circular (+) ss DNA IIa 6.4 No Helical rod
QX174 phage Bacreria Circular (+) ss DNA IIa 5.4 No Icosahedral

Rotavirus Mammals Segmented (+/-) ds RNA m 1.2 - 1.4 No Icosahedral


Poliovirus Mammals Linear (+) ss RNA IV 7.2 - 8.5 No Icosahedral
Rhinovirus Mammals Linear (+) ss RNA IV 7.2 - 8.5 No Icosahedral
Rabies Vertebrates Linear (-) ss RNA V t2 Yes Helical rod
lnfluenza Vertebrates Segmented C) ss RNA 13.6 Yes Helical rod
HIV Vertebrates Dimer (+) ss RNA VI 9.2 Yes Icosahedral
$ Approximate sizes in kilobases where 1 kb = 1000 nucleotides
Table 6-8
The six viral classes as proposed by David Baltimore.

The information presented in Table 6-g might seem a bit overwhelming. It is not
meant to be. And above all, this information is not to be memorized. Rither, it is
presented to give you a feel for the rich biological diversity in the viral world. As
we proceed in our discussions we will be considering a number of these viruses.
we will examine the coliphages T4, 1., and Qx{74, as well as the human
imunodeficiency virus (HIV), a retrovirus implicated in the pathogenesis of
human acquired immunodeficiency syndrome (AIDS).

Copyright O by The Berkeley Review 50 The Berkeley Review


Specializing in MCAT preparation
Biology Structure & Function in Cells & Viruses Viruses

Ad#U#pffi#itri1,,"'WffH, ffiffii|ffi11# s#i$ffi


Adsorption
\'-iral infections are generally restricted to specific organisms or types of cells, Bacteriophage
usually referred to as the host range of the virus. Viral specificity for a host cell
depends on the interaction of a particular virus with the host cell's surface
proteins, glycoproteins, and glycolipids.

For example, the bacteriophage T4 has a complex of tail fibers that recognizes a
specific lipopolysaccharide structure as well as a particular protein porin, both of
rrhich are located on the cell surface of the bacterium E. coli. The bacteriophage l,
nas a single tail fiber that recognizes an integral membrane protein responsible
tor the transport of the sugar maltose into E. coli. The HIV retrovirus has a
glvcoprotein (gp120) associated with its envelope that appears to bind to a
specific protein receptor (CD4) found on the surface of helper T cells and
monocytes, two cellular constituents of blood, and glial cells, a cellular
c,f,nstituent of the central nervous system.

Penetration
Once a virus adsorbs to the surface of a suitable host, its next step is to introduce
1ts genome into that host, a process called penetration. The type of virus and the
attachment to the host membrane determines how this will occur.

Bacteriophages
Viral DNA
-{fter a bacteriophage like T4 has adsorbed to the membrane of the host cell, the
orotein sheath that defines the tail begins to reorganize and penetrates the cell
ltali of the bacterium until it contacts the plasma membrane. An opening is
iormed in the plasma membrae and phage DNA is passed from the viral capsid,
lhrough the tail section, and into the bacterial cytoplasm (Figure 6-68). S capsid
(c)
The llershey-Chase Experiment
s"--,{n
"K
-\s shown in Figure 6-68 only the viral DNA enters into the host. The protein
components of the capsid, tail, and tail fibers remains outisde the bacterial cell.
This finding was demonstrated in a classic experiment performed by Alfred
Hershey an-d Martha Chase in 1952. They infecte d E. coli bacteria with 32P and
35S labeled T2 bacteriophages. After allowing for phage adsorption the infected
'tacterial cells were separated from any unattached phages by centrifugation and
then placed in a Waring blendand subjected to violent shearing forces. The fluid
',ras again centrifuged. Hershey^ and Chase found that the bacterial cell pellets ,rn"'&
containeda high percentage of 32P while^the supernatant, containing the sheared
phages, contained a high percentage of 35S. Since protein contains sulfur (in the
amino acids cysteine and methionine) and DNA contains phosphorous (in the Flgure 6-68
:tnsphodiester bonds of the backbone), Hershey and Chase concluded that it was Sequence of events showing (a)
adsorption of phage to bacterial
:he viral genome, and not the viral protein, that entered the bacterial cells and
cell, (b) penetration, and (c)
caused infection.
release of viral DNA into host
ilaked (Non-Envetoped) Viruses cytoplasm.
\aked viruses gain access to the host's cytoplasm only by recptor-mediated
.:fiocytosis. The receptors on the cell surface of the host are usually located near
specialized depressions called clathrin- co ated pits. Clathrin, a non-glycosylated
piotein, is believed to act as a scaffold that promotes vesicle formation. After the
-;irus has been taken into the host by endocytosis, the pH of the vesicle's interior
s lowered as hydrogen ions are transported into the vesicle by proton pumps in
-Jre vesicle membrane. The clathrin coat depolymerizes and the smooth-surfaced

t/y Copyright @ by The Berkeley Review 5l The Berkeley Review


n Specializing in MCAT Preparation
Biology Structure & Function in Cells & Viruses Viruses

vesicle is now referred to as an endosome. As the pH of the vesicle


falls, the
nucleocapsid disrupts the membrane of the end.osorile and is released
into the
cytoplasm. The naked virus has been brought into the host cell (Figure
6-69).

virus binding to membrane receptor


U.Naked
Plasma membrane
of host Viral genome

(b)
cytoplasm
Clathrin/
coated pit
'{w{ Y
Receptor-mediated
..(

endocytosis
Endosome

Figure 6-69
Penetration of a naked virus into a host cell. (a) Binding of the virus to
a membrane receptor is
followed by (b) endocytosis and (c) enclosure of the viral g-"no-e in an endosome.

Enveloped Viruses
viruses which are enveloped can either enter their host through receptor-
mediated endocytosis or by direct fusion with the plasma membrane. If an
enveloped virus enters-thrgrigh receptor-mediated endtcytosis, a process
similar
to the one just outlined is followed. The enveloped virus attaches to the plasma
membrane of the host and is brought into the ."tt uy end.ocytosis (rigure o-zo).
After a low enough pH is reached in the endocytotic vesicle, the vira'i envelope
fuses with the vesicle's membrane and the nucleocapsid is released into the
cytoplasm.

Enveloped virus binding to membrane receptor

Plasma membrane

+ of host Viral genome

(b)
Cytoplasm

Clathrin/ \)
coated pit -4t { .-(
\
Receptor-mediated
endocytosis
Endosome

Figure 6-7O
Penetration of an enveloped virus into a host cell by receptor-mediated endocytosis. (a)
Binding of
the viral envelope to a membrane receptor is followed by (b) endocytosis and (c) enclosure
of-the
enveloped virus in an endosome.

Copyright @ by The Berkeley Review 52 The Berkeley Review


Specializing in MCAT preparation
Biology Structure & Function in CeIIs & Viruses Viruses

-{n alternate approach is for the envelope of the virus to directly fuse with the
plasma membrane upon initial contact with a specific receptor (Figure 6-71). This
is how the HIV retrovirus is thought to enter its host cell. The fusion of the viral
membrane with the plasma membrane, whether it is through receptor-mediated
endocytosis or direct membrane fusion, is mediated by fusion proteins in the
liral membrane.

Enveloped virus binding to membrane receptor

Viral genome
Plas
q)
Cytoplasm
o.,4t
Figure 6-7 I
Penetration of an enveloped virus into a host cell by membrane fusion. (a) Binding of the viral
envelope to a membrane receptor is followed by (b) fusion of the viral and plasma membranes and
(c) release of the nucleocapsid into the cytoplasm.

Once the virus has entered the host cell, its genome must have access to the
cellular processes that allow for nucleic acid replication and protein synthesis.
This is brought about by a process called uncoating. Here the protein capsid is
removed from the viral genome, thereby allowing the viral nucleic acid to enter
the host cell's cytoplasm.

Expression
We have mentioned that viruses are obligate intracellular parasites that cause
infected host cells to produce viral gene products rather than host gene products.
How does this occur? In order to answer that question, we must first determine
if the infected cell is prokaryotic or eukaryotic; if the viral nucleic acid is DNA or
RNA; and if that nucleic acid is double stranded or single stranded.

Viral Replication & Expression in Prokaryotes


There are many examples of viral replication and expression in prokarytotic cells.
One of the most widely studied groups of viruses are those that comprise the T-
series of. bacteriophages that infect E. coli. Prcbably that most famous coliphage
fuom this series is T4.

The genome of the T4 phage is represented by linear, double-stranded DNA with


a rather unique feature. The nitrogenous base cytosine has been modified with a
hydroxymethyt group at the C-5 position of the ring. This modified base, 5-
hydroxymethylcytosine (HMC), is present only in the viral DNA. It is not found
in the bacterial DNA. The importance of this modified base to the viral genome
is that viral enzymes will easily be able to determine which is viral DNA and
which is bacterial DNA.

TheDNA of T4 could contain as many as 150 genes, of which about half code for
regulatory proteins and degradative enzymes. The bacterial enzyme RNA

Copyright @ by The Berkeley Review DC The BerkeleY Review


Specializing in MCAT Preparation
Biology Structure & Function in Cells & Viruses Viruses

polymerase, which is continually being modified by viral proteins, is used to


transcribe different genes in the viral genome. viral gene products not only
disrupt the replication, transcription, and translation of the host genome, but
they also aid in the production and assembly of progeny virions.

For example, within 2 minutes after infection the immediate eailq viral genes are
transcribed by the bacterial errzyrr.e RNA polymerase. Translation of thil plus (+)
mRNA transcript produces viral proteins that have a variety of actions. The
pol}rnerase enzyme is subsequently modified in order to transcribe the delayed
early, quasi-late, and late viral genes. Translation of these plus (+) mnlvR
transcriptsproduces viral proteins that eventually act to digest bacterial DNA

we will be returning to viruses in our discussion on Genetic Information.

Copyright @ by The Berkeley Review 54 The Berkeley Keview


Specializing in MCAT Preparation
Structure and Function
in Cells and Viruses
15 Passages

100 Questions

Passage Titles Questions


I. Sepsis 1-.5
U. EndoplasmicReticulumMicrosomes 6-tr
III. Essential Amino Acids 12-17
IV. Endotoxin 18 - 23
V. Spinal Meningitis 24-29
VL Nuclear Envelopes and Pores 30-36
VII. Lipids and Membranes 37 -43
V[I. Nonstandard Amino Acids 44-50
IX. Amino Acid Characteristics 51-58
X. Fluorescent Recovery after Photobleaching (FRAP) 59-65
XI. Gram-Negative and Gram-Positive Bacteria 66-12
XU. Raffinose 73 -79
XI[. Mitosis and Meiosis 80-87
XIV. Lipids: Four Groups 88-94
XV. BacteriophageLambda 95 - 100

,]fu
REBI{ELAY
L)R.E-u.l .-b.w*
Speci aLtztng in MCAT Preparation
Suggestions
The passages that follow are designed to get you to think in a conceptual manner about the processes
of molecular biology at the organismal level. If you already have a solid foundation in moleculai biology,
many of the questions you read here will seem to be very straight forward and easy to answer. But if you
are new to the subject or if you have not had a pleasant experience with molecular biology in the past,
some of them might appear to come from the void that spreads out beyond the Oort field ii the edges of
our solar system.

Pick a few passage topics at random. For these initial few passages, do not worry about the time. just
focus o1 what is expected of you. First, read the passage. Second, look at any diagrams, charts, or graphs
in it. Third, read each question and the accompanying answers carefully. Fourth, answer the questibns
the best you can. Check the solutions and see how you did. \A/hether you got the answers right or wrong,
it is important to read the explanations and see if you understand (and agree with) what is being
explained. Keep a record of your results.

After you feel comfortable with the format of those initial few passages, pick another block of
Passages and try to do them in one sitting. Be aware that time is going to become important. On average,
you have about 1 minute and 15 seconds to complete a question. Be creative in how you approach tils
next group. If you feel comfortable with the outline presented above, fine. If not, then iry different
approaches to a passage. For example, you might feel well versed enough to read the questions first and
then try to answer some of them, without ever having read the passage. Maybe you can answer some of
the questions by just looking at the diagrams, charts, or graphs that are presented in a particular passage.
Remember, there are many effective learning styles. You need to begin to develop alormat thit works
best for you. Keep a record of your results.

The last block of passages might contain at least a few topics that are unfamiliar even to those who
know a good deal about molecular biology. Find a place where the level of distraction is at a minimum.
Get out your watch and time yourself on these passages, either individually or as a group. It is important
to have a feel for time, and an awareness of how much is passing as you try to answer each question.
Never let a question get you flustered. If you cannot figure out what the answer is from information
given to you in the passage, or from your own knowledge base, dump it and move on to the next
question. As you do this, make a note of that pesky question and come back to it when you have more
time. When you are finished, check your answers and make sure you understand the solutions. Be
inquisitive. If you do not know the answer to something, look it up. The solution tends to stay with you
longer that way. (For example, what ls the Oort field, anyway?)

The estimated score conversions for 100 questions are shown below. At best, these are rough
approximations and should be used only to give one a feel for which ballpark they are sitting in.

Section VI
Dstimated Score Conversions
Scaled Score Raw Score
>13 80 - 100
ll-12 10-79
9- l0 60-69
7 -8 50-59
5-6 40-49
<4 0-39
Biology Sepsis Passage I

Passage I (Questions 1-5) 1. The Gram stain used to differentiate between two
main types of bacteria takes advantage of the fact
Sepsis is an extremely complex disease that disrupts that:
all physiological regulatory mechanisms involved in
maintaining homeostasis. This disorder, which in its A. neither type has a membrane-bound nucleus.
extreme form, may be characterized by organ B. Gram-positive bacteria do not have a plasma
malfunction, is brought about by the interaction of membrane.
microbial products with host cells. C. bacteria differ from one another in the
structures of their cell walls.
When a host organism first succumbs to the effects of D. different bacteria possess different
an acute bacterial infection, the inflammation response characteristic shapes, such as rod and spiral.
initiated by the body of the host is meant to preserve its
health. Normally, the inflammatory response is turned off
by a variety of mechanisms. Septic shock develops when
this homeostasis is disrupted.

Septic shock has generally been considered to be


,-ausedby Gram-negative bacteria. The major sepsis-
inducing factor of a Gram-negative bacterium is its
surface lipopolysaccharide (LPS). This bacterial product
\timulates the production of a number of cytokines and
-rther inflammatory mediators that have both pro-
rnflammatory and anti-inflammatory functions. Septic 2. According to the passage, cytokine generation and
.hock is associated with the excess production of pro- the inflammatory response are normally repressed
.nflammatory mediators. Among these cytokines are by a number of mechanisms. Which of the following
:umor necrosis factor (TNF) and interleukin-1. The target events would be LEAST likely to repress the
ri the inflammatory activation pathway is the endothelial inflammatory response?
, e11.
A. Down-regulation of cytokine receptors on host
Once LPSs are found in the host in quantities that cells.
:rnnot be managed by its defense mechanisms, they B. Activation of the complement system.
::cruit monocytes, macrophages, polymorphonuclear C. Endogenously generatedcytokineantagonists.
-ells, and endothelial cells to initiate an inflammatory D. Suppression of the immune system by
-:scade. This response may occur locally in tissue or hormones.
.rroughout the bloodstream.

In blood, LPS binds to a circulating molecule known


,; LPS-binding protein (LBP). Once formed, these
:rmplexes then bind CD14, a surface molecule on
-.onocytes. Binding of LPS/LBP to CD14 leads to
-.onocyte activation. Endothelial cells are also stimulated
:'. LPSs, acting with LBP and CD14 molecules.
:-.,rrvever, in this case, endothelial cells lack a membrane-
rund form of the CD14 molecule.

3. It is difficult to study sepsis, because most mediators


are capable of promoting the release of other
mediators and even of themselves. This self-
stimulation is BEST described as:

A. an endocrine function.
B. a exocrine function.
C. an autocrine function.
D. a paracrine function.

- :,ryright O by The Berkeley Review


The Berkeley Review
Specializing in MCAT PreParation
Biology Sepsis Passage I

4. Septic shock is not a new disease, yet its incidence is


on the rise. Which of the following would be a likely
reason for this increase?

A. Less invasive procedures are being used in


modern medicine.
B. A decline in immunosuppresive therapy has
been noted.
C. The use of antibiotics has created highly
resistant strains of bacteria.
D. There has been a decline in the population of
persons with treatable serious diseases.

5. In endothelial cells, LPS is also capable of


stimulation through LBP and CD14. Analysis of
endothelial cells shows a lack of a membrane-bound
form of CD14. This protein is MOST likely to be
found:

A. bound to the extracellular face of the


membrane on the endothelial cell.
B. bound to the cytoplasmic face of the
membrane on the endothelial cell.
c. within the cytosol of the endothelial cell.
D. within the blood and extracellular fluids of the
tissues.

Copyright @ by The BerkeleY Review 5a The Berkeley Review


Specializing in MCAT PreParation
Biology Endoplasmic Keticulum Microsomes Passage tr

Passage II (Questions 6-11) 6. To separate smooth microsomes from rough


microsomes, centrifugation within a sucrose
When tissues or cells are disrupted by gradient is used with the following result:
:omogenization, the endoplasmic reticulum (ER) is
-ra_smented into many closed vesicles called microsomes.
\licrosomes derived from rough endoplasmic reticulum
re called rough microsomes. The ribosomes are always
:,rund on the outside surface of these microsomes. 1 mlcrosomes
Smooth
J
Signal peptides were discovered in secreted proteins
:irt are translocated across the ER membrane prior to .Rough
mlcrosomes
.:eir transport to the Golgi apparatus and eventual
':,-retion from the cell. The signal hypothesis postulates
:,:t a leader peptide serves as a signal peptide, directing
.-: eventual secreted protein to the ER membrane. Once
-: peptide is inside the ER, the signal peptide is cleaved
' br a special protease. This result can BEST be explained by the fact that:

The signal peptide is thought to be guided ro rhe ER A. rough microsomes are made of different
*::rbrane by two components. One is the signal- proteins than smooth microsomes.
'-:.-snition particle (SRP), which binds ro the signal B. ribosomes contain large amounts of RNA.
,=:.rde and moves between the cytosol and the ER C. smooth microsomes have a much higher
-.:''rrane. The other component is the SRP receptor, an density than rough microsomes,
- =rral membrane protein found within the ER D. rough microsomes have much smaller
;:frane. The signal-recognition particle binds to the diameters when compared to smooth
microsomes.
: "1 peptide as soon as the peptide emerges from the
' i,rme. A subsequent pause in protein synthesis is
-,=:,.ed untii the ribosome carrying SRP binds to the
:,: ::ceptor. The translational arrest is then lifted with
* -.:rneous translocation of the protein into the
:- , :-rsmic reticulum.
The exterior of a rough microsome is equivalent to
the:

A. cytosol.
B. lumen of the nucleus.
C. lumen of the rough ER.
D. lumen of the Golgi.

8. The mRNA of a hormone normally produced by an


endocrine cell is translated by a free ribosome in the
presence and absence of microsomes. In this cell
free system, the hormone is found to be:

A. the same size in the presence and absence of


microsomes.
B. longer in the absence of microsomes.
C. shorter in the absence of microsomes.
D. longer in the presence of microsomes.

r I bv The Berkeley Review 59 The Berkeley Review


m Specializing in MCAT Preparation
Biology Endoplasmic Reticulum Microsomes Passage tr

9. It was discovered that washing microsomes with salt


removes the SRP as part of the salt extract. This
would be MOST likely to result in:

A. a quicker translational arrest.


B. elimination of the ability to import protein into
the ER.
C. the loss of the signal peptide.
D. an increase in the frequency of secreted
proteins.

10. One reason for the translational arrest observed as


the peptide emerges from the ribosome might be:

A. to prevent the release of the newly synthesized


protein into the cytosol.
B. an indication that the ribosome is non-
functional.
C. to ensure that all translation must occurs on the
membrane of the ER.
D. to allow the ribosome time to proofread the
signal peptide before it binds to the SRP.

11. During the course of a typical secretory pathway, a


molecule leaving the lumen of the ER will next
come in contact with the:

A. lateral region of the Golgi complex'


B. cis region of the Golgi comPlex.
C. medial region of the Golgi complex.
D. trans region of the Golgi complex.

Copyright @ by The BerkeleY Review 60 The BerkeleY Review


Specializing in MCAT PreParation
Biology Essential Amino Acids Passage Itr

Passage III (Questions 12-17) 12. According to Table 1, which of the following
statements must be TRUE?
Nine amino acids are required in the diet of adult
human beings. They are histidine (His), isoleucine (Ile), I. All animal proteins meet the ideal protein
leucine (Leu), lysine (Lys), methionine (Met), threonine requirements.
'Thr), valine (Val), tryptophan (Trp), and proline (Pro). II. Soybeans meet ideal protein requirements.
Each of these amino acids has a distinctive side chain that III. Lentils are low in sulfur-containing amino
,-annot be formed by enzymes in the human body. acids.

In the 1950s and 1960s, researchers studied A. I only


.equirements of each amino acid and produced an ideal B. II only
:rofile of amino acids for human nutrition. Generally, the C. II and III only
:-1esh of other animals, birds, and fish contains all nine D. I, II, and III
'mino acids in patterns that fulfill the "ideal" protein
::quirement. Except for soybeans, the proteins of
. egetable sources are usually lacking one or two of the

:ssential amino acids. Grains and nuts are usually low in


'. sine. Beans and peas tend to be low in sulfur-containing
.:uno acids. The combination of plant proteins of
---ierent classes (so that amino acid weaknesses in one
: rd are complemented by the other food) is called
'nrein complementation. An amino acid is termed For which amino acid are beans characteristically
niting" if it does not meet or exceed the ideal protein lower than the ideal protein standard?
,..:dard. The standard is based on the proportions of
:rrino acids the body requires for growth and A. Methionine
:rntenanCe. B. Lysine
C. Leucine
D. Phenylalanine
Protein
Food Lys Met / Cys Thr Trp Leu
Source
.::al 5.5 3.5 4.0 1.0 7.0
6.4 5.5 5.0 t.6 8.8
',1r11 1.8 -1 --J 4.6 t.4 9.8
:..+ 8.7 3.8 4.4 1.2 8.2
l:.icken 8.8 4.0 4.3 1.2 7.2
: -
., beans 6.9 3.5 4.3 1.5 8.4 14. Which of the following structures indicates lysine?
6.4 2.6 3.4 1.0 8.7

-.rti ls 6.1 1.5 3.6 t.0 7.0 HO HO


I :lmeal 2.9 3.5 4.0 0.6 3.0
@rilo orilo
HjN- C- C- O H3N- C- C* O
neal 3.7 3.6 J.J 1.3 7.5
CHt CH"
n -1.4 0.9 1.8 0.0 -J.t,
t'
l. Arnino acids as percent protein in foods and compared
jeal reference protein. o+"
HO
otilo
H1N- C- C- O
I

CH"
t" @rilo
HjN- C- C- O
CH"
t' CH:
CH:
I I

CH, /:\o
orNH: H-N-7,N-H

lH'
-_:ht O by The Berkeley Review The Berkeley Review
on Specializing in MCAT Preparation
Biology Essential Amino Acids Passage Itr

15. Of the choices listed in Table 1, which has only two


limiting amino acids?
A. Collagen
B. Soybeans
C. Cornmeal
D. Lentils

16. An unknown food is examined, and each of its


amino acids is analyzed by gas chromatography with
flame-ionization detection. The following pattern is
observed:

Lys Met/ Cys Thr Tip Leu


5.^/, 3.t 3.7 1.0 3.2

What is the MOST probable food category of this


unknown sample?

A. Bean
B. Grain
c. Milk
D. Fish

17. Which of the following amino acids is nonessential?

A. Phenylalanine
B. Valine
C. Isoleucine
D. Histidine

Copyright @ by The Berkeley Review 62 The Berkeley Review


Specializing in MCAT Preparation
Biology Endotoxin Passage IV

Passage fV (Questions 18-23) 18. Gram-negative bacteria are distinguished from


Gram-positive bacteria by:
_ Many potent antibiotics exist that are effective against
Gram-negative bacteria, but a potential clinical problem A. an anaerobic culturing process.
::rnains with the acute inflammatory response to B. an immunoassay.
rndotoxin released from these bacteria. The endotoxin is a C. a differential staining technique.
.-popolysaccharide, often called LpS. The acute D. high performance liquid chromatography.
-flammatory response to endotoxin is called septic shock.

The death of Gram-negative bacteria leads to the


::lease of endotoxin, a lipid component of the outer cell 19. A person has an infection caused by a Gram-
;- ill of most Gram-negative bacteria. Humans
respond to negative bacterium. What effect would an antibiotic
'rdotoxin with a release of cytokines and other cellular effective against Gram-negative bacteria have on
:---'diators. These mediators include tumor necrosis factor short-term concentration of endotoxin in the blood?
:r TNF-o), interleukin-1 (L-1), IL-6, leukotrienes, and
:::omboxane A2. At high levels, these compounds may A. Endotoxin levels would decrease rapidly as the
.:--gter the responses of septic shock, including fever, antibiotic bound and neutralized endotoxin.
l.-,lck. hypotension, clotting, and organ failure. B. Endotoxin levels would not change with
antibiotic treatment.
In the endotoxin molecule, a terminal disaccharide C. Endotoxin levels would decrease briefly as
::,rspholipid (Lipid A) contains the key strucrural feature bacteria were killed by the antibiotic.
:":r is responsible for toxicity in Gram-negative bacteria. D. Endotoxin levels would increase brieflv as
bacteria were killed bv the antibiotic.

oP(oxoH),

oH .A' *i' 20. The following table shows data from an experiment
on mice, using the experimental agent E5531 given

\^r"+^*
(Ho)rP(o)oY NHR, OH
simultaneously with 3 prg of E. coliLpS.

E5531 Plasma TNF Plasma TNF Mortality


(pglmouse (ng/ml) inhibition (%) (Vo)
oRr
0 (control 582+ 20 0 100
E. coli Lipid A 1 547 t45 6 80
J 432+ 40* 26 20
10 259 + 28+ 55 0r!

i.:searchers tested a synthetic structural analog of


30 198 !241 66 0*
- . I A. called E5531. It was designed to work as an 100 71 + l8t 88 0*
i;:sonist of endotoxin to avoid the cascade of events * p < 0.01 versus control.
,r'.,j1119 to septic shock. t p < 0.001 versus control.

Which of the following statements is FALSE


regarding these data?
oP(oxoH)"

Meo nl./ 'u"*'


A. A dose of 10 prg E5531 allowed all the mice
tested to survive without complete suppression

,Ho)rp(o)oY NHR.
+^"-, oH
B.
of plasma TNF changes due to LPS.
The selected dosage of LPS given to the mice
was lethal to about 7570 of those tested.
C. Plasma TNF responded inversely to the dose
oRr
of E553 1 .
D. Plasma TNF inhibition led to increases in
E5531 survival rates for test animals.

lright @ by The Berkeley Review tr.t The Berkeley Keview


Specializing in MCAT Preparation
Biology Endotoxin Passage IV

21. The following graph shows the results of a series of 23. 85531 might act as an antagonist of Lipid A in an
experiments involving administration of E. coli organism infected with Gram-negative bacteria by:
intraperitoneally to mice:
I. competing for cellular surface receptors in the
host.
U. directly cleaving the terminal portion of
No treatment
released Lipid A to promote inactivity.
Itr. interfering with bacterial release of Lipid A.

A. I only
FA
Y- 60
B. I and II only
>. C. I and III only
(€ D. I,II, and trI
-640
r1

01020304050
Time after E coll infection (hrs)

Which conclusion is supported by the data in the


graph?

A. The antagonist alone provided effective


treatrnent in promoting long-term survival.
B. This bacterial strain is resistant to antibiotics.
C. The combination of antibiotic and endotoxin
antagonist was most effective in promoting
long-term survival.
D. The antibiotic alone provided effective
treatment in promoting long-term survival.

22. What is the purpose of administering both an


antibiotic and an endotoxin antagonist during an
infection involving Gram-negative bacteria?

L To increase blood bacterial count while


decreasing plasma endotoxin concentration.
U. To increase concentrations of cellular
mediators to attack the bacteria.
ilI. To destroy strains of bacteria that may be
resistant to the antibiotic that is used.

A. I only
B. I and II only
C. I, II, and III
D. None of the above

Copyright @ by The Berkeley Review 64 The Berkeley


Specializing in MCAT
tsiology Spinal Meningitis Passage V

Ir';rss;age \r (Questions 24-29) 25. According to the passage, the epithelial layer of the
throat:
\f:lingococcal meningitis, commonly known as
. :' :neningitis, is caused by the bacterium Neisseria A. traps bacteria and promotes infection.
- :' .*. i:iidis. The disease is known for its extremely rapid B. serves as a barrier against infection by the
: ,,: - .\ infected person first develops fever and malaise. bacteria.
- ;,:.,:r. within hours, these symptoms evolve into C. secretes mucus, which destroys bacteria.
r ::: :eadache, neck rigidity, and an aversion to bright D. becomes infected in spinal meningitis, and is
.: ": Ii these symptoms go untreated, a patient can lapse the ultimate cause of death.
" : ,:ra and a fatal form of shock.
,{:rrngococci normally colonize the lining of the
^:: .: So common is the organism that at any given time,
- '; -, , '-: of healthy people carry meningococci. Spinal
-r- rlrris begins when the organism invades the blood 26. In an experiment, tissue samples are taken from both
:'.::- :nd crosses the meninges (the membranes which the spinal cord and brain of a human subject with
.: ::re spinal cord and brain) into the cerebral spinal spinal meningitis. Which of the following statements
' -. : This fluid acts as a culture medium for rapid growth is MOST likely to be true?
".- . :::tefia.
A. In the brain and spinal cord, gray matter is
- - 960, researchers from the Army Institute
- found in the exterior region, while white
- ,;.-:ated the importance of host defense against matter is found in the interior region.
'":: :rrJocci. The group drew blood from thousands of B. In the brain and spinal cord, gray matter is
: ::s as they entered basic training and followed them found in the interior region, while white matter
"' -i:- ut this period. As men became ill, their stored is found in the exterior region.
,,r"r ::e tested for their ability to kill meningococci and C. In the brain, white matter is found in the
- : -:J with anti-meningococcal activity from the sera exterior, while in the spinal cord white matter
:::.:jr! men. The group found that the disease occurred is found in the interior.
-:.:-.r in recruits who had low anti-meningococci D. In the brain, white matter is found in the
. . '' ln sera before they became ill. Most adults have interior, while in the spinal cord white matter
' :::1r,e antibodies against meningococci. The recruit is found in the exterior.
": .r,Jicated that individuals lacking antibodies who
' :-: ;.:.DoSed to meningococci stood a high chance of
: , , r.-re i11. It appears that most people develop
:;..',e antibodies after their first exposure to
' r - r:,;occi, which keeps them from becoming ill.
Lr

27, People with a genetic deficiency in their


complement system are unusually susceptible to
spinal meningitis. Since the disease is rare in the
United States, a substantial proportion of cases seen
here may result from this deficiency. However, the
defect is unlikely to be a significant cause of an
epidemic. Which of the following statements
supports this claim?

A. In Nigeria and Gambia (countries with


repeated spinal meningitis epidemics), patients
with complement deficiencies are commonly
encountered during epidemics.
.,1, i'r-;h of the following organelles would be found B. In Nigeria and Gambia, patients with
' -:--n i,{eisseria meningitidis? complement deficiencies are not commonly
encountered during epidemics.
\" \ucleus C. A population's level of antibodies against
E, \Iitochondria meningococci rises after an epidemic.
i--, Ribosomes D. A population's level of antibodies against
tt Endoplasmicreticulum meningococci declines before an epidemic.

; 65 The Berkeley Keview


m Speciatizing in MCAT Preparation
Biology Spinal Meningitis Passaqe V

28. Neisseria lactamicq, a relative of N. meningitidis, is


part of the normal oral flora in humans. Young
children who acquire N. Iactamica throat infections
are frequently protected against meningococci
infections. The MOST likely reason for this
protection is that:

A. toxins released from N. lactamica destroy N.


meningitidis.
B. N. Iactamica colonizes in the throat,
eliminating any viable environment for N.
meningitidis.
C. antibodies produced against N. Iactamica are
effective against N. meningitidis.
D. antigenic determinants on N. lactamica are
different from N. meningitidis.

29. According to the passage, neck rigidity is a symptom


of spinal meningitis. This symptom is a result of:

A. endotoxins released from the bacteria, which


affect only neck muscle.
B. rapid growth of the bacteria in the CSF,
causing inflammation in the meningeal lining.
C. N. meningitidis over stimulating the nerves
innervating neck muscle.
D. N. meningitidls under stimulating the nerves
innervating neck muscle.

Copyright @ by The Berkeley Review 66 The BerkeleY Review


Specializing in MCAT Preparation
Biology Nuclear Dnvelope and Pores Passage VI

Passage VI (Questions 30-36) 30. According to the passage, the perinuclear space is
continuous with the:
The nuclear envelope of a cell encloses its DNA and
:efines the nuclear compartment. The spherical inner A. nuclear lumen.
:uclear membrane contacts the chromosomes and nuclear B. cytosol.
R-\As. The inner membrane is surrounded by the outer C. endoplasmic reticulum lumen.
-uclear membrane, which is usually studded with D. mitochondriamatrix.
.irosomes engaged in protein synthesis.

The nuclear envelope in all eukaryotes is perforated by


lclear pores. A typical mammalian cell contains 3000
.-;h pores. The pore is embedded in a large disk-like
).:ucture known as the nuclear pore complex. The
,:mp1ex is formed by protein granules arranged in an
,:risonal arrangement. The pore is an aqueous channel in 31. According to the passage, a nuclear envelope:
,.:ich water-soluble molecules shuttle between the
-
---leus and cytosol. The effective size of the pore has A. ensures DNA replication.
::in determined by measuring the rates of diffusion of B. confines translation to the cytosol.
=::led, non-nuclear components. The results show the C. prevents synthesis of ribosomal RNA inside
:e of the pore to be 9 nanometers in diameter and 15 the nucleus.
- ::,rmeters long. To offer a comparison
of size, the D. breaks down during interphase of the cell
- ::reter of a ribosome is approximately 30 nanometers. cycle.

\uclear pores can open to accommodate objects larger


'- 9 nanometers in diameter. The pore is like a
-
:r:lragrn that opens to just the right size when activated
'- . si-unal on an appropriate protein. The selectivity of
--:.:ar transport lies in the presence of nuclear import
. .1s. which are present only in nuclear proteins. The 32. The MOST likely reason that non-nuclear
.,.1. located anywhere within the protein, generally components are used to measure the effective size of
. -:.sts of four to eight amino acids that bear positively- the nuclear pore is that nuclear components:
. -.:_.:d side chains.

A. cannot be radiolabeled.
-- a cell is replicating its DNA, then it requires 106 B. are difficult to isolate.
' r. :'le molecules every three minutes to package the C. are accommodated by nuclear pores.
-- -nation molecule correctly. During mitosis, the D. are smaller than the pore size.
-
-:.:ar envelope breaks down, and the contents of the
-: :us mix with the cytosol. Upon reformation of the
' -:.:.rs. many soluble proteins are excluded, including
-. '. iormer residents of the nucleus. In time, these
-:-:r residents find their way back inside the nucleus.

33. In an experiment, nucleoplasmin (an abundant


nuclear protein) is proteolytically cleaved into a tail
and a head component. Both are attached to a 20-nm
piece of colloidal gold, which is easily visible under
the electron microscope, and placed in the cytosol.
In time, only the gold attached to the tail piece is
found near the nucleolus. The nuclear import signal
is found on the:

A. head, and contains Phe and Trp.


B. head, and contains Lys and Arg.
C. tail, and contains Phe and Trp.
D. tail, and contains Lys and Arg.

t'
-:ht O by The Berkeley Review The Berkeley Review
I
Specializing in MCAT Preparation
Biology Nuclear Envelope and Pores Passage VI

34. In an experiment, a signal sequence is chemically


attached to random amino acids on phosphofructo-
kinase. The result of the experiment is that the
enzyme: r
(
A. remains in the cytosol, because the location of
the nuclear import signal is unimportant. J
B. remains in the cytosol, because the location of
the nuclear import signal is important.
C. is transported to the nucleus, because the d
location of the nuclear import signal is n
unimportant. ti
D. is transported to the nucleus, because the s
location of the nuclear import signal is E
important. d
&
p
q
fr

35. According to the passage, nuclear proteins are ,il


MOST likely to: fi
OI
A. be resynthesized after mitosis.
B. be destroyed during mitosis.
C. lose their nuclear import signal after transport fr
into the perinuclear space. I
D. keep their nuclear import signal after transport m
into the perinuclear space. F
lfii
6!
m
&
36. In an experiment, small RNA molecules injected
into a frog oocyte nucleus are rapidly transported T:
into the cytosol. When these same RNA molecules h
are injected into the cytoplasm, they remain there. frru
The BEST explanation for this is that receptors on ,t
the: r
fi
A. nucleoplasmic face of the pore recognize the n
nuclear import signal. I
B. cytoplasmic face of the pore recognize the
nuclear import signal.
C. nucleoplasmic face recognize a nuclear export
signal.
D. cytoplasmic face recognize a nuclear export
signal.

Copyright @ by The Berkeley Review 68 The BerkeleY Review


Specializing in MCAT Preparation
Biology Lipids and Membranes Practice Passage Vtr

Passage VII (Questions 37-43) Experiment I


Biological membranes are composed of lipids, In a double-labeling experiment, researchers gave a
proteins, and carbohydrates. Membranes control the flow suspension of growing bacterial cells a l-minute pulse of
of molecules into and out of a cell through the use of radioactively labeled 32pgo3-. This pulse label was
exocytosis, endocytosis, ionophores, pumps, gates, designed to add radioactively labeled phosphoryl groups
junctions, and other membrane-embedded proteins.
only to phospholipids currently being synthesized by the
cell. After the 1-minute pulse, a reagent specific for PE,
Simple lipids include the terpenes and the cholesterol- trinitrobenzenesulfonic acid (TNBS), was immediately
derived steroids. Complex lipids include fatty acids, added to the cellular suspension.
triglycerides, waxes, phosphoglycerides and sphingo-
lipids. Lipids are soluble in nonpolar organic solvents, o
such as chloroform. They are marginally soluble in water. il
Because of the relative insolubility of lipids in water, HzC- O- C- Rl
amphipathic membrane lipids like the phosphoglycerides lo
and sphingolipids form lipid bilayers. The hydrophilic Irr
HC- O- C- R2 +
portion of the bilayer faces the extracellular space and
cytoplasm of the cell, while the hydrophobic portion
lo
tbrms the interior of the membrane.
liloozN
HlC- O- P- O- CH.CH2NHI
' lo
The transverse diffusion (flip-flop) of a lipid molecule PE o- t
across the membrane is quite rare, but the lateral diffusion \- nrso.
of a lipid molecule along the entire length of a bacterial
:ell like Escherichia coli takes about one second.
il
o+
HzC- O- C- Rl
The composition of lipids in biological membranes lo
depends primarily on the source of the membrane and, to
: small extent, the diet of the organism producing the
Irr O- C-
HC- R2

loo.N
nembrane. For example, in Escherichia coli, the weight
:ercent of total lipid of the phosphoglyceride phospha- Irr
HrC_ O_ P_ O* CH2CH2 _ NH
Noz

:idylethanolamine (PE) is about 65Vo, while that of


:holesterol is }Vo. Even though most membranes are sg
:oughly one-half protein, the protein-to-lipid ratio varies,
:epending on the function of the membrane. Experiment 2

Membrane proteins can be integral or peripheral' In this double-labeling experiment, the procedure is
Integral proteins are tightly bound to membranes by exactly the same as in Experiment 1, except that a 3-
:ldrophobic forces, and peripheral proteins are loosely minute time period is allowed to pass between the end of
:,,rund to membranes through electrostatic interactions the l-minute pulse of radioactively labeled 32pgo3- un6
:nd hydrogen bonding. Unlike lipid molecules, membrane
the addition of TNBS.
::oteins cannot undergo transverse diffusion. They can
:rfuse through the membrane only in a lateral fashion.
Horvever, both the lipids and the proteins found in a
:embrane are asymmetrically distributed.

.{symmetric membranes are synthesized on the


37, The bacterium Escherichia coli is classified as:

-'::rnework of preexisting membranes. Enzymes in the


::inbrane synthesize membrane lipids while membrane-
A. Gram-positive, with a plasma membrane
surrounded by a thick cell wall'
:,:und ribosomes synthesize protein to be incorporated
::o the membrane itself. Selective labeling experiments,
B. Gram-negative, with a plasma membrane
surrounded by a thin cell wall and an outer
, -:h as the one described below, confirm that membranes
membrane.
:.:r be constructed on the framework of preexisting C. Gram-negative, with a plasma membrane
:r:mbranes.
surrounded by a thick cell wall.
D. Gram-positive, with a plasma membrane
surrounded by a thin cell wall and an outer
membrane.

l:pvright @ by The Berkeley Review 69 The BerkeleY Review


Specializing in MCAT PreParation
Biology Lipids and Membranes Practice Passage Vtr

38. Complete hydrolysis of the phosphoglyceride 4t. The inability of an integral protein to diftuse
shown below by a eukaryotic cell yields how many transversely in a membrane is MOST likely due to:
products?
A. strong hydrophilic forces between the
membrane and the Protein.
o
il B. the protein's asymmetric distribution.
HrC- O- C- (CH2)r4cH3 C. the protein's numerous hydrophobic amino
loHH acid residues in the membrane's nonpolar
Ittll c- (cHz):- c= c- (cH2)7cH3
interior.
lo D. strong hydrogen-bonding forces between the
Itt o protein and the membrane.
-l
H,C-O-P-o-cH2cH2NH3
o9
A phosphatidYlethanolamine

A.0
8.3 42. The fluidity of a membrane is controlled by the
c.4 composition of fatty acids in it and by its
D.5 cholesterol content. A decrease in the membrane
fluidity of a bacterial cell is caused by:

L the presence of cholesterol'


39. In Experiment 1 it is observed that none of the
II. long-chain fatty acids with a cis double bond'
radioactively labeled PE molecules was labeled III. an ln"rease in the length of a saturated
with TNBS. This indicates that: hydrocarbon chain on a fattY acid'

I. TNBS can readily cross the bacterial A. I only


membrane. B. II only
il. synthesis of PE occurs on the extracellular C. III onlY
face of the membrane. D. I and III onlY
III. synthesis of PE occurs on the cytoplasmic
face of the membrane.

A. I only
B. II only
C. III only
D. I and III onlY
43. Sperm whales (Physeter catadon) can dive to
depths of 1100 m. A mature whale's head contains
about 4 tons of different waxes (commonly referred
40. In Experiment 2 it is observed that roughly half of to as whale oil). One such wax is spermaceti' an
the radioactively labeled PE was labeled with ester of a 16-carbon fatty acid and a 16-carbon
TNBS. This indicates that: alcohol. This wax has a melting point between 42-
4? 'F. Whales use these waxes to:
I. TNBS can readilY cross the bacterial
membrane.
II. PE can fliP-floP across the bacterial fi
membrane. cH3(cH?)14 - c- o- (cH2)15cH3

III. synthesis of PE occurs on both the Spermaceti


extracellular face and the cytoplasmic face of
the membrane.
A. increase their average density'

A. I only B. control their buoyancy in sea water'

B. II only C. conserve energy at great depths'

C. III only D. all of the above.

D. II and III onlY

70 The BerkeleY Keview


Copyright @ bY The BerkeleY Review
Specializing in MCAT PreParation
Biology Nonstandard Amino Acids Passage VItr

Passage VIII (Questions 44-50) 46. How many codons does it take to specify the
nonstandard amino acids?
The twenty common amino acids dictated by the
standard coding tables are not the only amino acids to A. Three
occur in biological systems. Nonstandard amino acids B. Four
occur in proteins, in biologically active peptides, such as C. Five
neurotransmitters, and in several metabolic pathways. D. None of the above

4-hydroxyproline and 5-hydroxylysine, components of


collagen fibers, are formed by modification of the 47. The enzyme ornithine transcarbamoylase transfers
standard amino acids after a polypeptide is synthesized. the carbamoyl group of carbamoyl phosphate to
Collagen is the most abundant protein in vertebrates. It is ornithine, yielding citrulline. This reaction is
present in connective tissue, such as bone, teeth, diagrammed below:
ligaments, tendons, and epithelial tissue.

Some neurotransmitters are made by modifications of O,I


2 ATP+HCO3 +NH3 -qi+ 2ADP+P;
single amino acids. For example, y-amino butyric acid
r'GABA) is made from glutamic acid by the enzyme o\
glutamic acid decarboxylase. Dopamine is made from the tlrN - "to
C- O- Por
amino acid tyrosine. Other nonstandard amino acids ,r Carbamoyl phosphate
lunction in urea biosynthesis (citrulline and ornithine), /
rmino acid metabolism (homocysteine), and methylation
reactions (S -adenosylmethionine).
-^n' I '
in l*-;'""']tt
I
I
T" lttt
f", Fn,.,
?",
CH"
?,
CH,
''
+t. The lysine residue modified to make hydroxylysine
"-i-',P", ["-t--t
in the collagen polypeptide is modified during: loo. l"-["i']tn
A. post-translation. Omithine Citrulline
B. post-transcription.
C. post-replication. The carbamoyl moiety is indicated by:
D. post-degradation.
A. I
B. II
45. Which of the following amino acids is proline? c.m
D. IV
l A. B.
HO
OrnO
HO
@rll
-i
HIN-C-C-O H3N-C-C-O
O 48. In a series of experiments, one group of rats is fed
I laC-radiolabeled proline as part of their diet. A
n
second group of rats is fed laC-radiolabeled
CHz CHz
I I I

CH" CH, hydroxyproline. The rats are killed, and their


I- l-
H- C_ OH CH. collagen is separated and analyzed. In which group
I t' is radioactive collagen synthesized?
CHt CH"
.c!l
- NHr ^l'
" NH, A. Both, because the Proline and the
hydroxyproline are both incorporated into the
collagen polypeptide.
C. D. B. Only the rats receiving proline have
radioactive collagen.
oIl C. Only the rats receiving hydroxyproline have
oocr.-i- ooc.
Acs, radioactive collagen.
on c I' D. Neither, because the Proline and
/ ,t""/H hydroxyproline are digested in the small
""*H intestine.

g,s 7l The BerkeleY Review


on Specializing in MCAT PreParation
Biology Nonstandard Amino Acids Passage VItr

49. Urea is a vehicle for removing nitrogenous waste


from the body in a relatively nontoxic form. In what
organ is urea synthesized?

A. The liver
B. The kidney
C. The gall bladder
D. The pancreas

50. Prolyl hydroxylase, the enzyme that hydroxylates


proline, requires vitamin C as a coenzyme. What
(
would be the effects of vitamin C deficiency, called
scurvy? I
s

I. Easy bruising i
il. Poor wound healing fi
Itr. Loose teeth fr

A. I only
B. I and II only h
C. II and III only s
D. I, II, and III ri
il
0
a
fr
q
iG

Copyright @ by The Berkeley Review 72 The BerkeleY Reviev


Specializing in MCAT Preparation
Biology Amino Acid Qharacteristics Passage IX

Passage IX (Questions 51-58) 51. The pH of a solution can be expressed as:

Proteins are initially synthesized from the amino (N) A. - ln [H+].


terminus to the carboxyl (C) terminus during translation B. lo916 [H+].
as polypeptide chains from a pool of 20 common amino
acids. Each of these amino acids is coded for by at least C. ln [H+].
one codon in the genetic code. Once an amino acid is D. - log1s [H+1.
incorporated into a growing polypeptide chain, it can
undergo post-translational modification, thus allowing for.
an even richer diversity in protein structure.

Each of the common 20 amino acids contains an ct-


carbon atom, to which four substituents are bonded: a 52. A mixture of the common 20 amino acids in solution
hydrogen atom, an amino group, a carboxyl group, and a at physiological pH would predominately take on
side chain. It is the side chain that places each amino acid the characteristic of being:
rn a unique grouping. Nine amino acids have nonpolar
side chains, six have uncharged polar side chains, and five A. nonpolar.
have charged side chains. B. positively charged and monovalent.
C. negatively charged and monovalent.
The cr-amino and cx-carboxyl group of each amino acid D. dipolar.
'iear
a dissociable hydrogen atom with a characteristic
:Ku value. For leucine, an amino acid with a nonpolar
.ide chain, the pKa for the cx-amino group is 9.6, while
:re pK. for the o-carboxyl group is 2.4. The addition of
rne equivalent of a base (NaOH) titrates the cr-carboxyl
53. All of the following amino acids are found near the
;roup and converts the fully protonated form of leucine center of a long polypeptide chain. Which amino
:to the zwitterion form whose net charge is zero (Figure acid in this protein molecule is not ionizable?
. . The addition of a second equivalent of base titrates the
,r:-amino group.
A. Histidine
o o
B. Isoleucine
C. Lysine
- cooH
(9t o 9oo
coo
I D. Arginine
'I - C- H
:lrN
*oHo u,N-g-H *oHo H2N- C-H
I

CH" CH,
t' @ l- @ t-
HIC- C- H +H HrC-C-H
'l +H HIC-
-l C-H
I

CHr CH: CH:

Figure 1 54. The titration of leucine with sodium hydroxide


generates the titration curve shown below. The
If a small number of amino acids are linked together, isoelectric point for this amino acid is close to:
-: molecule is called a peptide or apolypeptide.
-'.:otropin-releasing hormone (TRH) is an example of a
.
eptide containing three amino acids joined by peptide * 2.0
- --Js (Figure 2). Proteins generally contain 50 or more od_
-.-:ro acids.
z t.J
C rn
o r.v
HHOHOHO E
@l I ll I ll I ll '= 0.5
H - N - C- C- N -C - C -N - C- C- NH2 cl
tD
IV

N-u
"-N\il A. 1.0
B. 3.0
Thyrotropin Releasing Hormone (TRH)
C. 6.0
Figure 2 D. 10.0

* @ by The Berkeley Review 1C The BerkeleY Review


oo -ght
Specializing in MCAT Preparation
Biology Amino Acid Characteristics Passage IX

55. Phenylalanine, an essential amino acid, is converted 58. In Figure 2, the N-terminus nitrogen in TRH is
to tyrosine in the following reaction: indicated by which Roman numeral?

A. I
o B. II
@ foo c.ru
H3N- C- H D. IV
cH"
t"
o
Phenylalanine

This reaction can be described as a:

A. carboxylation.
B. hydrolysis.
C. hydration.
D. hydroxylation.

56. Proteins are molecules that contain 50 or more


amino acid residues, and they can act as excellent
buffers because of:

A. their hydrogen-bonding capabilities in forming


secondary and tertiary structures.
B. the ease with which H+ and OH- ions can be
absorbed once the protein is hydrolyzed.
C. the ability of the terminal regions of the
protein to accept or donate H+ ions.
D. the wide range of pKu values found within the
protein.

5t. Complete hydrolysis of the tripeptide thyrotropin-


releasing hormone (TRH) requires how many moles
of water?

A.2
8.3
c.4
D.5

Copyright @ by The Berkeley Review 74 The BerkeleY Review


Specializing in MCAT Preparation
BiOlOgy Fluorescent Kecovery aft,er Photobleaching (FKAP) Passage X

Passage X (Questions 59-65) ]l

Experiment I I

6)

In 1970, a hybrid cell was artificially produced by the C)


O
fusion of mouse cells with human cells. Two differently O
!
labeled antibodies were used to differentiate between
human and mouse plasma membrane proteins. At t = 0
minutes, the labeled antibodies were confined to their
respective halves. At t = 40 minutes, the two different Time ---------->
.ntibodies were mixed over the entire surface of the
hvbrid cell. This experiment is shown in Figure 1:
Figure 3

59. In Experiment 1, the incubation temperature was


lowered significantly. The time required for the
human and mouse plasma membrane proteins to
reach the state pictured in Figure lb would be:

A. increased, due to increased fluidity of the


plasma membrane.
Figure I B. increased, due to decreased fluidity ofthe
plasma membrane.
C. decreased, due to increased fluidity ofthe
plasma membrane.
D. decreased, due to decreased fluidity ofthe
:,:eriment 2 plasma membrane.
To study the lateral diffusion rates of membrane
:- :eins, one can use fluorescence recovery after
: -:tobleaching (FRAP). Most commonly, fluorescent
- .:lovalent antibodies are attached to selective membrane
:-"::ins. The tightly bound fluorescent ligands are 60. The monovalent antibodies used in Experiment 2 are
"=.:hed in a small area by a laser beam, and the time
fragments of antibodies that have only one antigen-
, r,.in for adjacent membrane proteins carrying
antibody binding site. The MOST likely reason for
-::l:ached fluorescent antibody molecules to diffuse into using such antibodies is that:
"= rleached area is measured. This experiment is shown
' - .gure 2 and Figure 3: A. only monovalent antibodies can carry
fluorescence.
B. only monovalent antibodies can attach to
membrane proteins.
c. the use of monovalent antibodies prevents
cross-linking between neighboring molecules.
D. a bivalent antibody would attach to more than
one antigenic determinant.
lur',"l [l [-l *".nu.ru
{} :L

61. The results of Experiment I provide evidence for:

A. protein flip-flop.
B. lateral diffusion.
Figure 2 C. simple diffusion.
D. rotationaldiffusion.

ii :'_ri.lt O by The Berkeley Review 75 The Berkeley Review


Specializing in MCAT Preparation
BiOlOgy Fluorescent Kecovery aft,erPhotobleaching (FRAP) Passage X

62. The graph shown in Figure 3 shows that the highest 65. In eukaryotic cells, the plasma membrane contains
level of recovery is slightly lower than the beginning relatively large amounts of cholesterol. At such high
level offluorescence. This is because; concentrations, cholesterol has the effect of:

A. not enough time was allowed during the A. decreasing the fluidity of the membrane by
recovery process to obtain the full level of inhibiting hydrocarbon chains from
fluorescence. crystallizing.
B. the level of fluorescence had naturally B. decreasing the fluidity of the membrane by
declined. promoting hydrocarbon-chain crystallization. r
C. the laser beam only temporarily bleaches a C. increasing the fluidity of the membrane by
population of the fluorescent ligand. inhibiting hydrocarbon chains from
D. the laser beam permanently bleaches a crystallizing.
population of fluorescent ligand. D. increasing the fluidity of the membrane by
promoting hydrocarbon-chain crystallization.

63. The graph below depicts the FRAP results for four
different surface glycoproteins under similar
conditions:

o
C)
o
o
!

tr

The rate of diffusion is lowest for:

A. Glycoprotein A.
B. Glycoprotein B.
C. Glycoprotein C.
D. Glycoprotein D.

64. In an experiment, a single glycoprotein is inserted


into a synthetic lipid bilayer. Using FRAP, it is
determined that the rate of diffusion of the
glycoprotein is significantly higher in vito than in
vivo. The MOST likely explanation for this increase
in the rate of diffusion is that:

A. antibodies of a lighter weight are used for


tagging in this synthetic system.
B. cross-linking occurs between antibodies in this
synthetic system.
C. the lack of bulky intracellular oligosaccharide
chain interaction increases the rate of
diffusion.
D. the lack of bulky extracellular oligosaccharide
chain interaction increases the rate of
diffusion.

Copyright @ by The Berkeley Review 76 The Berkeley Revier


Specializing in MCAT Preparatia
Biology Gram-Negative and Gram-Positive Bacteria Passage Xl

Passage XI (Questions 66-72)

-\n important difference between prokaryotic cells and


:-riaryotic animal cells is the presence (in prokaryotes) of o^
: protective cell wall sunounding the plasma membrane.
to
a)r
Bacteria can be divided into two groups: Gram- (D

:,:sitive and Gram-negative. Division into these two


_.:rups is based on their reaction to the Gram stain, as ////////
MM'LM M-LM M-LM M
rJ
(D
"::lined in the procedure given below: rd
:
dt d d d d d d
'/////// cll oO
<o
M' M-M M-LM M-LM
Step 1 ///////
GGGGGGG
(Doq

A suspension of bacterial cells is stained with ,G o


crystal violet. This procedure takes about I minute. Face2

Step 2
Iodine (I2) is added to the suspension and o|!
complexes with the crystal violet to fix the cells.
This procedure takes about 3 minutes. o

Step 3 Face 1
Alcohol is added to the suspension and removes Transmembrane
:o1or from the cells. This procedure takes about 30 protein
seconds.
Figure I
Step 4
Safranin, a red-colored counterstain, is added to the Antibiotics like penicillin inhibit transpeptidation
suspension of cells. This procedure takes about 1 to reactions, while enzymes like lysozyme hydrolyze
I minutes. polymers of N-acetylglucosamine and N-acetylmuramic
acid. Both actions weaken the ce1l wall and lead to
Staining techniques can generally be divided into three eventual osmotic lysis. However, if the cell is in osmotic
- :,es: positive staining, negative staining, and differential equilibrium with its environment, intact spherical
.'-:ing. Positive staining employs dyes to stain cells in structures called spheroplasts and protoplasts can form.
':-.r to increase their contrast. Negative staining allows Spheroplasts retain some fragments of the cell wall, but
;:-is to be seen in outline against a stained background. protoplasts do not.
-:isvsnsitl staining can involve both positive and
:.rative staining.
Gram-positive and Gram-negative cells differ from
---' another in the structure of their cell walls. The cell 66. Escherichia coli has the characteristic shape of a
*:.1 of Gram-positive bacteria contains peptidoglycan, Gram-negative bacterium, namely a:
, ,.,trted polysaccharides, and teichoic acids. The cell wall

: Gram-negative bacteria contains peptidoglycan, A. coccus.


: : -' spholipids, lipopolysaccharides, and assorted proteins' B. rod.
-.:, Gram-positive cells, peptidoglycan accounts for about C. spiral.
: , :o 90 percent of the weight of the wall; in Gram- D. spiral helix.
:=::rive cells, it accounts for about 10 percent.
Peptidoglycan is composed of two acetylated amino
-;ars. N-acetylglucosamine (G) and N-acetylmuramic
p(1,4) linkage, and a small
'.:-: iM), linked together in a 67. Invariant cell structures in prokaryotes include all of
: *::Lber of amino acids, including D-glutamic acid and D-
the following, EXCEPT:
,!.:rine (Figure 1). Attached to each N-acetylmuramic
-,:-J residue is a tetrapeptide side chain. Transpeptidase
::rrmes catalyze cross-linking between some of these
A. a cell wall.
:e chains and increase the stability of the peptidoglycan.
B. a nuclear region.

:-rrmes called autolysins open up the peptidoglycan


C. ribosomes.

:::ng cellular growth.


D. a plasma membrane.

- :lyright @ by The Berkeley Review /l The BerkeleY Review


Specializing in MCAT PreParation
Biology Gram-llsgative and Gram.Positive Bacteria Passage XI

68. After being subjected to the Gram staining


procedure, Gram-positive and Gram-negative cells
show which of the following colors, respectively?

A. Violet; colorless
B. Blue;violet
C. Colorless; red
D. Blue; red

69. The Gram staining procedure is a:

I. positive staining technique.


rI. negative staining technique.
III. differential staining technique.

A. I only
B. II only
C. III only
D. I and III only

70. Based on Figure 1, the enzyme transpeptidase would


be expected to be found near which membrane face
in both Gram-positive and Gram-negative bacteria?

A. Face I
B. Face2
C. Face 3
D. Face 4

7t. The antibiotic penicillin weakens the peptidoglycan


layer in virtually all species of prokaryotes by
inhibiting transpeptidation. This can lead to lysis
hnd eventual death of the cell. Penicillin-induced
lysis can be prevented by:

A. inhibiting enzymes called autolysins.


B. increasing the growth rate of the cells.
C. inhibitingprotoplastformation.
D. decreasing the solute concentration outside the
cell.

72. One would expect a bacterial cell with the structures


shown in Figure 1 to be:

A. nonresistant to penicillin, because the


antibiotic can cross the outer membrane.
B. resistant to lysozyme, because the enzyme
cannot cross the outer membrane.
C. resistant to both penicillin and lysozyme,
because neither can cross the outer membrane.
D. nonresistant to lysozyme, because the enzyme
can cross the outer membrane.

Copyright @ by The Berkeley Review 78 The Berkeley Review


Specializing in MCAT Preparation
Biology Kaffinose Practice Passage XII

Passage XII (Questions 73-79) 73. Raffinose can be described as a:

Mammals first arose some 200 million years ago A. reducing sugar.
during the Mesozoic Era. Today, there are at least 4500 B. nonreducing sugar.
species of mammals. They can be divided into three C. disaccharide.
primary groups on the basis of their mode of D. glycoprotein.
reproduction . The monotremes are the only mammals that
lay eggs. The embryo of the marsuplal develops in the
uterus for a very short time after fertilization. As soon as
the immature marsupial is able to survive outside the
mother's uterus, it leaves and crawls into a marsupium
(pouch) where the mother's mammary glands are located
to finish the developmental process . Placental mammals
develop in the uterus to a much later stage of maturity 74. Fructose can be classified as:
before parturition.
I. an aldose.
The koala (Phascolarctos cinereus) is an arboreal il. a ketose.
marsupial native to Australia that feeds exclusively on the
leaves of the eucalyptus tree. One of the naturally ilI. a furanose-
IV. a pyranose.
occurring carbohydrates found in eucalyptus leaves is the
oligosaccharide raffinose (see structure below). Placental
A. I only
mammals, such as humans (Homo sapiens), typically
B. II only
obtain raffinose from leguminous seeds like peas and
c. II and III only
beans.
D. I and IV only
CH,OH Bond
I

0
o- 9ut
HOH H OH
HOH 't5. The bond between glucose and fructose is in the:
Galactose

I. o(l-+2)configuration.
IL B(2-+1)configuration.
Glucose Fructose III. a(l-+5) configuration.
ry. 9(5-+1) configuration.
For all mammals, carbohydrates provide a large
:crtion of the daily caloric requirement. However, before A. I only
:r oligosaccharide can be utilized by an organism, it must B. I and II only
..rst be degraded by enzymes (salivary amylase and C. III only
,: ancreatic amylase) into smaller units called D. III and IV only
':anosaccharides. Monosaccharides such as galactose,
:.ucose, and fructose are rapidly absorbed across the
:ucosal cells of the wall of the duodenum and ileum of
--.e small intestine by a carrier-mediated transport system.
I'iigosaccharides that are not hydrolyzed are not
. : sorbed.

Raffinose is an example of an oligosaccharide that 76. The bond between galactose and glucose is in the:
.nnot be hydrolyzed in the small intestine. However,
.:e this carbohydrate reaches the terminal portion of the A. cr(6+1) configuration.
:um and the beginning of the large intestine, it can be B. F(6+1) configuration.
, drolyzed into its constituent monosaccharides by C. p(l+6) configuration.
:srdent bacteria. Many of these monosaccharides can be D. cr( I -+6) configuration.
rerobically metabolized by the bacteria to produce
: npounds like lactate, methane, carbon dioxide, and
, Jrogen gas.

The Berkeley Keview


Specializing in MCAT Preparation
Biology Raffinose Practice Passage XII

77. Galactose and glucose are epimers of one another


about which carbon atom?

A. c-l
B. c-4
C. c-5
D. c-6

78. If the galactose residue were to be hydrolyzed from


raffinose, the disaccharide left over would be:

A. maltose.
B. lactose.
C. sucrose.
D. none of the above.
i
I
I
t
I
79. After ingestion of leguminous seeds, many people I!
experience a period of flatulence. This problem can d
be traced to: 1o

t
A. the loss of bacterial enzymes that degrade u
oligosaccharides. ,i
B. oligosaccharides that cannot be hydrolyzed by
human intestinal enzymes.
C. a decrease in the products of anaerobic I
metabolism. D
D. a decrease in intestinal motility. o
d
U
ni
ft
d
h
u
I
h
n[
II
iil
m
h
fl

n
ni

Copyright @ by The Berkeley Review ao The BerkeleY Keview


Specializing in MCAT Preparation
Biology Mitosis and Meiosis Passage XItr

Passage X[II (Questions 80-87) Normal Down's


Mother Father Child Child
Mitosis and meiosis are essential for all eukaryotic 6
organisms. After fertilization, the zygote depends on 0)

mitosis for growth and development. Mitosis is also


N^
rzg
q I I
important for replacement of certain tissue cell types coo 6.t
during the life of the organism. bo.- -
- I
,!'- t I I I
After cytoplasmic division (cytokinesis) of a cell
during mitosis, the resulting two daughter cells enter
I I I
interphase of the cell cycle. Interphase is a period of Autoradiograph of Family 1

intense metabolic activity. During interphase, each


daughter cell proceeds through an initial growth period, Normal Down's
followed by a period of DNA synthesis (S phase), and Mother Father Child Child
then another growth period before mitosis and 6
cytokinesis. Mitosis occurs in somatic cells and results in o
N^
daughter cells, which are diploid. 6g
C61
o3 I
In contrast, meiosis occurs in the sex cells and results I
in the production of gametes which are haploid. Meiosis
b0.- I
E- 2 I I-
consists of two cellular divisions called meiosis I and
1
meiosis II. Before the onset of meiosis I, chromosomes -
have duplicated during the S phase. Homologous Autoradiograph of Family 2
chromosomes pair (synapse) and form a tetrad consisting
of four chromatids. During meiosis I, there is a reductive Figure 1

division in which each tetrad separates to form dyads,


each containing two sister chromatids. The dyads are 80. Which of the following sequences represents the
separated during meiosis II into monads. Each monad correct order of the phases of the cell cycle?
represents a single chromosome, the carrier of genetic
information in the form of genes. A. Metaphase, telophase, S, G2, prophase
B. Interphase, S, G1, metaphase, prophase
There are many techniques in genetics that can be used
'.o analyze genes. In Southern blotting, DNA from a C. Telophase, anaphase, Gr, Gz, metaphase

;articular chromosome is cut into fragments using a


D. G2, anaphase, telophase, S, G1
:estriction endonuclease and then separated according to
size using agarose gel electrophoresis. The DNA in the
sel is next treated with alkali and denatured to yield
81. Which of the following mature cell types rematns ln
the G1 phase of the cell cycle?
.ingle-stranded DNA. A nitrocellulose paper (used
iecause of its ability to bind single-stranded DNA) is
:laced on top of the gel, and a buffer flow is used to I. Kidney epithelial cells
-:ansfer the DNA from the gel to the nitrocellulose paper.
IL Skeletal muscle cells
The exact DNA fragment pattern in the gel is maintained IfI. Nerve cells
rn the nitrocellulose paper. Next, a probe containing a
,.nown sequence of radioactively labeled DNA is used for A. I only
rr bridization to the DNA on the nitrocellulose paper. The
B. I and II only
:osition of the DNA fragments that anneal to the probes C. III only
.:e revealed by autoradiography. In Northern blotting' D. II and III only
l*\A fragments can be transferred to nitrocellulose paper.
l.r Western blotting, proteins can be analyzed using an 82. DNA synthesis during the S phase of the cell cycle
SDS gel.
can be detected by using radioactively labeled DNA
precursors. The BEST technique for monitoring the
The Southern blotting technique is used to analyze the incorporation of these labeled precursors is:
l,NA of two different families. Each family has a child
'. ith Down syndrome and
a child who is unaff'ected. The
.rtoradiographs of the DNA from the mother, the father,
A. Northern blotting.
B. Southern blotting.
.:d the children of each family are shown in Figure 1: C. Western blotting.
D. autoradiography.

; l,rpyright @ by The Berkeley Review al The BerkeleY Review


n Specializing in MCAT Preparation
Biology Mitosis and Meiosis Passage Xftr

83. Which radioactively labeled precursor would be 86. In the autoradiographs of Family I
and Family 2,
BEST for monitoring DNA synthesis during the S which parents show an abnormal division during
phase of the cell cycle? gametogenesis?

A. 3H-thymine A. Family (maternal); Family 2 (maternal)


B. Family (paternal); Family 2 (maternal)
B. 3H-cytosine
C. Family (maternal); Family 2 (paternal)
C. 3H-uracil
D. Family (paternal); Family 2 (paternal)
D. 3H-adenine

87. The mother of Family 2 has a haploid chromosome


84. Down syndrome is due to trisomy: number of 23. How many chromatids are present in
metaphase of the second meiotic division?
A. 13.
B. 18. A. 23
c. 21. B. 46
D. 22. c. 69
D. 92

85. During nuclear division, chromosomes or


chromatids move to the opposite poles of the cell.
This is called disjunction.If disjunction were to fail,
two chromosomes or two chromatids would go to
one pole and none would go to the other pole. This
is called nondisjunction. Primary nondisjunction
occurs at the first meiotic division, while secondary
nondisjunction occursat the second meiotic
division. Which of the following diagrams
represents secondary nondisjunction in
spermatogenesis?

A.

B.
@@@@
c.
@@oo
o@@@
@ooo
D.

Copyright @ by The Berkeley Review a2 The BerkeleY Revier


Specializing in MCAT PreParation
Biology Lipids: Four Groups Passage XIV

Passage XfV (Questions 88-94) The most abundant steroid in animals is cholesterol
(Figure 3). Cholesterol is found in cellular membranes,
Lipids can be divided into four distinct groups: fats and it is also the precursor to many steroid hormones and
rnd waxes, complex lipids, steroids, and prostaglandins bile salts.
.ind leukotrienes. In animals, an important use for lipids is
:he storage of energy, especially in the form of fat. If all
;hree alcohol groups of glycerol are esterified to the
:ndividual carboxylic acid groups of various fatty acids, a
iat called a triglyceride is formed (Figure 1).
Tnglycerides are simple lipids.

o
cH'
H?c- o- !
lo Figure 3

"l-to
I
n- fl cH'
Prostaglandins and leukotrienes are synthesized from
cH' arachidonic acid, a C20 unsaturated fatty acid (Figure 4).
".1-o-! o
A Triglyceride
cH'
se- [
Figure I Arachidonic acid
Figure 4
Fatty acids attached to the glycerol backbone can be
:-ther saturated or unsaturated, and they can be of varying Prostaglandins and leukotrienes mediate hormonal
,:rgths. The Crs saturated fatty acid attached to the action, and both cause inflammation and fever.
,.1cerol backbone in Figure 1 is stearic acid. Saturated
."ts are solids at room temperature. Unsaturated fats are
;;nerally liquids at room temperature.

The membranes that surround cells and cellular


,:"anelles contain complex lipids. These lipids can be
::'.ided into two groups: phospholipids and glycolipids. 88. The fat shown in Figure 1 can be described as:
?:ospholipids contain an alcohol, fatty acids, a phosphate
::lup, and either choline, ethanolamine, or serine. If the A. a carboxylic acid.
.l:oho1 is glycerol, the phospholipid is called a B. an alcohol.
:;:osphoglyceride. A common phosphoglyceride is C. an ester.
:rosphatidyl choline (Figure 2). It the alcohol is D. a ketone.
.:hingosine, the phospholipid is called a sphingolipid.
,,i-' colipids are sphingosine-based complex lipids that
::rtain carbohydrates like glucose and galactose.
89. The chemical structures shown in the passage are a
triglyceride (Figure 1), phosphatidylcholine (Figure
o 2), cholesterol (Figure 3), and arachidonic acid
ctt' (Figure 4). The arrangement of these lipids in order
:,-c- o-rj of decreasing polarity is:
lo t"'
-,|-o-'j
I CH,-
A. phosphatidyl choline > arachidonic acid > a

o triglyceride > cholesterol.


I or B. cholesterol > a triglyceride > arachidonic acid
!-n-'J-o&T-t"'
. .ICH: > phosphatidyl choline.
oo C. arachidonic acid > a triglyceride > cholesterol
PhosPhatidYl choline > phosphatidyl choline.
D. phosphatidyl choline > a triglyceride >
Figure 2 arachidonic acid > cholesterol.

a5 The BerkeleY Review


lew --:p1'right @ by The Berkeley Review
Specializing in MCAT PreParation
ron
Biology Lipids: Four Groups tucryeXIV

90. The following series of diagrams represents the 92. Arachidonic acid can contribute to the formation of:
hydrocarbon portion of individual fatty acids that
make up different triglycerides. What is the I. simple lipids.
increasing order of the melting points for the IL complex lipids.
following four triglycerides? Itr. prostaglandins.

A. I and II only
B. II only
C. III only
D. I,II and Itr

Tryglyceride I
CHr
93. A bacterial culture growing at 38 'C is transferred to
cHl an environment where the temperature is 25 'C. All
cHr of the following initial responses should be
observed, EXCEPT:
Tryglyceride II
A. an increase in the synthesis of unsaturated fatty
acids.
B. a decrease in the fluidity of the bacterial
membranes.
c. an increase in the synthesis of long chain fatty
acids.
Tryglyceride III D. an increase in the synthesis of short chain fatty
acids.

cHr
CH:
CH:
94. Hydrolysis of membrane phosphoglycerides might
Tryglyceride IV yield all of the following compounds, EXCEPT:

A. glycerol.
A. I, IV,Itr, II B. glucose.
B. II, I, W, III C. choline.'
C. III,IV,I, II D. serine.
D. III, I, IV, il

9t. Elevated levels of cholesterol in the blood serum can


lead to plaqueJike deposits on the inner walls of the
arteries, a condition called atherosclerosis. This
disease reduces the diameter of the blood vessels
and leads to adverse clinical conditions, such as a
stroke or heart attack. Atherosclerosis results from
the:

A. increased synthesis of bile salts from


cholesterol.
B. increased synthesis of steroid hormones from
cholesterol.
C. removal of excess cholesterol from cellular
lipid bilayers.
D. insolubility of cholesterol in water.

Copyright @ by The Berkeley Review 84 The BerkeleY Review


Specializing in MCAT Preparation
Biology Bacteriophage Lambda Passage XV

Passage XV (Questions 95'100) 95. The lambda phage is largely incapable of infecting
bacterial species other than E. coli. This is because:
Bacteriophage lambda is a nonenveloped DNA virus
rvhose host is the bacterium E. coli. The infection process A. E. coti has a circular chromosome, while other
begins when the virus is adsorbed to the surface of the bacterial species do not.
hoit cell. Once attached, the phage injects its DNA across B. other bacterial species have circular
the plasma membrane and into the cytoplasm of the E' chromosomes, while E. coli does not'
:oli it is transformed from a linear strand to a
cell, where C. other bacterial species lack a chromosomal
:ircular one. After this point, the infection may take one integration site for lambda DNA.
rf two different pathways: D. other bacterial species lack specific cell
surface proteins that lambda normally binds.
Lttic Pathway
After injection, the circularized DNA is transcribed
ind translated by the host cell's DNA. The resulting
::otein -products make up the future viral capsid. The
..mbda DNA then undergoes several rounds of
::plication, after which it is packaged into the newly
:ssembled capsids. Multiple new viral particles are
: rrmed, ultimitely causing the lysis, or bursting, of the
-,:st E. coli cell. During lysis, the new lambda phages are
:.ieased. 96. A strain of lambda phage exists that produces a
defective integrase enzyme, meaning it is unable
-', sogenic Pathway to:
-{fter injection, the circularized DNA directs the of E. coli
::rduction bf a viral protein known as integrase'This A. avoid causing the lysis upon
;ir\me causes the circular lambda DNA sequence to infection.
-:eErate itself into the bacterial chromosome' Once B. to infect E. coli.
.:ited, the lambda DNA (now termed a provirus) is C. to replicate its DNA.
"::Licated along with the E' coli host cell. The provirus D. to enter the lytic PathwaY.
, remain hidden and inactive in this latent state for
*.":
-l:ip1e replications. If the E. coli host is damaged by UV
;:t or ionizing radiation, the latent lambda provirus can
-.,.r'. itself from the bacterial chromosome and enter
":: -'. tic pathway.

97. Which of the following statements would represent


an evolutionary selective advantage for the lambda
virus?

L Upon damage to the host E' coli cell, a


lambda provirus can excise itself and enter
the lYtic PathwaY'
II. Lambda-does not always kill its host cell
immediatelY'
III. Lambda DNA is almost completely resistant
to mutation.

A. I only
B. II only
C. I and II only
D. I, II, and III

85 The BerkeleY Keview


r _Jrt O by The BerkeleY Review
Specializing in MCAT PreParation
Biology Bacteriophage Lambda Passage XV f,

98. A lambda DNA strand is labeled with radioactive


32P and packaged into a viral capsid. The newly
made virus is allowed to infect an E. coli cell. The
cell does not lyse, but instead begins normal
I
mitosis. At this stage, radioactivity would MOST
likely be detected:

A. in the nucleus of the E. coli host cell.


B. in the cytoplasm of the E. coli host cell.
C. in the ribosomes of the E. coli host cell.
D. outside of the E coll host cell.

99. A certain strain of E. coLi produces a cytosolic


endonuclease that cleaves lambda DNA at certain
sites. This should inactivate:

A. only the lytic pathway of infection.


B. only the lysogenic pathway of infection.
C. both the lytic and lysogenic pathways of
infection.
D. neither infection pathway.

100. Lambda phages entering the lytic or lysogenic


infection pathways do NOT differ from each other
in their:

A. mode of viral DNA injection.


B. mode of viral DNA replication.
C. immediate lethality to the host cell.
D. ability to remain latent for long periods.

Copyright O by The Berkeley Review a6 The Berkeley Re


Specializing in MCAT
Biology Structure and Function in Cells and Viruses Section VI Answers

1. C is correct. Recall that Gram-positive bacteria do not have an outer membrane, while Gram-negative bacteria do.
In addition, the peptidoglycan layer in Gram-positive bacteria is much thicker than the peptidoglycan layer in Gram-
negative bacteria. The stain does take advantage of the fact that bacteria have different cell wall structures. The
correct choice is C.
) B is correct. Activation of the complement system (a system of proteins found in the blood, which participates in
the immune response) brings about diffusible factors that stimulate the secretion of histamine from mast cells and
basophils. Even if we did not know the details of what complement did, we should be aware of the fact that it takes
part in the immune system response. Therefore, it is unlikely that activation of complement is part of the repression
of the inflammatory response. The rest of the answers all lead to suppression of response. The correct choice is B.
3. C is correct. This function involves the release of chemical messengers that are secreted into the extracellular fluid.
Those messengers then act upon the cell that secreted them. This is best describing the self-stimulation addressed in
the question. Paracrine function is secretion of chemical messengers that act locally, but not on the cell that secreted
the signal. The correct choice is C.
4. C is correct. We should rcalize that sepsis is the result of an acute bacterial infection. Therefore, if an individual is
infected with a bacterium that is unresponsive to antibiotics, the infection may becomes acute and lead to septic
shock. Thus, we are looking for an explanation for the rise in sepsis. The creation of highly resistant bacteria
through the overuse of antibiotics certainly would be a valid explanation for the increase in sepsis. The rest of the
choices would all lead to the conclusion that sepsis should be declining: Less invasive procedures should lead to a
smaller possibility of infection. Less frequent use of immunosuppresive therapy would not increase a person's
susceptibility to infection. Finally, a decline in treatable diseases would not create the potential for resistant strains.
The correct choice is C.
5. D is correct. We are told that this CD14 molecule lacks a membrane-binding domain. Therefore, it does not bind to
any plasma membrane, regardless of the face. We can automatically eliminate choices A and B. To distinguish
between choices C and D, we need to think about the function of CD14. It helps transduce a signal from the outside
wodd into the endothelial cell. Therefore, it is not likely to be found already inside the cytosol of the endothelial
cell. It is found instead in the blood and extracellular fluid of the tissue. The correct choice is D.

6. B is correct. From the picture in the question, we learn that the rough microsomes fall below in the sucrose gradient
when compared to smooth microsomes. This must mean that they are more dense. Choice C is eliminated. The
rough microsomes have ribosomes attached all over them. They are not made from radically different proteins.
Recall that ribosomes are made up of RNA and protein. This will certainly have the effect to add more mass without
affecting the volume of the ribosome as much. The net effect is that the rough microsomes are more dense and fall
lower than the smooth microsomes in a sucrose gradient. The correct choice is B.
7. A is correct. In the passage, it is stated that the rough microsome always has on its outside surface the ribosomes.
This is implying that the topology of the ER is conserved. In other words, the outside of the rough ER in an intact
cell is the cytosol. The ribosomes that are attached to the ER are attached on the cytoplasmic face of the ER
membrane. This case is no different for the microsomes. They have not been turned inside out. Therefore, it
becomes clear that the exterior of a rough microsome is equivalent to the cytosol. Consider the other choices. The
lumen of the nucleus is not correct. The lumen of the nucleus is enclosed by the nuclear membrane, and it is not
continuous with any other cellular fluid. The lumen of the ER is of course equivalent to the inside of the microsome.
That is why we take advantage of these mini-ER systems for experiments. Finally, it is evident that the lumen of the
Golgi has little to do with the exterior of the microsome. Choice D can be eliminated. The correct choice is A.
8. B is correct. This,problem requires that we think about the processing that occurs on a protein that is being
transported into the'ER. It is stated in the passage that the signal peptide that is responsible for bringing that protein
into the ER is cleaved once in the ER. Therefore, proteins that make their way into the ER will be shorter than they
would have been if they had remained in the cytoplasm. Since microsomes are mini-ERs, the same holds. In the
absence of the microsome, the protein should be longer, and indeed this is the case. Again, the signal peptide is not
cleaved if the protein does not enter the ER. Based on this information, all other answers can be dismissed. The
correct choice is B.

J rpyright @ by The Berkeley Review a7 The Berkeley Review


Specializing in MCAT Preparation
Biology Structure and Function in Qells and Viruses Section W Answers

9. B is correct. We are told from the passage that the SRP binds to the signal peptide and is ultimately responsible for
bringing the protein to the ER membrane. Let us think about what would happen if the SRP was no longer there.
Would *. t." an increase in the frequency of the secreted proteins? No, we would see a decrease since the proteins
that are being translated cannot make their way to the ER. This eliminates choice D. Would we see a loss in the
signal peptid"Z No, nothing would happen to this sequence on the protein except that it would no longer bind to the
Snp U"cause there is not one available. Therefore, we can choice C. Would we see a quicker translational arrest?
Assuming there is no SRP around, we would not see any translational anest because binding of the SRP to the signal
peptide ii responsible for the arrest. We can eliminate choice A. We would see the elimination of the ability to
import proteins into the ER. The correct choice is B.

10. A is correct. By having a translational arrest, the cell is ensuring that a protein that is needed on the outside of the
cell (or some organellelnside the cell) will not be released into the cytosol. If the protein is no longer being made on
the ribosome, this is a solid means of insurance. Consider the other choices. There is no evidence that the ribosome
is non-functional. It will translate the protein just fine, but its function is regulated by the SRP. Therefore, we can
eliminate choice B. The notion that all translation must occur on the ER membrane is ridiculous. This is obviously
not the case for all of those proteins that carry out their function within the cytosol. We can eliminate choice C.
There is no evidence in the- passage or our body of knowledge that such a translational arrest is a means of
proofreading. Therefore, the most likely reason lies in the idea of ensuring proteins that are destined to be secreted
are not inappropriately released into the cytosol. The correct choice is A.

ll. B is correct. The Golgi stack has two distinct faces: a cis face (or entry face) and a trans face (or exit face). The cis
face is closely associated with the transitional elements of the ER while the trans face is distended into a tubular
reticulum known as the trans Golgi network. Proteins and lipids enter a Golgi stack in small vesicles from the ER on
the cis side and exit for various destinations in vesicles from the trans side of the Golgi. Based on this information,
one can easily eliminate choice D. The medial region of the Golgi is the region in between the cis and the trans face.
Therefore, on" can easily eliminate choice C. There is no lateral region of the Golgi complex, so this choice can
easily be eliminated. The correct choice is B.

12. C is correct. Collagen is an animal connective tissue protein. It is at the bottom of the list in the table. It does not
meet the ideal protein requirements. Statement I is not true. Soybeans do meet the ideal protein requirements, as
seen in the table. Statement II is true. Lentils do not meet the ideal protein requirements for sulfur-containing amino
acids. Statement III is true. The correct choice is C.

13. A is correct. The passage indicates that beans are usually low in the sulfur-containing amino acids. Methionine is an
amino acid that containJ sulfur. Choice A is correct. Neither choices B, C, nor D contain sulfur. The correct choice
is A.

14. C is correct. Choice A is phenylalanine. Choice B is tryptophan. Choice C is lysine. Choice D is histidine. The
correct choice is C.
15. D is correct. We can read this answer from the table. Compare the values for each protein food to the ideal standard
given at the top. Collagen is deficient in all five of the listed amino acids. Choice A is incorrect. Soybeans have
idequate levels of the l]sted amino acids. Choice B is incorrect. Cornmeal has 3 limiting amino acids. Choice C is
incorrect. Lentils have2limiting amino acids (count the sulfur-containing amino acids as one choice, since only
methionine is actually needed by the body). The correct choice is D.

16. B is correct. First, compare the amino acid pattern to the ideal. This is not an ideal protein, so it is probably not fish-
which is a "complete" piotein. Choice D is incorrect. Beans, as we learned in the passage, are usually low in sulfur-
containing aminb acidi. The unknown is not deficient in these, so choice A is incorrect. Milk also meets the ideal
protein st-andards, not the pattern shown. Choice C is incorrect. Grains are low in lysine, as is the unknown. The
bther values are similar to cornmeal and oatmeal, both of which are grains. The correct choice is B.

17. A is correct. The opposite of essential is nonessential. In the passage, valine, isoleucine, and histidine are indicated
as essential. This means phenylalanine is nonessential. The correct choice is A.

Copyright @ by The Berkeley Review a8 The Berkeley Reviev


Specializing in MCAT Preparation
Biology Structure and Function in Cells and Viruses Section VI Answers

18. C is correct. The Gram stain is commonly used in microbiology. It involves staining samples with a purple dye,
such as crystal violet, using iodine as a mordant, and then staining with a pinkdye, safranin.-The differen."i in..tt
wall structure allow characterization of Gram-positive and Gram-negative bacteiia. This gives the clinician a broad
approach for treatment. Not all bacteria would grow under anaerobic conditions. Choice A is incorrect. An
immunoassay binds specific molecules, but would not be used to classify Gram-negative or positive. Choice B is
incorrect. HPLC is used to separate compounds based on their molecular characteristics. Bactbria are not separated
this way. Choice D is incorrect. The correct choice is C.

19. D is correct. The death of Gram-negative bacteria leads to the release of endotoxin from the bacterial cell wall. As
the antibiotic kills bacteria, they would release more endotoxin. At some point, endotoxin levels would rise and then
fall if the antibiotic were able to control the bacterial population. Choice C is incorrect. Choice B is incorrect. The
antibiotic does not bind or neutralize the endotoxin. Choice A is incorrect. Choice D is the correct choice. The
correct choice is D.
:r). B is correct. At the dosage of l0 ug, the mortality was \Vo, so all the mice in this group survived, even though TNF
was inhibited only 55Vo. Choice A is true. As E5531 levels were increased by group, the plasma TNF concentration
decreased. Choice C is true. Inhibiting TNF increased survival rates. Choice blr t.u.. The dose of LpS given
killed
all the control group. The dose was selected to do this, not to kill only 7 5vo of the control group. choice B is false,
and is the answer we want. The correct choice is B.

:1. C is conect. Neither the antibiotic nor the antagonist alone provided any benefit in terms of long term survival.
Choices A and D are incorrect. In the combined treatment, the antibiotic seemed to work together with E553 l. We
cannot make a judgment on the resistance of the bacterial strain based on this data. Choice B is incorrect. The
combination of E5531 and the antibiotic provided the only beneficial effects in terms of survival 50 hours later.
Choice C is correct. The correct choice is C.

D is correct. The purpose of the antibiotic is to decrease the number of bacteria. The purpose of the endotoxin
antagonist is to decrease unfavorable changes caused by the response of cellular
-"diuiorr to the endotoxin
produced. Blood bacterial count and endotoxin concentration should both decrease with this regime.
Choice I is
incorrect. The goal is to decrease cellular mediators such as TNF. Choice II is incorrect. This stratigy will not
work
if the bacteria are resistant to the antibiotic. The endotoxin antagonist has no direct effects on bacierial number or
reproduction. Choice III is incorrect. Choice D is the correct choice, none ofthe above. The correct choice is D.

.{ is correct. The similarity in structure between Lipid A and E553 I should suggest that E553 I can bind to the same
cellular surface receptors in the host. It is probably a competitor that does not elicit the same cellular responses.
Statement I is correct. Usually enzymes (proteins) are responsible for cleavage, not lipopolysaccharides.
Statement
II is incorrect. The release of Lipid A as part of the LPS is usually as a response io ieit damage or death (see
passage). E5531 combats the effects of release of LPS, not its release directly. Statement III ii
inconect. The
correct choice is A.

;JA, C is correct. The question is asking us to think about bacterial structure and organization with reference to
-rganelles. The cytoplasm of bacteria, unlike eukaryotes, contains no internal compartments and no organelles
:rcept ribosomes. The correct choice is C.
B is correct. The answer can be derived from a careful read of the passage. We are told that the bacteria that causes
:rs disease normally colonizes on the human throat. However, the disease begins only when the bacteria invades the
:-ood stream and eventually reaches the cerebral spinal fluid. This statement impliis that the bacteria must cross
: 'er the lining of the throat to reach the blood stream. Therefore, the layer of epithelial cells that lines that
r rrnally serves as a barrier of penetration to the blood stream. Once that barriei is broken, then the disease throat
takes
:," 1.J. The correct choice is B.

.ffiitrr,, D is correct. The concentration of myelin sheaths make tracts of nerve axons white. The rest of the CNS appears
*-.r. Within the brain, white matter is located in the inner region, whereas in the spinal cord, white matter is located
;: the exterior. This answer cannot be derived from the passage (meaning th-e question tests your personally
s-';-rired knowledge), and in fact is true of all brain and spinal cord tissue, regirdless-of whether a person-has spinal
merrngitis. The correct choice is D.

f by The Berkeley Review 89 The Berkeley Review


Specializing in MCAT Preparation
Biotogy Structure and Function in Cells and Viruses Section VI Answers

)'l B is correct. We are told from the question that this lack of complement is unlikely to be a cause of an epidemic.
We are now looking for a statement that will back this claim up. The second answer claims that in countries where
there is a high frequency of epidemics, persons with this lack of complement are not commonly found during the
epidemic. This certainly qualifies as evidence for the claim that the genetics is not a significant cause of an
epidemic. Statement A is completely the opposite, so it can easily be eliminated. The statements about the rise and
fall of antibody levels are true, and research into the cause of these fluctuations continues. However, these
statements do not offer us support for the claim made in the question. Again, our best evidence comes from the fact
that we do not see a lot of persons afflicted with this lack of complement during spinal meningitis outbreaks in
countries where epidemics are common. The correct choice is B.

28. C is correct. N. Iactamica is a relative of N. meningitidis. Therefore, it is not highly unlikely that they may be
similar in structure in some ways. Children who are infected by N. lactamica produce antibodies against this
organism. We are told that children who have had this infection are shown to be protected against spinal meningitis.
Therefore, it is logical to claim that those antibodies against N. lactamica are effective against N. meningitidis. Since
they are relatives of each other, they very well may have similar antigenic determinants (eliminate choice D) and
therefore antibodies against one may be effective against the other. There simply is no evidence in the passage or
logic to lead us to believe that choices A and B are correct. The correct choice is C.

to B is correct. One should realize from that passage that the cerebral spinal fluid is a culture medium for the rapid
growth of the bacteria. This increase in the population of the bacteria causes inflammation of the meningeal lining.
The swelling of the membrane is what causes the stiff neck. It also causes fever, headache, and potentially coma.
While this answer cannof be directly ]ifted from the passage, one should be able to relate the growth of the bacteria
to swelling of tissue. Furthermore, one may take a process of elimination to this answer.

For choice A, one has to ask themselves why the endotoxin from the bacteria affect only neck muscle. This is not
very likely. For choices C and D, one has to ask themselves how the bacteria would cause stimulation of nerve cell
innervating the neck muscle. There is no evidence in the passage for such interaction, while there is information
regarding the growth potential of bacteria in the cerebral spinal fluid. The correct choice is B.

30. C is conect. The perinuclear space is the space between the inner nuclear membrane and the outer nuclear
membrane. It is stated in the passage that the outer nuclear membrane is studded with ribosomes. This should
indicate that the membrane of the ER is continuous with the outer nuclear membrane. Therefore, the space inside the
"ER, the lumen, should be continuous with the periplasmic space. The periplasmic space cannot be continuous with
the nucleoplasm, because of the division by the inner nuclear membrane. Thus, choice A is eliminated. Furthermore-
the periplasmic space is divided from the cytosol by the outer nuclear membrane. Therefore, this eliminates choice
B. Finally, the mitochondrial matrix is in no way continuous with the periplasmic space. They are separated by
many membranes and should not be construed as continuous. The correct choice is C.

31. B is correct. The nuclear envelope ensures that translation of a particular mRNA polymer into a protein occurs in
the cytosol and not the nucleoplasm. One can gather this from the passage by considering the resting size of the pore
and size given for the ribosome. The ribosomes is clearly too large to fit into a nuclear pore, and there would not be
a nuclear import signal on the ribosome. Therefore, translation is separated and confined from the nucleus. Consider
the other answers. There is no evidence that the nuclear envelope ensures DNA replication. While nuclear envelope
breakdown is a part of the mitotic cycle, the envelope does not provide a means of insurance. We can eliminatc
choice A. Choice C is false. We know that ribosomal RNA is produced in the nucleus and the nuclear envelope harr
no known control over that process. Therefore, this answer can easily be eliminated. Finally, the nuclear enve
as stated above, breaks down during mitosis, not the interphase portion of the cell cycle. This makes choice D
incorrect. The nuclear envelope ensures that translation remains in the cytosol. The correct choice is B.

32. C is correct. First, the statement that nuclear components cannot be radiolabeled is simply not true. Think
labeling nucleotides that will eventually be incorporated into a nucleic acid polymer. Since this statement
obviously false, it can easily be eliminated. We have no evidence from the passage that nuclear components
difficult to isolate. We cannot make this assumption. Therefore, this is not the best answer, and choice B can
eliminated. Let us look at choice D, which states that nuclear components are smaller than the nuclear
implying that they would not be good measures of the size of the pore. This statement is also false. Think a
DNA and RNA polymerase. These molecules are very large and must be imported into the nucleus. Since

Copyright @ by The Berkeley Review 90 The Berkeley


Specializing in MCAT Prepara
Biology Structure and Function in Cells and Viruses Section VI Answers

statement is false, choice D can easily be eliminated. This leaves us with choice C. The nuclear pore, according to
the passage, opens up beyond its resting size to accommodate those molecules that are larger than the pore size, yet
are necessary inside the nucleus for proper function. Therefbre, it would not be a wise experimental choice (if one is
trying to measure the non-accommodating resting size of the pore) to use nuclear components. The correct choice
is C.

33. D is correct. It is clear from the question that only the tail portion entered into the nucleus. Since the tail portion is
attached to a 20 nm piece of gold, the nuclear pore must have opened up to accommodate the piece of gold.
Therefore, we can safely assume that the nuclear import signal is found on the tail piece. Since the gold attached to
the head piece did not make it into the nucleus, we can saf'ely say there is no nuclear import signal on the head
portion of the protein. Therefore, we can eliminate choices A and B. The next question is to decide the makeup of
the amino acids in the nuclear import signal. The passage tells us the signal is rich in positively charged amino acids.
The amino acids Phe and Trp are not positively charged. However, the amino acids Lys and Arg are basic, positively
charged amino acids. Therefore, they are likely candidates fbr the residues making up the nuclear in-rport signal. The
correct choice is D.

.i.{. C is correct. We are told from the passage that the nuclear import signal can be found anywhere within the protein
and still be functional. This implies that the location of the signal is unimportant. Based on this informarion alone,
we can eliminate choices B and D, because they claim the location of the signal is important. The question is telling
us that a nuclear import signal is attached to random amino acids onto an enzyme that usually finds its home in the
cytosol. In time, that enzyme should be inside the nucleus, as the protein now has a nuclear import signal. Based on
that information, we can eliminate choice A, which claims the molecule remains within the cytosol. Therefore, the
molecule ends up inside the nucleus, because the location of the nuclear import signal is unimportant. The correct
choice is C.

D is correct. The passage tells us that during mitosis, the nuclear envelope breaks down. It goes on to inform us that
after the nuclear division has taken place, the nuclear envelope begins to refbrm around the chromosomes. The
passage states that many of the previous molecules of the nucleus are exiled when the envelope reforms. However,
in time, these previous nucleus dwellers find their way back into the nucleus. It is that last sentence which is the
most important in answering this question correctly. If those molecules that were once inside the nucleus flnd their
way back to the nucleus after the envelope reforms, the nuclear import signal must not be lost on these molecules. In
other words, they must have the signal to get back into the nucleus. If these indeed are the same molecules that were
previously in the nucleus (meaning they were not resynthesized), the signal must have remained. Consider the other
answers. We cannot believe that all molecules that reside inside the nucleus are resynthesized after every nuclear
division. We have no evidence for this claim; and furthermore, it makes little sense. because it seems a terrible waste
of energy. Thus, we can eliminate choice A. Choice B can be eliminated, because it is false. The molecules from the
nucleus are not destroyed. Based on the discussion above, we can eliminate choice C, because the molecules most
likely retain their nuclear import signal. The correct choice is D.

C is correct. We are told from the question that a mcllecule that is placed in the nucleus of a frog oocyte is exported
out into the cytoplasm. Next, we are told that the same molecule, when placed directly in the cytosol, remains there.
This implies that the molecule has a nuclear export signal and it does not have a nuclear import signal. In other
uords, it can only leave the nucleus, and not reenter. Based on this information, one can easily eliminate choices A
and B. The question then becomes where is the receptor that will recognize this nuclear export signal. Will it be on
the cytoplasmic face of the nuclear membrane, or the nucleoplasmic face? The answer seems obvious. In order to
recognize the nuclear export signal, the receptor must be facing the inside of the nucleus. Therefore, the receptor
must be on the nucleoplasmic face of the nuclear membrane. The correct choice is C.

B is correct. This question is designed to test your basic knowledge of the bacterium Escherichia coli (abbreviated
t> E. coli). This prokaryotic organism is rod-like (bacilli), with a length of about 2 prm and a diameter of about 1

lm. Bacteria are classified as either being Gram-positive or Gram-negative. If a bacterium takes up the Gram stain
a crystal violet dye and iodine), then it is said to be Gram-positive. If it does not take up the Gram stain, it is Gram-
:-gative. Surrounding the cytoplasm of a prokaryotic cell is the plasma membrane. Surrounding the plasma
rrembrane is a peptidoglycan (murein or cell wall) layer which consists of covalently linked polysaccharide and
:.rl)'peptide chains.

; ) by The Berkeley Review 9l The Berkeley Review


,mr
Specializing in MCAT Preparation
Biology Structure and Function in Cells and Viruses Section VI Answers

Extracellular Face

Peptidoglycan f
(cellwall) Periplasmic
1 gel
Plasma I
memb.ane f
Cytoplasmic Face
Gram-positive bacterial wall Gram-negative bacterial wall
(e.g., Staphylococcus aureus) (e.g., Escherichia coli)

In the Gram-positive bacterium the peptidoglycan layer is quite thick (about 250 A) while in the Gram-negative
layer it is rather thin (about :041. fnis is where the basic similarities between the two types of bacteria end
Surrounding the peptidoglycan layer of Gram-negative bacteria is an outer membrane. The space between the
plasma membrane and the outer membrane of a Gram-negative bacterium is often called the periplasmic space (or"
more correctly, the periplasmic gel). The correct choice is B.

38. D is correct. Al1 hydrolysis reactions are favorable. Hydrolysis of this phosphatidylethanolamine will give palmiuc
acid, oleic acid, phosphate, ethanolamine, and glycerol. These individual structures are shown below. As we uill
learn later, each of these components will become important in metabolism. The correct choice is D.

o o
il ll
HrC- O- C- (CH2)r4CHj H)C- OH HO- C- (CH2)|4CH1 Palmitic acid

HC-
lft O
- C-
(CH2h
HH
tt
- C: C- (CH2)7CH3
4 H.O
l
HC- OH HO- C-
o
il
(CHz)r
HH
tl
- C: C- (CHzhCH3 Oleic acid
lo o
Itt
HrC- O- P- O-
o I

H2C- OH HO_ P_
il
HO-
@
CH2CH2NH3
"l CH2CH2NH-,
I
OH

o6 Ethanol oq Ethanolamine

A phosphatidylethanolamine Phosphate

39. C is correct. In bacteria, membrane lipids are synthesized primarily by integral membrane proteins. The question m
whether that synthesis occurs on the extracellular face (outside) or cytoplasmic face (inside) of the membrane. As u
membrane lipid is being synthesized it begins to incorporate molecules like glycerol, fatty acids, ethanolamine, anC
phosphate. In this case the phosphate which is incorporated into the membrane lipids is radioactively labeled.

Extracellular Face

Q rNnS complexed with


membrane lipids that
contain PE
,o*ion of f
rinio ulaverl

Q Rudiou"tively labeled
membrane lipid
Cytoplasmic Face

If TNBS is added to the cell suspension, we would expect it to bind to PE on the extracellular surface as well as
PE on the cytoplasmic face. If membrane lipids like PE were synthesized on the extracellular face, some of t

would be expected to be radioactively labeled and some would bind TNBS. This was not observed. If TNBS c
cross the cell's plasma membrane, it should be able to bind to radioactively labeled PE which was being syn
on the cytoplasmic face. Since it was observed that none of the PE molecules which were radioactively labeled '*
labeled with TNBS, it must mean that TNBS cannol cross the cell's plasma membrane and that the membrane lin

Copyright @ by The Berkeley Review 92 The Berkeley Re


Specializing in MCAT
Biology Structure and Function in Cells and Viruses Section VI Answers

are being synthesized on the cytoplasmic face. TNBS cannot cross the membrane because is bears a negative charge.
Remember, membrane lipids have head groups which are polar and part of that polarity is due to the negatively
charged phosphate groups. Like charges repel one another. The correct choice is C.

40. B is correct. First, go back and read the previous answer. TNBS is still negatively charged when it is added to the
bacterial suspension. Therefore, it still cannot cross the bacterial membrane. This will eliminate choice A.
Membrane lipids like PE are synthesized on the cytoplasmic face, not the extracellular face. This eliminates choice
C and D. This leaves us (by default) with choice B.

Extracellular Face

$ fNas complexed with


membrane lipids that
contain radioactively
labeled PE
dilTil:.{
Q Rudiou.tively labeled
Cytoplasmic Face membrane lipid

As outlined in the passage, phospholipids in membrane bilayers will take days to undergo a flip-flop (i.e., transverse
diffusion). Just because the phospholipids take days to flip-flop does not mean that membrane lipids like PE cannot
flip-flop in a shorter period of time. A relevant diagram for Experiment 2 is shown below. The correct choice is B.

11. B is correct. The passage states that integral proteins are tightly bound to membranes by hydrophobic forces. Those
hydrophobic forces arise from the interaction of the non-polar interior of the lipid bilayer and the hydrophobic
amino acid residues in that portion of the protein which is in contact with that non-polar interior. The only
hydrophilic forces of interest between the integral protein and the membrane will occur at the polar regions of the
lipid molecules. These forces, if they exist, are not as great as the interior hydrophobic forces (i.e., the exclusion of
water). Hydrogen bonds are mainly electrostatic interactions, and they occur in the limited polar region of the
membrane. Choice A and D are eliminated as possibilities, and choice C becomes a good candidate for the answer.

Outside

However, let's consider choice B. As stated in the passage, both the lipids and the proteins found in a membrane are
asymmetrically distributed. This means that an integral protein is different on one side of the membrane than it is on
the other side of the membrane. For example, the C-terminal region of a protein can be found on the inside of a cell,
rvhile the N-terminal region can be found on the outside. The N-terminal region on the outside of the membrane
might have more amino acids associated with it while the C-terminal region might have fewer amino acids. Also,
carbohydrate residues can be found attached to the N-terminal domain (on the outside of the cell).

It turns out that it is the asymmetry of the integral protein that makes it so difficult for transverse diffusion to occur.
The hydrophilic domains of the integral protein not associated with the interior of the membrane have a difficult
time rotating and trying to pass through a hydrophobic interior. If transverse diffusion were to begin, the
hydrophobic portion of the protein and the hydrophobic portion of the membrane would still (for the most part)
remain in contact with one another. The correct choice is B.

irr",*:-:t O by The Berkeley Review 93 The Berkeley Review


Specializing in MCAT Preparation
Biology Structure and Function in Cells and Viruses Section VI Answers

42. C is correct. The question asks about the fluidity in the membrane of a bacterial cell, not a eukaryotic cell.
According to the pasiage, the bacterium Escherichia coli contains 0% cholesterol in its membrane. Since cholesterol
is not present, choices A and D as answers. A decrease in the membrane's fluidity means that the
"ii.ninit"stiffer. If there were long-chain fatty acids with cis double bonds in the membrane, this
membrane is becoming
would introduce a kinl in the fatty acid and not allow for close packing of the hydrocarbon tails. The membrane
tends to be a bit more fluid. Eliminate choice B. If we increase the length of the hydrocarbon tail of a saturated fatty
acid, this would allow for more hydrophobic interactions between the individual methylene (-CHz-) groups. The
more hydrophobic interactions, the tighter the packing of the hydrocarbon tails, and the less fluid the membrane-
The correct choice is C.

43. D is correct. Spermaceti has two long hydrocarbon tails, which can pack together quite well. As mentioned in the
question, this wix melts between 42-41 "F. It tends to be more of a fluid than a solid above this temperature range-
As the sperm whale dives to great depths, the temperature of the water begins to drop to the low 30' F range. Cold
sea watei is circulated through special chambers in the whale's head. Since the cold water has a temperature below
the melting temperature of the wix, the wax begins to solidify. As the wax goes from a fluid state to a solid state, its
volume de-creasis. A decrease in volume is related to an increase in density. Recall that the density is the ratio of an
object's mass to its volume. As the whale's density increases, it is able to stay at these great depths for longer periods
of time without expending considerable energy.

In other words, by controlling the shape of the wax the whale controls its own buoyancy. This is analogous to thc
weights that a scuba diver uses on her weight belt as she descends into the water. When the whale ascends, the
temperature of the cold water in the chambers increases because of the increased circulation of warm blood to the
head. The correct choice is D.

44. A is correct. The passage states the amino acid residue is modified after the protein is synthesized. Only post-
translation indicates synlhesis is complete. Transcription is the production of a template m-RNA for proteir
synthesis. Choice B is incorrect. Replication is the duplication of DNA in the nucleus. Choice C is incorrecl
Iiegradation means the protein is destroyed. Choice D is incorrect. The correct choice is A.

45. D is correct. Choice A is hydroxylysine. Choice B is lysine. Choice C is hydroxyproline. Choice D is proline.
correct choice is D.

46. D is correct. The nonstandard amino acids are ones not specified by the standard tRNA codon table. They
modified versions of the standard amino acids (modified after translation). Therefore, there are no tRNA c
corresponding to the nonstandard amino acids. The correct choice is D.

47. B is correct. The box marked II indicates the carbamoyl moiety. Look at the molecule carbamoyl phosphate:
C=Ol-phosphate. The carbamoyl moiety is in brackets. That's your answer. The correct choice is B.

48. B is correct. From the passage, we learned that hydroxyproline is formed by modification of a synthesized 1

Thus, proline is incoiporaGd and then modified to hydroxyproline after the peptide is made. This
hydroxyproline itself would not be incorporated. Choices A and C are incorrect. Proline is not further broken
in the digestive ffact, so choice D is incorrect. The correct choice is B.

49. A is correct. The liver contains the enzymes of the urea cycle and produces urea from excess amino
Although urea passes through the kidney on its way to excretion in the urine, the kidney does not make urea
B is inJorrect. Neither the gill bladder or the pancreas has a role in urea biosynthesis. Choices C and D are i
The correct choice is A.

50. D is correct. Collagen would not be synthesized properly if hydroxyproline were not available due to impair
activity of prolyl hydroxylase. Collagen is a major component of connective tissue and plays an important role
blood vessel str.ngth, healing of wounds, and healthy gum tissue. All the symptoms listed are symptoms of
The correct choice is D.

Copyright @ by The BerkeleY Review 94 The Berkeley


Specializing in MCAT Pre
Biology Structure and Function in Cells and Viruses Section VI Answers

D is correct. This relationship is quite important and stems from the fact that it is often necessary to express the
concentrations of hydrogen and hydroxide ions in an aqueous solution. These concentrations can be quite awkward
to work with (e.g., 10 M to l0-14 M) and so a common logarithmic notation (based on logro) is used for simplicity.
One of the rules of logarithms states that for a given quantity X,

pX=log!=-logX
X
This allows the [Ho] to be expressed as,
pH = log
' " -f = - log [H*
[H*l
or even the [OHe] as,
pOH-log-l =-loglOH-l
IOH ]

Natural logarithms have as their base e = 2.'l 18 and are exponents to which e must be raised to give a number. These
are quite different from common logarithms. The correct choice is D.

it D is correct. Physiological pH is considered to be 7.4. All of the common 20 amino acids at physiological pH will
have their s-amino group in the protonated state and their cr-carboxyl group in the ionized (deprotonated) state. This
is because the average pKn of the c-amino group is about 9.5 and the average pIQ of the cr-carboxyl group is about
2.2. At physiological pH many of these amino acids will be in their zwitterionic form (where the net charge on the
amino acid adds to zero). Since each amino acid will bear at least two charges at physiological pH, they cannot be
nonpolar and they cannot be monovalent. The correct choice is D.

53. B is correct. Since all of these amino acids will be found near the center of a long polypeptide chain, their s-amino
group and a-carboxyl group will be tied up in a peptide linkage. The only protons that are able to ionize are those on
amino acids with ionizable side chains. There are only 7 amino acids with ionizable side chains, and three of them
are listed as answers. The one amino acid which does not have an ionizable side chain is isoleucine. The correct
choice is B.

-i.1. C is correct. The two pKa values for leucine were given in the passage. Leucine's cr-amino group has a pKa of 9.6
while its c,-carboxyl group has a pKa of 2.4. The pI can be calculated as follows:

ot=W=9Lr2a=6l
Note that leucine will carry no net electric charge at its pI. In the case of leucine this is its zwitterionic form (it is a
dipolar ion). The correct choice is C.

i5. D is correct. The only difference between phenylalanine and tyrosine is the presence of a hydroxyl (OH) group on
the ring.

3 -L:lli

A carboxylation reaction would have added a CO2 group. A hydrolysis reaction would have used water to break
(iyse) a molecule. The elements of water would have also been added to the products. A hydration reaction involves

t|eu ': .,ght O by The Berkeley Review 95 The Berkeley Keview


toll Specializing in MCAT Preparation
Biology Structure and Ftrnction in Cells and Viruses Section VI Answers B

the addition of water to a molecule without lysing that molecule. Since we do not see any of these three types of
reactions in the conversion of phenylalanine to tyrosine, it must be a hydroxylation reaction. The correct choice is
D.

56. D is correct. This means that certain amino acids within a protein can have side chains with dissociable hydrogen
atoms. The pKn's of these amino acid side chains can range from about 3.9 (aspartic acid) to about 12.5 (arginine).
This is quite a wide range with which a protein can donate or accept hydrogen ions (i.e., act as a buffer).
61
Proteins can form secondary and tertiary structure through internal hydrogen bonding interactions. This occurs
through amino acid interaction within the polypeptide chain itself. Once a protein is in its tertiary form it will stay in
that form until it is denatured. Protons in the medium surrounding the protein are (for the most part) not involved in
the hydrogen bonding that dictates the secondary and tertiary structure of a protein. If a protein were denatured (by
hydrolysis), the element of water would add across the peptide bond being broken. Once this happened the ability of 62
protons to bind to a free terminus would be diminished. However, protons can add to the N-terminus and the C-
terminus of a protein. This action only absorbs (at most) two protons. Many more protons could be absorbed by
amino acids that have ionizable side chains. The correct choice is D.

57. C is correct. The key to answering this question is to locate all of the amide bonds (see the dots . in the structure
below). Each amide bond will require I molecule of water. After hydrolysis we will get 4 compounds. They are
glutamic acid (Glu), histidine (His), proline (Pro), and ammonia (NH:).

pyro-Glu His pro

HHOHOHO
ot r il. r lla I lla
H- N- C- C- N_ C- C-N - C- C- NH"
t. \ I I

o=c.,./ " Fn,


o /-\
I

u- N\zN -H

Note that glutamic acid in the structure is referred to as pyro-Glu. This is because its side chain has condensed with
its own o-amino group to form a ring structure involving an internal amide bond. Hydrolysis of this bond simply
exposes the glutamic acid side chain. The correct choice is C.

58. A is correct. Hydrolysis of the internal amide bond in pyro-Glu and the terminal amide bond of Pro will allow us to
see the tripeptide in its "true" linear form.

N-teminus C-terminus
HHOHO o
or r il r rl I il
H- N_ C- C- N_ C- C_ N- C- c- oH + NH3
Ii l" i f", (9 IV

. l=
f"' H-N\zN-H
"o."_ro
The N-terminus is the cr-amino group of Glu. The correct choice is A.
EE
ffi. I
I
59. B is correct. To answer this question, one must be aware of the following idea: A drop in temperature will cause t
bilayer to become more viscous. The increased viscosity will cause a decrease in the fluidity of the membrane. Sincc I
the proteins are embedded inside the lipid bilayer, the time it takes to react the state in Figure 2 should increase
to the decreased fluidity of the membrane. In fact, organisms such as bacteria and yeast will change the compositi B
of the lipids making up their biological membrane in responses to drops in temperature. Based on this informati T
all other answers are easily eliminated. The correct choice is B. t

Copyright @ by The Berkeley Review 96 The Berkeley


Specializing in MCAT
Biology Structure and Function in eelts and Viruses Section VI Answers

60. C is correct' From the passage' we known that monovalent antibodies are used
in this process. Normally, antibodies
are bivalent, that is, they have two arms, each of which is capable of binding
t" i"iig""ic determinant. In our
immune system response' such anatomy is beneficial because antibodies ""
cari cross-link"antigen molecules into a
large lattice as long as.each antigen molecule has three or more antigenic,determinants
However, in this experiment, one can imagine^that_cross-linking between fluorescent i*hi"r, they usually do).
antibodies can greatly affect
the movement of proteins and thus the resulis of the lateral diffustn rates. The
correct choice is C.
61. B is correct. At t = 0 minutes, the labeled antibodies of the mouse and human are
on their own halves of the hybrid
cell' At t = 40 minutes' it is clear that the antibodies are mixed. The only way the
-i*lng .ouio occur would be if the
I
lipid molecules readily exchange places with their neighbors within a monollyer. This riovement
I is known as lateral
diffusion, and this experiment provides evidence for such a process. The correct choice
is B.
f 62. D is correct. This answer requires one to think about the experiment and interpret
the graph. The graph shows that
the asymptotic level of fluorescence reached is lower than ihe original level
of fluore"scerrc". why could this be?
Look for the obvious. In the experiment, we use a laser beam to bleach some
of the fluorescent ligand. We will not
be able to recover our original amount of fluorescent because we have permanently
bleached the ligand. we can
assume that the bleaching is permanent and not temporary on two accounts.
There ii no evidence from the passage
that demonstrates that the bleaching is only temporiry. Furthermore, if the
bleachi"g i; ;"i; remporary, over time
we should see the level recover to its original levei. we do not see this in the
asymptotic' Therefore, we can eliminate choice c. There is no evidence for lraph, a, ti,e level reached is
choices i utio b in either the passage or
our body of knowledge. The correct choice is D.

63. D is correct' This question requires graphical interpretation. The longer it takes
for the recovery process to occur,
the smaller the diffusion rate. Since all of the experiments *"." .u.ii"d out in
simila. .onditionr, we are really
looking a diffusion coefficien-ts. The greater thq rate of recovery, the greater the
diffusion coefricient of the
membrane glycoprotein. Therefore, it becomes clear that it takes glycoproteln
D the longest u*oun, of time to reach
the full rate of recovery. Therefore, glycoprotein D must have th-eiowest
diffusion rate and thus smallest diffusion
coefficient. The correct choice is D.

64. D is-correct. The question requires us to think about several things. First, there is
no evidence that the antibodies
used are lighter in the in vitro system. Furthermore, we have no-evidence
irh for a correlation between weight and
diffusion rates. Since we do not have this information, we cannot make the assumption.
Pl). choice A' Next, we are not told of any switching to a bivalent antibody. Therefore,
it.r.ior", we can eliminate
we have no reason to believe that
cross-linking would be occurring. we can eliminate choice B. So ntw we are left
to think about oligosaccharide
interaction on the surface of cell membranes. First, the oligosaccharides are
found on the extracellular face, and not
the intracellular face, of the plasma membrane. This alonJ eliminates choice
c. Let us finish the conclusion. with
only one glycoprotein, there is no interaction.betwe.en^ these bulky oligosacchario" groufr
as the proteins are
difTusing laterally. The less interaction leads to higher diffusion rates. The
iorrect choicJis D.
65. C is correct' The question requires us to know a little about cholesterol and its role in
our biological membranes. At
the high concentrations that cholesterol is found in eukaryotic cells, it acts to
increase the fluidity of the membrane.
while this may seem counter-intuitive, it is indeed the case. Therefore, one can easily eliminate
choices A and B.
Now the question becomes how does cholesterol increase the fluidity of the membrane.
well, it prevents the
hydrocarbon chains making.up the lipids from crystallizing by preveniing their interaction.
One can think of the
bulky cholesterol as getting in the way with its large, hydrophobic, classic"steroid rings. Therefbre,
cholesterol acts
to increase the fluidity of the membrane by inhibiting hydroc-arbon chain crystallization.
The correct choice is C.

t
se the
66' B is correct. This question is not based on any information in the passage, but rather tests you
might have learned in some of your science. courses. Many bacteria hav-e rather distinctive
though it would be rather difficult to associate all the species of-bacteria *itn
worth your while to remember at least one or two of the ciassic prokaryotic cells
on information you
shapes and sizer Even
u !iu.n characieristic, it mignt ue
Since and"their characteristi" ,t up.sl
edlr
Bacteria shaped like spheres are referred to as cocci. The best known examples comes
rsitiom from the genus Streptococcus.
The two species of streptococci that cause most of the streptococcal diseases associated
,ation- with humans are
Streptococcus pneumoniae (causes pneumonia) and, Streptococcus pyogenes (causes impetigo, a superficial skin

- opyright @ by The Berkeley Review 97 The Berkeley Review


Specializing in MCAT preparation
Biology Structure and Function in Cells and Viruses Section VI Answers I
infection commonly found in children). Bacteria shaped like cylinders are referred to as rods (also called bacilli).
The best known example comes from the genus Escherichia and has the species name Escherichia coli (which can
cause bladder and kidney infections). Bacteria shaped like spirals come in different sizes as well. There are those
bacteria which are short and twisted into a rigid spiral, and there are those bacteria which are elongated and twisted
into a flexible spiral helix. Representatives of these two forms of bacteria are not as commonly mentioned as the
ones above, but we will give two examples for completeness. A typical spiral bacterium comes from the genus
Spirillum. The species Spirillum volutans is one of the largest bacteria known. One of the better known bacteria that
represents a spiral helix comes from the spirochete genus,Treponema.The species Treponema pallidum causes
syphilis in humans. The correct choice is B.

67. A is correct. Invariant simply means that something (in this case a characteristic) is unchanging or constant.
Another way to look at this question is to ask yourself which are the most important features of a prokaryotic cell.
Certainly the nuclear region which contains the DNA is important. If new proteins need to be made to help in
cellular function, then the DNA must be transcribed into mRNA. This message must then be translated at the level
of the ribosome into a particular protein. Variation in either the nuclear region or the ribosomes could be deadly to
the cell. The same can be said about the plasma membrane. This is the membrane that prevents the cytoplasm from
leaving the interior of the cell. Lysis of the plasma membrane leads to immediate destruction of the cell.

The cell wall is the only structure mentioned which can be invariant. If the cell wall were to be removed from a
given prokaryotic cell, that cell would form a protoplast (see the passage). These cellular structures are still capable
of carrying out metabolic processes given the right environment. In fact, one genus of bacteria (Mycoplasma) lacks a
cell wall entirely, simply because it cannot synthesize the precursors needed for the formation of the peptidoglycan
layer. The correct choice is A.

68. D is correct. Recall that Gram-positive bacteria do not have an outer membrane, while Gram-negative bacteria do.
Also, the peptidoglycan layer in Gram-positive bacteria is much thicker than the peptidoglycan layer in Gram-
negative bacteria. In Step I both types of cells are treated with crystal violet. This procedure will stain both cell
types purple. In Step 2 iodine is added to fix the crystal violet in the cells. A complex of crystal violet and iodine is
formed. Again, both cell types will remain purple. In Step 3 alcohol is added and acts to decolorize the cells.
However, only the Gram-negative cells are decolorized by the alcohol. The Gram-positive cells remain purple. This
is due to the thickness of the peptidoglycan layer of the Gram-positive cells. Alcohol tends to dehydrate this layer.
thus making any pores within the layer itself rather small. These small pores hinder the passage of the crystal violet-
iodine complex during the extraction process. Since the crystal violet-iodine complex remains trapped in the
peptidoglycan layer, Gram-positive cells still display a violet color at this stage. In Gram-negative bacteria the thin
peptidoglycan layer does not significantly hinder the extraction process, and these cells therefore display no color
(i.e., they are colorless) at this stage. In Step 4 a counterstain (safranin) is added to the suspension. This red-colored
stain is added so the Gram-negative bacterial cells can be visualized. They now display a red color. The purple
colored Gram-positive bacteria also pick up the red stain and now appear blue. The correct choice is D.

69. D is correct. In Gram-positive bacteria the peptidoglycan layer is rather thick, while in Gram-negative bacteria the
peptidoglycan layer is rather thin. The Gram-staining procedure takes advantage of these different characteristics. It
is important to understand that the designations "Gram-positive" and Gram-negative" refer to the fact that a
particular bacterial cell can resist decolorization or be decolorized, respectively.

The technique employed by the Gram procedure is an entirely different matter. In order for a cell to be clearly seen
under a light microscope, organic dyes are used to stain cells. This technique is referred to as positive staining.The
dyes used can either be cations (positively charged) or anions (negatively charged). Crystal violet and safranin ara
both examples of dyes which are cations. One of the reasons that these two dyes work rather well in staining cells i-r
because the membranes ofcells show a high degree ofnegative charge.

Negative staining allows cells to be visualized in an outline form. This technique calls for the staining of the cell s
background while leaving the cell itself stain-free.

Dffirential staining is a procedure that does not stain bacterial cells equally.E,ither positive or negative stains are
used in this procedure. Therefore, we find that the Gram staining procedure is both a positive staining procedure anC
a differential staining procedure. The correct choice is D.

Copyright @ by The Berkeley Review 9a The Berkeley Revier


Specializing in MCAT Preparatior
Biology Structure and Function in Cells and Viruses Section VI Answers

70. B is correct. Recall that the enzyme transpeptidase links amino acids together between adjacent glycan chains. This
linkage helps to stabilize the fbrming peptidoglycan layer. If the enzyme were found near Face 1, it would not be
close to the peptidoglycan layer and would therefore be unable to catalyze the cross-linkage between adjacent amino
acids of the glycan chains. We can rule out choice A in both types of bacteria. We can immediately rule out two
other choices if we recall that Gram-positive bacteria do not have an outer membrane, and hence do not have
membrane Face 3 or membrane Face 4. Remember, the question asks for location of the enzyme in both Gram-
positive and Gram-negative bacteria. Choices C and D are eliminated. By default, this leaves choice B. Notice that if
the enzyme were near membrane Face B, it could easily catalyze the joining of amino acids in the peptidoglycan
layer. The correct choice is B.

7I. A is correct. In order for bacterial cell growth to occur, the peptidoglycan layer must first be opened up by the
action of autolysins. Specific sugar derivatives and amino acids are then added to the peptidoglycan layer as the cell
begins to grow. The strength in the peptidoglycan layer comes fiom the cross-linking of peptide bonds between
individual glycan chains by enzymes called transpeptidases.

If these enzymes are inhibited (by penicillin), then cross-linking can not be completed and the peptidoglycan layer
remains weak. The cell becomes susceptible to lysis. If the autolysins were inhibited, the peptidoglycan layer would
not be able to grow and therefore the cell would stop growing. Since the peptidoglycan layer would not be opened
up, the action of the transpeptidases would not be needed. Penicillin is not able to induce lysis in non-growing cells,
because inhibition of the transpeptidase enzyme would have no effect.

Increasing the growth rate of the cells would mean that penicillin could interact with the transpeptidases and inhibit
the action of this enzyme. Even if the growth rate were to increase, the growing cells would still lyse.

Under normal conditions, the solute concentration within a bacterial cell is much higher than the solute
concentration outside a bacterial cell. Water wants to flow down its concentration gradient and into the cell, causing
the cell to swell and eventually lyse. Lysis by penicillin can also be prevented by adding a solute such as sucrose to
the medium in which the cell resides. If the solute concentration inside the cell balances the solute concentration
outside the cell, then structures called protoplasts form when the cell wall of the bacterium becomes too weak.
Protoplasts are (spherical) cellular structures that do not have a cell wall. They have lost the ability to retain it.

However, theses structures still have their plasma membrane and all of their intracellular components. Inhibition of
protoplast formation wouid mean that the solute concentrations across the bacterial cell membrane(s) are not the
same. If penicillin is administered, and the bacterial cells cannot form protoplasts, they will lyse.

Decreasing the solute concentration outside the cell sets up an even larger solute concentration gradient between the
inside and the outside of the cell. More water will want to flow into the cell. If this happens, the cell swells and
eventually lyses. The correct choice is A.

C is correct. First, look at Figure l. Note that there is an outer membrane (with a pore), a peptidoglycan layer, and
then a plasma membrane. The action of both penicillin and lysozyme is described in the last paragraph of the
passage. Both work on the peptidoglycan layer. In order to work on the peptidoglycan layer, they both must have
3ccess to it. The only types of cells that will allow for such access are Gram-positive cells. Gram-negative cells have
that extra outer membrane and tend to be (more) resistant to penicillin and lysozyme.

Choice A states that the cell is nonresistant to penicillin, because the antibiotic can cross the outer membrane.
Penicillin is a large molecule and would have a difficult time crossing the lipid bilayer of the outer membrane. What
,rbout passage through the pore? Again, penicillin would be too large to pass through the pore. The same is true for
.-r sozyme. Enzymes are generally quite large (much larger than penicillin). If penicillin cannot pass through the
:cre, either can lysozyme. This allows us to eliminate choice A and choice D. Even though choice B is a true
:ietement, there is a better answer. Do not just sequentially read down the answers until you find one that is correct.
) ou oe€d to flnd the best answer. The correct choice is C.

B is correct. As shown by the molecular structure, rafTinose is clearly not a disaccharide. It is considered to be an
..gosaccharide (or even a polysaccharide). Since there are no amino acids attached to and of the three sugar
--f1dues, it cannot be a glycoprotein. This means that raffinose is either a reducing sugar or a non-reducing sugar.

m . by The Berkeley Review 99 The Berkeley Review


ffMl Specializing in MCAT Preparation
Biology Structure and Function in Cells and Viruses Section VI Answers

Sugars having anomeric carbon atoms that have not formed glycosides (containing an acetal linkage) are called
reducing sugars. Recall that the cyclic and linear forms ofboth aldoses and ketoses are readily interconverted. The
aldehyde function in galactose and glucose can easily be oxidized by an oxidizing agent like Benedict's reagent (a
solution ofcopper(Il) sulfate and sodium citrate in aqueous base) to the corresponding carboxylic acid (see below).

cHo
I
H-C-OH "ooo
I
H- C- OH
I
HO_ C- H
I
+ 2cl2+ + 5 oHo
I
HO-C-H + Cu2Olry + 3H2O
I
H-C-OH Benedict's ' H_C-OH
I I
red
H- C- OH reagent H- C- OH precipitate
I
(blue) I
cH2-oH cH2-oH

D-Glucose D-Gluconate
(open chain)

Fructose is an a-hydroxy ketone and a-hydroxy ketones are easily oxidized to the diketone by Benedict's reagent.
The Benedict's reagent, which is a blue solution, is reduced to a red precipitate. If the ;ldehyde and ke-tone
functional groups remain tied up in a glycosidic bond (as shown in the structure ofraffinose), then they cannot react
with the Benedict's reagent. The correct choice is B.
74. C is correct. Sugars with an aldehyde function are referred to as aldoses, while those with ketone functions are
referred to as ketoses. The open chain form of glucose can undergo intramolecular hemiacetal formation. In this
case, the hydroxyl oxygen of the C-5 carbon attacks the carbonyl carbon of the aldehyde to form the stable
hemiacetal. This hemiacetal, which forms a six-membered ring, is referred to as a pyrinose. With this much
information, we can eliminate choices A and D. The open chain form of fructose cin undergo intramolecular
hemiketal formation to produce a five-membered ring referred to as a furanose. The hydroxyl oxygen of the C-6
carbon attacks the carbonyl carbon ofthe ketone to give the hemiketal. The correct choice is i.
't5. B is correct. This answer depends on how you view the linkage between glucose and fructose. Locate the anomeric
carbons of each sugar residue. The anomeric carbon for glucose is at the C-l position while the anomeric carbon fm
fructose is at the C-2 position. If we are looking at this linkage from the point of view of glucose, then the linkage
will be 1-+2. Is this linkage in the a or the p position? The bond stemming from the anomiric carbon of glucose-is
pointing down, below the plane of the Haworth projection. This represents the c-configuration. Therefore, we catr
say, that from the point of view of glucose, the bond is in the cr(l-+2) configurarion. But what if we look at the bond
from the point of view of fructose? In this case, the linkage is 2-+1. Since the bond at the anomeric carbon is
pointing up, above the plane of the Haworth projection, it is in the p-configuration. Therefore, the 6x(l-+21
configuration by itself is not good enough. We also need to consider the p(2-+1) configuration as well. The correct
choice is B.

76. D is correcL First, you must find the anomeric carbon of each sugar. The anomeric carbons of galactose and glucosc
are both at the C- I position. The bond leading form the C- I position of galactose is connecring with the C-6 positim
of glucose. Note that the C-l bond of glucose is connecting with fructose. This allows us to eliminate choicei A ard
B. Now, is the l-+6 linkage in the a or the B position? In order to determine this we need to consider the
carbon that links the two molecules. That anomeric carbon belongs to galactose. Note that the bond stemming fn
the anomeric carbon of galactose is pointing down, below the plane of the Haworth projection. This represenithe
configuration. If the bond were pointing up, it would represent the p-configuration. The correct choice is D.

77. B is correct. Recall that when sugars differ from one another about one carbon atom they are referred to as epimr
The only place where the carbon atoms of galactose and glucose differ from one another is at the C-4 position.
galactose the hydroxyl at the C-4 position is above the plane of the ring, while in glucose it is below the plane of
ring. The correct choice is B.
78. C is correct. Hydrolysis of just the galactose residue from raffinose leaves a single disaccharide composed
glucose and fructose. Sucrose is composed of both glucose and fructose. However, in order to call this disacchar
sucrose, the two monosaccharides, glucose and fructose, must be linked together in the correct fashion. Note that
linkage between these two sugars is glucose-o(l+2)-fructose. This is the correct linkage between these two
residues. Lactose is a disaccharide composed of galactose and glucose while maltose is a disaccharide composed
two glucose residues. The correct choice is C.

Copyright @ by The Berkeley Review too The Berkeley


Specializing in MCAT
Biology Structure and Function in Cells and Viruses Section VI Answers

79, B is correct. If there had been a loss of bacterial enzymes that degrade oligosaccharides, then the bacteria would not
be able to produce the monosaccharides which (when metabolized) lead to the production of lactate
(CH:CHOHCOOo), methane (CF{a), carbon dioxide (COz), and hydrogen gas (H2). It is the gases rhar are produced
that lead to flatulence. These compounds can also cause an increase in the motility of the intestinal system, fluid
secretions, and cramps. The correct choice is B.

80. A is correct. If we start the cell cycle just after cytokinesis (division of the cytoplasm), we will find the following
order:

Gr Phase
* S Phase * Cr Phase
+ Prophase--+ Metaphase * Anaphase -+ Telophase--> Cytokinesis
iir;il; oNi^sviliri.rirt ii;ffi;

Note that in choice B the S phase is shown to follow interphase. The S phase does not follow interphase, rather it is a
component of interphase. In choice C it says that anaphase comes after telophase. This is not the case as outlined
above. Finally, in choice C the Gt phase comes before the S phase, not after it. The Gz phase comes after the S
phase. The correct choice is A.

sl, D is correct. Two mature cell types in the body which do not replicate at all are skeletal muscle cells and nerve
cells. These cell types are said to be in their "resting" state in which they are maintaining the metabolic functions of
the cell. The resting state for these cell is the Gt phase of the cell cycle. It is in the Gr ptrase that transcription and
translation for the various cell functions take place. The Gt phase is part ofinterphase. Interphase also includes the
G2 phase (where the chromatin condenses) and the S phase (where DNA is replicated). After interphase is
completed at the end of the Gz phase, a cell enters into the M phase. This is where mitosis and eventual division of
the cytoplasm (cytokinesis) occurs. The correct choice is D.

!:. D is correct. We are interested in following labeled precursors which are incorporated into DNA. Northern blotting
involves RNA analysis, while Western blotting involves analysis of proteins. This allows us to eliminate choices A
and C. As outlined in the passage, Southern blotting is used to locate certain genes in a segment of double-stranded
DNA. The question is asking us for a way to monitor the incorporation of labeled precursors into DNA, not to locate
a particular gene in the DNA. Autoradiography is employed to locate radioactively labeled molecules. Radioactive
decay from an isotope that has been incorporated into a molecule would reduce silver grains in the emulsion of a
sheet of film. Development of the film would show a deposition of silver grains, indicating the location of the
Iabeled precursors. The correct choice is D.

A is correct. This question is designed to see if you understand the differences between the bases in DNA and RNA.
Recall that DNA contains adenine (A), thymine (T), guanine (G), and cytosine (C). RNA contains A, G, C, but uses
uracil (U) in_place of T. If we were to use 3H-Uracil, the label would show up in RNA. If we were to use either 3H-
cytosine or 3H-adenine, the label would show up both in DNA and in RNA. This allows us to eliminate choices B,
C, and D. If we just use 3H-thymine, the label shows up in the DNA and not in the RNA. The correct choice is A.

C is correct. Down syndrome (trisomy 21) is the most common type of human aneuploidy (the change in number of
one or more chromosomes during gametogenesis). It results from the non-disjunction of chromosome 2l from either
the father or the mother. Individuals with Down syndrome show a number of physical and mental abnormalities.
Life expectancy is relatively short, with a mean at about l7 years. Individuals with trisomy l3 (Patau syndrome) and
trisomy l8 (Edwards syndrome) show severe physical and mental abnormalities and usually die soon after birth.
Trisomy 22 is common in abortuses. Since there was no mention of the cause of Down syndrome in the passage, it
u as assumed that you would have learned about this genetic condition in your general science classes. The correct
choice is C.

C is correct. The best way to see the difference between primary and secondary nondisjunction is to consider
diagrams of each case. Primary nondisjunction (as stated in the question) occurs at the first meiotic division while
secondary nondisjunction occurs at the second meiotic division. These types of disjunctions are drawn below. By
:omparing the answers with these diagrams, we see that C is the correct choice.

p,-;fi1 O by The Berkeley Review lol The Berkeley Review


Specializing in MCAT Preparation
Biology Structure and Function in Cells and Viruses Section VI Answers

Meiosis I Meiosis I

II
Meiosis
tt/ \
/ \ T

@@oo Primary Nondisjunction Secondary Nondisjunction

If a normal gamete were to unite with one of the gametes of primary nondisjunction during fertilization, half the
zygotes that would arise which would show trisomy (the gain of one chromosome) and half would show monosomy
(the loss of one chromosome). However, if that normal gamete were to unite with one of the gametes of secondary
nondisjunction, then half of the zygotes would be normal, a quarter would show trisomy, and a quarter would show
monosomy. The correct choice is C.

86. C is correct. Maternal simply refers to the mother, and paternal refers to the father. The procedure for establishing
the autoradiograms is well outlined in the passage. All we need to do is read the results.

In both families we are looking for that parent which contributes to DNA fragments to the Down's child. In Family I
note that the mother contributes the two fragments (one at about 5 kilobases and the other at about I kilobase).

Normal Down's Normal Down's


Mother Father Child Child Mother Father Child Child
6 6
O
N^ ( N^ <
v) a r53
E?
o-o 4 Ed4

oo .-
- I - - CJ-o
I
-I
il:i
.t-t I _i:J , I -I --I
I I I I
- -
Autoradiograph of Family I Autoradiograph of Family 2

The father only contributes one fragment (at about 2 kilobases). In Family 2 it is the father who contributes the tu,o
fragments to the Down child (one at about 3 kilobases and the other at about 2 kilobases). The mother onlv
contributes one fragment (at about 4 kilobases). The correct choice is C.

87. B is correct. In order to understand where this answer came from, it is important to understand meiosis. Before the
actual onset of meiosis, DNA has already been synthesized during the S phase of the cell cycle. In prophase I of the
first meiotic division the chromosomes become visible and centromeres begin to develop. Homologous
chromosomes begin to pair or synapse at the centromere. This structure of two sister chromatids is referred to as a
bivalent. When two bivalents come together the complex is referred to as a tetrad. A tetrad consists of four
chromatids. The number of tetrads is equal to the number of haploid chromosomes. This means that if we have 2-1
haploid chromosomes, we must have 4 times that many chromatids. In other words, we have 23 x 4 = 96 chromatids.

After telophase I of the first meiotic division has occurred, two haploid nuclei are formed. These nuclei are haploii
by definition, even though they contain pairs of sister chromatids. Since the sister chromatids are attached by the
same centromere they are considered to be part of one chromosome. The total number of chromosomes in each
nuclei has been reduced by one-half. This is why the first meiotic division is referred to as a reduction division. The
second meiotic division is similar to mitosis and is therefore referred to as a equational division.

Copyright @ by The Berkeley Review t02 The Berkeley Revieu


Specializing in MCAT Preparation
Biology Structure and Function in Cells and Viruses Section VI Answers

Since the total number of chromosomes in each nuclei has been reduced by one-half, we now have 46 chromatids in
each nuclei as we enter into the second meiotic division. During metaphase II of the second meiotic division the
chromosomes align along the equatorial plane. If we stop at metaphase II, we find that the daughter chromatids have
not yet separated. Therefore, we will still have 46 chromatids present in each of the two nuclei at metaphase II. It is
not until anaphase II that the centromeres begin to divide and the sister chromatids move to the opposite poles. After
telophase II there will be 23 chromatids in each of the four meiotic nuclei. The correct choice is B.

88. C is comect. Fats are esters. Recall that esters result from the union of an alcohol and a carboxylic acid. The
backbone of all fats is glycerol, a three-carbon molecule containing a hydroxyl group on each of the carbon atoms.
Fatty acids contain a carboxylic acid group.

o
cH,
H)C- OH !
"o-
o
I
cH'
HC- OH !
"o-
o
I
H,C- OH 16- ! cH,

Glycerol 3 Fatty Acids (Stearic acid)

3 H2o
$.tt.-
o
H,c-o-! ""'
ln
"l- o- f "",

H2c-
tflo- 6
I

ctt,

A Triglyceride

Since glycerol contains 3 hydroxyl groups, there is the possibility of joining 3 fatty acids ro one molecule of
glycerol. This will form a fat called a triglyceride. Each linkage will be an ester linkage. The correct choice is C.

{l'q A is correct. This question involves looking at the four structures in the passage and determining the order of
polarity. We want to arrange the polarity from the most polar molecule to the least. The most polar molecule bears a
full charge or charges. The only structure in our four diagrams that bears full charges is phosphatidyl choline. The
remaining three structures do not bear a full charge. However, they all contain oxygen atoms, which are highly
electronegative. We first look for that structure with the most oxygen atoms. It turns out that both the triglyceride
and arachidonic acid have two oxygen atoms. Which is more polar? At physiological pH, arachidonic acid has its
carboxyl hydrogen ionized (not shown in the diagram). The two oxygen atoms on the carboxylic acid group will
share the negative charge between them (tkough resonance). This leads to stabilization of that functional group and
an increased polarity. The triglyceride contains ester linkages. Because one of the oxygen atoms is tied up with two
other carbon atoms, the polarity of the ester linkage is reduced. However, it is not as reduced as the polarity of the
single oxygen atom of the hydroxyl group of cholesterol. With this arrangement of polarity, we can oider the lipids
in terms of decreasing polarity. The correct choice is A.

C is correct. The more saturated the fatty acid, the more likely it is a solid at room temperature. The more
unsaturated a fatty acid, the more likely it will be a liquid at room temperature. Triglyceride II contains three
hydrocarbon chains which are completely saturated. This allows these three chains to pack close together in a tight
parallel arrangement, thereby increasing the amount of attractive forces between the individual chains. If there are
more attractive forces between the aliphatic chains, then it will require more energy (heat) to pull them apart. We

,,: ,:.:ht O by The Berkeley Review l03 The Berkeley Review


Specializing in MCAT Preparation
Biology Structure and Function in Cells and Viruses Section VI Answers

would expect the melting temperature to be the highest for triglyceride II. Remember, we are being asked for the
order of the increasing melting point for the four triglycerides? We can immediately eliminate choice B.

We can distinguish between choices A, C, and D by looking for that triglyceride with the most sites of unsaturation
(i.e., double bonds). This turns out to be triglyceride III with 3 sites of unsaturation. The more double bonds there
are in a fatty acid, the more disorder there will be in the packing of the hydrocarbon chains. The more disorder in the
packing, the less interaction there will be between individual regions of each chain. Therefore, less energy will be
required to pull them apart. We would expect the melting temperature to be the lowest for triglyceride III. We can
now eliminate choice A.

All that remains is to locate that triglycerides with the next two highest degree of unsaturation. They are triglyceride
IV (2 sites of unsaturation) and triglyceride I (1 site of unsaturation). Therefore, the order for increasing melting
point of the four triglycerides is III, IV, I, and II. The correct choice is C.

91. D is correct. The structure of cholesterol is shown in the passage in Figure 3. Note that there is just one hydroxyl
group on the molecule. The rest of the molecule is composed of a non-polar cyclohexane and cyclopentane ring
system. Attached to the cyclopentane ring is a long non-polar hydrocarbon chain. Diets high in cholesterol increase
the blood serum concentrations of cholesterol. Because this compound is quite insoluble in water they begin to
adhere to the walls of arteries and help in the formation of plaques.

If there were an increase in the synthesis of bile


salts from cholesterol, then less cholesterol would be in the blood
serum and more would be excreted in the feces. As we will learn a bit later, bile salts act to emulsify dietary lipids in
preparation for digestion. Similarly, an increase in the synthesis of steroid hormones means a reduction of
cholesterol in the blood serum. Both conditions would act to reduce the formation of plaques. Once cholesterol is
integrated into a lipid bilayer it usually stays there until the membrane is degraded. There is no (known) mechanism
to remove excess cholesterol from cellular lipid bilayers. Once the membrane is degraded, the excess cholesterol in
the membrane can contribute to plaque formation, but it is the insolubility of cholesterol in water that allows for this.
The correct choice is D.

92- D is correct. The structure of a simple lipid is shown in Figure I of the passage. Arachidonic acid has a carboxylic
acid group which can react with an alcohol group of glycerol to form an ester linkage. Therefore, arachidonic acid
can contribute to the formation of simple lipids. As outlined in the passage, the only difference between a simple
lipid and a complex lipid is that there is a phosphate group involved in the formation of complex lipids. There is
nothing to say that we cannot attach arachidonic acid to the alcohol group of a complex lipid. Thus, arichidonic acid
can contribute to the formation of complex lipids as well. Finally, it was stated in the paisage that arachidonic acid
is a precursor to the synthesis of prostaglandins and leukotrienes. The correct choice iJD.

93. C is correct. The question states the temperature is being changed from 38 "C to 25 "C. The bacterial cultures are
going from a warm environment to a cold environment. At high temperatures membranes are rather fluid; at low
temperatures they tend to be less fluid. However, it is the desire of every cell to maintain some degree of fluidity in
their membranes. This will allow for transport across that membrane. Which of the three answeis in the quesiion
lead to an increase in fluidity? If the synthesis of unsaturated fatty acids is increased, there will be less packing
between the fatty acid side chains and hence more freedom of movement. Choice A will occur. Initially there will be
a decrease in the bacterial membranes. This is simply because the cells will not have had enough time to begin
synthesis of the components needed to maintain membrane fluidity. Choice B will occur. The synthesis of short
chain fatty acids will increase. Why? The shorter the chain, the less packing between the chains. The less packing
between the chains, the less of an attractive force holding the chains together. The membrane becomes more fluid-.
Choice D will occur. Choice C will not occur because the bacterial cell does not want to decrease the fluidity of its
membrane. If the fatty acid chains are longer, there will be more attractive forces holding the chains together. This
will lead to a more solid membrane. The correct choice is C.

94. B is correct. A typical membrane phosphoglyceride is shown in the passage in Figure 2. We can hydrolyze this
compound as shown below. The hydrolysis products are glycerol, stearic acid, phosphate, and choline. Whit about
glucose and serine? The amino acid serine can be part of a phosphoglyceride (attached to the phosphate group like
choline). However, as outlined in the passage, glucose is not part of a phosphoglyceride simply beiause
phosphoglycerides are based on a glycerol backbone. Sphingolipids are based on a sphingosine back6one. In this
case, glucose would be associated with sphingolipids and not with phosphoglycerides. The correct choice is B.

Copyright @ by The Berkeley Review to,4 The Berkeley Review


Specializing in MCAT Preparation
Biology Structure and Function in Cells and Viruses Section VI Answers

o
tn'
H2c- o- [
lo t",
"l- [ n-
I o
cH,
II 6l
il
H,C-o-p-o- V' N-CH,' Phosphatidyl choline
'lCH.
oo
+ 4 H2o fl uydrotysis
{2
o Futty Acid (Stearic acid)
cH,
H2C- OH - 'j
"o
o
I cH,
HC- OH - 'j
"o CH,
o Ol
I ll
HzC- OH + HO-P-OH
oo
I
+
"o& f- ""' CH:
Glycerol
Phosphate Choline

95. D is correct. The question asks why the lambda virus is incapable of infecting other bacterial species. If these
species lack appropriate cell surface proteins, lambda phage may never be able to become adsorbed, or attached, to
the surface of these cells. According to the passage, this attachment is the first step in the process of viral infection.
If it can't occur, the virus doesn't infect the cell. We can approach this problem by a process of elimination as well.
Answer choice A is incorrect because most bacterial species have circular chromosomes, not just E. coli.
Additionally, having a circular bacterial chromosome has little to do with whether the lambda phage can infect a
cell. Don't be confused by the passage, which states that the viral DNA is circular. Likewise, answer choice B is
incorrect, as we know that E. coli has a circular chromosome (as opposed to the linear ones seen in eukaryotes).
Choice C is tempting, because we know from the passage that lack of a chromosomal integration site would prevent
the lysogenic infection pathway from functioning. We cannot say, however, that this would interf'ere with the lytic
pathway; the virus might still be able to infect the cell and lead to lysis. The correct choice is D.

Sh. A is correct. If the integrase protein were def'ective in a certain strain of lambda phage, that strain would be unable
to enter the lysogenic infection pathway. This is because the viral DNA would never be able to integrate into the
bacterial chromosome; functional integrase is normally required for this insertion process. Therefore, this viral strain
would be able to enter only the lytic pathway, always leading to the lysis of the host cell. Choice B is incorrect,
because the integrase-defective viral strain would still be able to infect E. coli; it just wouldn't be able to adopt the
lysogenic pathway. Choice C is likewise invalid, as the lytic pathway does lead to viral DNA replication. We can
also eliminate choice D, because a lack of integrase would not prevent the virus from entering the lytic infection
pathway. The correct choice is A.

C is correct. The question essentially asks which statements represent evolutionarily advantageous situations for the
lambda virus. An evolutionary selective advantage is basically anything that better enables an organism to survive,
reproduce, or adapt to change. Statement I represents just such an advantage. Recall, a provirus is lambda DNA that
has integrated into the bacterial chromosome. If the host cell were damaged and later died, the lambda provirus
riould also perish. Therefore, it is a selective advantage for the lambda provirus to excise itself and enter the lytic
;ycle when the host cell is initially damaged. In this fashion, it can quickly replicate new viral particles and leave the
d1,ing host cell, potentially to infect new neighboring E. coll cells. This is an evolutionary advantage. Statement II is
likewise beneficial evolutionarily. If lambda always killed its host cell immediately (i.e., via the lytic pathway), it
nould soon run out ofviable host cells. It would also have less chance ofbeing dispersed to new areas. If itdidn't
always kill its host cell immediately (i.e., the lysogenic pathway), it could replicate along with the bacterium and

'rr,i1:-nt O by The Berkeley Review los The Berkeley Review


Specializing in MCAT Preparation
Biology Structure and Function in Cells and Viruses Section VI Answers

therefore be more successful at spreading to new areas. This is an evolutionary selective advantage. Statement III,
on the contrary, does not represent an advantage. If lambda DNA were almost completely resistant to mutation, it
could never evolve to meet new environmental challenges. In essence, it would be incapable of adapting to change.
This is not an evolutionary advantage. The correct choice is C.

98. B is correct. The experiment described in the question involves radioactively labeling the lambda viral DNA,
packaging this labeled DNA into a phage protein capsid, and allowing this new virus to infect an E. colicell. The
question effectively asks where the viral DNA would be after lysogenic infection not lytic infection (the cell does
not lyse). Recall from the passage, the lysogenic infection pathway leads to the integration of viral DNA into the
host's chromosome. In prokaryotes such as bacteria, chromosomes are located in the cytoplasm of the bacterial cell.
This is were we would detect radiation from the integrated, labeled viral DNA. Choice A is incorrect because
bacteria do not have nuclei. Choice C is invalid, because we would only see labeled viral DNA bound to ribosomes
if it were being translated. After integration via the lysogenic pathway, the viral DNA does not direct protein
synthesis and is therefore latent. Choice D is wrong, because we would only see labeled viral DNA outside fhe cell
if lysis, or cell bursting, had occurred, releasing new virus particles. Since the virus in question does not enter the
lytic pathway, this would not occur. The correct choice is B.
99. C is correct. A cytosolic endonuclease is an enzyme which can cleave DNA at specific sequences within the strand.
Endonucleases (also called restriction enzymes) are therefore capable of cleaving circular viral DNA. Such cleavage
would prevent viral DNA from being either translated directly or integrated into the bacterial chromosome. Injection
of lambda DNA is a step common to both infection pathways; the DNA injected at this step is degraded "by the
bacterial endonuclease. Therefore, both lytic and lysogenic pathways of infection would be inactivated. The lytic
pathway is prevented because lambda DNA is cleaved before it can direct synthesis of the protein capsid. ihe
lysogenic pathway is inhibited because lambda DNA is cleaved before it can either direct integrase production or
insert into the bacterial chromosome. This information leads us to eliminates answer choiceJA, B, and D. The
correct choice is C.
100. A is correct. The qtrestion essentially requires us to know the differences and similarities between a lambda phage
which adopts the lytic pathway of infection and a lambda phage which adopts the lysogenic pathway of infection.
The question asks about the similarities between the two pathways. From the passage, we learn thal the common
step to both pathways is the adsorption ofthe phage to the bacterial cell surface and the subsequent injection of viral
DNA. This makes choice A correct; the two pathways do NOT differ in this regard. We cin also proceed via a
process of elimination. Choice B can be ruled out, because the lysogenic and lytic pathways both adopt dffirent
methods of replicating viral DNA. The lysogenic approach involves replication along wittr the host chromosome,
while the lytic approach involves replication independent of the host chromosome. Therefore, this answer choice
represents a way in which the two pathways dffir. Choice C is likewise incorrect, because it also represents a
' difference between the two pathways. While the lytic pathway is immediately lethal to the infected cell ldue to
lysis), the lysogenic pathway may allow the cell to undergo multiple replications unharmed. In the same sense,
choice D can be eliminated. Only viruses entering the lysogenic pathway can remain latent for long periods. This is
another difference between viruses entering each of the two respective pathways. The correct choicels A.

Copyright @ by The Berkeley Review l06 The Berkeley Revier


Specializing in MCAT Preparation
ology A. '..,,E m€ l{inetics '. , ,

$€bff,$fi.....VII l. Transition State


''l.r'. Li.r .''
,, t'., Mic,haelis-&{enten E'.quation
5.
lA tid '1, &.'
Lineweaver-BurkPlot
Enzynre Inhibition , ,'.

B.,,,,' lfinryme1,:MgchanlsmS .',


l.
Chymotrypsinl\zlechanism
2.
RN-*A as an Active Enzyme
3.
Inactive Enzymatic Precurso.s
4.
Molecular Evolution
':' , ,5; ,, Trmsition State Analog ,'tt, ,

llrstabteTetrahedral Hiss;
C. Metabolic Molecules
-_! .\ @
t. AdenosineTriphosphate
s*t.s.- ''t\;n-":*--.*o-c-Asfto2 ',,r:','!,.',Cofaciors anA,Coaneyrnes
'],,,,,'
J ]],,,.

--ll'', Acyf rEn@€,i T f\ Aminoproduct:,,.,;':3'),, IiliCotinamideAdenineDinucleotide


htefhr€diafF: H- lj-- R--1/ rea'es rhe scene
Dinucleotide

li, ,,, ,,, ,l l,i


i:.
:.. trJi:ffinine
':., 'I
.. :',,,.
,:'_ r,'.'.::ii i l:i.,
1..'':,1,',.. lill :
r ',.t,il

i....;.r:."..lrf r I l:

K'i,Ei.VtI.EtW*
Spec,ializin$ irn' k[C Preparation :
:
I
Metabolic Components
Top lO Section Goals Li
n
-{r
oB Be familiar with the concept of transition states.
It is important to understand the difference between the activation energy of an uncatalyzed reaction :J_
-:
and thdt of a catalyzed reaction. :ri_-
::t:.
t::
av Understand how to read a Plichaelis-Menten graph.
Don't,worry about the algebra involved in deriving the Michaelis-Menten equation. lnstead, be able
lt_:

to understand the components of that equation in relationship to its graphi-al form. -rL: ;
x ]:;
Understand how to read a Lineweaver.Burk graph.
@? Be able to i^
s;
sure you understand the meaning of the different pararieters.
lrr nF

$r:triL
Be familiar with the concept of enzvme inhibition.
@v Enzyme inhibition is a favorite topic in biology. Know the ditferenc" U"t*""t"t u .ot rp"titi* una u
non-competitive inhibitor and how they function in the presence or absence of substiate.

Understand the basics of th" chy-otry


The details of this mechanism have been on the MCAT before. Those who took this exam needed
only a basic understanding of enzymatic mechanisms. Keep track of yoo,
"i*.tronri

ov Be familiar with the concept of molecular evolution.


On occasion, the MCAT has had a few questions concerning evolution. Be aware of the terms
co nue rge n t euol u t io n and d i ue rge n t euol u t i o i.

Know the basic components and functions of the molecule ATP.


@w ATP is the energy currency of the cell. We will encounter this metabolic component throughout our
entire review session. Know its components and understand its uses in metabolism.

Know the basic components and functions of the coenz;nnes NAD and FAD,
"v NAD and _FAD_are both coenzymes. They are not enzymes. A coenzyme helps an enzyme function
properly. NAD and FAD are frobably ttie two best known coenzym-es.

Be familiar with coenzyme A and its uses in metabotism.


@v Coenzyme A is another importart coenzyme, especially in the Krebs cycle and in fatty acid metabolism.
Have some understanding of where this molecule is used in the cell and why.
Biology Metabolic Components Enz;rme Kinetics

ftffi$iffi$ffilii$
Activation Energy & CatalYsts
Chemical reactions occur at a particular set of rates. In the reaction shown in
equation (7-1) there will be a rate under specified conditions at which molecule A
.un b""o*" molecule B or molecule B can become molecule A. Usually k1 is kr
referred to as the forward reaction rate while k2 is referred to as the reverse A-
A-<: B (7-1)
:eaction rate. The point of equilibrium is where [A] and [B] no longer change k2
',r-ith respect to one another. If k1 > k2, then at equilibrium [B] > [A]'

I this is true, then why don't all reactions come to equilibrium immediately? In
:rder to answer this question we need to consider the energy diagram shown in
Figure 7-'1,.Ifwe have two energies for A and B, we will find that at equilibrium
ts ivitl be in higher concentration. It is a bit more stable than A. It has less energy
:rd is therefot" *ot" stable. In order to get from A to B we have to go through a
::gher energy state. This is often referred to as the transition state (TS). Do not
--:jnk of this reaction as one molecule of A and something else, but rather think of
-: as a population of molecules.

E
a ^*r"J;;;.;;
of the
a
,- Uncatalyzed Activation EnergY
Reaction of the
Catalyzed Reaction
!

- A
-r'l (Substrate)
.J
!
B
(Product)

Progress of Reaction

Sgure 7-f
""u.s:r:ion States and the Effect of a Catalyst.

ir
--:,tn this population of molecules some will have more energy than others
r:lr- because some have been "heated" more recently than others. They have
* **i rr contact with something that has given them energy. The population as.a

, * : -e d"oes not have a particular energetic state but rather a range of energetic
,ril::s. In order to make the trek up to the transition state, some proportion of the
t;:':Lation of molecules of species A must have enough energy to be at that
: ir-.:5on state. If it were u r"iy small proportion of the population of species A,
:T,*: would expect to see t-he forwirdreaction occurring very slowly. If ryg
-,.,-e

t:= :o look at the population of molecules represented by species B' we would


fir,* -i.at it would be a harder trek to go back to species A because of
the greater
-:i::::ence in energy between B and A. Some population of molecules from
lli.r,,r'::.s B, though, would be expected to have an energy equal to the
difference

Review l09 The BerkeleY Review


-,.,q,,
-qht O by The BerkeleY Specializing in MCAT PreParation
Biology Metabolic Components Dnryme Kinetics

between the transition state and the average thermodynamic state of species B.
This will be different at different temperatures. At low temperatures fewer
members of the population of A and B will have enough energy to make the trek
across the transition state. This is why chemists, in order to achieve equilibrium,
often raise the temperature. If we raise the temperature, then more of the
population of species A, per unit time, will cross the barrier.

How9v,er, in biological materials raising the temperature is usually not a !


possibility. For any one particular organism there is a range of temperatures
which is fairly restricted (i.e., around body temperature). A diff"rent method
must be employed and that is to use a catalyst. A catalyst is a general name for
any substance (or substances) which can provide an alternative route or an T
alternative transition state. A catalyst can change the nature of the intermediate
state to some lower energy state, If the transition state (TS) is lower in terms of its ffi
own energy, then the nature of the relative proportion of the population of A and
B that can obtain a transition state at a-paiticular temperaiure is increased W
(without an increase in the temperature). This is indicated in Figure 7-1. I@m

M
we can think of the enzyme as being like a tunnel through a mountain (the
energy barrier). Instead of expending a lot of energy to go over the mountain, r,r,e ffim
can go through the mountain to the other side (products) by way of the tunnel.
As our molecule A goes through the tunnei (i.e., is worked on by the enzyme) it
ilh,
is converted to B. Our tunnel allows A to be converted into B. Wlttrin the iunnet.
iM
though, molecule A goes through a high energy transition state (less stable
before it becomes molecule B. It turns out that one can synthesize molecules that
look like the transition states of molecules that are being examined. These
synthetic structures are called transition state analogs, and they can easil,--
interact with our enzyme (fit into the tunnel). Since this transition state analos
has nothing to do wiih A going to B, it actually impedes the process. In other
words, transition state analogs are excellent inhibitors of the catilytic process.

Thus, enzymes are catalysts which are often employed to loz.oer the transitio:
state's activation energy. By definition, a catalyst will not alter the equilibrium ci
a reaction. However, a catalyst will alter the rate of a reaction. In other words, a:
enzyme accelerates the forward reaction as well as the reverse reaction t,-,
precisely the same factor.

illicftddili5r,M6iltEH EHdhtion
Derivation & Meaning
How do enzymes carry out their catalysis of a reaction? Enzymes contain spec:il
regions called active sites to which a specific substrate molecule will bin;
Enzymes also stabilize the transition state and they carry out acid-base catalr sirs
by precisely positioning the catalytic groups of certairramino acids (e.g., ,i=,
Glu, Lys, Arg, His, Ser, Cys, etc.) found within the active site pocket.

For a number of enzymes, the catalytic rate (V) changes with the substi
concentration (lsl). In equation (7-2) the enzyme (E) combines with the substr'
(S) to form an enzyme-substrate complex (ES) with some rate k1. This
complex can continue on to form the product with some rate k3 orit"
dissociate to the substrate and enzyme at some rate k2.

Copyright @ by The Berkeley Review llo The Berkeley Rer


Specializing in MCAT Prepar
Biology Metabolic Components Enzyme Kinetics

k1 kr
E+S- kt
S----.-..-.-.-E+P
r5-2)

muhen [S] is small and the [E] is constant, then V is essentially proportional to [S],
:::.'-:ing a first order reaction. However, when [S] is large, then V is essentially
i:nrJependent of [S], giving a zero order reaction. This can be seen in Figure 7-2.
1}e tvpe of curve obtained is hyperbolic.

l}Le velocitlz of the reaction given by equation (7-2) is shown in equation (7-3).
}e only way that we can obtain the product is through the ES complex.
lt, -J, V= kslESl

The total concentration of enzyme in the system in question is equal to the Substrate Concentration IS]
,smcentration of the free enzyme plus the concentration of the enzyme bound
Figure 7-2
"w:h substrate in the ES complex. This is shown in equation (7-4).
Reaction rates as a function of
substrate concentration.
fir-41 Etotul = [E]fr"" + [ES]6orr,.,6

ltur he following discussion we will consider the derivation of the Michaelis-


,\ufislten equation (7-14').It is important to know how to use the Michaelis-Menten
eumiation but not how to deriae it. The only reason the derivation is presented here
ur :or completeness. If you wish, you can skip this derivation and proceed
dinecfly to equation (7-'1.4).

We rvould like to describe the rate of an enzyme reaction in terms of some


flrmrameter that we can measure. It would be nice to measure ES. Unfortunately
fitinrnq !5 13*1gr difficult. Consider the rate of change of
[ES] over any given time.
Th:rs can be seen in equation (7-5). This is simply the rate of formation of ES
munus the rate at which ES is broken dovrm.

iN{I-5} dlEsl/dt = krlEllSl - (kz+k3)lEsl

ll,,cqnnor Michaelis and Maud Menten reasoned (circa 1913) that the concen-
lftamon of the intermediates in a steady state process remain the same while the
umrrentration of the reactants and products change. In other words, d[ESUdt = 0.
"lhr:s r!'ill
occur when the rate of formation of the ES complex and breakdown of
dftE ES complex are equal. This can be seen in equations (7-6) and (7-7).

{ilf-61 dlEsl/dt =0= kltEltsl - (k2 + k3)[ES]

lu - | kltEllsl - (k2 +k3)[ES]

M*arranging equation (7-7) gives equation (7-8).

illli",-fi lESl = tEltsl/(k2 +k3)/k1


$lhrli* can now define (k2 + k3)/ft1 to be the Michaelis constant, KN1. Substituting
tfms constant into equation (7-8) gives equation (7-9).

'iffi-9[ lESl = [E][S]/Krra

tfiaUrright @ by The Berkeley Review tll The Berkeley Review


Specializing in MCAT Preparation
BioIo gv Metabolic Components Enz5rme Kinetics

Now, rearanging equation (7-4) gives equation (7-10). And substitution of (7-10)
into (7-9) gives equation (7-11).

[E]r."" = [E]totut - [ES]uo.,r,a (7-10)

tES] = (lE]totur - [ES])[S]/KM (7-t1)

Solve equation (7-11) for [ES]. This gives equation (7-12).

[ES] = [E]total (tsl(tsl + Kria)) (7-12',)

Substituting (7-12) nto equation (7-3) gives equation (7-13).

V= k3[El16tar ([S]/([S] + Ku)) (7-73)

Let's define the maximal rate of a reaction as v-u*. This reaction rate can be
obtained when all the enzyme's active sites are saturated with substrate. In other
words, vmax = kglE]total. substituting this expression into equation (7-13) yields
equation (7-1'4). This equation is the Michaelis-Menten equation and it is an
important equation in enzyme kinetics.

V- V-u^ [S]
Michaelis-Menten Equation
[S] + Krra
(7-14)

If we plot the velocity of a reaction as a function of the substrate concentration,


we will get ahyperbolic curoe (Figure 7-3). This is the same type of curve that we
will see when we examine the binding of oxygen to myoglobin. In the case of an
enzyme, we find that when we saturate that enzyme with substrate, the enzyme
is operating at its maximal velocity (i.e., Voru*). In the case of an enzyme, when
[S] = KM, then V = Vmax/2. In other words, the Ky is equal to the substrate
concentration at uhich the reaction rate is half of its maximal ualue.

The Ky can also be related to the rate constants in a given reaction. Suppose we
have a situation in which kz>>>kg. This means that the enzyme-substrate
v-u*
x complex (ES) will have a tendency to dissociate to E and s rather than form E
O and P. In this situation, KM = kZ/kt. It is only when this situation is true that Kg
v-u*
0 is a measure of the binding strength of the ES complex.
2

.J In other words, a high K14 value indicates a weak binding of the


ES complex while a low Kyl value indicates a strong binding of
Substrate Concentration IS]
the ES complex.
Figure 7-5
Michaelis-Menten kinetics showing
variation of Reaction Velocity with
In practice it is not very convenient to estimate V*ar, in the manner just
Substrate Concentration.
suggested. This is because as we increase the amount of substrate the solution
becomes more and more viscous (e.g., like honey). The only values that are really
much use to us are those in the area where things arc first order.

In some cases we will not have a perfect hyperbola. For example, if we had a
protein that had a structure like that of hemoglobin, where each of the four
subunits has an active site, then those active sites might act independently of one

Copyright @ by The Berkeley Review tt2 The Berkeley


Specializing in MCAT
Biology Metabolic Components Enz5mre Kinetics

another or they might influence one another. If one of those subunits was
nteracting with a substrate, it could cause a change in the affinity (the Ky) of the
rther active sites with respect to their interaction with other substrates. Since this
:equently happens it turns out that there is cooperativity in multimeric proteins
-,rith more than one active site. Allosteric enzymes have a non-hyperbolic curve
i.e., more sigmoidal) when plotted as velocity versus substrate concentration.

Lineweaver-Burk Plot
Meaning
-: became of interest to try and find some way of dealing with Ky that did not
rr-olve plotting the hyperbola and then estimating the Vmaxlz. Lineweaver and
Burk developed a plot which is now referred to as a double-reciprocal plot. In
-:js plot we graph l/V as a function of 1ilSl. If we were to plot the hyperbola
.hown in Figure 7-3 onto this graph, we would end up with a straight line.We
:an obtain an equation for this plot if we take the reciprocal of the Michaelis-
l-fenten equation. This is shown as equation (7-15). Note that this equation is in
-:.e form of y 11 + b, the equation for a straight line.
=

1-[r*lIrl,
v - t-v,""J t-rsr] v-*
I
--15) Lineweaver-Burk Equation

-i"e solid dots on the Lineweaver-Burk graph are data points (Figure 7-4).If we
,h:ere to take the best fit through those data points and then extrapolate the line,

",e ivill intercept the Y-axis at L/V*u*. If we continue to the X- axis, we will
.::ercept at -1lKp1. Note the region of high IS] and low[S] values. If 1/[S] is Slope = KMff-u*
:!:proaching zero, then it must mean that [SI is approaching infinity. At the
r:ersection of the X-axis and Y-axis the [S] would be equal to infinity. This is the tlir
:,:int at which V became V*u* in Figure 7-3.If the line in Figure 7'4 is not r

':aight, it indicates that we do not have a hyperbola. If we do not have a Figure 7"4
r-.-l'erbola, then we have an enzyme which is undergoing some type of Lineweaver-Burk plot.
:,-:ernative reaction. This is very often a sign that we are dealing with a
r-:ltimeric protein which binds more than one molecule of substrate.

,,1en an enzyme is completely saturated with substrate, then the number of


'-lstrate moleculei which are converted to product per unit time is referred to as
r-e furnover number.

Enzyme:Inhibition,' :::: :

il.eversible & Irreversible


I- -re are two major types of enzyme inhib.ition: reaersible and irreoersible
rdribition. One example of an irreversible inhibitor is diisopropylfluorophosphate
DIIPF), a potent nerve gas. DIPF inhibits the enzyme acetylcholinesterase, which
i :'l-olved in the hydrolysis of the neurotransmitter acetylcholine to acetate and
-::'j:re. In other words, DIPF blocks cholinergic nerve impulses within the body.
: :j-:Trs out that DlPF-inhibited enzymes have an unusually reactiae serine residue
r: -:re active site. Other enzymes like elastase, trypsin, and chymotrypsin all have
'::;tive serine residues at their active sites as well. Thus, these enzymes are
::uped into a class of enzymes called serine proteases.

- :,:.,right @ by The Berkeley Review rl5 The Berkeley Review


Specializing in MCAT Preparation
Biology Metabolic Components Enzyme Kinetics

There are two types of reversible inhibition. They are competitiae


inhibition
and non-competitiae inhibition (of which allosteric inhibition
is a subset).
E + s=5 ES k3-E + P Competitive Inhibition
+k2 A competitive inhibitor (K1) usually resembles the normal substrate
I molecule
11d can readily bind at the active site of the enzyme to form an enzyme- J
1l*, inhibitor (EI) complex as shown in Figure 7-5.
['
EI T there is a competitiae inhibitor around, it will compete with the e
substrate for e
the active site on the enzyme. In Figure z-s we see that the enzyme
Figure 7-5 is being ,&
utilized in two pathways. one pathway utilizes the normal substrate
Competitive Inhibition, and the M
other pathway utilizes the competitive inhibitor. The consequence
of that ,&
competition is a decrease in the rate of catalysis of the enzyme. The
rate of
formatio_n of the product is dp/dt = ks[ES]. If some of the
IE'sl is removed to
become [EI], we essentially have a lower concentration of ES. ,ds,

&
A competitive inhibitor can be overcome at high concentrations of substrate W
Uninhibired (Figure 7-6). Initially we have a louter vmaxlo, u gr""" m0
vru* *bstrate concen- @
tration, but as we approach an infinite substrite concentration, the
concentration of the competitive inhibitor becomes negligibte
v\/ and the enzyme um
b 'rn4x catalyzed reaction will once again approach the,u*u f_i.
oa
aL ME
o ,,diil
when a competitiae inhibitor is present it changes the appareni Ky
of the q@
The word "apparent" is used because
:lry*:.
Ky-val1e "u.h the
for a given substrate. In other words, "^rymehas
a characteristic fi
0 Ktr,t Ky of a competitiverr-
isl inhibited enzyme appears to shift to a higher varue. 'Fio*"rr"r, if we add ,@m

ne@
Figure 7-6 enough substrate, we will eventually reach trie same vrr,u*. This
can be seen in
Competitive Inhibition "appears" to both Figure 7-6 and tn Figure 7-7 .

increase KM.
shown in Figure 7-7 is the Lineweaver-Burk representation of Figure
7-6. In
each plot the concentration of inhibitor and enzyme is constant.
Again, we find
that the Vrnu* remains constant while the apparent K1,4 changes. Note
that a
competitive inhibitor will increase the slope of tne tines" found in
the
Lineweaver-Burk plot.

[I] = 2Kt lll = Ki

Uninhibited

Slope Kv
= V*u*

V.o*

_1
Krra ?
Apparen
oo

_1
Kv

Figure 7-7
Lineweaver-Burk Plot for Competitive Inhibition.

Copyright @ by The Berkeley Review tt4 The Berkeley Revieu


Specializing in MCAT preparatior
Biology Metabolic Components Enz5rme Kinetics

An example of a competitiue inhibitor would be ethanol. Ethylene glycol, a


constituent of antifreeze, can be converted via a.number of reactions in the body
to oxalic acid (which causes kidney damage and often results in death). One of
those reactions involves the enzyme alcohol dehydrogenase.

It turns out that alcohol dehydrogenase will also convert ethanol to acetaldehyde
irvhich can then be converted to pyruvate--more on this in a later discussion). If
ethylene glycot is ingested, all one needs to do is drink intoxicating levels of
ethanol. The end result is that ethanol will compete with ethylene glycol for the
active site on the alcohol dehydrogenase enzyme, and since there is more ethanol
:han ethylene glycol around, the enzyme will convert much more ethanol to
:ceialdehyde than it will ethylene glycol to oxalic acid.

,\nother example of a competitive inhibitor is methotrexate, an anticancer drug.


As we will see later, rapidly dividing cancer cells require a continuous stream of
:recursor molecules (called deoxyribonucleotides) for the synthesis of DNA.
I.lethotrexate resembles a molecule called dihydrofolate, an intermediate in the
::ductive synthesis of tetrahydrofolate (Figure 7-8).

- re enzyme that carries out this reaction is dihydrofolate reductase'


l.lethotrexate binds to dihydrofolate reductase and prevents the conversion of
:ivdrofolate to tetrahydrofolate, which is not only an essential cofactor for the
.-,-nthesis of one of the bases in DNA (i.e., deoxythymidylate (abbreviated as
:TMP)) but it is also a cofactor in the synthesis of all other reactions that depend
,:. tetrahydrofolate (e.g., the amino acids methionine, histidine, and the purine
: :,.svnthetic pathways).

n H
I I

C- NH - C- COO C-NH_C_COO

+i,:. T +i::"
- N-H
"1r
l, { .",-
''-.]/
,i
"
____1H.N*fH
A.-,[ ,k"
"

Dihydrofolate H
Reductase
Tetrahydrofolate (THF)

nrgLrre 7-B
: -- ::3xate is a Competitive Inhibitor.

ght O by The Berkeley Review I l5 The Berkeley Review


Specializing in MCAT Preparation
Biology Metabolic Components Enzyme Kinetics

Non-competitive Inhibition
kr k3 Non-competitive inhibitors do not compete at the active site of an enzyme,
Ba$:ES __+E+P rather bind to some other site on the enzyme. Binding at this ,'ailosteric
but
+ k2+ site,,
will alter the conformation of the enzyme ihus giving rlversible inactivation of
I I the catalytic site. These inhibitors can bind tJbottlthe free enzyme and
the t
lfn k1 filn
EI + S: ESI
enz).nne-substrate complex (Figure 7-9).

One characteristic of non-competitive inhibition is that it cannot be


I
il
k2 overcome
by increasing the substrate concentration. This means that we will have alower
Figure ?-9 apparent v*"* for the reaction. However, the Ky will not change. why?
Non-competitive Inhibition. Consider one specific type of enzyme. The non-competitive inhibitor that
we add
to the reaction mixture containing this specific ut iy-" will bind to the enzyme
at some allosteric site and cause a conformational ihange to alter
the enzyme,s
V,nu* active site. This enzyme is no longer functional u" u unit. However,
"itulytic
there are still other enzymes in the reaction solution that may not have had an
:V 'max encounter with this non-competitive inhibitor. Those unry*", have active
ts] sites
g2 that are just as active as they ever were, and they still have the same Kp1 value
()
Non-competitive
as their brothers and sisters (because the K1,4 is a relationship b"i.,"""r,
Inhibition substrate and actiue enzyme). But since their numbers have been reduced (by
non-competitive inhibition), they have a lower overall vmax. This can
oKu be seen
in Figure 7-10.
tsl
Figure 7-lO The Lineweaver-Burk representation of the plot shown in Figure 7-10
Non-Competitive Inhibition isshown
Decreases Vmax.
in Figure 7-11. Note that the apparent K1,4 remains constait while the v-6y
clecreases.

Uninhibited

Figure 7-l I
Lineweaver-Burk Plot for Non-Competitive Inhibition.

Copyright @ by The Berkeley Review 116 The Berkeley


Specializing in MCAT
Biology Metabolic Components Enryme Mechanisms

r biochemistry one usually learns the mechanism for at least one of four classic
::lzymes. Lysozyme is an enzyme that hydrolyzes a specific glycosidic bond in
re polysaccharide component of certain bacterial cell walls. Ribonuclease is an
:lzyme that catalyzes the hydrolysis of phosphodiester bonds in RNA polymers.
Carboxypeptidase, is a zinc-containing digestive enzyme secreted by the
.rocrine cells of the pancreas. This enzyme hydrolyzes the carboxyl terminal
::ptide bond of protein polypeptide chains. Chymotrypsin catalyzes the
:'" drolysis of either ester or peptide bonds. All of these enzymes are referred to
:: hydrolases. Water is introduced between a particular bond that had originally
:'=en formed from the loss of water. In this section we will consider the reaction
:- echanisms for chymotrypsin.

chtffiff ffi,sflft...iiffiHili bm
-:.r'motrypsin catalyzes the hydrolysis of either ester or p"pt![" bonds. If a
',::::le is hydrolyzed, the products will be an amine and an acid. If an ester is
" , 3ro1yzed, the resulting products will be an alcohol and an acid.

-r:-,r- rvas this catalysis at the active site revealed? One of the amino acid residues
,: -r-e active site was identified using diisopropylphosphofluoridate (DIPF), an
r':=..-ersible inhibitor. It turns out that chymotrypsin has 28 serine residues, yet
: -.' one of those residues, Ser-195, reacts with DIPF. Another of the amino acid
':s-jues at the active site, His-57, was identified using tosyl-L-phenylalanine
-:" -:omethyl ketone (TPCK) and a process called ffinity labeling. Knowing the
: . . peptide sequence of chymotrypsin, making use of the different types of
,,i:,=-::rg techniques, and using x-ray studies, it was discovered that His-57 is not
--" adjacent to Ser-195 but also to another amino acid residue, Asp-102. These
:'':- residues, shown in Figure 7-12, lorm what is called a catalytic triad at the
- = site of chymotrypsin.
"
-:=: physiological conditions, where the pH is about 7.4,5ev195 will have a
" , ::ound 13. What this means is that for all practical purposes, Ser-195 will
li,t ::tain its hydrogen atom on the hydroxyl group of its side chain. (Remember,
,": :t',? pH of the solution is greater than the pKaof the ionizing side chain, the
',::,

'" ;":*'.:nnnt species is the conjugate base of the side chain, and when the pH of the
i r -" . ": is less than the pKa of the ionizing side chain, the predominant species is the
u.,:",,i .:,u,other way to think about this is as follows: pH ) pKq, then [HA] < [A-1 and iJ

r i < vtl(o, then [HA] > tA-1.) Therefore, at a physiological pH of about 7.4, only
l -: 1 r 10-6 molecules of Ser-195 will be in the form of the conjugate base.
i'e:-195 is reacting with DIPF, there must be something that is allowing the
xu i : : -lrat serine residue to be lowered. If the pKa of serine is lowered, there will
rr; r ::: of a chance of finding it in the form of the conjugate base. In this case the
rr -'u-. >:de chain would be in the oxyanion form. When alcohols lose their
:r:, :.ns they become alkoxide ions--and an alkoxide ion is a stronger base and
1"",,,:r'i.f , rE a better nucleophile--and nucleophiles love to give away electrons.

r r,i-:*. .-ut that the His-57 residue is unprotonsted when there is no substrate
"llr::: r the active site. Remember, histidine has a pKx of about 6.0, and since

rrr i- --f 9 bv The Berkelev Review tt7 The Berkeley Review


Specializing in MCAT Preparation
Biology Metabolic Components Enzyme Mechanisms

we are at a pH of about 7.4, roughly 96% of it will be unprotonated. The Asp-102


residue that is part of this triad has a pKa of about 4.4. At physiological pH it will
essentially be in the anionic form. It therefore can act as a base and interact with
His-57, thus making His-57 an even stronger base than it would have been if the
Asp-102 residue were not present. What this means is that His-57 can abstract a
proton from Ser-195 as the substrate comes into the active site. Once Ser-195
loses the proton from its hydroxyl function, it becomes a rather reactive alkoxide
ion.This portion of the mechanism is shown in Figure 7-12.

His57
\
I
Ser1e5-
/-;4
o- H ------ -!------: *V"- H
o
----------------o-
fl
c- Asproz

1t
I
His57

)
I
oF.oR- *\Z*-
Ser1e5- O:-------------' H H --------------- g- C- Asproz

Figure 7-12

We have mentioned that chymotrypsin catalyzes the hydrolysis of either ester or


peptide bonds. Let's consider the hydrolysis of a peptide bond in a typical
protein substrate (Figure 7-13).

His57

)
I

Ser1e5- OOr) u -
/4.N-
N.l ,, H
ofl
o- C-
---------------: Asproz
i./
i.. lt
R'-
' llr N-
C- Rr
il
H
o) His57
Substrate )

Ser195- O H
-
F'
N 7, N-
I

H
ofl
--------------- O- C- Asproz
t..
Unstable Tetrahderal Transition State

Figure 7-13

Copyright @ by The Berkeley Review lra The Berkeley Revier


Specializing in MCAT Preparatior
Biology Metabolic Components Enz5rme Mechanisms

-\lkoxide ions are strong bases and quite reactive. When the Ser-195 residue is in
re alkoxide ion form it is a strong nucleophile, and nucleophiles love to seek out
:arbonyl carbons and pass electrons to them. (Carbonyl carbons are partially
:csitively charged while the oxygen is partially negatively charged.) Flowever,
-:is will place too many electrons on the carbonyl carbon. The result is that the
=-ectrons in the double bond of the carbonyl will move to the oxygen.

lrce the electrons move to the oxygen we form an unstable tetrahedral


:".tttsition state. The function of the enzyme is to stabilize this transition state.
- ris is accomplished by hydrogen bond interaction of the negatively charged

-
':bonyl oxygen with the N-H groups of the main chain Ser-195 residue and a
l-r-193 residue located in the vicinity.

thi.s point the electrons essentially have two paths they can follow. The first is
":
--a: they can return to where they came from (reversing Figure 7-13) or they can
::-:eed in such away that there is collapse of the tetrahedral intermediate to
::n the acylenzyme. This occurs with expulsion of the leaving group, which in
,- -: case is the amino product (Figure 7-14).

His57

Ser195- O
I| /"r---.
J.
..,-"-.-)..Z*- H
L?
--------------- O- C- Asp1s2

R'-
-"
' l\HN: R,
C:
o)v 1t

L nstable Tetrahedral His57


Transition State

ie119-i- O :NVN- H --------------- O- C- Asp162


Rr- Acyl Dnryme
H
I d Amino product
Intermediate H - N- R, ' leaves the scene

tilgure 7-I4

' . ;:-,'1 enzyme intermediate that we have just formed provides a lower energy
-- ,", ar- to get from the substrate to the product. Once the amino product is
--
=: it is free to leave the scene. We could consider the steps that we have just
: - red the First Act in the opera Chymotrypsin (abit like the First Act in the
' -n r-r: opera Die ZauberJldte by Mozart).

- ::.e beginning of the Second Act we need to introduce water in order to


, -;. our acyl enzyme intermediate. As the His-57 function removes a
:,- : ::n atom from water, there will be a nucleophilic attack on the carbonyl
of the acyl enzyme intermediate by the oxygen atom on the water
, : --e. Again, a transient tetrahedral intermediate is formed as shown in
.: .-:l -l-5.

::rt O by The Berkeley Review t19 The Berkeley Keview


Specializing in MCAT Preparation
Biology Metabolic Components Enzyne Mechanisms

His57

o
H ----------- -"o-'J- Asproz

I
Hll
H I
His57

)
/t-/' ]O O
o
ll
Ser195- O H- H ------------Y9- C- Asp1s2
^\z^-
Rr- C- O

J^
e ,! C'
Unstabte Tetrahedrat
Transition State

Figure 7- l5
The His-57 residue is now in a position to donate a hydrogen atom to the ser-L95
residue. Again, we have an electron shift. The transient tetrahedral intermed.iate
collapses and the acid component of the substrate that we started with is free to
leave the scene. Note that we now have the enzyme in its original starting
condition--ready to accept another substrate molecule for catalysis (Figure 7-16)-

His57
\
I
o
ser1e5-o----.-------
l--\o o ll
----
-q\zN- s -- o- c- AsP162
J I
"
R1- Q- g il
lll
oo' H I
llis57

o
Ser195- O- H
o
t *- H --------------- o-
ll
C- Asp162
!
R1- C- O
Acid Product
ilt
OH
d
' can leave the scene

Figure 7.16

In the analysis of this mechanism we looked at the hydrolysis of a generic


peptide bond. However, chymotrypsin is rather selective in the peptide bonds
that it will cleave. For example, chymotrypsin will hydroryze the peptide bond
on the carboxyl side of Phe, Tyr, and Trp, three aromatic amino acid residues.

Copyright @ by The Berkeley Review 120 The Berkeley Review


Specializing in MCAT Preparation
Biology Metabolic Components Enz5rme Mechanisms

chymotrypsin will also hydroryze on the carboxyr side of large hydrophobic


residues like Leu, Ile, and Met. It turns out that the side chains of these'amino
residues can fit into a special nonpolar pocket near the ser-195 residue
of the
active site. This pocket allows {or chymotrypsin,s specificity.

ffil#dtiv.$Ll .ilil"'l'i'ltltti , ,,,.,,,,,,, .,::.,


::r. :,ii,:i:i

During the early 1980s Thomas Cech and his colleagues discovered that
RNA
can act as a catalytic enzyme. In the ciliated protozoan Tetrahymena
thermophila, a
414 nucleotide intron is excised from a 6.4 kb ribosomal RNA
precursor in the
Dresence of a cofactor which proved to be a guanosine (G) residue.
The release of
ihis 414 nucleotide intron and subsequent splicing oi th" juxtaposed exons
;howed that an RNA molecule can have cataiytic a"ctivity. in" nsnucleotide
ntron undergoes two more rounds of self-splicing, first ltsing a 15 nucleotide
:ragment and then losing a 4 nucleotide fragment."The linear fiagment
of rRNA
rat is left is called L19 RNA. (The "L" stanJs for the fact that it is linear and the
19" means that 19 intervening nucleotide sequences were removed
since the
:crmation of the 414 nucleotidsintron. It is now 395 nucleotides long.)
L19 RNA
*' rather stable and can act on other substrates. As we will see later,
Ihis enzyme
*. both a nuclease and a polymerase.

InactiV $ $iFr$6ut$#C
'lany enzymes are initially synthesized in the form of a proenzyme or zymogen.
-'"-rI1ogens are inactiae
precurslrs of the active enzyme. hrorder ior these
- become active, they must first undergo "rrryi.r",
so*e type of proteolytic activation
"' rich is an irreversible
step). As an examprl, chymotrypsin.
"or,rid",
-- "-motrypsin is the qctiT)e form of the enzyme. The inactive form of this enzvme
-:e zymogen) is synthesized in the pancrlas and is caned chymotrypst"G;.
-r-:e.chymotrypsinogen is synthesiled in the pancreas it is secreted into the
r:-stinal tract where it is activated by a proteolytic enzyme called trypsin.
The
':::"'ated enzyme, chymotrypsin, can then hydroryze proteins found in the
"-:-:tine. If the pancreas were to become damaged, the pancreatic secretions
':ld rapidly become activated and digest the pancreas, giving rise to the
. :. :ition called acute pancreatitis.

-"
=:e is a naturally occurring substance called a1-antitrypsin which is an
:-mely effective. inhibitor of trypsin and an even better inhibitor of elastase
,,:;- called antielastase).Th1s inhibitor prevents these digestive enzymes
from
',:=sting the rest of your body. For exampre, antitrypsin iJpresent in the tissues
::. lung and prevents elastase from digesting connective iissue proteins in the
= - -ar walls of the lungs. Genetic disorderi leading to a deficiency in this
'"' :-tor can result in a clinical condition known as eiphysemq.
Aperson with
: :--,','sema breathes much harder than the normal individual. Individuals who
--i , r,:e cigarettes also have the possibility of developing emphysema.
It turns out
"I
-i{arette smoke oxidizes a specific Met residue on antitrypsin and converts
'nethionine sulfoxide. The Met residue is essential for bindLg elastase. Once
1: .s converted into methionine sulfoxide, elastase no longer binds to
'Lr
the
." ,:::tr.
Mlotr eculru,,,E#otutidfi
-:rer digestive enzymes that are quite similar to chymotrypsin are trypsin
r '' :lastase. These three enzymes are secreted by thep^rl"."ui and all have the
" *::rr O by The Berkeley Review t2t The Berkeley Review
Specializing in MCAT preparation
Biotogy Metabolic Components Enzyme Mechanisms

ser-His-Asp catalytic triad that we have been discussing. They are serine
proteases and their mechanisms of action are also similar. It turns out that about
40'/. of the overall amino acid sequences in these proteins are identical. why?
Because these three enzymes evolved (after mutation and duplication) from a
similar ancestral enzyme. The evolutionary process that allowed for these three
distinctive enzymes is called divergent evolution.

The bacterial enzyme subtilisin (isolated fuornBacillus subtilus) is also a serine


protease.If you compare the sequences of amino acids in subtilisin with those in
chymotrypsin (or trypsin or elastase), you will find a remarkably different
composition. These two enzymes (probably) did not have a common ancestral
enzyme and therefore were evolutionarily independent of each other. Even
though subtilisin is a serine protease (as are chymotrypsin, trypsin, and elastase),
the amino acid residues located at the active site of the enzyme are in
functionally different positions than the amino acid residues at the active sites of,
say, chymotrypsin or elastase. \A/hat this means is that both the bacterial enzyme
and the mammalian enzyme have found a similar way to catalyze a particular
reaction. These two independent processes are probably due to an evolutionary
process called convergent evolution.

ffi il$iti$rHii| $tfi tCIffi ffi fi t6g


Transition state analogs are synthesized with the hopes that they will occupy the
active site of an enzyme. The role of an enzyme is to bind the transition state and
stabilize it. A transiiion state analog is thus a competitiae inhibitor of the actual
reaction.

The vertebrate immune system's single most important function is to recognize


self from non-self. If a foreign substance (an antigen) enters into the body, the
immune system will recognize that substance as being foreign and begin to
synthesize antibodies or immunoglobulins against it. Suppose the foreign antigm
that entered the body was a transition state analog. The antibody synthesized in
response to the transition state analog will bind to that analog in a very specifi,c
way. This is exactly analogous to the way in which an enzyme would bind a
substrate. What we have done is to make the immune system produce an
"enzyme," which is the antibody. If we remove the transition state analog from,
the antibod/, what will be able to fit into the space left behind? How about
substrate or the product from the reaction in which the transition state
was competitively inhibiting? This would suggest that such antibodies w
have catalytic activity. It turns out that they do! These synthetic ant
"enzymes" are not necessarily as good as the naturally occurring enzymes,
they do have enzymatic activity. This might be a way to design and synthesi
your own protein(s).

Copyright @ by The Berkeley Review 122 The Berkeley


Specializing in MCAT
Biology Metabolic Components Metabolic Molecules

\Ietabolism is the generalized word for all of the processes which occur inside
-:r'ing organisms. Metabolism can be divided into catabolism and, anabolism.
catabolism means the breakdown of complex molecules into smaller and Complex
Complex "food"
simpler products accompanied by the release of. energy.In most instances the
biomolecules molecules
:ody attempts to harness this energy in the form of adenosine triphosphate
-LTP). Anabolism means to build up or become more complex. It is the
:iosynthesis of small precursor molecules into larger and more complex ADP
+Pi
I
:-olecules. Entropy is reduced in this energy-requiring organizational process.
l\-e can diagram these two metabolic processes as shown in Figure 7-17. \il"
I l3
Adenosine Triphosp62f,g,,,,(ATP )
Il8
lll-".
'",-e
"
will first concern ourselves with catabolic processes and then after that we
rll look at selective anabolic processes. In order to begin our discussion of ATP / ll.
,::abolism, we need to look at the chemical properties of ATp. As we will see, u
{'TP is the cunency exchange in the cell. A good analogy for ATp are dollar bills; Simple Less
--u make them and you spend them in the course of a lifetime. nutrient complex
materials "waste"
\TP is composed of adenine (a base), D-ribose (a sugar), and three phosphate products
.:,rups. Adenine and D-ribose taken together make up the molecule adenosine. Figure 7.17
,:-,',-e add one phosphate to adenosine, we get adenosine monophosphate (AMp). Catabolism and anabolism are
:'"',-e add two phosphates to adenosine, we get adenosine diphosphate (ADp). the components of metabolism.
'--,d if we add three phosphates to adenosine, we get adenosine triphosphate
\T?). Note the numbering of the two rings shown in Figure 7-18. The adenine
** : system (called a purine ring)
has the numbers 1 though 9 as shown. The
-:."::se ring has
the numbers 1' through 5' (the "prime" markings are important).

AcidI anh

,.r
Lnhydridde
tiinka5
kages

I
e{l e{i o
ilil5',
P- O--,P-
-r -P-O -P- -.P - O_ CH 2tO Adenosine is just
,o oJ -t)o
^t Jo)Jfi
LJ n
)
Adenine & D-Ribose

rosphonmonoester
Phos mon
linkkase
(age

ADP ,

ATP
ilfltiumure
7-IB
,.i*jr. :-ne Triphosphate (ATP)

I li" *ght @ by The Berkeley Review 123 The Berkeley Review


I Specializing in MCAT Preparation
Biology Metabolic Components Dnzyrme Kinetics

The linkage between the C1' of the ribose ring and the N9 of the adenine ring is
an N-acetal linkage. It is also referred to as an N-glycosidic linkage. The linkage
between the C5' of the ribose ring and the first phosphate group is a phospho-
monoester linkage. The linkages between the first and the second phosphates
oo
ilil and the second and third phosphates are phosphoric anhydride linkages.
-

HjC- C- O- C- CH3
Acetic Anhydride A similar situation stems from organic chemistry where you may have
encountered acetic anhydride. Aceti<- anhydride is the anhydride between two
4t
+H.O ll -H"O molecules of acetic acid as shown in Figure 7-19.
'I
I
Both esters and anhydrides can be hydrolyzed with water. However, anhydride
o bonds hydroryze in a manner that releases more energy than hydrotysis of ester
il
2 H3C-C- OH bonds. If you were to hydrolyze acetic anhydride, the equilibrium constant
Acetic Acid would favor the acetic acid products more than you would have expected had
you made it analogous to an ester where you would get back an acetic acid and
Figure 7-19 an alcohol product. one of the reasons for this is that in the acid anhydride
The hydrolysis of acetic
situation the acyl groups do not have the option of becoming stabilized by
anhydride to two molecules of
resonance in their acid anion form. \Atrhen acetic anhydride is hydrolyzed you get
acetic acid.
two molecules of acetic acid which can be stabilized by resonance. This
stabilization of both molecules of (product) acid probably leads to this large
release of energy that we have mentioned.

ffipffiGfi$ iiilffi:i:ffi# si,rl,


Enzymes often require substances other than amino acids to carry out tl
functions. These substances are called cofactors. Consider an enzyme with
active site. This active site is isolated from the rest of the enzyme and within
cavity are lrydrophobic side chains. If you have a substrate (s) which is in solu
it tells you that it must have some degree of polarity-otherwise it would not
soluble in that solution. There are a number of reactions that must be carried
in an anhydrozs condition (without water). Suppose you wanted to do
to the substrate that required a non-polar environment. one way to achieve
situation in a cell that is full of water is to get the substrate to settle down within
the pocket of the active site (which is surrounded by hydrophobic residues). If
this happens, the substrate can be converted into product.

once the substrate settles down at the active site, what will happen? Recall that
we have a "tool kit" of 20 amino acids. Some of these amino acids are acidic (they
can catalyze a reaction by donating a proton) while others are basic (they can
accept a proton). These types of amino acids are often involved in acid/ba
catalysis. There are also amino acids which can be oxidized or reduced li
cysteine. There are also amino acids with hydrophobic side chains which
good for excluding polar compounds such as water.

There may be other things that you would like to do to this substrate--things
cannot be done with our 20 amino acid tool kit. For this you need
additional apparatus at the active site. In hemoglobin that additional app
was the heme prosthetic group. other prosthetic groups are necessary for simi
reasons. You can do things with these prosthetic groups that you cannot do w
any of the 20 amino acid side chains. Therefore, a prosthetic group at the acti
site may be necessary to achieve some particular type of catalytic process.
are a whole series of prosthetic groups that need not be bond in a cov
manner at the active site. These are the cofactors that we just mentioned. If
cofactors are required by an enzyme, they are called coenzymes. A coenzyme is

Copyright @ by The Berkeley Review 124 The Berkeley


Specializing in MCAT
Biology Metabolic Components Metabolic Molecules

, :n-amino acid, non-polypeptide, non-protein substance which is required for


-:-e activity of an enzyme. In many
cases the coenzyme is bound to the enzyme in
; :.on-covalent manner but there are also many cases in which the coenzyme is
: --"'alently attached. There also may be more than one coenzyme involved with a
:i-'gle enzyme.

lTicotilild i d';Huffi tfi €Diffi il;GlEutiffi lltil.ffi il


NAD
Business end
o
il
C
-:re coenzyme that will become very important to us is nicotinamide adenine
:.:rtucleotide (NAD) whose structure is shown in Figure 7-20. Note the reactive o 'N
^

p
-.:t or "business end" of the molecule.
" n - 'J - o-cH,
" I

,1ere does "dinucleotide" in NAD come from? The phosphorylated adenosine


::':etv of NAD is one nucleotide while the other phosphorylated moiety (the HO OH
*-:.'se and the
pyridine ring) of NAD is the other nucleotide. The "businesi end"
: \AD contains a pyridine ring linked to an amide. This is called nicotinamide
,: niacinamide). If we add a carboxylic acid function to the pyridine ring, we
" - have nicotinic acid. This is also referred to as niacin. why call it niacini

-: =re is a carboxylic acid function in the structure of nicotinic acid, but what
, : - rt the "nicotin" prefix? Tobacco was brought back from the New world to the
o
- : ivorld by a Frenchman by the name of Jean Nicot. The tobacco plant was
" .:.ed Nicotiqna tqbscum after Nicot in his honor. Nicotiana contains a iubstance
' '--ti nicotine. If you were to oxidize nicotine with HNo3, you would get
-:Cnic acid.
Figure 7-2O
Nicotinamide Adenine
Dinucleotide (NAD)

NHz
N

O O"-"" O'
I

CH:

Pyridine Nicotinic acid Nicotinamide


(or niacin) (or niacinamide)

'\,E:rre 7-21
.-. .:n-based ring structures.

- :.ans have the ability to synthesize the nicotinamide (or nicotinic acid)
::-rn of the NAD molecule provided they have enough of the amino acid
::",:tophan in their diet. Tryptophan is an essential amino acid. Once we have
': :;ced nicotinamide the rest of the NAD molecule can be synthesized. If we
,.: : diet low in tryptophan, we would not be able to synthesize as much
- :--:,amide and therefore would have a deficiency in the NAD coenzyme. Since
.:ramide is an amine and it is "vital" to the diet, we would call it a "vital
- -:=." Today these are known as "vitamins" (from "vital amine"). If we do not
- = enough of the NAD coenzyme to carry out our metabolic reactions, then we
-- :.ot function as well. This is the basis of a vitamin deficiency.

:,:r,s out that nicotinamide was one of the first vitamins ever discovered.
'.- *:.d the same time that this vitamin was being discovered, it was also
, :- -'-i:rced that the nicotine in cigarette smoking was bad for you. The general
": : had a rather hard time distinguishing between the words nicotine and
.namide. Nutritionists decided that the name "nicotinamide" had to be

::sht @ by The Berkeley Review 125 The Berkeley Review


Specializing in MCAT Preparation
Biology Metabolic Components Enzyme Kinetics

changed in order to make consumption of nicotinamide more psychologically


acceptable. So, instead of calling it nicotinamide, the same substance was
renamed niacinamide. Nicotinic acid was renamed niacin (Figure 7-2L)

The coenzyme NAD is held at a specific site on the surface of the active site of an
enzyme. NAD is held there by its R group (the adenine dinucleotide portion).
When you use a screwdriver the part that engages the screw is just the head of
the screwdriver itself. Without the handle the screwdriver would be useless. The
enzyme is holding the NAD coenzyme by its handle (the R group portion which
has negative and positive charges) and positions the nicotinamide portion

o toward the incoming substrate. This can be seen inFigweT-22.

iffiHiffiiii ilffffiiiiiffiffiiffiffiftfiffiliiiift,H#ffiil,,:,,,,',,,,,,,,,,',r,,',' f
O In Figure riboflavin (vitamin Bz), a coenzyme
7-23a we see the structure of
Substrate component. We cannot synthesize riboflavin. Instead, we must obtain it
Figure 7-22 elsewhere (e.g., our diet). Riboflavin is a component in the structure of a
Enzy mel co enzyme i nteracti on molecule called flaain mononucleoflde (FMN), which is shown in Figure 7-23b-
at the business end of NAD. Riboflavin has been phosphorylated to give FMN, the active coenzyme. Thc
bond involving the phosphate and oxygen atoms in FMN is a phosphate
monoester bond. Phosphorylation of riboflavin has increased the size of thc
"handle" that we mentioned concerning NAD. FMN is another type of enzFrrn
that is often involved in redox reactions.

o o
il
u^c2'Y** "- " ^'-2'{*\A ^- "
",.-lo-[ A
VNNLo
I
":.\,,1|,.4-,to I

H_C-H
"-f-"
H- C- OH
I

H- C- OH
(a) | (b) r

H- C- OH H_ C- OH
I I

H_ C. OH H_ C_ OH
I a_
cH2-oH H2C- O - PO3-

Riboflavin Flavin Mononucleotide


o
il
,.r2-{*$n- "
","-!,ll-*A*!o NHt

#r
I
H-C_H
I
(c) H- c- oH
I

H- C- OH
I

H-C-OH o o CH"
tllll
cH2-o-l-o-,t-o
,/i
H
oo oo
Flavin Adenine Dinucleotide

Figure 7-25
The Flavin Derivatives

Copyright @ by The Berkeley Review t26. The Berkeley


Specializing in MCAT Pre
Biology Metabolic Qomponents Metabolic Molecules

If we were to add a phosphate, ribose ring, and an adenine base to FMN, we


-.r'ould get the coenzyme
flaain adenine dinucleotide (FAD). This is shown in
Figure 7-23c. The flaain portion of FMN and FAD is the collection of three rings
:t the top of the molecule. Recall that we have mentioned that when you connect
: base (in this case, adenine), a ribose ring, and a phosphate together, it is
:eferred to as a nucleotide. The FMN molecule itself looks a bit like a nucleotide.
lhus, FMN is called flavin mononucleotide while FAD is called flavin adenine
:rnucleotide.

Coenzyme A (GoA) ,i: ,,,,,, , ,,,i,,i,,;,

--nother coenzyme that will be important to us is Coenzyme A (often abbreviated


-oA). The terminal sulfhydryl group is the active site of CoA. The rest of the
: rlecule can be thought of as being like a handle, similar to the handle on a
.::ervdriver. This is shown in Figure 7-24.

{.cvl groups can be linked to the sulfhydryl function to form acyl coenzyme A
:: acyl CoA). A common acyl group that is linked to CoA is an acetyl moiety,
1" :-. giving acetyl CoA. Because the AGo'for acetyl CoA is -7.5Kcals/mol, this

r . -ecule has a high acetyl group transfer potential. In other words, acetyl groups
r,,:- :e easily transferred from acetyl CoA to other molecules.

:*':der CoA as shown in Figure 7-24 for a moment. where did the components
, lr-{ come from? If we hydrolyze the amide bond towards the sulfhydryl
*: ::.rn, and then carboxylate the CH2 moiety next to the nitrogen, we will have
''' sieir€. If we hydrolyze the second
amide bond in from the sulfhydryl function,
ii- r --r.in carboxylate the CH2 moiety next to the nitrogen, we will have aspartic
:: Ii rve hydrolyze the first phosphoester bond closest to the sulfhydryl
,rrrL

: :i -::':'r. we find a segment that is synthesized fuorrr three acetate residues. The
" r;-: ler of the molecule is just resembles ADP.

TN,
N4-
't N'
coenzyme A
-JL
)
Ic- 9?"t", I o- I o- T
|1- .": - cHz - N- cHz- cHz- N- c-
i- - f - o- cH2 P- P-
"fZo..
{ H H HCH: Oo oo KfJ }
I

I-, :.i
-.: ':'!rru
end" /
" "
-er..'ed from from
Derived Derived fiom o- oH
AI
:; s:eine by aspartic acid by 3 acetates "o-P=o
-)\-
u;- :oxylation decarboxylation (many -)
steps)

. :," The Berkeley Review t27 The Berkeley Review


Specializing in MCAT Preparation
Metabolic
Components

15 Passages

100 Questions

Passage Titles Questions


I. Enzyme Inhibitors 1-5
II. Enzymes, Coenzymes, and Vitamins 6- 11
IrI. Plasma Glucose Measurement 12-11
IV. Brown Adipose Tissue ]8-23
V. Heme Metabolism 24-29
VI. Enzyme Kinetics I 30-36
VII. Vitamin Brz 37-43
VIII. Enzyme Nomenclature 44-50
IX. Lysozyme Mechanism 5l-58
X. Enzyme Kinetics II 59-65
XI. Adenosine Triphosphate (ATP) 66-12
XII. ProteinDegradation(Ubiquitin) 13-19
XIII. Nutrient and Proximate Analysis 80-87
XI\,'. Niacin Experiment 88-94
XV. Diffusion-Limited, Enzyme-Catalyzed Reactions 95 - 100
Suggestions
The passages that follow are designed to get you to think in a conceptual manner about the Processes
of moleclhr biology at the organismal level. If you already have a solid foundation in molecular biology,
many of the questionr yo, r"ud here will seem to be very straight forward and easy to answer. But if you
,r" ,r"* to the subject or if you have not had a pleasant experience with molecular biology in the past,
some of them might io come from the void that spreads out beyond the Oort field at the edges of
our solar system.
"pp""r

pick a few passage topics at random. For these initial few passages, do not worry about the time. Just
focus on what is exp6cted of you. First, read the passage. Second, look at any diagrams, charts, or graphs
in it. Third, read eich question and the accompanying answers carefully. Fourth, answer the questions
the best you can. Checkthe solutions and see how you did. \Atrhether you got the answers right or wrong,
it is important to read the explanations and see if you understand (and agree with) what is being
explained. Keep a record of your results.

After you feel comfortable with the format of those initial few passages, pick another block of
passages and try to do them in one sitting. Be aware that time is going to become important. On average,
yo.t nlve abouf 1 minute and 15 seconds to complete a question. Be creative in how you aPProach this
next group. If you feel comfortable with the outline presented above, fine. If not, then try different
appro-achei to a passage. For example, you might feel well versed enough to read the questions first and-
then try to answ-er sorie of them, without ever having read the Passage. Maybe you can answer some of
the questions by just looking at the diagrams, charts, or graphs that are presented in a particular passag€.
Rem-ember, there are many effective leaming styles. You need to begin to develop a format that works
best for you. Keep a record of your results.

The last block of passages might contain at least a few topics that are unfamiliar even to those who
know a good deal about molecular biology. Find a place where the level of distraction is at a minimum'
Get out your watch and time yourself on these passages, either individually or as a grouP. It is important
to have a feel for time, and an awareness of how much is passing as you try to answer each question.
Never let a question get you flustered. If you cannot figure out what the answer is from information
, given to you in the pu"rlg", or from your own knowledge base, dump it and move on to the next
and come back to it when you have more
[uestion. As you do [his, make a note of that pesky question
time. When you are finished, check your answers and make sure you understand the solutions. Be
inquisitive. If you do not know the answer to something, look it up. The solution tends to stay with you
longer that way. (For example, what ls the Oort field, anyway?)

The estimated score conversions for 100 questions are shown below. At best, these are rough
approximations and should be used only to give one a feel for which ballpark they are sitting in.

Section Vtr
Dstimated Score Conversions
Scaled Score Raw Score
>13 80 - 100
tt-12 70 -'79
9- l0 60-69
'7 -8 50-59
5-6 40-49
<4 0-39
Biology Enz5nne Inhibitors Passage I

Passage I (Questions L-5) Competitive inhibitors resemble the substrate and bind
reversibly at the active site. They increase the Ky and can
In the case of a simple enzyme-catalyzed reaction, an be overcome at high [S]. Uncompetitive inhibitors bind
r:rzyme E binds a substrate S with a high degree of only to the enzyme-substrate complex. They decrease the
.tt-rnity, proceeds through a transition state involving the Kv and Vmar and cannot be overcome at high [S].
:nzyme-substrate complex ES, and converts that substrate Noncompetitive inhibitors bind to either the enzyme or
-rto a product P (or products). This type of reaction is enzyme-substrate complex. They decrease the Vmax and
:escribed by Equation 1:
cannot be overcome at high [S].

E+S ES k'' E+P Many cellular enzymes are composed of more than a
single polypeptide chain. Instead of having just tertiary
structure, these enzymes have quaternary structure and
The rate (velocity v) of this reaction is dependent not multiple binding sites that can cooperatively interact with
::-r' on the enzyme concentration [E], but also on the one another. Instead of having hyperbolic kinetics, these
, -bstrate concentration [S] in the cell. Initially, the enzymes show sigmoidal or S-shaped saturation kinetics.
':l,rcity of the reaction is directly proportional to [E].
,'{::abolic reactions in the cell occur at higher rates when
r,.:re enzyme is available. However, the rate of a reaction
:i :enerally determined by [S]. When [S] is low, the rate
- i : reaction increases quite rapidly in a linear fashion.
;:en [S] is high, the rate of the reaction begins to
i:,;:ease less rapidly until saturation of the enzyme with 1. The diagram shown below represents an interaction
L:e substrate is reached. At this point, the rate of the between enzyme (E), substrate (S), and inhibitor (I):
1:in:tion is maximal and is given the term V*u*. The
,l-:haelis constant, KM, is the [S] at which the initial
:. -'ciry of the reaction is one-half the maximal velocity. 4ffii
\
g dffi+,r
V
l:e ;urve developed from this type of saturation kinetics
/t
, :::at of a hyperbola (Figure 1):
lln
\',n*
1l^ I
zr-illll\ fal /f;Ill\
: rrtmax (::::::,i'ri',,:'l
=r!r- fi:..:...iin'iji:lfSl
=
I2
>
YI\-
\:,!!!!!!!!4!!! !!,,,t \::::iljillili::l::i:li-
=

Which of the following reversible enzyme inhibition


KM tsl graphs BEST represents the diagram shown above?
[Note; Curve C represents the control.]
Figure 1 Figure 2

ii : : tollows classical Michaelis-Menten kinetics


B.
,,V,/
in Equation (2):
/Y4
,:.,r;T--ised

- ..1 _ V.u* [S]


- Kn,,.LSl 1
0 1/tsl

-.-:ce it is difficult to measure V6a1 irccurstely from a


l.::i:: plot of v as a function of [S], the reciprocal of
i;i l[-:on (2) is used to generate a straight line of the form D.
: :x + b. A double reciprocal plot (Figure 2) allows for l/vl /
iL :r :rr3 accurate

,iur,:"1",
l:e
determination of both Vmar and Kvt.

inhibition of an enzyme-catalyzed reaction can be


led in terms of Michaelis-Menten kinetics. The three
l4sl
-% 0
rn I'n'i :-r' types of reversible inhibition are competitive, l/tsl
rliilL :n p€titive, and noncompetitive.

lry',:r,sht @ by The Berkeley Review l3r The Berkeley Review


Specializing in MCAT Preparation
Biology Enzyme lnhibitors Passage I

2. Which of the following is NOT a characreristic of 5. The Krebs cycle requires eight enzymatic steps, and
the Michaelis constant (Ky)? in eukaryotic cells it occurs in the matrix of the
mitochondrion. One rate-controlling step involves
A. It is the substrate concentration atYmw<12. the conversion of isocitrate to cr-ketoglutarate:
B. Itcharacterizes substrate-enzymeinteraction.
C. It is equal toYmu</2. .3
D. It is not altered by noncompetitive inhibitors.
Isocitrate + NAD+1excesr,

cx-Ketoglutarate + NADH + H+ + CO2

Three reaction mixtures are examined:


3. What is the initial velocity of a reaction catalyzed by
the enzyme urease, if the concentration of urea in Mixture 1: Substrate andNAD+.
solution is 2.5 x 10-1KM? Mixture 2: Substrate, NAD+, and 2 mM ADP.
Mixture 3: Substrate, NAD+, and 5 mM ADP.
A. 0.20 vmar,(
B. 0.25 VmarK The initial reaction velocity (v) as a function
c. 0.75 VmaK substrate concentration [Sl of each mixture is
D. 0.80 Vmax plotted. Based on this analysis one might conc
that:

4. Three separate enzyme-catalyzed reactions are run


with the same substrate and enzyme at a constant >|
enzyme concentration. The only difference between .n
the three reactions is the type of inhibitor used o
during catalysis.

Reaction 1: No inhibitor added.


Reaction 2: Competitive inhibitor added in Substrate concentration
increasing concentrations.
Reaction 3: Noncompetitive inhibitor added in A. ADP increases the Ku of the enzyme.
increasing concentrations. B. ADP is an allosteric inhibitor of the enzyme.
C. a high [NADH]/[NAD+] ratio stimulates
The data from each reaction was plotted on the enzyme.
graph shown below:
D. a high [ATP]/[ADP] ratio inhibits the

A l4sl

The point that is common to both Reaction 1 and


Reaction 2 is:

A. Point A.
B. Point B.
C. Point C.
D. Point D.

Copyright @ by The Berkeley Review 132 The Berkeley


Specializing in MCAT
Biology EnzSrmes, Coenz;imes, and Vitamins Passage tr

Pa"ssage II (Questions 6-11)

){icotinic acid is also referred to as niacin. The amide


: nicotinic acid is called nicotinamide, a component of
:- coenzymes nicotinamide adenine dinucleotide (NAD)
.-i nicotinamide adenine dinucleotide phosphate
l;.\DP). NAD and NADP are loosely bound to a class of
:-,zvmes called dehydrogenases and can easily move H-dV,NH
'::,n one enzyme to another. They are involved in the
:.::sfer of hydrogens and electrons in oxidation-reduction
' :.;tiOns.
e
nslo
The only oxidation step in glycolysis is catalyzed by a
\r"5- rj_no
ll^
tt-b-o"
:-':r'drogenase enzyme. The proposed mechanism for this 60
':.;tion is outlined in Figures 1-4:
Product t"ro-]
Figure 4

\/v
N,Lo
Zbn A niacin deficiency leads to the disease pellagra, a
condition characterized by dementia, dermatitis, and
dianhea. Meat, fish, beans, and nuts are good sources of
ll^
HrN H -uL niacin. Very little niacin is found in either milk or eggs.

; r "-d\r*" Riboflavin (vitamin Bz) has a complex isoalloxazine


.z----\&o
s.' ring system showing conjugation of multiple bonds
." I
H_C_OH (Figure 5). In general, the more conjugated bonds a
rSubstrate molecule has, the longer the wavelength at which the
cHroPo;
molecule absorbs light.
Figure 1

Oxidized

:,.n:*" R
isoalloxazine
ring

ll ,r* * ,.-
u

:"gltr"l
Reduced
isoalloxazine
ring

RH

Figure 5

Riboflavin is a component of two closely related


coenzymes, flavin adenine dinucleotide (FAD) and flavin
}.T{D mononucleotide (FMN). These coenzymes are covalently
H- -H bound to a class of enzymes called dehydrogenases, and

"r,l ..9F they participate in the transfer of hydrogens and electrons


in oxidation-reduction reactions.

A vitamin Bz deficiency is rare, but it can occur during


pregnancy, in growing children, and during times of
stress. If a deficiency does occur, it usually occurs with
pellagra. Milk, eggs, liver, and meat are good sources of
Figure 3 vitamin Bz.

:';nght @ by The Berkeley Review 133 The Berkeley Review


Specializing in MCAT Preparation
Biology Enrymes, CoenzSmes, and Vitamins Passage Il

6. Electrons on the nitrogen atom of the imidazolium 11. In solution, succinate is oxidized to fumarate by the
ring of histidine (His) in Figure 2 ultimately end up enzyme succinate dehydrogenase.
on the:

o FAD o
A. carbonyl oxygen of nicotinamide. coo FADH2 coo
B. nitrogen of the nicotinamide ring. I I
CH" CH
c. sulfur atom of cysteine (Cys). t' il
CH" Succinate CH
D. carbonyl oxygen of the substrate. l'g
dehydrogenase
l6
coo coo
Succinate Fumarate

7. What is the name of the linkage at Q in Figure 4? The dehydrogenase enzyme contains a covalently
bound prosthetic group, flavin adenine dinucleotide
A. Phosphodiester
(FAD). Oxidized FAD absorbs light in the visible
B. Peptide region of the electromagnetic spectrum at 460 nm-
C. Mixed acid anhydride Based on the reaction shown above and the graph
D. Amide shown below, the color change during the course d
the reaction is from:

Blue Green Red


cone cone cone

8. The sulftrydryl group of Cys loses its hydrogen atom


between Figure I and Figure 2. This is best
explained by the need for the sulfhydryl group to
i1 X
become:
Pk
6O
A. a nucleophile, attack the aldehyde carbon of
boo
the substrate, and form an acyl thioester bond. '-o
Frb
B. an electrophile, attack the aldehyde carbon of a
the substrate, and form a hemiacetal.
C. a nucleophile, attack the aldehyde carbon of 400 500 600
the substrate, and form a hemiacetal. Wavelength (nm)
D. an electrophile, attack the aldehyde carbon of
the substrate, and form an acyl thioester bond. A. blue to red.
B. yellow to colorless.
c. yellow to blue.
D. colorless to red.

9, The BEST dietary source for precursors to the


dehydrogenise enzyme would be:

A. eggs.
B. fish.
C. meat.
D. beans.

10. Al1 of the following oxidation-reduction pairs could


be found at the active site of a dehydrogenase
enzyme, EXCEPT:

A. reduced NAD; reduced cysteine.


B. reduced NAD; reduced product.
C. oxidized NAD; reduced cysteine.
D. oxidized NAD; oxidized cysteine.

Copyright @ by The Berkeley Review 134


Biology Plasma Glucose Measurement Passage III

Passage III (Questions 12-17) 12. Which of these is the structure of glucono-6-lactone,
a product of Reaction t?
Researchers often need to measure plasma glucose
:'"els for reports on clinical experiments. A plasma A. B.
.:-,.:cose analyzer is a machine containing reagents and
o.\
_o
0t'a o
:lzvmes that allows rapid analysis of plasma glucose -c- H -
:',e1s. A blood sample is introduced into the analyzer,
.:J a probe determines the glucose concentration. The
I

H_C-OH *- i- o'
I
, *-icture of the probe is shown in Figure 1: HO- C- H HO-C- H
I
I
H-C-OH H- C- OH
I
H-C-OH H- C- OH
Platinum
I

Reaction 2 cH2oH 67c- oo

Reaction C. D.

CH.OH
t'
"""'YU:"" )-o

lmmobilized o
"J(f1>" OH
d
Oxidase

U)
13. The glucose analyzer could be used to follow
o-Ri ng changes in plasma glucose over time. Which of the
following graphs indicates how a person's blood
glucose would change over the three hours
Figure 1 following a meal?

The first reaction occurs on the membrane containing A. B.


:-:obilized glucose oxidase. Glucose oxidase catalyzes
": : -ollowing reaction:
o o
:-D-Glucose *02 + Glucono-d-lactone +H2 O o
o o

Reaction I

Tre second reaction occurs at the platinum electrode


.-:: :s shown below: 01234 01234
Time Fotrlowing A Meal Time Following A Meal
(hours)
Oo (hours)
HzOz 2H + 02 + 2e
c. D.
- Reaction 2

.i thin cellulose filter blocks chemical compounds c) C)

o
,' ,:. molecular weights above 200 between Reaction I
.": : Reaction2. The electrons produced at the electrode rFl

i': ,:nearly proportional to the H2O2 concentration, and


-:'r::ore to the concqntration of the substrate. The current
':: --red on the silver electrode to keep the electrode pair
::-::r1 is recorded as electron flow. Electron flow is
01234 01234
Time Following A Meal Time Following A Meal
*::;ied and reported by the probe to a computer, which (hours) (hours)
r.-:::.:i-ies and converts the electrical signal into a plasma
: -: rse concentration.

; - .:. rght @ by The Berkeley Review 135 The Berkeley Review


n Specializing in MCAT Preparation
BioIo gv Plasma Glucose Measurement Passage Itr

14. The user of the glucose analyzer first generates a 17. The function of the platinum electrode in this probe
standard curve like the one shown below. The is to act as:
computer then has an internal reference for
interpolating the actual glucose samples from the A. a cathode.
subject. If the electron reading were 1250, what B. an anode.
glucose concentration would be displayed on the C. an oxidizing agent.
computer screen ofthe glucose analyzer ? D. an oxidase.

bo


o

o
!
o
C)
g.l

60 80 100 120 140 160 180 200


Blood Glucose (mg/dl)

A. 110 mg/dl
B. 220mgldL
C. 330 mg/dl
D. 440mgldL

15. How would the measurement of the plasma glucose


concentration change, if a technician accidentally
added hydrogen peroxide to the plasma sample
instead of saline buffer?

A. The glucose reading would be erroneously


high.
B. The glucose reading would not be changed by
this addition.
C. The glucose reading would be erroneously
low.
D. The glucose reading would be zero, because
the hydrogen peroxide would destroy the
glucose.

16. What is the purpose of the cellulose filter between


the immobilized oxidase and the platinum electrode?

A. The filter blocks hydrogen peroxide.


B. The cellulose allows passage of many
oxidizabl'e substances, so that the readings are
accurate.
C. The cellulose blocks many other oxidizable
substances, so that the readings are accurate.
D. The filter protects the immobilized enzyme.

Copyright @ by The Berkeley Review r36 The Berkeley


Specializing in MCAT
Biology Brown Adipose Tissue Passage IV

Passage IV (Questions 18-23) 19. The most abundant adipose tissue is called "white,"
although in humans and many other animals, it is
White adipose tissue (WAT) is the most common fat actually yellow, due to stored carotenoids. What
storage medium in adults and children. The small provides the brown color in brown adipose tissue
amounts of brown adipose tissue (BAT) between the (BArX
shoulder blades and along the nape of the neck of
newborn infants is metabolically important for their A. Ribosomes in the cytosol
survival. B. DNA in the nucleus
C. Cytochromes in the mitochondria
The primary difference between the two tissues is that D. Bacteria in the lysosomes
tsAT has several times the mitochondria of WAT. Also,
the inner mitochondrial membranes in BAT are more
permeable to protons than those of WAT. The reactions of
:ridative phosphorylation, particularly the generation of
{TP QV ATP synthetase, depend on an inner 20. Norepinephrine first binds to the beta-adrenergic
::tochondrial membrane that is impermeable to protons. receptors and then acts through a second messenger
--, the case of BAT, this means that oxidative system to cause changes inside the adipocyte. This
::osphorylation is uncoupled, so that fewer ATP sequence of events is represented in the following
:rllecules are made when protons are pumped out of the diagram:
:ritochondria. This uncoupling is caused by a protein
:Aled thermogenin that is present in the inner membrane.
l:iermogenin allows the passage of protons back into the
-rtochondrial matrix, bypassing the ATP synthetase. This Hormone Act'ivated Extracellular
.rcoupling produces beneficial effects for the infant. o receptor

The adipocytes in BAT are controlled by extensive


i-"mpathetic enervation. When stimulated, these nerves
-:1ease norepinephrine directly at the adipocytes.
\rrepinephrine stimulates lipolysis and oxidation of the
lerated fatty acids. Through oxidation, both NADH +
:--: and FADHz are produced. These give rise to the cAMP
::rtons that are pumped across the mitochondrial ATP
::,embrane. The intracellular metabolic pathways and their
::izvmes are not significantly different between BAT and G protein
Intracellular
A'AT. GTP

In the diagram shown above, which of the following


intracellular events follows the binding of
norepinephrine to the p-adrenergic receptor?

A. The hormone is transported into the cytosol.


B. The production ofthe By subunit is increased.
C. Adenylate cyclase is inhibited.
IE. Which of the following would result from the D. Concentrations of cAMP increase.
uncoupling of fat oxidation and ATP production?

A. The production of heat


B. A local cooling effect
C. Conservation of body fuel stores 21. What is the principal storage component of an
D. A lowered resting metabolic rate adipose cell in white adipose tissue?

A. Fatty acids
B. Triglycerides
C. Phospholipids
D. Glycerol

- :n1'right @ by The Berkeley Review 137 The Berkeley Review


Specializing in MCAT Preparation
Biotogy Brown Adipose Tissue Passage IV

22. What role can BAT play in glucose homeostasis for


the whole body?

A. BAT provides glycerol for gluconeogenesis.


B. BAT provides glucose for gluconeogenesis.
C. BAT contains glycogen for glycogenolysis.
D. BAT provides fatty acids for gluconeogenesis.

23. Thermogenin, the uncoupling protein, is probably


MOST similar in structure to:

A. hemoglobin.
B. chymotrypsinogen.
C. ATP-ADP synthetase.
D. Nao-Ko pump.

Copyright @ by The Berkeley Review r38 The Berkeley


Specializing in MCAT
Biology Ileme Metabolism Passage V

Passage V (Questions 24-29) 25. The conversion of biliverdin to bilirubin can be


described as a:
The ability to transport oxygen throughout the body
results from the oxygen-binding capability of hemoglobin A. one-electronoxidation.
Hb). In particular, it is the heme prosthetic group of Hb B. one-electronreduction.
;:rat carries out this essential function. The prosthetic C. two-electronoxidation.
:roup consists of a porphyrin ring bound to a central iron D. two-electronreduction.
::om, which in turn directly binds oxygen.

In a disease of porphyrin anabolism, the enzyme 26. In the absence of uroporphyrinogen synthase,
-:oporphyrinogen synthase is deficient and a non- uroporphyrinogen I (uro I) is synthesized in place of
"-nctional porphyrin skeleton is formed. As a result, the biologically active uroporphyrinogen III (uro
::egular erythrocytes are made and eliminated. This m).
:-sease, congenital erythropoietic porphyria (CEP), is o
coo
::lieved to be transmitted in an autosomal recessive
..hion.

The breakdown of the heme prosthetic group produces


,--.e end product bilirubin, and the first step involves the
:.:avage of an alpha-methane bridge to form biliverdin.
l:rs reaction involves the release of carbon monoxide.
:-lrverdin is then converted into bilirubin, a reaction
::lined in Figure 1. In the liver, bilirubin is attached to
' r glucuronate residues, and the complex is secreted into
.--: is released along with bile. Studies show that bilirubin
, an anti-oxidant and has the ability to eliminate
" :roperoxy radicals. For this reason, it is postulated that
: :lution has selected bilirubin as the end product of
".:,e degradation.

o
coo

NADPH
NADP+
H+ +
'.Bilirubin
Biliverdin \ N
H
NH HN
H
Figure 1 N

ol
ooc

Conversion of uro I into uro III requires:

-j The breakdown of heme to bilirubin takes place in


A. an isomerase, converting a symmetric
molecule into an asymmetric molecule.
-:1e:
B. an isomerase, converting an asymmetric
molecule into an asymmetric molecule.
.\. liver.
C. a kinase, converting a symmetric molecule into
B. spleen.
an asymmetric molecule.
C, pancreas.
D. a kinase, converting an asymmetric molecule
D. gall bladder.
into a symmetric molecule.

,r.
-_:ht @ by The Berkeley Review 159 The Berkeley Review
Specializing in MCAT Preparation
Biology Ileme Metabolism Passage V

27. Which of the following statements is TRUE


regarding the heme prosthetic group?

A. Carbon monoxide has a greater affinity for Hb


than for an isolated heme group.
B. Oxygen has a greater affinity for Hb than does
carbon monoxide.
C. A small percentage of Hb can be expected to
be bound to carbon monoxide.
D. Hb is known to exhibit a noncooperative
binding mechanism,

28. Based on information in the passage, which of the


following statements is FALSE?

A. Persons suffering from CEP should exhibit a


decreased blood viscosity.
B. Persons suffering from CEP exhibit a retarded
oxygen transport system.
C. Persons suffering from CEP exhibit a lowered
arterial pO2.
D. Persons suffering from CEP should exhibit a
decreased resistance to blood flow.

29. A man heterozygous for CEP has a child with a


woman who is homozygous dominant for CEP.
What is the probability that their first son will show
the disease phenotype?

A. jVo
B. 25Vo
C. 50Vo
D. IOOVo

Copyright @ by The Berkeley Review 140 The Berkeley


Specializing in MCAT
Biology Enryme Kinetics I Practice P2ssage VI

Passage VI (Questions 30-36) Enzymes can also undergo one of three types of
reversible inhibition. Competitive inhibitors resemble the
In 1913 Leonor Michaelis and Maude Menten substrate and bind to the active site of the enzyme, thus
proposed that reactions catalyzed by enzymes proceed in preventing the substrate from binding to that active site.
two steps. First, an enzyme (E) rapidly binds a substrate The initial tendency of lowering the velocity at which the
(S) to form an enzyme-substrate complex (ES). Second, enzyme converts substrate to product can be overcome at
the enzyme-substrate complex is converted to product (P) high substrate concentrations. Noncompetitive inhibitors
in a rate-determining step. The rates of enzymatically can bind to either the free enzyme or the enzyme-
catalyzed reactions, in comparison to uncatalyzed substrate complex. This action also lowers the rate at
reactions, can easily exceed a factor of a million. Since an which substrate is converted to product. Uncompetitive
enzyme is a catalyst, it does not alter the equilibrium of a inhibitors bind only to the enzyme-substrate complex,
chemical reaction. thus lowering the rate at which substrate is converted to
product. Unlike competitive inhibition, uncompetitive
k1 k1 inhibition and noncompetitive inhibition cannot be
FrS-FS>ErP
Lrv- overcome at high substrate concentrations.
k2

Almost all known enzymes are proteins, and all of the


amino acid residues found in proteins are L-amino acids
rr (S)-amino acids. The exception is that L-cysteine is
designated as (R)-cysteine. The actions of enzymes can be 30. Trypsin is a digestive enzyme generally referred to
:egulated, and they offer a high degree of specificity and as a peptidase, because it hydrolyzes polypeptides.
:atalysis towards a substrate or reactant molecule. For The scissile peptide bond to be cleaved in the
:rample, enzymes readily degrade polypeptides polypeptide is on the carboxyl side of the amino
,r'nthesized from L-amino acids, but not from D-amino acid residues arginine (Arg) and lysine (Lys).
.:ids. Which of the following peptide bonds is cleaved by
trypsin?
When a substrate binds to the active site of an enzyme,
. usually does so by non-covalent interactions. Typical H O H^
:-nding forces involve hydrogen bonding, hydrophobic c)rrtOO
^HOH tl19
ooc- c- N - c-
rteractions, electrostatic interactions, and van der Waals
nrN-
f c- N- c- coo *",
?-
:teractions. The active sites of enzymes generally form fr, H cH3 H:c
" t",
:.efts or crevices in the molecule. Those crevices are lined
,i rth specific amino acid residues that help to stabilize the
i*' Alanine (Ala) CHz
I
CH" CH'
':rnsition state of the substrate and allow catalysis to II
:', I 1..'
LHr
::cur. Even though different enzymes can have l' t-
,:ecificities for different side chains next to the peptide @ NH3 @ NH3
:,:nd to be cleaved, they can have similar active sites. Lysine (Lys)

Enzymes like chymotrypsin, trypsin, elastase, and


HOCH. CH,OH
.- rtilisin are referred to as serine proteases, because they @rrr l' o o a' ir ro
-.','e a highly reactive serine residue at their active site. n.N- c- N- c- coo ooc- c- N - c- c- NHj
..:e presence of active site serine residues can be
? ttl
HHCHU
-::ermined by using an irreversible inhibitor like ?t,HH I

cH" cHz
--.sopropylphosphofluoridate (DIPF) as a chemical label. t' I

?t' III IV f"'


(H1C)2HC- O-
i O- CH(CHt2
NH
I
Oc
NH
I
@
i- HzN
,caa
NHz
- ta-\
HuN NHz
o
DIPF Arginine (Arg)

DIPF reacts only with serine residues at the active site A. I only
' :orming a diisopropylphosphoryl complex with the B. II only
,-:',me. This irreversible inhibitor has a similar action on C. I and III
":..'.'lcholinesterase, a serine esterase involved in the D. IV only
::olysis of the neurotransmitter acetylcholine to acetate
-.: : ;holine.

!t :r right O by The Berkeley Review t4t The Berkeley Review


0n Specializing in MCAT Preparation
Biology Enzyme Kinetics I Practice Passage VI

31. The action of the nerve gas diisopropylphospho- 34. The lysozyme enzyme has two amino acid
fluoridate (DIPF) on acetylcholinesterase and residues, aspartate 52 and glutamate 35, at the
serine proteases depends upon: catalytic site with pK6 values of approximately 4.0
and 6.0, respectively. Ifthese are the only ionizable
A. allostericinhibition. groups involved in catalysis, then the velocity of
B. competitiveinhibition. the reaction varies with pH according to which
C. phosphorylation of ADP. graph?
D. phosphorylation of the active site.

o
o
o
32. In the following graph, if Curve I repr€sents the
reaction velocity (V) as a function of substrate
concentration (S) plot for an enzyme, then Curve II o
o
represents the reaction of the same substrate in the &
presence of a fixed concentration of:

V-"*

o
o
o

o
C)
&

A. an irreversible inhibitor.
B. a competitive inhibitor.
C. an uncompetitive inhibitor. o
D. a noncompetitive inhibitor. 0)

o
C)

33. Of the four compounds tested at the same &


concentration, the MOST effective competitive
inhibitor of the common enzyme is:

il
o
o

o
o
&

A. A. Graph I.
Compound I.
B. Graph II.
B. Compound II.
C. Graph III.
C. Compound III.
D. Graph IV.
D. Compound IV.

Copyright @ by The Berkeley Review 142 The Berkeley


Specializing in MCAT
Biology Enzyme Kinetics I Practice Passage VI

35. At a pH of about 5, which of the amino acid


residues in the active site of the lysozyme enzyme
provides the general base catalysis and which
provides the general acid catalysis?

A. Aspartate 52 provides the general acid


catalysis, because its side chain p-carboxyl
group is about 9lVo deprotonated at a pH of 5.
Glutamate 35 provides the general base
catalysis, because its side chain y-carboxyl
group is abott9l7o protonated at apH of5.
B. Glutamate 35 provides the general acid
catalysis, because its side chain 1-carboxyl
group is about9Vo deprotonated at a pH of 5.
Aspartate 52 provides the general base
catalysis, because its side chain B-carboxyl
group is about9Vo protonated at apH of5.
C. Aspartate 52 provides the general base
catalysis, because its side chain B-carboxyl
group is about9lVo protonated at a pH of 5.
Glutamate 35 provides the general acid
catalysis, because its side chain y-carboxyl
group is about 9Vo deprotonated at a pH of 5.
D. Glutamate 35 provides the general base
catalysis, because its side chain y-carboxyl
group is about9Vo deprotonated at a pH of 5.
Aspartate 52 provides the general acid
catalysis, because its side chain p-carboxyl
group is about9lVo deprotonated at apH of5.

According to the following graph, the Vmax of the


enzyme reaction is:

A. 3tz
8.2
c. rtz
D. 2tz

I ";r'nr,,:tght @ by The Berkeley Review 146 The Berkeley Keview


Specializing in MCAT Preparation
Biology Vitamin Bl z Passage Vtr

Passage VII (Questions 37-43) 37. Based on the reaction shown in Figure 1, what type
of enzyme is methylmalonyl-CoA mutase?
Vitamin Brz is a large cobalt-containing molecule that
has a ring structure similar to heme, with Co as the central A. Oxidoreductase
ion instead of Fe. Vitamin Brz participates in only two B. Hydrolase
known reactions in the human body, catalyzed by the C. Transferase
enzymes methylmalonyl-CoA mutase and homocysteine D. Isomerase
methyltransferase.

38. In the enzyme methionine synthetase, both Brz and


o Methylmalonyl-CoA o tetrahydrofolate (THF) are cofactors. If B rz is low,
coo coo then THF is trapped as methyl-THF at this enzyme
I mutase I
H-C_CH. CH" and is not recycled. This is called the methyl-TllF
I I
o=c-s-coA CH" trap. How can 812 status affect folate status?
.-l
Methylmalonyl CoA 'rC-S-CoA A. A primary B 12 deficiency can lead
Succinyl CoA secondary folate deficiency.
Figure 1
B. A primary folate deficiency can lead
secondary B12 deficiency.
C. Folate and Bn are independent of each other.
D. B 12 is required to synthesize folate.

coo
o Homocysteine
coo
o
to
H-p-NHj
methyltransferase t@
H_C_NH,
I

CH"
I 39. The production of IF often decreases with age. To
CH.
I I prevent B12 deficiency in older adults, which of tb
CH. cH" following would be the BEST treatrnent?
I I
SH S
I
CH: I. Intramuscular injection of Brz
Figure 2 II. Increase dietary Bt2
ilI. Increase dietary Co

A. I only
B. II only
Btz is synthesized by anaerobic microorganisms. In C. I and II only
the diet, it is found in animal products, particularly in the
D. I and III only
meat and milk of ruminants, because their first stomachs
contain microorganisms that synthesize Btz for the
animal. Complete vegetarians need to include either a
microbiological or a synthetic source of Btz in their diets. 40. Bacteria in the human colon produce B tz. How
this production affect Brz status in the individual?
B12 is absorbed in the ileum of the small intestine.
Intrinsic factor (IF), a glycoprotein, is produced in the A. Colonic Brz is readily absorbed
stomach. In the intestinal lumen, IF binds cobalamin, and
vegetarians.
the complex is taken up by receptors in the ileum of the
small intestine.
B. Colonic Btz is not absorbed, because is
distal to the ileal receptor.
A deficiency of IF lbads to Brz deficiency, even if the C. Colonic B 12 decreases production of IF.
diet is rich in B12. Signs of a Btz deficiency include:
D. Colonic Bt2 decreases activity of
malonyl CoA mutase.
increased urinary methyl malonic acid, neurological
damage due to abnormal membrane lipids, and
macrocytic anemia.

Copyright @ by The Berkeley Review 144 The Berkeley


Specializing in MCAT
Biology Vitamin 812 Passage Vtr

{1. Why is increased methyl malonic acid (MMA) in the


urine a sign of B12 deficiency?

o
coo
I
H- C- CH3

o=c-oH
A. The blocked metabolic pathway to succinyl
CoA pushes the reactants into the MMA
pathway.
B. Brz is a competitive inhibitor of the MMA
pathway.
c. MMA is a by-product of the methionine
synthetase reaction.
D. MMA results from breakdown of surplus IF.

{2" Which of the following statements regarding


methionine (Met) is FALSE?

A. Met is an essential amino acid.


B. Met has a sulfur-containing side chain.
C. The carbon skeleton of Met can be used in
gluconeogenesis.
D. Met is the precursor of Tyr, a nonessential
amino acid.

.{"1. Why is IF resistant to digestion in the stomach?

A. Proteins are not digested by gastric enzymes.


B. IF is protected by nondigestible surface
carbohydrates.
C. IF inhibits the activation of pepsin.
D. IF is not resistant to digestion in the stomach.

l"rpyright @ by The Berkeley Review 145 The Berkeley Review


Specializing in MCAT Preparation
Biology Enz4yme Nomenclature Passage VIII

Passage VtrI (Questions 44-50) The substrate-binding site of a protein enzyme is


called the active site. Lining the active site are specific
Enzymes are catalysts that are quite specific in their amino acid side chains, which attract the substrate and
choice of reactants (substrates) and in the reactions they enhance catalysis. If the pH of the catalytic environment
catalyze. They are able to lower the energy barrier to a is optimal, the rate of reaction tends to be maximized. The
reaction, thereby increasing the rate (velocity) of the optimal operating pH of an enzyme therefore depends on
reaction in either the forward or the reverse direction. the pKa values of the amino acid side chains (Table 2) at
Catalysts have no effect on the position of equilibrium of the active site.
the reaction.
Table 2. pKu Values of Select Ionizable Groups
There are thousands of different types of enzymes that
catalyze a wide variety of reactions. Many of these cl-cooH cr.NH3+ Side chain
enzymes have common names that are descriptive of their a-Amino Acid p&r p&z plq3
functions and they are derived from the principal reactant
involved in the catalytic process. The suffix -ase is Arginine 1.8 9.0 12.5
attached to the common name to indicate an enzyme.
Aspartic acid 2.0 10.0 3.9
Urease, for example, increases the rate ofurea hydrolysis.
Cysteine 1.8 10.8 8.3
o
lt
H2N- C- NH2 Urease- Glutamic acid 2.2 9.5 4.1
+ H2O NH3 + CO'
Urea Histidine 1.8 9.2 6.0

Some enzymes have names that do not end in -ase.


Lysine 2.2 9.2 10.8
Pepsin and trypsin, for example, are proteolytic enzymes. Tyrosine 2.2 9.I 10.1

In order to assign a name unambiguously to an


enzyme, the Enzyme Commission (EC) of IUPAC places
each enzyme into one of six enzyme classes (Table 1),
44. Using the expression AG'' = -2.3 RT log
based on the type of reaction they catalyza Within each determine the change in the free energy for
of these six classes are subclasses and sub-subclasses. reaction shown below,if the concentration values
Each enzyme is assigned a specific, four-integer EC the reactants and products at equilibrium is
number, along with a common name and a systematic moles.
name.
A+B-i.-C
Table 1. Enzyme Classification
A. - 2.3 RT
Enzyme Class Type of Reaction Catalyzed B. + 2.3 RT
Oxidoreductase Oxidation-reduction
c. - 4.6 RT
D. + 4.6 RT
Transferase . Transfer offunctional groups

Hydrolase Hydrolytic cleavage 45. In the Krebs cycle, fumarate is converted


by the enzyme fumarase.
Lyase Addition of groups to double
bonds or the reverse
o o
coo coo
Isomerase Transfer of groups within I

CH
I
HO- C- H
molecules to give isomeric ll + HnO I

CH CH'
forms lg l-6
coo coo
Ligase Bond formation coupled with
ATP hydrolysis Fumarate Malate

Enzymes can eiist in alternate forms called This enzyme is a:


isoenzymes. Lactate dehydrogenase (LDH), for example,
has two different types of subunits designated as M and A. lyase involved in a hydration reaction.
H. LDH with M subunits is found primarily in muscle B. hydrolase involved in a hydrolysis reactioa-
tissue, while LDH with H subunits is found primarily in C. lyase involved in a hydrolysis reaction.
heart tissue. D. hydrolase involved in a hydration reaction-

Copyright @ by The Berkeley Review r46, The Berkeley


Specializing in MCAT Prel
Biology Enryme Nomenclature Passage VItr

16. The first step in glycolysis is the conversion of 49. The active sites of different enzymes can contain
glucose to glucose-6-phosphate by the enzyme unique catalytic groups. The properties of these
hexokinase. groups are pH-sensitive, and the initial rates
generally show bell-shaped curves.
o o
o o
t/ .>l o Vrnu*
o--/ P-1 o- p- o-
It lt
AMP
Ol
o

t
o o o-P=o I ()
;b:)' o
{)

*s._(t
Hrc H H,C
J-o os d

ADP I
8 9 10 ll1213
OH
pH
Glucose Glucose-6
phosphate
Which of the following amino acid pairs BEST
Hexokinase is a member of a class of enzymes for the curve shown
represents the catalytic group
called: above?

A. ligases. A. Histidine and aspartic acid


B. hydrolases. B. Cysteine and tyrosine
C. transferases. C. Arginine and lysine
D. oxidoreductases. D. Lysine and cysteine

.{-. Hexokinase also has the ability to transfer the 1-


phosphoryl group of ATP to water, but at a rate
which is about 40,000 times slower than its transfer 50. The isoenzymes of LDH can be resolved by using
to the C-6 hydroxyl of glucose, even though the electrophoresis. In the polyacrylamide gel shown
nucleophilic properties of the C-6 hydroxyl group below, Lane 1 represents one type of LDH
and water are similar. The large difference in rate isoenzyme, while Lane 3 represents another type:
can be explained by all of the following EXCEPT:
Lane 1 Lane2 Lane 3
(+)
A. the introduction of a large conformational
change in the enzyme by glucose.
B. the reduction of active site polarity by the
exclusion of water.
C. the presence of water in the active site at the
position occupied by the C-6 hydroxyl group. Ij:@
D. the increased nucleophilicity of the C-6
hydroxyl of glucose and the increased Origin
electrophilicity of the y-phosphoryl group of C)
ATP.
Equal amounts of the two isoenzymes are mixed
together. The subunits are next dissociated from one
another and then allowed to reassociate randomly as
*ri, The second step of the glycolytic pathway involves indicated in Lane 2. This indicates that LDH is a:
:he conversion of an aldose sugar into a ketose
sugar. Enzymes that catalyze this type of reaction A. dimer.
:re called: B. trimer.
C. tetramer.
\. transferases. D. pentamer.
B. isomerases.
C. hydrolases.
D. ligases.

: ':,ght @ by The Berkeley Review 147 The BerkeleY Review


Specializing in MCAT Preparation
Biology Lysozyms Flssftanisrn Passage H

Passage IX (Questions 51-58) Main Chain of Lysozyme Enzyme

The cell wall polysaccharide of bacteria is composed Cfu:S I

of two types of sugars, N-acetylmuramate (NAM) and N- o'rc-oo


acetylglucosamine (NAG). These sugars are joined to
each other by a glycosidic linkage.

Lysozyme is composed of 129 amino acids, linked Ring F


together through peptide bonds to form a protein that
displays cr-helical and p-sheet regions in its tertiary
structure (Figure 1). This enzyme, which is found in
NAM N-H NAG N-H
lal
human saliva and lacrimal fluid, and in hen egg whites, is o=f oi^._o o=F
an antimicrobial substance that aids in the hydrolysis of a CH: f c",
specific substrate bearing a NAM-NAG sugar linkage. I
I
Asp 52
Main Chain of Lysozyme Enzyme
Active Site
Figure 3

Main Chain of Lysozyme Enzyme

Cru:S I

o',t-oo
o
psheet v' Lysozyme
Rings A-B-C
Figure I

The proposed mechanism for the catalytic hydrolysis NAM N-H


of bacterial cell walls by lysozyme is outlined in Figures ^-l
u-L
o
o_
2-5. ln this mechanism, a portion of the substrate cHr i^._o
containing six glycosidic sugar residues fits into the active Asp 52 I
site of the enzyme at positions labeled A, B, C, D, E, and Main Chain of Lysozyme Enzyme
I

F. Two catalytic groups, glutamic acid 35 (Glu 35) and


aspartic acid 52 (Asp 52), reside close to and on opposite
Figure 4
sides of the glycosidic linkage to be cleaved. The
neighboring environments of Glu 35 and Asp 52 are quite
different from each other.
Main Chain of Lysozyme Enzyme
Main Chain of Lysozyme Enzyme

Cru:S
cru rs I
I

o',C- o - H
ntrt'n- t
cH20H
o
o o
Ring F
Rings A-B-C
Rings A-B-C

NAM N-H NAG N_H


NAM N-H
lal
o=f u^-l
-L
o
or ^-_o
o=
F ^,-o
cHr I
of
c",
CH, I
It' Aso 52 I
I
Asp 52
Main Chain of Lysozyme Enzyme
Main Chain of Lysozyme Enzyme

Figure 5
Figure 2

Copyright @ by The Berkeley Review t4a The Berkeley


Specializing in MCAT Prepar
Biology Lysoryme Mechanism Passage IX

51. Lysozyme works BEST on bacteria that are: 55. The pKa values for the side chain carboxyl groups
on aspartic acid and glutamic acid are usually cited
I. Gram-positive. as 3.9 and 4.1, respectively. Analysis of lysozyme's
II. Gram-negative. active site indicates that the pKa of Asp 52 is still
III. Missing their cell walls. about 3.9, but the pKn of Glu 35 is now about 6.6.
Glu 35 shows a 2.5 fold increase in pKa, because:
A. I only
B. II only A. the carboxyl group of Asp 52 is located in a
C. III only polar environment.
D. II and III only B. the ionized carboxyl group of Asp 52
destabilizes the protonated carboxyl group of
Glu 35.
C. the carboxyl group of Glu 35 is located in a
nonpolar environment.
D. the carboxyl group of Glu 35 is located in a
polar environment, where it is stabilized by
<) The configuration of the linkage between rings D hydrogen bonding.
and E in Figure 2 is:

A. cr(l-+4). 56. In order to establish clearly which bond is being


B. p(a-+l). cleaved by lysozyme, enzymatic hydrolysis using
C. cx(4-+1). water labeled with 18O is used. This heavy isotope
D. p(1+4). of oxygen is found attached to sugar residue D at
the:

A. reference carbon.
B. anomeric carbon.
C. C-5 carbon.
D. C-6 carbon.
53. Which of the following general classes of enzymes
BEST represents lysozyme?
57. According to the proposed mechanism outlined in
A. Hydrolase the passage, lysozyme is involved in acid catalysis.
B. Transferase This mechanism involves:
C. Oxidoreductase
D. Ligase A. noncovalent catalysis coupled with heterolytic
bond cleavage.
B. covalent catalysis coupled with heterolytic
bond cleavage.
C. noncovalent catalysis coupled with homolytic
bond cleavage.
D. covalent catalysis coupled with homolytic
51. Amino acid charge distribution at the active site of bond cleavage.
an enzyme is arranged to:

A. destabilize the transition state in order to 58. Based on the representation of lysozyme shown in
increase the rate of catalYsis. Figure 1, glutamic acid would be expected to be
B. stabilize the transition state in order to found in the:
decrease the rate of catalysis.
C. destabilize the transition state in order to I. active site.
decrease the rate of catalYsis. II. cx,-helical regions.
D. stabilize the transition state in order to increase IIr. interior of the protein.
the rate of catalYsis'
A. I only
B. II only
C. I and II only
D. I and III only

l:n:"rright @ by The Berkeley Review 149 The BerkeleY Keview


Specializing in MCAT PreParation
Biology Dnzyme Kinetics II Passage I

Passage X (Questions 59-65) The turnover number of an enzyme is the number d


substrate molecules processed per enzyme per secooil
Enzyme activity depends not only on temperature and when the enzyme is completely saturated with substrar-
pH, but also on the available concentrations of substrates This value is equal to k3 and is sometimes called ftc
[S] and inhibitors [I]. As [S] in an enzyme-catalyzed catalytic constant, k.ut. A measure of an enzymeh
reaction increases, so does the rate of catalysis and the catalytic efficiency can be obtained from the ratio d
formation of products P. At a constant enzyme kcat/Ku. Table 1 lists some values for kcat and Ku.
concentration [E], the relationship between the initial
reaction rate (velocity v) and [S] describes a hyperbola
(Figure 1). Table 1. Enzyme and Substrate Parameters
Enzyme (and substrate) KM (M) k.", (s-1)
V-u" Acetylcholine esterase 9.5 x 10 -5 1.4 x 10 a
(Acetylcholine)
Carbonic anhydrase 2.6xlo-2 4.ox1os
(Bicarbonate)

O V-u* Chymotrypsin 6.6x10-4 1.9x102


(N-Acetyltyrosine ethyl ester)
0)
2 Fumarase 5.ox1o-6 8.0x102
(Fumarate)

Pepsin 3.0 x 1o -a 5.0 x 10 -l


(Phe-Gly)
Ribonuclease '7.9'x rc'3 i.9 x lo2
(Cytidine 2',3' cy clic phosphate)
Km tsl Urease 2.5x10-2 1.ox1oa
(Urea)
Figure I

When an enzyme reacts with a substrate, it forms the


enzyme-substrate complex ES, which then breaks down to
reform the enzyme and release the product. This reaction
is governed by individual rate constants kl, kz, and kt.
Rate constant kr has limits between 108 to 199 14-1r-1.

k3 .E
E+S ES +p 59. When the substrate concentration is very much
lower than the Michaelis constant, the MichaeUs-
Menten equation reduces to:
The relationship between the initial reaction velocity
and [S] was examined during the early 1900's by Leonor A. ,r-V**.
Ky
Michaelis and Maude Menten. They concluded that the
initial reaction velocity v can be given by:
B. ,n' - vt*
Ky + [S]'
(r) u= J'adu-
K* + [S]
V*u* [S]
^
(-. ,.v
---, Krra
where [S] is the initial substrate concentration, Vmax is the
maximal velocity of the reaction under saturating [S], and
Kv is that [S] where the reaction operates at one-half its D.
maximal velocity. The lower the value of Ku, the higher
the affinity the enzyme has for its substrate. Ku is called
"=+P
the Michaelis constant and Equation (1) is called the
M ichaelis - M enten e quation.

Copyright @ by The Berkeley Review 150 The Berkeley Keviev


Specializing in MCAT Preparation
Biotogy Enz5rme Kinetics II Passage X

6lD. When the substrate concentration is very much 63. By taking the reciprocal of the Michaelis-Menten
higher than the Michaelis constant, the Michaelis- equation, a straight line can be graphed on a double
Menten equation reduces to: reciprocal plot. This graph (shown below) more
accurately represents the parameters of the curve in
Figure L
{. r,' = [S]
Ky + [S]'
B. v = Vmax.
c' u=
rst
isr'
V-u* [S]
D.'v=-.
tsl
-4 -3 -2 -1 0 I 2 3 4 5
lrtsl
6,1. When the substrate concentration is equal to the
Michaelis constant, the Michaelis-Menten equation Ku of the enzyme with
The these kinetic data is:
reduces to:

A. + 0.5.
A. ,, - vtu* - B. - 2.0.
2Krtr C. +2.0.
D. + 5.0.
B. u-2V-u*.
Ky
C. V = 2V-r*.

D. u-Vtu*. 64. Which of the following pairs of enzymes have


2 achieved a state of virtual catalytic perfection?

A. Carbonic anhydrase and chymotrypsin


ttrL The relationship between IEl, tsl, [ES], and [P] can B. Acetylcholine esterase and carbonic anhydrase
be expressed graphically in terms of steady-state C. Fumarase and acetylcholine esterase
enzyme kinetics. Which of the following graphs D. Chymotrypsin and pepsin
BEST represents this relationship?

A. B.

65. The enzyme triose phosphate isomerase catalyzes


the conversion of glyceraldehyde-3-phosphate into
dihydroxyacetone phosphate in both yeasts and
o o humans. This enzyme has a reported efficiency of
o C)

o 2.4 x log M-1s-1, which suggests that it:


U U

Time Time
A. evolved to near maximum efficiency early in
its evolutionary history and has changed very
little since then.
C. D.
B. evolved to near maximum efficiency late in its
evolutionary history and has changed very
o little since then.
c6
H k C. did not evolve to near maximum efficiency
C) o during its evolutionary history, because it is
found in distinctly different species.
U U D. did not evolve to near maximum efficiency
during its evolutionary history, because of its
Time Time inability to change.

l:nlright @ by The Berkeley Review r5l The BerkeleY Keview


Specializing in MCAT Preparation
Biology Adenosine Triphosphate (ATp) Passage XI

Passage XI (Questions 66-72) 67. Which of the following cellular transport processes
requires ATP?
Adenosine triphosphate (ATp), shown in Figure l,
serves as the primary energy intermediary between the A. Osmosis
higher-energy phosphate compounds and the lower- B. Diffusion
energy phosphate compounds, Other energy-rich C. Active transport
nucleoside triphosphates that function in this capacity are D. Passive transport
guanosine triphosphare (GTP), cytidine triphosphate
(CTP), and uridine triphosphate (UTp). 68. When a compound is used to inhibit ATp production
in the cells of an organism, what happens to that
Table I indicates ATP's energetically central position. organism?
The hydrolysis of ATP to ADP (adenosine diphosphate)
and Pi (inorganic phosphate), or to AMP (adenosine A. It continues regular metabolic processes
monophosphate) and PPi (pyrophosphate), provides without change by switching ro a higher-
energy for many endergonic reactions in the body, such as energy phosphate compound.
muscle contraction and transport of ions against B. It continues regular metabolic processes using
concentration gradients. GTP.
C. It is unaffected by the cessation of ATp
i* production.
o"" oo .,o f
/i(o{.o/ . D. It slows all metabolic activity and
ort
r | !nffs
o-P-o-P-o-T-o-r", ^
dies.

69. Which compound would release the MOST


energy after the phosphorylation of ADp?
HO OH A. Phosphoenolpyruvate
Figure 1
B. 1,3-bisphosphoglycerare
C. Acetyl phosphate
D. Phosphocreatine

Compound AG" (kJ/mol) 70. Which of the following metabolic processes


Phosphoenolpyruvate NOT generate a net yield of ATp or GTp?
- 61.9
1,3-Bisphosphoglycerate - 49.4 A. Glycolysis
Acetyl phosphate - 43.1 B. Citric acid cycle
Phosphocreatine - 43.1 C. Protein synthesis
PPi -+ 2 Pt - 33.5 D. Oxidativephosphorylation
ATP -+ AMP + PP1 - 32.2
AIP -+ ADP + P1 - 30.5 71. What type of chemical bond is indicated by
Glucose- 1,-phosphate - 20.9 arrow in Figure l?
Fructose-6-phosphate - 13.8 A. Phosphoanhydridebond
Glucose-6-phosphate - 13.8 B. Phosphoesterbond
Glycerol-3-phosphate - 9.2 C. O-Glycosidic bond
Table 1. Standard free energies of phosphate D. Peptide bond
hydrolysis of some compounds.
72. When ATP is hydrolyzed to AMp and ppi, which
the following statements must be TRUE?

66. Based on the data in Table 1, which


of the following
I. PP1 is very stable due to electrostatic
compounds can be phosphorylated exergonically by interactions between its phosphate residues-
ATP? II. More energy is released than in the hyd
of ATP to ADP.
A. Phosphoenolpyruvate IfI. PPi is insoluble in the cytosol of the cell.
B. Glucose-1-phosphate
A. I only
C. Acetyl phosphate
B. II only
D. Pyrophosphate
C. II and trI only
D. I,II. and III

Copyright @ by The Berkeley Review 1|52


Biology Protein Degradation (Ubiquitin) Passage XII

Passage XII (Questions 73-79) 73. Why do lysosomal enzymes have pH optima close to
5?
Proteins are degraded inside cells by selective and
:onselective means. Lysosomes are used ior nonselective A. So that they will be active when released to the
:roteolysis. These membrane-bound organelles contain cytoplasm
.bout fifty hydrolytic enzymes, including several types of B. So that they will be inactive when released ro
;roteases. The lysosomal enzymes have pH opiima of the cytoplasm
.rout 5. Lysosomes fuse with vacuoles inside the cell and C. To degrade basic proteins more efTiciently
.'''drolyze the contents, thereby recycling intracellular D. To degrade acidic proteins more efficientiy
: rmponents and digesting foreign particles.

Conversely, the ATP-dependent proteolytic system is


,,'ry selective. The molecule ubiquitin, a 76_amino
acid
.-.ot"rl,- _binds to lysine residues on condemned proteins.
- te DNA sequence coding for ubiquitin is trignty
- rnserved throughout eukaryotes.

An
_enzyme system called the ubiquitin_conjugate
74.
[!1t is meant by saying rhar a DNA sequence is
highly conserved in eukaryotes?
::grading enzyme (UCDEN) recogniies ttre ubiqiitin
.:gs and destroys the labeled protein. The ubiquitin
:'.)tem is used to destroy abnormal proteins and short A. T!" sequence varies widely among all
,'.ed enzymes that are at control points in metabolic eukaryotes studied.
:.thways. As shown in Table 1, the N-terminal residue of B. The sequence differs only among phyla of
--e protein determines how quickly it is ubiquitinated eukaryotes studied.
and
:;stroyed. C. The sequence is virtually identical in all
eukaryotes studied.
D. The sequence is completely identical in all
eukaryotes studied.

Cytochrome c I 50 hours
.{ protein in the eleclron-ranspon chain
Clyceraldehyde-3-phosphate 130 hours
Jehydrogenase (GAPDH)
A protein required for glycolysis
.\ldolase t5.
I 1 8 hours Based on information given in the passage and in
{ protein required for glycolysis
Table 1 about its role at metabolic iontrot points,
\-Terminal residue (stabilizing) which.of the following proteins is MOST likely to
\Iethionine (Met), Serine (Ser), be modified quickly by ubiquitin?
> 20 hours
\lanine (Ala), Threonine (Thr),
\aline (Val), Glycine (Gly) A. Cytochrome c
B. Tyrosineaminotransferase
shof$ill+ i6,i ffil#iiiir C. Ornithinedecarboxylase
T1'rosine aminotransferase 120 minutes D. Glyceraldehyde-3-phosphate dehydrogenase
.{ protein required for amino acid catoblism
R\A Polymerase I 78 minutes
.{ eukaryotic protein that synthesizes rRNA
Cmithine decarboxylase 12 minutes
.\ protein required for polyamine synthesis
\-Terminal residue (destabilizing)
I.oleucine (Ile), Glutamic acid (Glu) = 30 minutes
Tvrosine (Tyr), Gluramine (Gln)
76. Of the following choices, which N-terminal amino
= 10 minutes acid provides the MOST stability for an enzyme?
?henylalanine (Phe), Leucine (Leu) = 3 minutes
\spartic acid (Asp), Lysine (Lys)
A. Lysine
erginine (Arg) = 2 minutes B. Glycine
Table 1. Half-lives of long-lived and short-lived cellular C. Glutamate
:lzymes as a function of their (potential) N-terminal residues. D. Leucine

- .,,-vright @ by The Berkeley Review 153 The Berkeley Review


Specializing in MCAT preparation
Biology Protein Degradation (Ubiquitin) Passage Xtr
3
77. Under what physiological condition would 79, The antimalarial drug chloroquine, pictured below, Prs
lysosomal protein degradation be the highest? penetrates the lysosome and accumulates as a weak
base inside. What effect does this have on the
A. During a fast lysosomal degradation of proteins? ,*fu
B. Following a meal frunl
C. During exercise &rE A
D. During pregnancy fr"r*1.'
ha['lr

.l
a,r - 1cH2)r- N(c2Hs)2 f llifti
il- ltofu
cHr
'lfl'mrfilMt
Chloroquine
dfiMffi
78. The following diagram indicates the steps involved
in the attachment of ubiquitin to a protein: A. The rate ofprotein degradation is decreased. '@mfueil;
firuffit
B. The rate ofprotein degradation is increased.
c. Chloroquine inhibits the proteases by 'mitEm
modifying their active sites.
M 'llrdhT

ubiquitin- 3- oO + BI-SH D. Chloroquine activates the proteases try


modifying their active sites.
, llr
o"
vll\
aue* ee'
o
il 10fi
Ubiquitin- C- S- Er
M
,flcp lrq
m

il
m
Ubiquitin- C- S- E2

n
ConOemned protein (CP)
. l16
ll\ ez-sn
u'o
il
Ubiquitin- C- N- Lys- CP

What is the purpose of the reaction involving ATP


in Step 1?

A. The hydrolysis of ATP to AMP and the


subsequent hydrolysis of PP1 provides the
energy ofthree high-energy phosphate bonds.
B. The hydrolysis of ATP to AMP provides the
energy of two high-energy phosphate bonds.
C. The hydrolysis of ATP to AMP and the
subsequent hydrolysis of PP1 provides the
energy of two high-energy phosphate bonds.
D. The hydrolysis of ATP to AMP provides the
energy of three high-energy phosphate bonds.

Copyright @ by The Berkeley Review 154 The Berkeley


Specializing in MCAT
Biology Nutrients and Proximate Analysis Passage XItr

Fassage XIII (Questions 80-87) 80. The dehydration step of proximate analysis is often
by lyophilization (freeze-drying). The sample is
Nutrients are grouped into six classes: protein, frozen, and the water is evaporated from the frozen
:..:bohydrate (starches and sugars), and lipid, all of which sample in a vacuum. What is this process called?
::ovide energy; and water, vitamins, and minerals, which
.:: noncaloric. Protein and carbohydrate each provide 4 A. Melting
, - al/gram, and triglyceride (a type of lipid) provides 9 B. Vaporization
i:aVgram. C. Condensation
D. Sublimation
Average daily water intake for an adult is about 1.5 to
iiters from liquids, foods, and metabolic water.
-.-tamins
are divided into water-soluble and fat-soluble
:umins. The water-soluble vitamins are thiamin (Br), 81. When vitamins and minerals are combusted, which
-::oflavin (82), niacin (B3), pyridoxine (Bo), folate, compounds produce the gas phase and which
,.:alamine (Btz), folate, biotin, pantothenic acid and produce the ash?
-,:orbic acid (vitamin C). The fat-soluble vitamins are
"amin A (retinol), vitamin E (alpha-tocopherol), vitamin A. Both are in the gas phase.
- dihydroxy-cholecalciferol), and vitamin K. Minerals B. Both are in the ash.
:;;ude the major or macrominerals: calcium, phosphorus, C. Vitamins are in the ash, while minerals are in
- -lorine, potassium, sulfur, and magnesium; and the trace the gas phase.
:,nerals: iron, zinc, iodine, copper, manganese, fluoride, D. Vitamins are in the gas phase, while minerals
-::omium, selenium, molybdenum, cobalt, vanadium, tin, are in the ash.
,r--on, and nickel.

Vitamins and minerals function in specific ways in


.:: body. Some are important as cofactors or coenzymes, 82. Which of the following is NOT a trace mineral?
;'..rle others maintain acid-base balance, promote nerve
-- j muscle activity, or maintain fluid and electrolyte A. Magnesium
-.-.:nce. Vitamins are organic molecules, while minerals B. Zinc
*: inorganic ions and atoms. Many of the B vitamins C. Manganese
, :;tion as cofactors in the reactions of energy D. Iodine
::-abolism, such as niacin in NAD. Vitamin C works as
"": :ntioxidant to keep metals in a reduced form in some
r::i'mes. The fat-soluble vitamins are also important.
.:rmin A is crucial for vision. Vitamin D helps regulate 83. A portion of chicken is analyzed by the Kjeldhal
-.:lum homeostasis. Vitamin K is required for blood technique and found to contain 5 grams of nitrogen.
- -ino How many grams of protein are in it?

The nutrient content of food can be determined by A. 31 grams protein


: rimate analysis, whish involves a series of chemical
, B. 0.8 grams protein
,::..i ses. First, the sample is dehydrated, and the mass of C. 80 grams protein
" .::r is calculated by difference. Next, a lipid extraction D. 3.1 grams protein
' : organic solvent is performed to calculate the mass of
" :. A Kjeldhal analysis determines nitrogen content.
'.: mass of protein is determined based on the
i :-:nption that a protein contains 16%o nitrogen Fiber is 84. If you were to analyze a new food, which
:, :esidue unaffected by acid and base hydrolysis. components would you test to determine the calorie
by content?
"nins and minerals are separated combustion,
:, -idng in a gas phase and an ash. Finally, carbohydrate
:l.culated as the total weight minus all other nutrients I. Carbohydrates
,:: : :lber. il. Vitamins
ilI. Lipids

A. I and III only


B. II only
C. II and III only
D. I, II, and III

:''right @ by The Berkeley Review 155 The Berkeley Review


Specializing in MCAT Prepmation
Biology Nutrients and Proximate AnalYsis Passage )fltr
!
L
85. Which of the following statements is FALSE?

A. Excess water-soluble vitamins are excreted in


the urine. t
@
B. Fat-soluble vitamins require binding proteins rf
C.
for transPort in the blood.
Ascorbic acid is a fat-soluble vitamin'
&
D. An antagonist for Vitamin K could lead to
longer blood-clotting times.

86. If a person consumes 1800 kcals, and if 3070 of the


energy in that food is from triglyceride, how many
grams of triglyceride are consumed?

A. 540 grams
B. 60 grams
C. 67 grams
D. 54 grams

87. About 250 mL of water is produced through


metabolism and is called metabolic water. Which of
the following reactions would NOT produce
metabolic water?

A. The last step ofthe electron-transport chain


B. Synthesis of a protein from amino acids
C. Glycogen production
D. The breakdown of triglYcerides

156 The BerkeleY


Copyright @ bY The BerkeleY Review
Speciatizing in MCAT Pre
Biology Niacin Experiment Passage XfV

Passage XIV (Questions 88-94) 88. If none of the fatty acids were reesterified into
triglycerides in the adipocyte, what ratio of glycerol
Niacin is a B vitamin that is required in milligram to fatty acids would be seen in the blood?
quantities by the body as a cofactor in many oI th"
enzymes of energy metabolism. It is also used in gram A. 2:3
quantities, at pharmacological levels, to lower levels of
cholesterol and triglycerides in the blood. This beneficial
B. l:2
effect is believed to result from reduced fatty acids
C. 1:3

leaving the adipose tissue and entering the liver. Once in


D. l:4
the liver, free fatty acids are believed to influence the
hepatic assembly of lipoproteins.
89. For the turnover measurements, the isotopic
o enrichment of fatty acids and glycerol extracted
il
C- OH from the blood must be measured. tf tne isotopically
Niacin
ry enriched molecules use-d were 11,23,4:t3Ci-
palmitate and 11,1,2,3,3-2Hl-glycerol, then what
instrument would be used for measurement of
isotopic enrichment?
. A research group is testing the following
:bout niacin's
hypothesis
mechanism of action: A. Scintillationcounter
!{rpothesis
B. Mass spectrometer
C. Geiger counter
Inside adipocytes, niacin inhibits reesterification of D. UV spectroscope
:atty acids that were liberated by hormone-sensitive
-pase.
:ryeriment I 90. To extract glycerol from the blood, a student uses a
series of cation and anion exchange resins. A portion
Six hyperlipidemic and hypercholesterolemic subjects of deproteinized blood is adjusted to pH 7.b. The
.:; first studied with isotopically labeled palmitate and blood is applied to a cation resin therto an anion
;11'cerol to quantify turnover of free faity acids and resin. Following a water rinse through the exchange
:-r'cerol. The isotopicalty labeled compoundi are used as resins, where is the glycerol?
:3cers to study the entry of fatty acids and glycerol into
"":e blood from adipose stores. The subjects ihen receive
:-acin for 2 months, building up to 3 grams per day. After Blood sample
;'s period, the subjects repeat the turnover study with
:reled palmitate and glycerol. The following table +
rjicates the data obtained from this study:

Phase Fatty acid turnover Glycerol turnover


Cation exchange
resin W Ed
fumol/kg/min) (pmoVkg/min) +
Anion exchange
Pre-
3.4 1.2 resin
q.rl,'.#
niacin
Post-
t#
2.1 1.3
niacin
Table I:_Turnover of fatry acids and glycerol (pmol/kglmin.1 in Collection
-
subiects before and after niacin intervirition.
tube

A. The glycerol is attached to both the cation and


the anion resins.
B. The glycerol is attached to the cation resin.
C. The glycerol is attached to the anion resin.
D. The glycerol is in the collection tube.

tll:]'right @ by The Berkeley Review 157 The Berkeley Review


Specializing in MCAT preparation
Biology Niacin Experiment Passage XIV

91. If fatty acids are released from the adipose tissue,


repackaged in the liver, and sent back for storage in
the adipose tissue, what metabolic effect does this
cause?

A. Ituses energy through a substrate cycle.


B. Itcreates energy through a substrate cycle.
C. Fatty acid cycling damages the liver.
D. Fatty acid cycling damages the adipose tissue.

92. Which of the following statements is supported by


the data in Table 1?

A. The pre-niacin period involved no lipolysis.


B. Niacin treatment promoted both lipolysis and
reesterification.
C. The pre-niacin period did not involve
reesterification.
D. Niacin treatment increased reesterification of
free fatty acids in the adipose tissue.

93. If a new experimental drug were studied that


increased fatty acid reesterification to 1007o, what
effect would this have on the body?

A. The body would switch to ketone bodies for


peripheral tissue energy metabolism.
B. The brain and nervous tissue would have no
ready supply offuel.
C. The muscle and heart would have no ready
supply of fuel.
D. The body would switch to cholesterol for
peripheral tissue energy metabolism.

94. In a separat6 experiment to learn more about


lipolysis, the blood of a group of subjects is studied
before and after being given caffeine. Caffeine is a
stimulator of hormone-sensitive lipase, an enzyme in
the adipose tissue that hydrolyzes triglycerides into
free fatty acids and glycerol. What effect would
caffeine have on glycerol and fatty acid
concentrations in the blood?

A. Glycerol and fatty acid concentrations would


both increase.
B. Glycerol concentrations would increase, and
fatty acid ioncentrations would decrease.
C. Glycerol concentrations would decrease, and
fatty acid concentrations would increase.
D. Glycerol and fatty acid concentrations would
both decease.

Copyright @ by The Berkeley Review r5a The Berkeley


Specializing in MCAT
Biology Dif frrsio n. Limited, E;rrn1rm,e' Qataly zed Reacti o ns Passage XV

lTrssage XV (Questions 95-100) 97. Based on information in the passage, which of the
following statements is TRUE?
l:r metabolic pathways, the product of one reaction
-r::, serves as a reactant for a subsequent reaction. For A. Many enzymes have inefficient catalytic
:i: i-,1son, many enzyme-catalyzed reactions need to be properties.
,:- :n rn one direction. To obtain a negative Gibbs free B. Diffusion-limited reactions are faster in the
r::r and thereby drive a reaction in one direction, many cytosol than in the nucleus.
inr.,-ilS€S are coupled to the hydrolysis of ATP into ADP C. Multienzyme complexes should increase the
r: P,. The ratio of ATP to ADP is high, which ensures rate of diffusion-limited reactions.
riL: : ";rlable cellular source of energy. D. The hydrolysis of ATP into ADP and ! has a
positive Gibbs free energy.
-- :ell's livelihood is dependent upon its ability to
,l:*' rut its metabolism efficiently. Efficient metabolism
rr --:es efficient enzymes, working at rates greater than
; 98. It is discovered that in a particular cell, a membrane-
u L: r:r3S of unavoidable, competing side reactions. The bound compartment is 20Vo of the total cellular
rr ; irich is rate-determining in many enzyme-catalyzed volume. The concentration of reactants inside the
r:.!d:-rls is due to the collision frequency of enzyme and compartment can be:
;i..'1,n::::e. rather than the actual enzymatic function of the
lr,: :r:. These reactions are termed diffusion-limited, and A. 5 times greater than in the cytosol.
:: "::rr],, they can be accelerated by increasing the B. 10 times greater than in the cytosol.
:.;rtration of the reactants. However, the cell's C. 15 times greater than in the cytosol.
'' r":-rsm has limits on the volume of reactants it can D. 20 times greater than in the cytosol.
. : and most diffusion-limited reactions are not
rllr;: ::Ered in this fashion. 99. The following table shows experimental data for
-:; diffusion-limited reactions taking place outside and
presence of intracellular membranes increases the
within a cell membrane:
lilL: :: ieactions limited by diffusion. The formation of
tilu iiriirle compartments concentrates substrates and
r i' :,3s. and it minimizes the distance molecules must
Internal Time required for
Tlial
t"l ,:. Diffusion-limited reactions that are carried out
Membrane substrate to hit target
n r[]': Ihe membrane itself, using membrane-anchored A None 15 minutes
nr,"r':-:s. are also accelerated. The reason is that the B Large l minute
,ii ff,,r::les and enzymes are limited to movement in only
C Small 9 seconds
\r :imensions. By eliminating a dimension of
IIr u , ::,ent, there is an enhanced opportunity for substrate

,iLu r :rzvme collision. Based on information presented in the table, it can


be concluded that the frequency of collisions in:

uu 'i-hich of the following statements describes an A. Trial A is 15 times greater than in Trial B.
- rEanism approaching equilibrium? B. Trial C is l0 times greater than in Trial B.
C. Trial C is 100 times greater than in Trial A.
.{. It has a high intracellular ratio of ATP/ADP. D. Trial B is 10 times greatff than in Trial C.
B. It attains the most efficient state of
metabolism. 100. According to the passage, an efficient enzyme-
C. There is a constant input of energy into the calalyzed reaction requires that DNA-binding
organism. proteins should:
D. It experiences intracellular death and decay.
tiillltr A. contain many regions of acidic amino acids.
-: rhe process of diffusion, the distance a molecule
B. be translated in the cell's nucleus.
:rvels from its origin is directly proportional to the C. randomly jump onto and off of a DNA
:quare root of the time traveled. Based on this molecule until they hit an appropriate binding
riormation, it can be concluded that diffusion is: site.

.{. D. bind to any region of the chromosome and then


efficient over long and short distances. slide along the DNA until they reach an
B. efficient over long distances, but inefficient appropriate binding site.
over short distances
C. inefficient over long and short distances.
D. inefficient over long distances, but efficient
over short distances.

irr',,:-,sht @ by The Berkeley Review 159 The Berkeley Review


Specializing in MCAT Preparation
Biology Metabolic Components Section VII Answers

l. B is correct. We first need to consider the diagram with the enzyme interacting with the substrate and inhibitor
(Figure I below). Notice that there are two sites at which binding can take place. The substrate (square) and the
inhibitor (diamond) bind at different locations. This is an indication that the inhibition is not competitive. Instead, it
should look like noncompetitive inhibition. A noncompetitive inhibitor can bind to the free enzyme, or it can bind to
the enzyme-substrate complex.

.'-\
uJ-- txljr Fl /',---\-
A

lln
1t^ I

@
l-s-l
G-b
VY 0 l/tsl

Figure 1 Figure 2

Now that we know the interaction is noncompetitive, we must choose the correct graph. A noncompetitive i
decreases the maximal velocity of a reaction (V.u*). The more inhibitor added to the reaction, the more the Vmax
decreased. The slope of the line becomes steeper. However, one important characteristic of a noncompeti
inhibitor is that the KM remains the same. The constant Ky is important, because it allows us to eliminate e
choice except B (see Figure 2 above). The correct choice is B.
) C is correct. The Michaelis constant, Ku, tells us something about a given enzyme and its relationship to a
substrate. Kl,r has a simple definition. It is the substrate concentration, [S], that gives a half-maximal reac
velocity. In other words, when [S] = KM, the enzyme is said to be half-saturated with substrate (i.e., vo = y
This is what is given in choice A. The Klt also characterizes the substrate-enzyme interaction. It is a measure of
enzyme's affinity for a substrate only when the step leading from the enzyme-substrate (ES) complex to the
has a rate (k3) that is much smaller than the rate (le) of the ES complex dissociating back to the free substrate
enzyme. A small Kvr indicates a strong binding, while a high Krvr indicates a weak binding between enzyme
substrate. This is indicated by choice B.
kr k3
E+STES+E+P

The Krra value between enzyme and substrate is not changed if a noncompetitive inhibitor is added. Only the V
changed (it.decreases). This is what we see in choice D. The Ku is not numerically equal to V mu4/2, because
[S] and not the value at Ymaxl2. The value of Ymaxl2 might be given in units of pM/min, while the value of
might be given in units of mM. The correct choice is C.

3. A is correct. This question is asking you to rearrange the Michaelis-Menten equation so the initial velocity
solved for in terms of Vrnu*.

v -
V'* [S] when rearranged gives:
V.u* - V.*
Ku + [S] [u*lll Krur
+ I
lsl tsl tsl
In the question we are told that [S] = 2.5 x 10-l Ku. This is [S] = 0.25 Ku. Substitution of this value
equation shown above gives the following:

V*u* Vtu*
- V*u*
., - =-J43r- - = o.2o V*u*
K"*l Kr *1 I +1 4+1 5

tsl 0.25 Krra 0.25

The correct choice is A.

Copyright @ by The Berkeley Review 160 The Berkeley


Specializing in MCAT
Biology Metabolic Components Section VII Answers

B is correct. It is important to understand the difference between competitive and noncompetitive inhibitors, how
they act, and what factors they affect in comparison to a control (i.e., with no inhibitor). A competitive inhibitor
binds reversibly to the active site of an enzyme. It competes with the substrate for the active site and increases the
Kna of the enzyme. At a substrate concentration that is high enough, the substrate outcompetes the competitive
inhibitor. Therefore, at a high substrate concentration, in the presence of competitive inhibitor, the Vmax is the same
as that of the control. If we increase the concentration of inhibitor, the Vmax remains the same (at a substrate
concentration that is high), but the Klvr continues to increase. This is indicated in Figure I below.

2x inhibitor 2x inhibitor

+ (^-l
.{- Inhibitnt llv
I 4t
r Inhibi,o.

A0 A0
Figure I Figure 2

A noncompetitive inhibitor binds at a site other than the active site and does not compete at the active site for
substrate binding. Binding of a noncompetitive inhibitor inactivates the enzyme and therefore decreases the Vmax. It
does not alter the Kru. It we continue to add noncompetitive inhibitor, the Vmax continues to decrease, but the Ku
remains the same. This is indicated in Figure 2 above. Combining both of these diagrams gives the graph shown in
the question. Reaction I is the control, and Reaction 2 involves the competitive inhibitor. The graph of the
competitive inhibitor is shown above in Figure 1. The point that is common to both the control and the competitive
inhibitor is Point B. The correct choice is B.
D is correct. Note that the graphs for each mixture are not hyperbolic. They are sigmoidal. They follow non-
Michaelis-Menten kinetics. However, we can use the ideas put forth by Michaelis and Menten to answer the
question. Let's consider the Krebs cycle reaction and the graphs as they are given in the question.

Enzyme
Isocitrate + NAD+ (excess) + o-Ketoglutarate + NADH + H+ + CO2

Based on the reaction equation and the three mixtures we see that excess NAD@ is used in each case. We also see
that as we move from Mixture I to Mixture 2 to Mixture 3, the concentration of ADP in solution increases. What
can we conclude from this? Does ADP increase the Ku of the enzyme? No, because as we increase the
concentration of ADP in solution, the curves move to the left. This is characteristic of a decrease in Krra. Eliminate
choice A. Is ADP an allosteric inhibitor of the enzyme? No, because as we add more ADP, the curve moves to the
left, indicating that /ess substrate is needed to reach half-maximal velocity. Eliminate choice B.
Does a high [NADH]/[NADo] ratio stimulate the enzyme? This is telling us that we have more NADH (the reduced
form) than NADo (the oxidized form). If we have less NADo (i.e., it is not in excess), then the reaction rate will
slow down. There will not be enough of the NAD coenzyme to facilitate catalysis. Eliminate choice C. A high
IATP]4ADPI ratio means that there is less ADP than ATP. This would mean that the graph would resemble that of
Mixture l. In other words, a high ATP concentration inhibits this reaction. This makes sense, because if we have
plenty of ATP in the cell, why waste energy making more? The cell makes more ATP only if its concentrations of
this nucleotide have been depleted. The correct choice is D.

'f*"
B is correct. In order to go from Figure 2 to Figure 3 in the passage, we must have a movement of electrons from
the imidazolium nitrogen of histidine to the nitrogen atom of the nicotinamide ring. We can see this if we consider
the flow of electrons in Figure A below. [This diagram corresponds to Figure 2 in the passage.] After electron
movement and the formation of new bonds, we get Figure B. [This corresponds to Figure 3 in the passage.]

- ru1'right @ by The Berkeley Review r6l The Berkeley Review


Specializing in MCAT Preparation
Biology Metabolic Components Section VII Answers

In Figure A, the electrons on the nitrogen atom of the histidine ring move to an area of electron deficiency. Areas of
electron deficiency are indicated by atoms bearing a positive charge (or partial positive charges). Areas of electron
deficiencies are sometimes referred to as electron slnks. In Figure B, we see that there isnow a pair of electrons or
the nitrogen atom of the nicotinamide ring and that the nitrogen atom of the imidazolium ring of histidine bears a
positive charge. Even though the electrons could end up on the atoms indicated in any of the other choices (and ir
theory they might, for a fleeting moment), they do not produce the stablest end product. The correct choice is B.

Figure A Figure B

7. C is correct. Consider the bond in the question as they are shown in Figure 1 below. If we were to hydrolyze
bond with water, we would get a carboxylic acid functional group and a phosphoric acid functional group (Fi
below). These are two different acids. If we mix them together and lose the element of water between them
anhydrous reaction), we get a mixed acid anhydride linkage. Note that this linkage does not resemble the lin
either a phosphodiester bond, a peptide bond, or an amide bond.

\9no
'6-
I
g- p- gv
l,n
o

I
+ HO- P-
o
ll
lo
o
O

tf- c- oH o- H-C-OH U
l"
cHroroi
la
cH20Po3

Figure 1 Figure 2

Examples of a phosphodiester linkage, peptide linkage, and an amide are given below. Notice that a peptide li
isjust a special class of an amide linkage.
R
ooclol
lllllCH' t "
O-p-O-p-O-p-O-R
r r r -- R-C-N-R'
| ,C--
o I o I o As HtN l-o
l__J
Phosphodiester
I
Peptide
I

Amide

The correct choice is C.

8. C is correct. If the sulfhydryl sulfur were to become an electrophile, it would bear a positive charge and
be able to attack a (partially) positively charged carbon atom of the aldehyde functional group. Therefore,
eliminate choices B and D. When the sulftrydryl sulfur loses its hydrogen atom, it becomes a nucleophile, a
that seeks out electron-deficient centers (like the carbonyl carbon atom) and pass electrons to them. The
now becomes one of what is formed after this happens. Look at Figure 2 in the passage. Do we see an acylthi
or a hemiacetal? Consider the word "acylthioester" for a moment. Let's break this into its components.
acyl, thio, ar'd ester. We know what an esler looks like--it is an R-CO-O-R'linkage (see below). The prefix
from thiol, which is just R-SH. Thiols are the sulfur analogs of hydroxyl groups. A thioester would then look
CO-S-R' (see below). An acyl group is derived from an acetyl group (CH:-CO-R) and looks like R-CO. It is
carbonyl group attached to something. Notice that an acylthioester needs a carbonyl group in the i
structure in Figure 2 of the passage. We do not see that. Therefore, we can eliminate choice A.

Copyright @ by The Berkeley Review 162 The Berkeley


Specializing in MCAT
ffiflology Metabolic Components Section VII Answers

o o
il il
R-C-O-R' R_C-S-R'
Ester Thioester

\\.hat is a hemiacetal? A hemiacetal can be formed when an aldehyde undergoes a nucleophilic attack by a hydroxyl
group (or in this case, a sulfhydryl group). The general reaction is shown below:

o
ooH
fl) rl
R':R*C-S-R':R-C-S-R'
,"t-\ I I I I
RH "' HH HH
i
t
Hemiacetal

: he hemiacetal that is formed is usually too unstable to isolate. However, as shown in Figure 2 of the passage, this is
ire structure that is linked to the enzyme. The correct choice is C.
C is correct. As you read in the passage, dehydrogenase enzymes can contain either niacin or vitamin Bz. If the
:nzyme contains niacin, it is referred to as an NAD-linked dehydrogenase. If the enzyme contains vitamin B2, it is
.;lerred [o as an FAD-linked dehydrogenase. Yery little niacin is found in either milk or eggs. This allows us to
:liminate choice A. Even though fish is a good source of niacin, it is not mentioned as being a good source of
";tamin B2. Eliminate choice B. Similarly, beans are a good source of niacin but are not listed as being a good
i -'urce of vitamin Bz. Eliminate choice D. This leaves meat as a common source of both niacin and vitamin Bz. The
correct choice is C.

B is correct. This question is asking whether you can recognize the difference between something that is oxidized
"".i something that is reduced. Only one of the oxidation-reduction pairs is not found at the active site. Consider the
--.gures I -4 in the passage. In choice A, we find the reduced form of NAD (i.e., NADH) and the reduced form of
; i steine (i.e., R-SH) at the active site in Figure 4. Eliminate choice A. In choice C, we find the oxidized NAD (i.e.,
\AD@) and the reduced cysteine at the active site of Figure 1. Eliminate choice C. In choice D, we find the oxidized
5 \D and the oxidized cysteine (i.e., R-S-R') at the active site of Figure 2. Eliminate choice D.

i: choice B, we also find the reduced form of NAD at the active site of both Figure 3 and Figure 4. However, the
::,rduct, which is in the active site of Figure 4, has been oxidized (and not reduced). How can we tell whether the
::lduct has been oxidized? The oxidation state of the carbonyl carbon in the substrate in Figure I is +1. The
:tidation state of the carbonyl carbon in the product in Figure 4 is +3. As we move from the substrate to the
:r-rduct, there is a 2-electron change at that carbonyl carbon. Ifthere has been a loss ofelectrons, then it signifies an
- ri.lation. A gain of electrons would signify a reduction.

{nother way to consider this is to think of just the substrate and the coenzyme. Since they are both involved in an
:riidation-reduction reaction, then one must start off in the oxidized form, while the other is in the reduced form.
{tter the reaction is over, the one that was originally in the oxidized form becomes reduced, and the one originally
.: the reduced form becomes oxidized. In Figure l, the coenzyme NAD is in the oxidized form (NAD@). Therefore,
:he substrate must be in the reduced form. At the end of the reaction, NAD is in the reduced form (NADH), and the
::oduct is in the oxidized form. The correct choice is B.
B is correct. The reaction in the question tells us we need to consider the coenzyme FAD. The structure of this
:r-€IlZltrle is given in the passage in Figure 5. Note that the oxidized form is highly conjugated. We are told in the
:L\sage that "...the more conjugated bonds a molecule has, the longer the wavelength at which the molecule absorbs
;rght." In the question we are told that "oxidized FAD absorbs light in the visible region of the electromagneric
ipectrum at 460 nm." We are also given a graph of light absorption at wavelengths in the visible region of the
:lectromagnetic spectrum.

-\t the beginning"of the reaction, we have a solution of succinate (reduced) and FAD (oxidized and highly
;urnjugated). The oxidized form of FAD is absorbing light at 460 nm. By looking at the graph, we see that rhe blue
;one is being absorbed. If a wavelength of light is being absorbed, we perceive its complementary color. In this case,
:he complementary color to blue is yellow (a mixture of the green and red cones). Therefore, our starting solution is
'n ellow. We can eliminate choice A and choice D.

i1 ur,,*:ht O by The Berkeley Review 163 The Berkeley Review


Specializing in MCAT Preparation
Biology Metabolic Components Section VII Answerg

At the end of the reaction, we have an oxidized product (fumarate) and a reduced coenzyme IFADHz). Note that fu
structure of FADHz in Figure 5 of the passage is not as highly conjugated as the structure in the oxidized state. Thc
fewer conjugated bonds a molecule has, the shorter the wavelength at which that molecule absorbs light. Bec€
FADHz has so few sites of conjugation, we would expect absorption to take place outside the visible range of
electromagnetic spectrum. The final solution will be colorless. The correct choice is B.

12. D is correct. Choice A is p-D-glucose. Choice B is saccharic acid. Choice C is fructose. Choice D is gl
lactone. A lactone is recognized by the fact that it is a cyclic ester. The correct choice is D.

13. C is correct. Blood glucose would rise in response to a meal, then fall when insulin was released by the pancreas
response to elevated blood glucose. A slight dip is common when the glucose level is lower than at the
state, and then it corrects itself. Blood glucose neither rises linearly nor falls linearly. Choices A and D are i
Blood glucose does change in response to a meal, so choice B is incorrect. The correct choice is C.
14. A is correct. If we read the standard curve, this is an easy question. Find 1250 on the y-axis. Move horizontally u
you intersect the line of the standard curve. Move down to touch the x-axis, and read off the correct answer, I
The correct choice is A.
15. A is correct. HzOz from any source would reach the electrode and be oxidized, although the person reading the
results assumes that only HzOz from the glucose oxidase reaction is reacting. However, if extra HzOz somehou
into the machine, then the reading would be misleadingly high. Choices B and C are incorrect. HzOz does
oxidize glucose. The correct choice is A.
16. C is correct. If the filter blocked hydrogen peroxide, then nothing would reach the electrode and be oxidizsd
make a signal. Choice A is incorrect. If many oxidizable substances passed through the filter, then the readi
the electrode would be amplified by other oxidation reactions. This would make an incorrect reading of p
glucose. Choice B is incorrect. Choice C is the correct answer: The filter keeps large oxidizable substances
from the electrode. Choice D is incorrect. The filter in question is downstream from the immobilized enzyme.
does not protect the enzyme. The correct choice is C.

17. B is correct. Oxidation occurs at the anode. The hydrogen peroxide is oxidized. The platinum acts as a
agent. Platinum acts as an anode. Choices A and C are incorrect. An oxidase is an enzyme. It plays a role in the
reaction. Choice D is incorrect. The correct choice is B.

18. A is correct. In the uncoupling situation, fewer ATPs are produced per unit of fuel. The dissipation of
electrochemical H@ gradient produces heat, which is probably important in regulation of body temperarlr
neonate.s. Choice B is incorrect. More fuel must be burned to make ATP than in the regular, coupled state. Bodl'
stores would be recruited, not conserved. Choice C is incorrect. All this oxidation would lead to an increase in
resting metabolic rate. Choice D is incorrect. The correct choice is A.
19. C is correct. The passages tells you that the number of mitochondria is greatly increased in BAT versus
"Chrome" usually refers to a colored substance. The cytochromes contain iron and are brown in color. The WA
the same cytosolic components as the BAT, so they are not causing differences in color between the two tt
tissue. Choices A, B, and D are incorrect. The correct choice is C.

20. D is correct. The diagram is a second messenger system. The hormone does not enter the cell, but relies i
its communication with the cell interior via the second messenger, which is a G protein, in this case. Choice
incorrect. The s-subunit is involved in activating adenylate cyclase. The p- andy-subunits are neutral in this
Choices B and C are both incorrect. The correct choice is D.

21. B is corrett. The fat cell is a storage depot for triglyceride. A triglyceride contains glycerol esterified to three
acids. It is neutral and hydrophobic. This is the form that is used for storage in fat cells. Fatty acids are presenl
blood and are oxidized for fuel in the tissues. Phospholipids are located in the membranes, predominantly. Gly
is a component of triglycerides, and it also exists free (unbound to anything) in the blood. The comect choice fu

Copyright @ by The Berkeley Review 16,4 The Berkeley


Specializing in MCAT
Biology Metabolic Components Section VII Answers

1) A is correct. Although the adipose tissue does not perform gluconeogenesis, the liver tissue does and can use the
released glycerol for the production of glucose. This plays a role in glucose homeostasis. BAT does not contain
glycogen or release glucose. Choices B and C are incorrect. Fatty acids cannot be made into glucose, by
gluconeogenesis or any other pathway in the human body. Choice D is incorrect. The correct choice is A.
11
C is correct. Thermogenin is probably most like a protein with a similar role in the body. Chymotrypsinogen is a
hormone that, when cleaved properly, digests certain peptide bonds in food. Choice B is incorrect. Hemoglobin is a
transporter of oxygen in the blood. Choice A is incorrect. Choices C and D exist in the membrane of cells and allow
things to transport through the membrane. However, ATP-ADP synthetase is located in the mitochondrial inner
membrane and allows protons to cross, driving ATP synthesis. Thermogenin allows protons to cross the inner
mitochondrial membrane for free (i.e., without an input of energy). The sodium-potassium pump requires energy.
Choice D is incorrect. The correct choice is C.

t1. B is correct. This question can be answered using previously acquired knowledge. The human red blood cell
contains hemoglobin molecules. Therefore, if we know where the human red blood cell is degraded, we know where
the breakdown of heme occurs. A typical human red blood cell has a life span of 120 days. Old cells are removed
from the circulatory system and degraded in the spleen. The correct choice is B.

{lflll
D is correct. Looking at Figure l, we see that the way to determine whether the conversion is a reduction or an
ilIfi
oxidation is to make note of the cofactor involved (in this case, NADPH). The cofactor itself is oxidized as a result
rfl0
of the reaction, and the oxidation involved two electrons. The two electrons given off had to go somewhere, and
they went to the reduction of biliverdin into bilirubin. Remember, we cannot have an oxidation without a reduction.
Therefore, we can conclude that the conversion is a two-electron reduction. The correct choice is D.
80tI
A is correct. We have to look at the two structures given in the question. First, there is no difference in the chemical
ritml
constituents of the two molecules. However, the atoms are arranged differently. For this reason, an isomerase is the
ml
enzyme that would be involved. Next, in looking at the picture one should arrive at the conclusion that uro I is a
lml
symmetric molecule, symmetric around the center point of the molecule (where the iron would be bound). By
srvitching one set of constituents, uro III is produced. Switching these constituents produces an asymmetric
molecule. The correct choice is A.
C is correct. The passage informs us that carbon monoxide is released when the heme group is converted to
biliverdin. From one's previous knowledge of Hb, one should remember that carbon monoxide has a very strong
allinity for the iron atoms of the hemoglobin molecule. That is why the gas is poisonous. Judging from this reaction,
\\ e can assume that the body naturally produces a small amount of this gas. However, the amount is very small
relative to the amount of oxygen in our body. Nonetheless, we cannot ignore the fact that CO is indeed produced,
and a small percentage will indeed bind to Hb. The correct choice is C.

:$" C is correct. The passage informs us that persons suffering from this disease have abnormal red blood cells that are
eliminated by the body. Therefore, we can assume individuals with this disease have a lower than normal red blood
:ell count. Does the red blood cell level contribute to the partial pressure of oxygen? The answer is no. The partial
Jressure of oxygen is determined only by the amount of oxygen dissolved in the blood, not by the amount of oxygen
:ound to hemoglobin. For this reason, persons with CEP would not have a lowered partial pressure of oxygen. The
correct choice is C.
.\is correct. This is a very straightforward genetics question. We know from the passage that the disease in
question, CEP, is transmitted in a autosomal recessive fashion. The father is heterozygous for this trait, making his
:.notype Cc (letters chosen arbitrarily). The mother is a homozygous dominant, making her genotype CC. The
llrestion asks for the probability that a son will suffer from the disease. Doing the cross between parents, it becomes
rpparent that no child will suffer from the disease, as no recessive allele is contributed by the mother. Therefore,
::ere is aOTo chance. The correct choice is A.

frlt B is correct. Trypsin is an enzyme that cleaves peptide bonds on the carboxyl side of the amino acids arginine (Arg)
r:rJ lysine (Lys). The important clue to answering this question comes from the last sentence of the second
:aragraph. It says, "...enzymes readily degrade polypeptides synthesized from L-amino acids, but not D-amino
,;ids." This tells us that we must find that structure with the two amino acids in the L configuration. How do we do
::.,s? We need to recall our stereochemistry from organic chemistry.

il.r r))'i! "*


-ci'. 9 by The Berkeley Review 165 The Berkeley Review
Specializing in MCAT Preparation
Biology Metabolic Components Section VII Answers

Recall that a Fisher projection is a way of representing a tetrahedral carbon (and its substituents) in three
dimensions. The structures of the dipeptides in the question are drawn in Fisher projections. Each dipeptide has trr "
chiral carbon atoms, each with four different substituents. Focus on the four bonds attached to those chiral carbon'
Vertical lines represent bonds pointing away from your point of view, while horizontal lines represent bontJ:
pointing towards your point of view.

Rank the four substituents on each chiral carbon in order of decreasing priority. Arrange the order so that thr
substituent with the lowest priority is on top. In each case the substituent with the lowest priority is hydrogen (H). Ii
two substituents need to be exchanged so that hydrogen is on top, then do not forget to exchange another pair cr
substituents so that the absolute configuration is retained. Next, trace the order of priority of the three remaining
groups. If thetracing is in the clockwise direction, the stereocenter is designated as R (rectus, Latin for right). If the
tracing is in the counterclockwise direction, the stereocenter is designated as S (sinister, Latin for left). However"
since amino acids are designated by the older D and L prefixes, we must make a conversion. It turns out that the oltj
D prefix is analogous to the old R prefix, and the L prefix is analogous to the S prefix. We are now ready to find the
answer.

Consider Structure I (see below). Both chiral carbon atoms have their hydrogens pointing up. All we need to do is
arrange the substituents in order of priority. Let's do lysine first. Nitrogen (N) has an atomic number of 7, while
carbon (C) has an atomic number of 6. Nitrogen is assigned the highest priority. Next, we need to distinguish
between the carbons attached to the chiral carbon. The carbon atom with the oxygen atom attached has the nerl
highest priority. This is followed by the methylene (-CHz-) carbon atom. The tracing is from nitrogen to carbonln
carbon to methylene carbon, giving a clockwise direction. This stereocenter is R, which in the old nomenclature i:
D. Right away, we know that Structure I cannot be the answer, because we need both chiral centers to be in the L
configuration. We just found that one chiral center is in the D configuration.

R (D) R (D)
HO
@ru ro
H1N- C- C- N_ C_ COO
;rb ta I
HCHr_ b
"?1
CH,
t- Alanine (Ala)
CH.
I
CH, I
t'
o NH3

Lysine (Lys)

Let's continue this example by finding the configuration about alanine's chiral carbon. Again, the hydrogen atom r$
pointing up and therefore has the lowest priority. Nitrogen still has the highest priority, followed by the carbor$
carbon, andfinally the methyl carbon. The tracing is again clockwise, giving the D configuration.

D D L
HOH^ CH.OH
olll lo otttHOCH, rO O l- ll l(9
OOC_ C- N - C_ C_ NH" H1N-C-C-N-C-COO ooc- c- N - c- c- NHI
trl rtt llt
H.C H CH. CH,HH HHCH,
't' I I

CHt CH. CH,


I I t-
II
CHt

i"' fn, ilI


NH
IV fn'
NH
I I I

O NH3
,caa @ OC.a\
HzN NHz H:N NH,

Arginine (Arg)

Following this same procedure for Structure II, we get a configuration of L-Lys and L-Ala. Structure III has
configuration of D-Arg and L-Ala. Structure IV has a configuration of D-Ala and L-Arg. The correct choice is B

Copyright @ by The Berkeley Review 166 The Berkeley


Specializing in MCAT Prepar
Biology Metabolic Components Section VII Answers

31. D is correct. As stated in the fourth paragraph of the passage, DIPF is an irreversible inhibitor. Since this inhibitor
reacts with active site serine residues, it must enter into the active site, bind to a specific serine residue (see below),
and shut the enzyme down. Allosteric inhibition refers to the fact that binding is taking place at a site other than the
active site. Since this is not the case here, we can eliminate choice A.

HrC- Serine

F o
I I
{HrC)2HC- O- P- O- CH(CHj}2 "p Serine-CH'OH + (H3C)2HC-O-P-O-CH(CH3)2 + HF
il il
o o
DIPF Phosphorylated active site

If DIPF were a competitive inhibitor, it would mean that it is of the reversible type, which is not what is stated in the
passage. We can eliminate choice B. What about phosphorylation of ADP? If DIPF could phosphorylate ADP (and
in the process form HF), then it would be unable to react with the active site serine residue. This allows us to
eliminate choice C. In order for DIPF to bind irreversibly to the active site serine residue, it must phosphorylate that
residue as indicated above. The correct choice is D.

1' B is correct. Notice that initially (at low [S]) the reaction velocity for Curve II is lower than the reaction velocity for
Curve I. However, as the [S] is gradually increased, the reaction velocity of Curve II approaches that of Curve I.
Eventually, the two reaction velocities (in theory) will reach the same velocity (i.e., V63*). This is exactly what is
mentioned in the last paragraph of the passage. The correct choice is B.

11 A is correct. As outlined in the lastparagraph of the passage, competitive inhibitors show the same maximal
velocity. This means that the maximal velocity of the competitive inhibitor crosses the y-axis on the graph at the
same point as the maximal velocity of the control. Only Lines I and III intersect at that point. This allows us to
eliminate choice B and choice D.

The most effective competitive inhibitor is that inhibitor that requires more substrate to reach half-maximal
saturation. In other words, the most effective competitive inhibitor is that inhibitor with the largest apparent Kvr.
This would give the smallest value for -llKrrl. Another way to look at this is that the most effective competitive
inhibitor is indicated by the line with the steepest slope. The correct choice is A.
3-t. C is correct. Almost all enzymes are proteins. Since proteins are composed of amino acids, they are quite sensitive
to pH changes. We are told in the question that aspartate 52 (Asp-52) has a pKn of about 4.0, while glutamate 35
(Glu-35) has a pKs of about 6.0. Based on our discussion of pKns we know that at the pIG of a particular amino acid
side chain the dissociable hydrogen atom is half on and half off. The dissociable hydrogen on the side chain spends
half of its time in the protonated state and half of its time in the deprotonated state.

At low pH values (close to l), we would expect both the Asp-52 and the Glu-35 side chain carboxyl groups to be
protonated. As the pH begins to increase and approach the pK2 of Asp-52, we find that the side chain carboxyl of
Asp-52 begins to lose its hydrogen atom. That carboxyl group begins to take on a negative charge. By the time we
are one full pH unit away from the pKn for Asp-52 (at a pH of 5), we find that'the side chain carboxyl group spends
most of its time negatively charged. This negative charge stabilizes the substrate at the active site. At a pH of 5, we
are one full pH unit below the pKa for Glu-35. This means that the side chain carboxyl of Glu-35 still has its
dissociable hydrogen atom. It is this hydrogen atom that eventually gets transferred to the substrate at the active site.
As we increase the pH even more, say, to one full pH unit above the pK3 for Glu-35, we find that the side chain
carboxyl of Glu-35 now spends most of its time negatively charged. Since we do not have a proton to donate to the
substrate anymore, the reaction rate begins to decrease. Therefore, we would expect an increase in reaction rate as
we approach a pH of 5, and a decrease in the reaction rate as we move to pH values higher than 5. This is exactly
what we see in the bell-shaped curve in Graph III. With this information, you should be able to reason why Graphs I,
II and IV are invalid solutions. The correct choice is C.
35. B is correct. Aspartate-52 provides the general base catalysis, because its side chain p-carboxyl group is about 97o
protonated at a pH of 5. At a pH of approximately 5, we find that Asp-52 would acts as the general base (-COOo),
while Glu-35 would act as the general acid (-COOH). Without even dealing with percentages, we can immediately
eliminate choice A and choice D.

- . lnght @ by The Berkeley Review 167 The Berkeley Keview


Specializing in MCAT Preparation
Biology Metabolic Components Section VII Answers

Let's consider choice B and choice C. In choice B, we are told that G1u-35 acts as the general acid, while Asp-52
acts as the general base. We know this is correct. Are the percentages correct? Will the side chain carboxyl group of
Glu-35 be about 9Vo deprotonated at a pH of 5? Let's give a rough estimation first. At a pH of 6, we know that the
side chain carboxyl of Glu-35 is 507o protonated and 50Vo deprotonated. This is because the pKa of the side chain
carboxyl of Glu-35 is given as 6. We can see this quantitatively by using the Henderson-Hasselbalch equation, as
shown in the first column below:
Using pH = 6 Using pH = 5

pH = pK2 + tog J4I pH = pKn + tog [A-]


tHA]

6=6+log [A-] 5=6+log [A-]


tHA] [HA]

o= 1o, [A-] -t = 1o, [A ]


IHAI tHA]

100- [A ] t'-t _ [A] or t'r= [HA]


IHA] [HA] tAl
[Al or [HA]=[A] [Ha] or l0 [HA]
1= tO =
tHA] tA-l l tAl

Let's try this same approach with a pH of 5. This is shown in the second column above. At a pH of 5, the side
carboxyl of Glu-35 is more on the protonated side and less on the deprotonated side. After applying the
Hasselbalch equation, we end up with l0 = [HA]/[Ao]. This states that there is ten times more of the proto
(HA) form of the carboxyl side chain than the deprotonated (Ae) form. In other words, Glu-35 is about 9l
protonated or about 9Vo deprotonated. A similar calculation can be made for Asp-52.

The best way to get a feel as to whether or not an amino acid is protonated or deprotonated, and by how much, is
use the Henderson-Hasselbalch equation in a series of simple calculations. Start like we did in the first col
where the pH is equal to the pKn of the dissociable hydrogen of interest. Next, move one full pH unit away from
pKa (in either direction). You will find results similar to the ones in the second column. Move two full pH
away from the pK6 of interest. Try three full pH units away. You should begin to see a pattern develop. In fact,
you understand this pattern, you will not need to use the Henderson-Hasselbalch equation for the calculation. Y
will just be able to do a very quick and fairly accurate estimation. The correct choice is B.
36. C is correct. The maximal velocity of the reaction is taken from the intersection of the line with the y-axis.
value of the point at that intersection is 2. All we need to do is remember that we have a reciprocal. Therefore,
ll2.The correct choice is C.

37. D is correct. An oxidoreductase catalyzes a reaction in which one substrate is oxidized and another is
'Choice A is incorrect. A hydrolase catalyzes the hydrolytic cleavage of singe bonds. Choice B is
transferase moves groups from one substrate to another. Choice C is incorrect. An isomerase moves a group
one position to another within the same molecule. The correct choice is D.

38. A is correct.This is a question of which came first, Btz or folate deficiency. If Btz is low (i.e., a Btz defic
the enzyme homocysteine methyltransferase can't work properly. Any THF that is around is trapped as methyl-
at this enzyme. Folate functions as THF in many aspects of one-carbon metabolism; and in this case, it is deli
a methyl group to homocysteine methyl transferase. Even if folate status were normal, this trap due to
deficiency would cause a secondary folate deficiency. The correct choice is A.

39. A is correct. An intramuscular injection of Brz bypasses the problem with IF and makes the vitamin
available to the blood. Statement I is correct. An increase in dietary B tz would not help if the IF were not
because the vitamin is absorbed as a complex with IF. Statement II is incorrect. Co itself is not the vitamin-
vitamin contains Co in the Btz molecule, but Co by itself would be of no benefit. Statement III is incorrecl
correct answer is A.

Copyright @ by The Berkeley Review l6a The Berkeley


Specializing in MCAT Pre
Biology Metabolic Components Section VII Answers

40. B is correct. If you remember the anatomy of the GI tract, the ileum is the last section of the small intestine, and the
colon or large intestine follows it. The receptor of Btz is in the ileum, so any B12 produced past this receptor (in the
colon) is not absorbed. The makes choice A incorrect. Choice B is correct. Choices C and D imply that the colonic
B tz is absorbed to have reactions on enzymes in the body, and are therefore incorrect. The correct choice is B.

41. A is correct. If methyl malonyl-CoA cannot be metabolized into succinyl-CoA, then the CoA is hydrolyzed and the
excretion product is MMA. Choice A is correct. Brz is not a competitive inhibitor of the MMA pathway. Choice B
is incorrect. MMA is not a by-product of homocysteine methyl transferase; this is another enzyme altogether.
Choice C is incorrect. MMA is not made from IF. Choice D is incorrect. The correct choice is A.

42. D is correct. Met is an essential amino acid. This may seem like a trick because of the reaction with homocysteine
methyl transferase. However, this homocysteine had to be made using a Met in the first place. Choice A is true. We
can see from the diagram that choice B is true. Met is used in gluconeogenesis, so choice C is true. The precursor of
Tyr is Phe, so choice D is false. Since you want the false answer, choice D is it. The correct choice is D.
43. B is correct. IF is a glycoprotein, which is a protein-carbohydrate combination. Just like the protective mucus of the
GI tract, glycoproteins have nondigestible carbohydrate bonds. IF is protected from digestion, because our bodies do
not have the digestive enzymes to break it down. Choice A is incorrect, because pepsin (a protein-digesting enzyme)
is present in the stomach. Choice C is incorrect, because IF does not influence the activation of pepsin. Choice D is
incorrect, because the passage tells us IF makes it to the ileum from the stomach. It must be resistant to gastric
enzymes. The correct choice is B.

44. B is correct. The values of the reactants and products at equilibrium is 10 moles. Using the equation AG'' = -2.3 RT
log Ksq, where.Ksq = [C]/[A][B], we find that AG"'= + 2.3 RT.

AG'' = - 2.3 RT log K". -) AG"'= - 2.3 RT log [C] = - 2.3RT log [10]
tAl IBI tl0t [10]

AG'' = - 2.3 RT log 0.10 --) AG"'= - 2.3 RT (- 1.0) = + 2.3 RT

The standard free-energy change is an important concept to understand, because it will be able to tell us whether a
reaction is spontaneous (exergonic) or nonspontaneous (endergonic). The correct choice is B.
{5. A is correct. Lyases are involved in the cleavage of bonds like C-C, C-N, and C-O. Lyases can cleave double bonds
to make single bonds, or single bonds to make double bonds. Functional groups can be either added across or taken
away from these bonds, respectively.

OO
coo coo
ill
CH Fumarase HO - C- H
ll +H"O+l
'
cH CHt
lolg
coo coo
Fumarate Malate

In the example of fumarate going to malate, we see that water is being added across a double bond. This type of
reaction occurs quite frequently in biochemistry. If we add the element of water to a bond, it is referred to as a
hydrationreaction. Ahydrolysis reaction involves the use of water (hydro-) to break (-lysls) abond. The correct
choice is A.
16. C is correct. Hexokinase aligns the 1-phosphoryl group of ATP with the C-6 hydroxyl of glucose and catalyzes a
transfer of that y-phosphate group to the C-6 position of the sugar. Since this reaction involves a chemical group
transfer, the enzyme is a transferase.

Ligases involves the hydrolysis of a high-energy bond, such as those found in ATP; but the energy in that high-
energy bond is used to drive the condensation of two molecules via a ligation reaction. Hydrolases involve the use of
water to break a specific bond. Oxidoreductases are enzymes involved in electron transfer through oxidation-
reduction reactions. The correct choice is C.

- rr:''right @ by The Berkeley Review 169 The Berkeley Review


Specializing in MCAT Preparation
Biology Metabolic Components Section VII Answers

o o
o
o t/> oI o
o- P- o- P- O- AMP ol A
,I lt
(o o o- P= o
I

;o.' a
o
HzC H HzC

l-o on
,/f__ o
on
1n\ Hexokinase "r.

T. "si?
I

"l\Y OH OH

Glucose Glucose-6
phosphate

47. C is correct. We can think of the nucleophilic properties of the C-6 hydroxyl of glucose and water as being similar,
if we consider both molecules as having the R-OH format. The R for water is a hydrogen atom, while the R fr
glucose is the rest of the molecule other than the C-6 hydroxyl. Even though the hydroxyl functional group in botl
cases acts as a nucleophile, the R group in each case is quite different.

The presence of glucose at the active site (along with ATP) induces a large conformational change in the enzym
and allows the active site to close around the substrates with the exclusion of water. If water is excluded from rh
active site, the polarity of the active site's environment is reduced. The active site becomes hydrophobic. This
that the C-6 hydroxyl of glucose and the y-phosphoryl group of ATP are not solvated. Therefore, the C-6 hr
group's oxygen atom becomes more nucleophilic, while the y-phosphoryl group's phosphate atom becomes
electrophilic. These events allow for an accelerated transfer ofthe phosphoryl group to glucose. If water were al
position occupied by the C-6 hydroxyl group, it would be able to attack the y-phosphoryl group of ATP. The
would be the hydrolysis of ATP to ADP and (inorganic) phosphate. In other words, the rate of ATP hydrolyri
would increase. As stated in the question, this is not what is observed. The correct choice is C.
48. B is correct. It is important to know the differences between an aldose sugar and a ketose sugar. Sugars
saccharides, polyhydroxyl carbonyl compounds that contain an aldehyde or a ketone functional group and at I
two hydroxyl groups. Sugars with aldehyde functional groups are referred to as aldoses, while sugars with ket
functional groups are referred to as ketoses. Even though you do not need to know the complete structures of
aldose and the ketose in the second step of glycolysis, you do need to know (recognize) the aldehyde and
functional groups of a sugar. We can draw them as shown below:
o-H
--c,
; EnzYme cH2oH
H_C_OH + C=O
l- H Ho-l -
_lt "o*
Aldose
"
Ketose

What type of enzyme assists in the conversion of an aldose into a ketose? Notice that in going from an aldose
ketose, there is no loss or gain of atoms. These two molecules differ in the sequences in which their atoms are
together. The empirical formula is the same in both cases. These molecules are isomers of one another.
the type of enzyme that catalyzes this reaction is an isomerase. The characteristics of the other three choices
outlined in the passage in Table l. The correct choice is B.

49. B is correct. The pK2s for these amino acid pairs are given in Table 2 in the passage. The optimal activity of
enzyme is where the initial velocity is the greatest, which is at V6ax (i.e., the maximal velocity). The pH at V
about 9.0. We want that amino acid pair whose pKa values border as close as possible to a pH of 9.0.

We can get a'rough estimate of this by adding the two pKa values of the amino acid pair together and then di
by 2. For choice A, we get a pH value of 4.95, which is way to the left of the graph and nowhere near the
activity of the enzyme. Choice B gives a pH value of 9.2, which is extremely close. Choice C gives a pH v
I 1.65, which is a little further away. Choice D gives a pH value of 9.6, which is close to the optimal activity,
as close as choice B. The correct choice is B.

Copyright @ by The Berkeley Review 170 The Berkeley


Specializing in MCAT
Biology Metabolic Components Section VII Answers

50. C is correct. The passage mentions that lactate dehydrogenase (LDH) has /wo different types of subunits,
designated as M (for muscle) and H (for heart). Let's assume that the band shown in Lane I is composed of all M
subunits (i.e., Mx, where x is the number of subunits), while the band shown in Lane 3 is composed of all H subunits
(i.e., Hx). Therefore, the bottom band in Lane2 is all M and the top band is all H. The three bands in the middle are
mixtures. This is indicated in Figure I below.
Lane 1 Lane2 Lane 3 Lane 1 Lane2 Lane 3
(+) (+)
ll:rrr:]:] flir:l]l 5 L:--= Frl -= sEE H4
a
\./\
r------------r ,/ 4
r-\,/ \,/ + !! u1H,
gl Hx gL H,
x\-
=J
I I
J lttt
x\- r !! tvt2H,
z >
t L_____J 2 Lt------ 2 !l vr.H'
Origin ".r::-:" lI rr
t"t4
Origin I 1----. 1
ll
c) (-)

MX MX

Figure 1 Figure 2

Consider Band 3 in Lane 2. This band falls halfway between the isoenzyme that is all M and the isoenzyme that is
all H. We would expect the isoenzyme indicated by Band 3 to contain an equal mixture of M and H subunits. If
Band3weretocontainoneMandoneHsubunit,thenBandlwouldcontain2Msubunits,andBand5would
contain 2 H subunits. This means that Band 2 would have to contain 1.5 M subunits and 0.5 H subunits. Similarly,
Band 4 would have to contain 0.5 M subunits and 1.5 H subunits. Since we cannot have anything but a whole
number for a subunit (otherwise, the subunit would be nonfunctional), we need to make an adjustment in our
analysis.

Instead of a 1:1 mixture of M to H in Band 3, let's assume a 2'.2 mixture of M to H. This would allow Band 1 to
contain4MsubunitsandBand5tocontain4Hsubunits.Band2nowcontainsaratioof3M:lHsubunits,while
Band 4 will contain a ratio of 1M:3H subunits. This is shown in Figure 2 above. This analysis fits the gel pattern and
indicates that there are 5 isoenzymes of LDH, each being a tetramer. The correct choice is C.

A is correct. Bacterial cells that are Gram-positive have a plasma membrane surrounded by a thick peptidoglycan
layer, the cell wall. Within this layer are two types of sugar residues, N-acetylmuramate (NAM) and N-
acetylglucosamine (NAG), which can act as a substrate for lysozyme. This enzyme has a clear path to these sugars.
In a-Giam-negative bacterium, there is an additional outer membrane that contributes to the bacterial cell wall. This
outer membrane acts as a barrier that helps to prevent lysozyme from degrading the peptidoglycan layer. If bacteria
are missing,their cell walls, chances are that they have already lysed due to osmotic differences between the inside
of the cell ind the outside of the cell. If they have not lysed but have formed protoplasts instead, then the lysozyme
enzyme is facing a plasma membrane that does not have the NAM and NAG sugar residues linked as they were in
the peptidoglycan layer. The correct choice is A-
i,] D is correct. You need to recall your carbohydrate chemistry to answer this question. Rings D and E have been
labeled with the C-1 and the C-4 positions already (see Figure 2 in the passage). In order to determine the
configuration of the linkage, we first need to locate the anomeric carbon atom of the ring'

The anomeric carbon in the open-chained form is the most oxidized carbon atom. That turns out to be the C- I carbon
or the carbonyl carbon of the aldehyde functional group. When the ring begins to close, the oxygen of the C-5
hydroxyl group attacks the C-1 carbon. The carbonyl oxygen at the C-l position picks up a hydrogen atom to form
the C-i hydroxyl. That hydroxyl can reside either above or below the plane ofthe_ring, depending on the direction
of attack. If the C-l hydroxyl is above the plane of the ring, it is said to be in the p-configuration; if it is below the
plane of the ring, it is in the cr-configuration.

By examining the linkage between rings D and E, we see that the hydroxyl at the C- 1 position of the D ring is in the
0lconfiguration. The linkage extends from the C-l carbon of ring D to the C-4 carbon of
ring E. Hence, the
configuration of the linkage is said to be 0(1+a). The correct choice is D.

: . right @ by The Berkeley Review l7t The Berkeley Keview


Specializing in MCAT Preparation
Biologr Metabolic Components Section VII Answers

53. A is correct. All enzymes are based on the type of reaction they catalyze. Many enzymes are known by a common
name, usually derived from the principal reactant involved in the catalysis. For example, arginase reacts with the
amino acid arginine.

Arginase falls within a specific class of enzymes, just as lysozyme does. There are six classes of enzymes
established by the Enzyme Commission of the IUPAC. In the answers to this question, we are presented with
enzymes from four of those classes. A hydrolase enzyme is involved in hydrolysis reactions. Water iJ used to cleave
bonds like C-C, C-N, or C-O. This is exactly what we see in the case of lysozyme (and what we will see in the case
of arginase in a later discussion). A transferase enzyme transfers a functional group. An oxidoreductase is involved
in an oxidation-reduction reaction. A ligase is involved in bond formation (think of ligation) that is coupled to the
hydrolysis of ATP. We do not see the last three examples involving lysozyme catalysis. The correct choice is A.

54. D is correct. Almost all enzymes are sequences of more than 100 amino acids. However, there are only a few amino
acids that can be used to make up the catalytic group of amino acids at the active site. These few amino acids form
an enzyme-substrate complex through a variety of bonding interactions. As the substrate forms product, it goes
through an intermediate species (the transition state). The transition state has a free energy that is higher than either
the substrate or the product. The charge distribution at the active site of an enzyme stabilizes this transition state,
thereby lowering the barrier to activation lAGf ) between substrate and transition state. If the barrier to activation is
lowered, the rate of a reaction can be accelerated significantly. The correct choice is D.
C is correct. The last sentence of the third paragraph of the passage reads, "The neighboring environments of Glu
35 and Asp 52 are quite different from each other." In the question it states that the pku valuls for Asp 52 and Glu
35 are usually cited as 3.9 and 4.1, respectively. The difference between the two side chains is the additional
methylene (-CHz-) group in aspartic acid. Under normal conditions, the influence of this additional methylene g1oup
has negligible consequences. Thus, under normal conditions we would expect both aspartic acid and glutamic acil
to have similar pKa values for their side chains. Since both side chains are on polar (charged) amino acids, ,yys lryerrldl
expect their environments also to be polar.

However, we know that the two environments are different from each other. We can assume that one environmenr
polar and the other nonpolar. And we know which is which, based on information contained in the question:
"Analysis of lysozyme's active site indicates that the pKn of Asp 52 is still about 3.9, but the pK' of Glu 33 is nom
about 6.6." The pK6 value of the aspartic acid side chain has not changed. The pKs of the glutamic acid side chair
has increased by 2.5 times. This tells us that Asp 52 is in a polar environment, and that Glu 35 is in a non
environment. In order to remove the dissociable hydrogen from the side chain of Glu 35, the active site must bi u
pH close to a pKn of 6.6. In other words, it is harder to remove the dissociable hydrogen from the carboxyl si
chain of Glu 35 than it is from the carboxyl side chain of Asp 52. Recall from organic chemistry that dissociation
a hydrogen from a carboxyl group is more favored in a polar environment than it is in a nonpolar environment.

Even though choice A is a correct answer, it does not address the question. What about choice B? The
carboxyl group ofAsp 52 is in a different environment than the protonated carboxyl group of Glu 35 (see Figure 2
the passage). If anything, the negative charge of the ionized carboxyl group would help stabilize the lparti
positive charge of the hydrogen atom of the protonated carboxyl group. However, in this situation the two
groups are distant from one another, and a stabilizing or destabilizing influence is negligible. In choice D, we
that if the carboxyl group of Glu 35 were in a polar environment, then its pKa value would be around 4.1. Hydrc
bonding would help in stabilization. But we already know that Glu 35 is in a nonpolar environmenr. With
reasoning, we can eliminate choices A, B, and D. The correct choice is C.

50. B is correct. This question involves careful inspection of Figure 2, Figure 4, and Figure 5 in the passage. In Fi,
2, the C-l and C-4 carbons are identified. Based on this, we can establish where the C-5 and C-6 carbons
located. We know where to find the anomeric carbon (at the C-1 position). The reference carbon is that carbon
establishes whether a sugar is in the D or L configuration. By definition, the reference carbon is the last
carbon farthest from the most oxidized carbon. In the case of sugar ring D, the most oxidized carbon is at C- l, w
the last chiral carbon is at C-5. Therefore, C-5 is the reference carbon. This is great, because it says that choice
and choice C are one in the same. Since there should not be two correct answers to the same question, we c
eliminate both df them.

We are left with choice B and choice D. After examination of the hydroxyl groups on the C-6 carbons of Figurc
and Figure 5, we can conclude that they are one in the same. We can eliminate choice D. By default, we are left ni
choice B.

Copyright @ by The Berkeley Review 172 The Berkeley


Specializing in MCAT
Biology Metabolic Components Section VII Answers

Main Chain of Lysozyme Enzyme Main Chain of Lysozyme Enzyme


I
Glu 35 |
cru ls I
.c- O otttrrrtrrttrtt
_
o',Ct o - H
'rt H
CH"OH I
o t1' o
lrttttt"ttttttttt'
Rings A-B-C Rings A-B-C,

NAM N-H NAM N-H


^-l
u -L
o
o_ ^._o o=
l^
fcHr ol^-_o
CHr i i
I
I
Asp 52
I
nsn s2
Main Chain of Lysozyme Enzyme Main Chain of Lysozyme Enzyme

However, by examining Figure 4 and Figure 5, you should be able to see how the hydroxyl group of the water
molecule forms a bond with the C-l anomeric carbon, and how the remaining hydrogen of the water molecule forms
a bond with the carboxylate oxygen of Glu 35. This is indicated by the dashed lines in the two diagrams shown
above. The correct choice is B.
:/. A is correct. General acid-base catalysis involves the transfer of a proton in the transition state (see the protonated
form of Glu 35 in Figure 2 in the passage). This transfer does not involve covalent bond formation.
A covalent bond between two atoms like C and H involves the sharing of a pair of electrons. When a C-H bond
breaks, the electrons must do something. If one electron leaves with the hydrogen and the other stays with the
carbon, the reaction is referred to as homolytic cleavage, and two radicals are produced (e.g., H. and C.). If both
electrons leave with one atom, the reaction is referred to as heterolytic cleavage (e.g., .6.e and Ho or Co and
'11'o). In Figure 3 of the passage, we see a carbocation at the anomeric position. The correct choice is A.
58. C is correct. Glutamic acid is an acidic amino acid with a negatively charged side chain atpH7.4. The side chain is
polar and tends to interact with other polar molecules (like water). We would expect this amino acid to be found on
the outer surface of the enzyme, where it can be in contact with the aqueous medium. Note that in the diagram, the
a-helical region is towards the outer surface of the enzyme. We would not expect glutamic acid to be buried in the
interior of the protein, where the amino acid residues tend to be quite hydrophobic and nonpolar. Finally, as outlined
in the passage, Glu 35 (glutamic acid) is part of the catalytic group at the active site of the enzyme. The correct
choice is C.

:9. C is correct. When the substrate concentration is quite low, [S] is insignificant compared to Ku. In the
denominator, Krra + [S] becomes Krr,r + [0], which is just Ky. What this is telling us is that the initial reaction
velocity is almost proportional to [S] at very low values of [S]. In other words, at low [S] we observe a straight line
in the graph in Figure 1 of the passage. This is characteristic of a first-order reaction. The correct choice is C.

'ultrt-
B is correct. When [S] is quite high, it means that Ky is quite small. In the denominator, Ku + [S] becomes [0] +
[S], which is just [S]. This reduces the Michaelis-Menten equation to the following:
Vt^* [Sl=
v= vmax
tsl
In other words, at very high [S] the reaction approaches some maximal velocity (V.ur). Note that the velocity of this
reaction is now independent of [S]. This is characteristic ofa zero-order reaction. The correct choice is B.

n,[- D is correct. If [SJ = Krra, then substitution into the Michaelis-Menten equation reduces it to:

y V.u* [S] V.u* [S] V*u*


= = =
lsl + tsl 2tsl 2

This expression is telling us that Ky is that [S] where the reaction operates at one-half its maximal velocity. The
correct choice is D.

|,.'flr'!nsht @ by The Berkeley Review 173 The Berkeley Review


Specializing in MCAT Preparation
Biology Metabolic Components Section VII Answem

62. C is comect. This question can best be answered by considering the reaction of E and S. As E and S combine,
[Sl
and [E] both decrease in order to make the ES complex. Therefore, we would expect the
[ES] to increase. As t]c
[ES] begins to increase, some of it begins to get converted to P. As a result, the tpl begins io increase. The corrett
choice is C.

E kr ks
C) E+S:.ES-E+p
kz
U

Time

63. A is correct. The reciprocal of the Michaelis-Menten equation is given below. This formula is called
Lineweaver-Burke equation. Note that it is in the form of the general y-iniercept equation for a straight line: (y
=
+ b).

t=/ru\ t + I
v \v.u*/[Sl v-^,
The y-axis is l/v. The slope of the line is Ku/Vmax. The x-axis is l/[S]. The y-intercept is l/Vmax. How do we
the value of Klt? In Figure I of the passage, Ku is a particular [S] at which the enzyme is half-maximally satt
In this case, Ku is also a particular [S]. However, the Ku here is a reciprocal value (i.e., l/Krvr). And since we
a quadrant that has a negative sign notation for the x-axis, we place a negative sign in front of l/Ky. It now
l/Krra. All we need to do now is extrapolate the line from the y-axis to the x-axis as shown below.

e^vapolation/ ,

/i
(e/
r2345 -4 -3 -2 -1 0 I 2 3 4 5
14sl - l/Kv l{sl
The rest is just algebra. We find that:
l-.
Krr'r

K1a=-1 =+0.5
-2

The Krr,l value in Figure I of the passage is a positive value. Likewise, the Ku of the double reciprocal plot
also be a positive value. The correct choice is A.

64. c is correct. In order to answer this question, we must consider the values for Ku and kcat in Table I in the
If an enzyme is (almost) catalytically perfect,
it must mean that it is extremely efficient at what it does. A
an enzyme's efficiency is given by kcat/Ku (from the passage). We also know that rate constant kt has I
between 108 to lOe M-lsec-I. This is tetling us that if kr has these limits, then kt must have them as well
amount of product that can be made is determined by how much of the ES complex there is available; and
availability is determined by how fast E and S can come together. You do not ne;d a calculator to do this pr
All you need to work with are the exponents.

Copyright @ by The Berkeley Review t74 The Berkeley


Specializing in MCAT
Biotogy Metabolic Components Section VII Answers

Consider choice A: Carbonic anhydrase has a Kv =2.6x 10-2 M and a kqal = 4.0 x 105 sec-1.If we consider the
exponents alone, we find that kcat/Ku = 195716-2 = 107 M-lsec-I. The actual value is 1.5 x 197 14-1r""-1.
Chymotrypsin has a Kv = 6.6 x 10-4 M and a kcat= 1.9 x ld sec-1.If we considertheexponents alone, we find that
k"^t /Kr,r = 192119-+ - 106 M-lsec-1. The actual value is 2.9 x 1gs 14-1t."-1.

Therefore, for carbonic anhydrase, the efficiency is about 197 14-1t."-1. For chymotrypsin, the efficiency is about
196 14-1t""-1. These exponents (107 and 106) are not too far away from the limits of 108 to 199 14-1r".-1. Therefore,
this is a pretty efficient enzyme pair. But we do not know if it is the ruosr efficient pair.
ConsiderchoiceB: AcetylcholineesterasehasaKlr=9.5x10-5Mandakcat=1.4x104sec-1. Ifweconsiderthe
exponents alone, kcat /Krr,r - 104i10-5 - 109 M-tsec-l. From choice A, we know that the efficiency for carbonic
anhydrase is 107 M-lsec-1. These two sets of exponents (109 and 107) are even closer to catalytic perfection.

ConsiderchoiceC: FumarasehasaKu=5.0x 10-6Mandak,r.r1 =8.0x 102sec-l.Therefore,kcat/Ku -l}2lrc'6 -


168 14-1r""-1. From choice B, we know that the efficiency of acetylcholine esterase is 109 M-lsec-I. So far, these
two sets ofexponents (108 and 109) appear to be the most effictent.
ConsiderchoiceD:PepsinhasaKl,,r=3.0x10-4Mandakqnl=5.0x10-lsec-l.Thekcat/Ku=16-1719-4,whichis
193 14-1t..-1. From choice A, we know that the efficiency of chymotrypsin is 196 14-1r""-1. These two sets of
exponents (106 and 103) make this enzyme pair the least efficient. The correct choice is C.
b5. A is correct. According to the passage, the rate constant k1 (for the reaction shown below) has limits between 108 to
1ge 14-lr-1.
kr,h,+p
E+s--I!ES
k2

This value is predicted from diffusion theory and represents a very extreme case, where (k: >>> k1) and every
substrate that collides with an enzyme is converted into an ES complex. This means that the efficiency of the
process is determined by how fast a substrate can be placed into the active site of the enzyme (forming the ES
-omplex).
Having limits between 108 to 169 14-lt-1*"ans that the enzyme is near catalytic perfection. Since triose
phosphate isomerase has an efficiency of 2.4 x 108 M-ls-1, it means that this enzyme is almost perfect. There is little
reason for it to change. This leads us to choices A and B.

Now, did this change occur early or late in evolution? The passage does not help you with this part of the answer.
Common sense will. Organisms are believed to have gained the ability to respire (use Oz) some 2 billion years ago
(the numbers are not important here). Multicellular organisms are believed to have appeared some 700 million years
ago. Humans are multicellular organisms; yeasts are not. Long ago (early in evolution), there was a divergence in
our ancestral relationship with yeasts. At that divergence point, the isomerase enzyme was passed to each organism.
The enzyme must have been quite catalytically efficient at that time; otherwise, both lineages might have gone the
way of extinction. Since each lineage did not become extinct, the same enzyme (essentially) is present today. If the
maximal efficiency of the enzyme came about late in evolution, we would expect that one lineage might have a
nearly perfect enzyme, while the other lineage would not. This is because of different environmental forces acting
on the two species.
Even though both choice C and choice D are incorrect (primarily because of the first part of the statement), let's
briefly look at the last part of choice D. The isomerase enzyme does have the ability to change through amino acid
substitution (most likely through point mutations or single base changes). If a mutation is to be passed on to future
generations, it must allow the organism to survive and reproduce. Organisms do have the ability to change. The
correct choice is A.

66. B is correct. ATP is the major carrier of chemical energy in all living cells. Many of the reactions in which ATP is
an intermediate involve a phosphoryl-group transfer from ATP to another molecule, or from an energy-rich
molecule to adenosine diphosphate (ADP) to form ATP. The simplest example of this type of group transfer reaction
can be taken from Table I in the passage. This table summarizes the phosphoryl-group transfer potentials of
hydrolysis of some important compounds in metabolism. If ATP is hydrolyzed to ADP and Pi (inorganic phosphate),
water acts as the phosphoryl-group acceptor. The standard free energy of hydrolysis (AG") for this reaction is -30.5
kJ/mol. This value lies between that of the hydrolysis of phosphoenolpyruvate (-61.9 kJ/mol) and the hydrolysis of
glycerol-3-phosphate (-9.2 kJ/mo1).

- . ri right @ by The Berkeley Review t75 The Berkeley Review


Specializing in MCAT Preparation
Biology Metabotic Components Section VII Answers

Molecules that have a phosphoryl-group transfer potential greater than that of ATp (e.g., phosphoenolpyruvate)
transfer a phosphoryl group to ADP to form ATP. Molecules that have a phosphoryl-grJup't.unif"r potential
lesi
than that of ATP (e.g., glycerol-3-phosphate) acquire a phosphoryl group from ATpio Torm ADp
and the molecule
being phosphorylated. Which of the compounds listed in the answerihoices can be phosphorylated exergonically
by
ATP? We know from general chemistry that reactions that are exergonic are spontaneoui und proceed with a rellas!
of free energy. In this case, the change in free energy (AG) is negative, because the products of these types of
reactions have less free energy than the reactants. Reactions that are endergonic are not spontaneous, anO
ttrey
require an input of free energy. The AG for these reactions is positive, becaus6 the products huu"
than the reactants. We are therefore looking for compounds that have a Lower AG" value than AT-f. -o." free energy

Phosphoenolpyruvate, acetl phosphate and pyrophosphate all have AG"' values that are higher than ATp, allowing
us to eliminate choices A, C, and D. Glucose- I -phosphate has a AG" value of
-20.9 kJ/moj, which is clearly low&
than the AG" value for the hydrolysis of ATP. The correct choice is B.

67- C is correct. Molecules can move across cell membranes by several processes. Diffusion is the net movement of
molecules from a high concentration to a low concentration. Osmosis ii the net diffusion of water from a region
d
low solute concentration (i.e., high water concentration) to a region of high solute concentration (i.e., lorri water
concentration). Passive transport (i.e., simple diffusion) does not require carrier molecules or an expenditure
energy' but rather is the net movement of molecules down their concentration gradient across a membrane. Acti
transport is a carrier-mediated process that requires the input of cellular energylnd makes possible the transporr
molecules across a membrane against their electrochemical gradient. Onl-y active t.unrport works against
t
concentration gradient and requires an input of energy in the form of ATP. Ciroices n, S, and D are incorrect.
correct choice is C.
68. D is correct. Any compound that inhibits ATP production, such as cyanide in the electron-transport chain, is
c
a poison. ATP is the cell's energy culrency, and without it the cell'i metabolic processes slow down and the
eventually dies. A human being has an energy reserve of about four minuGs of ATp and other phosphe
compounds in every cell. This is one of the reasons brain damage is likely to occur after someone has
been withorl
heartbeat for about four minutes and then is revived. The cell is set up to use ATP, so it cannot suddenly
switch
other phosphate compounds and survive indefinitely. Choices A, B, and C are incorrect. The correct choice is D.

69. A is correctt If the phosphoanhydride bond in any one of the answer choices is hydrolyzed to yield the prod
compound plus Pi, the energy release is in the form of heat. For example, if tli,e phtsphoaniydride bond
phosphoenolpyruvate is hydrolyzed to give pyruvate and Pi, the standard frei-energy .trung" of hydiolysis -
is
kJ/mol. If the hydrolysis of an energy-rich compound releases more free energy t[an the -hydrolysis of A1'p,
thguq! phosphoryl-group transfer these energy-rich compounds can transfer i phosphoryt
ATP. Note that the reaction for the phosphorylation of ADP is the reverse of the ieaction tor
lroup to ADp to
hydrolysis of i-tre
This means that the hydrolysis of an energy-rich compound can be coupled to the synthesis of ATp. We can
see
in the coupled example shown below:
o
ooo. o-U-no o AG-
\"/ | cOO o
' it 6^ +Hro + t-o * no-lJ-oo
.c(7i
/\
H' 'H
I
cHr o
o
Phosphoenolpyruvate Pyruvate Phosphate

oo o ooo
ililo
Adenosine-O-P-O-P-O -
ilililo
Adenosine-O-P-O-P-O-p-O
lt
oo
+
"o -!-oo ----------------
ttt + HnO +7))
I
ooo
oo o
o ooo
Adenosine diphosphate (ADP) Adenosine triphosphate (ATp)

1'
Phosphoenolpyruvate + ADP + Pyruvate + ATp

The compound among those in the answer choices that releases the most energy upon hydrolysis would also
the most heat after the phosphorylation of ADP. This compound is phosphoenolpyruvaie. Choices B, C, an
all lower in energy released upon hydrolysis and are incorrect. The correct choice is A.

Copyright @ by The Berkeley Review 176 The Berkeley


Specializing in MCAT
Biology Metabolic Components Section VII Answers

70. C is correct. Of the metabolic processes presented to us, three generate high-energy phosphate compounds (i.e.,
ATP or GTP), and one does not. We are looking for the one that does not. Three of the metabolic processes involve
the oxidation of fuel molecules: glycolysis, citric acid cycle, and oxidative phosphorylation. All of these pathways
produce either ATP or GTP. These metabolic processes are catabolic, and they release energy in the form of ATP
and heat. Protein synthesis, being anabolic, requires an enormous input of energy (in the form of GTP)-possibly as
much as 90Vo of the chemical energy used by a cell during the course of its biosynthetic reactions. Choices A, B, and
D are incorrect. The correct choice is C.
71. B is correct. The bond indicated by the arrow in the ATP molecule, between the 5' carbon of the ribose ring and the
o-phosphate, is a phosphoester bond. There are two phosphoanhydride bonds. One is located between the a- and B-
phosphates, and the other is located between the p- and y-phosphates. Eliminate choice A.
Phosphoester NH:
ypGbbndl
-o ^o ^o / z)-fl
\,3J\"-,
""_i_,_i_"_i-"{:-,,
L-<"> /- \N-crY'*idi'
"'+ ll I tJl jfl-i{
u
?
T
Phosphoranhvdride !"
"J
The only type of glycosidic linkage found in ATP is an N-glycosidic linkage, located between the I' carbon of the
ribose ring and the N-9 nitrogen of the nitrogenous base. Eliminate choice C. Peptide bonds are formed between an
N-terminus nitrogen and a C-terminus carbon of amino acids. Eliminate choice D. The correct choice is B.
72, B is correct. Pyrophosphate (PPi) is a compound that contains two phosphate residues, and depending upon the pH
of the solution, can exist in several ionic forms. For example, there are times when the molecule might bear two
negative charges, and there are times when it might bear three or four negative charges. The electrostatic repulsion
between the negatively charged oxygen atoms of PPi provide a portion of the driving force tbr its hydrolysis to two
molecules of inorganic phosphate (Pi). Since PPi is rather unstable, statement I is inconect. Based on the values for
the standard free energies of phosphate hydrolysis in Table I in the passage, we see that more energy is released for
the hydrolysis of ATP to AMP and PPi than for the hydrolysis of ATP to ADP and Pi. Statement II is correct. Since
PPi can bear a variety of negative charges in the cell, it is quite soluble. Statement III is theretbre incorrect. The
correct choice is B.

11 B is correct. The pH of the cytoplasm is approximately 7. These lysosomal enzymes would be virtually inactive in
the cytoplasm. This is a protective mechanism, so that a rupture in a lysosome does not lead to the destruction of all
intracellular components. Choice A is incorrect. The acidic or basic nature of the proteins does not matter. They are
all degraded with the same degree of efficiency. Choices C and D are incorrect. The correct choice is B.
'{. C is copect. A region of DNA is considered to be highly conserved if it is very similar in the genetic material of
two or more organisms. That means choice A is incorrect. The phyla have nothing to do with conservation in this
question. Choice B is incorrect. "Highly conserved" means virtually identical, while "completely conserved" would
mean completely identical, so choice D is inconect. The correct choice is C-

C is correct. Ubiquitin attaches most quickly to those enzymes that are at metabolic control points in a biochemical
reaction. Some pathways operate pretty much constitutively (all the time), such as glycolysis, the Krebs cycle, the
electron-transport chain, and oxidative phosphorylation. Conversely, your body closely regulates pathways
involving gluconeogenesis, amino acid catabolism, nucleic acid synthesis, or the synthesis of spermine and
spermidine, two polyamines used in DNA-packaging.

Cytochrome c ig a peripheral membrane protein found on the side of the inner mitochondrial membrane that faces
the intermembrane space. This protein passes electrons from Complex III of the electron-transport chain to Complex
IV. Under normal cellular conditions, this enzyme has a half-life of about 150 hours and, as indicated in Table I in
the passage, is a long-lived enzyme that has a stabilizing N-terminal amino acid residue. It turns out that the N-
terminal amino acid in human cytochrome c is glycine (Gly). Since the ubiquitin system is used to destroy abnormal
proteins and short-lived enzymes, we can eliminate choice A.

:r right @ by The Berkeley Review t77 The Berkeley Review


Specializing in MCAT Preparation
Biology Metabolic Components Section VII Answers

Glyceraldehyde-3-phosphate dehydrogenase is a glycolytic enzyme involved in the conversion


of glyceraldehyde-3-
phosphate to--1,3-bisphosphoglycerate. Its long half-life of 130 hours indicates that the
N-terminal amino acid has a
stabilizing effect. Eliminate choice D.
The remaining two proteins are both short-lived enzymes. Tyrosine aminotransferase has a half-life
of l2O minutes
and is involved in the catabolism of the amino acids phenylalanine and tyrosine to acetoacetate
and fumarate-
Ornithine decarboxylase has a half-life of about 12 minutes and is involved in the synthesis
of spermine ard
spermidine, two polyamines used in the packaging of nucleic acid. Even though uoin of these pioteins
havc
relatively short half-lives, and they each have a destabilizing N-terminal amino acid,"we want to select
the one that ir
most likely to be modifiea by ubiquitin. The protein we want is the one with the shorrest half-life, ornithine
9u19_ttv
decarboxylase. A short half-tife usually means that a molecule is being turned over quickly, and in
the case of
rapidly dividing cells, ornithine carboxylase is required in large amounts. With such a short half-life,
UCDEN ir
probably degrading ornithine decarboxylase just as fast as new proteins are being synthesized
to replace it. ThG
correct choice is C.
76. B is correct. Look at Table I in the passage. The stablest enzyme has the longest half-life. Lysine and
leucine
have half-lives of about 3 minutes. Eliminate choices A and b. Glutamic acii has a half-life
of abour 30 min
Eliminate choice C. Glycine has a half-life of more than 20 hours. The correct choice is B.
77. A is correct. During a fast, amino acids are still needed for the repair and synthesis of proteins. Since
the person i
not eating, these amino acids must come from thebody's storedprotein, ihe muscles. Following
a meal, diet4
amino acids are readily available, and lysosomal degradation of proteins should be low. Choice
B is incorre
During exercise, some proteins may be degraded, but not ur rnuny as are degraded during a fast.
Choice C
incorrect. During pregnancy, women are usually well-fed and not fasiing. ChoicJD is incorrec"t.
fhe correct
is A.
78. C is correct. The conversion of ATP to AMP and PPi (pyrophosphate) involves the hydrolysis of
phosphoanhydride bond between the B- and y-phosphates of ATP. Th; pyrophosphate
thus formed is rap
hydrolyzed to two molecules of P1 (inorganic phosphite). In this reaction t"qu"n"", i*o prrorpr1oanhydride
have been hydrolyzed. choices A, B, and D are incorrect. The correct choice is c.
79. A is correct. The action of lysosomal enzymes requires a pH of approximately 5. Chloroquine, in the unch
form, diffuses across the single membrane of the lysbsorne and accumulates inside. Due to
the acidic medium i
the-lysosome, chloroquine becomes protonated and begins to accumulate in the charged form.
This increases the
within the lysosome, leading to inactivation of the enrym"s that require a tow
iH
decrease in protein degradation. Choice B therefore can be eliminated. Since there is no
optimum and a subseqr
indication in the pa!
that chloroquine modifies the active site of a protease, we must assume that it does not, so choices
C and D can
be eliminated. The correct choice is A.

80. D is, correct. Melting is the change of a solid to a liquid, so choice A is incorrect.
Vaporization is the change
liquid to 3 vapor phase, and that is not what is involved in freeze-drying. Choice B is therefore incu
Condensation is the change of a vapor to a liquid or a solid. Choice C is incotect. Sublimation is the
change
solid to a vapor, which is an accurate description ofthe freeze-drying process. The correct choice is D.
81. D is correct. In the third-paragraph of the passage, we learn that vitamins are organic compounds. Therefore"
can be combusted to yield COz and H2o, among other gases. Minerals are just in6rganic ions and atoms
and q
remain in the ash portion. The correct choice is D.
82. A is correct. In the second paragraph of the passage, we find that magnesium is not a trace mineral; il
macromineral. Zinc, manganese, and iodine are trace minerals. The correct choice is A.
83. A is.correct. According to the last paragraph in the passage, a protein is about 16To niftogen The correct
obtained by dividing 5 grams nitrogen by 167o n_itrogen per sample of protein, which givJs 31 grams
of p
answer for choice B is obtained by multiplying 5 grams by 16To. This gives 0.8 grams"of protein,
which is i
Choices c anu n are the same answers mutiipfi"a Uy ro*" power of to] rne coriect choice is
A.
84. A is correct. The calorie content of food is another way of saying its energy content. Only proteins, carboh
and lipids provide energy. Statements I and III are correct, which means thit choice A is also correct.
Vita
not provide energy themselves, although they are involved in metabolic reactions. This means that
incorrect, which allows us to eliminate choices B, c, and D. The correct choice is A.

Copyright @ by The Berkeley Review t7a The Berkeley


Specializing in MCAT prel
Biology Metabolic Components Section VII Answers

85. C is correct. Choices A and B make you think about solubility. The blood is an aqueous medium. Water-soluble
vitamins could travel freely in it, while fat-soluble vitamins (like fats) would need a transport protein. Excess water-
soluble vitamins would be excreted into the urine by the kidney. Vitamin K is important for blood clotting, so an
antagonist would decrease the ability to clot, leading to longer clotting times. Choices A, C, and D are thus all true.
The false (and best) answer is choice C. Ascorbic acid (vitamin C) is water-soluble. The correct choice is C.

86. B is correct. In the first paragraph of the passage, we learned that a triglyceride contains 9 kcal/gram. 30% of 1800
kcal = 0.30 x 1800 = 540 kcal. 540 kcall9 kcal per gr&rn = 60 grams triglyceride. The answers in choices A, C, and
D are the results of various incorrect manipulations of the data. The correct choice is B.
87, D is correct. We are looking for the answer that does not produce water. The last step of the electron-transport chain
uses electrons to reduce oxygen, producing water. Choice A is true. Synthesis of protein or glycogen involves
condensation reactions and the production of water. Choices B and C are true. The breakdown of a triglyceride
requires the use (not the production) of water in a hydrolysis reaction. Choice D is the false answer, because it does
not produce water. The correct choice is D.

88. C is correct. This question cannot be answered based on information in the passage and requires previous
knowledge. The release of fatty acids from glycerol is regulated by the actions of insulin, epinephrine, and glucagon
through a reaction catalyzed by an enzyme called hormone-sensitive triacylglycerol lipase. Adipocytes hydrolyze
triglycerides (i.e., triacylglycerols) to free fatty acids and glycerol, as shown in the reaction below;
o
il
O H,C_O_
"l C_ R H,C- OH
'l o
lt .3 H,o il
R-c-o-c-H o Ho-c-H + 3 HO- C-R
I ll -------:---------
Linase I
H?C-O- '
C- R H2C- OH

Triglyceride Glycerol Fatty acid

One of the intermediates in the synthesis of triglycerides is the molecule glycerol-3-phosphate. If the concentration
of this molecule is low in the adipocyte, the free fatty acids produced in the reaction shown above are released into
the bloodstream. They are not reesterified to triglycerides. The ratio of glycerol to fatty acids would therefore be l:3.
The correct choice is C.
!p. B is correct. The isotopically labeled molecules Iisted in the question are not radioactive. Therefore, scintillation
counting or using a Geiger counter would not work. Both l3C and 2H ate stable isotopes of their parent atoms.
Choices A and C are incorrect. The only difference between l2C and l3C is one neutron in the nucleus. The same is
true of hydrogen and deuterium (2H). The molecules can be separated by mass only, using a mass spectrometer.
Choice B is correct. A UV spectroscope would not help the situation either, since mass is the only critical piece of
data to gather. Choice D is incorrect. The correct choice is B.

',l'l], D is correct. The pH is adjusted to 7.0 so that glycerol is not charged. Therefore, glycerol would not interact with
either the cation exchange resin (where the beads are negatively charged) or the anion exchange resin (where the
beads are positively charged). Glycerol would pass through the apparatus and end up in the water in the collection
tube. Choice A, B, and C are incorrect. The comect choice is D.

A is correct. Since this cycling is well-regulated and operates almost 24 hours per day, it is doubtful that the
normally cycling quantities of fatty acid damage either the liver or the adipose tissue. Choice C and D are incorrect.
The cycle does, however, require energy to operate. It is a futile cycle, one that operates in a circle but uses energy.
Choice B is incorrect. The correct choice is A.

D is correct. The turnover of fatty acids and glycerol means they are entering the bloodstream and exiting the
bloodstream. Turnover is a measure of how rapidly this entering and exiting occurs. In the pre-niacin state, you
see that lipolysis of triglycerides in the adipose tissue must be occurring, since there is a non-zero value ior both"an
fatty acid and glycerol turnover. Choice A is incorrect. You can also see that some fatty acids are being reesterified,
even in the pre-niacin state. If there were no reesterification, then the fatty acid turnover value would be three times
the glycerol turnover value. Since this is not the case (1.3 x 3 = 3.9), then there must be some reesterification in the
pre-niacin state. Choice C is incorrect. Niacin treatment did not increase lipolysis, since the glycerol value did not
change in the pre and post states. Only the value for the fatty acid turnover decreased, so the change was in the
reesterification of fatty acids. Choice B is incorrect. The correct choice is D.

I ': " nght @ by The Berkeley Review 179 The Berkeley Review
Specializing in MCAT Preparation
Biology Metabolic Components Section VII Answers

93. C is correct. These free fatty acids are crucial for use as energy during rest and during exercise. The hean
preferentially uses free fatty acids as fuel, as does the skeletal muscle at rest. This drug probably would be called a
poison, since a large dose of it would mean death. Anyway, the body could not switch to metabolizing ketone
bodies, since they are formed from fatty acids. Choice A is incorrect. The brain and nervous tissue rely primarily on
glucose as fuel, so choice B is also incorrect. Cholesterol does not contain usable energy for human beings, so
choice D is incorrect. The correct choice is C.

94. A is correct. Hormone-sensitive lipase hydrolyzes triglycerides into glycerol and fatty acids. These are both released
from the cell. Blood levels of both would increase following the ingestion of caffeine. Choices B, C, and D are
incorrect. The correct choice is A.

95. D is correct. Equilibrium is a stable condition, a state where no further net change is occurring. In a living syster4
this condition means vital reactions are no longer netting vital products. Without the expenditure of energy and rhe
net production ofvital products, the organism will decay and approach death. The correct choice is D.

96. D is correct. The question describes the relationship between time and distance traveled when molecules ac
diffusing through a medium: If a molecule requires I second to travel I micron, then it requires 9 seconds to travel
microns, and 16 seconds to travel 4 microns. From this information, we can see that as the distance increases.
efficiency of diffusion decreases. Therefore, diffusion is inefficient over long distances, but efficient over
distances. The correct choice is D.

97. C is correct. We are told in the passage that it is the frequency of collisions between substrate and enzyme thar
the rate-determining step in diffusion-limited reactions. It should follow that if we can bring the substrate and
enzyme closer together, we can increase the rate of reactions. A multienzyme complex is exactly what its
implies, a group of enzymes involved in a reaction pathway. By bringing the enzymes close together, we de
the distance one product has to travel to the next enzyme to continue the reaction chain. By decreasing the di
we increase the frequency ofcollisions and increase the rate ofreaction. The correct choice is C.

98. A is correct. The question informs us of a membrane-bound compartment that occupies 20Vo of the total
volume. If we put all of the substrate into this compartment, its concentration could be as much as 5 times
than it is in the cytosol. How do we arrive at this figure? Let us assume that the concentration in the cytosol is I
of solute to I unit of volume. In the compartment, the volume is reduced to 0.2. Now, the concentration is I unir
solute to 0.2 units of volume, which gives us a concentration that is 5 times greater than in the cytosol. The
choice is A.
99. C is correct. This answer can be arrived at simply by looking at the time it takes for these molecules to hir
target. In Trial C, it takes 9 seconds for the molecule to find its target. If we convert 9 seconds into minutes. sr
0. l5 minutes, so the frequency of collisions in Trial C is 100 times greater than in Trial A. The correct choice ig

100. D is correct. This answer can be arrived at by thinking about movement in different dimensions. Recall
membrane3 accelerate diffusion-limited reactions by limiting movement to only two dimensions. The same ki
thinking can be applied to DNA-binding proteins, as they diffuse through the nucleoplasm in search of their bi
site(s). It is not efficient for the binding protein to jump onto and off the DNA molecule at random places to
binding site, because it might miss something. It is more efficient for the binding protein to limit its move
going up and down the DNA molecule until it finds its appropriate binding site, indicated by a strong bi
affinity. The correct choice is D.

Copyright @ by The Berkeley Review rBo The Berkeley


Specializing in MCAT
A. Metabolic Pathways

,t,, ','.' :'i; ' ,,biSaiihuride Metabolism ::,,,,i,r:,,,ri.,,: ,::,

1.,
1.,.,'',.
5;,',.,,fhe.Kgehs'Cyile,., rl :

Metabolic ,1
t,,.,,,'. ,,..,

Pathways r:,.,.,,,,,,.',t'''r0*idativeP'hosbhorylati,
..',',.', 6.,,,, Pen@ pho$ghate,Faih#a y.,,,,,,,,,,,.
l,:,t.,,t.,:7'"rt,,.Glwbneogenesis ''lrr.'.''*rl
....''.:l.::$;.:::.:l.sa*'5r'Aci,do.xidation:;.l''',,;;:'...
g. The Urea Cycle

. eg',Pa$sgget,,,*-'O**.*t"

.,,,,, 1,: 11:1;l|:ffi1*l-El


:)t: ':4tt ,t:t.::
.:..1.
uzo_-ll7
ti.
.Fumat'ate,".r.
raoHl*$41',,,,,,,;'
:'
FAD]I -,/.11i:: ir".r.

r
.& f 'rr4rNtiDt, +rebA
",'r
-r--4\
\ '}NADH+H+
9?ttYi,.'. :tr;"6 :., ':,' ,{9?,.,t'.:' .,, : ,'-. ., r' ,.
',

$$a
n
R'. arvr;'I;h'i w*
Specialiiin$ in' MC$'Preparation
Metabolic Pathways
Top lO Section Goals

Understand the behind the


Glycolysis is one of the pathways central to metabolisnr. You should have a basic understanding
;f'h;i"= it operates, ivhut ii generates, and how it is tied in with the Krebs cycle'

Be famillar with the common of that can be metabolized.


Suears other than glucose can be used for metabolism. Sucrose, lactose, fructose, & galactose are
Uu? a few sugais that are commonly metabolized by a wide variety of organisms.

Be able to link the concePts of glycolysis with tlre l{rgbs cycte.


Once the commitment to aerobic respiration is made, pyruvate is converted to acetyl-CoA which
then enters the Krebs cycle. ThJs link unites glycolysis and the Krebs cycle and is important.

Understand the im of the NADII ET NADII2.


the Krebs rycle. |AD_H2 is generated.iljl" Krebs cycle.
NADH is generated in both glycolysis and
Understan? how these reduiing c6mponents are important for the generation of ATP.

Know hory the electron transport chain operates.


"3 It is important to have an understanding of how electrons flow from NADH and FADH2 down the
electron transport chain to oxygen, the rfltimate electron acceptor.

'v Understand the chemiosmotic hypothgsis and the concept of a proton gradient.
Understand how a proton gradient is generated, and how it leads to the eventual synthesis of ATP
in oxidative phospfiorylad6n. Be famiiiar with inhibitors and how they work.

ov Be familiar with the general concepts of the pentose phosphate pathway.


Do not memorize the structures of the pentose phosphate pathway. Just understand why this
^also found
pathway is important. Many of its component} are in the Calvin iycle.

Understand the relationship


^%
- V
t( Be aware that gluconeogenesis is not the complete reverse oi glycolysis. There are irreoersible step*
which must be"bypassed. Know the importance of gluconeogenesis in metabolism.

Know the concepts involved in the p-oxidation pathway.


"v Fatty acid oxidation is important in metabolism. There are many classic biochemical reactions:n
thiJ pathway that ilhistrate the importance of undersianding organic chemistir--

Be familiar with the urea cycle and its relationship to the lite. LE44ey:-
reactions. Understand how nitlogenous w-astes are removed from the body and in what for:n',
Biology Metabolic pathways
Glycolysis

cl#,c I l
Phase I
The ideas that we have recentry
been considering can be put to immediate use by
:xamining the reactions in glycolysis.
The first st?p - giirryrrr"is shown
1 in Figure 8-1. The most common in Step
and wide-spread enzyme that makes
qlucose when it enters the use of
ceil is hexokinase.
:he transfer of a termina.l phosphar"-g;o"pA kinase is an enzyme that invorves
;ompound. In this case.it is b-grucos".
of an ATp ,.,it to some other
D-glucose it does so atthe C-o p"osition.
ivtr"" ATp transfe* ii, phorphate onto
we will end up with a compound calred
clucose-6-phosphate. Note that
we have usea r arp. rr,i, i, ur., inaestment
and is an important contror point step
in giy"otysir. why did we bother to attach
:hosphate onto glucose in the first a
pra?& Gr,r.oru dols not il";;"y charges
:an pass back and forth across the and
cel's membrane reratively eas'y. However,
rnce it is phosphorylated,it picks_up
some negative charges and can no
sass across the cel|s membrine. longer
Grucose, in.th"e.form ,r gi".;-6_phosphate,
inside the cell. The AGo, for this ,uu.tior, in is
=apped Step 1 is _4.0 kcal/mol.

Ho
\ /t- HO
\ az-
C
AGo'= -4.0 C AGo'= +0.4
, I
CH"-OH
H_C_ OH
H.C-
I
OH Phosphogluco t'
I Hexokinase I isomerase c=o
HO - C_H
HO- C.H
I

H.C-
I
OH I
HO - C.H
I ADP H.C- OH Step 2 I
H.C_ OH
H.C_ OH I
I
I
cH'-oH
Step I H-C_ OH
I
H-C_ OH O
IrrO
cH2- o- P_ O cH2- o_ P_ O
Glucose -o
I
I

o n
I
Glucose-6-phosphate Fructose-6-phosphate

Figure B- I

step 2 in glycorysis irwolves an equilibrium


between two isomers, glucose_6-
:hosphate and fructose-6-phosphatd. rr,"
reuction takes prace at the C-l and C_2
:arbons of glucose-6-phosphaie. and
proceeds through an enediolintermediate.
lhe compound that emeiges from ihis reaction
is- Fructose-6-phosphate as
;hown in Figure g-1. Thelnryme involved-
here is called phosphoglucose
isomerase. Fructose-6-phosphate is
the phosphorylated versio'n-of D-fructose
reto sugar). The AGo'for (a
thii reaction is +0.+ kcal/mol.
Step 3 involves a second inrsestment
of an ATp molecure and is yet another control
:iif:t:flf:Y.'_'::l* general.class of enzyme involved here is
called pho sphof a transferase
ructokinar:. enzyme cataryzes the conversi;Hll":r';r"-T
Ti:
phosphate to Fructose-$6-diphosplate.
ir;allmol
The lGo' for this reaction is -3.4
as shown in Figure g_i.

opyright @ by The Berkeley Review


185 The Berkeley Review
Specializing in MCAT freparation
Biology Metabolic Pathways Glycolysis

o
ilo
cH2-oH AGo'= -3.4 cH2 - o- P- o
t"t'l
C= O Phosphofructo C= O OO
I kinase I
HO- C-H HO- C.H

tl
"_l-
o" a.'ip io* o"
"_l-
H-c-oH I . Step 3 H-c- oH I
cH2- o- P- o cH2- o- P- o
oo oo
Fructose-6-phosphate Fructose- 1,6-diphosphate

Figure 8-2

In Step 4 fructose-1,6-diphosphate undergoes an interesting reaction in which


bond between the C-3 and C-4 carbons is broken. This reaction involves a
aldol condensation and the products are dihydroxyacetonephosphate (
and glyceraldehyde-3-phosphate. DHAP and glyceraldehyde-3-phosphate
the two simplest and smallest carbohydrates. Both are trioses. If DHAP were
react with glyceraldehyde-3-phosphate, forming fructose-1,6-diphosphate,
reaction would be called an aldol condensation. The enzyme involved here
aldolase. It is in the class of enzymes referred to as lyases. The AGo'for
reaction is +5.7 kcal/mol as shown in Figure 8-3.

^, C=O OO
AG" = +5.7 |

Aldolase cH2-oH

Iilo
cH.-
'l
o-P -o
oo
Fructose- 1,6-diphosphate Glyceraldehyde-3-phosphate

Figure B-3

In Step 5 DHAP can be converted into glyceraldehyde-3-phosphate.


compounds are isomers and the reaction is catalyzed by triose p
isomerase. All of the other significant reactions in glycolysis proceed
glyceraldehyde-3-phosphate. Even though about 96% ol the of the
phosphate molecule at equilibrium is DHAP, once glyceraldehyde-3-
is removed, DHAP is isomerized to glyceraldehyde-3-phosphate (recall
ChAtelier's principle). In other words, from one molecule of fructose-
diphosphate you can get two molecules of glyceraldehyde-3-phosphate.
AGo' for this reaction is +1.8 kcal/mol as shown in Figure 8-4.

Copyright @ by The Berkeley Review ta4 The Berkeley


Specializing in MCAT
Biotogy Metabolic Pathways Glycolysis

o AGo = +1.8
rr O Triose Phosphate c
cH2- O- P- O
rl isomerase I

C=O
r O^s,,, Step 5
H-C-
IrOOH O

cH2-oH cH2-o - P-O


I

oo
DHAP
Glyceraldehyde-3-phosphare

fqgure 8.4

--::his point we have completed phase I of glycolysis. we have


converted the 6
-::rons the original glucose molecule into 2 three carbon phosphorylated
-of
::ermediates. Because these two phosphorylated intermediates are intercon-
'-:.]ble, we have a "pool" of triose phospirates. you should remember, we
i::;ally have millions and millions of glucoie molecules forming twice as many
r:i 'ions and millions.of triose phosphate molecules. up to this point the o.r".uil
r-iation states of the carbons has remained constant. There have been
no
'-:ation or reduction reactions.

Fhase II
- rr: now consider Phase II of glycolysis. In step 6 we will see
a change in the
'-'alion state of the C-1 carbon of glyceraldenyae-e-pnosphate. The oxidation
r;;;j of the C-1 carbon on glyceraldehyde-3-phosphate is +t. The
enzyme
s ''r ;eraldehyde-3-dehydrogenase will convert giyc"tutd"nyde-3-phosphate'into
.r-Diphosphoglycerate (abbreviated as 1,3-Dpt). Note that the C-l carbon
of
:-DPG has an oxidation state of +3. We have a change in oxidation states form
*i :: +3. This is a tzao electron oxidation. when you look at 1,3-DpG, note the
: t:; ;r.thyd-ride linkage and the monophosphoestei linkage. How did this mixed
,;1j6lg linkage get there? we needed a phosphat6 in order to make
':-r this
.-g l1 and that phosphate comes from inorganic phosphate (p) and not from
*rlF, llhis inorganic phosphate and glycerataenyae-e-phosphate
have to be put
ri';*::.er. There must also be an oxidizing agent th-atwill-oxid-ize the
C-l carbon of
I :-:aldehyde-3-phosphate. This oxidizing agent is NADo. The AGo'value for
r' .* :.acfion is +1.5 kcal/mol (see
Figure g-5).

Mixed Anhydride

AGo'= +1.5 "\\


Linkase
?
O
o
ii.' G ly c e r dlde hy de - 3 - p ho sp
dehydrogenase
hat e o.\'^, o- P-O
*31 ll
o
H.C-OH O T
|ilOOH
H-C- O
lr, - o- !- ot NAD* NIDH
I +Pi cH2-o-P-O
oo +H+ I

oo
Step 6
"
;eraldehyde-3-phosphate 1,3-Diphosphoglycerate
-:

&5

It. -',ed anhydride linkage in 1,3-DPG is relatively unstable, thus giving that
l: : ::d a high phosphoryl transfer potential. If we were to carry out the

9 by The Berkeley Review la5 The Berkeley Review


Specializing in MCAT preparation
Biology Metabolic Pathways Glycolysis

hydrolysis of 1,3-DPG, the A Go' would be about -11.8 kcal/mol. However,


instead of using water in this reaction we will use ADP. The enzyme involved
here, phosphoglycerate kinase, will not operate unless there is ADP present. The
product of this reaction, as shown in Step 7 (Figure 8-6), is 3-phosphoglycerab
and ATP. We now have a return on our investment in the form of ATP. In fact
we have a return of 2 ATP's (recaIl we split our 6-carbon glucose molecule inb
two 3-carbon triose phosphate molecules).

oo AGo = -4.5
lO A
o.
\'-'
o-P-o
ll
Phosphoglycerate o.
t'c' o"
Yo kinase
I

H-C- OH
til
O
, ATP
riloOH O
H-C-

I
cH2-o-P-O
oo Step 7 oo
1,3-Diphosphoglycerate 3-Phosphoglycerate

Figure 8-6

In Step 8,3-phosphoglycerate is converted to 2-phosphoglycerate by the


phosphoglyceromutase (see Figure 8-7). The phosphate group at the
position of 3-phosphoglycerate is not the same phosphate at the C-2 positi
2-phosphoglycerate. This is because the enzyme itself is aphosphorylated
During an intermediate state the phosphorous from the enzyme gets
the C-2 position of 2-phosphoglycerate while the phosphorous from the
position of 3-phosphoglycerate gets transferred to the enzyme. An exchangp
taken place. The AGo'for this reaction is +1.1 kcal/mol. [Note: Recall that
we discussed hemoglobin, we encountered 2,3-bisphosphoglycerate (2,7
This molecule binds in the middle of the hemoglobin molecule and stabili
deoxy state of that molecule so more oxygen is released to the tissue. It is d
point in the cell that 2,3-BPG can be made. The enzyme that would cataly
reaction would be 2,3-BPG phosphatase (phosphatases catalyze the hydrol;
phosphate esters).1

Figure B-7

In Step 9 we have the loss of water from 2-phosphoglycerate. This inr


hydroxyl function at the C-3 carbon and the hydrogen at the C-2
Figure 8-7). The enzyme enolase catalyzes the reaction of 2-phosphogll
phosphoenolpyruvate (PEP). PEP has an unstable high energy phosphate
Why? The phosphate moiety attached to the C-2 carbon is in a
arrangement with the rest of the molecule because of the enol

Copyright @ by The Berkeley Review la6 The Berkeley


Specializing in MCAT
Biology Metabolic Pathways Glycolysis

:lectrostatic repulsion is quite pronounced in this situation. The AGo' for this
::action is +0.4 kcal/mol.
': v'e hydrolyzePEP, we would get the enol of pyruvate plus pi. The enor form
, ould be in equilibrium with the keto form. These are referred to as tautomers.
:-iorvever, the equilibrium between the enol and the keto forms is very much in
:..r'or of the keto form. This is shown in Figure 8-8.
o. oo
::ep 10 is the last reaction in glycolysis (see Figure 8-9). The enzyme pyruvate t\-zo
cc
o
\ z
o

r,:inase will catalyze the transfer of a phosphoryl group from PEp to ADp and !H r'=
C -r
il+i -r>
L:
r---'e us pyruvate and ATP. Pyruvic acid may also be called 2-keto-propionic acid CH: CH.
: a-keto-propionic acid. This phosphorylation is nonoxidatioe. Recall that when
rDP is converted to ATP the AGo'is +7.3 kcal/mol. The high energy bond in enol-Pyruvate keto-Pyruvate
IEP has a AGo'of about -14.8 kcal/mol. coupling these two reactions will give Figure B-B
,-- cverall AGo' of -7.5kcal/rnol for Step 10. Note that since we started with a 6- Enol-keto equilibrium with
,::ron glucose molecule we now have two 3-carbon pyruvate molecules. In this pyruvate.
:: we have actually made 2 ATP's (net gain).

AGo'= -7.5
oo Pyruvate
o-l\ z-o-A
-z-\-
t\/
CO ll C
I O
Kinase
c-o*P-o
ill
I

C=O
CH, O^ ADP ATP
I

CH:
Step 10
?:osphoenolpyruvate Pyruvate

''gure 8.9
-',' that we have finished glycolysis, we need to consider the overall
,..,rtption and generation of ATP. Recall that in Step 1 and in Step 3 we invested
-TP each. Thus, our total investment is 2 ATP's. in Step 6 we got 2 ATP's back
' :. our investment (remember, we have two 3-carbon compounds at this
:-:r. And it is in Step 10 that we have made a net profit of 2 ATP's from our
...,."1 investment.

'- . return to Step'6 in glycolysis (Figure 8-5). In order for this reaction to take
r:. \ve used two molecules of NADo and we produced two molecules of
i.DH + H@. Remember, we actually have two molecules of glyceraldehyde-3-
, r:hate at this point. In order to continue with glycolysis we must have some
r :o restore the reduced NAD's back to their oxidized form. How do we
'-:.erate NAD@?

' :r: dr€ several ways to do this andpyruaate is involved in all of them. If we
, :'.'ruvate itself to regenerate the NADo, then pyruvate will get reduced.Look
-- = C-2 carbon of pyruvate. It is in the keto form and the oxidation state is +2.
= lid a 2 eiectron transfer, we would end up with a carbon that has a zero
-:.:ion state. The bompound that we can pioduce from this reaction ls L-
..:aie as shown in Figure 8-10. The enzyme involved here is lactate
. : -:' lrogenase. This is one of the possible fates of pyruvate. in this way we can
::ze NAD and pay off our debt in Step 6.

:ht O by The Berkeley Review ta7 The Berkeley Review


Specializing in MCAT Preparation
Biology Metabolic Pathways Glycolysis I
il

oaa
o
Inctate
_o
().()
rt*,:
&
.ro
C dehydrogenase C
I l0
+2C=O H-C-

^ZN
OH
I I

CH: CH:
Pyruvate H* +
Lactate

Figure 8-lO

The reaction in Figure 8-10 occurs in the cells of an organism when oxr-gmL
becomes a limiting factor. For example, during vigorous exercise actiae sl<el-,:,il
tyuscle produces lactate. Lactate turns out to be a dead end in metabolism, and as
we will later discover, is transported by the blood to the liver where it ls
converted back to pyruvate.

Pyruvate will then be converted into glucose by a process calieJ


gluconeogenesis. Lactobacillus casei, a member of a family of bacteria that are u-.e,j
in the preparation of yogurt, excrete lactic acid into the surrounding medium a_o a
waste product. This is what gives yogurt its slightly acidic taste. The reacticrru
shown in Figure 8-10 therefore allows glycolysis to continue under anaerobis
conditions. Note that lactate is still at the same oxidation state as glucose
(Lactate: C3H6O3 x2 = C6H72O5,. Glucose: C5H12O5).

j_
o.o o
C
I

c=o
I
_Z\- i=o-n-i',
Pyruvate
decirborylase I
Alcohol
dehydrogenase oH A
d
i
CH: H+ iO, CH: NADH
NraO* CH:
Pyruvate Acetaldehyde
+ H
Ethanol

Figure B- II

This is not the only way to regenerate NADo. Yeasts use a slight modificahc,n
Instead of doing a direct reduction of pyruvic acid with reduced NADH, yeL.is
first decarboxylate the pyruvic acid to acetaldehyde as shown in Figure 8-11. The
enzyme that catalyzes this reaction is pyruvate decarboxylase (which contai:m
thiamine pyrophosphate (TPP) as a coenzyme). Pyruvate decarboxylase is a lyase.
-{

Acetaldehyde will next react with the reduced NADH to form ethanol.
enzyme involved in this step is alcohol dehydrogenase. This process,
conversion of the sugar glucose into ethanol, is called alcoholic fermentation, -1-
ethanol, a waste product, is excreted into the surrounding medium.

The Big Picture


Let's consider the overall picture of glycolysis. This is shown in Figure 8-12.
way to become intimately familiar with this pathway is to follow a radioacti
labeled carbon through each series of steps (1-10) that we outlined and see wh
that label ends up on the end products. For example, if the C-1 carbon of glu
were radioactively labeled (designated as *C-1), where would that label

Copyright @ by The Berkeley Review la8 The Berkeley


Specializing in MCAT
ll
Biology Metabolic Pathways Glycolysis
l

iJ it does at all) inpyruaate,Iactate, or even ethanol? We will do it for a *C-1


,abeled glucose molecule as shown in Figure 8-19.

{€ CHo {€ CHo * Cuz_gr-r


rll
H-C- oH H_c- oH
^-
Phosphogluco i= o
I Hexokinase i Isomerase i
: " / nu- L-n HU- L-H
t/\t \ =:
H-C- OH
I

.^,.
ATp
nbp Hc- oH H c- oH
|
H.C- OH H-C- OH O H-C_ OH O
I|il_Iil
cH"-oH cH2_ o_ P_ o cH,_ o_ P_ O

Clucose Clucose-6-phosphare I Fructose-6-phosphate I

;l -:J1:_l_" ""'flx:{::""
^,"1*"n,-o-!-o-
ATP: I

TriosePhosphate,
i''- '- |
i=o
o

Isomerase " l'2 "


//7 1",-o" I

9 I '"J"" c=O O
"1 GlYcer- tl * Aldolase I

'rv- !--
\.'" I " 1-.pi"'ii"t, ' '..'r-
o. o- P- o auinYde- \r H . o
o"
l0dehydrogenasell "_l-
H-C- OH O - /=-== H-C- OH O H-C- OH O
"r
ffc"r-o-f-o
| /
{
\ \ *a",-o-p-o-
ekt il I il
cur-o-e-o-
-
l -
o NADH NAD+
"""" ! |

^
Pi o o-
t,3-Diphosphoglycerate + H* +
Glyceraldehyde- Fructose-1,6-diphosphate
3-phosphate
Phospho- lf
ll-ooo
glycerate ll\ #
kinase '{l \ c",- cH)- oH NAD+
lrp NADH
Ethanol \ f
o.\ ,o oo
-a4", - irooo, \=-,* "-
.C dehydrogenase \
I I H_ C= O
H.C- OH O H-C- OH
NADH * ln,
ff"tct,
-"t il NAD.
' cH-- o- P- o *,H'
'l Acetaldehyde
Lactate
3-Phospho$ycerate
o
Iactate
dehydrogenase
o,!l#::j;",, I. ,o,

ll nhospnogryc"ro-
Jl mutuse \ ott
I

oa\ ,o rr
o:\ ,o Pyruvdte zo
CO CO Kinase
C
til Enolase lil I
H-C- O- P- O c-o-P-o ,/\ c=o
l*" I ill
*cHz ADP ATP *l,
HO-H2C O o CH,

2-Phosphoglycerate Phosphoenolpyruvate Pyruvate

Figure a- 12 .i.

lhere is usually one point of confusion in this process and that comes at Step 4
::.d Step 5. When fructose-1,6-diphosphate splits to DHAP and glyceraldehyde-
:-;hosphate, the label is initially on the *C-1 carbon of DHAP (which was the
;beled nC-1 carbon of fructose-1,6-diphosphate). However, since DHAP is in
:::"ilibrium with glyceraldehyde-3-phosphate, that labeled carbon at the *C-1

- rpvright @ by The Berkeley Review la9 The Berkeley Review


Specializing in MCAT Preparation
Biology Metabolic Pathways Glycolyir

position of DHAP is now the labeled carbon at the *C-3 position of glycer-
aldehyde-3-phosphate. \44ren you split fructose-1,6-diphosphate the numiering
system of the carbon atoms in the subsequent productJ are now based o1 u thrd
carbon molecule. In other words, the phosphate attached to the C-l position off
DHAP is the same phosphate attached to the C-3 position after DHAP hns
isomerized to glyceraldehyde-3-phosphate. Therefore, the labeled *C-l carbon cf
DHAP is the same labeled *C-3 carbon of glyceraldehyde-3-phosphate.

Regulation
If you look at the glycolytic pathway presented in Figure g-12, you will notice
three places where the reactions are essentially irreveisible. Those reactions a-re
catalyzed by_,the enzymes hexokinase (AGo' - -4.0 kcal/mole), phosphofructo-
kinase (AGo'= -3.4kcal/mole), and pyruvatekinase (aco' = -z.s ical/moleil
respectively. It turns out that these reactions all involve control points in ttrc
regulation of the glycolytic pathway. In particular, regulation ui th" level od
phosphofructokinase is the most important. (we will r"e why in future
discussions.)

lvhen the cell has plenty of glucose it will make a lot of ATp. Times are good fm
the cell and so it doesn't want to waste its source of energy (glucose) uy irtltizrng
it to make more ATP than it really needs. one thing *," would titu to ao
with its supply of glucose is store it. As we will later see,"utt
the storage form qd
glucose is glycogen.

However, before the cell can begin to store this glucose it must somehow slow
down the glycolytic pathway. High levels of ATp tend to allosterically inhiw
phosphofructokinase. Not_only do high levels of ATp inhibit phospdofructo-
kinase, but high levels of H@ inhibit it as well. one place we could get in increa_ce
in hydrogen ions is from the conversion of pyrursate to lactate (see F-igure 8-10). If
you were to produce too many hydrogen ions, then your blood pH would begio
to drop and you would experience acidosis. one other regulatory molecule"of
phosphofructokinase is citrate, an intermediate in the Krebs cycle. if the levels of
citrate are high, it must mean that glycolysis is functioning at some optimal rahe
(because there is plenty of glucose around). once again, why waste the glucose
that has been made available to you. High levels of iitrate also inhibit
phosphofructokinase.

If high concentrations of ATP inhibit phosphofructokinase, then it must mean


that when the concentrations of ATP are low the activity of phosphofructokinase
is enhanced. what indicates when the levels of ATp are low? If ATp is being
utilized, it is most often converted to ADP and even AMp. It turns out that high
levels of AMP stimulale phosphofructokinase.

Copyright @ by The Berkeley Review 19() The Berkeley Review


Specializing in MCAT Preparation
Biology Metabolic Pathways Disaccharide Metabolism

DisHbdhffi dd' MGtd$6ti$m


Disaccharides such as maltose, sucrose and lactose can be hydrolyzed
into their
:onstituent monosaccharide residues which can then enter into"the
glycolytic
rathway. In humans we find that maltose can be hydrolyzed by thJ L"ri^u
maltase into two molecules of B-D-glucopyranose while iucrose
can be hydro-
]i'zed by sucrase into a-D-glucopy.anose ind
B-D-fructofuranose . Lactosecan be
rvdroiyzed by lactase to B-D-glucopyranose and
B-D-galactopyranose. These
:nzymes are located in the epithelial cells that line the smill intestine.
In bacteria
-actose is hydrolyzed by the enzyme B-galactosidase.

]nce maltose is hydrolyzed, the monosaccharide residues are immediately


ready
:o enter into the glycolytic pathway. However, the
B-D-fructofuranose residue
::om sucrose and the B-D-galactopyranose residue from galactose
both need to
:e converted to a form which cun ente. into the glycolytic
iutn*uy.
-he hydrolysis of sucrose into g-D-glucopyranose and
B-D-fructofuranose is
tlow_n in Figure 8-13. The glucopyranose residue will readily enter
into
:lvcolysis.

.-oH
cH2-oH
HO
o HzC o
Sucrase, { oH
H:O
+ K"J
nU tlcH^
-) I-l on'
OHH HOH OHH
a-D-Glucopyranosyi- B-D-fructofuranose a-D-Glucopyranose B-D-Fructofuranose
Sucrose (Enters Glycolysis)

Figure B- t 3

i{cwever, the fructofuranose residue must first be converted into a form


which
--'n enter into the glycolytic pathway. Fructofuranose can either be converted
-- lo
p-D-fructofuranose-6-phosphate by the enzyme hexokinase
or into B-D-
-:uctoJuranose-1-phosphate by the eniyme fructokinase. Both reactions are
:rosphorylation reactions and both require ATp.

CH,.O-PO.2-
- J
| lrtose
C= O Phosphate

l"r-on {"'"'"
oT
HO-H2C

K"J
O OH
10"
-
""W DHAP \ o.,
C
V V/
H CH2-OH Fructokinase
+ ADP
I

H-C- OH
tt
OHH
2-o.Po32-
I

CH2-O-POj2-
- OHH
B-D-Fructofuranose B-D-Fructofuranose-
} \" Glyceraldehyde
3-phosphate
1-phosphate I

H-C_ OH
I

cH2-OH
D-Glyceraldehyde Glycolysis

iigure B-14

:pyright @ by The Berkeley Review l9l The Berkeley Review


Specializing in MCAT preparation
Biology Metabolic Pathways Disaccharide Metabolism

The hydrolysis of lactose into B-D-galactopyranose and a-D-glucopyranose


is
shown in Figure 8-1.5- once again, the grucopyranose residue ivitt reaaity
enter t
into the glycolytic pathway.
{
rm
cHz-oH
G
Lactase,
d
*+
H'O
H
OHH
H

M
d
HOH HOH
h
HOH HOH h
B-D-Galactopyranosyl-cr,-D-glucopyranose B-D-Gatactopyranose g-D-Glucopyranose
(Enters Glycolysis) wn
Lactose

Figure B-15

The galactopyranose residue must now be converted into a form which


can enter
into the glycolytic pathway (this sequence is not shown). Note that galactose
is
an epimer of glucose at t_he C-4 position. It turns out that galactlse can be
converted into glucose-6-phosphate in 4 steps. Recall that glucoJe-6-phosphate
m
an intermediate in glycolysis.

Copyright @ by The Berkeley Review 192 The Berkeley


Specializing in MCAT prepara
I

Biology Metabolic Pathways Krebs Cycle

ltr6$$l'i
lhe citric acid cycle is also called the tricarboxylic acid cycle or the Krebs cycle.
":n eukaryoflc cells the citric acid cycle o"".rrc
inside lhe mitochondria.
Glvcolysis, however, occurs in the cel|i cytosol. Recall that the end product
of
:jvcolysis was pyruaafe. Under anaerobic conditions, in the absen." bf o*yg"r,,
:r'ruvate was used as an oxidizing agent to reoxidize NADH to NADd so
:lvcolysis could continue. For example, in exercising muscle and in certain
:acteria pyruvate can be converted into lactic acid wrile in yeast pyruvate
can
!e converted into carbon dioxide and, ethanoL Under aerobic in the
:resence of oxygen, we will find that all of the carbons of "orrd-iiio.,r,
pyruvate will
=r-entually be converted into carbon dioxide.

The first reaction to consider is the oxidative decarboxylation of pyruvate


to form
acetyl CoA. This is a complicated reaction involving three caialytic cofactors
'Siamine pyrophosphate (Tpp), lipoamide, and fla"vin adenine dinucleotide
FAD)) and two stoichiometric cofactors (CoA and NADo;. this reaction
is
by a group of three enz,ylres (pyruvate dehydrogenase component,
:1:u1{r"9
iihydrolipoyl transacetylase, and dihydiohpoyl dehydrolenase) which are
:ranged into a structure called the pyruvate dehydrogenase iomplex.

TPP adds to the carbonyl function of pyruvate. The addition compound


,lecarboxylates and the remaining portion -oi pyruvate (a hydroxy-ethyl
moiety)
*r transferred to lipoamide where it is oxidiied to form aletyl-iipoamide.
The
:cetyl moiety of acetyl-lipoamide is next transferred to CoA to form acetyl-CoA
:nd dihydrolipoamide (the reduced form of ripoamide). NADo is converted
to
\ADH + Ho as it reoxidizes dihydrolipoamide to lipoamide. The essentials of
-ris reaction can be seen in Figure g-16.

NADH
+H+
oo
ililo
NAD+ |
\/ o
il
cH3.C-C-O + CoA-SH CH3'C- S- CoA + O=C=O
-3 +2 +3 Pyruvate
-3 +3 +4
Dehydrogenase
Pyruvate Acetyl-CoA
Complex
AGo'= -8.0

Figure 8- l6
The AGo'for this reaction is about -8.0 Kcal/mol and is very much in favor
of the
:roducts. Not only is entropy increasing because we are converting one molecule,
:vruvate, into two molecules, acetyl-CoA and carbon dioxide, but the carbon
lioxide iiself is free to leave as a gas. This makes the reverse reaction rather
iifficult.

l\re mentioned that the reaction shown in Figure g-16 is an oxidation pyruvate
is
:n g-keto acid" The methyl carbon has an overall oxidation state of -3, the
:arbonyl carbon +2, and the carboxylic acid carbon +3. The oxidation state of the
:arbon in carbon dioxide is +4. what about the acetyl group of acetyl-CoA? The
:rethyl carbon still has an oxidation state of -3. The cirbonyl carbon, however,
row has an oxidation state of +3. Remember, sulfur is more electronegative than
:arbon. \A/hat is the difference in oxidation states between the reactant, pyruvate,

l Copyright @ by The Berkeley Review 193 The Berkeley Review


t Specializing in MCAT preparation
Biology Metabolic Pathways Krebs Cycle

and the products, carbon dioxide and acetyl-CoA? The oxidation state of the
methyl carbon does not change, so we can ignore it. The combinedoxidation states
of the carbon of the carbonyl function and the carbon of the carboxylic acid
function of pyruvate is +5. The combined oxidation states of carbon dioxide's
carbon and the carbonyl carbon of acetyl-CoA is +7. This is a two
oxidation process because we have increased from +5 to +7 as we have gone from
reactants to products. If there is an oxidation, there must also be a reduction-
NADo undergoes a two electron reduction to form NADH + H@

The two carbon unit that we have synthesized in the form of acetyl-CoA
unique because it has a high energy thioester bond. The problem that we i
faced with is turning the two carbons of acetyl-CoA into torbon dioxide. we w
to use the energy of the thioester bond in acetyl-coA to help turn both of the
carbons into carbon dioxide. The only way we are going to be able to accompli
this is to extend the carbon chain. we can do this by taking this two carbon ai
unit and combining it with a four carbon acceptor molecule called oxaloacetic
(oAA). where did oAA come from? we will discuss this at a later time.

Figure B- l7
The alpha hydrogen of acetyl-CoA witl form a bond with the carbonyl
oAA as shown in Figure 8-17. The reaction proceeds via an aldol con
to form the intermediate citryl-CoA. Citryl-CoA is then hydrolyzed to fi
citrate with the subsequent regeneration of coA. It is the hydrolysis of
thioester bond that allows this reaction to be favorably pulled to compietion.

The enzyme that catalyzes this reaction is citrate synthetase. It is in the clas
enzymes called a lyase. Note that there is no net gain or loss of hydrogens in
reaction. Therefore, it cannot be a dehydrogenation--and it certainly cannot
hydrogenation. There is no net change in oxidation state either. Those molq
who gained electrons also lost electrons, and those who lost electrons also
electrons. Think of it as a reaction in which you are adding to the double
the carbonyl group in OAA. Thus, it is alyase.

Even though citrate is a symmetrical rtolecule, the two -CH2-COOo


not react identically. why? It has to do with the asymmetrlc nature of the
aconitase in relationship to the symmetrical nature of citrate. In other
two -CH2-COOo groups of citrate are not equivalent. Aconitase, which
citrate to cis-aconitate, can distinguish between them. Thus, citrate is a
molecule and is optically inactive.

There is a hydroxyl function at the beta position of citrate. you might


we could oxidize the hydroxyl group to make the beta-keto function so we
decarboxylate that carbon. We cannot do that because the hydroxyl

Copyright @ by The Berkeley Review 194 The Berkeley


Specializing in MCAT Pre
Biology Metabolic Pathways Krebs Cycle

:.rtiary and would require very forceful conditions to oxidize it. What we can do,
rough, is a little rearrangement. Tertiary alcohols are known for their ability to
.:se their hydroxyl function and form a fairly stable tertiary carbonium ion as
::.own in Figure 8-18. You can think of this carbonium ion as being an
-::termediate in the reaction scheme. What can we do with this tertiary
:arbonium ion? Note that it has some alpha hydrogers. When alpha hydrogens are
:lose to a carbonium ion we may have an elimination reaction. This will give us
:is-Aconitate.

coo
rrom. .cetrt coa I
) CH'
-oH -Ht o t'
OOC- C- OH ooc- c @ OOC- C
I R Atttnituse lt
CH. \\ AGo'= +2.0 C.H
r'n)
,9 lg
Coo --
.l" Hydroxyl coo
Cirrate Tertiary carbonium ion cis-Aconitate
intermediate

Figure B-18

-: ive add water across the double bond in cis-Aconitate, we will produce
,.;ocitrate as shown in Figure 8-L9. Aconitase, which catalyzes the reaction from
-,lrate to cis-Aconitate, also catalyzes the reaction of cis-Aconitate to isocitrate.

o
coo
I

CH,
t- Hzo
OOC_ C OOC_ C-H
tt Aconilase I

:
C-H H-C- OH
I O AGo=-0.5 lg
coo coo
: cis-Aconitate Isocitrate

figure B-19

. rcitrate has a secondary hydroxyl group that is capable of being oxidized toa
i ::rbonyl function. We can oxidize isocitrate with NADo.
5

:* coo
o
coo
,:
l"' N,Ao' \ADH + H-
I

CH.
:ri ol\,/o
lrtrL L-n
t'
- CH,
I Isocitate t-
o: c=o
" f- dehvdrogenose
coo " AGo'= -2.0 loo '
-* Isocitrate B-Keto Acid c-Ketoglutarate
-ra

F"igure B-2O
' rirl,
;
l-.e enzyme that carries out that reaction is isocitrate dehydrogenase. Note that
:: intermediate that will form is a B-keto acid, which is quite unstable. This will
,dt!.
:=-ult in a spontaneous decarboxylation at the C-3 position of isocitrate to give a-
-l:
netoglutarate (o-KG) as shown in Figure 8-20.
'q

il -'.,pyright @ by The Berkeley Review 195 The Berkeley Review


(n Specializing in MCAT Preparation
Biology Metabolic Pathways Krebs Cycle

If you compare the relative stabilities of an alpha and a beta keto acid, you will
find that the more stable form is the alpha keto acid. The general mechanism for
the spontaneous decarboxylation of isocitrate can be seen in Figure 8-21. NoE
e-i"JT
n-c.-J f? that the enol form is in tautomerlc equilibrium with the keto form.
cH2:c: o

B-Keto Acid
We now have an o-keto acid which is reminiscent of where we started back at
pyruuate. Pyruvate was also an cr-keto acid. The cr-ketoglutarate will lose ttre
f. carboxyl group that is alpha to the carbonyl. After going though a reaction that b
l*co, similar to the reaction with pyruvate, we will get succinyl-CoA and carbon
?" dioxide (Figure 8-22). This reaction is catalyzed by the cr-ketoglutarate
R-C=CH2 dehydrogenase complex. Many anaerobic organisms do not have this enzyme
Enol and therefore cannot carry out this reaction.

1l o o
fl
coo NA,,DH + H+
coo
R-C-CH3 1"" N$D* 1",
Keto coA + ir, i;, 1 co:
a-KG Dehydrogenase
Figure 8.2 I i=;
I n Complex i=;
I

Coo'r LGo' = -7.2 i-con


cr-Ketoglutarate Succinyl-CoA

Figure 8.22

We can take advantage of the high energy thioester bond in succinyl-CoA to


some GTP (guanosine triphosphate) as shown in Figure 8-23. GTP can readily
converted into ATP (and we will find out how later).

o
coo o
I
GTP coo
cHz GDP+P1
lTlil I

t-
fnt + coA-sH
Succinyl CoA CHz
c=o Synthetase
IA
I coo"
S-CoA Go'= -0.8 Succinate
Succinyl-CoA

Figure 8-25

If you like, you can think of these high energy compounds (ATP, GTP, etc.)
nucleoside triphosphates (NTP)--a more generic name. The enzyme
catalyzes the conversion of succinyl-CoA to succinate is succiny
synthetase. This enzyne is in the class of enzymes referred to as ligases. \A/hy?
ligase will put two molecules together by using a high energy phosphate
When you use GDP and P1 to make GTP, a water molecule is prod
However, the water molecule that hydrolyzes the thioester bond and the w
molecule produced in the formation of GTP cancel each other out. Therefore,
is actually an anhydrous reaction. Also note that succinate is a symmet
molecule and the two ends, -CH2-COOe, are indistinguishable from one

ln the reaction shown in Figure 8-23 we have made a GTP. Since we started
with two pyruvate molecules (remember, the glucose molecule gets split),

Copyright @ by The Berkeley Review 196 The Berkeley


Specializing in MCAT
Biology Metabolic Pathways Krebs Cycle

nave a total of 2 GTP's formed in the Krebs cycle. In other words, at this
point we
have a total of 4 net NTp's since the start of giycolysis.

l.the following set of reactions we will be reconstructing succinate back into


OAA so we can repeat the Krebs cycle. The only differenJe between succinate
and OAA is the keto group on OAA.

The first step in the regeneration of OAA is an oxidation in which succinate is


:onverted to fumarate by the enz)ryne succinate dehydrogenase. Fumarate is in
fre cis configuration. If it were in the trans configuratiJn, it would be called
-'nleate. since we h_ave a dehydrogenation we need an oxidizing
agent. The
-'ridizing agent is flavin adenine dinucleotide (FAD). Let,s muk" some
ieneralizations. Every time you have seen an NAD involved in an oxidation
:eaction it has been involved with turning an alcohol into a ketone or an
:ldehyde (or vice versa). It does so by a hydride ion transfer. In the reaction OO
coo
rvolving FAD, shown in Figure 8-24, thereis no alcohor. In circumstances where i"" FAD FADH2 foo
-2 cH) \ ,/ -t c-H
rere is no alcohol, and particularly where you form double bonds, the oxidizing-
.2 CH2 Succinate -l
:educing agent will turn out to be FAD. (Note the oxidation states of the c-2 anl C-H

loo " Dehydrogenase


C-3 carbons as we proceed from succinate to fumarate.) loo "
Succinate AGo'= o Fumarate
-r Figure 8-25 we find that fumarate will react with water to produce malate.
I'Ialic acid was first isolated from apple trees and it is this subitance that gives Figure B-24
:oples their tart taste. The enzyme that catalyzes this reaction is fumarase. I-t is a
int dration reaction and falls in
the Iy ase clasi of enzymes.

\ote the hydroxyl group on malate. we coooo coo


can convert this hydroxyl group into a l.-^t
lieto group by the use of an oxidizing agent like NAD@. This is the reaction that c-H Hzo H-c- oH

return us to oAA and bring us full cycle (Figure g-26). The enzyme C-H Fumara.se CH.
"'.-i-11 tot'o
-::r-olved here is malate dehydrogenase. oAA is now ready to react with coo AGo = -0.9 coo
rother acetyl-CoA molecule and repeat the whole procedure again. Fumarate Malate

have not made any more NTp's. what has happened to all that energy? It Figure B-25
"ie
--.as not been liberated yet. we have not used oryg"tr. ar far as this is conce-ied,
=-"'erything could have been done anaerobically. we have made a lot of reduced
:Lrmpounds such as FADH2 and NADH + Ho which can be classed together as
reduced coenzymes. These reduced coenzymes represent a "dry brush" that could O NADH O
:e ignited with oxygen to turn them into their oxidized form, and in doing so foo NAD+ *.H* foo
'rerate a lot of energy. Part of this energy H-c- oH \ L ,/- c= o
will get turned into ATp. Even tholugh '
CH,
;','e have made only 2 GTP's by going through the Krebs cycle, we Malate CH'
have stored"a t-o
lon a Dehydrogenase coo
::eat deal of potential energy. This potential energy can be used to make a whole
:-cck of ATP's as soon as the oxygen is made available to us. Malate AGo = +7.1 OAA

-he function of the Krebs cycle is to turn those hard to oxidize carbons of acetic Figure 8.26
.:id into something that can be oxidized. By the time we went around the Krebs
--;c1e, two carbons had come off as carbon dioxide. we had oxidized two
-:rbons and we regenerated the starting material. Eventually the two carbons
rat we put into the cycle will come off as carbon dioxide, too.

lhe ,Krebs cycle can be thought of as a catabolic process in which the overall
'Go'value for the complete oxidation of an acetyl unit from acetyl CoA is about
-9.8 kcalslmol. It would be difficult to reverse this cycle.

-t this point in our story we have turned glucose (or some carbohydrate) into
:arbon dioxide via glycolysis and the Krebs cycle. In eukaryotic cells glycolysis

lopyright @ by The Berkeley Review 197 The Berkeley Review


Specializing in MCAT preparation
Biology Metabolic Pathways Krebs Cycle

occurs in the cytosol while the Krebs cycle occurs in the matrix of the
mitochondria. Recall that the average oxidation state of the carbon atoms in our
carbohydrate was 0. The oxidation state of the carbon in carbon dioxide is +4.
This process was therefore an oxidation. In order to balance that oxidation \^.e
needed to have a corresponding reduction. where did the electrons go? Those
electrons ended up in either NADH or FADH2. In order for these processes tc
continue we must reoxidize NADH and FADH2 back to NADe and FAD
respectively. we can summarize the essentiats of glycolysis and the Krebs cycle
as shown in Figure 8-27.

Mitochondrial
membranes
CoA
2 foruvate foruvate
o
ti 2 ATP
Matrix
Y
Acetyl
+NADH

I
{

il

o 2 NADH
I il
V Oxaloacetate
g m
NADH
M^ff Citrate
,m

n
Glucose
Cytosol t Krebs
Cycle {
W
r@

Fumarate
Isocitrate
m
|-FADH2 NADH<.$> ge, @
Succinate V fin
6[-ketoglutarate r{il
I CoA @[
rd
crp{ succinyr
' coA i:'^^
\co: + NADH im
ffi
Figure B-27 fuM

Un
The external oxidizing agent, otherwise known as the external electron accepto:. @0
is oxygen. As oxygen is used as the external electron acceptor, it witl t'e
converted into two water molecules by picking up four hydrogens and fo..r
W
electrons per oxygen molecule (O2). This can be summarized as shown in Figr:re
8-28.

Glucose
(CH2O)" NAD-. FAD 2H2O
\/- \/-
CO2 *I- NADH * 11*. -\ 02 + 4H+ + 4e-
FADH2

Figure B-28

Copyright @ by The Berkeley Review l9a The Berkeley Revier


Specializing in MCAT Preparatin
Biology Metabolic Pathways Electron Transport

-\ very important feature of the mitochondrion is that it is a double membrane


,.rganelle. The nucleus and the plant chloroplast (site of photosynthesis) are also
jouble membrane organelles. As we will learn, much of the ATP that we utilize
:n our metabolic processes is synthesized within the mitochondrion. The
Outer membrane
:iitochondrion is the site of oxidative respiration in the cell.
-'
''hen we discuss the chemical reactions that take place in the mitochondrion, it
--r important to have a firm grasp of the organelle's anatomy. As we mentioned,
:,e mitochondrion is composed of a double membrane system. There is an outer
rtembrane and an inner membrane. Between these two membranes is the inter-
membrane space. The inner membrane is folded into structures called cristae.
ltis increases the surface area of the inner membrane. The central cavity of the
-:ganelle is called the matrix. These structures are shown in Figure 8-29. Let's
:-<e a brief look at these structures.

'.e outer membrane is quite permeable and it contains protein channels called
: rrins that allow molecules with molecular weights as high as 10,000 daltons to
::ss through. [One dalton is nearly equal to the mass of a hydrogen atom. In Ribosome
, -:,er words, 10,000 daltons is about 10,000 times the mass of a hydrogen atom.]
.i-.iecules such as amino acids, carbohydrates such as glucose, and small
Figure 8.29
: -,r-peptides can pass through these porins. The outer membrane is composed
, about half proteins and half lipids. The anatomy of a generalized
mitochondrion.

:,.r'in8 inwards we next come to the intermembrane space. This area of the
:::ochondrion contains a variety of enzymes and also has a high proton concen-
: j::on because of protons which are pumped into this space from the matrix. In
::r words, the concentration of protons in the matrix is lower than the
. :entration of protons in the intermembrane space. This leads to an
-:trochemical gradient being established between the intermembrane space
-- : the matrix. Gradients have the potential to do work. That potential is utilized
-he svnthesis of ATP.

' inner membrane is folded into cristae in order to increase its surface area. A
=
::er surface area means more proteins. In fact, the inner membrane is
:posed of about 75 percent proteins and about 25 percent lipids. Some of the
-::ins in this metnbrane are permeases that allow for the passage of ADP into
natrix in exchange for ATP. ATP is synthesized within the matrix of the
:,:hondrion from ADP and Pi (inorganic phosphate) at the expense of a
,rn gradient. Because the inner membrane is quite impermeable to hydrogen
. protons), there are special permeases that allow for this ion's passage into
:.atrix from the intermembrane space (down their electrochemical gradient).
: .rample, there are permeases that allow for the simultaneous passage of
::anic phosphate and hydrogen ions into the matrix. Other molecules like
::n (O2) can readily diffuse through the inner membrane and into the matrix
- : mitochondrion.

-.r the matrix of the mitochondrion we find hundreds of different types of


: .:res. This is a region rich in enzymatic activity. One of the important
*-.emical pathways that occurs in the matrix is the Krebs cycle (also called the
".::oxylic acid cycle or the citric acid cycle). The Krebs cycle is the final
r rLorr pathway for the oxidation of fuel molecules like glucose. It also
-,ies intermediates for biosynthetic reactions and it generates carbon dioxide

:ht @ by The Berkeley Review 199 The Berkeley Review


Specializing in MCAT Preparation
Biology Metabolic Pathways Electron Transport

(COz) which can diffuse out of this organelle. ATP is synthesized by an ATP
synthase protein which is situated on the matrix side of the inner membrane
Also, within the matrix we find mitochondrial DNA.

So far we have turned glucose (or some carbohydrate) into carbon dioxide via
glycolysis and the Krebs cycle. In eukaryotic cells glycolysis occurs inthe cytosol
while the Krebs cycle occurs in tlne matilx of the mitochondria. We mentioned
that the average oxidation state of the carbon atoms in our carbohydrate was (L
The oxidation state of the carbon in carbon dioxide is +4. This process wffi
therefore an oxidation In order to balance that oxidation we needed to have a
corresponding reduction. Where did the electrons go? Those electrons ended up
in either NADH or FADH2. In order for these Processes to continue we must
reoxidize NADH and FADH2 back to NADo and FAD, respectively.
external oxidizing agent that we will use for this reoxidation is oxygen. We witr
see that as oxygen is used as the external electron acceptor, it will be converted
into two water molecules by picking up four hydrogens and four electrons per
oxygen molecule (O2). This process, which is referred to as electron tra
and oxidative phosphorylation, occurs on the inner membrane of
mitochondria.

Mitochondria are about the same size (1500 nm by 500 nm) as the prokary
bacterium Escherichiq coll. Mitochondria have both an outer and an i
membrane. Between the two membranes is the intermembrane space. The ou
membrane is permeable to many small molecules and ions. The inner
however, is essentially impermeable to most polar molecules and ions and it
highly folded into structures called cristae. Because prokaryotic cells do not
mitochondria, they carry out the reactions of electron transport and oxida
phosphorylation on their cytoplasmic membrane (i.e., the inner pl
membrane). (Prokaryotic organisms have an outer membrane, followed by a
wall, and then an inner membrane.)

Mitochondria contain their own DNA. They arise from the growth and di
of existing mitochondria within the cell. Human mitochondrial DNA (mt
for short) is circular and contains 1,6,569 base pairs. This DNA encodes for
tRNAs and 2 rRNAs. It also codes for 7 subunits of the NADH-Q
enzyme. This is a complex located on the inner membrane and is responsible
proton trans-Iocatio?? across the inner membrane. Cytochrome red
cytochrome oxidase, and ATP synthase subunits are also encoded by
mitochondrial DNA. We will learn the functions of these complexes in just a bit.

How are the reduced NADH and FADH2 coenzymes reoxidized? Recall that
Krebs cycle occurs in the matrix of the mitochondria in eukaryotic cells.
reduced coenzymes, therefore, are released into the mitochondrial ma
Associated with the inner mitochondrial membrane ale sequences of prc
complexes that act as electron carriers' Taken collectively, the
reiluctase (also called Complex I or Site 7), succinate-Q reductase (also
Complex II), cytochrome reductase (also called Complex III or Site 2),
cytochrome oxidase (also called Complex IV or Site 3) protein complexes
often referred to as the respiratory chain (due to their eventual involvement
oxygen) or electton tuansport chain. There are many of these respiratory ch
associated with the inner membrane. In turn these protein complexes
composed of an array of prosthetic groups (e.g., FMN, Fe-S, FAD, heme, and
which can act as electron carriers. As we will see, the electrons from NADH (

Copyright @ by The Berkeley Review 200 The Berkeley


Specializing in MCAT
Biology Metabolic Pathways Dlectron Transport

into the electron transport chain at the level of the NADH-Q reductase complex
rvhile the electrons from FADH2 enter by way of the succinate-Q reductase
complex and a molecule called coenzyme Q (also known as ubiquinone). This can
be seen in Figure 8-30.

Let's consider each of the enzyme complexes shown in Figure 8-30 in a little more
NADH
detail. We'll start with the NADH-Q reductase complex first. Reduced NADH +
He passes two electrons and two hydrogens to the oxidizedflaain mononucleotide
FMN) prosthetic group associated with this enzyme complex. This results in the NADH-Q
:roduction of reduced FMNH2 and the regeneration of oxidized NAD@. The Reductase
electrons from FMNH2 are passed to a series of iron-sulfur clusters (Fe-S) in which
]re iron atoms cycle between the reduced ferrous (feZe, and the oxidized ferric
Fe3o; states. There is an uncertainty as to how many Fe'S clusters there are in
Co mplex I FADH2
ne NADH-Q reductase complex. The electrons will eventually be passed from a
:educed Fe-S moiety to the oxidized form of coenzyme Q (Figure 8-31).
I
C Qc= Succinate-Q
NADHY- F\,IN FC]+-S).,- FC3+-S\ZCOAHZ Reductase
Y
0 Complex tr

Cytochrome
Reductase
NADH-Q Reductase
Complex trI
Figure 8-3 I I
Cyt c
.-t the end of the Fe-S complex there is a quinone. It is called coenzyme Q (or
CoQ). The oxidized quinone is just called quinone while the reduced quinone is
-il)ed dihydroquinone. Coenzyme Q was first discovered back in the 1930's and it
+ or1 ge n
,.as turned up in every organism which is capable of doing this type of electron fc".. h--. ta
:ansport. In the late 1940's the international fraternity of biochemists realized I bxioase I
::,at they were finding CoQ everywhere and so they thought it would be nice to
-eII it ubiquitous. Coenzyme Q in the oxidized form is now called ubiquinone Complex IV
,"-rile coenzyme Q in the reduced form is called ubiquinol. The structure of
*riquinone (CoQ) and ubiquinol (CoQH2) and a semiquinone (free-radical)
Figure B'3O
:,:ermediate is shpwn in Figure 8-32.

o. OH
-\>wute.
I
H*+e-
H,co-14)-
LE]-
"..n-A'_.". Hrco-UJ-
cHl
R
",."\,J- I
R

o OH OH
Ubiquinone Semiquinone Ubiquinol

Figure B-52

l:ca1l that as succiriate is oxidized to fumarate in the Krebs cycte by the enzyme
.::cinate dehydrogenase, FADH2 is generated. This FADH2 needs to be
'',:':idized. Succinate dehydrogenase turns out to be part of the succinate-Q
-.juctase complex. FADH2 is immediately passes its electrons to an Fe-S
::rtein(s) which then funnels them into the oxidized form of CoQ. FADH2 is
-. rridized to FAD as shown in Figure 8-33.

- ,:i'right O by The Berkeley Review 201 The Berkeley Review


Specializing in MCAT Preparation
Biology Metabolic Pathways Electron Transport

What we have done so far is to collect all of the electrons from all of the various
substrates into one pocket which is CoQH2. No matter what is happening we
will always have a large turnover of CoQ. our next interest is to reoxidize CoQ.
FAD FADH2 The electron carriers between CoQ and oxygen are called cytochromes
Cytochromes are electron transporting proteins that contain a heme prosthetb
group with an iron atom that alternates between the Fe2o and Fe3e conditioru
within the cytochrome reductase complex are two cytochromes, b and c1, and an
Fe-s protein. As coQH2 transfers one electron at a time to an Fe-s protein in thc
complex it is converted to CoQH. (semiquinone). The reduced form of
cytochrome b (Cyt b2o) reacts with CoQH.to give the oxidized form
cytochrome b (Cyt 63e) and CoQH2.

Figure 8.3
Cytochrome b3e can then oxidize another molecule of CoeH. to Coe. This
shown in Figure 8-34. Coenzyme Q is a molecule that carries two electrons at
time. However, the Fe-S protein can carry only one electron at a ti
Cytochrome b is the go-between that allows this interaction to occur,
electrons are eventually passed to cytochrome c. The iron atom of the
group of cytochrome c is bonded to a sulfur atom of a Met residue on one
and to a nitrogen atom of a His residue on the other side.

3+ 2+
C1t b Cyt b
Figure 8-54

The electrons that end up on the reduced form of cytochrome c in Figure


o next passed to the cytochrome oxidase complex. This complex consists of
o
o heme groups, heme a and heme a3, and two copper ntorns, one associated
a heme a and the other with he'me a3.
o
c3". Cyt a3 x These copper atoms can alternate between the +1 and +2 oxidation states-
0) electrons are passed from cytochrome c to the heme a and heme a3
(D
the cytochrome oxidase complex and then to oxygen. The transfer of
electrons to molecular oxygen leads to its reduction to two molecules of
:r{:,o q-4H* This is shown in Figure 8-35.

$cm cf weps At this point we have completed electron transport and have reoxidi
T
WTh. E- reduced coenzymes. This means that there must have been a strong ne
lligm D55 AGo' for this process. The question we want to get at now is how A
synthesized. So far we have not seen anyway for a high energy phosphaF
to be formed (none of these compounds are phosphorylated).

Curyqmm6[r @ @n Ih tserkeley Review 202 The Berkeley


Specializing in MCAT
Biology Metabolic Pathways Oxidative Phosphorylation

oxiildti#$ri:mm:u$HH fition
lhere are three areas in this oxidation reduction scheme where ATp can be
..'-nthesized due to a conservation of energy. It turns out that the passage of two
.lectrons from NADH down this chain allows for the synthesis of 3 molecules of
\TP. This is described by the P/o ratio, The P/o ratio is the number of high
:nergy phosphate bonds made per atom of oxygen used. The P/o ratio for
F--\DH2 allows for the synthesis of 2 molecules of ATP. The reason that FADH2
:enerates only 2 ATPs is because the electrons from FADH2 enter into the
=lectron transport chain at a lower energy level than the electrons from NADH.

:J of the ATP molecules that we will be producing via the electron transport
rain depend on the presence of oxygen. By taking ADP and Pi and making ATp
".-e are doing a reaction which is referred to as a phosphorylstion reaction. The
--->toric term that is associated with these types of reactions is oxidative
nhosphorylation (ox-phos). oxidative phosphorylation will yield a flock of
-{TPs. All the other ATPs that we have made (i.e., in glycolysis and the Krebs
--' c1e) can be distinguished from ox-phos ATPs because they did not directly
:epend on oxygen. Al1 six of the high energy bonds that were formed in botir
:-,. colysis and the Krebs cycle are not referred to as oxidative phosphorylations
: -rt rather substrate level phosphorylations.
:-lon' are the A T Ps synthesized during oxidative phosphorylation? The
:echanism, which was first proposed in 1961 by Peter Mitchell, is called the
;hemiosmotic hypothesis. Mitchell said that ATP synthesis and the electron
-:arLsport chain are coupled together by a proton gradient that had established
.:-'elf across the inner mitochondrial membrane and not by a high energy
::osphorylated intermediate. The coupling factor for this mechanism is an
::z\/me called the FsFl ATPase. This enzyme allows for the synthesis of ATp at
:-e expense of the free energy that is released as a proton (Ho) passes through
:.is F6FlATPase from the intermembrane space to the matrix of the
:-rtochondrion. How was the proton gradient established?

: the following diagrams the membrane that is drawn will be taken to be the
'.tner membrane of the mitochondria. Associated with this membrane are the
::rLplex of proteins we collectively call the electron transport system (or the
:=:piratory complex). This particular array of enzymes allows for a smooth and
: -'ntinuous flow of electrons from one end to the other. As we have mentioned,
::.ese respiratory complexes are repetitive within the inner membrane.

-: there are reduced substrates (e.g., NADH or FADH2) contributing electrons


=-fier from within the mitochondrion or from without (e.g., the electrons from
::.e NADH synthesized during glycolysis), then the electrons will move down
::e electron transport chain and eventually come into contact with oxygen. The
,:,\'gen will be reduced to water. Mitchell's hypothesis is that there is a
::arrangement of these enzymes as they pass back and forth through their
:::dized and reduced states such that hydrogen ions are removed from the interior
:.atrix) of the mitochondrion and pumped out into the exterior (intermembrane
;:ace) of the mitochonidrion. Hydrogen ions are thought to be translocated across
-:e rnner mitochondrial membrane at each of the three enzyme complexes (i.e.,
:.re NADH-Q Reductase, Cytochrome Reductase, and Cytochrome Oxidase
--.mplexes). This is schematically shown in Figure 3-36. If this process repeats
,:-.elf, then the pH of the interior will increase (the acidity will decrease) with
:=spect to the exterior. That is what constitutes the proton gradient.

- rpyright O by The Berkeley Review 203 The Berkeley Review


Specializing in MCAT Preparation
Biology Metabolic Pathways Oxidative Phosphorylation

If there is this gradient of hydrogen ions where the


[Ho] is greater on the outside
than on the inside, then it will allow the flow of i,yd.ogJ' ions
back into the
Hish [H+l matrix through the FoFlATPase complex. This is u.ron-"qiitiurium
situation and
represents potential worf,If thelrydrogen ions are allowed to
enter through this
ATPase compler; then this work energy can be converted into
useful chemical
energy through the synthesis of ATp as shown in Figure g_37.

Inner mitochondrial
membrane

Figure 8-36

f
Low [H']

FlFoATPase

Figure 8-57

This ATPase complex is not part of the respiratory unit (electron transport
<

The oxidation takes place in the respiratory unit whiie phosphorylation


place at the ATPase complex. These ATpase units occur ali orru, th" iorr".
of the inner membrane of the mitochondrion and essentially alternate with
units of the respiratory assembly. These two units taken together, the elecl
transport chain and the F6FlATpase comprex, constitute thi oxidatiae
of ATP.

The hydrogen ions cannot freely pass across the inner mitochondrial
membra
If they could flow freely back and forth across the membrane, no work would
done. This membrane-must exclude hydrogen ions except at the ATpase
port
entry. Not only is this membrane impermeable to protons but it is al
impermeable to almost all charged molecules unless there are specific in
protein ports which allow the molecule in question to come across. For exa
the membrane would be impermeable to ATp, ADp, pi, NAD, and N
unless there was a specific port to allow these molecules purrug".

Consider for a moment the integrity of the mitochondrial membrane. If


were to happen to the integrity of the mitochondrial membrane, it would have
affect on the oxidative phosphorylation but not on the substrate level
phorylation of ATP. Physical damage to the mitochondrial membrane can in

phosphorus from compounds inside the matrix to ATp, then the removal of
membrane should not have made a difference and phosphorylation should

Copyright @ by The Berkeley Review 20,4 The Berkeley


Specializing in MCAT prel
Biology Metabolic Pathways Oxidative Phosphorytation

be taking place. But that was not the case because every time a hole was punched
in the mitochondrial membrane, oxidative phosphorylations ,,o longu,
proceeded. substrate level phosphorylations did cbntinle though.

It turns out that there is more than one way to punch a hole in a membrane. one
way to do this is to chemically modify the membrane or even modify the passage
of the hydrogen ions across that membrane. suppose hydrogen ions could cross
the membrane without the membrane realizing it. If yort modify the way in
tllt.h the hydrogen ions can enter into the matrix such "orld
that they sneak past the
ATPase complex without doing any work, then that will wipe out oiidative
phosphorylation.

{ iv9 aaa a hydroxyl group to dinitrobenzene, we will make a compound" called


2,4-dinitrophenol (2,4-DNp). This is one of the first compounds discovered that
.auses a problem with the integrity of mitochondrial membranes. 2,4-DNp is
a
sufficiently strong acid. Recall that phenols are acids--they are slightly acidic. If
1,4-DNP were capable of moving across a mitochondriai membiane and then
cissociating inside, it would have transported. a hydrogen lon across the
nembrane. The method of getting a protonicross the membiane is referred to as
a proton transport.

,\'hat will be the effect of the presence of 2,4-DNp on a system such as


this where
-he hydrogen ion gradient
was the coupling aspect of electron transport and ATp
:ormation? It will uncouple it. If we weie to uncouple this hydrogen ion gradient,
would be just getting -52 kcals and no work. This rn"ut r thai all the ihemical
;'.'e
:rergy will be dispersed as heat.

-r similar heat production mechanism is utilized by hibernating


animals and
:ervborn human babies. There are specialized fat ceik in which mitochondrial
:-spiration is naturally uncoupled from the synthesis of ATP. These fat cells have
: \'ery large number of mitochondria, and since the mitochondria have
:"'tochromes which contain iron, they give the fat a brownish color. In infants
:-ts brown faf is located in_the interscapular space. Adult humans have very little
::orvn fat. Embedded in the mitochondria of the brown fat is a special transport
:.'stem that allows protons to move back across the inner membrane. since the
-. drogen gradient is dissipated, there is no means to capture the energy of ATp.
-:-otead, this energy,is lost as heat.

',-ecan calculate the overall generation of energy from glucose. In the process of
i--rcose going to pyruvate we found that 2 net ATps were generated directly. In
::s process we also produced 2 NADHs which were extramitochondrial. Tlrese
:r
=a-mitochondrial NADHs are transported into the mitochondrial matrix by
::j1er one of two shuttle systems--the glycerol phosphate shuttle or the malate-
ispartate shuttle. rf the glycerol phosphate shuttle is utilized, each cytoplasmic
's,\DH yields two ATPs. rf the malate-aspartate shuttle is used, each cyto-
:,,=mic NADH yields the normal three ATps.

- re Krebs cycle the 2 pyruvates that we generated from glycolysis produced 6


i'D2s. we also proddted 2 more high energy phosphate bonds in ihe form of
lll-P from the Krebs cycle. The NTps that we have produced so far are referred
's substrate leael NTPs because they have not come into contact with oxygen.
:
" also found that the Krebs cycle produced 2 FADH2s (one for each pyruvite)
,": 8 NADHs (four for each pyruvate). These NADHs and FADHfs will be
::a-mitochondrial.

riright @ by The Berkeley Review 205 The Berkeley Review


Specializing in MCAT Preparation
Biology Metabolic Pathways Oxidative Phosphorylation

Recall that we mentioned that the p/o ratio for NAD is 3


ATps while the pio
ratio for FAD is 2 ATps. Consequently, if we utilize the marate-aspartate
shuttle, the 2 NADHs from glycolyiis wiil give us 6 ATps while
the 8 NADH'
fr91n the Krebs cycle wilr give us 24 ATps. fhe 2 FADH2,
tro*it Krebs cycle
Bacteri will give us 4 ATPs. we have 2 ATps made in glycolysi and we have
" 2 GTps
Photons (which can be converted into ATp) from the r-rebs cycle. Therefore,
g+ the total
ATPs produced is 38. If we had used the glycerol phosphate shuttre,
ATP \ we would
have produced 3d ATps. It turns out thit-the thirmoiynamic efficiency
Synthetase for the
complete oxidization of glucose is about 40% under stairdard
"oi.raitiorrr.
Halobacterium halobium (from the Greek hals f or salt) is an archaebacterium
AD that requires an optimal sodium chloride concentration of about 4.3 I\{-
+Pi Compare this to the salt concentration in sea water which is about
0.6 M. one
place where these arachaebacteria can be found is at the south
Reconstituted Vesicle end of san
Francisco Bay. Its membrane can be separated into various fractions,
Figure B-38 one being
a purple-m-embrane protein called baiteriorhodopsin which
contains seven
transmembrane helices and a light absorbing ptorlh"ti" group
called retinal (a
chromophore). The retinal found in bacterioihodopsin isjaen'ticat
to the retinal
in. the rhodopsin of the rod cells of vertebiates. when
loun! incident photm
flux impinges on this purple membrane in the absen"" or o"yg"r,,
it acts like a
NADH proton pump and translocates protons from the cytosol of the lell
b the outsil
environment. This proton gradient can be used to synthesize ATp.
If oxygen i
prl""t, this organism can also carry out oxidative phosphorylation to
ATP.
Rotenone
NADH-Q G)
Reductase

Comple xr FADH2

CoQ..
I Succinate-Q
Reductase

I
Cytochrome
Complex II

Antimycin
experiment (circa 1974) added support to peter Mitchell's chemiosmo
hypothesis (circa 196r) involving the coupling of a proton gradient with a
svnthesis.

Reductase
AG)
one experimental means that was employed in elucidating the pathwav
electron flow in the electron transport thuitt *u, by th! use-of sp*
Complex trI inhibitors. Rotenone is a toxic plant poison that inhibits electron tranq
within the NADH-Q reductase complex. Antimycin A, a toxic antid
isolated from a strain of streptomyces, blocks electron flow b
cytochromes b and c1 (at the cytochrome reductase complex). Addition
{"tAscorbate ascorbate reduces cytochrome c and allows for electron flow from cytochru
Oxygen c to oxygen. The cytochrome oxidase complex can be inhibitei by
CN,N3, molecules as cyanide (cNe), azide (N3e), or carbon monoxide (co). The
of these inhibitors can be seen in Figure g-39.
',to cl
Water

Figure 8.39

Copyright @ by The Berkeley Review 206 The Berkeley


Specializing in MCAT pre
Biology Metabolic Pathways Pentose Phosphate pathway

PuGffisG,:..ifililo$flnfiffi1li iiiiiriiiiiiiiiiiiii*i ,,iliii' ', ,;,;,,,,,;:, I

Let's consider the pentose phosphate pathway. The purpose of the pentose
phosphate pathway is to generate reducing power in the form of trllopH and
five carbon sugars such as ribose-5-phosphate. These reactions occur in the
cytosol of the cell and, as we will learn later, some of them will be involved in
the formation of other sugar molecules from CO2 in photosynthesis.

Recall that during glycolysis, glucose can be phosphorylated to form glucose-5-


phosphate. The enzyme that cataryzed this reaction is hexokinase. Glucose-6-
phosphate has many fates and one of them involves the pentose phosphate
pathway. rf we dehydrogenate (remove hydrogens) glucose-6-phosphate at the C-1
carbon, we will get a molecule called 6-phosphoglucono-d-lactone. This
oxidation reaction yields NADPH and is catalyzed by glucose,6-phosphate
dehydrogenase. See Figure 8-40.

o
o
il
o- P- o- cH.
o9
NADPH O- P- O_CH.
II
+
oo NApP.
T ,F o,,

- "6 /
o-H
\ \-^
ctuco,se 6-phosphate no\fl--1-
'
Qenyarogenase
l"
Glucose 6-phosphate 6-Phosphoglucono-
Slactone

Figure B-4O

}e predominant form of glucose-6-phosphate in solution is in the ring form.


l"ote that the C-1 position is bonded to two oxygen atoms. As shown in Figure &
{0, oxidation at the c-l carbon will give 6-phosphoglucono-d-lactone. If we have
: carbonyl function that is next to an oxygen atom in a ring structure, we have a
,',;lic ester or lactone.

o o o
Oll O oat o"
o-P-o ,o
I

o- 9Ht
C
I
NADPH F 'o
H.C- OH + H* H-C- OH cH2-oH
I
Ho - c_H NADP- |
I

c= o cPt
coz i= o
1I{ \' - ^[-o, 1
- Hzo ".f-o"
I
H-C- OH
6-Phospho-
plutonate H-C- OH
| "-t-o"
I
H-C- OH
Lactonase | " ,,,
cHr.o-Po,) dnatc
I I

OH cHr-o-po.,2 cHr-o-Po,2
6-Phosphoglucono- 6-Phosphogluconate B-Keto Ribulose
6-lactone Intermediate 5-phosphate

Fi'gure B-41

l:.e next reaction is the hydrolysis of that cyclic ester. The enzyme that catalyzes
---: reaction is called lactonase. Lactonase hydrolyzes the lactone and lets the
."-r
3 open to form 6-phosphogluconate. Recall that the best way to release carbon
:.--xide is to have a keto function at the beta position. In the structure of 5-

;rright @ by The Berkeley Review 207 The Berkeley Review


Specializing in MCAT Preparation
Biology Metabolic Pathways Pentose Phosphate pathway

phosphogluconate we have an alcohol function at the C-3 position. If we could


relieve that hydroxyl group of its hydrogen, we would have an intermediate that
is a beta-keto acid. This reaction is catalyzed by NADpo and the enzyme d-
phosphogluconate dehydrogenase. The transitional state, which is the beta-keto
acid, rapidly decatboxylates to form ribulose-5-phosphate. This set of reactions is I

shown in Figure 8-41. The reactions shown in Figure g-40 and Figure g-41 il

constifute an oxidatiae way to form S-carbon sugars.


I

The enzyme phosphopentose isomerase is able to isomerize ribulose-5-


phosphate to ribose-5-phosphate. This reaction is shown in Figure g-42 and
il
proceeds through an enediol intermediate. This reaction should look familiar to
you. Recall the interconversion between glucose-6-phosphate and fructcse-6- ,il

phosphate in glycolysis. The intermediate in that reaction was an enediol, too. ilt
fi
o.H
\\r
ilh

CH.-OH CH.OH C
l' il I il.
C=O C- OH H-C- OH ,@
I I I
H-C- OH H-C- OH H-C* OH @il!
I Phosnho- I I
m
H-C- OH pr,,ioro H.C_ OH
I
H-C- OH
I
CH2-O-PO32- isomerase cHz-o-Poj2- cH2-o-PO32-
Ribulose Enediol Ribose
5-phosphate Intermediate 5-phosphate

Figure 8-42
Formation of Ribose 5-phosphate

What is the value of having something like NADP? It turns out that this molecuc
serves a special purpose. AII of the reactions that we have considered so far a:e
energy releasing reactions in which ATP was generated. Because they rr*e
energy releasing reactions they were also breakdown reactions. The entropy tr:w
increasing.

All of the reactions that are degradative and energy releasing are referred to am
catabolic reactions (catabolism). NAD is the coenzyme that is involved in t}'cmer
reactions. In contrast, there are reactions that require energy to decrease Fre
entropy by putting things together. These are biosynthetic reactions and ::e
collectively characterized by the term nnabollsn. NADP is the coenzyme that m
involved in these reactions.

There are a variety of things that can happen to the S-carbon sugars that we tu
just mentioned. For example, enzymes such as a transketolase, a transaldo
or even an epimerase can make use of a specific S-carbon sugar as their
Let's examine some of these reactions.

The transketolase enzyme can catalyze the synthesis of xylulose-S-phosp


and erythrose- -phosphate from glyceraldehyde-3-phosphate and f
phosphate. The coenzyme involved in this reaction is thiamine pyrophosi
(TPP). This is shown in Figure 8-43. Note that the "boxed" portions of
molecules are those portions that are being transferred.

Copyright @ by The Berkeley Review 204 The Berkeley


Specializing in MCAT
Biology Metabolic Pathways Pentose Phosphate Pathway

T&;oH I
i= o I ,,on,- l- cH"oH I
I|-.r-------------rketolase o.H
HO- C-H
li I

o.H C
I

C H-C- OH TPP HO_ C_H H-C- OH


I I I I
H-C_ OH H-C- OH H.C- OH H-C- OH
I I I I

cHr-o-Porz CH2-O-PO32- CH2-O-PO32- cH2-o-Po32'


Glyceraldehyde Fructose Xylulose Erythrose
3-phosphate 6-phosphate 5-phosphate 4-phosphate

Figure B-45
Intermediates in the Pentose Phosphate Pathway

Xylulose-5-phosphate can also be converted to ribulose-S-phosphate by the


enzyme phosphopentose epimerase. This enzyme is a member of the isomerase
class of enzymes. Note that these two molecules are epimers at the C-3 carbon.
This is shown in Figure 8-44.

I cH2-oH CHr-OH
t-
I c-o NAD-iinked C= o
Ir
I HO- C-H H.C- OH
it Phosphopentose I

lti
H-C- OH epimerase H.C- OH
I

cH2-o-Po32 cH2-o-Po32-
Xylulose Ribulose
S-phosphate 5-phosphate

Figure 8-44

-low could we make this epimer? The hydroxyl function at the C-3 carbon of
,.r-lulose-5-phosphate could be oxidized to the corresponding keto function. This
'",-ould eliminate the chirality. Reduction of that keto function could then give the

=pimeric hydroxyl function at the C-3 carbon of ribulose-5-phosphate. The


:oenzyme NAD islnvolved in this oxidation-reduction scheme. NAD would be a
:atalyst as it is not involved stoichiometrically.

lhrce we have ribulose-S-phosphate it can undergo an isomerization rcaction to


jve ribose-S-phosphate as shown in Figure 8-42. Ribose-S-phosphate could
:,,-entually become incorporated into biomolecules such as RNA, DNA, ATP,
,:d NAD.
Irythrose-4-phosphate can react with DHAP via an sldol condensation and the
aldolase enzyme to give sedoheptulose-"1.,7-diphosphate (first isolated from
.',-ocados, which belong to a family of plants generally known as sedum plants--
-.ence the prefix). Sedoheptulose-1,7-diphosphate can be converted to sedo-
":i eptulose-7-phosphate by a hydrolase enzyme known as sedoheptulose-1,7-

Jiphosphate phosphatase. This is shown in Figure 8-45.

-,,.pyright @ by The Berkeley Review 209 The Berkeley Review


Specializing in MCAT Preparation
Biology Metabolic Pathways Pentose Phosphate ? athw ay

CH.-OH
?H2-O-PO32- t'
o c= sed-1.7-dP c=o
I
o.. Ho- i-H phosphatase-
HO- C-H
'c ,H s-l- on + H2o
H-C-
I

I OH
H-C- OH
t+ CH2-o-Po32- u-b- oH
l-t
I

H-C- OH
H-C- OH
I f= o
- Atd"k* H-C- OH I

H-C- OH
I
CH2-o-Po32- cH2-oH CH2-O-PO32- CH2-o-Po32-
Erythrose DHAP Sedoheptulose Sedoheptulose
4-phosphate 1,7-diphosphate 7-phosphate

Figure 8-45

s-edoheptulose-z-phosphate can react with glyceraldehyde-3-phosphate


to form
ribose-S-phosphate and xylulose-S-phosphate as shown #rig.rru
s-46. Thi$
reaction is catalyzed by the transketolase enzyme.

o.H
.'c,
HO- C-H Trans-
I I
H-C- OH o...c, H ketolase H-C- OH
I -
H-C-
I
OH + H-C-
I

OH +
I
H-C- OH H-l- on H-C- OH
I I
cH2-o-Po32- cH2-o-Po32- cH2-O-PO32-
Sedoheptulose Glyceraldehyde Ribose
7-phosphate 3-phosphate 5-phosphate

Figure 8.46

sedoheptulose-7-phosphate and glyceraldehyde-3-phosphate can also react


form fructose-6-phosphate and erythro"e-+-pho"pirate. This reaction
catalyzedby a transaldolase enzyme as shown in figure A_+2.

Trans-
aldolase o..,11
.C
H-C- OH o.. ,H :+
I

H-C-
I
OH + C
I

H-C_ OH
H-C- OH u-l- on I

H.C_ OH
I I
I
CH2-O-PO32- CH2-O-PO32- CH2-o-lqz-
Sedoheptulose Glyceraldehyde Erythrose
7-phosphate 3-phosphate 4-phosphate

Figure B-47

Copyright @ by The Berkeley Review 2to The Berkeley


Specializing in MCAT prep
Biology Metabolic Pathways Pentose Phosphate Pathway

Note the portion of sedoheptulose-7-phosphate that is being transferred to


glyceraldehyde-3-phosphate. The reactions shown in Figure 8-42 through Figure
8-47 represent anon-oxidatiae way to generate S-carbon sugars.

The pentose phosphate pathway, which occurs in the cytosol of cells, is one way
to get five carbon sugars and NADPH. There is also a mitochondrial-linked
method of obtaining NADPH (which we will examine during fatty acid
metabolism). Most eukaryotic cells have mitochondria. However, one of the most
common types of (mature) eukaryotic cells that do not have mitochondria are the
erythrocytes (red blood cells). If mature erythrocytes do not have mitochondria,
then they must obtain their NADPH from the pentose phosphate pathway.

-\s oxygen becomes reduced it goes through a number of intermediate stages


such as the superoxide radical, hydrogen percxide, and hydroxyl free radical
oefore it is converted into water. All of these intermediate species are "toxic" in
-he sense that any one of them (outside the usual enzymatic channel of becoming
'-';ater) can cause cellular damage (which is a means of aging). There are various
:ompounds and enzymes which can scavenge these radicals and convert them
-nto less toxic substances. For example, compounds like vitamin C, vitamin E
iocopherols) and ergothioneine can act as free radical scavengers (or anti-
:xidants). Superoxide dismutase (SOD) can catalyze the conversion of the
'uperoxide radical into hydrogen peroxide and oxygen. Hydrogen peroxide can
:eact with an enzyme called catalase and be converted into water and oxygen.

NADP"
NADPH + H+

Glutathione
reductase
Reduced Oxidized
Glutathione Glutathione
(GSH) (GSSG)

\/\,/
w:-
' Glutathutne
n:\-,
peroxidase

F-rgure a.4a

:---.-drogen peroxide can also react with an enzyme called glutathione peroxidase
rnl be reduced to water. Reduced glutathione (GSH) reacts with the peroxide to
r::=L water and the oxidized form of glutathione (GSSG). Oxidized glutathione is
::luced back to the sulfhydryl form (GSH) by NADPH. The enzyme that
:-:lvzes this reaction is glutathione reductase. This is shown in Figure 8-48.

lLere are cases in which humans lack one or more of these "housekeeping"
s?',"rnes. For example, there are individuals who are catalase negative (they lack
f:-. enzyme) and who still manage to survive. Individuals have not been found
,,,':.o are completely lacking in either superoxide dismutase or in glutathione
re:.'rxidase. This suggests that these two enzymes are quite essential for normal
lL'::cgical function.

l l.;rlright @ by The Berkeley Review 2ll The Berkeley Review


I Specializing in MCAT Preparation
Biology Metabolic Pathways Pentose Phosphate p athw ay

If an individual were deficient in- the enzyme glucose-6-phosphate dehydro-


Figure 8-40), then that person would not 6e abie to synthesize
g,enase (see
NADPH' This sex-linked trait is the single largest class of all human mutations
and_ is quite prevalent in malaria-infested regions of the
world like the
Mediterranean area, central Africa, the Middle Eist, Lrdia, southeast Asia,
and
China. It turns out that glucose-6-phosphate deficiency is one means of
protection
against the malarial p2rasite
falciparum.lbtner means of protection
involves the sickle-cell trait.).pla-smodium
How is it that a glucose-6-phosphate deficiency
affords protection against this parasiie?

Glucose-6-phosphate dehydrogenase deficiency is sex-linked. It is on the X-


chromosome. Females are XX while males are Xy. If a male has glucose-G
phosphate dehydrogenase deficiency, he is said to be hemizygous for that defecL
A female may carry the defect on one of her X-chromosomei 6ut not on the other
one. she is said to be heterozygous for that defect. Heterozygotes for this
deficiency turn out to have red blood cells that are about ten times more resistant
to the malarial parasite than normal wild type individuals who do not have
this
deficiency. why? The malarial parasite requires products of the pentose
phosphate pathway and reduced glutathionJ for thlir survival. If there
is a
glucose-6-phosphate dehydrogenase deficiency, then these products
are limited-
Note that the female of the species is the one being protected because she is
the
one who is heterozygous for the trait. she is the one-who bears the offspringl

Individuals with glucose-d-phosphate dehydrogenase deficiency are unable to


form NADPH and therefore unable to form ieduced glutathione (GSH) as
shown in Figure 8-48. If GSH is not formed, then p"ro"*id", are free to lurk
within the cell and cause damage. Erythrocytes have a typical lifetime of abogt
Jz! days before they are degraded in the spreen. However, the erythrocytes
individuals with this deficiency have a relatively short life span (about t{ da1
femglyli9 anemia (lysis of the red blood cells) iesults and hlmogl obrn is retea:
into the blood. As the hemoglobin is destroyed the protein portLr, (i.e., globi
and the iron can be recycled. However, the iron-free porphyrin portioriof
heme is degraded in the liaer (and spreen and bone marrow) to a compo,
called bilirubin.If the concentration of bilirubin were to exceed what thl hrr
could excrete, a condition of hyperbilirubinemia results in which bilirubi
diffuses into the tissues and turns yellow. This condition is referred to
jaundice.

Copyright @ by The Berkeley Review 212


Biology Metabolic Pathways Gluconeogenesis

Glucoheogen€siS::
Gluconeogenesis is the synthesis of glucose from non-carbohydrate precursors
such as lactate, amino acids like alanine, and glyceroL Recall that during vigorous
exercise lactic acid accumulates in skeletal muscle. We had mentioned that lactate
is a dead end in metabolism, unless is can be converted back to pyruuate.In order
to convert lactate back into pyruvate it must first be carried by the blood to the
liver. At the liver, lactate is converted to pyruvate, which can be converted into
glucose by way of gluconeogenesis. Glucose can leave the liver and return to the
skeletal muscle to once again undergo glycolysis. These series of reactions, as
shown in Figure 8-49, ate referred to as the Cori cycle.

Figure 8-49
The Cori Cycle.

llloiecules like lactate, alanine, and glycerol can be converted into glucose by
:luconeogenesis. Note that all three of these compounds contain three carbon
"::oms (C3). It turns out that animals cannot get a net conversion of a two carbon
C2)compound like acetyl CoA into glucose. Recall that when acetyl CoA enters
-nto the Krebs cycle and combines with OAA, two carbon atoms leave as CO2. In
:ther words, the OAA that is regenerated is not synthesized de novo when acetyl
CoA is oxidized in the Krebs cycle. Animals lack the necessary enzymes that will
.llow the conversion of a two carbon acetyl unit into glucose. However, plants
:nd many bacteria can make use of these two carbon precursors because they
::-ioe the enzymes necessary for the proper convetsions. They use the glyoxylate
cl'cle and bypass,the two decarboxylation reactions in the Krebs cycle by
:onverting the two carbon acetyl units into a four carbon succinate molecule.
,.'-ith this in mind, let's consider gluconeogenesis.

--',-ithin
the cytosol of (eukaryotic) liver cells the ratio of NADH/NAD@ islow,
-:us favoring the oxidation of lactate to pyruvate. Once you have pyruvate you
right think that you can convert it to phosphoenolpyruvate (PEP) by a simple
:er.ersal of the reaction at Step 10 in glycolysis. Recall that the AGo for the
-onversion of PEP to pyruvate in glycolysis is -7.5 Kcals/mol. If we were to
:everse this reaction, the AGo' would be +7.5 Kcals/mol. This reaction must be
:,passed for one with a more favorable standard free energy change.

ihis bypass is catalyzed by the enzyme pyruvate carboxylase. Pyruvate


:arboxylase is located inside the mitochondrial matrix of the liver cells and
:-rntains a biotin prosthetic group that carries activated CO2. CO2 was
rctivated" at the expense of a molecule of ATP. In other words, ATP facilitated
--:re attachment of CO2 to biotin. Pyruvate then diffuses into the mitochondrial

lopyright @ by The Berkeley Review 2t3 The Berkeley Review


Specializing in MCAT Preparation
I
Biology Metabolic Pathways Gluconeogenesis

matrix and is carboxylated.to.form oAA. pylu,vate carboxylase


is an example of a ilm:
Iigase enzyme because a-high energy bond is being used.
ir *".gv was not used, qpq
the enzyme would have been a lyase. This is showi in Figure g_5'0.
OA
M
:mum

rsrlr,
ngm

@ I
Omu

I Pyruvate I erytor
dfiilrt
lCarboxylasel
idffie
OAA W
Figure 8,5O Ws'r

It turns o-u! pyruvate carboxylase is actiaatedby high leaels of acetyr CoA. I:


@
!n1t ",lftrmffi
tf9r9 alenigh levels of acetyl coA, then acetyl CoA mult not be ;hdir"
condensing with
oAA. why? Because the oAA levels are low and. there are not enough
to meet
the demand' The forma.tion of oAA from pyruvate is called
an anaplerotic regctior: msprT
(from the Greek, meaning to "fill up"). wiat happens to this
oeaz If the cell L. Mturffi
low in ATP, oAA will enter the Krebs cycle und .ondense
with acetyl CoA. Thi_. dryft
will lead to the eventual synthesis of more ATp. However, if the cell has
plenty c: w@ffi
ATP, then OAA will be utilized for gluconeogenesis.

Glxose
Lactate
'K"
N.+o- , f*r={ 9o'
oL ApP
.crP
\
NADH pyruvare n^
+ H+ potuntial co:
ll t- ADH + H+
ll ry,iy
n

l,_ *AD+
Cycle Malate
Cytosol

ry\'m'ate OAA Malare


7Z-\
ff, i|,1 TT: NIo.
Mitochondrial Matrix

Figure B-51

oAA is next reduced to form malate by NADH-linked malate dehydrogenase


This enzyme is in the oxidoreductase crass of enzymes. Malate is iran$ortec
across the mitochondrial membrane and is reoxidized to oAA
in the cytosol t,r
an NADo-linked Ttljt" dehydrogenase. oAA is decarboxylated ani
phosphorylated by GTp to givephoiphoenolpyruaafe (pEp). This reaction is
catalyzed by PEP carboxykinase and can be r"".t itr Figure g-d1.

right @ by The Berkeley Review 214 The Berkeley Review


Specializing in MCAT preparation
Biology Metabolic Pathways Gluconeogenesis l{t

rl
i

In reaction sequences shown in Figure 8-5L we have the potential for a futile
I

cycle. Why? If we go from PEP to pyruvate, we synthesize one ATP. Pyruvate to


OAA costs us an GTP. OAA to PEP costs us another ATP. We end up with a net
loss of one NTP. If we were to continue around this cycle, we would eventually
run out of NTPs and still not have gotten anywhere. It would be a futile effort.
I{owever, there are controls that regulate this potential futile cycle. If we were to
remove those controls, a lot of heat would be generated.

Once you have PEP the rest of the reactions in glycolysis are reversible and the
equilibrium will favor moving back towards glucose until fructose-L,6-
diphosphate is reached. This is true as long as there are high levels of ATP (i.e.,
the energy charge is high). High leaels of ATP turn out to allosterically inhibit
pyruvate kinase, the enzyme which converts PEP to pyruvate'

Recall that the AGo' for the conversion of Fructose-5-phosphate to Fructose-1,6-


diphosphate was about '3.4 kcals/mol. This reaction was catalyzedby phospho-
:ructokinase.Instead of trying to reverse this reaction it would be much easier to
:rydrolyze the phosphate at the C-1 position.

Recall that hydrolysis reactions are always favorable. The enzyme that catalyzes
this reaction is in the hydrolase class of enzymes and is called fructose-1,6-
diphosphate phosphatase, This is shown in Figure 8-52. Again, there is the
rossibility of another potential futile cycle.

ADP o
ilo
cH?-oH ,
*l-l CH"-O-P-O
O
I

C=O C= O,.,
I
lv
HO_ C-H F ructo se- 1, 6- Dip ho sp hate
HO- C-H
I
I

H.C_ OH Phosphatase H-C- OH


I I

OH O o r OH ll
H.C- H-C- O
ril o
cH?-o-P-o Pi cH"-o-P-o
-l
I

O^
I
oo
Fructose-6-phosphate Fructose- 1,6-diphosphate

Figure 8-52

iructose-6-phosphate is in equilibrium with Glucose-6-phosphate. Glucose-6-


rhosphate can be used to make glycogen (a storage form of glucose) or it can be
:onverted into glucose. The brain uses about 120 grams of glucose per day as an
:nergy source. It would be advantageous, then, to have a means for the
:onversion of glucose-6-phosphate into glucose.

R.ecall that the AGo'value for the reaction of glucose to glucose-6-phosphate is


:bout -4.0 kcals/mol. This reaction is catalyzedby hexokinase. It would be easier
:o overcome this enerly barrier by simply hydrolyzing the phosphate from the C-6
:osition of glucose-6-phosphate. This reaction is catalyzed by glucose-6-
phosphate phosphatase. These reactions represent yet another potential futile
,tlcle.This is shown in Figure 8-53.

Copyright @ by The Berkeley Review 2t5 The BerkeleY Review


Specializing in MCAT Preparation
Biology Metabolic pathways Gluconeogenesis

HO
,{, HO
I Step
ATP
1 AGo'= -4.0
'i' I
H-C- OH H-C- OH
I
I
HO- C-H Glucose-6-phosphate HO_ C.H
I
I
H-C_ OH H.C- OH
I
I
H-C- OH H-C- OH
I O
Hzo lll t:t
cH2-OH cH2 - o- p- o"
Glucose Og
Glucose_6_phosphate

Figure 8-58

Recall that glycorysis.(from grucose to pyruvate)


will give us a net yield of 2
Glt'coneogenesis (from pyruvate io gtn"or";
+Tlr'
th-ink of this cycle as one large po"tential
*itt".ori,r, a erpr. we cm
futile loop t}rut gi .J.,, u net loss of I
ATPs. "orra

Copyright @ by The Berkeley Review


216. The Berkeley
Specializing in MCAT prer
Biotogy Metabolic Pathways Fatty Acid Oxidation

F
Let's look at fatty acid metabolism. We will start with
fats or triglycerides which
have the general structure shown in Figure g-54. The
carboxyl function of the
fatty acid is in an carboxyester linkage iui*, tnu hydroxyl
or ine glycerol. The
structure shown in Figure g-54 has three carboxyester rinkages.
The most
.ommon fatty acids are 16 or 1g carbon atoms long and they 3
lre the energy H2C- O c-
storage form that is most frequently used in animals cH2-(cH2),iCH3
uid ptur",tr.' lo
I tt
is more €nergy available per unit weight of triglyceride than H-9- O- C- CH2_(CH2),,_CHr
Tut". of hydrates
lo
-rke glycogen. You can see that i? you think o"f -CH2- u', u"i"j u,r"
form in which lil
:arbon exists. If we were to oxidize that unit, then it wou*ld H2C- O- c-
take 1.5 oxygen CH2-(CH2)n_CH3
::rolecules to bring it to Co2 and H2o. In contrast,
if you were to think of a A Triacylglycerol
--arbohydrate structure like -CHoH-, then you would fir,d thut it
is already
:artially oxidized. As a result, only 1 oxygen molecule is required Figure 8.84
to oxidize that
:arbon and hydrogen completely to CO2Ind H2O.

-: the amount of energy that had been concentrated in these neutral fats
-'istead stored in the-form were
of glycogen, then you would be grossry overweight.
here are a couple of reasons tot tfrit. Fats are at a lower oxid.ation
state. The
;'"-erage oxidation state of the carbons in a fatty
acid molecule is -2. The average
" ddation state of the carbons in a carbohydrate
is 0. In the pro."r, of turning thit
::-rbon into Co2 there is a rot more oxidation taking
plaie. The result is more
being produced from the buming of a gram oifut
":.9-y
:*bohydrate. Fats are also fairly hydroplobic"and so when "o-pured
to a gram of
thiy are stored there
-: not very much water. This is quite different from the stoiage
of glycogen
:rere we find the molecule to be iull of hl/droxyl groups.
There is a lot of water
,;:red along with the glycogen. one disadvantage lrrat iat
has is that it cannot be
::tabolized anaerobically. Fats have to be metabolized aerobicaily.

-;t s consider the way in which fat is metabolized. This pathway involves g
::7\'matic steps. The first step that we want to consider involves
the enzyme
-pase' To mobilize triglycerides (or neutral fats), the first step
't,;'clysis of the carboxyester bond. This
would be the
is accomplished by hydrolysis of the
:-:-r-ceride into a molecule of glycerol and 3 fatty acid residue's.
This is shown
_ f tgure 6-55.

o
il
c- cH2-(cHrn-cH3 HO- C- Rl
",f-
IO H2C- OH
lil Lipase o
I tl
H-q- O- C- CH2-(CH,)n-CH3 +
L_____________z H-C- OH + HO- C- R2
lo
til
3 H2O I

H2C- OH o
il
H'C- O- C- CH2-(CH)n-CH3 HO- C- R3
Glycerol
Fatty Acids

ffqnare 8-58

r: : rriefly consider what happens to the


"a:x::ron shown in Figure fatty acids and, glycerol after the lipase
8-55 has taken place. The fatty uliar tnut are produced
* .L- :e undergo a series of reactions
ur,d b" converted to acetyl CoA. Glycerol
' L- :e also be converted to acetyl CoA, but by a different series of reactions.

ur -rsht @ by The Berkeley Review 2t7 The Berkeley Review


Specializing in MCAT preparation
Biology Metabolic Pathways Fatty Acid Oxidation

Acetyl CoA will then be able to enter into the Krebs cycle and energy, in the form
of ATP, will be produced as we have previously discussed. A very general vier,r-
of this outline is shown in Figure 8-56.

FATS
Glycogen

Glycerol in
Occurs FattY Acids
cytosol
;ytosol
r---l
the

\l
I

DHAP
il
r Occurs in the I

I mitochondrion
I it I

Pyruva te tsAcetylCoA

Figure 8-56 ll

i
Glycerol Metabolism I
The activation of glycerol is catalyzed by glycerol kinase and ATP. Glycerol-} (
phosphate is produced. The coenzyme NAD@ reacts with glycerol-3-phosphate I
to form DHAP. This is catalyzed by glycerol phosphate dehydrogenase ani
occurs in the cell cytosol. This general reaction sequence is shown in Figure 8-57. T
(
NADH UI

ArP oPt NAD+ + H+ M

H2C- OH
I
\_-,/ 't\,/ OH
H'C- ' I H2C- OH
I
ffi

H-C_ OH t--------------+ H-C-OH r- c=o ,i!


I
Glycerol
I $
H2C- OH H,l- opo.z- GlYcero! H2C- OPO32-
Kinase phosphate 0m

Glycerol Glycerol delrydrogenase DHAP W


3-phosphate d

Figure 8.57

Activation of Fatty Acids


The second step that we want to consider is the actiuation of the fatty acids. We n'il
find that fatty acids are degraded 2 carbons at a time. If we are going to sph:
these carbons off two at a time, then we will need some type of "handle" at the ft
position of the fatty acid. What we will need is a keto group at the p-position. L
we had a p-keto acid, the carboxyl function would decarboxylate and we wouli
lose that carbon atom. As a result this is not handled as a p-keto acid but rather a.
n
the thioester. The thioester of a p-keto acid does not decarboxylate. How do rre h
form this thioester linkage? ffill
r die

Wfr1

Copyright @ by The Berkeley Review 2ta The Berkeley Revieu {i,m


Specializing in MCAT Preparation
Biology Metabolic Pathways Fatty Acid Oxidation

The driving force for forming the thioester linkage


between the sulfhydryr group
of CoA and the carboxyr gtolp ofa fatty acid is erp.
If we are going to do this
reaction with ATp, then one phosphate grou-p
will not be enoughlTheiydrolysis
energy of a thioester is about -z.b kcal/mol-. If we allowed
thi rrr" of only one
1sh gne.rsr phosphate bond, then we would have a lC"tor -7.3 kcar/mol.
clearly this reaction will not proceed very well. what we find ""ry
here is that ATp is
rydrolyzed to AMp and pyrophosphate (ppJ. pyrophosphate wilr be
nvdrolyzed in the presence of water and pyrophorpt
two morecules of
-'t.hophosphate, giuing a AGo'of another -73Kcal/mol. "t"r"io
The activation of the fatty
acid is now complete as shown in Figure 8-58. This
activation reaction takes placl
;'n the outer mitochondrial membrane and is catalyzed,by
the enzyme acyl CoA
srnthetase (which is in the ligase enzyme class).

o
rr O
R-C-O + ATP R_ C- AMP + PP;
Fatty Acid Acyl-CoA
Acyl Adenylate
Synthetase

-.
R_C-AMP + HS-CoA :4 R_ C_ S_CoA + AMP
Acyl Adenylate Acyl-CoA

Figure B-EB

lnce,the fatty acid is activated in the cytosol of the cell it


needs to be transported
- to the mitochondrial matrix where it can be oxidized. Activated fatty acyl CoA
: olecules are shuttled across the inner mitochondrial membrane by .u*itir,u.
l:ce the activated fatty acid is released in the mitochondrial matrix,
carnitine
' j] return to the cytosolic medium and the process will repeat itself.
The B-Oxidation pathway
l:-lc.e the acyl-CoA molecule is in the mitochondrial
matrix, we can begin B_
: ':dation. The first four steps of
B-oxidation are highly reminiscent of the steps
-:-at we saw in the Krebs cycle. The third step in our breakdown
reaction is the
: -:mation of a trans double bond
between the alpha and the beta carbons in the
ic'l-CoA structure.'This is accomplished by an oxidation of the acyl_CoA
'::ucture with FAD. lhe e11zme involved is acyl-CoA dehydrogenase. This
. r:dation yields an dnoyl-CoA. Acyl-CoA slightry ,eru^bie, succinic acid
r :ereas enoyl-CoA slightly resembles fumnric qcid'n the Krebs cycle. This
is
;. lrvn in Figure 8-59.

g FiDH'?
il\/ "to Ho
p.- cH2 - cHz-cHz- t- s- coa * R- cH2-a=
- l-'J- r-
Oxidation |
"oo
H
Acyl-CoA
Fatty Acyl-CoA dehydrogenase Enoyl-CoA

ffigure B-59

r ::ep four we have hydration of the double bond in


enoyl-CoA. The two ends of
1'-' molecule are not symmetrical and therefore the hydration
of enoyl-CoA is
,::-tospecific. The enzyme that catalyzes this reaction is enoyl-CoA
hydratase. we
r
- only get the L-isomer, which is L-Hydroxyacyl-coA as shown in Figure g-60.
l'right @ by The Berkeley Review 219 The Berkeley Review
Specializing in MCAT preparation
Biology Metabolic Pathways Fatty Acid Oxidation

This reaction is analogous to the hydration of fumarate to malate in the Krebs


cycle.

Ho"rpHoHo
ril\ttil
R-CHr-9=C-c-s-
'lHydration'ri CoA -- R-CHr-c-C- C- s- CoA ,l
H Enoyl-coA HH lw
Enoyl-CoA Hydratase L-Hydroxyacyl-CoA r@

Figure 8-6O

In step fiae we have another oxidation reaction where we introduce a keto group at
the B-position. This reaction is catalyzed by the enzyme L-3-hydroxyacyl-CoA
dehydrogenase. The coenzyme involved is NADe. The reaction is shown h
Figure 8-61. This reaction is analogous to oxidation of malate to oxaloacetate in
the Krebs cycle.

NADH

NAD* +T*
HOHO,IOHO
I I
" ,-coa - ^jS41 R-cH2-[-l-[-,-"oo
R-cH2-9-f-c-l oxidation I

H H L]-Hydroxyacyl-CoA H
L-Hydroxyacyl-CoA dehydrogen'ase Ketoacyl-CoA

Figure 8-61

In step six we have the cleavage of ketoacyl-CoA between the s and B carbon_.. -4,
second molecule of CoA-SH is used in the process. The products are an acetvL
CoA molecule (which can be utilized in the Krebs cycle) and a fatty acyl-CoA
molecule that has been shortenedby 2 carbon atoms. This is shown in Figure 84I-
The enzyme that catalyzes this thiolytic cleaaage is B-ketothiolase. The fatty aqll.
CoA molecule that we just produced is already actiaated, which means that we
not have to repeat the activation step shown in Figure 8-58.

Figure 8-62
ltg i
The Oxidation of Palmitic Acid (a C16 even-numbered fatty acid)
If we were to completely oxidize palmitate using the B-oxidation pathway, '@',]i

would go through the cycle that we just described 7 times to get 8 two-ca 1m

acetyl CoA fragments, 7 FADH2 molecules and 7 NADH + He molecules. mfr


can be seen in Figure 8-63. In return for these products we will get a wealth
ATP's via the Krebs cycle, electron transport and oxidative phosphorylation.
point of this process is that the sum of the ATP liberated by the comp
oxidation of a f.atty acid makes this a very elegant breakdown process.

Copyright @ by The Berkeley Review 220 The Berkeley


Specializing in MCAT Prepara
Biology Metabolic pathways Fatty Acid Oxidation

cH3-CH2-l-cH2_cH2_l-cH2_CH2_l_cH2_CH2_l_CH2_CH2_l_CH2_CH2-._CH2_""r_'_""r_aoo:

#8 #7 #6 #5 #4 #3 #2 #I

now in a position to calculate


'v:,ul: of one morecule of palmitic acid.the net production of ATp from the
oxidation In the
ar{ we hydrolyzed the pyrophosphate bond. In activatio" ,t"p we used 1 ATp
other words, we have used 2
u:"rgl-phosphate bonds to aciivate the fatry acid for
lql
rf the B-oxidation process we will have formed the B-o"iautior,. At rhe end
eqrrirrai" nt of 129high energy
:hosphate bonds. The rest of the steps in our process
are outlined in Table g_1.

Process ATP Equivalents


Activation
-02
Oxidation (7 rounds)
FADH2 (worrh 2 ATp's each) +14
Z
7 NADH (worrh 3 ATp's each)
+21
Acetyl-CoA (catabolism)
8 Acetyl-CoA's enter Krebs Cycle

8Acetyl-CoAx3NADH +72
8Acetyl-CoAxlFADH2 +16
8Acetyl-CoAx l GTp +08
+ 129
H
Table B,l lH o
RAI-^ [- s-con
=".,19t. fgl process is about -2g40 Kcal/mol. The fraction of energy
T0c
:'''ailable (whichFis
is -2340 Kcal/mol) that is actually trapped as ATp is about 40% cds-Enoyl-CoA
i29 x 7.3 = 9 4'1 .7, and then g4]^.T l2g40 x 100'/,
= +OXS.'' n
Lnsaturated Fatty nliOs ll lro*rroro
)-rt.all fatty acids are completery saturated with hydrogen
atoms. some have
u
::uble bonds in them. These are-referred to as unsaiuratid yatty HO
acid.s. There are
:"'^'o type of situations concerning
double bonds in unsaturatei fatty acids. one
ii:uation is that you arrive at a double bond in the
n^tf H
[, s_coA

;'' :i-le the other situation


B,y-position after B-oxidation
is that you arrive at a double bond at the a,B_position lrazs-Enoyl-CoA
a:'ter B-oxidation.

-'he
I
li double bond is at the B,y-position, we need to convert
h: a,B-position. The enzyme tirat catalyzes this reaction is itantoisomerase.
a double bond at lJ
5r r\-vn in Figure 8-64. onq.q,you have the
This is B-oxidation
a,B-position established you are at the
k;el of enoyl-CoA in B-oxiaation. The enoyl-C^oA double bond Figure 8-64
is trans. If it were
'cs, it would need to be converted to trans Ly an isomerase.

:ryright @ by The Berkeley Review 221 The Berkeley Review


Specializing in MCAT preparation
Biology Metabolic Pathways Fatty Acid Oxidation

The Oxidation of Pelargonic Acid (a C11 odd-numbered fatty acid)


Pelargonic acid is a C-11 odd chained fatty acid. If we were to completely oxidize
pelargonic acid using the p-oxidation pathway, we would go through the cvcle
that we have been discussing 4 times to get 4 two-carbon acetyl CoA residues
and 1. three-carbon propionyl-CoA residue. This is shown in Figure 8-65.

cH 3-cH2-cH2- -cH2-cH2- l-CH2-CH2,|-CH2-CH2- -CH2_COO-


| |

#5 #4 #3 #2 #1

Figure 8,65

Propionyl-CoA Metabolism in Animals


In order to utilize propionyl-CoA we must carboxylate it to fora.
Methylmalonyl CoA. This reaction is catalyzed by propionyr CoA carbon
(a biotin enzyme). This enzyme is a ligase because ATp is simul
converted to AMP and PPi. Pyrophosphate is subsequent$ hydrolyzed:,r
molecules of orthophosphate. D-Methylmalonyl-CoAis converted to L-\fic
malonyl-CoA by a racemase enzyme (which is in the class of enzvmes refm
as isomerases). L-Methylmalonyl-CoA is next converted to succinyl-CoA rw
enzyme methylmalonyl-CoA mutase. This is a unique enzyme beca&
contains vitamin 812 @obalamin) as its coenzyme. [There is only one ;-ffi
reaction known in mammals dependent on vitamin B12 and that is the fonr:,mu
of methionine by methylation of a homocysteine residue.l once succinyl{cA
formed it can then enter into the Krebs cycle.

Can Fatty Acids be converted into Carbohydrates?


Would it be possible for animals to convert fatty acids into carbohydrates? tr*ul
know that when fatty acids are degraded by the $-oxidation pathway, a wealth n
2-carbon acetyl-CoA units are synthesized. These acetyl-CoA units will enter i:-ru
the Krebs cycle and condense with OAA to form citraie. F{owever, as citrate goem
around the cycle there are two decarboxylation rcactions, catalyzed by isocif,am
dehydrogenase and a-ketoglutarate dehydrogenase. Even though we will sts
form oAA as we continue around the cycle, we will have lost 2 carbons in tF.e
process, which means that we would be unable to convert fatty acids in:-r
carbohydrate material. Plants and many bacteria, though, can utilize acetyl-CoA
for energy production and biosyntheses by using a reaction sequence cailed tle
glyoxylate cycle. However, animals can degrade carbohydrates to acetyl-coA
units and then take those acetyl-CoA units to synthesize fatty acids.

Not only does the Krebs cycle function in the oxidative catabolism of aminLo
acids, fatty acids, and carbohydrates, but it also serves as a primary starting poir,l
in many biosynthetic reactions for which it is able to provide precursors. fJ tnese
precursors were removed from the Krebs cycle for various other metabohl
pathways, then the rate at which the Krebs cycle operated would begin to declinre
(and might even stall). One of the most importanf a naplerotic reacti6ns (from the
Greek, to "fill up") is the synthesis of oAA from pyruvate and Co2. This reacticni
is catalyzed by the enzyme pyruvate carboxylase. For example, if a cell r.t,a-.
exclusiaely catabolizing fatty acids to obtain a high level of ATp, and if there
o,,a-<
no carbohydrate catabolism taking place, then the anaplerotic reaction of
pyruvate to OAA wouldlot take place. The result is that the Krebs cycle woulo
niinilffifir
run down. In other words, fatty acid degradation needs the flickering flame ot
{i$rrrrrlm!

lalbohydrate degradationto keep it going (i.e., a little conversion of plruvate to


oAA is needed to ensure that fatty acid degradation will continue).

Copyright @ by The Berkeley Review 222 The Berkeley Review


Specializing in MCAT preparation
Biology Metabolic Pathways Urea Cycle

UrearrG$dt6
-: ive hydrolyze proteins, we will be able to obtain a variety of amino acids. The
':',iro acids that we obtain can then be used in a variety of reactions, one of
'nich is protein synthesis. However, for most amino acidj the amino group can
:= removed to yield the s-keto acids which can then be used in citric icid
cycle
-::ermediates to eventually give ATp, Co2 and H2o. This is shown in Figure g-
Let's focus on the Phase I and Phase II (we have already looked at phlase
"o
:,
III
lhe Krebs cycle).

H3\
Proteins cr-Keto Acids Krebs cycle
tT: ffi Intermediates
il
il il pnu,. rrr
u U
Protein Synthesis ATP, CO2, H2O

Figure 8-66

Phase I
-:. this phase of amino acid degradation (the major site in mammals being the
':t'er) we are essentially dealing with two reactions. The first is amino transfer
-:om an amino acid to c-ketoglutarate as shown in Figure g-67. (The portions
-:.at are in boxes will form the cr-keto acid.) o-ketollutarate is the icceptor
:-rolecule of the amino group from most amino acids. TLe enzyme that
catalyzes
ris reaction is an aminotransferase (also called a transaminase). There will be a
-:ecific aminotransferase for each amino acid.

- rr example, if you wanted to transfer the amino group from alanine to a-


:.etoglutarate, then you would use alanine aminotranslerase. The mechanism
of
-:-ese reactions involves the coenzyme pyridoxal phosphate (plp)
which is
rerived from vitamin B5 (pyridoxine).

T--;r
,lrllcoo
tvt \7

o Ic=o
I

oo
o 9oo
(ur H ilo I
PI,P ll ll ri
NH?- C- H
'l
NH3- C- c-o + CH,- Rr- C- C- O" + CH"
| Amino- l'
H 9n' .'
coo"
transferase a-Keto
Acid
CH.r-a
|

c-Amino COO,J
Acid cr-Ketoglutarate Glutamate

Figure 8-67

-he glutamate that isproduced in Figure g-67 gets oxidatively deaminated as


'hown in Figure 8-68: Glutamate dehydrog"ni"" catalyzes tiris reaction and
=rther NAD@ or NADP@ can be utilized. Note that nitrogen is released in the
-'orm of NH+€.

iopyright @ by The Berkeley Review 223 The Berkeley Review


Specializing in MCAT preparation
Biology Metabolic Pathways Urea Cycle

o o
@ too H+l
coo
./tr
NHl-
"l C- H C=O
CHT A NAD. t Lrzv =- L11, T NADH + NHa+
a"" 1or NADP+) Glutamate | - (or NADPH)
lO dehydrogenase ?", a
coo coo
Glutamate cr-Ketoglutarate

Figure 8-68

The sum of the reactions shown in Figure 8-67 and in Figure 8-68 can be seen in
Figure 8-59. In terrestrial vertebrates the excess NH+e produced in Figure 8-68 is
converted into urea and then excreted. In most aquatic organisms, NH4@ itself is
excreted, while in birds and terrestrial reptiles the NH4+ is converted to uric acid
before it is excreted.

+ H2O
g-Amino Acid + NAD+=- ct-Keto Acid + NADH + H+ + NH4+
(or NADP+) (or NADPH)

Figure 8-69

Phase II
Let's consider the degradation of the carbon skeleton of the amino acids.
that carbohydrate metabolism will eventually yield pyruvate which can enter
Krebs cycle through acetyl-CoA. we also mentioned that fatty acid degradatim
generates acetyl-CoA as well. When amino acids are degraded they will
metabolic intermediates which can funneled into the Krebs cycle.

Carbohydrate
Metabolism
\ Fatty f(rhG)

@,""1$Acetoacetyr
I tm
@*1"#{
@t", ,ti)ru*."t"
Knehs citrate
[wJ
\-_lspJ\uyanel.
@'*'r'-.****@
Figure B-7O

Copyright @ by The Berkeley Review 224 The Berkeley


Specializing in MCAT
Biology Metabolic Pathways Urea Cycle

Some of these amino acids will provide the carbon skeleton framework
for the
net synthesis of carbohydrate via gluconeogenesis. These amino acids
are termed
glucogenic and they will be degraded io pyruvate, o-ketoglutarate, succinyl-
CoA, fumarate, and oxaloacetate. rnt is shtwn in Figure "g-20. oth", amino
acids will be degraded to acetyl-coA and acetoacetyl-c11. These
amino acids
are termed ketogenic because they will eventually give rise to ketone
bodies.
Remember, mammals do not have a pathway *rat"witt allow acetyl-CoA
-co2 (or
acetoacetyl-CoA) to be converted to carbohydrate (due to the loss or
in the
Krebs cycle).

{s we have seen, amino groups will flow from the various amino acids to
glutamate and then to ammonii by the glutamate dehydrogenase reaction. The
free ammonia that we produced in the glutamate dehldrogenase reaction
combines with Co2 to form carbamoyl phosphate. This is r'r,o*.r" in Figure g-7L.
Thiq reaction is_ catalyzed by carbamoyl phosphate synthetase, requirei
2 ATps,
and occurs in the mitochondrial matrix. since carbamoyl phosphatels now
a high
energy compound, we can think of it as an activated carbamoyl donor.

oo
CO2 + NH4+ + Z ATp+ H2CE::+ urN- 3-o - i-o"
t'
+ 2 ADp + p; + 3 H+
oo
Carbamoyl Phosphate

Figure 8-7 I

ln the next reaction, carbamoyl phosphate reacts with a molecule of ornithine


(an
amino acid that does no-t appear in proteins). Carbamoyl transcarbamoylase
iransfers the carbamoyl group from carbamoyl phospr,ut" to ornithine to
produce citrulline as shown in Figure g-72. citrulline is another example
of an
amino acid that does not occur in proteins. Citrulline can leave the mitochondrial
matrix and enter into the cytoplasm.

NHq
t-
CHr Ornithine
t'
CHr
t- +.
f"'t
H-C-NHr
oo
Carbamoyl Phosphate
f"t.
H-c-NHl
l6-
looo coo"
Ornithine Citrulline

Figure B-72

-: we take citrulline and let it be aminated by aspartate via the enzyme argino-
.-:ccinate synthetase, we will form arginosuccinaie, which can be cleaved
:-rzyme arginosuccindse to give arginine
r! tire
and fumarate. This reaction is a way to
',nthesize arginine from ornithine. How do you get ornithine? If you hydroiyze
::ginine with water, you will get ornithine and urea. Urea can then be excreied.
lhe overall pathway for the urea cycle is shown in Figure g-73.
[once fumarate is
::nerated it can react with water and be converted to malate. Malate reacts with

,rpyright @ by The Berkeley Review 225 The Berkeley Review


Specializing in MCAT preparation
Biology Metabolic Pathways Urea Cycle

NADo and is converted to oxaloacetate. oxaloacetate can react with glutamate


to
give aspartate and o-ketoglutarate.]

2ATp 2ADp+p1
\/oo
NH+* + COF><:+ [- o- i- o
",*-
Carbamoyt 6
phosphate
synthetase

H H
+l +l
H3N- C- COo HrN-
"l C- COO
Ornithine
CH"
Ornithine ?"'
cH"
transcarbamovlas e r' Citrulline
t' CHr
t-
CH" CH"
rl t'
'NH.l H.N- C- NH.
I u'
o
H2N- C- NH2
Urea
[
ll .-
*Yc- CoO
H3N-
Arginase o":;:,';:;':""11(
i:l
coo-
+ll Aspartate
H H
+l +l
H?N-
-l C- COO HrN-
'l C- COO
CH" CHn
I
Arginosuccinase t'
CH, CHr
t' t-
CHr CH"
t- t'
H-N_ C- NH? H-N- C- NH.
il-
NHz coo u'
I N
+ _l
CH
Arginine il OOC- C- CH,_ COO
CH t'
I H
coo Arginosuccinate
Fumarate

Figure 8-73
The Urea Cycle

Copyright @ by The Berkeley Review 226 The Berkeley Review


Specializing in MCAT preparatim
Metabolic
Pathways

15 Passages

L00 Questions

Passage Titles Questions


L Glycolysis and Electron Transport Chain 1-5
II. Electron Transport Chain 6-11
ilI. Cholesterol Metabolism and Regulation 12-tl
IV. Very Low-Density Lipoproteins (VLDLs) 18-23
V. Calvin Cycle 24-29
VL Lactose Intolerance 30-36
VII. B-Oxidation 37-43
V[I. GlycogenMetabolism 44-50
IX. Glycolysisand2,3-Bisphosphoglycerate 51-58
X. Leucine Catabolism s9-65
XI. Trehalose Experiment 66-12
XII. Fuel Oxidation during Exercise 13 -19
X[I. Urea Cycle 80-87
XIV. Gluconeogenesis and the Cori Cycle 88-94
XV. Starch Blockers 95 - 100

R.E.tV,,I .b.W@
Speciahzing in MCAT Preparation
Suggestions
The passages that follow are designed to get you to think in a conceptual manner about the processes
of molecular biology at the organismal level. If you already have a solid foundation in molecular biology,
many of the questions you read here will seem to be very straight forward and easy to answer. But if you
are new to the subject or if you have not had a pleasant experience with molecular biology in the past,
some of them might appear to come from the void that spreads out beyond the Oort field at the edges of
our solar system.

Pick a few passage topics at random. For these initial few passages, do not worry about the time. Just
focus on what is expected of you. First, read the passage. Second, look at any diagrams, charts, or graphs
in it. Third, read each question and the accompanying answers carefully. Fourth, answer the questions
the best you can. Check the solutions and see how you did. \A/hether you got the answers right or wrong,
it is important to read the explanations and see if you understand (and agree with) what is being
explained. Keep a record of your results.

After you feel comfortable with the format of those initial few passages, pick another block of
Passages and try to do them in one sitting. Be aware that time is going to become important. On average,
you have about 1 minute and 15 seconds to complete a question. Be creative in how you approach this
next group. If you feel comfortable with the outline presented above, fine. If not, then try different
approaches to a passage. For example, you might feel well versed enough to read the questions first and
then try to answer some of them, without ever having read the passage. Maybe you can answer some of
the questions by just looking at the diagrams, charts, or graphs that are presented in a particular passage.
Remember, there are many effective leaming styles. You need to begin to develop a format that works
best for you. Keep a record of your results.

The last block of passages might contain at least a few topics that are unfamiliar even to those who
know a good deal about molecular biology. Find a place where the level of distraction is at a minimuru
Get out your watch and time yourself on these passages, either individually or as a group. It is important
to have a feel for time, and an awareness of how much is passing as you try to answer each question.
Never let a question get you flustered. If you cannot figure out what the answer is from information
given to you in the passage, or from your own knowledge base, dump it and move on to the next
" question. As you do this, make a note of that pesky question and come back to it when you have more
time. When you are finished, check your answers and make sure you understand the solutions. Be
inquisitive. If you do not know the answer to something, look it up. The solution tends to stay with you
longer that way. (For example, what is the Oort field, anyway?)

The estifrated score conversions for 100 questions are shown below. At best, these are rough
approximations and should be used only to give one a feel for which ballpark they are sitting in.

Section VIII
Estimated Score Conversions
Scaled Score Raw Score
>13 80- 100
lt-12 70-79
9- 10 60-69
7 -8 50-59
5-6 40-49
<4 0-39
Biotogy Glycolysis and Dlectron Ttansport Chain Passage I

Passage I (Questions l-5) 1. Louis Pasteur discovered that by depriving a culture


of yeast cells access to oxygen, their giowth rate
Glycolysis involves a series of reactions that converts de.creased by a factor of 6, while th6ir glucose
glucose into pyruvate. Under aerobic conditions pyruvate
utilization increased. Under aerobic conditions,
enters the citric acid cycle, where it is completely yeasts can synthesize 36 NTps per molecule of
oxidized to COz and HzO. During glycolysis anA tnl glucose oxidized. The rate of glucose consumption
citric acid cycle, high-energy phosphate bonds are under anaerobic conditions would increasi by
produced in the form of ATp and GTp (collectively approximately what factor?
known as nucleoside triphosphates or NTps) by a process
called s ubstrate -level p ho spho rylation. 4.2
8.3
The NADH and FADHz generated from glycolysis and c.4
the citric acid cycle transfer their electrons to the electron- D.6
transport chain in the inner mitochondrial membrane. As
the electrons flow down a series of electron carriers to
molecular oxygen, protons are pumped into the
intermembrane space from the matrix, eitablishing an 2. If oligomycin is added to a growing culture of yeast
electrochemical gradient that is used to do work. efp is cells, the culture will:
synthesized from ADP and pi as protons move through an
FoFlATPase located in the inner membrane. This process A. continue with aerobic respiration, synthesize
is called oxidative phospho rylation. 36 net NTps, produce COz and HzO, and
increase their growth rate.
Outer Membrane
Cytosol B. switch to anaerobic fermentation, synthesize 4
net NTPs, produce COz and CH:CHzOH, and
i?f,?tgtE4.:?i:?r?[gr,?f??3rf f ?r?f ?r,?f ?r?r?r?r,fi ??rE? increase their growth rate.
88888888Jd88J888888S888899e8 Jg888g8sig J8J8 J9868
Intermembrane
C. switch to anaerobic fermentation, synthesize 2
Space net NTPs, and produce COz and CH:CHzOH,
and decrease their growth rate.
D. continue with aerobic respiration, synthesize 2
net NTPs, produce COz and HzO, and
decrease their growth rate.
NADH NAD+ FADH2
FAD
+H+
Matrix ADP H+
ATF 3. Both
+Pi _oligomycin and 2,4-DNp are simultaneously
added to an aerobic cultureof growing yeast cells.
In this case:
The FoFrATPase can be inhibited by oligomycin,
rvhich binds specifically to rhe Fo portion of the complex.
A. oxidative phosphorylation is stimulated, a
When oligomycin is pfesent in a concentration sufficient proton gradient
ro allow binding to every one of the Fq subunits, ATp
will not be formed, electron
transport will increase, CHgCHzOH and COz
cannot be synthesized by oxidative phosphorylation. are generated, and 36 net NTps are made.
Under these circumstances, electron transpoit stops, the
electrochemical gradient increases, and the energy
B. oxidative phosphorylation is stimulated, a
proton gradient will be formed, electron
associated with the electron-transport chain becomes
rnsufficient to pump any more protons from the matrix
transport will increase, HzO and CO2 are
generated, and 2 net NTps are made.
into the intermembrane space.
C. oxidative phosphorylation is inhibited, a
Other chemical agents can interfere with the proton gradient will not be formed, electron
production of NTPs as well. For example, arsenic acid transport will increase, HzO and CO2 are
H:AsO+) has a chemistry similar to that of phosphorus generated, and 4 net NTps are made.
acid (H:PO+). The comlound 2,4-dinitrophenol (2,4_ D. oxidative phosphorylation is inhibited, a
DNP) uncouples electron transport from oxidative proton gradient will not be formed, electron
rhosphorylation by transporting hydrogen ions from the transport will increase, HzO and COz are
intermembrane space to the matrix of the mitochondrion, generated, and 2 net NTps are made.
lhus bypassing the F6F1-ATPase complex.

Copyright @ by The Berkeley Review 229 The Berkeley Review


Specializing in MCAT preparation
Biology Glycolysis and Dlectron Transport Chain Passage I

4. Step 6 in glycolysis is catalyzed by glyceraldehyde


3-phosphate dehydrogenase. If arsenate is supplied
in place of inorganic phosphate, the enzymatic
reaction proceeds quite well as shown below. This
linkage between the C-1 carbon and arsenate is
unstable and decomposes to 3-phosphoglycerate and
inorganic arsenate.

o.- -H
C
I
H- C*
I
OH Enzyme H- c- oH
HrC- O- POr2 H2C- O - PO12-

Glyceraldehyde 1-Arseno-3-
3-phosphate phosphoglycerate

Addition of arsenate to a culture of cells growing


anaerobically on glucose:

A. inhibits glycolysis at Step 6 and prevents


further growth of the cells.
B. does not inhibit glycolysis but prevents further
growth of the cells, because ADP cannot be
{fi
phosphorylated in Step 7.
C. inhibits glycolysis at Step 7 but prevents
I
I
further growth of the cells, because ATP r
cannot be produced in a net yield. if
D. does not inhibit glycolysis but induces the !
culture to switch from aerobic to anaerobic
metabolism. so they can continue to grow.

-t. In terms of its effect on glycolysis, arsenate is BEST


described as:

I. a competitive inhibitor.
II. a noncompetitive inhibitor
IIL an uncoupling agent.

A. I only
B. II only
C. I and III only
D. II and IiI qnly

Copyright @ by The Berkeley Review 230 The Berkeley


Specializing in MCAT Pre
Biology Dlectron-Transport Chain Passage II

t Outer
t Membrane

ot
o+
++++++ o
I Inner

I Membrane

Matrix
NADH
+H+ NAD+ ADP
Y
+Pi g+
Transport
Protein
ATPase
Figure I

Passage II (Questions 6-ll) The FAD-linked dehydrogenase at complex II


interacts with coenzyme Q in very much the same
Mitochondria are organelles that have two membranes fashion.
(Figure 1). The outer membrane contains proteins called
porins that permit the passage of molecules with Complex III contains a number of cytochromes, heme-
molecular weights of less than 10,000. The inner containing proteins involved in one-electron transfers, and
membrane is selectively permeable to different molecules an iron-sulfur protein. This complex, also called the
and is quite impermeable to polar and ionic substances. cytochrome bc t complex, passes the electrons to a small
The matrix is enclosed by the inner membrane and peripheral membrane protein called cytochrome c (Cyt c).
contains the enzymes necessary for the citric acid cycle,
fatty acid oxidation, and even amino acid oxidation. Cytochrome c diffuses through the membrane and
Between the two membranes is the intermembrane space. passes its electrons to complexIV, known as cytochrome
oxidase. These electrons are accepted by cytochromes a,
Most of the electrons that enter the electron-transport a3, and two copper ions before they are passed to
:hain come from the action of dehydrogenase enzymes. molecular oxygen. Electron flow through this complex
llany of the dehydrogenases are NADo-specific. These results in protons being pumped into the intermembrane
enzymes remove two hydrogen atoms from their space.
substrate; one in the form of a hydride ion (:He) and the
cther leaves as a proton (H@). Dehydrogenases that are Protons from the intermembrane space can return to
FAD-specific can also remove two hydrogen atoms. the matrix though the Fo transmembrane proton channel.
Attached to the Fo component on the matrix side is an
The electron-transport chain operates within the inner ATPase designated as Ft. As protons pass into the matrix
mitochondrial membrane and is composed of a series of through the FoFrATPase, ATP is synthesized from ADP
electron carriers, many of which are integral membrane and Pi.
proteins that bear prosthetic groups capable of carrying
-rut electron-transfer reactions. Inhibitors like dicyclohexylcarbodiimide (DCCD) and
oligomycin inhibit proton transport through F6. Amytal
The NADo-linked dehydrogenase at complex I not and rotenone inhibit electron transport from complex I to
rnly transfers hydride ions to coenzyme Q, but it can also coenzyme Q. Finally, electron transport and oxidative
pump protons into'the intermembrane space. As QHe phosphorylation can be uncoupled using hydrophobic
liffuses through the membrane to complex III, it picks up agents like 2,4-dinitrophenol (DNP), FCCP, and CCCP.
r proton from the matrix and forms QHz. Electrons are
lassed to complex III, and protons are pumped into the
.ntermembrane space.

Jopyright O by The Berkeley Review 231 The Berkeley Review


Specializing in MCAT Preparation
Biology Electron Transport Chain Passage II

6. Elecffons from pyruvate enter the electron-transport 10. The flow of electrons from succinate to coenzyme Q
chain at: occurs by way of the FADHz that is produced in the
oxidation of succinate to fumarate.
A. Complex I
B. Complex II
FADH2
C. Complex III or IV FAD
D. none of the above
Succinate \ '- Fumarate

7. The chemiosmotic hypothesis states that ATP is


synthesized in the mitochondrial matrix because of As electrons flow down the electron-transport chain,
the: free energy decreases. This decrease can be
quantified if we know the standard reduction
A. passage of electrons from NADH and FADH2 potentials (E6') of redox pairs and by using tbe
to molecular oxygen. equation
B. electrochemical gradient that forms across the
inner mitochondrial membrane. AGo,_ _ (n)(F)(AEo,)
C. increased permeability of ADP into the
mitochondrial matrix at a specific protein
antiport. where AGo'is the standard free-energy change, n is
the number of electrons transferred, F is the Farada5r
D. increased permeability of ADP into the
constant, and AEo' is the change in standarrd
mitochondrial matrix at a specific protein
symport.
reduction potential between the oxidized and
reduced species.

8. Synthetic phospholipid vesicles were constructed, The following information is given:


and purified Fq protein was incorporated into the
membrane. High levels of Ko were added to the
vesicles. A short time later, valinomycin was added
to the exterior medium. H@ translocation was not Fumarate + 2H@ + 2eo -----+ Succinate 0.03
observed in vesicles missing the Fo protein. After
valinomycin was added, however, He translocation
FAD + 2H@ + 2eo
-+ FADH2 - 0.22
occurred, because valinomycin was able to:
We can conclude that in the reaction involving
A. transport H@ from the exterior to the interior conversion of succinate to fumarate:
of the vesicle.
B. transport Ho from the interior to the exterior A. AGo'_ - (0.50)(F).
of the vesicle. B. AGo'- +(0.s0XF).
C. create a diffusion potential that allowed an C. AGo'- +(0.10)(F).
efflux of Ke and an influx of Ho. D. AGo'_ _ (0.IOXF).

D. create a diffusion potential that allowed an


influx of K@ and an efflux of H@. 11. In prokaryotic and eukaryotic cells, the e
transport chain and oxidative phosphorylation
9. Addition of DCCD to cells inhibits Ho transport coupled. Oxidative phosphorylation in pro
through the mitochondrial Fo protein. Addition of cells occurs in the:
2,4-DNP would be expected to:
A. matrix, while electron transport occurs on
A. decrease electron transport and decrease ATP inner mitochondrial membrane.
synthesis. B. intermembrane space, uhile electron tr
B. increase electron transport and increase ATP occurs on the inner mitochondrial
synthesis. r C. cytosol, while electron transport occurs oo
C. increase electron transport and decrease ATP outer membrane.
synthesis. D. cytosol, while electron transport occurs on
D. decrease electron transport and increase ATP plasma membrane.
synthesis.

Copyright @ by The Berkeley Review 232 The Berkeley


Specializing in MCAT
Biotogy Cholesterol Metabolism and Kegulation Passage III

Passage III (Questions 12-17) Most of the cholesterol synthesized in the liver is
exported in the form of a bile salt or a cholesteryl ester.
In 1912 Brown and Goldstein began to examine a Bile salts are stored in the gallbladder until they are
dominant human hereditary condition called familial needed in the small intestine. Cholesteryl esters are
hypercholesterolemia (FH). Their work on the transport synthesized from cholesterol and a fatty acid from
of exogenous cholesterol in the bloodstream and its coenzyme A through the action of acyl-CoA-cholesterol
receptor-mediated endocytosis and metabolism by target acyl transferase (ACAT). High intracellular levels of
tissues (Figure l) won them the Nobel Prize in 1985. cholesterol stimulate ACAT, promoting esterification of
cholesterol for storage.
Cholesteryl
esters lesterol Unlike bile salts, cholesterol and cholesteryl esters are
rather insoluble in water. These lipids are sequestered in a
micelle-like particle called a low-density lipoprotein
LDL (LDL) and transported from the liver in the bloodstream
receptors to various target tissues. LDL receptors on the surface of
/l
I

LD L Extracellular
Lipoprotein ( ,/l space
a target cell cluster into coated pits, and binding of LDL
to these receptors initiates endocytosis. The internalized
coated pit becomes an endocytotic vesicle. Several of
these vesicles fuse to form an endosome, which in turn
fuses with a lysosome. Enzymes within the lysosome
hydrolyze the cholesteryl esters and the protein portion of
\- the LDL particle, releasing cholesterol, fatty acids, and

\ ffi\ 'lj,"'3jfiil''
amino acids into the cytoplasm. The LDL receptors are

rffid5p recycled and move back to the plasma membrane to form

\ W.-ffi
another coated pit. High levels of intracellular cholesterol
inhibit LDL receptor synthesis.

\\ Endocytotic \ \
vesicle \ \
{-1>" I \

{-st\ A
&*t*d
qs4ffi1 #
tffi
[::"::i:>w \
12. Based on information in the passage, what is meant
by "exogenous cholesterol"?

A. Cholesterol in the blood that may enter the cell


Endosome I

B. Cholesterol inside the cell


/ I C. Cholesterol sequestered inside arterial plaques
Amino
-!-R-Zacids a
=-WA ,,^ I D. Cholesterol in the nucleus of the cell
,'-* / CO-

13. Brown and Goldstein studied people with FH. The


primary defect of FH is ineffective or missing LDL
receptors. How would FH affect the metabolism of
cholesterol?
Figure 1. LDL receptor-mediated endocytosis of
chlosterol and its metabolism. I. Increased intracellularcholesterol synthesis
il. Decreased activity of ACAT
Cholesterol biosynthesis begins with acetyl-CoA and III. Increased LDL cholesterol in the blood
Jccurs in four stages, primarily in the cytosol of
nepatocytes. The ra-te-limiting step in this pathway A. I only
rnvolves HMG-CoA reductase, an enzyme that can be B. II only
:ilosterically inhibited by high levels of intracellular C. I and III only
:holesterol, unidentified cholesterol derivatives, and D. I, II, and III only
hormonally regulated by glucagon (inactivation) and
insulin (activation).

Copyright @ by The Berkeley Review 235 The BerkeleY Review


Specializing in MCAT PreParation
Biology Cholesterol Metabolism and Regulation Passage III

14. In Canada, public health guidelines are given in SI


units. For cholesterol (MW - 386.64), the public
health guideline is plasma total cholesterol levels
below 5 mM. What is this in mg/dl, the units for
this guideline in the U.S.?

A.77.2mgldL
B. 193.3 mg/dl
C. 1933.0 mg/dl
D. 772,0 mg/dL

15. All of the following events happen after the


lysosome fuses with the endosome EXCEPT:

A. lipases degrade cholesteryl ester to free


cholesterol.
B. proteases degrade apolipoproteins to amino
acids.
C. LDL receptor synthesis increases.
D. cholesteryl ester is stored in droplets in
cytoplasm.

16. What would be the physical results of changing to a


no-cholesterol diet?

A. Intracellularcholesterol synthesisincreases
B. Intracellularcholesterolsynthesisdecreases
C. Extracellularcholesterolsynthesisincreases
D. Extracellular cholesterol synthesis decreases

17. Mevinolin is the active ingredient in a class of


cholesterol-lowering drugs. It works as a potent
competitive inhibitor of HMG-CoA reductase and
lowers plasma cholesterol levels. In the following
Lineweaver-Burk plot, which line BEST indicates
mevinolin's action on HMG-CoA reductase?

Without
mevinolin

A. Line A
B. Line B
C. Line C
D. Line D

Copyright @ by The Berkeley Review 234 The Berkeley Revier


Specializing in MCAT Preparatic
Biology Very Low-Density Lipoproteins (VLDLs) Passage IV

Passage IV (Questions 18-23) 18. From the plasma samples she draws, the researcher
conducting the three experiments described in the
Phospholipids, triacylglycerols, cholesterol, and passage must separate the VLDL from the other
cholesteryl esters are hydrophobic molecules that are lipoproteins. What is the BEST way to accomplish
essentially insoluble in water. In order to be transported in this?
the bloodstream, these lipids combine with carrier
molecules called apolipoproteins to form one of four A. Extraction with organic solvent
major classes of plasma lipoprotein particles. Different B. Centrifugation in a density gradient
combinations of lipid and apolipoproteins produce plasma C. Thin-layerchromatography
lipoproteins ofvarying densities and sizes (Table 1). D. Extraction with 5Vo NaCl solution

Density Diameter
Particle (g/ml) (nm)
High-density l0
1.13
lipoprotein (HDL)
Low-density 20 19. Based on the information in Table l, which are rhe
1.04
lipoprotein (LDL)
largest and MOST buoyant lipoproteins?
Very low-density
0.98 50
lipoprotein (VLDL) A. High-densitylipoproteins
Chylomicron 0.95 500 B. Chylomicrons
C. Low-densitylipoproteins
Table 1. Major classes of plasma lipoproteins. D. Very low-density lipoproteins

De novo lipogenesis (DNL) is the synthesis of fatty


acids from acetyl-CoA. DNL occurs in the cytosol of the
hepatocyte with the assistance of the enzyme complex
iatty-acid synthase. After DNL, fatty acids are esterified
to glycerol. The product, triglyeride, is packaged into
VLDLs and excreted from the liver. 20. Which lipoprotein contains dietary lipids?

A researcher uses intravenous l-13C acetate to label A. High-densitylipoproteins


rhe triglyceride that is synthesized de novo during the B. Low-densitylipoproteins
iollowing experiments. She then analyzes the C. Chylomicrons
.ncorporation of l-l3C acetate using gas chromatography- D. Very low-density lipoproteins
mass spectroscopy (GC/MS).

Etperiment I
Subjects fast from 3:00 pm until noon the following
day. Blood samples are taken every hour from 6:00 2I. Why does the researcher use 1-l3C acetate to label
am until noon. the VLDL triglycerides?

Experiment II I. The acetate is metabolized to acetyl-CoA.


II. 8 acetyl-CoAs are used to make I palmitate
Subjects fast from 8:00 pm until 6:00 am. A glucose molecule.
drink containing 25 grams of glucose is given once UI. l-l3C acetate contains a stable isotope that is
per hour from 6:00 am until noon, during which time safe for use in humans at high doses.
hourly blood samples are taken IV. l-13C acetate contains a radioactive isotope
that is safe for use in humans at high doses.
-.rperiment I ll '.
A. II and III only
Subjects fast from 8:00 pm until 6:00 am. A fructose B. I and IV only
drink containing 25 grams of fructose is given once C. I, II, and IV only
per hour from 6:00 am until noon, during which time D. I, II, and III only
hourly blood samples are taken.

ilopyright @ by The Berkeley Review 235 The Berkeley Review


Specializing in MCAT Preparation
Biology Very Low-Density Lipoproteins (VLDLs) Passage IV

22. Analysis on the GC/MS requires transesterification


of the triglycerides to form fatty acid methyl esters.
Which is the BEST reagent to use for this purpose?

A. Methanol + HCI
B. Methanol
C. Ethanol + HCI
D. Ethanol

23. Subjects participating in which experiment would


show the LEAST incorporation of l-l3C into VLDL
triglycerides?

A. Experiment I
B. Experiment II
C. Experiment III
D. Subjects in all three groups would show the
same degree of l-13C incorporation.

Copyright @ by The Berkeley Review 236 The Berkeley Revier


Specializing in MCAT Preparatia
Biology Calvin Qycte Passage V

Passage V (Questions 24-29) ATP and NADPH are generated as products of the
light reactions of photosynthesis. These iwo metabolites
Within the chloroplasts of green plants, COz can be are then used to convert COz and HzO into carbohydrates
fixed in the form of simple organic compounds. The during the light-independent or dark reactions of
synthesis of these compounds takes place in a cyclic photosynthesis. These dark reactions make up the Calvin
pathway called the Calvin cycte (Figure l), in which cycle. All of the reactions of the Calvin cycle, except
specific metabolites (Table l) are continually regenerated. those catalyzed by enzymes Er, Eto, and btr, can be
found in animal tissue. Many of the reactions are common
to both glycolysis and the pentose phosphate pathway.

Key intermediates in the Calvin cycle, like 3-pG and


Ru-1,5-BP (Figure 2), were identified as rhe resulr of
experiments using green algae and radioactively labeled
carbon dioxide ( l4CO2).

Hrc- o- po.}

^o
u-t^-0
c-- o
I
L
I
H- C- OH
I
H_C-OH H* C- OH
I I
HrC- O - POr2 HrC- O- POr2'
Ru-1,5-BP
| 3-PG Ru-1,5-BP
Etl{-CO"
+ n,o
Figure 2. Calvin cycle intermediates.
I
3-PG
{P Experiment I
lr
tE" Step A:
Ni
Y
anp A mixture of green algae is exposed to COz and light for
DHAP an extended period of time.
1,3-BPG
\
p+\\ NADPH
Step B:
The light source is removed, and 14COz is added to the
\\ mixture.
G-3-P NADP@
+Ho Step C:
Metabolites are analyzed for l4C labeling.
IrluT. l.The Calvin cycle. These reactions fix atmospheric
CO2 in the lorm of carbohydrates.
Experiment II
Step A:
Table l.
A mixture of green algae is exposed to tacoz and light
for an extended period of time.
Abbreviation CompoundName
Step B:
Ru-1,5-BP Ribulose- 1,5-bisphosphate The light source is removed, and COz is added to the
3-PG 3-Phosphoglycerate mixture.
1,3-BPG 1,3-Bisphosphoglycerate
G.3-P Step C:
Glyceraldehyde-3-phosphare
Metabolites are analyzed for l4C labeling.
DHAP Dihydroxyacetone phosphate
F-1,6-8P Fructose- 1,6-bisphosphate
F-6.P Fructose-6-phosphate
Experiment III
G.6-P Glucose-6-phosphate Step A:
Xu-5-P Xylulose-5-phosphate A mixture of green algae is exposed to l4CO2 in darkness
E-4-P Erythrose-4-phosphate for an extended period of time.
s- 1,7-BP . Sedoheptulose-
1,7-bi sphosphate Step B:
S.7-P Sedoheptulose-7-phosphate After all forms of COz are removed from contact with the
R.5-P Ribose-5-phosphate mixture, a light source is introduced.
Ru-5-P Ribulose-5-phosphare
Step C:
Metabolites are analyzed for 14C labeling.

Jopyright @ by The Berkeley Review 237 The Berkeley Review


Specializing in MCAT preparation
Biology Calvin Cycle Passage V

24. The first five steps in the Calvin cycle are catalyzed 28. Based on the information in Experiment I, which of
by enzymes Et,E2, E:, E+, and Es (Figure l). These the following graphs BEST represents the levels of
reactions correspond to: l4C in Ru-1,5-Bp and 3-pG?

A. carboxylation, phosphorylation, reductive A. B.


dephosphorylation, enolization, and
condensation. U O
.+
'+
Ru-1,5-BP
B. hydrolysis, phosphorylation, reductive
dephosphorylation, isomerization, and
condensation. o
€ (6

C. carboxylation, phosphorylation, oxidative Ru-1,5-BP


dephosphorylation, enolizationn and & &
condensation. Time Time
D. hydrolysis, phosphorylation, oxidative
dephosphorylation, isomerization, and C. D.
condensation.
tU +O
25. Based on the metabolic pathway in Figure l, what is >r >r
the overall balanced reaction for the Calvin cycle?
o
o
A. COz + 2ATP + 4HzO + NADPH + H+ -+ (! €
& Ru-1,5-BP
Glucose + 4Pi + 2ADP + NADP|
Time
B. 6COz+ 18ATP + I2HzO + 12NADPH + 12H+ Time
-+ Glucose + l8Pi + 18ADP + l2NADp+
C. 6COz+ I2ATP + 6Hz0 + I2NADPH + l2H+
-+ Glucose + l2Pi + 12ADP + I2NADP+
D. 29. Based on the information in Experiment III,
COz + ATP + HzO + NADPH + H+ -+
of the following graphs BEST represenrs the
Glucose+Pi+ADP+NADp+
of raC in Ru- 1.5-BP and 3-PG?

26. After 3 turns of the Calvin cycle, 6 molecules of A. B.


glyceraldehyde-3-phosphate are formed. These 6
U O
"molecules will be converted into: .+
Ru-1,5-BP
>'
A. 3 molecules of Ru-1,5-BP and I molecule of
c-3-P. O
(! o
B. 3 molecules of Ru-1,5-BP and I molecule of
glucose. - 6 €
& Ru-1,5-BP
C. I molecule of Xu-S-P, I molecule of E-4-P, &
and I molecule of S-1,7-BP. Time
.D. I molecule of F-1,6-BP and 2 molecules of
- glucose. c. D.

sU $
O
27. If radioactively labeled carbon dioxide (l4COz) is
used as a substrate for the Calvin cycle, it will >a

appear in the carboxyl group of 3-phosphoglycerate.


A short time later, the label will be found in glucose- o
c!
()
(g
o
6-phosphate at carbon atoms: !
& &
A. C-l and C-d. Time
B. C-l and C-3.
C. C-2 and C-5.
D. C-3 and C-4.

Copyright @ by The Berkeley Review 234 The Berkeley


Specializing in MCAT
Biology Lactose Intolerance Passage VI

Passage VI (Questions 30-36) 3f. Which biochemical reaction during fermenration


changes milk so that yogurt and cheese made from it
Human beings are the only animals that are known to can be consumed without complication by some
drink the milk of other species and to continue to drink lactose- intolerant individuals ?
milk after weaning. The majority of the adult population
of the world cannot digest lactose, the sugar in milk. A. The lactic acid produced during fermentation
Almost all human infants have sufficient lactase activity breaks down lactose.
to break lactose down into glucose and galactose. But B. The alcohol produced during fermentation
with age, the activity of lactase generally declines, except breaks down lactose.
in some people of northern European origin and certain C. Microorganisms produce disaccharides during
groups of Africans, such as the Tutsi and the Felani. fermentation.
D. Microorganisms break down disaccharides
The symptoms of lactose intolerance are unpleasant during fermentation.
but not often fatal, unless milk is the only available food.
After the ingestion of 30-50 grams of lactose as a test
dose, the lactose-intolerant person usually experiences
diarrhea, abdominal cramps, and intestinal gas. Even if
fluid milk is not tolerated, some lactose-intolerant people
may still be able to consume fermented milk products,
such as yogurt or cheese, without complications. 32. What is the cause of the intestinal symptoms
associated with the ingestion of fluid milk by a
lactose-intolerant person?
30. Which of the following diagrams represents the
chemical structure of lactose (galactose-p(1-+4)- I. Bacteria in the colon ferment the lactose to
glucose)?
produce gases.
A. il. Colon contents are hypertonic to surrounding
cH2oH cH20H
cells, and water enters the colon by osmosis.
UI. Bacteria in the colon produce irritating acids
from the lactose.

A. I only
B. I and II only
C. II and III only
D. I, II, and III
B.

33. Lactase isolated from the mold Aspergillus niger is


C. available commercially for lactose-intolerant people
to add to milk products before ingestion. How does
this commercial lactase work?

A. It is carried to the small intestine, where it


functions exactly like the missing human
lactase.
B. It breaks down the lactose in the food and is
inactivated in the stomach.
D.
c. It induces the production of lactase by the
intestinal mucosa cells.
D. Mold is cultivated in the milk, and it digests
the lactose.

Jopyright @ by The Berkeley Review 269 The Berkeley Review


Specializing in MCAT Preparation
Biology Lactose Intolerance Passage VI

34. After the ingestion by a test subject of a dose of


lactose to check for lactase deficiency, which of
these physiological variables should the clinician
measure?

L Breath hydrogen
il. Blood glucose
III. Blood lactose

A. I only
B. I and II only
C. II and III only
D. I. II. and III

35. Where in the human body is lactose synthesized?

A. The thymus
B. The small intestine
C. The mammary gland
D. The adrenal gland

36. Where is the enzyme lactase secreted during


digestion, in lactose-tolerant individuals?

A. The mouth
B. The stomach
C. The small intestine
D. The colon

Copyright @ by The Berkeley Review 240 The Berkeley


Specializing in MCAT
Biology B-Oxidation Passage VII

Passage VII (Questions 37 - 43) During the early 1950s another German chemist,
Feodor Lynen, discovered that one of the end products of
o
B-oxidation was not acetate, but rather acetyl coenzyme
il A. Working on a theory proposed by an American
R-CH,- C- oH + ATP + CoA-SH biochemist, Albert Lehninger, that fatty acids are
Catalyzed by I somehow activated before they are degraded, Lynen and
enzymes I Cytosolic ll his colleagues discovered that coenzyme A (CoA-SH) is
present in J Fauv Acid ll
esterified to the carboxyl group of a fatty acid in an ATP-
theouter
mitochondrial
I
I
oll
il
dependent enzymatic reaction.
membrane
t R-CH)-C-S-CoA + AMP + PP1 Activation of long-chain fatty acids occurs in the
Fatty acyl-CoA cytosol of a cell, and they are transported across the
Camitine mitochondrial inner membrane and into the mitochondrial
transports
the fatty acid
u..oi, th" I
Inner
Membrane
inner membrane\ V7777VA Fatty acyl-Camitine
e;::"Wffi matrix by a shuttle system involving L-carnitine. Short-
chain fatty acids can cross the inner mitochondrial
membrane as free fatty acids. Once they are in the matrix,
they are activated. These fatty acids can be obtained in
and into the MATRI' their free form from the blood or released from cellular
matrix of the
mitochondrion .^:z:Y triacylglycerols by the enzymatic action of lipase.

Oxidation of a fatty acid occurs exclusively in the


R-CH2-CH2-CH2-C - S - CoA mitochondrial matrix. This is in contrast to fatty-acid
FAD synthesis, which occurs exclusively in the cytosol of a
Fatty acyl-CoA \ 7
\ \\__, FADH2 cell. In other words, fatty-acid catabolism and fatty-acid
Ho anabolism occur in different compartments of the cell.
\ lll Even though fatty-acid synthesis utilizes acetyl-CoA as a
R-CH2-C=C-C-S-CoA precursor molecule, the reaction mechanisms are more
H complex and are not a direct reversal of B-oxidation.

HOHO 37. Once a fatty acyl-CoA molecule completes the four


ttll sequential reactions in B-oxidation, it produces an
)< R-cH2- c- c-c - t- .it
. NAD@ acetyl-CoA molecule and a fatty acyl-CoA molecule
co- H H which has been shortened by two carbon atoms. The
\*oo" reactions sequences of p-oxidation is BEST described
\
R-CHr-
ffrff
C- C-C- S- CoA
in terms of:

A. oxidation; hydrolysis; oxidation; thiolysis.


CoA-SH / H B. oxidation; reduction; dehydrogenation; hydrolysis.
C. dehydrogenation; hydration; oxidation; esterification.
\/
ff / il
R-CHr-C-S-CoA + CHr-C-S-CoA
D. oxidation; hydration; reduction; cleavage.

38. Consider caprylic acid, a C8 saturated fatty acid


Fatty acyl-CoA AcetYl-CoA
(2 carbons shorter) found in small amounts in butter:
o
At the turn of the 20th century (circa 1904), the
CH:-CH:-CHu-CH1CH2-CH2-CH2 - ! -on
German chemist Franz Knoop began to feed animals fatty
acids with a phenyl group (C6H5) attached to the last Caprylic acid
;arbon atom in the fatty acid chain, the omega (to) carbon.
.\nalysis of the urine of these animals led him to conclude Complete oxidation of one molecule of this short-
that fatty acids are degraded in two-carbon fragments by a chained fatty acid to CO2 and HzO gives a net yield
process that involves ckidation at the p-carbon to give a of:
3-keto acid. This is followed by cleavage of the p-keto
acid to give two products: acetate and a fatty acid A. 63 ATPs.
derivative (which is two carbon atoms shorter than the p- B. 62ATPs.
lieto acid). The pathway for fatty-acid degradation C. 61 ATPs.
became known as B-oxidation. D. 60 ATPs.

Copyright @ by The Berkeley Review 241 The BerkeleY Review


Specializing in MCAT Preparation
Biology B-Oxidation Passage Vtr

39. If we were to label caprylic acid with a phenyl ring at 42. Morc energy is available from the oxidation of fatn
the C-8 position, then which of the following acids than from the oxidation of carbohydratei.
chemical compounds would be present in the highest because:
concentration in the urine of a rabbit fed with this
modified fatty acid? I. fatty acids are stored in an anhydrous form.
whereas carbohydrates are not.
A. B. il. the hydroxyl hydrogens in a carbohydrate are
o released as bare protons with no electrons.
ll o IIII. f atty acids are more reduced thal
c-o o carbohydrates.
I\_./\n'J-." A. I only
B. II and III only
c. I and III only
C. D. D. I, II, and III :

o t
ll o
c-o 43. One of the three major biological roles for fatty
il

acrJs
_) is that they can be stored as triacylglycerols (neura.
I fats) and used as food molecules. The genera
/r\
$
structure for a triacylglycerol is shown below. The R. m

\J group represents the hydrocarbon chain of the samr


fatty acid:
ll
il
ri0

fi lm
40. Suppose we were to label the C- l, C-3, C-4, C-6 and H2C- O- C- R ,il!

C-7 carbon atoms of caprylic acid radioactively with


l4C, as shown below (where an * 14C):
Io
Irr
fl
= fi
H-C_ O_ C- R

** * * ff
lu
Itr
@

cHr-cH2-cH2-cH2-CH2-CH2-CH2 - C- OH H,C- O- C- R
*
Caprylic acid Triacylglycerol

If this molecule were completely metabolized, then in The melting points of four common Cra fatty
which of the following molecules would the label be are:
located predominately?
Stearic acid (+69.6 oC)
Oleic acid (+13.4 oC)
A. B. Linoleic acid (-5 oC)
o Linolenic acid (- I I oC)
il
o=c=o CH, - C- S-CoA
* * A decrease in the melting temperature (Tp) of
Crs fatty acids would indicate;
C. D.
o o I. an increase in fluidity.
il il II. a decrease in unsaturation.
CHr - C- S-CoA cHr - C-
* t< *
S-CoA
III. a decrease in the number of van der \l
interactions.

4L. A rabbit completely oxidizes I mole of caprylic acid


to COz and H2O. The net yield of water produced is: A. I only
B. I and II only
A. greater than 0, but less than 20 moles. C. II and III only
B. greater than 20, but less than 40 moles. D. I and III only
C. greater than 40, but less than 60 moles.
D. greater than 60, but less than 80 moles.

Copyright @ by The Berkeley Review 242 The Berkeley


Specializing in MCAT
Biology Glycogen Metabolism Passage VIII

Passage VIII (Questions 44-50) 45. Based on Figure I, which of the following
statements is FALSE?
Human beings store excess energy in glycogen or in
triglyceride. Glycogen in muscle and liver accounts for A. Carbohydrate oxidation increases with
about 1200 kcal, while energy stores in triglyceride are carbohydrate overfeeding.
virtually limitless. Glycogen is stored in the liver at 44 B. Carbohydrate is used to fill glycogen stores
g/kg liver tissue and in the skeletal muscle at l4glkg beforc de novo lipogenesis begins.
muscle tissue. The human liver weighs about 1.8 kg. This C. Glycogen stores cannot be saturated.
organ metabolizes excess dietary carbohydrate into D. Net de novolipogenesis (DNL) is used in part
triglyceride and releases it in the form of very low-density to dispose of excess carbohydrate consumed
lipoproteins (VLDL). during Days 5-10.

Swiss researchers studied the disposal of excess


dietary glucose following a glycogen-depleting fast. 46. What changes in VLDL should be seen in subjects
Three male subiects were fed a l07o carbohydrate, 75%o given a massive overfeeding of carbohydrate for one
iat, and l57o protein weight-loss diet for 3 days. Their week?
glycogen stores were also depleted by exercise.
A. Increased VLDL production
Sometime late on Day 2 of this dietary experiment, the B. Decreased VLDL production
sub.jects entered a respiratory chamber, a room that C. No change in VLDL production
permits the researchers to sample all the air exhaled by D. Complete inhibition of VLDL production
rhe test subjects. The subjects stayed in the chamber for
ren full days. Beginning on Day 4, they were switched to
: diet consisting of 85Vo carbohydrate,3To fat, and ll7o 47. The respiratory quotient is defined as the ratio of
protein for one week. This diet increased in calories from COz to Oz in expired air. If glucose (CoHtzOe) were
in excess of 1000 kcal to an excess of 2000 kcal. The the sole fuel for these test subjects, and if it were
:ollowing diagram (Figure 1) shows daily carbohydrate oxidized completely to COz and HzO, what would
.ntake and its disposal during the 7 days of progressive the respiratory quotient be?
; arbohydrate overfeeding :

A. 0.70
B. 0.82
c. 0.95
D. 1.00

800
O
48. Shown below is the structure of linked glycosyl
E 600 residues in glycogen. What are the linkages at I and
O T?

E +oo

u 2oo

oI
I

67 8 cH2oH
Days

Figure 1

l{. Which tissue in the human body has the largest net
supply of glycogen?

A. Liver A. is a-1,6, and II is p-1,4.


B. Kidney B. is s 1,4, and II is p-1,6.
C. Muscle C. is o-1,4, and II is a,-1,4.
D. Heart D. is a-1.6. and II is cx-1,4.

- rpyright @ by The Berkeley Review 243 The Berkeley Review


Specializing in MCAT Preparation
Biology Glycogen Metabolism Passage VItr

49. In an experiment with rats, a researcher labels the


glycogen stored in the liver. Animals are first fasted
and then refed with a labeled precursor to glycogen.
Which of the following tracers would NOT
effectively label the glycogen?

A. 1-l4C palmitate
B. l-l3C glucose
C. l-l4C alanine
D. l-l3C pyruvate

50. The experiment described in the passage is an


extreme version of carbohydrate loading. Athletes
also carbo-load by eating low-carbohydiate foods
and exercising followed by a high carbohydrate feast
right before an event. What benefits might carbo_
loading possibly have?

A. Increases glycogen stores to boost endurance


B. Increases fat stores to boost endurance
c. Decreases glycogen stores to boost endurance
D. Decreases fat stores to decrease endurance

Copyright @ by The Berkeley Review 244


Biology Glycolysis and 2, 3 -Bisphosphoglycerate Passage IX

Passage IX (Questions 51-58) Under aerobic conditions in animal cells, pyruvate


diffuses from the cytosol to the mitochondrial matrix,
Glycolysis involves ten enzymatic steps, during which where it is converted to acetyl-CoA before entering the
slucose is converted to pyruvate. In animals, this is the citric acid cycle. Under anaerobic conditions, pyruvate is
cnly metabolic pathway that produces ATP in the absence converted into lactate in animal cells, while in yeasts it is
of oxygen. converted into ethanol and carbon dioxide.

H .O
^
Ll
\at Nine of the intermediates in glycolysis are
,l
C H-C=O phosphorylated to prevent them from leaving the cytosol.
ADP t
c2 H- C- OH H- C- OH This ensures that during specific reactions, phosphate
rl I
cr Ho- c- H HO- C- H groups are donated to ADP to make ATP.
.t I
c4 H- c- oH H- C- OH
'l I
c5 H- C- OH H- C- OH
'l I
c6 cH2-oH cHr-o-Por2'
Glucose
Glucose
6-Phosphate

st"e,
ll
cHr-o-Porl CH,-OH
t- 51. In prokaryotic cells, glycolysis occurs in rhe:
c=o ADp H-c=o
i- ri t ATP A.
I
mitochondrialmatrix.
"o-
l- o" ":::-:" B. cytoplasm.
"- Step 5 I C. lumen of the Golgi complex.
H-C-OH
I
H- C-
I
OH D. lumen of the smooth endoplasmic reticulum
cHr-o-Po.2- cHr-o-Por2-
Fructose Fructose
1,6-Bisphosphate 6-Phosphate
NADH
o Pr* 52, Lactic acid is the end producr of glycolysis in:
"r.o ll ' +H*
g NAD+ r
o.ttc'\)c o.. ,o - po,2'
A. red blood cells.
r \t '-
B. brain tissue cells.
c=o H- C- OH step 6 H- C- OH C. adipose tissue cells.
I

cH?-oH cHr-o-Por2- cHr-o-Por2- D. heart and muscle tissue cells.


Dihydroxy- Glyceraldehyde 1,3-Bisphospho-
acetone 3-Phosphate glycerate
Phosphate
11 ,. ADP
step z l[ 53. Glycolytic enzymes can catalyze all of the following
o
ll\ erp reactions in glycolysis EXCEPT:
oat ooo
\\,
C 'o C A. phosphorylations.
.l I
H- L- r:r-t{-r-- -------------'-' H_ C- OH B. isomerizations.
!",_o" step a I
cH2-o-Po32'
C. enolizations.
D. ligations.
2-Phosphoglycerate 3-Phosphoglycerate

step e 1\'",o
nr\/ll.,' ATP o.t ,o
C C
I
54. The net oxidation state ofcarbon in glucose is:
I
c- o-Po12' c=o
il- Step ro I
CH:
A. -1
cHz
8.0
Phosphoenolpyruvate Pyruvate C. +l
D. +2

Copyright @ by The Berkeley Review 245 The Berkeley Keview


Specializing in MCAT Preparation
Biology Glycolysis and 2,S.Bisphosphoglycerate Passage D(

55. In red blood cells, the glycolytic intermediate 1,3- 57, Pyruvate-kinase deficiency is inherited as a rare
bisphosphoglycerate (1,3-BpG) can be converted to
2,3-bisphosphoglycerare (2,3-BpG) by the action of
autosomal recessive trait. If
one parent has thc
disease and the other parent is heterozygous for the
a mutase enzyme. disease, what is the probability that one of their
children will show a deficiency in pyruvate kinase?
o
o.s/ o - Po,l o.\ ,o
C
r
H-C-OH .+
Mutase f A. jVo
I
H - C- OPOra B. 25Vo
cH.-o-Por2- cHr-o-po.2' C. 50Vo
1,3-Bisphospho- 2,3-Bisphospho-
D. 757o
glycerate glycerate

This bifunctional enzyme can also convert 2,3-BpG 58. Based on the glycolytic pathway, which of
to 3-phosphoglycerate and Pi, metabolites used in following structures is formed after a series
glycolysis. How many net ATps are produced enzymatic reactions that includes phosphorylati
during glycolysis if this 2,3-BpG shunt is operating dephosphorylation and reduction, a
in red blood cells? isomerization?

A.0
8.2 A. B.
c.4
D.6 oat
C 'o cH,-o-PO,r'
I t-
u - c-
t"
o-po.l c=o
I
cH2-oH cH2-oH
56. Pyruvate kinase is an important regulatory enzyme
in glycolysis, and it catalyzes the conversion of pEp
to pyruvate. Which of the following oxygen- C. D.
hemoglobin dissociation curves (dashed line) BEST
represents an individual with a mild deficiency of
o.a,H o.
\}/ oo
C C
this enzyme? I I
H- C- OH H_ C- OH
I I
cHr-o-POr2' cHr-o-Po.2-
A. B.
d
o (,N
3
o
=
d
U) rn

Poz poz

D.

N
o

a
poz Poz

Copyright @ by The Berkeley Review 246 The Berkeley


Specializing in MCAT
Biology Leucine Catabolism Practice Passage X

Passage X (Questions 59-65) 59. Which of the following types of reactions is catalyzed
by Enzyme l?
In animals, amino acids can undergo oxidative
degradation. Leucine, an essential ketogenic amino acid, A. Hydration
can be degraded to acetyl-CoA and acetoacetate as B. Transamination
outlined in Figure l. Each reaction is catalyzed by a C. Carboxylation
specific enzyme and involves either an important cofactor D. Dehydrogenation
or coenzyme. Many of these reactions are analogous to
reactions found in other metabolic pathways.

60. In human beings, the ultimate fate of the c-amino


o o group in leucine is BEST represented by which of the
coo
o foo I
following metabolic structures?
HlN- C- H o=c
I

CH, CHr
t- l- A. B.
HrC- CH Enzyme I H1C-
-l CH H^
l(g|
CHr CHr H_ N- H
o
I
il
Leucine o-Ketoisocaproate H H2N - C- NH2
@ Ammonium ion Urea
NADH + H.,\lCoA-SH -. I

^
OA I Enzyme 2 c. D.
NAD, CO2 --l+ o
coo
S-CoA S-CoA o @t
'
o=f oo?"t ,.oo o= ! il H1N-C-
'I H

C- H
\ ,/ |
HN
.c H
N
CH.
CH. I c=o t"
CHt
s,C-
| Enzyme 3 u,c- cn otrc- N N
H
lO
H coo
CH: CHI
Uric acid Glutamate
Isovaleryl-CoA
B-Methylcrotonyl-CoA
I cotactorA I -.,1
\t/ 7 ucol
Enzyme a )( tniotin- Dependenr)

El-i-rc]
L--J :
-/l\

S-coA
61. The carboxylation reaction catalyzed by Enzyme 4
a biotin-dependent carboxylase, which also requires
is

o=i
S-CoA
nro
( o=i
cofactor A. Cofactor A is:

A.
nrc-
,9=" llnzYme
?*'
5 H,c- c- oH B.
HzO.
ATP.
f C. NADH.
CH, CH,
| '6
coo -- l-9
coo
D. FADH2.

B-Methylglutaconyl-CoA p-Hydroxy-
B-methylglutaryl-CoA

/ 62. In the reaction catalyzed by Enzyme 4, the


,/ Enzyme 6
bicarbonate ion (HCO3e ) is converted into
/ Compound C. Compound C is:
o
o 3
ooc- cH. - 'llc- cHl + HrC- C- S-CoA A. H2CO3.
Acetoacetate Acetyl-CoA
B. NH:.
C. HzO.

Figure I D. COz.

- :pyright @ by The Berkeley Review 247 The Berkeley Review


Specializing in MCAT Preparation
Biology Leucine Qatabolism Practice Passage X

63. The enzyme holocarboxylase synthetase appears to


be responsible for attaching biotin to at leist four
different apocarboxylase enzymes in mammals.
Biotin is involved in specific carboxylation reactions.
In the absence of the holoenzyme, which substrate
begins to accumulate most rapidly?

A. Aceryl-CoA
B. o-Ketoisocaproate
C. B-Methylcrotonyl-CoA
D. p-Methylglutaconyl-CoA

64. Which of the following types of reactions is catalyzed


by Enzyme 6?

A. Thiolysis
B. Aldol condensation
C. Reverse aldol condensation
D. Hydrolysis

65. If leucine were radioactively labeled with 14C at the


d-carbon atom, we would expect to find that label
entering cellular respiration at which of the following
stages?

A. Glycolysis
B. Krebs cycle
C. Electron transport
D. Oxidative phosphorylation

Copyright @ by The Berkeley Review 24a The Berkeley


Specializing in MCAT
Biology Trehalose Dxperiment Passage Xl

Passage XI (Questions 66 - 72) The following graph indicates the resulrs of a pilot
study performed with four test subjects using the
Trehalose is a naturally occurring sugar with the experimental protocol just described:
following structure:

cH?oH
Legend
n 0 g fehalose
tr 20 g trehalose
ffi 30 g trehalose
I 40 g trehalose

Figure 1. Trehalose
I 50 g trehalose
tr

Trehalase is an enzyme present in the microvilli on the


*30
surface of cells in the duodenum, the first twelve inches
o
-rf the small intestine. This enzyme hydrolyzes trehalose, bo CJ

r sugar found in certain beetle larvae and in mushrooms, 820 o


o
-nto two monosaccharides. Researcher are studying :E
ruman tolerance for different amounts of the sugar, E
recause it is of interest to people in the food processing
ndustry as a possible food preservative. o
-o
The following experiment was devised to learn more 3
,bout individual tolerances for trehalose. Each subject
.,, as given five incremental doses
of trehalose, or fewer if
iey felt gastrointestinal distress and had to stop the
:rperiment. Gastrointestinal distress was defined for this Subject 1 Subject 2 Subjecr 3 Subject 4
:rperiment as diarrhea, bloating, or gas pains. Breath
,:mples were collected in leak-proof syringes and Figure 2. Effects of trehalose ingestion on concentration
,jected directly into a gas chromatograph for analysis of of hydrogen in the brearh.
- i drogen concentration.

Part l: Subject drinks 200 mL of distilled water. 66. What process causes the increase in breath hydrogen
Breath hydrogen is collected and analyzed when a trehalose dose is consumed?
after 2 hours.
A. Digestion of the trehalose in the small intestine
PartZ: Subject drinks 200 ml of distilled water leads to the production of hydrogen gas in the
containing 20 g of trehalose. Breath intestinal tract.
hydrogen-is collected and analyzed after 2 B. Bacteria in the colon ferment the trehalose and
hours. produce hydrogen.
C. The interaction between bicarbonate from the
Part 3: Subject drinks 200 ml of distilled warer pancreas and trehalose leads to the production
containing 30 g of trehalose. Breath of hydrogen.
hydrogen is collected and analyzed after 2 D. Hydrogen is produced when trehalose is
hours. denatured in the stomach's acid.

Part 4: Subject drinks 200 ml of disrilled water


containing 40 g of trehalose. Breath
hydrogen is collected and analyzed after 2 67. Which subject exhibited a dose,response effect
hours. during all five parts of the experiment?

Part 5: Subject drinks 200 ml of disrilled warer A. Subject I


containing 50 g of trehalose. Breath B. Subject 2
hydrogen is collected and analyzed after 2 C. Subject 3
hours. D. Subject 4

:;'right @ by The Berkeley Review 249 The Berkeley Review


Specializing in MCAT Preparation
Biology Tfehalose Experiment Passage il

68. Which two monosaccharides does the enzyme 72. Which subject exhibited a threshold effect during tbe
trehalase produce when it hydrolyzes trehalose? experiment? (Exhibiting a threshold effect means
that after a certain dose, the subject's responsa
A. Glucose and galactose changes dramatically.)
B. Glucose and fructose
C. Galactose and fructose A. Subject I
D. Glucose and glucose B. Subject 2
C. Subject 3
D. None of the subjects experienced a threshof,d
effect.

69. What results should be expected, if lactose were


substituted for trehalose in this experiment, using
the same subjects?

A. Since lactose and trehalose are both


disaccharides, the same pattern of intolerance
sould be expected.
B. Lactose would be tolerated at higher doses
than trehalose, since it is a commoner
component of the diet.
C. Lactose would be tolerated at lower doses than
trehalose, since it is a less common component
of the diet.
D. One cannot determine the answer from the
information presented in this passage.

70, What can you conclude about Subject I from the


results shown in Figure 2?

A. Subject 1 has a high tolerance for trehalose in


the diet.
B. Subject 1 has a low tolerance for trehalose in
the diet.
C. Subject I regularly consumes trehalose-
containing foods.
D. Subject 1 almost never consumes trehalose-
containing foods.

71. The basic function of the gas chromatograph, like


the one used in the experiment described in the
passage to analyze the concentration of hydrogen in
the breath samples, is to separate gaseous
compounds in a mixture on the basis of:

A. their densities.
B. their molecular interactions within a column.
C. the masses of each isotope in the compound.
D. their solubilities in organic solvents.

Copyright @ by The Berkeley Review 250 The Berkeley


Specializing in MCAT
Biology Fuel Oxidation during Dxercise Passage XII

Passage XII (Questions 7 3-79) 75. Which of the following compounds is an important
fuel source for resting muscle?
Exercise is a time of dynamic change in fuel oxidation
ior skeletal muscles. During rest, skeletal muscle respires A. Oleate
aerobically. Skeletal muscles respire anaerobically for the B. B-hydroxy butyric acid
tlrst 45-90 seconds of moderately intense exercise. As C. Lactic acid
erercise progresses, the muscles switch to aerobic D. Glycogen
respiration.

Table 1 outlines more specifically the fuels used


Juring different phases of exercise. The following
:bbreviations are used: FFA (free fatty acids), GMG
slucose from muscle glycogen), BG (blood glucose), Gly 76. What is the carrier molecule for blood glucose?
glycogenolysis), and Gng (gluconeogenesis).
A. Albumin
B. Glucose-bindingprotein
Liver Glucose Output C. Hemoglobin
PhaseT FFA GMC BG from Gly from Gng D. There is no carrier.
Rest +++ + 75Vo 25Va

5- 10 +++ + J I
10-40 + ++ ++ I t
40-90 ++ + +++ T t 77. During exercise, which of these hormones IS
by 240 +++ ++ 557o 457o involved in blood glucose maintenance?
t Time in Minutes
A. Insulin
Table 1
B. Glucagon
C. Glycogenphosphorylase
D. Oxytocin

73. Why is the first interval of moderately intense


exercise a period of anaerobic respiration?

A. Oxygen supply is transiently lower than


oxygen demand. 78. Which fuel provides the MOST energy per gram?
B. The Krebs cycle enzymes required for aerobic
respiration must be activated. A. Fatty acids
C. Fatty acids must be metabolized anaerobically. B. Blood glucose
D. Glycolysis cannot keep pace with the need for C. Glycogen
energy. D. Amino acids

?{. How does the blood glucose level change during


moderate exercise? 79. Which of the following statements is TRUE of
exercise that continues for more than four hours?

A. Blood glucose rises rapidly during exercise, A. Muscle glycogen is repleted by


and then plateaus at a higher level. gluconeogenesis.
B. Blood glucose .falls rapidly during exercise, B. The rate of gluconeogenesis continues to
and then plateaut at a lower level. decrease.
C. Blood glucose is relatively unchanged by C. The availability of free fatty acids from
exercise. adipose tissue begins to decline.
D. Blood glucose rise and falls in a cyclic fashion D. The rate of glycogenolysis continues to
as exercise progresses. decrease.

Copyright @ by The Berkeley Review 251 The Berkeley Review


Specializing in MCAT Preparation
Biology Urea Cycle Passage XIII

Passage XIII (Questions 80-87) Glutamate, once it passes into the mitochondrial
matrix from the cytosol, can lose its amino nitrogen as the
Every amino acid that is catabolized by the human ammonium ion in a reaction involving water and NADe
body must have its o-amino group removed from its or NADPo, or it can be converted to cr-ketoglutarate, as
carbon skeleton. Amino groups that are not reused in the indicated by the reverse reaction in Figure 2.
synthesis of nitrogenous compounds are converted to urea
in the urea cycle (Figure 1): The urea cycle involves four enzymatic steps, each of
which catalyzes a unique intermediary compound
2ADP necessary for the synthesis of an amino acid (Figure 3).
+Pi
l o o o g
IIr znrp o o @ too @
?oo foo foo
pnospnate
-cg|:nr'] .-\L-E, HCo3e + NH4o H3N-C- H -l C-
H1N- H H1N-C-H
-l HrN-C-H
-l
CH, CH' CH, CHr
I l- I t-
CH, cH' CHt CH'
I t' I t-
CH, CH" CH" CHt
I Citrulline Aspartate <---. t^t-
t' t- t-
ornirhin" I f
NHt NH
oT" Ot NH

U
l I
o=c H,N=C
-l H2N= C
I I

@ NH:
I
NH NH:
I
ooc- c- c- coo
l'' "*,+,L^.,]-*J :',"ff).
o HzH O

Ornithine Citrulline Arginino- Argininc


| ",€;;'-*"1 oxa,o. ) succinate

| .fJEHfY{ r G*",u,.1
Malate
I Krebs
Citrate
I
Figure 3

lEo cycte +
Fumarate cr-keto-
t.
succrnare
t'"'r""
Succinvl-
coe,'
80. Urea is primarily synthesized in the:

A. kidney.
B. blood.
Mitochondrial C. liver.
matrix D. muscle.

Figure 1

Amino u"ia, have their c-amino group removed


by transaminases,"in
enzymes that interconvert o-amino and 81. According to the passage, what is the
a-keto acids during transamination reactions. The keto- balanced reaction for the urea cycle?
acid acceptor of many amino groups is c-ketoglutarate.
A. 2NH+o + HCO:o + 3ATP + HzO -+
o o Urea + 2ADP + AMP + PPi + 2Pt
coo
I o @i* B. 2NH+e + HCQo + 3ATP + Aspartate +
c=o
I
CH, +
o foo
H3N-C-H
H3N-C
-H "ooo
I
c=o Urea + 2ADP + AMP + PPi + 2Pi
I I

| -o
CH' R
to
CHz R C. NH+@ + HCqe + 3ATP + H2O -+
coo coo Urea + Fumarate + 2ADP + AMP + PPi +
Amixo Keto
acid acid
cr-Keto- Glutamate D. NHao + HCqe + 3ATP + Aspartate + H1
glutarate
Urea + Fumarate + 2ADP + AMP + PP; +
Figure 2

Copyright @ by The Berkeley Review 252


Biology Urea Cycle Passage XIII

82. The two nitrogen atoms in urea can be traced back to 86. Argininosuccinicaciduria (ASA) is a rare autosomal
NH4o and: recessive disease caused by the absence of
argininosuccinase (E4 in Figure l). Its primary
A. citrulline. symptom is the accumulation of argininosuccinate in
B. arginosuccinate. the blood. A man whose morhJr had ASA and
C. arginine. whose father did not carry the ASA gene marries a
D. glutamate. woman with the disorder. What is the probability
that their child will NOT express the trait?

A. 0.25
83. The carbon atom in carbamoyl phosphate comes B. 0.s0
directly from the: c. 0.'7s
D. 1.00
A. glycolyric pathway.
B. citric acid cycle.
C. electron-transportchain.
D. oxidation of fatty acids. 87. Hyperammonemia can be caused by a deficiency in
arginase (E5 in Figure l). This disorder can be
controlled by the restriction of protein in the diet and
by diversion therapy (use of an alternative metabolic
{. The ammonium ion is quite toxic to brain tissue. pathway to reduce the toxic concentrations of a
Which of the following amino acids is able to react particular metabolite). Administration of sodium
with the ammonium ion and convert it into a non_ benzoate leads to the formation of hippurare, a
toxic compound? compound easily excreted in the urine.

o
A. B.
o-\ -$-fi,-8-oo
o o
coo
@t
HjN- C- H
ot coo
H3N- C- H
I I
CH, CH,-a
t- |
SH coo " Hippurate

D. The rationale for the use of sodium benzoate as a


o therapeutic procedure for hyperammonemic
o
@t
coo Orcoo individuals is that it:
HrN- C- H
H3N- C- H
I CH.
CH,
t'
t'
I
A. reduces the concentration of ammonia in the
CH.
| "g
F:n o _ blood by increasing the synthesis ofglycine.
coo
H-N--N-g B. reduces the concentration of ammJnia in the
_ blood by increasing the synthesis of aspartate.
C. increases the concentration of urea in the blood
so it can be eliminated in the urine.
D. increases the concentration of Krebs cycle
intermediates in order to produce more
ils Which of the following types of reactions can result aspartate for the urea cycle.
in the formation of ammonia by the removal of the
o-amino group from an amino acid?

.{. Oxidative deamination


d
B. Transamidation
C. Reductive deamination
D. Dehydrogenation

ii,-:i_eht @ by The Berkeley Review


2s3 The Berkeley Review
Specializing in MCAT preparation
Biology Gluconeogenesis and the Qori Cycle Passage XfU

Passage XIV (Questions 88-94) 90. Which of these statements would describe th
metabolic fate of lactate under anaerobic conditiom!
Under normal physiological conditions, the brain uses
about 120 grams of glucose each day as its only fuel L Lactate is converted to pyruvate.
source, compared to the roughly 160 grams of glucose n. Laclate is converted to alanine.
that the body uses each day. Since the body stores less ilI. Lactate is converted to lactose.
than a day's supply of glucose, this carbohydrate must be
synthesized from noncarbohydrate precursor molecules A. I only
during times of fasting or intense physical exertion. B. II only
C. I and II only
The synthesis of new glucose occurs during a process D. I, II, and III
called gluconeogenesis.It takes place primarily in the
liver and, to a lesser extent, in the cortex of the kidney.
Glycerol, lactate, pyruvate, and many amino acids (e.g., 9t. What metabolic condition could activate the
alanine) are all important precursors of glucose. Cori cycle, in which there is glucose output
muscle tissue and lactate output from the liver?
Lactate is an end product of anaerobic glycolysis and
is produced primarily in erythrocytes and skeletal muscle. A. Pregnancy activates the reverse Cori cycle.
When lactate is released into the bloodstream, it travels to B. Exercise activates the reverse Cori cycle.
the liver where it is taken up by the hepatic portal vein c. Fasting activates the reverse Cori cycle.
and reoxidized to pyruvate in the cytosol ofhepatocytes. D. There is no metabolic condition that
activate the reverse Cori cycle.
The cycle of reactions involving the production of
glucose from lactate in liver tissue and the production of
lactate from glucose in muscle tissue is called the Corl 92. If lactate dehydrogenase were inhibited
cycle (Figure 1). anaerobic conditions, what metabolic effect
this have?
Glucose Glucose
+ A. Glycolysis would eventually stop due to a
4^rP jll ofNAD@.
2 GrP ll [,o," B. Glycolysis would continue, but without
production.
Pyruvate Pyruvate
C. Glycolysis would eventually stop due to a
+ ofATP.
il n D. Glycolysis would continue, using FAD as
electron transporter.
Lactate Lactate

In liver tissue In muscle tissue 93. Which of these statements correctly de


Figure 1. The Cori cycle. alanine and its biochemical derivation?

A. Alanine is a nonessential amino acid


produced by transamination from pyruvarc-
88. Under what conditions would muscle release the
MOST lactate per minute? B. Alanine is an essential amino acid
produced by transamination from lactate.
A. During sleep C. Alanine is an essential amino acid
B. During studying produced by reduction from lactate.
C. During vigorous exercise D. Alanine is a nonessential amino acid
D. Following a light meal produced by reduction from pyruvate.

89. Which of the following enzymes is involved in the 94. When glucose produced by gluconeogenesis
first step ofgluibneogenesis in the liver? the liver, it exits through which blood vessel?

A. Pyruvatedehydrogenase A. The hepatic artery


B. Pyruvate carboxylase B. The hepatic vein
C. Lactatedehydrogenase C. The hepatic portal vein
D. Lactatecarboxykinase D. The hepatic portal artery

Copyright @ by The Berkeley Review 254 The Berkeley


Specializing in MCAT
Biology Starch Blockers Passage XV

Passage XV (Questions 95-100) 95. One volunteer for the study was not allowed to
participate, because she produced no increase in
In the early 1980s, a popular weight reduction fad was breath hydrogen following the administration of
the "starch blocker" tablet. Starch blockers were lactulose in the first phase of the study. Why would
developed and marketed to inhibit the intestinal this disqualify her from the parriciparion?
carbohydrate-hydrolyzing enzyme, o-amylase. In theory,
a dieter could consume unlimited amounts of complex A. She had too many colonic bacteria that ferment
carbohydrates, take a pill containing the starch blocker, nondi gestible carbohydrate.
and be protected from weight gain, since the starches B. She did not have enough ofthe colonic bacteria
rr ould not be digested. Instead, they would pass through
that ferment nondigestible carbohydrate.
the small intestine mainly undigested. C. She had too many colonic bacteria that ferment
digestible carbohydrate.
Further claims about starch blockers focused on the D. She did not have enough of the colonic bacteria
ibsorption of glucose and the lowered insulin response to that ferment digestible carbohydrate.
:ngested glucose after they were used. Several researchers
planned the following experiment to examine the effects
,f starch blockers. Breath hydrogen content was measured
:-r determine fermentation of undigested starch by colonic 96. Figure 1 indicates the breath hydrogen response to
::cteria. Eight men and women participated in each of the all three phases ofthe study:
'rree phases ofthe study:
90

Eto
tt:ase I: Lactulose Test
eo
15 grams of the nondigestible carbohydrate lactulose f50
.. ere administered, and breath hydrogen concentration L

, rs measured over 4 hours. 3:o


ca

10

::'.cse II: Test meal with placebo


0 60 120 180 240 300
Time (minutes)
Subjects ate a test meal ofregular foods, containing 50 Figure l. Breath H2 response as a function of time.
-:.ms of starch. Subjects took a placebo capsule
" ltaining calcium carbonate with the meal. Breath Which of these statements could be supported by the
-.. Jrogen concentration was measured over 4 hours.
data in Figure 1?

I. The starch blocker acted in a manner that was


" similar to the placebo.
".":se III: test meal with starch blocker
U. I actulose promoted a rise in breath hydrogen
5ubjects ate a testlneal ofregular foods, containing 50 beginning about an hour after it was given.
-*:ns of starch. Subjects took a test capsule containing III. Intestinal c-amylase was nor inhibited by the
starch blocker.
" :rmilligrams of phaseolamin, an c-amylase inhibitor,
the meal. Breath hydrogen concentration was
- t.-rured over 4 hours. A. I only
B. I and III only
C. II and III only
D. I, II, and III

97. Intestinal o-amylase is synthesized in the:

A. small intestine.
B. gall bladder.
C. large intestine.
D. pancreas.

' r-ight @ by The Berkeley Review 255 The Berkeley Review


Specializing in MCAT preparation
Biology Starch Blockers Passage XY

98. The following graphs show the glucose and insulin 100. How does an actual inhibitor of cr-amylase affect th
responses of test subjects to Phases II and III of the digestion ofthe simple sugars glucose and fructose?
study.
A. The digestion of simple sugars is unaffected
the starch blocker.
Placebo B. The digestion of simple sugars is affected i
Starch blocker
the same way as starch digestion.
9 r<n F
v. TL^
r rrs ll-^^+!^-
utEsJrtuil ^fur ^:--l^ suBalrs :^ts
stlrtprg ^--^^-^
o^
Ae
,o0
roo
D.
blocked by an inhibitor of o-amylase.
Simple sugars are destroyed by
d- 5u actions of the cr-amylase inhibitor.
o-
0 30 60 90 120 150 180
Time (minutes)
Figure 2. Glucose response.

r40
Placebo
Starch blocker
7o 100

F> 60

f20
0 30 60 90 120 150 180
Time (minutes)
Figure 3. lnsulin response.

These data indicate that when phaseolamin was


given to these subjects, it produced:

A. an increased glucose response and a decreased


insulin response compared to the placebo test.
B. a decreased glucose response and an increased
insulin response compared to the placebo test.
C. no change in glucose or insulin responses
compared to the placebo test.
D. decreases in both the glucose and insulin
responses compared to the placebo test.

99. The test meal contained 50 grams of starch in foods


the subjects ordinarily consumed. Other components
of the test meal were protein, simple sugars, and fat.
Which of the following foods is mostly carbohydrate
in the form of starch?

A. Turkey 'r-
B. Rice
C. Butter
D. Orange juice

Copyright @ by The Berkeley Review 256 The Berkeley


Specializing in MCAT Pre
Biology Metabolic Pathways Section VIII Answers

1. B is correct. The faster the growth rate of an organism, the shorter its doubling time. The guiding principle in
problems such as this one is that growth rate is directly proportional to the amount of ATP available to each cell per
unit time. In the presence of plenty of 02, prokaryotic organisms (such as E. coll can make 38 NTPs per molecule
of glucose oxidized, whereas eukaryotic organisms (such as yeasts) can make either 36 or 38 ATPs. For the
purposes of this question, we were to assume 36 NTPs per molecule of glucose oxidized (under aerobic conditions).
When oxygen is removed, the yeast cells switch over to anaerobic metabolism, and the net yield of NTP per
molecule of glucose falls to just 2. This is called anaerobic fermentation.

Depriving yeast cultures of Oz causes each cell to produce 18 times less NTP per molecule of glucose consumed
(from3612 = l8). Pasteur found that yeast cells grew 6 times less rapidly under these conditions, implying that they
were forming NTPs at a rate only 6 times less rapidly than before. This means that they must be consuming glucose
more rapidly than before (as was noted in the question). In particular, they must be consuming glucose at a rate
approximately 3 times faster (from l8/X = 6, or X = 18/6 = 3). The correct choice is B.

C is correct. In the presence of oligomycin, the Fs transmembrane protein is blocked. The proton gradient formed
by electron transport and proton-pumping can no longer be relieved. The result is that the cells revert to anaerobic
metabolism and just do glycolysis, synthesizing only 2 net NTPs per molecule of glucose consumed. The metabolic
products will be ethanol (CHaCHzOH) and carbon dioxide lCOz). The overall effect is the same as if you had
removed the cells from oxygen, or had added an electron-blocking agent such as cyanide. The correct choice is C.

3. C is correct. In this case, oxidative phosphorylation is still prevented by the binding of oligomycin ro the Fo
transmembrane protein; but the presence of 2,4-DNP means that no proton gradient is formed, so electron transport
can and will continue. The end products of glucose metabolism will be COz and HzO. If anything, the rate of
electron transport will be a little faster, since no NTPs are being formed during electron transport (i.e., there is no
resistance to electron transport). But overall, the growth rate of the culture would be expected to decline. A net of 4
NTPs are synthesized per molecule of glucose metabolized. Why? 2,4-DNP uncouples all of the NTPs that are
normally formed by oxidative phosphorylation, but leaves unaffected any NTPs formed by substrate-level
phosphorylations. Those substrate-level phosphorylations include the 2 net ATPs formed in glycolysis and the 2 net
GTPs formed during the Krebs cycle. Therefore, in the presence of excess 2,4-DNP, the amount of NTP formed per
glucose molecule when metabolized under aerobic conditions can be expected to decline from 36 to 4. The correct
choice is C.

4. B is correct. During normal glycolysis, one of the high-energy phosphate compounds, which serves as a source of
phosphorylating energy for the synthesis ofATP, is 1,3-bisphosphoglycerate.

-o
U ATP
^lO
U. o- no
- c- O- IlP-O ADP I
''c-
ro '-
H-C-OH \ I
H- C- OH
I I
H2C-O - POj2- Phosphoglycerate Hrc- o - Po.2-
kinase
1,3-Bisphosphoglycerate 3-Phosphoglycerate
StepT

In the presence of excess arsenate ions, the synthesis of ATP in Step 7 does not occur. In Step 6, the compound 1-
arseno-3-phosphoglycerate is hydrolyzed to inorganic arsenate and 3-phosphoglycerate, the intermediate we would
see at the end of Step 7.

The overall result is that glycolysis appears to proceed normally, but there is no net formation of ATP. Two ATPs
are invested in Step I of glycolysis; but in the presence of excess arsenate, only two ATPs can be synthesized (in
Step l0 of glycolysis when phosphoenolpyruvate is converted to pyruvate). Therefore, there will benonet gain in
ATP and no conservation of chemical energy. All of the free energy difference between glucose and pyruvate will
be lost as heat. The cells must stop growing, since they are no longer able to make any ATP under these
circumstances. The correct choice is B.

Copyright @ by The Berkeley Review 2s7 The Berkeley Review


Specializing in MCAT Preparation
Biology Metabolic Pathways Section VIII Answers

5. C is correct. Arsenate competes with phosphate for the active site of the dehydrogenase enzyme at Step 6 in
glycolysis. Note that arsenate is an analog of phosphate. If we were to flood the system with phosphate, then
phosphate would outcompete the arsenate, and the (majority of) cells would continue to thrive. If arsenate were a
noncompetitive inhibitor, then: (l) it would not resemble phosphate, and (2) no matter how much phosphate we
added to the system, the inhibition would not be overcome.

Arsenate can also be considered an uncoupling agent, because it uncouples the phosphorylation of ADP to ATP in
Step 7 but allows glycolysis to proceed. The action of arsenate is analogous to the action of 2,4-DNP in the electrot-
transport chain, in that they both uncouple a phosphorylation event. The correct choice is C.

6. A is correct. It is important to know the basics of metabolism. Pyruvate is the end product of glycolysis,
occurs in the cytosol. Pyruvate can pass from the cytosol to the mitochondrial matrix through a pyruvate-l
symport. Once in the matrix, it is oxidized to acetyl-CoA. In this process hydrogens and electrons are passed
NAD@, and NADH + Ho is formed. In Figure 1 of the passage, we see that NADH + Ho enter the electron-
chain at the level of complex I (also called the NADH-Q reductase complex). The correct choice is A.
7. B is correct. As shown in Figure 1 of the passage, hydrogen ions are vectored across the inner mi
membrane from the matrix to the intermembrane space. This not only establishes a difference in the [HCI]
the matrix and intermembrane space, but it also establishes a charge difference. Note the net positive charge on
surface of the inner membrane facing the intermembrane space and the net negative charge on the surface of
inner membrane facing the matrix. This electrochemical gradient allows the hydrogen ions to come back into
matrix through the FoFrATPase. In the process, ATP will be synthesized from ADP and Pi.

Choice A says that electrons are passed from NADH and FADHz to Oz. This is true, but it does not answer
question. In choices C and D, ADP is indeed transported into the matrix, and it can occur by way of either
antiport or a symport. Even though ADP enters the matrix, it does not account for the mechanism behind A
synthesis. The correct choice is B.

8. C is correcL First, we need to draw a picture, so that we can visualize what is happening. In Figure A below, we
a phospholipid vesicle with an Fq protein. Inside the vesicle is an elevated concentration of K@.
inR,r* A
Fo Protein
\\
{H+
Phospholipid
lH+l Valinomycin
Bilayer \\

efflux

Vesicle Vesicle

Figure A Figure B

Based on information in the passage, we know that the Fe protein facilitates the specific transport of Ho
membrane. If the Fo protein is not in the membrane, He cannot be transported across. If the Fo protein is ir
membrane, H@ can be ffansported across if there is some type of gradient. In the case of the inner membrane
mitochondrion, H@ is vectored into the intermembrane space. This allows for a chemical and electrical
Ho across the inner membrane. In the case of the inner mitochondrial membrane, the F6 protein allows H@ to
to the matrirdown its chemical and electrical gradient.

In the case of these synthetic vesicles, the situation is pretty much the same. In order for Ho to enter the
there needs to be a gradient. Within the vesicles is a high concentration of K@. Once the antibiotic valinom
cyclic peptide) is added to the solution, it diffuses through the membrane and into the vesicle's interior, binds a
ion, and then transports that ion to the exterior of the vesicle (Figure B). Since K@ is leaving the vesiclg

Copyright @ by The Berkeley Review 254 The Berkeley


Specializing in MCAT
Biology Metabolic Pathways Section VIII Answers

referred to as efflux. Valinomycin can transport up to about 104 K@ ions/second through the membrane. This
decreases the [Ko] inside the vesicle, which in turn means there is less positive charge at the interior. The Fo prote in
now allows the H@ to pass to the interior of the vesicle. Since H@ is entering the vesicle, it is referred to as influr.
The correct choice is C.

9. C is correct. DCCD is inhibiting the passage of H@ through the Fo portion of the FoFtATPase protein. This is
telling us two things: First, since FP cannot get back into the matrix, these ions are accumulating in the
intermembrane space. Eventually, there will be such a high [Ho] in the intermembrane space that H@ will no longer
be able to be vectored from the matrix to the intermembrane space. In other words, NADH and FADHz will not be
able to drop off their electrons and hydrogens. The electrons will not be able to be passed down the electron-
transport chain. Second, since He cannot pass thorough the Fo pore, ATP cannot be synthesized. The net result of
DCCD addition is elimination of electron transport and ATP synthesis.
2,4-DNP is an uncoupling agent and has the ability to pick up a H@ in the intermembrane space and transport it
across the inner mitochondrial membrane. This transport of H@ through the membrane bypasses the FoFtATPase
system. This reduces the [H@] in the intermembrane space. In turn, this allows more H@ to be pumped from the
matrix to the intermembrane space. When this happens electrons are dropped off at the electron transport chain and
o
the rate of electron transport increases. However, since DCCD is blocking passage of H through the FoFtATPase,
ATP cannot be synthesized. Even if DCCD were not present, we would still see a decrease in ATP synthesis because
of the uncoupling action of 2,4-DNP. The correct choice is C. I

10. B is correct. In the question we are given the equation that relates the standard free energy (AGo') to the standard
reduction potential (Eo'). We are told that F is the Faraday constant and that n is the number of electrons involved in
the transfer. We do not need the value of the Faraday constant. Even if we did, we could just consider the value as a
constant (i.e., unity) to simplify calculations.

\co,- _ (nXFXAEo')

We find the value for the number of electrons in the passage (see second paragraph) and in the table in the question.
FADH2 can pass 2 electrons to the electron-transport chain. Our equation now looks like the following:

aco' - _ (n)(F)(AEe') = _ (2XF)(AEo')

In the table in the question, we also find the values for Eo'. Note that the redox pairs are written as reduction half-
reactions.
Redox Pair Eo' {voltsl

Fumarate + zHo + 2eo -"-+ Succinate 0.03


-
FAD + 2H@ +Zea --- FADH2 - 0.22

The question now becomes one of arriving at the correct value for AEo'. The question it states that AEo' is the change
in standard reduction potential between the oxidized and reduced species. What is the oxidized species and what is
the reduced species? We need to consider the reaction given in the question.

FADH'
FAD
Succinate \ '- Fumarate

In this reaction, succinate is the electron donor. Succinate gives up its electrons to FAD, the electron acceptor. The
electron donor isrcalledthe reductant,while the electron acceptor is called theoxidant. Succinate is oxidizedto
fumarate, while FAD is reduced to FADHz. We can expand on our equation as shown below:

AGo' - - (nXFXAEo') = - (2XF)(AEo') = - (2)(F)(Eo' (acceptor) - Eo (donor))

right @ by The Berkeley Review 259 The Berkeley Review


Specializing in MCAT Preparation
Biology Metabolic Pathways Section VIII Answerg

If we substitute the appropriate values, we get:

AGo'- -(2XFXAE9')=- (2)(F)(Eo'(accepror) -Eo (donor))= - (2XF)t(- 0.22) - (0.03)l=- (2XF)(- 0.25)

We can clean this up a bit:


AGo'- + (0.50XF)
This question tests our knowledge on oxidation-reduction reactions and the manipulation of signs. The
choice is B.
11. D is correct. It is important to know the differences between prokaryotic and eukaryotic cells. Mitochondria are
found in prokaryotic cells. They are lound only in eukaryotic cells. Therefore, we can immediately eliminate
A and B.
All prokaryotic cells have a plasma membrane that surrounds their cytosol. If the prokaryotic cell is a Gram-
cell, it also has an outer membrane (outside the peptidoglycan layer). A Gram-positive bacterium has oniy
plasma membrane surrounded by a peptidoglycan layer. Since a prokaryotic cell requires energy (ATp) to survir
will want that ATP delivered to reactions in its cytosol as quickly as possible. Oxidative phosphorylation occurs
the cytosol. Since oxidative phosphorylation and electron transport are coupled, it would mean that elecl!
transport occurs on the plasma membrane. And this is what is observed in both Gram-positive and Gram-
bacteria. The correct choice is D.

12. A is correct. Exogenous means "from the outside." Its antonym is endogenous. For the cell, exogenous
"extracellular." This rules out answers containing cholesterol inside the cell. So choices B and D are wrong.
C is wrong, too. Although cholesterol in arterial plaques is exogenous to the cell, it is not free to move around-
word sequestered means "trapped" or "stored away." The correct choice is A.
13. C is correct. From the diagram, we see that LDL particles deliver cholesterol to cells. They must interact u'ith
LDL receptor. If the LDL receptor is damaged or missing, LDL particles and their cholesterol remain in the
This makes statement III correct. If LDL can't enter, then the negative feedback from exogenous cholesterol
available. Therefore, the cell alters itself to increase intracellular cholesterol. This means levels of
reductase are increased. Statementl is correct. Also, ACAT activity increases to hydrolyze stored cholesterll
into free cholesterol. Statement II is wrong. The correct choice is C.
14. B is correct. This is a simple change in units. (5 mmol/liter) x (386.64 mg/mmol) x (l liter/10 dL) = 193.3
The other choices are incorrect due to either multiplying or dividing error by a factor of l0 (choice C) or flippi
units upside down (choices A and D). The correct choice is B.
15. C is correct. Three of the answers are true, but the answer we want is the false one. We are told in the
the lysosolne fuses with the LDL receptor-LDl particle complex. Hydrolytic enzymes degrade both
esters to free cholesterol and the apolipoprotein into amino acids. Free cholesterol is released. This is a si
the cell decreases LDl-receptor synthesis. The free cholesterol can be reesterified into cholesteryl ester and
the cytoplasm. Choices A, B, and D are all true. The false answer is choice C. The correct choice is C.

16. A is correct. First of all, there is no extracellular cholesterol synthesis. Choices C and D are therefore inc
diet provided no cholesterol, then the cells would need to synthesize more cholesterol to meet their requi
Choice B is wrong. The correct choice is A.
17. B is correct. Mevinolin is a competitive inhibitor of HMG-CoA reductase. On the Lineweaver-Burk
competitive inhibitor has the same Vmax as the enzyme acting alone, but the Ku is higher. Only Line B c
y-axis at the same point as the line for HMG-CoA reductase. This means choice B is correct. Choice C i
noncompetitive inhibitor. Choices A and D are just tricks. The correct choice is B.

18. B is correct. The names of the lipoproteins comes from their activity during centrifugation. Extraction
organic solvent would separate fat-soluble components from water-soluble components, but this is not
want. Choice A is thus incorrect. TLC requires organic solvents as well, so the lipids would all mix

Copyright @ by The Berkeley Review 26,0 The Berkeley


Specializing in MCAT
Biotogy Metabolic Pathways Section VIII Answers

means choice c is also incorrect' A salt solution.would not help:


choice D is incorrect. Take advantage of the table
is how ttev sr,oi,iJb" sepa.ated, which is best accomprished
:::,Tl'rffi#l'ff"1'ii1}t"fl:1"';T;.this by
19. B is correct' Read this from the table. chylomicrons. have
the largest diameters and the lowest densities.
they are relatively large and more buoyantihan Therefore,
the other lipoproteins. The correct choice is B.
20. c is correct' This is not in the passage. we can eliminate VLDL, because
the passage tells us those lipids are made
in the liver' This eliminates choice"D. If we remember how ihe lipoproteins transform, we know
that vLDL
rhe rest comes do*n,o.'"^o.v. HDL does nor conrain
3iiitr'Xk?i;::;H*ffiifii::'i$:"o' dietary ripid;

21. D is correct' Acetate is linked to CoA by the enzyme thiokinase,


forming acetyl
Palmitate containsl6 carbons, which come from 8 acetyl c"A;, ;; statement II coA. Statement r3c
I is correct.
isotope of carbon' Statement III is correct' because_stable is also correct. is a stable
isotopes are safer for use with human beings
at high doses. onlv staiement IV is incorrect, so choice than
:X1t::1";.tsotopes D is the bert-answe.. The correct
'r',
A is correct' Eliminate choices c and D, since we are trying
to make methyl esters. Transesterification requires a
nucleophile in an acidic environment. The correct
choice is A.
23. A is correct' The group that is fasting is not doing much DNL,
because that is a pathway that is most activated when
;ffi:;l?::#::lXil:,::j|f
converted to rat.-rhe sroups ;;i;; carbohyirate a" -".. oNL than the rasting

24. B is correct' In all four answer choices, we find that


the steps in the calvin cycle catalyzed by E2(phosphorylation)
and Es (condensation) are the same. Therefore,
we do not need to concern ourselves with them,
Et, Es, and E+. Let's consider these one at a time. only with enzymes

Enzyme Et is ribulose-1,5-bisphosphate carboxylase. It is also called


rubisco.In the reaction, we see that coz and
H2o are involved' The reactant Ru-1,5-BP is first carboxylated,
which means that the reaction is a carboxylation
reaction' However' after the carboxylation event,
the transient six-carbon intermediate is immediately
two molecules of 3-Pc' Thus, this reaction is also a hydrolyzed to
hydrolytic re""ri"". This tells us that the reaction
El corresponds to either a carboxylation or (once the Cbz has catalyzed by
been added) a hydrolysis.
Enzyme E5 is involved in a dephosphorylation reaction,
but is it a reductive or oxidative reaction? The
here is NADPH (+ H@)' This is the reduced form coenzyme
of the coenzyme. It reacts with 1,3-BpG, which is the
form of the three-carbon compound. NADPH is converted oxidized
to NADpo. This is the oxidized form of the
l'3-BPG is converted to 3-PG' In the process inorganic phosphate coenzyme.
(Pr) has been lost (i.e., a dephosphorylation)
medium' 3-PG is the reduced form of the three-carboncompouno. to the
written' it is therefore areductive dephosphorylation. we.;
itris sequence constitutes a redox reaction. As
coenzyme is reduced and which is oxidized. ";tiit;"swer this by considering which form of the
Oxidized form of the
coenzyme (no H_atoms)
Reduced form of the
coenzyme (has H-atoms) r1
________- NADP+ Lostaphosohare
NADPH +Pi -< - (dephosphorylarion)
+
o--.
I -o - 'o'' 'H* |
oxidized.l
"lJ:""'"'
u,c-o-eo,]
\ E3 - ":i_"1 |*"0,..0
I srl_o_po,r, )
1,3-BPG 3-PG

At this point'we know that enzyme E3 is involved in a reductive dephosphorylation.


choices C and D. Once we can determine what enzymeE+
This allows us to eliminate
does, we will have the answer.

I . ryright by The Berkeley Review


@
26t The Berkeley Review
Specializing in MCAT preparation
Biology Metabolic Pathways Section VIII Ans

Enzyme E+ should look familiar to you (think of glycolysis). It is an isomerase enzyme that makes possible
interconversion between G-3-P and DHAP. Isomerases simply rearrange the carbon and hydrogen atoms Lrm
molecule. This is exactly what we see below. Notice that there has been no loss of atoms.

o-- H
CH,-OH (^,
I I
c=o H-C-OH
I I
H,C- O - PO,2 E^ Hrc- o - Po,2'

DHAP G-3-P

An enolase enzyme promotes the reversible removal of water from a molecule. We do not see the loss of u ata
the addition of water) between G-3-P and DHAP. The correct choice is B.

25. B is correct. The function of the Calvin cycle is to convert COz into carbohydrates. As shown in Figure l-
carbohydrate that is produced is glucose, and glucose is a six-carbon sugar (i.e., C6H1206). Glucose is an i
molecule to remember. If we end up with a six-carbon sugar, we must start with six molecules of COz. \\-e
eliminate choices A and D.
The difference between choices B and C involves the number of ATP molecules and the number of water
on the reactant's side. Let's consider the ATP molecules to see if we can deduce the correct answer.

In Figure l, there are only two places where ATP is used. If we react a molecule of COz with the five
compound ribulose-1,5-bisphosphate, a six-carbon transient intermediate is formed, which is rapidly hydroll
yield two molecules of 3-phosphoglycerate. Since six molecules of COz react with six molecules of iibulc',
bisphosphate, the result will be twelve molecules of 3-phosphoglycerate. The next step involves the conversion
phosphoglycerate into 1,3-bisphosphoglycerate. Twelve molecules of 1,3-bisphosphoglycerate are required ro
twelve ATPs. That takes care of the first part of the cycle that generates ATPs. The second part of the c1'cre
generates ATPs is in the conversion of ribulose-5-phosphate to ribulose-1,5-bisphosphate. Since six molei
ribulose-1,S-bisphosphate are needed to react with six molecules of COz, then six ATps are also required ro
the conversion from six molecules of ribulose-5-phosphate to six molecules of ribulose-1,5-bisphosphate. Thu
requirement for ATP in a balanced reaction for the Calvin cycle is 18. Without going any furthei, we can ehmi
choice C. The correct choice is B,
26. A is correct. This question involves carbon balancing. Glyceraldehyde-3-phosphate is based on the noc
glycerol, which is a three-carbon compound. At some point, glycerol was oxidized to the aldehl-de
(glyceraldehyde) and then phosphorylated, giving glyceraldehyde-3-phosphate. The point is that the mol,
three-carbon compound. We have 6 of these molecules for a total of l8 carbons. Note that the Calvin cycle
for all of the carbon atoms--not one is lost through decarboxylation.
In choice B, we have 3 molecules of Ru-1,5-BP and 1 molecule of glucose. Glucose is a six-carbon
this leaves 12 carbons to account for. How many carbons does Ru-1,5-BP contain? Look at the name. \\e
that the molecule contains 2 phosphate groups, one on the C-l carbon and the other on the C-5 carbon. \[e
conclude from this that Ru-1,5-BP contains 5 carbon atoms. We could also arrive at this same idea based
name" ribulose" (Ru). In the Calvin cycle, we saw that one place ribulose can come from is ribose. Ribose u
carbon sugar that is found in numerous compounds, including nucleic acids (DNA and RNA). If ue
molecules of Ru-1,5-BP, it means we have 15 carbon atoms. Since we need only 12 carbon atoms, ttrmc
carbon atoms to account for. Eliminate choice B. We can follow the same procedure for choice C. Xu-
contains 5 carbon atoms, E-4-P has 4 carbon atoms, and S-1,7-BP has 7 carbon atoms. This is a total of l6
atoms. In this case, we have 2 carbon atoms that are not accounted for. Eliminate choice C.

Choice D is interesting. Glucose contains 6 carbon atoms. Since there are 2 glucose molecules, we have ll
atoms. F-1,6-BP is on the pathway to glucose. It also has 6 carbon atoms. The total between these three mc
l8 carbon atonts, which is how many we have with 6 molecules of G-3-P. At first glance, this seems like a

How do we distinguish between choices D and A? To remove glucose as our product carbohydrate, we musl
the Calvin cycle intermediates in order to regenerate 6 molecules of Ru-1,5-BP. If we do not, there will be
1,5-BP to condense with the COzthat is hanging out withrubisco (Er). This is exactly what will h

Copyright @ by The Berkeley Review 262 The Berkeley


Specializing in MCAT Pre
Biology Metabolic Pathways Section VIII Answers

convertthe6moleculesof G-3-Pinto2moleculesof glucoseand I moleculeof F-1,6-BP.However,if weconvert


the 6 molecules of G-3-P into 3 molecules of Ru-1,5-BP and I molecule of G-3-P, then we can put that I molecule
of G-3-P in a conceptual holding tank. The 3 molecules of Ru-1,5,-BP can combine with 3 molecules of COz. After
the transient intermediate splits, there are again 6 molecules of G-3-P. This is what we started with in the question. If
these 6 molecules of G-3-P are converted into 3 molecules of Ru-1,5-BP and I molecule of G-3-P (which we also
place in a holding tank), then there are enough Ru-1,5-BP intermediates to continue the Calvin cycle. The two G-3-p
intermediates waiting in the holding tank can be combined to form a product carbohydrate (like glucose). The
correct choice is A.

D is correct. The structure of 3-PG is given in the passage. The question states that the labeled l4C is at the
carboxyl carbon (indicated by the dot (.) in the structures below). This is also the C- I carbon atom. 3-PG is
eventually converted to G-3-P. Since there is no rearrangement of that C-l carbon atom, the label remains in the
same place. Figure I indicates that G-3-P and DHAP combine to form F-1,6-BP. Since DHAP and G-3-P are in
equilibrium with one another through an isomerase enzyme (E+), the label also appears at the C-1 carbon in DHAP.
Again, there has been no realrangement ofcarbon atoms (because carbon-carbon bonds have been broken).
o...ro o o-...H .
f E3 i E1 cH2-oH
H-C-OH +", + H-C-OH :=:::+ f=O
Hrc-o - por2- Hrc- o- por2' H.c- o- po.2-

3-PC c-3-p DHAp


G-3-P and DHAP combine in an aldol condensation to form F- 1,6-BP. This is achieved by the covalent union of the
two C-1 carbons from both metabolites, The six-carbon sugar is formed, and the label appears at the two central
carbon atoms, which are the C-3 and C-4 carbons.

o-\c- -H
HrC- O- POr2-
I I
C:o H_C-OH
.l .l
HO- C- H F-
"o- E" HO-C- H
al '- .l
H- C- OH H-C-OH
I I
H- C- OH H-C-OH
I I
Hrc- o - Po.2- HrC- O- POr2

F-1,6-8P G-6-P

F- 1,6-BP is converted to F-6-P and then to G-6-P with no rearrangement of carbon atoms. The labels do not change.
The correct choice is D.

D is correct. From the passage, we know that ATP and NADPH are synthesized during the light reactions and that
these metabolitd3 are used during the (lighrindependent) dark reactions. When light is shining on the algae, ATP
and NADPH are synthesized; when there is no light, ATP and NADPH are not synthesized.

in Step A of Experiment I, green algae are exposed to light and COz for an extended period of time. A lot of ATP
and NADPH are synthesized. In Step B, the light source is removed, and l4COz is added. At this point, the cells are
in darkness, so ATP and NADPH cannot be synthesized. This means that the supply of these metabolites to the
Calvin cycle begins to decrease. The reactions that rely on these metabolites are:

NADP+
coz NADPH +Pt
ATP
ADP
I + H2O ATP OI' +H+ l
\/ \._ /- /-
Ru-5-P
u 3-PG \ 1.3-BPc \ o-r-,
Et3 E1 E2 Ej

ii the supply of ATP decreases, less Ru-5-P is converted to Ru-1,5-BP. Similarly, less 3-PG is converred to 1,3-
BPG, and less 1,3-BPG is converted to G-3-P. As we start our addition of lacoz, that label is incorporated into the
R.u-1,5-BP that is still present and is converted to 3-PG. But 3-PG is not as readily converted to 1,3-BPG. Therefore,

263 The Berkeley Review


Specializing in MCAT Preparation
Biology Metabolic PathwaYs Section VIII A

to increase. This is e
the l4C label accumulates in 3-pG and the level of radioactivity of that metabolite begins
what we see in the graph shown in the passage.

Some of this labeled 3-pG is still converted to labeled 1,3-BPG, which in turn is still
converted to labeled G-
ATp NADPH around, but their concentrations are ever-decreasing. The I
This is because there is still some and
find its way to Ru-5-P, and some of this metabolite will be converted into labeled Ru-
in G-3-p will eventually
Bp. The levels of raOloaciiuitv in this metabolite will be quite low, because not enough of it is being synthesi:
The correct choice is D.

29. B is correct. In Step A of Experiment III, the algae are exposed the algae to lacoz in darkness' The labeled I

a high level of radioac


distributed (as we have previously discussed) . At the end of Experiment I, 3-PG had
and Ru- 1,5-BP had a low level of radioactivity
the algae, and then
In Step B, all forms of COz are removed from the environment immediately surrounding
to synthesize ATP.and NADPH. The reactions in the Calvin cycle
lighh ;re turned on. The light reactions begin
3-PG is converted to the It
;tffi;il;';;;"ii;r ian on"" again operate at an increased rate. The labeled
Ru-1'5-BP. However'
I,j-SpC, and so on. Evenrually, the labelld Ru-S-P will be converted to the labeled
there is no COz, Ru-1,5-Bp cannot be converted to 3-PG. Therefore, Ru-1,5-BP
begins to accumulate and
high level of radioacrivity, while 3-PG begins to disappear and shows a low level of radioactivity. The
choice is B.

30. C is correct. This question tests our recognition of the chemical structures of common sugars. Choice A is
Choice B is sucrose. Choice C is correct, lactose. Choice D is cellobiose. The correct choice is C.
31. D is correct. Bacteria are the most common organisms used in the fermentation of milk to produce y
However, cheese may be cultivated by the activity of both bacteria and molds. Alcohol is not produced in
amounts by these particular fermentations (yeast is used for alcoholic fermentation). So, choice B is incorrecr
acid gives yogurt its sour taste, but it does not break down lactose. Choice A is incorrect. Disaccharides are
dimers. Microorganisms break down disaccharides such as lactose during fermentation. This makes the fermented
product more tolerable to those lacking lactase. Choice C is incorrect. The correct choice is D.
32. D is correct. Even if one has forgotten most of the digestive physiology that one learned, it is still possible to reason
this out. Lactose is not broken down nor is it absorbed in the small intestine, if lactase is missing. Therefore, the
lactose passes intact into the colon, where there are lots of bacteria. They rapidly ferment the lactose, producing gas
and acidic compounds. The gas production is mentioned in the passage as a symptom, so statement I is correct
Working from there, the bacterial production of excess acids irritates the colonic epithelium, and the bowel contens
are passed through faster. Statement III is also correct. This produces diarrhea, as mentioned in the passage. Finally.
both the unabsorbed lactose and the acids increase the osmolality of the colon contents, so that more water remains
in the feces. This also increases diarrhea. Statement II is correct. The correct choice is D.
33. B is corrcct. The enzyme is produced by (isolated from) a mold, but only the enzyme is commercially
available-not the mold. This means that choice D is incorrect. Exogenous lactase does not induce the production of
anything in the intestinal mucosa, so choice C is also incorrect. Enzymes are broken down in the stomach, so lactase
cannot pass to the small intestine. Choice A is incorrect. The lactase is added to the food before ingestion. It breaks
down lactose in the food on the plate, in the mouth, and a bit in the stomach, before it is inactivated by stomach
acids. The correct choice is B.

34. B is correct. This question addresses your test-taking skills, as well as your ability to extract answers from the
passage. If the lactose is digested to glucose and galactose, the blood glucose would rise. This makes statement II
correct. There is no mechanism in the digestive tract for the uptake of lactose itseli so the clinician would see no
increase in blood lactose. Statement III is incorrect. Thus, eliminate choices C and D as incorrect answers. Finally.
we are left with statement I. Even if statement I is true, we can see at this point that choice B is a better answer.
since stateme.nt II is also correct. It does not say this in the passage, but if the test subject is lactose-intolerant, then
the gas produced by the bacteria in their colon would increase and should be detectable in their breath after lactos€
ingestion. Hydrogen is a common gas to test for this assay. The correct choice is B.

35. C is correct. This is a give-away question. Lactose is a sugar found only in milk, which is synthesized only in the
mammary gland. The correct choice is C.

Copyright @ by The Berkeley Review 26,4 The Berkeley Revier


Specializing in MCAT Preparation
Biology Metabolic Pathways Section VIII Answers

36' C is correct. Lactase is released from the brush border membrane of the duodenum, the first
twelve inches of the
small intestine. This is the only location in the body where lactase is secreted. The correct
choice is C.

37. c is correct. In the first reaction of B-oxidation, we use the oxidized form of FAD to remove two hydrogens from
the fatty acyl-CoA molecule.,The more hydrogens a carbon has on it, the more reduced
that carbon atom is. The
fewer hydrogen atoms the carbon atom has, the more oxidized it is. Therefore, the second
molecule (enoyl-CoA) in
the p-oxidation pathway is more oxidized than the fatty acyl-CoA molecule. This first reaction
could be thought of
as an oxidation reaction. Note that if the reaction is an oxidation reaction, then the
oxidized form of FAD is reduced
to FADHz. Also. if we remove hydrogen atoms from a molecule, then we can call that type of reaction a
dehydrogenation reaction What this means is that we can call this first reaction either
an oxidation reaction or a
dehydrogenation reaction. So far, this supports all four answers.

The second reaction involves water (H2o). Note that we add the water across the double
bond. Instead of cleaving
the molecule with water (a hydrolysis reaction), we are instead hydrating that
double bond. This type of reaction is
called a hydration reaction, and from it we get the hydroxyacyl-CoA deriiative. Choices
A ancl B are eliminated.
In the third reaction' we use the oxidized form.of the NAD@ coenzyme to remove the
hydrogen atoms associated
with the B-carbon of the hydroxyacyl-CoA derivative. If we lose hyirogen atoms, the molecule
becomes oxidized.
This is therefore another oxidation reaction. Since we are losing hydiogens, it is also
.uit"d u dehydrogenation
reaction. This reasoning eliminates choice D.

What about the last step? we use coenzyme A (i.e., coA-SH) to cleave the p-ketoacyl-CoA
molecule at the B-
position' CoA-SH contains. a thiol (SH) group. We cleave the molecule, which ii
like lysing the molecule (breaking
it apart). The reaction is indeed a cleavage, but more properly it can be called a ilriotlri, reaction.It
is not a
hydrolysis reaction, because we do not use water to cleavi that uona. However, in the
answir for choice C, thiolysis
or cleavage is not mentioned. These are one and the same thing, and they do occur.
The molecule breaks apart. ihai
much is obvious. But wh.at else is happening at this step? we ire adding a molecule
of CoA. When CoA reacrs with
the carboxyl group of the fatty acid, which has been shortened by two carbon atoms,
there is an esterification
reaction (see the second paragraph ofthe passage). The correct choile is c.
38. c is correct. Activation of the Ca fatty acid with CoA will cost I ATP. We thereby generate the fatty acyl-CoA
molecule, plus an AMP and pyrophosphate (PPi). The driving force behind this activaiion
is the hyclrolysis of ppi to
2 P1, which is equivalent to the energy contained in a high-energy phosphate bond (or another
ATp equivalent).
Hence, activation of the fatty acid costs us 2 ATp equivalents.

The activated fatty acyl-CoA proceeds through 4 rounds of


B-oxidation and is converted into 4 acetyl-CoA
molecules. Each acetyl-coA then enters the Krebs cycle. From the Krebs cycle, we get t
Crp, I FADHz, and 3
NADH molecules. The NADHs drop off their electrons and hydrogens at the elect.on-tlnsport chain
and produce 3
ATPs per NADH. The FADHz molecule does the same but prod-uces only 2 ATps per rinHz.
The GTp can be
converted to'Rtp in a separate reaction. Therefore, we find that 1 acetyl-CoA molecuie is
the potentially equivalent
of l2 ATPs' Since we have 4 acetyl-CoA molecules, we can in theory syhthesize + x tZ +S nrps.
=
Complete oxidation of the Cg fatty acid yields 3 FADHz and 3 NADH molecules from
the p_oxidation pathway
itself. Note that cleavage of the last 4-carbon compound gives 2 molecules of acetyl-CoA. Both
of these aceiyl-CoA
molecules enter the Krebs cycle, but neither one undergoes
B-oxidation again. Thus, from p-eli6ution we find that
the3FADHzmoleculesproduce3x2=6ATPs,andthe3NADHmolecilesproduce3x5=gATps.Fromthese
two reduced coenzymes, we get a total of l5 potential ATps.

Let's complete our ATP calculation. We add all the ATPs we can generate from acetyl-CoA,
FADFb, and NADH,
which is 48 + 15 = 63 ATPs' However, we invested 2 ATP equivalents to activate our fatty
acid. Therefore, we
produce a net total of 63 - 2 = 6l ATPs for the complete oxidation of the
Cg saturated futty u.ij. The correct choice
isC. r
39. B is correct. when Franz Knoop first ran a similar experiment in Germany in about 1904,
he attached a phenyl ring
to the omega (rr1 or the last) position of various fatty acids and then fed those fatty acids
to Jogr. If rhe fatty acid fed
to the animals had an even number of carbon atoms, he discovered large quuniiti., of pheiylacetic
in their urine
(choice B).

Copyright @ by The Berkeley Review 265 The Berkeley Review


Specializing in MCAT preparation
Biology Metabolic Pathways Section VIII Answers

Note that caprylic acid is an even-chained, saturated fatty acid. There would be some phenylbutanoic acid (choice i,
in the urine, but this could still be metabolized to phenylacetic acid. If the fatty acid fed to the animals had an i,a.r
number of carbon atoms, their urine would have contained benzoic acid (choice A). The urine would also h".-,e
contained some phenylpropanoic acid (choice C), but this compound could still be metabolized to benzoic acid. Tfo
correct choice is B.
40. A is correct. If we degrade caprylic acid through B-oxidation, we get four units of acetyl-CoA, all having labals.
Two of those residues have the label at the C- I carbon, as in choice B. One residue has label at both the C- I and C-l
carbons, as in choice D. One residue has label just at the C-2 carbon, as in choice C. Based on this alone, you rrug:r
be tempted to pick choice B, because that species would predominate. But, what about the carbon dioxide? Acen -
CoA is completely oxidized to COz and HzO in the Krebs cycle, which means that all the carbon atoms of a f.:3
acid can potentially be lost as COz. Since there are five labeled carbons in caprylic acid, there will be five molecr.rrru,
of laCOz being produced by the Krebs cycle. This makes choice A the best answer. The correct choice is A.
41. A is correct. The molecular formula of caprylic acid can be written as CsHtOOz. We are told that this compoun; s
completely oxidized to COz and HzO, a reaction we can write as follows (remembering to balance it):

CsH16O2 + ll 02 -------) 8CO2 + 8 Hzo


+
The 8 moles of water shown in this equation are produced at the end of the electron-transport chain when ox\ l:rl
(Oz) combines with the electrons and the protons. Then more water is produced when Abp combines with F- u
yield ATP. If the complete oxidation of one molecule of caprylic acid has a net yield of 61 molecules of ATp. rf;em
maybe the answer we want is 61 molecules of water:

6IADP+61P1+61H+ 61ATP+61 Hzo


+
But the overall equation for the complete oxidation of one molecule of caprylic acid is given by:

CgH16O2 + 1102 + 61 ADP + 61 P1 +61 H+ 8CO2 + 8 H2O r6lATP+6 1H2O


+ +
Looking at this equation you might be inclined to choose 69 moles of HzO (which would lead you to select choice l
as the best answer). But be careful. To regenerate the ADP and Pithat is needed to resynthesize the ATp, ware: $
needed to hyftolyze that phosphoanhydride bond between the beta and gamma phosphates of ATP. What this meanw
is that 6l of those 69 moles of water are continuously being recycled, so that new ATP molecules can be made. T c
only water that is produced in a net quantity is the 8 moles from the complete oxidation of caprylic acid at the le',sL
of the electron-transport chain. The correct choice is A.
42. D is correct. Fatty acids are rather nonpolar; and because they are nonpolar, they can be stored in an anhydrous slajur
(without water). The storage form of carbohydrates in animal tissue is glycogen. Recall that glycogen is compc,re,u
of glucose residues. Glucose is a very polar molecule. It has many hydroxyl groups associated with it. Glucose end
glycogen are both stored in a hydrated form. It turns out that on a gram-for-gram basis anhydrous fatty acids u 6n,
metabolized yield approximately six times more energy than hydrated glycogen.
When molecules like fatty acids or carbohydrates are oxidized, energy is abstracted in the form of protons (Ho i tild
electrons. Recall that these protons and electrons are dropped off at the electron-transport chain from redu;C
coenzymes like FADHz and NADH (+ Ho). In other words, if a carbon atom has more hydrogens attached to it, rhrsn
it has more electrons and protons to give to these reduced coenzymes, which in turn will have more to give to rn
electron-transport chain. Note that the methylene carbons 1-CHz-) of the saturated fatty acids have t*o hydro:o
atoms per carbon, whereas the majority of carbon atoms in a carbohydrate (like glucose) have just one hydrosm
atom attached to each carbon atom. This tells us that the fatty acid is more reduced or, if you wish, the carbohydremr
is more oxidized.

Copyright @ by The Berkeley Review 266 The Berkeley Revieu


Specializing in MCAT Preparatior
Biology Metabolic Pathways Section VIII Answers

What about the hydroxyl hydrogens of a carbohydrate? If the hydroxyl oxygen were to lose a hydrogen, it would do
so as a bare proton. The electrons stay with the oxygen atom of the functional group and give it a negative charge.
Putting all of this together, it is clear that choice D is the best answer. The correct choice is D.
43. D is correct. A decrease in the melting temperature of a fatty acid means that less heat is needed to disrupt the
interaction between any two fatty acid constituents of that type when they are next to one another in a triacylglycerol
molecule. In other words, the hydrocarbon chains of the fatty acids are not well ordered or well packed together. If
this is the case, there are a lot fewer van der Waals interactions between the individual hydrocarbon chains. Fewer
van der Waals interactions means that the carbon atoms in the hydrocarbon chain are farther apart. This could be
because of cis double bonds in the hydrocarbon chain itself. Cis double bonds cause kinks to form in those
hydrocarbon chains and tend to keep them apart from one another. The more kinks, the less chance there is for
stronger van der Waals interactions. Adding kinks through cis bonds means that the carbon atoms participating in
those cis bonds are unsaturated (i.e., they have fewer hydrogen atoms). What about unsaturation with trans double
bonds? Fatty acid hydrocarbon chains can have trans double bonds, but they tend to allow the hydrocarbon chains to
come closer to one another than cis double bonds. The van der Waals interactions are greater and the fluidity
decreases (compared to the cis conformation). The conect choice is D.

44. C is correct. There is much more skeletal muscle than liver tissue in a anyone's body. A 70-kg man has about 4O7o
of his weight in the form of skeletal muscle and a liver weighing 1.8 kg. This means the muscle contains 0.40 x 70 x
14 = 4OO g glucose, while the liver has 1.8 x 44 - 80 g. The correct choice is C.

45. C is correct. We can see that carbohydrate (CHO) oxidation increases from Day 3 to Day 4. The diet changes on
Day 4 to high CHO intake. Choice A is true. We can see from the chart that the glycogen stores are beginning to fill
before DNL kicks in. Choice B is correct. From the diagram, we see that net DNL occurs on Days 5-10. Choice D is
correct. Since we are looking for the false choice, choose C. We can see from Figure 1 in the passage that glycogen
stores are eventually saturated. The correct choice is C.

#. A is correct. Net DNL is occurring during the overfeeding of CHO. Since the liver packages the fat that it makes
into VLDLs, we would see an increase in that class of compounds in the blood of test subjects on this high-CHO
diet. The comect choice is A.
17. D is correct. In the question we are given the fact that glucose has a molecular formula of GHtzOe, and that it is
completely oxidized to COz and HzO. If a biological molecule like glucose is being oxidized, molecular oxygen
(Oz) is being used. This allows us to set up a rough equation that needs to be balanced:

C6H12O6 + 02 CO2 + H2O Unbalanced equation

C6H12O6 + 602 6CO2 + 6H2O Balanced equation

In the batanced equation we see that there are 6 COz molecules produced for every 6 Oz molecules consumed.
Therefore, the ratio of COz to Oz is 6 to 6, or 1.00. The correct choice is D.

{8. D is correct. If the hydroxyl (OH) group at the anomeric carbon is down (i.e., below the plane of the ring), the
position is alpha (cr). Note that in the diagram every anomeric carbon has the OH group below the plane of the ring.
Therefore, the linkages for I and II are both in the o-position. This allows us to eliminate choices A and B. Linkage
II begins at the C-1 carbon and ends at the C-4 carbon. This is an s-1,4 linkage. Linkage I begins at the C-1 carbon
and ends at the C-6 carbon. This is an G-1,6 linkage. We can eliminate choice C. The correct choice is D.

{9. A is correct. Both the stable and tadioactive forms of carbon can be used as tracers, so ignore the 13C and 14C
tricks. Glucose can proceed to glycogen by the direct pathway. Alanine and pyruvate proceed to glycogen by
gluconeogenesis. Palmitate is a fatty acid, and it is not made into glucose. The correct choice is A.

50. A is correct. From Figure 1, we can see that the first few days of excess CHO feeding mainly increase the glycogen
stores and not the fat stores. This eliminates choices B and D. The focus is on glycogen. Depletion of glycogen by
exercise and a low-CHO diet, followed by a high-CHO diet might increase glycogen stores and thus increase the
endurance of the athlete. The correct choice is A.

Copyright @ by The Berkeley Review 267 The Berkeley Review


Specializing in MCAT Preparation
Biology Metabolic Pathways Section VtrI Answerc

51. B is correct. The important differences between eukaryotic and prokaryotic cells cannot be emphasized enough-
Eukaryotic cells have mitochondria, so they have a mitochondrial matrix. They also have cytoplasm, in which Sc
mitochondria and Golgi complex reside. And they have an endoplasmic reticulum, extending from the cell's nucleus.
The only item here common to the prokaryotic cell is the cytoplasm. Prokaryotic cells do not have mitochondril"
Golgi, or an endoplasmic reticulum (because they do not have a nucleus). Therefore, in prokaryotes, glycolysir
occurs in the cytoplasm. The correct choice is B.

52. A is correct. Besides knowing the general differences between prokaryotes and eukaryotes, it is also good to k
some specific fact about each type of cell. Lactic acid is the end product of glycolysis under anaerobic conditi
Under aerobic conditions, the end product of glycolysis is pyruvate (which can then enter the citric acid cycle).
our cells were deprived of oxygen, then lactic acid would be the end product in each one of the four choi
However, there is only one best answer here, not four. Therefore, we must assume that the question asks about cr
that are utilizing oxygen. If oxygen is present, then glucose is completely oxidized to CO2 and HzO (by way
glycolysis, the citric acid cycle, electron transport, and oxidative phosphorylation). But if lactate is being prod
in a cell and is not being oxidized to COz and HzO in the presence of Oz, then either the citric acid cycle, ele,
transport, and oxidative phosphorylation are not being utilized or those pathways are missing from the cell. It
out that mature red blood cells lack not only a nucleus, but they lack membrane-bound organelles like mitoch<
as well. The correct choice is A.

53. D is correct. All we need to do is look at each step in the glycolytic pathway and see what general type of reactim
catalyzed. There are two phosphorylation reactions, one at Step 7 and one at Step 10. Eliminate choice A. Therc a
isomerization reactions at Step 2 and at Step 5. The only enolization reaction occurs at Step 9. Only ligaril
reactions are left. Ligation reactions are catalyzed by enzymes called ligases. The word ligate means "to bind." h
ligation reaction two molecules are joined together, at the expense of hydrolyzing ATP to ADP and Pi. The
choice is D.
54. Biscorrect. If weassignhydrogenavalueof +l andoxygenavalueof -2,theoxidationstateof carbonmusil
whatever it takes to achieve charge neutrality. Consider the C-l (aldehyde) carbon. There is no change
electronegativity between the carbon atoms C-l andC-2. However, the C-l carbon bears a hydrogen (+l) and
oxygen (-2). This adds up to -1. Therefore, in order to be neuffal, the C-l carbon must have an oxidation state of

Consider theC-2 carbon. The hydrogen directly attached to the carbon brings a value of +1. The oxygen of
hydroxyl group has a value of-2, and the hydrogen ofthe hydroxyl group has a value of +1. This adds up n
Therefore, the C-2 carbon must have an oxidation state of 0. In fact, the oxidation state of the C-3, C-4, anO
carbons are each 0.

Consider the C-6 carbon atom. The two hydrogen atoms directly attached to the C-6 carbon bring a value of +2-
hydroxyl oxygen is again -2 and the hydroxyl hydrogen is again +1. This adds up to +1. In order to be neutral,
6 carbon must have an oxidation state of -1. Adding up all the oxidation states of the six carbon atoms gives a
oxidation state of 0. The correct choice is B.
55. A is correct. We need to consider the diagram outlining glycolysis. In particular, we need to focus our atte
Step 7. The 2,3-BPG shunt bypasses the enzyme phosphoglycerate kinase, which at Step 7 in glycolysis
1,3-bisphosphoglycerate to 3-phosphoglycerate. In this reaction, ADP is converted to ATP; and since this
occurs twice per glucose molecule oxidized, we should generate 2 ATPs at this step. But we are bypassing this
so we do not get those 2 ATPs. In fact, the only ATPs that are synthesized are those at Step 10 in the
phosphoenolpyruvate to pyruvate. The 2 ATPs made here repay the 2 ATPs that we used as investments at
and Step 3 in the first part of glycolysis. Therefore, the net production of ATP is zero (0). [You may wonder
red blood cell can survive, if glycolysis yields zero net ATP. It turns out (which was not important for this q
that about 257o of the glucose metabolized in the glycolytic pathways of the red blood cells is converted to 2,
Recall that this molecule is used to stabilize the deoxy state of hemoglobin. The remainingT5Vo of the
utilized by the glycolytic pathway can generate those much needed 2 net ATPs.l The correct choice is A.

56. B is correct. If there is a mild deficiency in this enzyme, PEP cannot be converted to pyruvate as readily-
begins to increase in concentration as do other glycolytic intermediates behind it. An increase in 3-phosphogly
leads to an increase in both 1,3-bisphosphoglycerate and 2,3-bisphosphoglycerate. Recall from our discussi
hemoglobin that 2,3-BPG stabilizes the deoxy form of hemoglobin and results in a decrease in the a
hemoglobin for oxygen. A decrease in its affinity for oxygen means that more oxygen has been released

Copyright @ by The Berkeley Review 264 The Berkeley


Specializing in MCAT
Biology Metabolic Pathways Section VIII Answers

tissue. The oxygen-hemoglobin dissociation curve would then shift to the right. Since the deficiency is mild (rather
than extreme), the shift is only slightly to the right. Also, note that a deficiency in this enzyme leads to a decreased
synthesis of ATP in red blood cells. The correct choice is B.

57. C is correct. This can be worked out by using the Punnett square shown below. We can use the notation large p for
the good gene and small p for the defective gene. If the trait is expressed, the genotype is pp. If it is not e>riressed
the genotype is either PP, Pp, or pP.

P p

p Pp pp

p Pp pp

There is a 50Vo chance that a child of this couple will express the trait. The correct choice is C.
58. B is-correct. The glycolytic pathway is not a one-way street. There are reversible reactions, as indicated by the
equilibrium arrows in Figure 1 of the passage. The entire reaction series follows a sequence. The first event is a
phosphorylation, followed by a dephosphorylation. If we study the glycolytic pathway from glucose to pyruvate,
we
can,count eight 8 phosphorylation events. There is one at Step 1, one at Step 3,
t*o ui St"p 6]two at Step 7, and rwo
at Step 9. At Step 7, ADP is phosphorylated to make ATP. At rhe iame time, t,3-bisphosphoglycerate is
dephosphorylated. But no reduction is involved in this step. This is the clue rhat something is a bil lut of the
ordinary.
What we are looking for is a phosphorylation step that is immediately followed by a dephosphorylation-and-
reduction step. The only oxidation-reduction in glycolysis occurs where we see the .o"nry*" NAD. If we examine
glycolysis in the reverse direction starting with 3-phosphoglycerate, we will see that in order to form 1,3-
bisphosphoglycerate a phosphorylation event must occur. Even though no ATP is indicated to be going to ADp at
the equilibrium arrows of Step 7 for this reverse reaction, it must be happening-otherwise, 3-pf,ospioglycerate
could not be phosphorylaje.d to 1,3-bisphosphoglycerate. Notice that onci 1,3-bisphosphoglycerite is pro-duced, it
can be dephosphorylated it (which means the loss of Pi) and reduced to glyciraldlnyle-:-pnosphate. This is
indicated by the reverse of Step 6. The reverse of Step 5 is an isomerization reaction that converts glyceraldehyde-3-
phosphate to DHAP (see the structure below).

cH,-g-p9.:-
Dihydroxyacetone
phosphate
eo
I

I
cH2-oH

These three teversed steps in glycolysis do occur, and they are part of the pathway called gluconeogenesis, the
synthesis of $ucose from specific precursor molecules. The correct choice is B.

59. B is correct. Many of the L-amino acids can lose their s-amino group in a transamination reaction. This reaction is
carried out by enzymes called transaminases or aminotransferases.In this reaction, the g-amino group of leucine is
transferred to o-ketoglutarate (a component of the Krebs Cycle), an o(-keto acid. This generatei L-glutamate and
another cr-keto acid, which in this case is cr-ketoisocaproate.

o o o o
coo coo coo
I
@ ?oo @t I

o=c H3N-C- H H1N-C-H o=c


| .. I -l
CH: + cH- * I

cH,
t- t' Aminotransferase
CH,
t' t-
CH, HjC- CH CH, HlC- CH
l-6 I t-g -l
coo CH, coo CH:
(I-Ketoglutarate Leucine Glutamate 0-Ketoisocaproate

r;ight @ by The Berkeley Review 269 The Berkeley Review


Specializing in MCAT Preparation
Biology Metabolic Pathways Section VIII Answers

Note that there is no net deamination in this reaction sequence. Transferring the amino group to glutamate allows
glutamate either to use that amino group in biosynthetic reactions or to eliminate that amino group (via the urea
cycle) as a waste product.

We can eliminate a hydration reaction as a possible answer choice, because we are not adding water to leucine. Thc
water molecule HzO has not been added (does not appear) on s-ketoisocaporate. Also, we are not adding a CQ
residue by carboxylation. nor are we removing hydrogens by dehydrogenation in this reaction. The correct choic
is B.

60. B is correct. This question asks you to remember how the human body disposes of nitrogen as a waste produa-
Ammonia is quite toxic to animals. Since the pKs of ammonia is about 9.5, we would, at physiological pH, exp€cr m
find this molecule present as the ammonium ion. The amino nitrogen is excreted as the ammonium ion by
aquatic organisms. These animals are referred to as being ammonotelic. Reptiles and birds excrete the ami
nitrogen as uric acid, and are therefore referred to as being uricotelic. Human beings and many other terrestri
animals excrete the amino nitrogen as urea, and because of this are referred to as being ureotelic. Recall that urea
produced in the urea cycle. Glutamate is simply an intermediate in the pathway for nitrogen elimination in
The correct choice is B.
6t. B is correct. Adding HzO to the reaction would either hydrate the reactant or hydrolyze it. Neither of those
is occurring. NADH and FADHz are both reduced coenzymes. Adding either to the reactant means
hydrogens across a carbon-oxygen double bond or across a carbon-carbon double bond, respectively. Again, we
not see either case here. This leaves, by the process of elimination, ATP as our choice.

S-CoA
o I
S-CoA
I
HCOj o=c
o=c I
C-H
I
ll
C_H H3C- C
il 'l
HlC- C (A biotin-dependent carboxylase enzyme) CH.
I
CH:
1"6
coo
p-Methylcrotonyl-CoA p-Methylglutaconyl-CoA

COz is a gas that tends to diffuse away. Trapping that COz and adding it to the reactant (via the biotin-dependem
enzyme), requires energy in the form of ATP. Carboxylation reactions involve energy. The correct choice is B-

62, C is correct. If HCO3o reacts with the reactant to add COz, then an OH moiety is left over. A hydrogen (Ht
one of the methyl groups of the reactant can combine with this OH group to produce HzO. Since there is no
group on the reactant, none can be formed in the products. Similarly, since there is no COz group on the
decarboxylation reaction will not yield COz in the products. Finally, the addition of a carboxyl group to the
(via HCOf9), does not form H2CO3 (carbonic acid). The correct choice is C.

63. C is correct. The enzymes mentioned in the question are proteins. The prefix apo- rcfers to the protein
the enzyme, while the prefix holo- refers to the enzyme having all of its necessary parts (cofactors,
prosthetic groups, etc.) in order to function. Holocarboxylase synthetase attaches biotin to a specific
enzyme. This makes the apocarboxylase enzyme complete (i.e., we could now call it a holoenzyme itselfl and
to carry out a carboxylation reaction. If the holocarboxylase synthetase enzyme is deficient, biotin
attached to the apoenzyme, and the reaction does not occur. This means that the substrate that accumulatcs
rapidly is the reactant for this reaction, which is p-methylcrotonyl-CoA.
Neither acetyl-CoA nor p-methylglutaconyl-CoA can be formed, since we cannot get past the reaction
Enzyme 4. Therefore, they do not accumulate. cr-Ketoisocaproate does begin to accumulate, but not as r
methylcrotonyl-CoA. The correct choice is C.

64. C is correct. If this reaction were a thiolysis, we would need to add the element of sulfur (S) across the
was broken. Similarly, if it were a hydrolysis, we would need to add a molecule of water (HzO) across the
was broken. We do not see this in either case, even though we had a lysis in both situations.

Copyright @ by The Berkeley Review 270


Biology Metabolic Pathways Section VIII Answers

The product of an aldol condensation is a molecule with both an alcohol


and an aldehyde functional group. These
two groups are the source of the term -aldol, and these are exactly what we would
have if we removed the CoA-S
portion of B-hydroxy-p-methylglutaryl-coA and added a hydrogen atom. The
mechanism for this is shown below:

o () o
fl Oil
ENZ-Base: H - H2c- c- S-CoA H2C-C- S-CoA e H)C-C-
I
@
S-CoA + ENZ-Base
- H

Acetvl-CoA Enolate ion

( o /'----- -o o
oOOC- CH2 xtt)/ fl alil
oo
C CHr
- - H,C- C- S-CoA + "ooc- cH, -c - cH, - C- S-coA
t-
Acetoacetate CH,
An alkoxide ion

ooOC- CH2
?"/---\ fi
-l- a"r-C- S-CoA H:
\., otil
oHO
Base-ENZ OOC- CH2 - 9- CHz -C - S_CoA + :Base-ENZ
I
cH, CH..
An alkoxide ion -
B-Hydroxy-B-methylgluraryt-CoA

However' it is evident the reaction outlined in Figure I in the passage is proceeding in the direction of a reverse
aldol condensation. The correct choice is C.
65' B is correct' Glucose. enters the glycolytic pathway and is converted to pyruvate in
a series of ten reactions.
Pyruvate can react with coenzyme A (coA) in a decirboxylation reaction
to glve u""tt-coe, which is one of the
end products of the degradation of leucine outlined in Figure 1. once acetyl-ioA
is fo'rmed, it can enter the Krebs
cycle and be completely oxidized to coz and Hzo. But acetyl-CoA cinnot d;ur;i
enrer glycolysis, electron
transport, or oxidative phosphorylation.

we do find that the carbon atoms of acetyl-coA can end up in the glycolytic intermediates
through the process of
gluconeogenesis. Be aware,th_at acetyl-CoA cannot be used as.a pr"cu.sbr
to yield a net synthesis ofglucose in
mammals' Recall that acetyl-CoA is a two-carbon compound. Whln it enters
the Krebs cycle,2 carbon atoms are
lost as Co2; yet those 2 carbon atoms are not necessariiy the same 2 carbon atoms
derived fiom acetyl-CoA. This
means that if we were to label acetyl-CoA, we would find some of.it in the glycolytic
intermediates. s:re just would
not be able to get a net conversion ofa two-carbon compound to a six-carborisugar.

The energy tied up in the bonds of acetyl-CoA can be transferred (via the Krebs
cycle) to the coenzymes NADH and
FADHz' These coenzymes can then transfer this energy, in the form of electrons and 'hydrogen
ions, to tr,e eLciron-
transport etrain and eventually to the process of oxidative phosphorylation, where
choice is B.
eip is generated. The correct

66. B is correct. The normal digestion of trehalose would not lead to the production of hydrogen,
so choice A is
incorrect' No hydrogen is given off from interactions with bicarbonate or with gastric
.ont"ntr. Choices C and D are
therefore incorrect. The reason for.increased hydrogen is the passage ofundigeJed trehalose
from the small intestine
into the colon, where many bacteria begin to metabolize it. ttrey
iroduce thE hydrogen uy i".menting the trehalose.
The correct choice is B.
67. c is correct' Exhibiting a dose-response effect means that every time the dose is increased by a certain
increment,
the subject's response follows a similar patterns. ofincreases. suUlect 3 has an increase
of 10 ppm of hydrogen for
every l0 g iribrease in trehalose, so choici C looks like the best answer. Subject 2
shows no dose-response effect, so
choice B is incorrect. Subject I shows no particular pattern, either. Choic" e i, in"orr."i
SrU:*t 4 did not complete
the study. Choice D is incorrect. The correct choice is C.
68. f) is correct.Figure I shows us that trehalose is made of two glucose residues joined in cr-l-l linkage. The correct
choice is D.

--rpyright @ by The Berkeley Review 271 The Berkeley Review


Specializing in MCAT preparation
Biology Metabolic Pathways Section VIII Answers

69. D is correct. This is one of the few times that "I can't tell from this passage" is the correct answer. Although
is a disaccharide, this does not mean it behaves in the body exactly as trehalose does. Choice A is incorrect.
may or may not be a commoner component of the diet, depending on what culture you are examining. But you
can't predict someone's lactose tolerance based solely on their degree of tolerance for another sugar. Choices C
B are incorrect. The correct choice is D.
70. A is correct. The enzyme trehalase is found in the brush border of the human duodenal mucosa. It may be present
large or small quantities, depending on one's genetics. It is not induced by eating trehalose like the lac operon
induced by lactose in bacteria. A person's tolerance for trehalose is not the result of their habitual consumption
trehalose-containing foods. Choices C and D are therefore incorrect. Subject I has almost no increase in h
upon ingesting increasing doses of trehalose. This means the digestion of this sugar by trehalase is more effici
The subject's tolerance for doses of trehalose is high. The correct choice is A.

7t- B is correct. A centrifuge separates the component parts of a mixture by density, so choice A is incorrect. A
spectrometer characterizes compounds based on masses of isotopes, so choice C is incorrect. Extraction
compounds based on their solubility in organic solvents, making choice D incorrect. The correct choice is B.

72. B is correct. Up to a certain dose, Subject 2 does little bacterial fermentation of trehalose. But after that
point, a threshold, their enzymes can handle no more, and there is a suddenly big increase in the amount of h
in their breath. The correct choice is B.

73. A is correct. In the initial phases of exercise, the oxygen consumption is not high sufficient to provide
oxygen to the body's muscle tissues for the oxidation of fuel molecules through the TCA cycle and ox
phosphorylation. During this brief period of lowered oxygen relative to demand, only glycolysis is carried out
TCA cycle enzymes are constitutively on; they do not need to be activated by exercise. Thus, choice B is i
Fatty acids are always metabolized aerobically. They cannot enter glycolysis, which is the only anaerobic
oxidizing pathway. Choice C is also incorrect. Glycolysis can keep pace with the energy demands of
intense exercise, since this is what occurs. Choice D is incorrect. The correct choice is A.

74. C is correct. The maintenance of glucose homeostasis has many complex and interacting safety checks. Sina
brain and nervous system are both almost entirely dependent on glucose as their fuel source, blood glucose
maintained within a fairly narrow range for all times, during a variety of activities. This may be accompli
through the intake of dietary glucose, through the use of glycogen stores, and through gluconeogenesis. Choi
B, and D are incorrect. The correct choice is C.
75, A is correct. Oleate is a fatty acid, which makes up abofi 40Vo of the circulating fatty acids. Resting muscle,
see in Table 1, uses FFAs as fuel primarily. Glycogen is not used at rest, so choice D is incorrect. p-hydroxy
acid is a ketone, used during fasting and very strenuous, sustained execise. Choice B is also incorrect. Lactic
a waste product of metabolism in the muscle. It is produced during exercise and exported to the liver for
gluconeogenesis. This ,means choice C is incorrect, too. The correct choice is A.

76. D is correct. Since glucose is a small, water-soluble molecule, it does not need a carrier molecule. Usually,
proteins such as albumin serve as carriers for large, insoluble molecules. Choice A is incorrect. There is no
binding protein, so choice B is incorrect. And hemoglobin carries oxygen. Choice C is also incorrect. The
choice is D.

77. B is correct. Insulin synthesis is a response to dietary carbohydrate entering the blood. Its function is to
glucose transfer from the blood into cells and also into the glycogen stores. Insulin levels in the blood are low
exercise, so choice A is incorrect. Glycogen phosphorylase is an enzyme involved in glucose maintenanc€"
hormone. Choice C is incorrect. Oxytocin is a hormone involved in labor and childbirth, so choice D is i
too. Glucagon promotes glycogenolysis, so that muscle can use the available glucose for fuel and the liver can
down and release stored glucose (glycogen) to the blood supply. The correct choice is B.

78. A is correctlCarbohydrates provide about half the energy per gram that lipids do, so choices B and C ac
incorrect. Proteins, or amino acids provide about the same amount of energy value per gram as ca
Choice D is incorrect. The correct choice is A.

79. D is correct. Muscle glycogen contributes none of the energy demanded by the body during the first four
exercise. Choice A is false. Gluconeogenesis increases during exercise, as you can see from Table 1. Therc

Copyright @ by The Berkeley Review 272 The Berkeley


Specializing in MCAT
Biology Metabolic Pathways Section VIII Answers

reason to expect it to decrease after four hours. Choice B is incorrect. Adipose tissue has about 100,000 kcal of
stored energy in the form triglyceride. It is released as free fatty acids and glycerol into the blood. There would be
no significant change in available fat stores for exercise that continues after four hours. Choice C is incorrect. Liver
glycogen's contribution to blood glucose does drop over the four-hour exercise period, however, and that decrease
would probably continue. The conect choice is D.

80. C is correct. Glutamine is the major form of ammonia transport and is carried in the blood to the liver. The a-
nitrogen is then removed as ammonia in the liver mitochondria. At a neutral pH, most of the ammonia that is
released is in the form of the ammonium ion (NH+@). This toxic ion is converted to urea in the urea cycle and later
exported to the blood from the liver. From there, it travels to the kidneys and eventually eliminated in the urine. The
correct choice is C.
81. A is correct. Urea contains two nitrogen atoms. One comes from the condensation of NFI+CI with HCOre and the
other comes from aspartate. In order to place urea on the product side of the equation, there must be a balance of 2
nitrogens on the reactant side. This allows us immediately to eliminate choices C and D. We can eliminate choice B
also, because if we add aspartate on the reactant's side of the equation, would have 3 nitrogen atoms on that side of
the equation and only 2 nitrogen atoms on the product's side of the equation. The correct choice is A.
82. D is correct. We know one of the nitrogen atoms comes from the ammonium ion. As shown in the diagram of the
urea cycle, the ammonium ion nitrogen is found in the structures of carbamoyl phosphate, citrulline,
argininosuccinate, and then arginine before it gets to urea. Where does the other nitrogen atom come from? The only
other reaction entering the urea cycle comes from aspartate, which is formed from the reaction of oxaloacetate and
glutamate. Glutamate passes its nitrogen to oxaloacetate and in the process is converted to a-ketoglutarate and
aspartate. There are no nitrogen atoms in oxaloacetate or in the citric acid cycle. The correct choice is D.
E3. B is correct. Carbamoyl phosphate is formed from the reaction of the ammonium ion with bicarbonate (HCO3o).
The bicarbonate ion is carbon dioxide (COz) in disguise. The vast majority of metabolic COz comes from the citric
acid cycle (Krebs cycle). Carbon dioxide is not produced in glycolysis, from the electron-transport chain. or in the p-
oxidation of fatty acids. The correct choice is B.
&{. C is correct. The only two choices we have that allow a stable reaction with the ammonium ion are choice B
(aspartate) and choice C (glutamate). The ammonium ion reacts at the side chain carboxyl group to form the amide.
The more favorable reaction is the one with glutamate, because its side chain carboxyl group is farther away from
the positively charged cr-amino group. There is less steric repulsion. Once the ammonium ion reacts with glutamate,
the amino acid glutamine is formed. Glutamine is a neutral, nontoxic compound. It can readily pass through cell
membranes and eventually into the blood, where it is carried to the liver for removal of the nitrogens. The correct
choice is C.
Li. A is correct. We need to be careful of words here. The word amidationrcfers to the introduction of an amino group
into an organic compound. However, we want to remove an amino group from an amino acid. Therefore, the
reaction is a deamination. This points to choice A and choice C. A transamination reaction involves the transfer of
the amide nit?ogen (from glutamine) to another compound, so eliminate choice B. A dehydrogenation reaction
involves the removal of hydrogens (not nitrogens) from an organic compound. Eliminate choice D. Is the removal of
the amide nitrogen an oxidative or reductive deamination? In the passage, we see the conversion of glutamate to o-
ketoglutarate. The removal of the amide there results in a carbonyl group at the s-carbon. This is an oxidation
reaction. The correct choice is A.
ffi. B is comect. Draw a pedigree as shown below. This indicates how a gene is transmitted from one generation to the
next. Let's represent the disorder by aa.

I
-II
^FI;]
*
III ""ii:'ii';,",:';il:;""'D "l "" | |

273 The Berkeley Review


Specializing in MCAT Preparation
Biology Metabolic Pathways Section VIII Answers

Individuals who do not express the trait are represented by either AA, Aa, or aA. The man's mother had ASA,
so trer
genotype must be aa. His father did not carry the gene, so his genotype is AA. This means that
the man's genotype ir
either Aa ot aA.It does not matter which he is, because he still doli not express the trait. Now, the
man mai.i., e
woman who has the disorder, so her genotype must be aa.What is the probability that their child will
be affected!
Since we do not know the sex of the child, we represent it by a diamond in the pedigree. A simple punnett
squarg
tells us that the probability the child will express the trait is 507o. The correct ctroice is B.
87. A is correct. By increasing the synthesis of glycine, we are pulling more of the ammonium ion out of solutim*
thereby decreasing its concentration. If we increase the syntheiis of aspartate (by some mechanism), thc
concentration of ammonia will begin to fall, because we are passing those nitrogens to tire urea cycle and eveniua$p
to urea. However, in the reaction drawn in the question, glycine is reacting with sodium benzoate, not asparmn
Based on this mechanism, we would not have a way to increase the synthesis of aspartate. Eliminate choice
B. \||l
cannot increase the concentration ofurea, because th^ere is a deficiency in the arginaie enzyme that converts
argi
to ornithine and urea. Eliminate choice C. Again, if we were to increase the Krebs cycle intermediates to prl
more aspartate, and if aspartate were to drop off its nitrogens at the urea cycle, there w-ould still be a lack of
argi
and a reduced production of urea. The intermediates would back up, and the concentration of ammonia *,
increase (hyperammonemia). The best way to eliminate nitrogen is by some other route. The correct
choice is A

88. C is correct. Think of gluconeogenesis as having two roles: maintenance of blood glucose by the liver
regeneration of NAD in the muscle tissue. Lactate is an end product. When lactate is formEd from
pyruvate, N-.:
is oxidized to NAD@. This NADo is free to reenter glycolysis. under aerobic condirions, NADH would
be oxi
in the.electron-transport chain. However, this metabolic detour through lactate is necessary during anae
conditions. The purpose of gluconeogenesis is to maintain constant blood glucose levels, especially duriig
vig
exercise and during a fast. Following a meal, gluco_se is provided by the diet, so choice D is incorrect.
D;ing
or studying, little anaerobic activity is happening. Choices A and B are inconect. Exercise involves rapid gl
to produce energy. vigorous exercise is anaerobic. The correct choice is c.

89. B is correct. Pyruvate is first carboxylated to make oxaloacetate (OAA). This is the first step in glucone
Pyruvate dehydrogenase is involved in the conversion of pyruvate to acetyl-CoA and COz. ihoice
.q is i
Lactate dehydrogenase is the enzyme the uses NAD(H) to interconvert pyruvate and lactate. Choice
C is i
Finally, there is no lactate carboxykinase, so choice D is false. The correct choice is B

90. D is correct. Pyruvate is converted to lactate, not the reverse, during anaerobic conditions, so statemen[
incorrect. Lactate is not converted to alanine, but pyruvate is, during anaerobic conditions. Statement II is
inc orrect L ac rarei' d;';:;".;;#;;;il: #i'
i#;", ;" i,T :i : lffi;TJiJl"in"#
" III is inconect. The correct choice
alphabetically. Statement is D. "iJT"}:::# 5
91. D is correct. So, you've never heard of the reverse Cori cycle? Pay attention to your intuition, because in this
you to thecorrect answer..Ther"_o
las go1lt1ng lo reverse Cori cycle. Muscle tissue neverreleases glucose i
blood. It does not have the enzymes to do so. Choices A, B, and C are incorrect. The correct choice is D.

92. A is comect. NAD@ is required in glycolysis by the enzyme glyceraldehyde-3-phosphate dehydrogenase. q.'
this cofactor, the enzyme could not function, and glycolysis would stop. Ctycotysis produces a net
lain of 1 ,
so it would not stop due to a lack of ATP. Choice C is incorrect. Since glycolysis is a fixed pathway of enzy
cannot change its ATP output or its cofactor requirements. Choices B and D are incorrect. The correct choicr

93. A is correct. Since the body produces alanine, by definition it must be nonessential. Eliminate choices B
Since the amino group is what distinguishes pyruvate from alanine, reduction is not the metabolic ormr
interconverts them. It is transamination. Choice D is incorrect. The correct choice is A.

94. B is correct. This is an anatomy question. Usually an artery flows into an organ, and a vein flows out of the
However, the hepatic portal vein (choice C), is a specialized venous system that carries nutrients absorbed fr
. i;:;ilJir*d;;;;d;* is inro rhe liver for rhis particutar
system. This was mentioned in the passage so choice C could be easily eliminated as a possible correcr
There is no hepatic portal artery, so choice D is wrong. Choice A is aiso incorrect, because the hepatic a
bringing blood to the liver. The hepatic vein drains the liver and carries glucose and other nutrients releasod
liver to the rest of the body, making choice B the best answer. The correct choice is B.

Copyright @ by The Berkeley Review 274 The Berkeley


Specializing in MCAT
Biology Metabolic Pathways Section VIII Answers

95. B is correct. Carbohydrate that can be digested by the body's own enzymes (i.e., digestible carbohydrate) is
digested in the small intestine and does not reach the bacteria in the colon. Choices C and D are incorrect. Hydrogen
in the breath during the lactulose test is generated by bacteria in the colon as they break down otherwise
nondigestible carbohydrates, like lactulose. Since her breath hydrogen did not rise, her bacteria were not of the type
that ferments lactulose. A change in breath hydrogen was the measured variable in the study, so she could not
participate. Choice A is incorrect. The correct choice is B.

96. B is correct. The points for the breath hydrogen produced following the starch blocker are not statistically different
from the placebo points. Statement I is correct. The rise in breath hydrogen following lactulose administration was
after the l2O-minute mark. Statement II is incorrect. Starch in the small intestine is digested by o-amylase. If this
enzyme does not work, then the starch passes to the colon for fermentation by bacteria. This fermentation gives off
hydrogen, which is measured in this experiment. In people who have enough bacteria in their colon to ferment
lactulose, there are only two possible outcomes: either c-amylase worked, and no extra hydrogen was produced, or
c-amylase did not work, and the colon bacteria produced measurable hydrogen. Since there is no difference in the
amount of hydrogen gas subjects produced in the breath between a meal with a placebo and a meal with a starch
blocker, we can assume that the amylase was not inhibited. Statement III is correct. The correct choice is B.

97. D is correct. The digestive enzymes trypsin, chymotrypsin, elastin, amylase, and lipase (among others) are produced
by the exocrine portion of the pancreas. They are secreted into the pancreatic duct and then empty into the small
intestine. Choices A, B, and C are incorrect. The correct choice is D.

98. C is correct. The figures show no significant differences between the placebo and starch blocker tests for insulin
and glucose levels. There was essentially no change due to the administration of the starch blocker. It did nothing to
inhibit starch digestion. Choices A, B, and D are incorrect. The correct choice is C.
99. B is correct. Starch is a plant's storage form of carbohydrate. Turkey is not a plant, and its storage form of
carbohydrate is glycogen, just like other animals. Choice A is incorrect. Butter is mainly fat, not carbohydrate.
Choice C is incorrect. Orange juice contains carbohydrate, but predominantly in the form of simple sugars. Choice
D is incorrect. The correct choice is B.
100. A is correct. Simple sugars are the monosaccharides, and they are absorbed by the body intact, without any
digestion. Amylase is not needed for their digestion. The inhibitor has no secondary effects that destroy simple
sugars. Even if a real starch blocker existed and blocked amylase in vivo, simple sugars would not be affected by it.
Eliminate choices B, C, and D. The correct choice is A.

i.:pyright @ by The Berkeley Review 275 The Berkeley Review


Specializing in MCAT Preparation
$t$r6
A. Classical Genetics
S€ctid,n',,,,,IX l. Patterns of Inheritance

. :i , , '5.r :t :,Gene.tiC LOei &,,j{1191*,$.,,,,,..1,',.1


Genetic
:

4. The Pedigree
B. Genetic Information
',1ntu*mation
155 l51.11
l. Central Dogma of Molecular Biology
2. Function of Nucleic Acids

'"'** C. DNA Synthesis


,'*-'
,
t. DNA Polymerase ..

l51q l4p , l{1q,1,19, ., ,, 2. DNA Replication

:5
/lil{ ;;:lJ',, /:il| ('
14111 ,, I' 14N,14N,. ': 1419,, 1419"
"
'',' ,taN,,15
Practice Passages Er Answers
:. . :r: r1:r:: i::r: I r r:j,i.r:t:r i. I iri, I r,

lil il ;i*;"; ill


;::,,r[,[.
:'.
,'1[

14Nlr4X I l5Nll4N
' :';',lgqo ,' , Savo ,

Direction of Sedimentatioa' r:*

tfre
PEffi
L)n.n.v.r.-ilff
Specializing'in' MCAT Preparation
Gen€tic Information
Top lO Section Goals

Ee famitiar,r,yith the transmission of genetic traits.


Understanf how genetic traits are transmitted from one generation to the next. Think of meiosis in :
this context, especially genetic recombination.
0

uw Be familiarwitlr the'work of Mondel. lld

1lt

Pioneering wolk
Trace the Gregor Mendel^and his garden
9f peas. Understand how to ,rr"l prrn.r"tt
tS
square, and De able to obtarn srmple ratros of genetic events.
W

ttfi
M
r{m

rffiil

frh
mr
Don't get lost in the details of difficult pedigrees. At most,_be able to take a pedigree back to three
generations. Be able to follow the lineages oi the individuals. r o M

ffit
]WM
ih
tffiun
r@U'
\me
k
TWW

tu
fomr
Eq familiar 4th the gener{_Irrocess of DNA replication.
Don,tget1ostinalltheenzymesinvolvedin.DNAreplication.J"'
Mru,u
the overall process, and feel comfortable with some of the names and structurbs.
Mm

UB Understand,how tlre chain reaction works.


The discovery of the polymerase chain reaction ranks among the greatest scientific air.ouetler of
iffilum'

'uffiii$
\\e1$\\trn\tny.\\ert\astbrtlriurrl ar\rt\esun\h\spru,e{s,an{\\rs ter\axr \obe on \heMCN\. Wuil'\l
,lwlitl

t- ,,&i,rtlru

Be able to relate the discussions of this section with genetic expression.


@B Besides DNA replicatiory there is also DNA transcription and mRNA translation. Once a cell divides,
{i!*'-
Mrru'

it must synthesize proteins in order to survive. You will need to understand this process. ,&:r
flfF*
Biology Genetic Information Patterns of Inheritance

P', #iiiiufriffiffi 6e
Let's begin a discussion on patterns of inheritance. There were many early ideas
about heredity. It was widely believed that strange creatures could be breed by
cross breeding different species. For example, the minotaur from Cretan
mythology was a creature with the torso and head of a human and the body of a
bull. The giraffe of the African plains was thought to be a cross between a
leopard and a camel. This is even reflected in the scientific classification
testablished by Carl Linnaeus) of the giraffe; Giraffa camelopardalls. By the time
the Middle Ages arrived it was believed that crosses were viable only within a
particular species.

cne of the early models proposed for heredity was that of pangenesis. This
-leory stated that each part of the body produced tiny particles called pangenes
or gemmules. Pangenes were though to be miniature replicas of each organ or
].sue of the body. These pangenes were carried to the reproductive organs by
he circulatory system where they were packaged into the sperm or the egg.
During fertilization the male and female pangenes united in the femal"'s rro-b.
Er-entually a new organism would result. If a pangene was healthy or defective,
:: l'ould be passed on to the offspring. The combining of pangenes led to the idea
:f "blending inheritance." This simply meant that the individual was a mixture of
two parents. The male pangenes were thought to be the major contributors to
--t'le

ris notion of blending. This was the prevailing theory up to the end of the last
:entury and it is interesting to note that it was proposed by Charles Darwin in
1S68.

-: the male and female contributions to the offspring resulted in blending, then at
'"-me pointin the future all of the offspring should resemble one another. We
r.ow that this is not the case. Flowever, it seemed to present a paradox for the
:,eory proposed by Darwin. In 1760 Josef Koelreuter, a German botanist, was
:ossing different species of the tobacco plant. The offspring that were produced
;,' ere fertile and they were able to produce a new generation of plant which
was
::*tly variable. A few members of this new generation resembled the original
srecies of tobaccb plant. Even though these results were not in agreement with
::,e ideas of blending inheritance they did provide clues for the mechanism of
:e:edity.

i+":elreuter's work was taken seriously by many investigators over the next
:rundred years and in the 1790s an investigator named T. A. Knight crossed two
:rrae-breeding lines of the garden pea (Pisum satiaum). Within the pea plant
:[rrver are the male (anther--produces pollen) and female (stigma-produces
*:gs) sexual organs. A true-breeding plant is one that if left to itself will self-
;q:llinate and always produce the same kind of plant. One true-breeding plant
ll,':-.r-ld always produce purple flowers while the other true-breeding line would
ilL'-,q dvs produce wlute flowers. These plants were designated as the parental (p)

#.:,eration. When the pollen from a purple flower was sprinkled on the eggs
r::1a white flower, only purple flowers were produced in the first filial (F1)
Fneration (Figure 9-1).

:;rq\nght @ by The Berkeley Review 279 The Berkeley Review


Specializing in MCAT Preparation
Biology Genetic Information Patterns of Inheritance
l
P Purple Flowers X White Flowers {

Fl Purple
I G
Flowers
:.:

F2
+ -.
Purple Flowers + White Flowers
-6

Figure 9- I

The purple flowers of the F1 generation were allowed to self-pollinate. In the +t


next generation, the second filial (F2) generation, both purple and *hite flo.,r,ex
appeared. This says that some characteristic trait was hidden (masked) in the Flt
generation but reappears in the F2 generation. Knight's comments on tl:_o
phenomenon were that there were more purple flowers in the F2 generation tha:, ;-_,
while flowers. He left it at that.
-.:?

-Ar

pl,li:

l--,i-

s-glgi

$[=rat
uE:€;
!L

nu:-:ch

fu'{r,de

able 9

:--en I
:(ffi
-^ )

"- J^l
:--qC_[
ro 'J +i-

::l oi tl
iener
ro
tlL -
+-l
.r

iital,
n-atir
:.:ifera

each c
:--5 tha
:i lreq
Copyright @ by The Berkeley Review 2AO The Berkeley r right :
Specializing in MCAT
Biology Genetic Information Gregor Mendel

fu', ildl
Gregor Mendel was born in Austria tr.1g22. He entered a monastery in Brtinn
and received a formal education. He later attended the University of Vienna and
after two years returned to the monastery because he failed the exams that would
have given him a teaching certificate. while at the monastery he carried out
examinations on the common garden pea. His worked marked the beginning of
modern genetics.

\A/hy did Mendel choose the garden pea? There were a number of reasons. (L)
A
great deal of work had already been carried out with the garden pea. Many
e_arlier investigators (e.g., Knight) had produced hybrid peas by crossing
different true-breeding lines. (2) Many different true-breeding varieties weri
available for use in experiments. (3) Mendel choose 32 of the mariy different true_
breeding pea plants of which to work with. From these 32 varieties he choose
lines that could be distinguishedby z different paired traits (e.g., alternative
characteristics like purple versus white, smooth ,rets.r, wrinkled-, etc.) (4) pea
plants were rather easy_ to grow, they have a short generation time, and they are
small and would not take up much space in the garJen.

As we have mentioned, both the male and female sex organs are contained.
within the flower of the pea plant. If the pea plant is left undiiturbed, it will self- Dominant vs
pollinate. Flowever, if the anthers are removed before pollination and pollen is Trait Recessive
Flower Color Purple vs White
introduced from another pea plant, then cross-fertiliiation will result. The Seed Color Yellow vs Green
fertilized eggs in the stigma develop into the embryo (the seeds). Each is the Seed Shape Round vs Wrinkled
Pod Color Green vs Yellow
product of a separate fertilization. The pod that contains the seeds has the Pod Shape Round vs Constrictec
characteristics of the parents while the seeds themselves belong to the next Flower Position Axial vs Top
Plant Height Tall vs Dwarf
generation.
Table 9.1
Mendel choose 7 traits that were alternatively expressed for 7 characteristics of
the pea plant. Those 7 traits are shown in Table g-t. Note which is dominant and
rvhich is recessive.

Mendel choose parent plants that had breed true for many generations. He then
performed crosses by cross-fertilization for each of the 7 pairs of traits shown in
Table 9-1. For example, from the purple flower he used the pollen and from the
''vhite flower he used eggs. Mendel also performed the reciprocal crosses (e.g.,
rollen from the whTte flower and eggs from the purple flower). These pian"ts
:epresented the parental or P generation. The offspring of each of these z crosses,
:eferred to as the first filial or FL generation, expressed one of the two parental
:haracteristics.

\{endel allowed the F1 pea plants to self-pollinate for one generation. He then
.cored the characteristics of the second filial or F2 generation. Mendel found that
.ach of the 7 characteristics reappeared in the F2 generation. For example, in the
F2 generation round and wrinkled seeds were observed as were purple and
.rhite flowers. This observation did not agree with the notion of biending
rheritance that we previously mentioned. In contrast to the earlier experimental
:,bservations that Knight made on the garden peas, Mendel counted the number
:f different types of plants in the F2 generation.

- or each of the 7 traits in the F2 generation, Mendel found close to a 3 to 1 ratio of


:lants that expressed one parental trait compared to the other parental trait. The
:rost frequently expressed trait was always the one that was exclusively
-opyright @ by The Berkeley Review 281 The Berkeley Review
Specializing in MCAT Preparation
Biology Genetic Information Gregor

expressed in the F1 generation. Mendel proposed that the trait that


expressed in the F1 generation was dominant while the other unexpressed
was recessive. In other words, if a purple flower was crossed with a white
and the F1 generation gave all purple flowers, then those purple flowers
dominant over the white flowers. Even though the white flower trait is
in the F1 generation it reappears in the F2 generation.

Mendel proposed that the parent plants do not transmit their physiological
or forms directly to their offspring (i.e., there are no pangenes) but ra
transmit "hereditary factors" which act later in the offspring to produce the
Today we call these hereditary factors "genes."

Each individual possesses 2 factors with respect to each trait. For example,
Purple P"gP White
can be one factor for purple color and another factor for purple color, one
Purple White White for purple color and another factor for white color, or one factor for white
Figure 9.2 and another factor for white color (see Figure 9-2). One of these two factors
contributed by each parent to the offspring.

The alternative forms of the factors determining a given trait are called
The flower color trait has a purple allele and a white allele. An individua-l
have two different alleles or two identical alleles. If the alleles are different,
the individual offspring is referred to as being heterozygous. If the two
are identical, the individual offspring is referred to as being homozygous-
example, in Figure 9-2 the Purple/Purple alleles would be con
homozygous as would the White/\4/hite alleles. The Purple/\Alhite alleles
be heterozygous.

The two alleles contributed by the parents to an offspring do not influence


other. For example, if an individual pea plant has a purple and a while
then those alleles will stay purple and white. They will not form in
alleles as expected from the blending inheritance hypothesis.

Let's consider one of Mendel's experiments on flower color. In this ex


two true-breeding pea plants are crossed. The plant with the purple
(dominant) is represented by an upper case W while the plant with the
flowers (recessive) is represented by a lower case zo. [The letter "W" is
chosen from the recessive allele.] The purple flower can only produce W
while the white flower can only produce zo gametes. We can express this
a Punnett square as shown in Figure 9-3. Note that four heterozygotes
produced. However, since W is dominant oyer u) | we see that the F1
yields all purple flowers.

In the next step Mendel allowed the Fl generation to self-pollinate. In


Punnett square in Figure 9-3 we find two homozygous individuals (WW
ww) and two heterozygous individuals (Ww and Wzo). However, since
allele is dominant over the w aIlele,3 of the offspring from this cross will
purple flowers while just 1 will have white flowers. The color of the flower
observed is referred to as the phenotype. The total number of alleles
individual contains is referred to as the genotype. Therefore, the phenotypic
of the F2 generation is 3:1. However, the genotypic ratio of the F2
L:2:L (because there is one WW, two Ww, and one ww). Similar expeti
be performed for the other traits that Mendel studied.

Copyright @ by The Berkeley Review 242 The Berkeley


Specializing in MCAT
Biology Genetic Information Gregor Mendel

*w
Whire
Parental
Generation

ww
(P) (Wn) (Wn)

0
Purple @@ F1 (A11 Purple)

ffi @lryilEl
ww
(wr)
tr @F;[it]ll

First Filial
Generation
(Fr)
*w Puryle

(ww)W
o
Purple @@
@''"'
F2 I White)

ffi
(3 Purple:

@lEl,t;l (nn )

tr
figure 9.5
clt{.'y;l w
fr i'ased on experiments like these, Mendel proposed that:
s
ts 1,. The traits that were expressed could either be dominant or recessive. For
t example, purple flower color in the pea plants was dominant over the
s recessive white flower color. The parental generation does not transmit their
M traits or th6ir form to their offspring but rather they transmit hereditary
E "factors" while include information about the traits to be expressed. Each
ml individual possessed two of these hereditary factors for each trait (one
coming from each of the two parents).

1 The alternative form of a hereditary factor, which leads to an alternative form


of a given trait, is called an allele. For example, the hereditary factor for a
purple flower would be one allele while the hereditary factor for the
corresponding white flower would be the other allele.

i" An individual who possesses two identical alleles is said to be homozygous


(e.g, WW for ihe puiple flower color or ww for the white flower color) while
an individual who possesses two different alleles is said to be heterozygous
(e.g., Ww for the purple flower color).

The two alleles contributed by the parents to an offspring do not influence


', each otherin any way. In other words, the two alleles come into a cross and

-,:rvright @ by The Berkeley Review 243 The Berkeley Review


Specializing in MCAT Preparation
Biology Genetic Information Gregor Mendel

leave a cross unchanged. They do not form intermediate alleles as would t',e
expected from the blending hypothesis.

5. The presence of hereditary characters in an individual does not ensure that it


Pollen will be expressed. For example, in the F1 generation the white allele is
ll2W ll2w present in the purple and white hybrid but it is not expressed (it is recessirel,lL

1/4wW. tIl,4,'Ww
L/2 W (le Recall that in relationship to what was expressed we mentioned the
(PrifbXe)
h
b! and genotype of an individual. we said that the phenotype is the ou
bo
appearance of a given trait. It is what you can actually see (e.g., flower color
:1/,4,,I&W l/4ww genotype is the genetic makeup of an individual (e.g., purple flower color rr-
ll2w tP.urplc) (White) be WW or Ww while white flower color would be ww).
Figure 9.4
when Mendel crossed the two true-breeding lines of pea plants he found rn
F2 generation a 3:1 ratio of purple flowered plants to white flowered ph
Another way to express this (other than Figure 9-3) is shown in Figure 9-4. Ir
two alleles are alternate alleles and equally contributing, then haH of tt
should be w (upper case) and half should be w (lower case). This would be
for both the pollen and the eggs. [when Mendel did this work the punnett sq
had not been invented. Lrstead, he used algebraic expressions.]

Mendel rcalized that not all the purple flowered plants should have the
genotype. He said that some of the purple flowered plants would har-e
He predicted that 1/3 of the purple flowered plants should
9tlfgt"i genotype.
ww while 2ig ahould be ww. (we are yust coniidering the prrrpl" florve:sd
plants and not the white flowered plants. The genotypes of the p"tpt" flou-erEil
plants in the F2 generation are w.w, ww, and ww. see Figure s-s.) He tested I
prediction in two ways.

His first test was to allow the F2 plants to self-pollinate. If a purple flowered H!
plant with the genotype ww self-pollinates, then the offspring will onlr ncr
purple flowered plants. If both purple flowered F2 plants with ihe genotrlm
ww self-pollinate, then both purple and white flowered plants are produced.,
This confirms the prediction that 1/3 of the purple flowered F2 plants will be ltw
while 2/3 of the purple flowered F2 plants wlli be ww. [a similar test could be
done for just the white flowered pea plants.l

His second test involved a test cross. In this cross Mendel used a purple floweredi
plant from the Fl generation in which the genotype was now knbwn (i.e., it rt-m
either WW or Ww). In order to determine the genotype of this unknown purple
flowered plant, he crossed it with a white flowered plant which rr-as
homozygous recessive (i.e., ww) as shown in Figure 9-5.

If the unknown allele distribution in the purple flowered plant is ww, then all of
the offspring in the test cross with the homozygous ww white flowered plard
will be ww. The offspring will be heterozygous and they will all have purple
flowers. F{owever, if the unknown allele distribution in the purple flowered plant
is ww, then the results of the test cross will give half heterozygous ww purple
flowered plants and half homozygous ww white flowered plants (Figure 9-5).

Copyright @ by The Berkeley Review 2A4 The Berkeley Kevieu ,I

Specializing in MCAT Preparation


Biology Genetic Information Gregor Mendel

White (rn,o)

8{*Wry Allele distribution


is unknown. It is
Since all purple flowers, Since half purple flowers and
either WWor Ww.
unknown flower was half white flowers, unknown
homozygous (WW ) domi nant. flower was heterozygous (Ww).

Figure 9-5

\'Iendel's experiments therefore confirmed his hypothesis. Alternatiae alleles


segregatefrom each other in heterozygous indiuiduals and retain their identity. This is
i<rrown as Mendel's First Law of Heredity (also called the Law of segregation).
Even though Mendel showed this to be true for garden peas, it has been shown
io be applicable to all eukaryotic organisms (including humans).

RrGg (Fl Generation)

Pollen (Male)
1/4 RG 1/4 Rg ll4 rG F2 Generation
1ll6 ltr6 1n6 1.116 Resulted from parents
RRGG RRGg RrGG RrGg with two different

o o o o
1t16 U16 1 /16 1 t16
characteristics.

RRGg RRgg RrGg Rrgg

o @ o @
U16 t/t6 1/ 16 U16
Round, Yellow

RrGG RrGg rrGG rrGg

o o o o
Round, Green

1n6 1t16 1/16 1 t16 Wrinkled, Yellow


RrG$ Rrgg rrGg rrgg

o @ o @ Wrinkled, Green

Copyright @ by The Berkeley Review 245 The Berkeley Review


Specializing in MCAT Preparation
Biology Genetic Information Gregor Mendel

Recall that Mendel identified 7 different pairs of traits. He next examined two
different traits segregating in the same plant. The two different traits he
considered were round versus wrinkled seeds and yellow versus green seeds.
The round seeds are dominant over the recessive wrinkled seeds. The yellow
seeds are dominant over the recessive green seeds. [Mendel had determined
which seeds were dominant and recessive in a previous crossing experiment
involving only one of the pairs.] After establishing pure breeding lines of pea
planis with these traits, Mendel crossed a true-breeding pea plant with round
seeds which were yellow with a true-breeding pea plant that has wrinkled seeds
which were green. He wanted to know if a particular allele for one trait (such as
seed color) would influence which allele the gamete had for the other trait (such
as the seed shape). The type of cross Mendel constructed is referred to as a
dihybrid cross (Figure 9-5). lNote that in this example we will let the uPPer cas€
R represent round seeds, the upper case G represent yellow seeds, the lower case
r represent wrinkled seeds, and the lower case g represent green seeds. The
reason behind this is because we already used the letter W in the previous
example with flower color.]

Mendel crossed true-breeding RRGG pea plants with true breeding rrgg Pea
plants. This gave an F1 generation in which all the pea plants had round yellow
seeds (i.e., RrGg). These dihybrid individuals were next allowed to self-fertilize
"If the segregation of alleles affecting seed shape were independent of the
segregation of those affecting seed color, then the probability that a particuJar
puir of seed shape alleles would occur together with a particular pair of seed
color alleles would be simply the product of the individual probabilities that each
pair would occur separately. Thus, the probability that an individual n'ith
wrinkled, green seeds would aPpeal in the F2 generation would be equal to ttn
probabilit| of observing an indlvidual with wrinkled seeds (1/a) limss 'ho
probability of observing in individual with green seeds (714), or116J

The composition of the dihybrid cross shown in Figure 9-6 can be predicted iI
mechanism behind placing an R allele or an r allele into a gamete is i
of the mechanism behind placing a G allele and a g allele into another
(separate) from each othn
[Recail Mendel's First Law: Alternatiue alleles segregate
heterozygous indiaiduals and retain their identity. Another way to put this is
"two members of a gene pair segregate from each other into the gametes, so
1/2 of the gametes ott" member of the pair and the other 1/2 of the ga
"itty
carry the other member of the gene pair." A gene pair could be a dominant
(allele) and recessive gene (allele).1 Notice that in the dihybrid individuals I

genes involved in the shape of the seed and the color or the seed can each
iepresented by a pair of alternative alleles. This means that one would ,

R6, Rg, rG, and rg gametes. The probabitity that a gamete would be Rg is
on Mendel's First Law. In other words, the probability that a gamete would
RG is simpty Ll2 x ll2 ot,L14. The same would hold for the Rg, rG,
gametes. fi-tii ir where thellacomes from in the dihybrid cross shown in
6-0. rnt would also explain why the probabi]ity of finding an individual
wrinkled., green seeds in the F2 generation isLl6.It is simply the-probabil
1/4) times the probability of obsenir
observing lhe male gamete rg (w_hich is
female gimete rg (whifh is also t|u). tr, other words, the probability of findi
rrgg zyfote-onld be L I 4 x\ I 4 or L I 15.

The results of Mendel's dihybrid cross are shown in Figure 9-7. These v
obtained from the F2 generation. Mendel examined 556 seed types and
that 315 were round and yellow, 108 were round and green, 101 were u

Copyright @ by The BerkeleY Review 28,6. The Berkeley


Specializing in MCAT
Biology Genetic Information Gregor Mendel

and green, and 32 were wrinkled and green. This gave a ratio of 9:3:3:'1,,
respectively.

If you look at the Punnet leuare in Figure 9-6, you will notice that there arc 9116
F2 Generation Seed # Ratio
round and yellow seeds, 3/15 round and green seeds, 3/15 wrinkled and yellow
seeds, and116 wrinkled and green seed. This is a ratio of 9:3:3:1. There are 4 Round & Yellow 315 9
Round & Green 108 J
phenotypes in the F2 generation. How many genotypes are there for each of the 4 Wrinkled & Yellow 10r 3
phenotypes? Wrinkled & Green 32 I
556 t6
\vhat does this dihybrid cross mean? It simply means that the hereditary factors
lgenes) for color (yellow and green) and shape (round and wrinkled) assort Figure 9-7
independently of one another. This is referred to as Mendel's second Law of
Hereditary (or the Law of Independent Assortment). Another way to put this
rvould be that the segregation of one gene pair is independent of other gene pairs
during the formation of the gametes. one addition to this statement (which
\'{endel did not postulate) is that independent assortment of the genes will occur
rt they are located on different chromosomes or are far apart on the same
;hromosome.

\Iendel published his findings in 1866 but did not receive much attention until
about 1900, some 16 years after his death. \A/hen Mendel published his papers he
lid not know about meiosis, chromosomes, DNA, or even genes. today we
i,now his hereditary factor to be genes and his law of segregation to be meiosis. Flower Seed
Color Color
lhe word

're
"gene" was first coined in 1911 by wilhelm Johannsen, a Danish
geneticist. Genes, which are the basic units of hereditary (DNA), are located at
tl * chromosome
.recific locations along the chromosomes. In fact, they are arranged linearly "t
{ong the chromosomes. Recall that we mentioned that alleles are one of two or
=ore alternative states of a given gene. For example, the white and purple flower 2-
:olor of the pea plants that Mendel examined is simply due to alternate forms of
:le same gene. 3-
- the genes are on different chromosomes, they will assort independently
:ecause the chromosomes themselves assort independently. However, if two
renes are on the same chromosome, then they may assort independently if
:ossing-over occurs between them. As we will see, the order and spacing of
4ffi

Flower
,l' / ,l
Positio^ / Plant
renes on a chromosome can be determined from the frequency with which the Height
Pod color Pod
:--fferent gene pairs tecombine. Genes that are far apart will recombine more
. l. Shape
:equently than genes that are closer together. Y
5ffi
lhe frequency with which crossing-over occurs allows one to map the relative
:,:sitions of the various genes on chromosomes. For example, consider the seven
:aits which Mendel studied. These traits can be located on the chromosomes of 6-
f,e pea plant as shown in Figure 9-8. Seed color and flower color can be found on WR
:"romosome #1 while seed shape can be found on chromosome #7. 7ffi
*ta
1',-ou studied the segregation of seed color and seed shape, you would find that I
Seed Shape
:i:ev wouid assort independently (because they are on different chromosomes).
It.rrver color and seed color should assort independently because they are rather
m: apart on chromosome #1. Note the location of the genes for plant height and Figure 9-B
;,:d shape. Those two genes are quite close to one another and therefore should
rrt assort independently.

-.:nyright @ by The Berkeley Review 287 The Berkeley Review


Specializing in MCAT Preparation
Biology Genetic Information Gregor Mendcl

Let's place the traits for seed color and seed shape on the chromosomes as shown
in Figure 9-9. We will let G stand for the seed color and wR stand for seed shape
In the gonial cells we will just use chromosome #1 and chromosome #7.

ffi,
o',''',,ffi
Rt/ ffe
/)
"w" ,v,
M
WV
ir,
ffifl \\

Gonial Cells

%f)
&A
&1/
HR
wRd b*
Figure 9.9

After replication we get homologous pairing and then crossing-over as shown


Figure 9-10.

Flgure 9-lO

Note the arrangement of the crossover events as they proceed through Te


I. This is shown in Figure 9-11. Two possibilities for Telophase I are shown

Copyright @ by The Berkeley Review 2Aa The Berkeley


Specializing in MCAT Prep
Biology Genetic Information Gregor Mendel

Figure 9- I I

lValter Sutton
r 1902 the American geneticist walter sutton made a logical argument that
.:ated that chromosomes were the places where Mendel's hereditary iactors were
. rcated. Sutton postulated:

If Mendel is right, then the sperm and the egg must contribute equally. Also,
since the sperm has very little cytoplasm and is mainly composed of nuclear
material, the hereditary factors must be contained within the nucleus.

- The meiotic behavior of chromosomes is the same as that of Mendel's


hereditary factors.

cametes from meiosis have one copy of genetic material for each hereditary
factor. Mendel's hereditary factors are distributed in the same way.

Chromosomes assort independently in meiosis as do Mendel's hereditary


factors.

-'\'ever, there was problemwith this because it was mentioned that there
a
::e many more independently segregating pairs of genes (hereditary factors)
.: there were chromosomes.
'-"iomas Hunt Morgan
1910 Thomas Hunt Morgan performed a crucial experiment that
,,:'onstrated that sutton's theory was correct. Morgan showed that a gene
. :olling eye color in the fruit fly Drosophila melanogaster was located on the X
-:-:omosome. As we will see, the results from Morgan's experiments led to the
:.,ceptance of sutton's theory. In 1909 F. A. Janssens suggested that homologous
-:1romosomes exchanged material during meiosis. At first Janssens theory was
- 'ot widely accepted but later experiments would prove him correct. One of those

:rperiments was performed in 1931 by the geneticist Curt stem who proved that
:rossing-over in the X chromosomes of Drosophila melanogaster involved the
rhysical exchange of chromosomal material. with independent segregation of
:hromosomes as well as crossing-over along the chromosomes, there canbe more
-ndependently segregating units than chromosomes.

Copyright O by The Berkeley Review 249 The Berkeley Review


Specializing in MCAT Preparation
BiologSy Genetic Information Gregor

Let's consider Morgan's experiment for a moment. The fruit fly D


melanogaster normally has red eyes (referred to as the wild type because it
normal). During one of his experiments he had noticed a mutant male fruit i
with white eyes. Morgan crossed this mutant white eyed fly with a wild type
eyed female fly. All the progeny (offspring) in the Fl generation had wild !
eyes (i.e., all the offspring had red eyes). Next, a male and a female fruit fly v
red eyes from this F1 generation were allowed to mate. In the resulting
generation all the female fruit flies were of the wild type while only half of
male fruit flies were of the wild type. The remaining male fruit flies had
white eyed mutation. After examining all of the Progeny in the F2 generab
Morgan concluded that eye color segregated among the progeny as predicted
Mendel. However, the white-eyed characteristic only expressed itself in the
male fruit flies. This is a very unusual result.

After looking at the chromosomes of both sexes of Drosophila melanol


Morgan discovered that the female had two copies of the X chromosome
the male had only one coPy of the X chromosome and a copy of a Y
In other words, females flies have an XX genotype while males have an
genotype. It tumed out that the trait for white eyes lies on the X chromosome
ii missing from the Y chromosome. If a given trait is determined by a gene on
X chromosome, it is said to be sex linked (or X linked).

White-eved Red-eYed

LWd;bWi
Lffi@X@Wl \\il
u

Figure 9-12

29o The Berkeley


Copyright @ by The BerkeleY Review
Specializing in MCAT
Biology Genetic Information Gregor Mendel
As shown in Figure 9-12,.the F1 generation all
have red eyes. This means that the
gene for red eyes is dominant. Notice that
the female fly itt *," Er generation has
a mutant X chromosome. Her eyes are
red because her one good X chromosome
is stillable-to express red eye coior. However,
when this F1 flmale mates with an
F1 male who has red eyes, she passes her
good X chromosome and her defective
X chromosome to her progeny in the F2 generation.

The Fl male passes his y chromosome and. his


good X chromosome to the
orogeny-in the F2 generation as well. If a female rty"in
ure F2 generation were to
receive the defective X chromosome, her eye
color will still be red because she
rvould have received a wild-type x chromosome
from the male of the F1
generation. However, if a male received the
defective X chromosome from the
:emale oJ the Fl generation, then he would
have white b".u.rre there would
:e no other X chromosome with the gene for-the red "y",
.oto, urrurtuble. why?
3ecause the genotype of the male is Xi while
that of the"fu
female is XX.
Curt Stern
h 1931 Curt stern did,an experiment that proved
that crossing over invorved the
;hysical exchange of chromosomal material. stern examined i*o g"r,", on
chromosomes of the female the X
ny Drosophila *rtonogo,itii. ii" ,*o genes that
{Ttl
:e was interested (Figure 9-13) in were the recessive
gene for carnation eye color
car) and the dominant gene for the bar_shaped
eye (B).

Fr Femare 'ilfr|
o,o_) |ii
t,y' L;"'-""'
.rm car
B

tr .o:l',i:?i-:r'
..orr,T!-ou", ! +- and B
firriil /fiD

?.T#i' 'If,
H tr f,-
t*Fl
'tA
. \/[
----" I
to.''
ro.,,"rromFl ru,. Gameres
cun'.,.r-
, from Fr MaieGametes ja
u
Mare
)1r'

''B
,/\
U )\
./ \
'TB "TH
,lt/\ fi
=

rH
Bar
H''
Carnation
tr
Normal
fi=
Carnation Bar

Parental Combinations Recombinant Combinations

Figure 9-I5

, rpyright @ by The Berkeley Review 291 The Berkeley Review


Specializing in MCAT prCparation
Biology Genetic Information Gregor Mendd

One of the X chromosomes could be identified because it had a "gap" in its


structure (due to inability to stain properly) while the other X chromosome could
be identified because it had a piece of a Y chromosome attached to it. These two
chromosomes could be clearly distinguished from one another under the liglt
microscope.

As shown in Figure 9-13 the experiment starts out with the female's FL I
chromosomes. Each has the respective genetic locus (location of a gene on r
chromosome) for the carnation eye color and the bar-shaped eye. Stern crossed"
these F1 females with the sperm from a male fruit fly who had carnation co
eyes. He then separated out the cross-over events in the F2 progeny'

He found normal shaped eyes that were carnation colored and normal
eyes that were bar-shaped in the progeny that had experienced crossing-over.
the progeny that had not experienced crossing over, he observed carnati
colored eyes that had the bar-shape and normal colored eyes without the
shape. Stern concluded that the genetic exchanges of various traits on a gi
chromosome (such as eye color or eye shape) involve the actual exchange
portions of those chromosomes in a event referred to as crossing-over. In
words, whenever genes recombine, chromosomes recombine. [Note that in
diagram in Figure 9-13 the "+" sign indicates the wild type individual or
normal individual.l

Alfred Sturtevant
Alfred Sturtevant (a student of Morgan's) constructed the first genetic map
L93L. Sturtevant examined three traits in the female fruit fly Droso
melanogaster. Th"y were yellow body color (designated as y), white eye color
and miniature wings (min). These traits resided on the X chromosome and
sex-linked. These lower cases refer to the recessive alleles' The normal
color is gray (+), the normal eye color is red (+), and the normal wing (+) is
50% longer than the miniature wing. [Note that here we are using the
terminology which means the dominant allele. We could have used upper r

Y instead of the +, an upper case W instead of the +, and upper case MIN i
of the +, respectively. The meaning is the same.]

Figure 9-14

Sturtevant crossed a female fruit fty that was homozygous recessive for the
traits with a male fruit fly that was normal for these traits. All of the
progeny were heterozy1ous as shown in Figure 9-14. The important point
lhaf the progeny females are heterozygotes. This means that if crossir

Copyright @ by The BerkeleY Review 292 The Berkeley


Specializing in MCAT
Biology Genetic Information Gregor Mendel

occurs between any two traits, gametes will result with different combinations of
the alleles.

In order for sturtevant to see all the possible recombinant types, he crossed the
F1 females shown in Figure 9-14 with males that were tecessirre to all three traits.
This cross is shown in Figure 9-15.

Female Male

ywmin v w min
X Chromosome +..+-4-
X Chromosbme X X Chromosome
Y Chromosome
+ +
0
Progeny Fruit Flies
Figure 9. l5
The progeny were next scored for the phenotype expressing these three traits.
sturtevant's results are shown in Table 9-2. when you look ut this table, keep in
mind the cross shown in Figure 9-15. Let's consider the body, uyu, #ir,g
phenotype (1st row) which reads + + +. How is this phenotype ottaineaz mo[
at the female's X chromosomes in Figure g-l.s. rf the femlleX chromosome
rvhich reads + + + combines with the male X chromosome which reads y ta min,
henthe resultant progeny will be a female with the phenotype +++. Similarly, if
lhe female X chromosome which reads + + + combinei with the malL y
;hromosome, then the resultant progeny will be male with the phenotype + + +.
This is one parental phenotype (i.e., + + +) that is observed in ribte g-1. we can
lo a similar analysis for the y za min phenotype (2nd row) in Table 9-2 as well.
This is also a parental phenotype. The number of progeny observed for the
larental phenotype + + + is 758 while the number of progeny observed for the
rarental phenotype y m min is 700. Because these are random events we do not
*et the same numbers.

PHENOTYPES CROSSOVBR TYPES


Number of
Body -Eye Wing Progeny Body/Eye EyeAiling Body/Wing
++ + 758
yw min 700
++ mrn 401 401 40i
Single Crossover yw + 3t7 317 317
+w min 16 1; 16
y+ + t2 t2 t2
Double crossover +w + 1 1 1
y+ min 0 0 0
TOTAL 2205 29 719 746
Recombination Frequency (o) 32.608 33.832
Iable 9-2
;\'here did the other six classes of phenotypes come from? Consider the 3rd row
:f phenotypes in which we have the + + min phenotype. How did we get this
:henotype? There must have been a crossover as shown in Figure 9-16. \A/hen

Jopyright @ by The Berkeley Review 293 The Berkeley Review


Specializing in MCAT Preparation
Biology Genetic Information Gregor

the two female X chromosomes separated we would have y tn + and + +


Suppose the female + + min combined with the male X chromosome which
y w min. The resulting female progeny would have the phenotype + + mi
[Remember, a genotype of (+ + min) / (y w min) would give a phenotype of +
min because the dominant + + genes would be expressed over the recessive y
genes. The min / min genotype is still recessive and so the phenotype would
min.l Similarly, if. the female + + min combined with the male y ch
the resulting male progeny would have the phenotype + + min. A si
analysis can be done for the 4th row of phenotypes in Table 9-2. In this case
get the phenotype y w +. The number of progeny observed for the + +
phenotype is 401 while for the y w + phenotype it is 317.

Progeny Fruit Flies

Figure 9.16

Consider the 5th row of phenotypes in Table 9-2 which reads + w min. We
obtain this type of phenotype by doing a cross as shown in Figure 9-12. Aftu
female X chromosomes crossover and pull apart we will have a y +
chromosome and a + zu min chromosome. If the female + ut min
combines with the X chromosome of the male which rcads y za min, the
progeny will be female with the phenotype + za min. There are 16
observed with this phenotype. Similarly, if the female + zn min
combines with the male Y chromosome, the resulting male progeny will have
phenotype + w min. We can do a similar analysis if the female y
chromosome combines with either the male X chromosome which rcads y m
or the male Y chromosome. In both cases the progeny will have the
w min. The number of progeny observed in this case is 12. Some will be
and some will be males.

+s
Progeny Fruit Flies

Figure 9- l7

Copyright @ by The Berkeley Review 294 The Berkeley


Specializing in MCAT
Biotogy Genetic Information Gregor Irlendel

Notice that the cross in-Figure 9-r7 yields 16 progeny with the phenotype
+ zo
y:" 11112 progeny with the phenotype y + +. Notice that the cross in Figure 9-
15 yields 401 progeny with the phenotype + + min and 317 progeny
,"ith th"
phenotype y w +. why the difference in the number of progenyi Ii has
to do
with the distance the genetic markers (i.e., the genes) uru u*ulifrom one another.
The further they are away from one another, the more chancl there
will be for a
cross. The closer they are to one another, the less chance there will be for a
cross.
Irr.other words, the y and zu genetic markers are closer together than the
zp and.
min genetic markers. Therefore, we can modify our chroirosomes to represent
this as shown in Figure 9-18.

Female Male
ywmin ywmin
X-as
x+
++ +ft
x-1
v
Progeny Fruit Flies
Figure g- l8

Consider the 7th rowinTable 9-2. we can explain the phenotype+ 70 + by


a
double crossover as shown in Figure 9-19. The number oi ptog".y observed
for
:his crossover is 1. [I have left out the details of expTaining how the
:hromosomes combine to get the genotype and then the ob^served"phenotype
:ecause it is explained in the previous examples.] In the gth row we see
the
:henotype y + min. we can obtain this phenotype from the double crossover
;hown in Figure 9-19 as well. The number o-f observed progeny for the
:henotype y + min is 0.

Female

Progeny

Figure g- l9
,-r,etotal number of progeny observed. in this experiment is 2205. The total
-rmber of progeny observed to have crossover events between the y and w
;:netic markers is 16."+ 12 +'1, = 29. Therefore, this amount of cross-ing over
:=presents (2912205) x 100 or 't .g'l,s% of the total number of crossover events.
:-nilarly, the total number of progeny observed to have crossover events
::tween the zu and min genetic markers is 401 + i17 + I = 719. This represents
'7912205 or 32.6080/"
of the total number of crossover events. Finally, the total
' :rnber of crossover events between
the y and min geneticmarkers is 401 + 317 +

:ryright @ by The Berkeley Review 295 The Berkeley Review


Specializing in MCAT preparation
Biology Genetic Information Gregor Mendel

16 + 12 = 746. This represents 74612205 or 33.g32% of the total number of


crossover events. What we have just defined in this discussion is the map
distance between a gene pair. This distance is usually expressed in terms of i
map unit. In other words, if the frequency between two genes (e.g., y andmin) is
33.832'/", then those genes are said to be separated by about 33.g map units. A
map unit is sometimes referred to as a centimorgan (cM) in honor of r. H.
Morgan.

Knowing the map units between genetic markers allows us to draw a genetic
map. we can do this for the analysis in Table 9-2. This is shown in Figure 9-20.

v w min

Figure 9-2O

,n

ir
t:.

ta
'ffiri,

:,i:

:E:

:: !r

-!e

Copyright @ by The Berkeley Review 296 The Berkeley Review


Specializing in MCAT Preparation
Biology Genetic Information Genetic Loci & Alleles

ffiGHGffi$i:::ruUffi lliffilliffilil iG ;iiii i

So far we have only considered two alleles at each genetic locus. For example, in
Mendel's garden peas we had a flower color locus in which one allele was able to
express a purple flower while the other allele was able to express a white flower.
Recall that the purple flower allele was dominant over the recessive white flower
allele. It tums out that a given genetic locus can have many alleles. One example
is the ABO blood group.

We have considered only one gene controlling a particular phenotype (e.g.,


flower color, seed shape, etc.). However, as we will see, many genes may control
a particular phenotype. For example, in the biosynthesis of many compounds
within the cell there can be many sequential steps. Each step is controlled by a
separate enzyme which in turn is controlled by a particular gene. One such
biochemical pathway of interest involves the synthesis of the tryptophan. This
amino acid is considered an essential amino acid. An essential amino acid is an
amino acid that an organism cannot synthesize itself and therefore must obtain
that amino acid in its diet.

Tryptophan
One class of mutants that we should consider are the auxotrophic mutants. An
auxotroph is a mutant that will grow only when its medium is supplemented
rvith a particular compound which is not required by the normal wild type
organism. The wild type organism is referred to as a prototroph. An auxotroph
rvill not grow on a minimal medium. For example, an auxotroph might require
certain vitamins, amino acids, purines, or even pyrimidines for its growth. A
prototroph will grow on a minimal medium.

One specific type of auxotroph that we will consider is an auxotroph for the
amino acid tryptophan. Tryptophan auxotrophs will grow on a complete
medium. They will not grow on a minimal medium unless that minimal medium
has been supplemented with tryptophan.

IrL order to study these tryptophan auxotrophs we need to isolate them. As we


itill see, they are not all the same. How could we isolate such an auxotroph? L:r
-]re case of a penicillir mutant all we needed to do was plate bacteria on a agar
plate, add penicillin, and find which bacteria survived. In the case of the sugar
utiJization mutallts all we needed to do was look at the array of petri plates that
ive generated and find the white bacterial colonies. F{owever, in the case of a
:yptophan auxotroph we need to find a colony of bacteria that does not grow.
l{ow can we isolate a bacterium that will grow in a complete medium but will
:.ot grow in a minimal medium?

In 7952 Esther and Joshua Lederberg devised an experimental procedure called


teplica ptating. This procedure was designed to isolate auxotrophs. The
:rocedure behind replica plating is straight forward. Bacteria are incubated for a
:eriod of time on a master plate which contains a complete medium. Both the
'uxotrophs and the prototrophs
will grow on this master plate. These bacteria
-e distributed in Such a way that they will form individual colonies. Next, you
lently touch the colonies of this master plate with a velvet stamp. The velvet
:tamp has fine hairs which can pick up some of the bacterial cells of the master
:late. The velvet stamp is then touched to a new plate so that the bacterial
:clonies are set down in the same orientation that they were in on the master
:late. This new plate is referred to as a replica plate (see Figure 9-21). If our

Jopyright @ by The Berkeley Review 297 The Berkeley Review


Specializing in MCAT Preparation
Biology Genetic Infonnation Genetic Loci & Alleles

replica plate contained just a minimal medium, then only the prototrophs would
grow. The auxotrophs would not grow because they need to have a particula_r
supplement in their medium. The replica plate can be compared with the master
plate to determine which colonies are auxotrophic. once you have determined
which colonies on the master plate are auxotrophic colonies, you can then test
those colonies to see exactly what compound it is that they need for their growth-

Master Plate Replica Plate

Figure 9.21
Replica Plating Technique.

Let's consider some tryptophan (abbreviated as Trp) auxotrophs. A bacterium"


which is a tryptophan prototroph is designated as Trp+. This bacterium is the
wild type. It is normal. It does not need tryptophan in order to grow because rt
can make this amino acid from the compounds given to it in lts medium. -{
mutant bacterium which requires tryptophan for its growth is designated as Try-.
It turns out that the study of tryptophan auxotrophs was important in th;
elucidation of the biochemical pathway for the synthesis of tryptophan (Figure
e-22).

Gene 2
Gene
n
1t
I o
Anthranilate
Anthranilate phosphoribosyl
synthase lransferase
Chorismate'->
trpE
Anthranilate + trp"t N-(5'-Phosphoribosyl) -
D anthranilate -

Gene 5 Gene 4
ry C1 fl Enzyme 3 (f Gene
s
Enzyme 5
+
Enzyme 4
{,
-r

\;t
Indole (-' Indole-3-glycerol r-J Enol- 1-o-carboxy-
phosphate trp C2 phenylamino- I
1-deoxyribulose T
+
NHq phosphate
Enzvze 6 \
iI - Hnc- C-COO
t-
m
I
m

e+
H
"-*u Tryptophan
h
,ffi

H m

Figure 9-22 (
The Tryptophan Biosynthetic Pathway. Lffi

ffi
In Figure 9-22 we see part of the pathway that leads to the synthesis od m
tryptophan. We start with chorismate because it has a number of differeni ru

Copyright @ by The Berkeley Review 294 The Berkeley Reviw ;


Specializing in MCAT Preparatiol
Biology Genetic Information Genetic Loci & Alleles

,,nemical routes that it can take. one is towards the synthesis of tryptophan.
-
-:ismate is converted to anthranil ate by the enzyme anthranilati iyninase.
-: gene that codes for this enzyme is referred to as trpE.Anthraiitate
is
- erted to a compound called PRA by a transferase enzyme. The gene that
*es for this enzyme is designated as frpD. pRA is converted to a molecule
r--=d CDRP by an isomerase enzyme. The gene that codes for this enzyme is
:..-gnated as trpC1. CDRP is converted to a molecule called iGp by a synthase
'* -'.'me. This gene that codes for this enzyme is designate d as trpC2.IGp can be
::-','erted to indole by the alpha subunit of the tryptophan synthase enzyme.
-r-: gene that codes for this enzyme is designated as trpA. Finally, indole
cin be
::-,,'erted to tryptophan by the beta subunit of the tryptophan synthase enzyme.
- = gene that codes for this enzyme is designate d as trpB.
'n orE: Do not get lost in the names which are involved in this pathway. The
: , -rt is to understand what happens to the synthesis of tryptophan if a mutation
.:e to occur in a gene coding for one of the enzymes in this biochemical
: t:nrVay.]

, - -: three mutants that we want to consider in this pathway involve


the trpE,
-:'-{, and trpB genes.If there are mutations in
these genes, then the designation
; lryE-' trpA-, and trpB'. Consider the entries in Table 9-3. what we would iike
, .Tlo\4/ is whether or not certain types of bacteria will grow if their minimal
:.=dium is supplemented with (a) nothing, (b) anthraniGte, (c) indole, or (d)
:-. ntophan. when we consider this table, we will look at the type or condition of
:.: bacterium (e.g., trp+), and then follow the row that the bacterium is in, across
':^d towards the right, as we supplement its medium with the four items that we
*-.t mentioned.

Tryptophan Auxotrophs of E. coli


Growth on Minimal Medium Plus:
k Nothing Anthranilate Indole Tryptophan

TrP+ + + + +
TrpE- + + +
TrpA- + +
TrpB- +

Table 9.5
:+'ptophan Auxotrophs.

\ote that the trp+ prototroph will grow if nothing is added to the minimal
nedium. This is simply by definition (as it is the wild type bacterium). This
'racterium
can utilize tire components of that minimal medium to synthesize
iryptophan. The trp+ prototroph will still grow if you add either anthranilate,
ndole, or even tryptophan itself. This is why we see all pluses (+) in that row.

Consider the hpB'*rturlt. This bacterium will not grow if nothing, anthranilate,
or indole is added to the minimal medium. This auxotroph will only gro.vr. is
tryptophan is added. Thus, as we go from left to right in this row we see three
minus signs (-) and one plus sign (+). What is the explanation for this behavior? It
must mean that the mutation in the trpB gene leads to a defective enzyme that

Copyright @ by The Berkeley Review 299 The Berkeley Review


Specializing in MCAT preparation
Biology Genetic Information Genetic Loci &

will not allow indole to be converted to tryptophan. In other words, there is


block at the level of the trpB gene. Note that anything preceding indole (even
the enzymes are of the wild type) cannot be converted to tryptophan eith
\A/hy? Because once the products get to indole it is indole that uitimately must
converted to tryptophan.

Consider the tupE- mutant. This mutant will not grow if nothing is added to ,

medium. Frowever, this auxotroph wilt grow if either anthranilate, indole,


tryptophan is added. Therefore, as we go from left to right in the table, we
minus sign t) and three plus signs (+). we can use the same analysis as abr
The mutation n the trpE gene leads to a defective enz).me which cannot con
chorismate acid to anthranilate.

Finally, consider the trpA'mutant. This mutant will not grow is nothing
anthranilate is added. However, this auxotroph will grow if either indole
tryptophan are added. Thus, as we go from left to right across the row in
table we find two minus signs (-) and two plus signs (+). Again, we can use
same analysis as above. The defective enzyme is a result of a mutation in
trpA gene. This enzyme cannot convert the precursor of indole to indole.
the pathway must be blocked at the level of the trpA enzyme.

Epistasis and Pleiotropy


If any one of these genes that is involved in the synthesis of tryptophan
defective (possibly due to a mutation), then that gene will not be able to prodt
the enzyme which converts a particular reactant to a particular product.
means that tryptophan is not synthesized. In order for the cell to survive it
be supplemented with this amino acid. Even if two or more of the genes in
biosynthetic pathway for tryptophan are defective, then the cell willitill req
the presence of tryptophan to grow. Therefore, the genes involved in
biosynthetic pathway of tryptophan ale said to act in an epistatic fashi
Epistasis occurs between different pairs of genes. It does not occur between
members of an allelic pair. [In other words, two different genes which are
alleles of one another may affect the same outcome, which, in our example, is
inability to synthesize tryptophan.l In order for tryptophan to be syntheiized,
dominant allele of all the genes involved in this biosynthetic pathway must
present (in the absence of tryptophan).

There are other sets of genes which may act in an additive fashion. For
in yeast there are 7 genes which control the synthesis of the enzyrrle i
This enzyme hydrolyzes the disaccharide sucrose into its two m
constituents, glucose and fructose. If the yeast cell possesses the dominant
of any one of these 7 genes, it will be able to ferment sucrose. If the cell
more than one dominant allele, it ferments sucrose at a quicker rate. It is
that many human traits (e.g., height) are controlled by such gene families.

We mentioned that the dominant gene in Mendel's pea plants expressed a


color while the recessive gene expressed a white flower color. As far as it
known, the gene for white flowers in the pea plant only causes the flowers to
white. Flowever, there are examples in mice in which a dominant gene ca
yellow coat color. If the mice had one copy of this gene, they had yellow coats.
the mice had two copies of this gene, it would be lethal for the mice. In this
not only does the gene affect coat color but another copy of it affects via
This is an example of pleiotropy. It is where an individual allele has more

Copyright @ by The Berkeley Review 60() The Berkeley


Specializing in MCAT
Biotogy Genetic Information Genetic Loci & Alleles

one effect on the phenotype (e.9., a mouse with a yellow coat or a mouse that is
dead).

Chromosomes
We have mentioned that humans have 23 pairs of chromosomes. Of those 23
pairs of chromosomes 22 pairs appear nearly identical and are called autosomes.
These autosomes are numbered from I to 22 and are arranged in order of
decreasing size. This is simply a convention. The size of the chromosomes ranges
from about 200 mb (the largest chromosome) to about 50 mb (the smallest
chromosome). The other pair of chromosomes is the sex chromosomes. The sex
chromosomes are either designated as XX (female) or XY (male).

Let's consider an individual human chromosome such as chromosome 19. This


chromosome contains about 60 x 106 base pairs in length. This is a fairly small
chromosome as far as chromosomes are concerned. The largest human
chromosome is chromosome 1, then chromosome 2, and so on. Chromosomes
can be stained to reveal a banding pattern as shown in Figure 9-23. These
banding patterns are quite specific and they are constant from chromosome to
chromosome. Within these different banding patterns we can assign genetic
markers. The long arm of the chromosome is assigned the letter q while the short
arm is assigned the letter p. Each arm is further divided into regions such as 1, 2,
3, and so on. Each region is divided into band numbers.

band

r
number
13.3 Insulin receptor gene and
tr LDLR gene are located
F
13.2 in this area of the arm
63

r p 1
a region I 3' 1

t
centromere

50 q L
Excision repair gene
is located in this area
of the arm

L Chromosome 19

Figure 9-23

.i.ere are a variety of known genes on this chromosome. For example, the gene
:: low density lipoprotein receptors (LDIR) is located in the P arm while the
:"ne for excision repair (ERCCI) is located in the q arm. The gene for the insulin
:r,.eptor is also located in the p arm.

:. one haploid set of 23 chromosomes there are about 3 x 109 base pairs of DNA.
.rce we are diploid there should be about 6 x'L09 base pairs of DNA in every
:=-1. It has been estimated that there are about 100,000 genes per haploid set of

-:pyright @ by The Berkeley Review 50t The Berkeley Review


Specializing in MCAT Preparation
Biotogy Genetic Infonnation Genetic Loci &

chromosomes. The size. of can range from about 1,000 base pairs
about L,000,000 base pairs. 1g"1"
The function of only about a thousand. out of
hundred thousand genes is known. In the years to colne you probably
witr
hearing a lot about the human genome project. This project'is a irast
underbali
designed to map the. entire human genome. one fr ihe goals of the
Genome Project is to find out the function of all of those genIs.

Copyright @ by The Berkeley Review 302 The Berkeley


Specializing in MCAT
Biology Genetic Information The Pedigree

\Alhen one established a pedigree a number of different symbols are utilized. For
example, a square represents a male while a circle represents a female. If this
male and female matry, the square and circle are joined by a line. If they have
children, an inverted "T" is dropped to their offspring. Their offspring are
attached to this inverted "T" by short vertical lines. The birth order of the
offspring is arranged from oldest to youngest and runs from left to right.
Separate generations will reside on separate horizontal lines. The most ancestral
generation is always at the top. Each generation is numbered with Roman
numerals. The oldest generation is given the Roman numeral I, the next oldest
r,leneration the Roman numeral II, and so on. If a square or a circle is filled in
,r'ith a dark color, then that individual is affected with a given defect. If there is a
siash through the square or circle, that individual is heterozygous for a given
jefect. An example of this symbolism is shown in Figure 9-24.

Heterozygote

Yigure 9-24

r ::a11that when Mendel crossed the ww geno,lype from the F1 generation with
in the F2 generation the gienotypes WW, Ww, Ww, and ww.
:-=U, he obtained
-.:.e flowers with the WW and Ww genotypes were purple while the flowers
--*r the ww genotype were white. We mentioned that this was the characteristic
: i ratio. In human hereditary we are able to observe an individual similar to the
;=rotypes in the F2 generation. We do not know what has happen in the
::-','ious generations (i.e., the F1 generation). What took place in the previous
- -rerations of a human lineage is something that must be deduced. This
: .iuction is based on the rules for pedigree analysis.

ft'enetic Diseases:
-- :ur body we have about a hundred trillion cells and in each cell we have about
- --'-rndred thousand genes. In most humans all of those genes in each of those
:-; rvorks properly from day to day. It is known that one defective gene in one
::.ese cells may be enough to cause a cancer. If an individual inherits a
I :.:.tive gene from one of both of his/her parents, then this may cause a genetic
t'.iease. Today, about 4,000 genetic diseases are recognized.

::i --:r- chromosome 4ssortment during meiosis may lead to genetic


,:. ,rmalities. For exafirple, if there is only one copy of any one of the
',: -.:rosomes (called monosomy), the individual will not survive development.
' ::' arly, three copies (trisomy) of all but chromosom es L3, L5, 78, 21, and 22 is
, --:i and individuals who are trisomic for these chromosomes are severely
, r=-ied. Trisomy for chromosomes 13, 15, and 1,8 causes severe developmental

:rght O by The Berkeley Review 303 The Berkeley Review


Specializing in MCAT Preparation
Biology Genetic Information The Pedigree

problems and such individuals die a few months after birth. Trisomy for
chromosomes 21 or 22 survive to adulthood but are severely affected. The mos;
common from of trisomy is for chromosome 21. This is Down's syndrome. It is
an example of aneuploidy (that condition in which nuclei have an unbalanceo
set of chromosomes--that is, they do not contain an exact multiple of the haploii
number of chromosomes). If the nuclei have a normal complement o:
chromosomes, it is referred to as being euploid. u

I
The incidence of Down's syndrome increases dramatically with the age of the
female. For example, the incidence of having a chitd with tiisomy 21 for mothes I
who are between the ages of 20 and 30 is about 1 in 1,400. For mothers who or-s {
45 years of age the incidence is about 1 in 16 births. All of the eggs that a woma:: {
will ever produced have developed to the level of prophase t uy tne time she rs I
born. some of these eggs then complete meiosis every menstruil cycle. In othe ,l
words, the older the woman is, the older are her eggs that complete Meiosis I ani ni
Meiosis II. It is thought that as time passes there is damage to the eggs and tha: u
interferes with the normal disjunction of chromosome 21.
,{
Nondisjunction of the sex chromosomes can also occur during meiosis. F;r s
example, nondisjunction of the two X chromosomes in females can lead tc a
gametes that are XX or gametes that do not contain an X chromosome (referred. :r,
as simply o). If the female XX gamete combines with a male y gamete, fr*
resulting offspring will be XXY male. This condition is refeired to as
Klinefelter's syndrome. If the o gamete from the female combines with ar.l 1.
gamete from the male, the resulting offspring will be an Xo female. Thi_e r
referred to as Turnerts syndrome.

The many thousands of genetic diseases resulting from single gene defects can be
classified into one of the four following traits:

\ (1) Autosomal recessive


' (2) Autosomal dominant
(3) Sex-linkedrecessive
(4) Sex{inked dominant

Pedigree analysis allows one to determine which of these four classification-c a


given genetic disease falls into. Let's consider some examples of the more
common genetic diseases.

Hemophilia, a blood clotting condition due to a lack of a protein called Factor


VIII, is an example of a sex-linked recessive disease involving the L
chromosome. If a male inherits an allele that has the mutation or Factor VIII, ru
will develop hemophilia. Perhaps the most well documented case invoh-ei
Queen Victoria of England and her offspring. Queen Victoria had 9 childrr-
Two of her daughters were carriers while one son was a hemophiliac. Of her $0
children, grandchildren, and great grandchildren, 19 were males and 21. n e:e
females. Of those 19 males, 10 (i.e., 10140 - 25%) developed hemophilia. Note th,-d:
those 10 individuals who developed hemophilia were all males. None of the 11
female descendants were affected with hemophilia. This is because it L. n
recessive condition.

Consider a sex-iinked disease versus an autosomal recessive disease. If a grl mr


disease effects both sexes equally, then it is likely to be autosomal. If males as
affected almost exclusively, the disease is said to be sex-linked. Roughli' c'-:e

Copyright @ by The Berkeley Review 3o'4 The Berkeley Revicn


Specializing in MCAT Preparatic
Biology Genetic Infonnation The Pedigree

quarter of the children should be affected in either case. Also, both parents may
appear normal.

Consider dominant versus recessive diseases. An individual who is affected with


a dominant genetic trait will have one parent who is also affected. Individuals
affected with a recessive trait may have parents who both appear to be normal. It
is possible with a recessive trait to have one parent who is also affected if that
parent is homozygous for the defect.

An example of a hereditary disease that is autosomal dominant is Huntington's


disease' This disease leads to the progressive deterioration of brain celli and
eventual death. The onset of the symptoms of this disease do not usually
manifest themselves until the individual is in mid-life (i.e., 30 to 50 years of age).
A test is available to determine if an individual will get the disease. As you -an
imagine, this has created quite an issue for those individuals who think they
might be afflicted with the disease. Is it better to know or not to know?

Autosomal Recessive
s-uppose you are a genetic counselor and you are presented with the pedigree
shown in Figure 9-25. In this pedigree there are two couples. This is generatiJn I.
\Ve will number these individuals in these two couples r,2, g, and 4lfrom left to
ngh0. Individuals 1 and 2 are a couple and individuals 3 and 4 are a couple. The
;ouple on the left have 4 children while the couple on the right have 3 children.
This is generation IL Again, the individuals in generation II can be numbered 1
though 7 (from left to right). one member of each family mates (i.e., II-4 and II-5)
end have 4 children. This is generation III. once again, the children are
rumbered 1 though 4 (from left to right).

Il

ilI
rigure 9.25

I- rte that the female atII-2 has a genetic disease while the female at III-1 and the
:-a,le atIII-3 have a genetic disease. Is this disease sex-linked or autosomal and is
: lominant or is it lecessive? Even though our sample is small we can say that
::*. disease is autosomal. \,vhy? The disease is affecting both males and females.
l::s disease is also recessive. \Arhy? The parents of child ll-2 are both normal as
;,:e the parents of children III-1 and III-3. Thus, the disease is autosomal
r*cessive.

:r'right @ by The Berkeley Review 305 The Berkeley Review


Specializing in MCAT Preparation
Biology Genetic Information The Pedigree

The next step in analy-zing this pedigree would be to figure


heterozygotes. since the disease is autosomal and it is recessive,
out the
then the onh-
way that you could get affected individuars would be if both pur"rrt,
ur"
for the disease. Thus, the parents of the siblings in generation iIt ";;;
are both carriers
as are the parents of siblings II-1 through II-4. wh; about
the parents of siblings
II-5 though rr-7? rn this case either parent I-3 or parent t-+ wouta be the
carrier.
It is either one or the other. However, with the information given you do not
know which one is the carrier. [They could both be heterozyfotes and
still not
have any diseased children just by chance!] Those individuaf who
are carriers
are indicated in Figure 9-25.

should the couple I-3 and I-4 have any more children? If one of them is
heterozygous, then there should be no chance of them having a child
with the
disease. However, the couple I-1 and r-2 and the couple II-a and
II-5 should te
advised that if they have more children, they run the risk of having
children ryith
the disease.

C_onsider the siblings in generation IIL \44rat is the chance that siblings III-2 anc
III-4 are carriers of this disease? They have a 2/3 chance of beiig carrier:
Suppose that sibling III-2 matures to adulthood and marries. \Alhat
is the chance
that her children will be carriers? It would be ll2.
rl

For many diseases there are tests that can determine if an individual
is a carris fl
of a given disease. For example, genetic analysis for sickle-cell anemia can i0
easih:
determine if a person has the gene or not ior that disease. There is a tesf t.", ffi
Huntington's disease that is about 95% accurate. A new development that rras ffi
just approved this year by the National Institute of Health is gene e
therapy. T|,Js
?nu oltherapy is accepted for somatic cell lines but not fot ger"m cell lines. \,\'h.,,: T
Consider the sickle-cell gene for a moment. That gene is noipresent just to M
be a::
annoyance for individuals. It has had a tremendous selective advantage
over tne m
years in malarial infected environments by giving resistance to malaria @
to thcr"e
individuals who are heterozygotes for sickle-cell anemia. If you were :r
introdr,rce a normal gene into the germ line of individuals who are M
heterozygotes
for-sickle-cell anemia, and that g"." p.opugated, then resistance to malar:a '@
would begin to decrease. m
lm
ffi
,M

Copyright @ by The Berkeley Review 306 The Berkeley


Specializing in MCAT prep
Biology Genetic Information Central Dogma of Molecular Biology

Up until 1'944 it was assumed that the carrier of genetic information was
chromosomal proteins. During the late 1940s and early tesos it was proven that
DNA was the carrier of genetic information (and not proteins as had otherwise
been assumed). Once it was realized that DNA was involved in the genetic of an
organism it was suggested that the flow of information went from DNA to
protein (DNA -+ Protein). Is the flow of information from DNA to proteins a
direct process? Could information flow in the reverse direction--from proteins to
DNA?

Within a eukaryotic cell there is a defined nucleus and within that nucleus
resides the cell's DNA. synthesis of proteins, however, occurs in the cytoplasm of
the cell. Because proteins are being synthesized in a different -cellular
compartment, it was a direct argument that there was not a direct template of
DNA to protein. What is the intermediate step in this process?

Within the cytoplasm are structures called ribosomes. This was the site where
protein synthesis was occurring. Analysis of the ribosome revealed that it was
composed of two different types of biomolecules--ribosomal proteins (several
iypes) and ribosomal RNA (rRNA). Maybe there was an RNA intermediate
between the flow of information from DNA to protein. where was rRNA
synthesized? Inside the cell's nucleus (and then lateiexported to the cytoplasm).
Thus, it was imagined that DNA could give rise to a paiticular type oi nNa ana
then that RNA could be exported from the nucleus where it would eventually
give rise to proteins. This is almost entirely true but nevertheless somewhat
confusing.

If we look at the analysis of certain proteins-ik-e silk, we wilt find that it is


composed almost entirely of three amino acids--Ser, Gly, and Ala. other proteins
h\: l"f have a very high content of Cys. There are a variety of different proteins
have quite a variety of different amino acids. However, when we analyze
ryhich
the base content of the rRNA in ribosomes from a variety of different organisms,
rve find that they all have an rRNA base content which is practically identical.
This did not seem right. If we had an organism that synthesized pioteins of a
particular type, we would expect to find an intermediate instructi,onal process
that would be different in its base content than we would for an organism with a
r-ery different set of proteins.

Francis Crick suggested that there must be at least two or three different types of
RNA of which rRNA would be only one. His suggestion was taken to helrt in
196lby Francois Jacob and Jacques Monod. They postulated that DNA can also
form a type of RNA called messenger RNA (mRNA) which had the instructions
that resulted in the actual protein. They suggested that mRNA is the actual
iemplate for synthegzing proteins and that rRNA is simply a portion of the
architectural framework of the ribosome.

-{ bacterium does not utilize all of its DNA all the time because it has more
nJormation than it could possibly need at any one given moment. If the food
'upply for this bacterium was changed from glucose to lactose, and back and
Copyright @ by The Berkeley Review 307 The Berkeley Review
Specializing in MCAT preparation
Biology Genetic Information Central Dogma of Molecular Biotogi

forth a few times, that bacterium would adapt and change the proteins it
was
synthesizing. The mRNA made under one set of conditionJ,"iff U" different
from
the nRNA made under a different set of conditions.

For example, when a bacterium is infected by a phage, most of the


operations ot
the host bacterium cease. In 1962Jacob, Meselson,"and sydney Brenner were
able to isolate *RN+ a T2 viral phage. No new ribosomes are synthesized"
{rom
Thus, no new rRNA is being made by the infected host. However, all of a
sudden
the phage DNA will give rise to many proteins that were never seen before
the
infection. These proteins must have been synthesized from mRNA that lt,a_.
m1f9 specifically by the T2 phage DNA. In other words, there is T2 viral specific
mRNA and bacterial specific mRNA--and both of those mRNAs can be translated
on the same ribosome.

IA/hat physical properties should mRNA have? one property is for the
mRNA tc,
have a rapid turnover. If the conditions change in tire cell, you do not want the
I
old mRNA making the same old protein anymore. However, rRNA has a rather
J
long half-life. This is why nobody had discovered up to this point. It had
a very short half-life and was rapidly turning over'RNA
rl

in the ce[. In fac't, only about I


4% of the total RNA in the cell is ever mRNA. Almost all of the RNA in a
cell i_.
rRNA.
il
n
At the time there was not enough evidence to prove this hypothesis of mRNA
fr
being a template for protein s;mthesis. Crick siw this and tus"d this article ot
ffi
faith on what he called a dogma. This lineage of informational flow from DNA
,d
-+ RNA -+ Protein soon became the central dogma of molecular biologr,_
,t
Informational flow from DNA to RNA is called transcription wh"i-le
informational flow from RNA to protein is called translation. Because DNA car
replicate itself we can draw an arrow on DNA as shown in Figure 9-26.
I
m
c
T
DNA : RNA Protein m

U --> im
il
m
Figure 9-26 U
fim

@
M
iln
m
@
@[

,S
m
W
ffi"i
ilh
@
@
GX

Copyright @ by The Berkeley Review 30a The Berkeley Revier


Specializing in MCAT preparation
Biology Genetic Information Functions of Nucleic Acids

nilffiftt$fi $.::::df rffi Uol$ild,i::fi,Uidi


What are the functions of these nucleic acids? Fred Griffith (circa 1928) was
looking at infections caused by the pneumococcus bacterium. This bacterium can
cause pneumonia in mice. The pneumococcus comes in two varieties--the normal
house and garden variety that is surrounded by a smooth (S) polysaccharide
capsule and the mutant variety that does not have this capsule and appears
rather rough (R). The smooth pneumococcus kills mice while the rough
pneumococcus do not kill mice. If Griffith heat-killed the S form of the
pneumococci and then injected the mice with it, the mice lived. However, if he
made a solution of the heat-killed S form mixed with the live R form, and then
rnjected that into the mice, they would die. Somehow the heat-killed S form of
lhe pneumococci transformed the live R form into the lethal S form that is able to
kill mice.

it was not until the mid-1940's that this phenomenon was understood. Oswald
-{very, Colin Macleod, and Maclyn McCarty published a landmark paper based Bacteriophage
rn Griffith's findings that stated that DNA was the transforming factor (genetic
material) and not protein as others had believed.

.r the intervening years it was suggested by Roger Herriot that a bacteriophage


-rke the T2 virus is actually a core of DNA surrounded by a protein coat and that
re virus somehow injects its DNA into a host cell causing that cell to be
::ansformed to produce more phage progeny. This hypothesis was tested by
\lfred Hershey and Martha Chase in 1952. Their findings tumed out to be yet
confirmation that DNA is indeed the genetic material. Figure 9.27
'r-rother

lhe T2 bacteriophage consists of a head which carries its DNA. Surrounding the
:ead is a protein coat. Stemming off of the head is a tail with fibers that have the
::ility to recognize specific markers on a given host, such as the bacterium E. coli.
lljs is shown in Figure 9-27. Once the phage attaches to its host it then injects its
l\A into that host via its tail mechanism (which acts like a hypodermic needle).

,:: the time that Hershey and Chase were doing this experiment they were not
.;re which portion of the phage was being-inlected into the host cell. Was it the
::otein from the protein coat or was it really the DNA? Hershey and Chase had
.::ess to various radioactive isotopes, including 32P and 35S. Other radioactive
j-rtopes include 14C and 3H. Another non-radioactive isotope is 15N. They
:=asoned that the phage DNA could be easily labeled with 32P is because of the
::'osphate atom in the DNA backbone. DNA does not contain any measurable
.::rount of sulfur. They also knew that proteins were composed of amino acids
.:.1 two amino acids had sulfur atoms (Cys and Met). Amino acids do not
:: rtain any measurable amount of phosphate in their structures. Therefore, they
- - rld have proteins labeled with
35S.

-. :u1ture of E. coli bacteria were infected with phage that had been labeled with
llP and 35S" After a brief period of time the infected solution was agitated in a
',-arrng Blendor in order to separate the phage from the bacterium. The solution
:s centrifuged into a supernatant and a pellet. The bacteria were collected from
:: pellet and analyzed for either 32P or 35S. It was discovered that the bacteria
: :riained a high percentage of 32P while the phage protein in the supernatant
..rtained a high percentage of 35S. This was a conformation of the Avery
, ,periment that DNA was the genetic material.

- :yright @ by The Berkeley Review 5()9 The Berkeley Review


Specializing in MCAT Preparation
Biology Genetic Infonnation Flrnctions of Nucleic Acids

fames watson and Francis Crick deduced the structure of DNA based on an x_
ray diffraction photograph taken of DNA fibers by Rosalind Franklin
and
Maurice wilkins during the early 1950's. watson and Crick proposed
that DNA
was a double helical molecule. (Linus pauling, who was utro
*ortir,g on DNA at
th-e time, proposed that it was a triple heiix.) The base pairing scheme that
watson and Crick proposed, adenine (a purine) pairs witf, thymine (a
pyrimidine) while guanine (a purine) parrs with (a pyrimidine), war
slpported by evidence from earlier studies by Erwin"yiorirr"
Chargaff in which he found
thatadenine and thymine ratios and guanine and cytosine"ratios were almostl:t
in all of the organisms that he examined.

ba;e_ pairing of adenine with thymine and guanine with cytosine


Jhe
through hydrogen bonding. Adenine and thymiie have two hydrogen
between-them while guanine and cytosine have three hydrogen bonds.
Note
adenine hydrogen bonds l and 5 positions while thynirne hydrogen br
via. its 3 and 4 positions. "iat
This is shown in Figure s-iaa. similarly) guanin
hydrogen bonds via its 2,1, and 6 positions whiL cytosine hydrogenboLas
vl
its 2,-3, and 4 positions. This is shown in Figure g-2gb. Not ojy do"the
hydn
bonds between the base pairs help to hold the DNA double helix togeiher,
the stacking of the bases also plays a major role due to the interaction
of the
electron clouds between juxtaposed bases.

f\Sure 9.2a
These bases are said to be complementary to one another. Not only is
complementarity between specific bases but the two DNA strands thai cor
the double helix run antiparallel to one another. In other words, one DNA s
will run in the 5' -+ 3 direction while the other strand will run in the 3'
direction. As the DNA double helix winds around an imaginary axis, I

grooves are formed. These two grooves are called the majoigroove and
minor groove. These grooves (as we will later see) make it much easier
certain proteins to bind specific sequences in the DNA duplex. This can
in the simplified DNA double helix shown in Figure g-29.

Figure 9-29

Copyright @ by The Berkeley Review 5lo


Biotogy Qenetic Information Functions of Nucleic Acids

How can we denature a double stranded (duplex) molecule of DNA?


one way
would be to add a denaturing agent. Recail that when urea was added
to
proteins, it denatured those proteins. one way to denature DNA
would be to
change the pH of the solution or even heat the solution. These
procedures will
first denature the hydrogen bonds in the AT rich regions of the duplex
DNA anJ
then in the GC rich-regions. why? Because adenine and thymine
are held
together try only two hydrogen bonds whereas guanine and
cytosine are held
by three hydrogen bonds. If we just denitured the AT rich region
l"-g-"lh".t of a
DNA double helix, it would be a partial denaturation. If we continue
the
denaturation process, we can get complete denaturation of the DNA
duplex.
suppose we have a partially denatured piece of DNA and remove
the heat or
adjust the pH back to normal. \Atrat will happen? one finds that
the DNA duplex
joins back together--a process cailed
renaturition or annealin g. rn
fact, it
out that DNA is constantly denaturing a few hydrogen"bonds and turns
then
reforming those same hydrogen bonds igain--a process"called
breathing. in
other words, DNA is a dynamic structure aid not a rigid
crystal.

If we were to heat DNA srowry, we wourd be able to look for a melting


iemperature (Trn). The Tp is that temperature at which half
of the helica"l
structure of the DNA is rost. As we begin to slowly heat DNA
there will be a
gradual dissociation from double strand-ed DNA to single
stranded
)NA double helix is rich in GC base pairs, it will have"a higher TpDNA. If the
value than
fNA with an abundance of AT base pairs. This can be seen iriFigure r-so.
ilow_is this phenomenon detected? one way would be by viscosity
separation.
-\nother.way would,be by-measuring the reritive absorbance of the"DNi.
if you
..ave a structure such as DNA that is held together
by hydrogen bonds between
:he base pairs, it should be possible to break thor"
r,yarog"r-,", by increasing the
'emperature and allow the two strands to separate. This is ,".r"rJibl"
nelting and can be observed by folowing the absorbance properties "ullua(at
260 nm)
rf the solution in which the meit is taking"place. we can foro# the
absorbance of
lis melt because of the different absorptlon properties between double stranded
DNA_ and single stranded DNA. Dowpk stranded
DNA will have a rower
=bsorbance (by about 40 to 50%) than sifgle stranded DNA.

(J
(.)

-o
!
a

o
&
Tm

Figure 9.3O

lopyright O by The Berkeley Review 3ll The Berkeley Review


Specializing in MCAT preparation
Biology Genetic Information Functions of Nucleic Acids

why is there a change in the absorbance between double stranded DNA ani
single stranded DNA? The stacking of the bases in the DNA double helix is quite
important. This close association of the bases brings about a phenomenon called
hypochromicity. Hypochromicity simply means that the absorption of the total
molecule is less than the sum of its parts. In other words, each of the base pairs
has its own particular absorption spectra, and when we allow all these base pairs
to form the double helix we do not rcalize their full absorptive capability.

When light impinges on these stacked base pairs in the DNA double helix there
is a light induced electronic transition. In other words, certain electrons will be
sent to higher energy levels. Whenever there is an electronic transition, there i.s
polarization. Once one of the base pairs becomes polarized, the ability for the a_r:
adjacent base pair to become polarized is diminished. Thus, when the bases are
tightly stacked, as in the double helix, there is a decreased absorption. This is the
basis for this phenomenon of hypochromicity. If you heat the DNA double helir
and break the hydrogen bonds between the bases, you will get an increase i:r
absorption--an effect called hyper-chromicity.

The Tp of DNA naturally depends on its base composition. We can talk about
that base composition in terms of the percentage of GC base pairs in the Dl\A
duplex of interest. Most mammalian DNAs are about 50% GC. The GC conteu
of the phage T2 is about 35% while the GC content of the bacterium E. coli is
about 50%. If this phage were to infect an E. colibacterium, one could determine
which DNA in the bacterium is actually the bacteria's own DNA and which is tlre
phage's DNA simply by the difference in GC content.
[n
Suppose you were to take a piece of DNA and denature it for awhile by addin*
tu
heat. At some point you stop this process and begin to reduce the temperatura
Would the DNA rehybridize? The answer is yes. If you cool the DNA quicklr
q
you will find that some portions of the single stranded DNA will find each oths
til
Gm
and reanneal. However, there will still be a great deal of single stranded DNA
qW
that did not renature. If you cool the DNA slowl/, then there will be more ti_u'.e
for the single strands of DNA to find their partners and join back together agairu
rw
dro
This is shown in Figure 9-32. The more complex the DNA, the less perfect will b'e
the renaturation process.
,&m

Single
o Stranded
C)
DNA
Lr Cool
U) Quickly

O Double
Cool
Stranded
Slowly
O DNA
&
lncrease T Stop Decrease T

Figure 9-52

Once DNA was proposed to be double helical in nature, the question off
replication immediately surfaced. How is it that DNA can replicate itself? There

Copyright @ by The Berkeley Review 512 The Berkeley Revic


Specializing in MCAT Preparatior
Biology Genetic lnformation Functions of Nucleic Acids

are several possible modes. Two of them are the conservative mode and the
semiconservative mode as shown in Figure 9-33.

Parental
DNA

Daughter v-- v
\\
GG
DNA

!=
Conservative
(a)
\\
/:
--:
,/-:
=-=
Semiconservative
(b)

Figure 9-33

in the conservative mode of DNA replication the parental strands remain


together after replication of two new daughter strands. This is shown in Figure 9-
33a. In the semiconservative mode of replication (proposed by watson and
Crick) the two parental strands separate and two new daughter strands are
svnthesized off of the respective parental template. The parental strands do not
:ejoin but rather remain with the newly synthesized daughter strands. This is
shown in Figure 9-33b.
./
It turns out that DNA is rcplicat{semiconservatively, as watson and crick had
hvpothesized. This was proven by Matthew Meselson and Franklin stahl in
1957. Meselson an{ stahl grew the bacteriurn E. coli in an ordinary medium
containing laNH+Cl. The nitrogen in this compound, 14N, is the regular house
and garden variety nitrogen. They also grew a different batch of E. coli tn a
medium with heavy nitrogen, 15N. When new DNA is synthesized these
nitrogen atoms (14N o.11551; will be incorporated into the structure of that
molecule. The light DNA (containing 1aN) could be separated from the heavy
DNA (containlng 15N) by the use of density gradient sedimentation.
"qnilibti.r-
-\ gradient ranging from 1.66 to'1,.76 g/cm3 was established in a concentration of
cesium chloride (CsCl). The density of the house and garden variety DNA is
about 1.7 g/ cm3. Two bands were clearly visible as shown in Figure 9-35a. This
ivas the starting point{or their experiment.

\4eselson and Stahl grew some E. coli a medium with 15N for many generations.
The DNA in these bacteria thus contained 15N in their DNA. Once they had a
batch of E. coli will all 15N in their DNA they started the experiment. At time
zero they switched the 15N to 14N in the growth medium (see Flgure 9-34).

Copyright @ by The Berkeley Review 3r3 The Berkeley Review


Specializing in MCAT Preparation
Biology Genetic Infororation Flrnctions of Nucleic Acids

E. coli
ttN
O
l' N

'o*/''*O O o."?i"',,""

/\
OOO
toVtt* toN toN toVtt*
Figure 9.54

Now, in each subsequent generation the E. coli would be utilizing laN instead
r5N. Analysis of the DNA afte-r one generation of growth (E. coli replicatic
revealed a hybrid DNA band(lap1lsp; between thaiof the 15N DNA band a
the 1aN ONA band. This is shown in Figure 9-35b and implies
semiconservative mode of DNA replication. If the mode of DNA replication
been conservative you would still expect to see two bands just like that shown i
Figure 9-35a. Why? Because the heavy band would be from the parental s
while the light band would be from the daughter strands (see Figure 9-33a).

toN toN
toVtt" toVtt^
ttN

@ E
(a) (b)

Figure 9-55

What would the bands look like after two generations of E. coli
Again, if this were a conservative mechanism, you would still see two bi
One would be heavy 115N; and the other would be light (14N), except that
there would be three times as much 14N as 15N. tf the mode of replication
semiconservative, then you would find one all 1aN band and one hybrid 151
band as shown in Figure 9-35c. Thus, we find that the mode of DNA repli
is indeed semiconservative.

Copyright @ by The Berkeley Review 3t4


Biology Genetic Information DNA Polymerase

ul$ iffiul#ffic$fip$
DNA replicates semiconservatively. rn 7955 Arthur Komberg and his colleagues
incubated extracts of E, coli with radioactively labeled deoxyribonucleoside
triphosphates at the a-phosphate position. They discovered that a very small
amount of nascent DNA was synthesized. The enzyme that catalyzes this
:eaction was called DNA polymerase I (since it was the first such enzyme
liscovered).

-\'hat is a deoxyribonucleoside triphosphate? Let's consider the structure of


J.eoxythymidine as shown in Figure 9-36a. There are a few important features to
::ote about this molecule, so we will consider them one at a time. The sugar in
lNA is deoxyribose, and in particular it is B-D-2'-Deoxyribose. The "deoxy"
:erm simply means that the ring structure lacks an oxygen atom at the 2'-carbon.
lhe C-1 carbon of the ribose ring is bonded to either the N-9 nitrogen of a purine
iase or the N-1 nitrogen of a pyrimidine base by an N'glycosidic linkage. This
-:r'ukage was formed via a dehydration reaction which means that it can be
:';drolyzed. In the case of deoxythymidine it is bonded to the N-1 nitrogen of the
::trogenous base thymine. This structural unit, the ribose ring bonded to either a
::.rrine or a pyrimidine, is called a nucleoside (note the "side" ending). In DNA
:rere are four nucleosides: deoxyadenosine, deoxyguanosine, deoxythymidine,
urd deoxycytidine.

(b) o
o il Acid Anhydride
Thvmine ll O-P-O.
"C tot',O ^ Linkage
-- \c -cH.
"-"til
o:c \ ..c-H
-.!
U ./\
o/ "!ao o
H-N

o-c -*-"-n
I
-" -"-"t,
il
N
,,,

;v"
o N-glycosidic
$ linkage

4',
Phosphomonoester
I'
ftD-2'-Deoxyribose
Linkage

HOH HOH
Deoxythymidine Deoxythymidine- 5'-triphosphate
(a nucleoside) (a nucleotide which can be abbreviated as dTTP)

ffiEure 9-56

I ;",-e add a phosphate group to the Slhydroxyl function of the ribose ring of any
,: :,.!r nucleosides, we will form a phosphate monoester bond (Figure 9-36b).
lhe name of our molecule now changes from a nucleoside to a nucleolic[g (note i

:e 'fide" ending). Be careful of what you mean when you use the "side" and the
':lie'endings. For example, we could name one of these molecules as a
,lr*,rlwribonucleoside$'-monophosphate (or as a deoxyribonucleotide). There
m';l also be two or three phosphate groups attached to the S'-carbon of the
'T:5,:se ring. Recall from organic chemistry that two phosphate groups are
unfra:hed to each other by a phosphoanhydride linkage. If we added two
:'!:ti:sphates to our molecule, we would call it a deoxyribonucleoside-5'-
ru,nt'-osphate. If we added three phosphates to our molecule, we would call it a

i iru'-ight @ by The Berkeley Review 315 The Berkeley Review


Specializing in MCAT Preparation
Biology Genetic Information DNA Polymerasc

deoxyribonucleoside-5'-triphosphate. This can be abbreviated as dNTp. The four


deoxyribonucleoside triphosphates (dNTps) of interest to us are dATp, dGTB
dTTP, and dCTP.

DNA Polymerase I adds about 20 deoxyribonucleotide residues (e.g., dATp,


dGTP, dTTP, or dCTP to the 3lhydroxyl function of a pre-existing DNA strand.
what this means is that a primer is needed in order for DNA polymerase I to
function. The dNTPs are called for by a DNA template and aie added to the
newly synthesized DNA chain in 5' -+ 3' direction (which means the DNA
template is read in the 3' -+ 5'direction). The incoming dNTp will hydrogcr
bond with its complementary base and then there will be a nucleophilic attack by
the 3'-hydroxyl of the primer strand on the o-phosphate of the incoming dNTp.
A phosphodiester bridge is formed between the a-phosphate of the incoming
dNTP and the 3'-hydroxyl function of the ribose ring of the primer strand
Pyrophosphate is released and subsequently hydrolyzed in order to help drive
this reaction to completion. This is shown in Figure 9-37. Note that the
phosphodiester bond will only be formed if the incoming dNTp is
complementary to its respective base on the template strand.

Primer
I
o
I Bo*"
HuCr
T l,r"{ T

H
E E
M M
P P
L OH L
A A
I
T O-P=O T
E E

-H
I
o
I Bare

HOH

Figure 9-57

Copyright @ by The Berkeley Review 3t6' The Berkeley


Specializing in MCAT
Biotogy Genetic Information DNA Reptication

D'N ...,.,,,.,'.iffilflllilitiiffi
,,.1r ,,..._.,,1:u.....,.

DNA is-_ composed polydeoxynucleotide strands arranged in an


:j_ l-o
antiparallel fashion. DNA- is uzually a right-handed helix with the
iurine and
pyrimidine bases arranged on the inside of the heli* while the deoxyribose
and
phosphate moieties are arranged on the outside of the helix. The DNA
double
helix can be represented as shown in Figure 9-3ga or in Figure 9-3gb.
Both
representations will give us similar information

bp = base pairs
(A=T or G=C)
c##"
trH:::
Single

(a) (b)

Figure 9.iB

-\ great deal of structural information about the DNA double helix can be gained
rrom X-ray analysis. For example, there are two forms of DNA which
can Le in a
right-handed helix. They are the A-DNA and the B-DNA (which was proposed
bv watson and Crick in 1953). Recall that we have mentioned that the
B:DNA
containsa major and a minor groove (see Figure 9-3ga). proteins that recognize
specific DNA sequences can gain access to the DNA double helix via
the riajor
groove.

There are a number of bonds in the DNA polymer about which rotation
can
For example, the backbone of the DNA polymer can be rotated about 6
:rccu_r.
bonds in each monomeric unit. Many of the differences between the A-DNA and
the B-DNA arise from the different conformations of the ribose ring.

Recall that the ribose ring in DNA can also be called


B-D-2-Deoxyribofuranose.
These furanose rings are not planar. They can pucker. In the A-DNA the minor
Sroo-ve essentially vanishes due to s specific puckering of the furanose ring. This
results in the base pairs of the A-DNA helix being tilted away froin the
perpendicular axis by about 19 degrees. However, becalse of a different type of
puckering, the B-DNA has its bases arranged perpendicular to the axis of the
relix, thus allowing foq. the distinct characteristics of the major and minor
grooves as shown inlig;re 9-3ga.

The type of helix that is found in A-DNA is also found in regions of double
stranded RNA (e.g., involving hairpins) and in RNA-DNA hybrids.

Copyright @ by The Berkeley Review 317 The Berkeley Review


Specializing in MCAT preparation
Biology Genetic Information DNA Replication

NHz

Adenine#i
N-Glycosidic
I Bond
o-cH2

Syn Conformation Anti Conformation


Left Handed Helix, Z-DNA Right Handed Helix
Purine is Syn, Pyrimidine is Anti Either A-DNA or B-DNA

(a) (b)

Figure 9.39

A third type of DNA helix involves rotation about the N-glycosidic bond that
connects the furanose ring to the base. This type of DNA is called Z-DNA
(because the phosphates in the backbone zigzag due to a repeating dinucleotid.e
unit rather than a mononucleotide unit). Z-DNA is a left-handed helix and
when the pyrimidine bases and the ribose units are far apart it is in an antil
conformation while it is in a syn conformation when the purine bases and the
ribose units are close together. This is shown in Figure 9-39a. The N-glycosidrc
bonds in both A-DNA and B-DNA are in the anti conformation. This is shown in
Figure 9-39b.

DNA does not have to be linear, rather it can be covalently joined at its ends 1

forming a circular structure. For example, the DNA found in mitochondria an'j.
in the chloroplasts of plant cells is circular. The topology of both linear and ,1

circular DNA is rather interesting. DNA is typically right handed. If we were to u

twist this DNA molecule around its own axis in the right-handed direction, tha: il

we would introduce into that double helical structure a phenomenon known a-"
"t
positive supercoiling. Conversely, if we were to twist this DNA molecule
around its axis in the left-handed direction, then we would introduce negative M

(!
supercoiling. Supercoiling greatly changes the overall form of the DNA not onk-
by making it more compact but also by altering accessibility to the major and M

minor grooves. m

ffi

The number of times that one DNA strand can be wound around another DNA t
strand is referred to as its linking number (L). Topoisomers are DNA molecule: s!
that differ only in their linking number. The degree of the linking number in
DNA can be altered by enzymes called topoisomerases. Type I topoisomerases t
reversibly cleave one strand of DNA and relaxes negatively supercoiled DN-{ A
while Type II topoisomerases (e.g., DNA gyrase) reversibly cleaves both strands ff
of DNA and adds negative supercoils. tu
a
In order to introduce a supercoil into a DNA double helix it costs energy. For M
example, DNA gyrase is a topoisomerase (Type II) found in E. coli which m
converts the potential energy of ATP into the torsional energy of a negativeh
supercoiled structure. It turns out that negatively supercoiled DNA prepares the

Copyright @ by The Berkeley Review 318 The Berkeley Review


Specializing in MCAT Preparation
Biology Genetic Information DNA Reptication

DNA duplex for processes such as replication, recombination, and transcription


where the separation of the two helical strands is required.

In 1958 Arthur Kornberg discovered the enzyme DNA polymerase I which


plays
a crucial role in the replication and repair of DNA. This enz)rme adds about 20
deoxyribonucleotide-residues 1e.g., dRTt, dGTp, dTTp, or dcrp (which
collectively can be called dNTps)) to the 3'-hydroxyl function of a pre-existing
DNA strand.

Primer
I primer
o I

'2"{ Base ----- Brse


2Pi
o

"'i"!
".J
t
Base

"rJ

H )'H
T T
E E
M M
H P P
('on _ L L
\ro
(]-P-o
A A
T o-P-O T
@ o
I E I E

-T" "ro
o
d I
HII
Base ---.. Base I Base

t/"!
r
HzC

H HOH

Figure g-4O

ivhat this means is that a primer is needed in order for DNA polymerase I
to
runction. The dNTP's are called for by a DNA template and are added
to the
newly synthesized DNA chain in 5'-+ 3' direction (which means that the DNA
template is read in the 3' -+ 5'direction).

The incoming dNTP will hydrogen bond with its complementary base and then
lhere will be a nucleophilic attack by the 3'-hydroxyl of the primer strand on
the
n-phosphate of theljncoming dNTPi phosphodieiter bridge is formed between
:he cr-phosphate of the incoming dNTp and ihe 3'-hydroxyllunction of the ribose
:ing of the primer strand. Pyrophosphate is released and subsequently
hydrolyzed in order to help drive this reaction to completion. This is shtwn in
Figure 4-40. Note that the phosphodiester bond wifl only be formed if the
ncoming dNTP is complementary to its respective base on the template strand.

Polymerase Chain Reaction


\ laboratory procedure called the polymerase chain reaction (pcR) is having a
significant impact on what can be done with DNA. suppose we have a relativ6ly
iong piece of double stranded DNA as shown in Figure 9-41. within this DNA is
a portion that we wogld like to examine. However, we do not have very
much of
fiis DNA available to us. The idea is to amplify the segment of pNA we are
nterested in so we can studv it.

Copyright @ by The Berkeley Review 319 The Berkeley Review


Specializing in MCAT preparation
Biology Genetic Information DNA Replication

We want to examine this area of the DNA.

Figure 9-41

The concept of the polymerase chain reaction would only have been possible
after the discovery of one of the organisms that grows in hot springs. These
organisms (e.g., bacteria, fungi, algae) are thermophilic--they love heat. ln
particular, there is a bacterium called Thermoaquaticus (Taq) which has a DNA
polymerase referred to as the Taq DNA polymerase. This polymerase is stable at
temperatures up to 90 "C and does not denature at that temperature. With this
polymerase in mind, let's amplify the DNA in Figure 9-41.

Suppose we know some base sequences as outlined in Figure 9-42a.If we were tc


somehow denature that area of the DNA, we could manufacture complementan-
base sequences with a free 3'-OH function at the end. These complementan-
sequences are made such that their 3'-OH functions point inward. In other
words, we can synthesize a primer as shown in Figure 9-42b.

How can we do this? If we heat this DNA to about 90 'C, the ends will separate
first. We can then add the short chain DNA primers that are complementary to
the base sequences of that portion of the DNA that we already know. If we cool
the solution to about 50 'C, our primers will hydrogen bond to their complement
areas as shown in Figure 9-42b.

We want to examine this area of the DNA.

ll|
.C.T. T.A-C.T.

lt
.G.A. ArT-G-A.

(a) I H"ot on,l cool in the


I pr.r"n". of excess
artifical nrimers
I
3' -C-T-T.C T-A-C-T.5'
.i-i-i.C-on s'
Primer Primer
3'HO-.T-A-C-T.
llrl
5' .G-A-A A-T-c-Ar J'

(b)

Figure 9-42

Once we have our primers in place we can then add dNTP's and the Taq DNA
polymerase. This enzyme will add the dNTP's as shown in Figure 9-43

Copyright @ by The Berkeley Review 32o The Berkeley Revien


Speciatizing in MCAT Preparation
Biology Genetic Information DNA Replication

Remember, the Taq DNA polymerase is stable at high temperatures (around 70


'C). By this time we have twice as much DNA as we did before.

3' TrA-CrT- 5t

Primer Primer
3'HO_-T-A-C-T-
.G-A. trrl
A- T-G-A-

I t"o o*o Polymerase


dNrP's

I
A-C-T- 5,
'' :E:i:l:
Primer

.G.A- 3'

Figure 9-45

But we still want to amplify our DNA in order to get a better yield. How do we
amplify the DNA that we have produced in Figure 9-43? we heat the DNA to 90
'c in the presence of our primers (which we have in excess). The DNA strands
will separate and the primers will hydrogen bond as before. we now have four
single strands of DNA with their respective primers. This is shown in Figure 9-
44. These four strands correspond to their respective strands from Figure 9-43.

3' -c- T- T-C- Original DNA


TrA{rTr 5,
5' -i- [- A-C-o" r'
Primer

5, -G- A- A-c ***rr*-**o*rrJ-*ti$ll{*li*ti{- c* A {


Aa. T.\ 3'
tttt 5'
3'HO_.T-A-C_T-
Primer

3' \- c $ T s 1 *' g,-r-rr*rrrrr***P*Sitili{{l\Tili{ A{-T. 5'


5' -i-tr-A-C-oHs'
Primer

S'.G-A.A-G A-T.G.A. 3'


tltt
3' HO- -T-A{-T. 5'
Primer

Figure 9-44

Taq DNA polymerase in the presence of the dNTP's will once again synthesize
new DNA. This is shown in Figure 9-45. At this point we have now made four

Copyright @ by The Berkeley Review 321 The Berkeley Review


Specializing in MCAT Preparation
Biology Genetic Inforrnation DNA Replication

times as much DNA. This process can be continued indefinitely to amplify the
DNA.

3'-c-T-T-C
tt
5' -G-Ar

5' \\\\t\\\\\\\\\a\' -
3' ,,111111t,,,,,,,,,t,,,,,,,,r!rrrrrr*arrrrrrrrrrrtrrrrrrrrrrrrrrrrrrrrr,,,,,,,,,,,,,,,,,,,,,,,,, f T-
t - A-i tlf : i,
3' *-Qr. frJxQsrmnrrrrurrrrrrsrllrrnlrrrru T-A-C-T.
lllr 5'
5t -G- A- d-Q trttttttrtrtrrrrrrrrrrrrurluunrnrr'lrrrrrrrr ,,.,.,,,,,,,,.,,,,..........l|trr,.-
3'

5'-G-A-Arc- A-T-G-A. 3,
3, .rllllll,,rlrrrr,,,,,,,,,,rtttttrtrrrrllrrrrrrrtt*t.trrrrrrrrrrr"rrrr.rrrrrrrr,,,,,,,,,,,r,,,,,,,,,,,,,{-l-J-{- t'

Figure 9-45

one of the problems with this process is that Taq DNA polymerase does not
have 100% fidelity. Once in awhile an incorrect baies will be added. Flowever,
the number of errors will be quite small compared to the quantity of DNA tha:
can be obtained. A general temperafure cycle for the polymerase chain reaction is
shown in Figure 9-46.

Condition for
denaturation

o920 !
il
o
a {
d
H m
o.
m
o
F W

il

Time (minutes)

Figure 9-46
--wnu'---..-.-..-t
are some of the applications of the polymerase chain reaction? one
application involves analysis of the DNA from organisms which have been
deceased for a long time. For example, the tissue of ancient Egyptian mum:nies
can be analyzed for their DNA content as well as ancient bones from various
burial mounds throughout the world. DNA from these organisms can then be
compared with the DNA from modern organisms can to see if there has been a
significant change (evolution).

Unwinding of Supercoiled Parental DNA


The synthesis of new daughter DNA is coupled to the unwinding of parentarn.
DNA at sites called replication forks. DNA replication does not begin random-E
but rather at sites referred to as an origin (or ori for short). For example, in E. ;:rfu
DNA replication will begin at a locus called ori C. In order for the DNA doubre
Copyright @ by The Berkeley Review 322 The Berkeley Revier
Specializing in MCAT Preparatin
Biology Genetic Information DNA Keplication

helix to unwind, DNA gyrase negative supercoils ahead of the advancing


replication fork. This is shown 1d{s
in Figuie S-aZa.itNhy? As the replication fork i!
initially unwound, positive supercoils are introduced thus making DNA
separation rather difficult.)

(a) 3',

DNA
DNA Gyrase adds Gyrase
negative supercoils

Origin of Helicase unwinds


<-o"oJl..iion--1> DNA (ATP-driven)

Template

\)1
DNA sinele-srranded
bindin! proreins

Figure 9.47

Once the negative supercoiling is introduced, a protein complex


called a helicase
binds to the ori site and catalyzes the ATp-driven unwinding of
the duplex
DNA. single-stranded binding proteins then stabilize the unwound portion
of
the parental DNA. This is shown in Figure g-47b.

\Ahen a replication fork has been created, both strands of the


parental DNA serve
as templates for the synthesis daughter DNA. Becausl DNA is replicated
?f.tl"ry
semiconservatively (proposed by watson and Crick in 1953 and
proved by
Meselson and stahl inr9s7), and because the DNA strands are
antiparalel, thl
synthesis of new daughter DNA must be in the 5' --+ 3' direction
for one strand
but in the 3' -+ 5' direction for the other strand. Synthesis of one daughter
strand
Ln the 3'-+ 5'direction seems to be a paradorbecause
of the faci that DNA
polymerase will only synthesize new DNA in the
5'+ 3'direction. This is shown
Ln Figure 9-48.

"Appears" to grow
3'-r 5'

Copyright @ by The Berkeley Review 323 The Berkeley Review


Specializing in MCAT preparation
Biology Genetic lnformation DNA Replication

Continuous and Discontinuous DNA Synthesis


This problem of one daughter strand of DNA paradoxically growing in the 3' -+
5' direction was tesolved by Reiji Okazaki (ckca 1969).

Parental As the replication fork proceeds in one direction, one daughter strand is
{- nN.q,
synthesized continuously in the 5'-+ 3'direction. This daughter strand is called
the leading strand. The other daughter strand is called the lagging strand
because it is synthesized discontinuously. Even though the lagging strand is
synthesized discontinuously, it is synthesized in the 5' -+ 3' direction (as shown
in Figure 9-49).
Primases
( How is the lagging strand synthesized? when the DNA template has been
11- exposed at the replication fork, nascent DNA cannot be made until a primer is
synthesized.

Figure 9-49

Pri

Figure 9-5O

An enzyme called a primase (complexed to other polypeptides to form arL


* aggt"gut" called a primosome), is a specific RNA polymerase that synthesizes
about 5 nucleotides of RNA which are complementary to one of the DNA
templates. DNA polymerase cannot start the synthesis of nascent DNA (we
rvifl
come back to this point later)' This is shown in Figure 9-50'

DNA PolYmerase Itr lloloenzYme


Once the primer has been created, the enzyme DNA polymerase III holoenzymc
enters the scur,e and catalyzes the synthesis of about a thousand phosphodiester
bonds (i.e., about one thousand dNTP's have been used) before it dissociates
from the DNA temPlate.

.W}yisDNApolymeraselllusedinsteadofDNApolymerasel?D\A
" D\A
-{olymerase III is quicker and it can add more dNTP's to the growing
itrina.Recall that we mentioned that DNA polymerase I only adds about lO
dNTP's to a pre-existing DNA chain'

This segment of RNA primer and newly synthesized DNA is called an OkazaE
fragmeit. As more Okazaki fragments are synthesized in the 5' --> 3'direction flt
324 The Berkeley
Copyright @ by The BerkeleY Review
Specializing in MCAT Pre
Biology Genetic Information DNA Reptication

eventually leads to overall growth of the daughter DNA in the 3' -+ 5' direction.
This is shown in Figure 9-51.

f
Growing
3',

DNA
Replication Gyrase
Fork

Hel
Primosome
DNA Polymerase III (Primase)
Holoenzyme

Fragment

Parental
ok
Fragment A
3', \5'

Figure 9.51

DNA Polymerase I and DNA Ligase


DNA polymerase I has a 5' -) 3' exonuclease activity that allows it to remove the
short segments of RNA primer (starting from the free end). \A/hen the primer has
been removed, DNA polymerase I then adds deoxyribonucleotides to the free 3'-
hydroxyl function of the chain undergoing elongation.

t
Growing DNA
Replication Gyrase
Fork
DNA Polymerase I uses its
5'ro 3'
exonuclease activity
to remove RNA primer

DNA Polymerase I next


Parental adds dNTP's to
DNA where RNA primer once
Nascent was located. t P
3', DNA
nue ligase.ioins/
the fragments.

Figure 9-52

once the gap that was once occupied by the RNA primer is filled with dNTps,
DNA ligase will join the free 3'-hydroxyl group of new daughter DNA just
synthesized by DNA polymerase I with the S'-phosphate group of the okazaki
fragment just ahead of it. Remember, DNA ligase only joins chains of DNA that

Copyright @ by The Berkeley Review 325 The Berkeley Review


Specializing in MCAT preparation
Biology Genetic Information DNA Replication

are double-stranded and not two molecules of single-stranded DNA. This is


shown in Figure 9-52. In bacterial cells the source of energy for this reaction
comes from NAD+ while in animal cells it comes from ATP.

Replication Forks (revisited)


The main chromosome in the prokaryotic bacterium E. coli is circular (there are
also minor chromosomes called plasmids). Recall that we have mentioned that in
the E. coli replication starts at a unique site called ori C (an origin of replication)-
In Figure 9-47a we drew a segment of DNA and located the origin of the
replication fork. In Figure 9-47b we proceeded to draw a typical replication fork
Flowever, what we did not show was that replication can be bidirectional. In
other words, as the parental DNA is opened, replication will proceed at two
replication forks simultaneously. This can be seen in Figure 9-53.

Parental DNA

DNA Gyrase
+ adds negative
Origin of supercoils

6-\
Replication
Fork
Growing Growing
Origin {- Replication + Replication{
o*r,
I t
V'
Fork Fork

6;.r"N
Growing
Replication
Forks I
Helicase\**y'
/il Figure 9.53

+ ) Origin Eukaryotic chromosomal DNA is a linear polymer and undergoes active Dl


replication during the S-phase of the cell's life cycle. Because the DNA
1
eukaryotic cells is quite long compared to E. coli's DNA, it is repli
bidirectionally from many origin sites as shown in Figure 9-54. This ensures
\ / there will be rapid replication of the genome.

5' 3',

Figure 9-54

Copyright @ by The Ber\eview 326 The Berkeley


Specializing in MCAT
Genetic
Information
L5 Passages

100 Questions

Passage Titles Questions


I. HbA/HbS Gel Electrophoresis 1-5
il. Gregor Mendel and Inheritance 6-11
ilI. Incomplete Dominance and Codominance 12-11
IV. DNA Replication 18-23
V. Color Blindness 24-29
vI. Alleles and Coat Color 30-36
vII. DNA Calculations 31 -43
VIU. Semiconservative DNA Replication 44-50
IX. DNA Structure 51-58
X. Mitochondrial DNA (mtDNA) 59-65
XI. Viral Complementation 66-72
xII. Atrial Natiuretic Peptide Experiment 73-79
XIII. Griffith's Pneumococcus 80-87
XIV. Drosophilia eyeless Gene 88-94
xv. Meiotic Nondisj unction 95 - 100

REKI{EIEY
l)n.n,.v.r.ilw'
Speciahztng in MCAT Preparation
Suggestions
The passages that follow are designed to get you to think in a conceptual manner about the processes
of molecular biology at the organismal level. If you already have a solid foundation in molecular biology,
many of the questions you read here will seem to be very straight forward and easy to answer. But if you
are new to the subject or if you have not had a pleasant experience with molecular biology in the past,
some of them might appear to come from the void that spreads out beyond the Oort field at the edges of
our solar system.

Pick a few passage topics at random. For these initial few passages, do not worry about the time. just
focus on what is expected of you. First, read the passage. Second, look at any diagrams, charts, or graphs
in it. Third, read each question and the accompanying answers carefully. Fourth, answer the questions
the best you can. Check the solutions and see how you did. \A4rether you got the answers right or wrong,
it is important to read the explanations and see if you understand (and agree with) what is being
explained. Keep a record of your results.

After you feel comfortable with the format of those initial few passages, pick another block of
passages and try to do them in one sitting. Be aware that time is going to become important. On average,
you have about 1 minute and 15 seconds to complete a question. Be creative in how you approach this
next group. If you feel comfortable with the outline presented above, fine. If not, then try different
approaches to a passage. For example, you might feel well versed enough to read the questions first and
then try to answer some of them, without ever having read the passage. Maybe you can answer some of
the questions by just looking at the diagrams, charts, or graphs that are presented in a particular passage.
Remember, there are many effective learning styles. You need to begin to develop a format that works
best for you. Keep a record of your results.

The last block of passages might contain at least a few topics that are unfamiliar even to those who
know a good deal about molecular biology. Find a place where the level of distraction is at a minimum.
Get out your watch and time yourself on these passages, either individually or as a group. It is important
to have a feel for time, and an awareness of how much is passing as you try to answer each question.
Never let a question get you flustered. If you cannot figure out what the answer is from information
given to you in the passage, or from your own knowledge base, dump it and move on to the next
question. As you do this, make a note of that pesky question and come back to it when you have more
time. When you are finished, check your answers and make sure you understand the solutions. Be
inquisitive. If you do not know the answer to something, look it up. The solution tends to stay with you
longer that way. (For example, what ls the Oort field, anyway?)

The estimated score conversions for 100 questions are shown below. At best, these are rough
approximations and should be used only to give one a feel for which ballpark they are sitting in.

Section lX
Dstimated Score Conversions
Scaled Score Raw Score
>13 80 - 100
n-12 '70 - 79
9- l0 60-69
7 -8 50-59
5-6 40-49
<4 0-39
Biology nbAlllbs Gel Electrophoresis Practice Passage I

Passage I (Questions 1-5) Protei n Protei n


sample s ample 2

Human adult hemoglobin (HbA) is a tetrameric H E


protein, which has a molecular weight of approximately tffii U{
W
65,000 amu. This protein consists of two (identical) a- W
globin polypeptide chains containing 141 amino acids Cathode C
*w w

each, and two (identical) p-globin polypeptide chains NU


tt
containing 146 amino acid residues. Each globin tto
Ilo
polypeptide contains one iron atom (55.8 amu), and each
iron atom can complex with one molecule of molecular Separated
ll I
tt 5

oxygen (32.0 amu).


components
ll3
|#
Large proteins like hemoglobin can have many acidic
Anode O
ll s
JL o
and basic groups, giving the molecule both negative and
positive charge. These types of molecules are called Lane I Lane2
polyampholytes, and they will always have an isoelectric
point (pI) where the net average charge is zero. The pI of Figure 2. Separation of protein samples by PAGE.
HbA is 6.9 An ampholyte that has a net charge of zero is
called a zwitterion.

Sickle-cell hemoglobin (HbS) and the wild-type HbA


differ from one another by the substitution of a single
amino acid residue at position 6 in the B-globin chain.
Individuals with normal adult hemoglobin have the
hemoglobin composition ozAFzA. Individuals with sickle-
cell trait have both HbA and HbS and a hemoglobin 1. What is the approximate percentage of iron rn
composition of a2Ap2A and cr2AB2s. Individuals
with hemoglobin?
sickle-cell disease have just HbS and a hemoglobin
composition of ozABzs. The relevant portions of the two A. 0.09 7o

beta chains for both HbA and HbS are shown in Figure 1: B. 0.17 vo
c. 0.26 vo
D. O.34 7o

HbA H3N-Val-His-Leu-Thr-Pro-Glu-Glu-Lys-
o
HbS H3N-Val-His-Leu-Thr-Pro-Val-Glu-Lys-

Bl 2 3 4 s 6 1 8 t HbS contains a valine residue at the p6 position,


while HbA contains a glutamate residue.
Figure 1. The amino acid sequence ofthe B-chains for both
HbA and HbS.
oifl o
@Yfl o
H3N-C-C-O
I

This difference makes it easy for different proteins to HlN- C- C- O CH,


I
be separated from one another by a process called H3C- CH CH,
polyacrylamide gel electrophoresis (PAGE), diagrammed l-o
CH: O=C-O
in Figure 2. The gel is formed from the polymerization of
acrylamide and N,N'-methylenebisacrylamide and then Valine (Val) Glutamate (Glu)
suspended between an upper and lower buffer solution. pKa= 4.4
The buffer solution usually has a pH of about 9 and is the
same in both reservoirs, as well as in the gel itself. In HbS, the pI is expected to be:
Different protein samples are placed in designated wells
in the gel, and a direct curr€nt of 300 volts is applied for A. equal to the pI for HbA.
about 60 minutes. After separation, the protein bands in B. slightly greater than the pI for HbA.
the gel can be visualized by a variety of techniques (e.g., C. slightly less than the pI for HbA.
staining or UV inadiation). Electrophoretic separation is D. representative of a nonzwitterionic species.
quite important in clinical laboratories, as it can be used
to identify abnormal hemoglobin patterns.

Copyright @ by The Berkeley Review 329 The Berkeley Review


Specializing in MCAT Preparation
Biology IlbA/HbS Gel Electrophoresis Practice Passage I

3. If HbA and HbS undergo gel electrophoresis at a


buffered pH of 9, then the protein band shown in
Lane 2 of Figure 2 represents:

A. HbA, because of its movement towards the


negatively charged anode.
B. HbS, because of its movement towards the
positively charged anode.
C. HbA, because of its movement towards the
positively charged anode.
D. HbS, because of its movement towards the
negatively charged anode.

4. Changes in the nucleotide sequence of DNA


If each amino acid in a protein
represent mutation.
is coded for by three bases in DNA, then the
difference between HbA and HbS must be the
result of a:

I. base-substitution mutation.
u. base-addition mutation.
III. base-deletion mutation.

A. I only
B. II only
C. III only
D. II and III only

5. Sickle-cell trait is a genetic disorder inherited as an


autosomal recessive disease. If a man has sickle-
cell trait and marries a woman who is heterozygous !
for a type of hemoglobinopathy referred to as HbC, rN

what is the probabiliry thar their first child will nor I


inherit any genetic disorder related to hemoglobin?

A. 0.25
B. 0.s0
C. 0.75
D. 1.00
ffi
,fl
il
&
il
d

Copyright @ by The Berkeley Review 330 The Berkeley


Specializing in MCAT
Biology Gregor Mendel and lnheritance Passage Il

Passage II (Questions 6-11) 6. Mendel first allowed plants of a given variety to


produce progeny by self-fertilization over many
The first quantitative studies of inheritance involved generations in order to:
the garden pea and were carried out by Gregor Mendel.
His choice of the pea was fortuitous, because a large A. carry out an experimental cross.
number of true-breeding varieties (uniform from one B. assure himself the plants were of the true-
generation to the next) was available. Mendel selected breeding variety.
seven easily distinguishable traits of these plants (e.g., C. observe segregation of alternative traits among
purple versus white flowers, and round versus wrinkled progeny.
seeds). D. carry out a reciprocal test-cross.

Pea plants contain both male and female sex organs.


This was also advantageous to Mendel, because he could
either let self-fertilization take place within an individual
flower or perform an experimental cross. A cross involves
removal of a flower's male parts before fertilization can
occur and introducing pollen from a strain with alternative
characteristics. 7. Mendel discovered that out of the Fz generation for
any trait, l/2 of the individuals were not true
Mendel first allowed plants of a given variety to breeders. This class of plants, if allowed to self-
produce progeny by self-fertilization for many pollinate, should produce Fr individuals that exhibit
generations. He then conducted crosses. Mendel took a dominant and recessive traits in a ratio of:
pea plant producing white flowers and introduced pollen
from a purple-flowered plant. He allowed the hybrid A. 1:1
offspring produced by these crosses to self-pollinate for B. 2:l
many generations. Mendel kept a record of the number of C. 2.5:1.5
offspring ofeach type and in each generation. D. 3:l
The hybrid offspring from the experimental crosses are
termed the first filial (Ft) generation. These offspring
always resemble one of their parents. For example, a
cross between wrinkled versus round seeds always
produce F1 progeny with round seeds. Mendel referred to
the Ft trait as being "dominant" over the invisible,
"recessive" trait. Mendel allowed individual F1 progeny 8. What is the ratio of homozygous to heterozygous
plants to self-pollinate, and he observed the offspring in individuals in an Fz generation?
this second filial (Fz) generation. He found that some
individuals in this generation exhibited the recessive trait. A. l:1
B. 3:l
Out of 7,324 Fz individuals resulting from the round- C. 9:3
seeded F1 self-pollinufion, 5,414 had round seeds and D. 16:l
1,850 had wrinkled seeds. The ratio was essentially 3:1.

Mendel examined all seven traits with alternative


forms and in every case obtained the same ratio results:
3/4 of the Fz individuals exhibited the dominant form of
the trait, and ll4 displayed the recessive form. He
accounted for his data by forming a central assumption
that alternative forms of a trait are specified by alternative 9. The ploidy number (n) of a cell refers to how many
alleles, which are discrete. sets of chromosomes are found in its nucleus. For a
human cell in prophase I, the cell is said to be:
In other words, the fgrctors specified by a pair of
alternating alleles are separate. This is known as the lqw A. haploid (n = 1).
ofallele segregation. B. diploid (n = 2).
C. triploid (n = 3).
D. tetraploid (n = 4).

Copyright @ by The Berkeley Review 331 The Berkeley Review


Specializing in MCAT Preparation
Biology Gregor Mendel and Inheritance Passage Il

10. In an effort to determine whether a purple-flowered


pea plant is heterozygous or homozygous, a test
cross is performed with a white-flowered pea plant.
Based on information in the passage, the MOST
likely conclusion is that the purple-flowered
individual is:

A. heterozygous, if all offspring have purple


flowers.
B. heterozygous, if half of the offspring have
purple flowers.
C. homozygous, if all offspring have white
flowers.
D. homozygous, if half of the offspring have
purple flowers and the other half have white
flowers.

11. The following diagram, illustrating a cross of


wrinkled and round seeded peas, is inaccurate
because the:

P generation

Round Wrinkled
seeds seeds

X
0
H
0.)

6,)
bo

F! X
0
f)
oa
o
o
4
FD

Round seeds Wrinkled seeds

A. P generation is not made up oftrue breeders.


B. ratio of rcitrnd to wrinkled seeds in Fz is
incorrect.
C. segregation is incorrectly shown as being
between pods.
D. number of progeny in Fz is incorrect.

Copyright @ by The Berkeley Review 332 The Berkeley Revieu


Specializing in MCAT Preparation
Biology Incomplete Dominance and Qodominance Passage III

Passage III (Questions 12-17) 12, Suppose two pink Japanese four-o'clocks are
crossed. What colors would the offspring, the Fz
Although many genetic traits are inherited in a generation, exhibit?
classical dominant/recessive pattern, this is not true of
every genetic trait. Certain genes follow the trends of A. 1:2:1,pink:white:red
either codominance or incomplete dominance. B. 1:2:l,red:pink:white
C. All flowers either pink or red
When a heterozygote for a particular trait has a D. 2:2 pink:white
of its two
phenotype that is intermediate between those
parents, the genes exhibit incomplete dominance.
Although many examples are known in both plants and
animals, a well-studied example of incomplete dominance
in color occurs in a flower called the Japanese four-
o'clock. When a homozygous white flower (n) and a 13. Which of the following terms BEST fits this
homozygous red flower (RR) are crossed, the Ft definition?
generation is entirely pink (Rr). Both the red color and the
white color are expressed together in the pink offspring. Genes that occupy corresponding loci on
homologous chromosomes and govern variations of
the same characteristic.

.F'f*-l A.
B.
Homologous pairs
Pheromones

.[*l*l C.
D.
Segregants
Alleles

Figure 1. Cross of red and white


flowers, producing all pink offspring.

t4. Which type of genetic cross is represented in


Another type of genetic expression is codominance, in Figure I ?
which both alleles are expressed independently and
simultaneously in the heterozygote. For example, a horse A. A monohybrid cross.
with a red coat (RR) is crossed with a white horse (rr). B. A test cross.
The fbal (Rr) has a mixed red and white coat, in which C. A dihybrid cross.
red hairs and white hairs are interspersed. Horse breeders D. An inbreeding cross.
refer to these crossed breeds as being "roan-colored."

A classic example of codominance in humans occurs


in the ABO blood group patterns. A and B refer to two
specific glycoproteins'that can be present on the surface
of red blood cells. The A and B glycoproteins are 15. Suppose a gene independent of the one governing
codominant. If either proteinis present, it is expressed. the color of a Japanese four-o'clock determined its
The lack of either protein results in the O blood type, leaf form in a classical dominant/recessive pattern,
which is recessive to A and B. with straight leaves (L) being dominant over cudy
leaves (l). Given the following cross:

RR Ll (Parent 1) X n Ll (Parent 2)

What is the probability of producing a plant with


pink flowers and curly leaves?

A. 0.25
B. 0.50
c. 0.75
D. 0.00

Copyright @ by The BerkeleY Review 333 The BerkeleY Review


Specializing in MCAT PreParation
Biology lncomplete Dominance and Codominance Passage III

16. The paternity of a royal heir is in dispute. The queen


(known to be the baby's mother) has type AB-
blood, while the king has B-. The baby has A+
blood. The positive or negative Rh factor is
transmitted as a classical Mendelian dominant/
recessive trait, with + being dominant. Is the child
the offspring ofthe king and the queen?

A. Absolutely
B. Possibly
C. No
D. Unable to tell from this passage

17. Two people with type O- blood have a child


together. What blood types could their child have?

I. o-
II. A-
m. B.

A. I only
B. I and II only
C. I and III only
D. I, II. and III

Copyright @ by The Berkeley Review 334 The Berkeley Ke


Specializing in MCAT Prepar
Biology DNA Replication Passage IV

Passage IV (Questions 18-23) 18. The replication of the entire E. coll genome requires
a relatively short time, approximately:
The entire Escherichia coli (E. coli) genome contains
4.7 x 106 nucleotide base pairs. At special sequences A. 160 minutes.
known as replication origins, two replication forks move B. 100 minutes.
in opposite directions, with polymerization occurring at a C. 80 minutes.
rate of 500 nucleotides per second. D. 60 minutes.

The average human chromosome is a single DNA


molecule containing about 150 million nucleotide base
pairs of DNA. In an experiment to determine the pattern 19. Which of the following patterns BEST represents
of eukaryotic chromosome replication, human cells are DNA replication of a eukaryotic chromosome?
grown in a culture labeled for a short time with 3H-
thymidine. The cells are lysed and placed on the surface A.
of a glass slide. The glass slide coated with a
is
photographic emulsion so the pattern of labeled DNA can
be determined by autoradiography. Results of the
experiment determine that polymerization occurs at a rate
of 50 nucleotides per second.
B.
Autoradiography results of two experiments on
identical pieces of DNA from the same organism are
shown in Figure l. The patterns of replication are
demonstrated by these results:
C.

€ D.

\ Replicati onongin /
Experiment A I

20. The slower rate of DNA replication on a eukaryotic


chromosome could be explained by:
Experiment B
A. the presence of multiple chromosomes.

+ B. the presence of exons and introns within the


DNA molecule.
ds DNA C. the difficulty in replicating DNA packaged as
chromatin.
D. the difficulty in replicating DNA packaged
inside the nucleus.
Figure 1

21. Replication forks are MOST likely to be activated


during which ofthese phases ofthe cell cycle?

A. Gr
B.S
C. Gz
D.M

Copyright @ by The Berkeley Review J'D The BerkeleY Keview


Specializing in MCAT Preparation
Biology DNA Keplication Passage [Y

22. In a modification of the experiment discussed in the


passage, a eukaryotic cell was pulse-labeled with
3H-uridine. The autoradiograph results would reveal
the presence of a label:

A. only in the nucleus.


B. in both the nucleus and the cytoplasm.
C. only in the cytoplasm.
D. only in the nuclear membrane.

23. Which of the following statements could be TRUE


regarding the autoradiography results shown in
Figure I ?

I. The organism is most likely eukaryotic.


II. The polymerases used in Experiment A are
slower than those used in Experiment B.
III. Further incubation after the addition of an
unlabeled medium occurred in Experiment B.

A. I only
B. I and II only
C. I and III only
D. II and III only

Copyright @ by The Berkeley Review 336 The Berkeley


Specializing in MCAT
Biology Color Blindness Fassage V

Passage V (Questions 24'29\ 24. Which of the following statements is LEAST likely
to result from gene duplication?
In human beings, the ability to perceive color depends
on genes that are both autosomal and X-linked. These A. Synthesis of similar amino acid sequences
genes initiate the production of visual proteins that are B. Synthesis of similar tRNA sequences
iensitive to red (565 nm), green (535 nm), and blue (420 C. Independent mutation of the DNA sequences
nm) wavelengths of light. Each visual pigment is found D. Synthesis of longer mRNA sequences
separately in a unique cell called a cone. These cells are
concentrated in an area called the fovea, located in the
retina of each eye.

The genes that code for the red and green pigments are
close to one another on the X chromosome. The X
chromosome contains one gene for the red pigment and
up to three genes for the green pigment. The gene for the 25. What percentage of females would be expected to
biue pigment is found on chromosome 7. Genes which are exhibit red-green color blindness, if one genetic
closely related to one another belong to gene families. locus were involved?
The X-linked genes appear to have evolved through gene
duplication during the past 30 million years. A. O.04Vo
B. O.l2Vo
C. 0.36Vo
Red-green color blindness is a sex-linked recessive D. O.64Vo

trait and is found in about 8Vo of the male population and


in less than l7o of female population, even though about
l57o of the female population are carriers. A defect in the
synthesis of the green-sensitive pigment is referred to as
tie deutan type of color blindness. Approximately 6Vo of
the 87o of all males who are color-blind have a deutan
defect. The remainin g 2Vo have a protan defect, a type of
color blindness resulting from a defect in the synthesis of
the red-sensitive pigment. Both defects lead to poor 26. Since two genetic loci are involved in red-green
recognition ofvarious shades ofred and green colors. color blindness, the incidence of color blindness
among females in the United States is 0.47o The
percentage of females that would be deutan color-
The frequency for X-linked genes in males is the same blind is:
as the phenotypic frequency, simply because the
phenotype of both the recessive and the dominant alleles A. O.04Vo
is expressed. The frequency of normal and mutant alleles B. O.l27o
can 6e calculated using the concepts of Hardy-Weinberg C. 0.367o
equilibrium and the.equation : D. 0.647o

P2+ZPq+q2=l

where p is the frequency of the dominant allele, and q is


the frequency of the recessive allele. Color blindness is
27. The percentage of women who are heterozygous
carriers ofprotan color blindness is about:
not a deleterious trait and therefore does not affect either
the viability or the reproductive success of the individual.
A. 27o
B. 4Vo
C. 87o
D. llVo

Copyright @ by The BerkeleY Review scl The BerkeleY Review


Specializing in MCAT PreParation
Biology Color Blindness Passage Y

28. A woman with normal vision marries a man who is


affected with deutan color blindness. Her sons will
have:

A. deutan color blindness.


B. homozygosity for the trait.
C. heterozygosity for the trait.
D. normal color vision.

29. A dominant autosomal allele C is required for proper


development of the cones in the retina. The
recessive allele c in the homozygous form results in
complete color blindness. A woman who has a
deutan defect is homozygous for the autosomal C
allele. She marries a man who is completely color-
blind but carries the dominant deutan allele. What
types of visual problems will their children have?

A. Daughters will have normal vision; sons will


have normal vision.
B. Daughters will be red-green color-blind; sons
will be completely color-blind.
C. Daughters will have normal vision; sons will
be red-green color-blind.
D. Daughters will be completely color-blind; sons
will be red-green color-blind.

t
I
,[

Copyright @ by The Berkeley Review 3;'4 The Berkeley


Specializing in MCAT
Biology Alleles and Coat Qolor Passage VI

Passage VI (Questions 30-36) Neither the black nor the brown coat color is expressed
when the hamster is homozygous for allele a, found at a
A gene is a unit ofinheritance located at a specific site different locus on the chromosome. The suppressive
(locus) on a chromosome (DNA). Genes exist in pairs in influence of allele d on an entirely different allele is an
diploid organisms, and the individual genes of a gene pair example of epistasis.
are refened to as alleles. If two alleles of a gene pair are
Ba
identical, the organism is homozygoas for that gene pair.
If the two alleles are different, the organism is
heterozygous for that gene pair. The complete set of genes
m
mba
in an organism determines its genotype, while the
expression of those genes as a collection of detectable
characteristics determines its phenotype. for allele a has a
A hamster that is homozygous
In a heterozygous individual, a dominant trait is due to genotype that produces a white (albino) coat. If a hamster
a gene that expresses itself in the presence of its allele. A has the + (wild-type or normal) allele at one of its a loci,
recessive trait in a heterozygote is due to a gene that does its coat color will be normal.
not express itself in the presence of its allele. Incomplete
dominance in a heterozygote involves the expression of Two experiments were conducted to investigate the
both alleles, with the effect of one allele appearing grcater genetics of coat color in hamsters.
than the other allele. In a heterozygote, a codominant trait
involves both alleles producing a combined effect. In this Experiment I
case, two gene products can be detected. Each product is
associated with one of the allelic forms. A group of true-breeding hamsters with the genotype B/B;
+./+ is crossed with a group of true-breeding hamsters with
Alleles on separate chromosomes can segregate from the genotype b/b; a/a to give the first filial 1nt)
one another at the time of gamete formation. This is the generation.
basis of Mendel's second law (independent assortmen$ ard
results in interchromosomal recombination, which always Experiment II
produces a recombination frequency of 507o in the second
filial (Fz) generation. One hamster from the F1 generation of Experiment I
was back-crossed to the parent with the double recessive
However, two or more pairs of alleles can be associated genotype (blb: a/a). The progeny of the resulting F2
with the same chromosome. If this is the case, the genes
generation are shown below:
are said to be linked. If two dominant alleles are linked,
those alleles are in coupling. If a dominant allele is linked
with a recessive allele, those alleles are in repulsion. If Blackoffspring: 66
there is crossing over between any two non-sister
chromatids, intrachromosomal recombination results, Brown offspring: 34
which always produces a recombination frequency of less White offspring: 100
than 50Vo in the Fz generation. The recombination
frequency (percent crossing over or ToCO) can be Total offspring: 200
determined fronithe following equation:

Number of Cross over Individuals in F2


x IOOVa
Total Number of Individuals in F2

The black coat color in hamsters is due to a dominant


gene, B, at a particular locus on a chromosome. The
recessive allele, b, at this same locus results in a brown 3 0. Based on the information given in the passage,
coat color when it is in the homozygous condition: which of the following statements is correct?
i-
B A. The a allele is dominant over the + allele.
ffi B. The a allele is recessive to the + allele.
m b
C.
D.
The a allele and the + allele are codominant.
The + allele shows incomplete dominance.

Copyright @ by The BerkeleY Review 539 The Berkeley Review


Specializing in MCAT Preparation
Biology AIIeles and Coat Color Passage VI

31 . A true-breeding strain of black hamsters would show 3 5. Based on the information in Experiment II and in the
which of the following genotypes? passage, which of the following BEST describes the
two genetic loci?
A. B/b: +/+
B. B/b; +/a A. They are linked.
C. B/B: +/+ B. They are unlinked.
D. B/B: +la C. They are epistatic.
D. They assort independently.

3 6. The genetics of coat color in albino hamsters is m


example of:
32. A different true-breeding strain of hamsters has the
genotype b/b; a/a at both loci. The phenotype of the A. dominance.
coat of these hamsters would be: B. recessiveepistasis.
C. incompletedominance.
I. black. D. codominance.
II. brown.
III. white.

A. I only
B. II only
C . III only
D. I and III only

n
ff
(
fr
3 3. Basedon the information in Experiment I, what is
the MOST likely phenotype of the progeny from the o
u
first filial (F1) generation?
m
fl
A. All black 0
B. All brown
C. All white
D. Both black and brown

3 4. Based on the information in Experiment II, estimate


the minimum genetic map distance between the two
genetic loci. (Note: One genetic map unit (m.u.)
gives a recombinant frequency (RF) of 1 percent.)

A. 1? m.u.
B. 34 m.u.
C. 52 m.u.
D. 68 m.u.

Copyright @ by The Berkeley Review 34o^ The Berkeley Reviw


Specializing in MCAT Preparatic
Biology DNA Calculations Passage VII

Passage VII (Questions 37-43) However, the separation of cellular constituents and
subsequent cytokinesis requires a constant time of 20
In 1953 James Watson and Francis Crick submitted a minutes after the completion of the corresponding round
landmark one-page paper to the British science journal of DNA replication. Therefore, bacterial cells with
Nature proposing a three-dimensional model for the doubling times that are shorter than 60 minutes must
structure of DNA. Using X-ray diffraction patterns begin DNA replication before the end of cytokinesis of
obtained by Rosalind Franklin and Maurice Wilkins, the preceding cell cycle.
Watson and Crick suggested that native DNA consisted of
two antiparallel polynucleotide strands that wind about Replication of the DNA duplex in E. coli is an ordered
each other in a helical fashion along a common axis. The process and is promoted by at least 20 enzymes and
helix is righrhanded with a diameier of about 204. This proteins. A helicase enzyme uses the energy of adenosine
is due to adenine-thymine (A-T) and guanine-cytosine triphosphate (ATP) to unwind the DNA at the orlC locus
base-pairings (G-C). to give two replication forks, from which the parental
strands serve as templates for the synthesis of new
Each base pair has an average molecule mass of 660 daughter DNA. Since all known DNA polymerases
Daltons (D). In an ideal situation, the DNA helix has 10 require a primer before new DNA can be synthesized, a
base pairs (bp) per turn of the helix, resulting in a helical short RNA primer of about 5 nucleotides is synthesized
twist of 36' per bp. The rise of the helix per turn (pitch) is by a DNA-directed RNA polymerase called a primase.
34A, while the rise per base pair is 3.4A. Shortly after
Watson and Crick proposed their model for the structure Once the primer has been established, DNA
of DNA, they postulated that DNA replicates polymerase III begins to synthesize new DNA in the 5' to
semiconserva-tively. The result is two molecules of 3' direction (at a rate of about 1000 nucleotides per
double-stranded DNA (a duplex), each containing the second) on the leading strand in a continuous fashion and
original parental strand of DNA and a newly synthesized on the lagging strand in a discontinuous fashion.
daughter strand of DNA. This hypothesis was verified in Discontinuous DNA synthesis occurs in the form of lkb
1958 by Matthew Meselson and Franklin Stahl. to 2kb segments of DNA calledOkazakifragments.DNA
polymerase I removes the RNA primers using its
In 1957 Arthur Kornberg, using the bacterium exonuclease activity and then slowly adds new
Escherichia coli, discovered the enzyme DNA nucleotides at a rate of about l0 per second. The
polymerase I, which is involved in the DNA-directed fragmented stretches of newly synthesized DNA are
synthesis of DNA. In 1969 Paula Delucia and John finally joined together by DNA ligase.
Cairns discovered a mutant of E. coli that did not display
DNA polymerase I activity. This mutant was named the
polAl mutant. Even though the polAl mutant had about
l%o of the normal polymerizing function of DNA
polymerase I, it reproduced at the same rate as its parental 37. If the average molecular weight of an amino acid
strain, thus suggesting that there might be polymerase l0 D, and if each amino acid is specified
residue is I
activity by an enzyme or enzymes other than DNA by three contiguous bases on a single strand of
polymerase I. DNA, then for a 40-kD protein the contour length
and molecular weight of a given section of DNA
This observation -led to the discovery of DNA would be about:
polymerase II and DNA polymerase III. DNA
polymerase III is also involved in the DNA-directed A. 3'7oL,7.0 x 104 D
synthegis of DNA. However, the physiological function of B. 37004, 7.0 x 104 D
DNA polymerase II is still unknown. C. 310L,1.0 x 105 D
D. 37ooA.7.o x los D
ln E. coli, the bidirectional replication of the roughly
4000 kilobase pairs found in the circular DNA
chromosome is initiated at a single locus referred to as
oriC. [One kilobase (kb) is equal to 1000 nucleotide base
pairs (bp) of duplex (double-stranded) DNA or 1000
nucleotides of single-stranded DNAI. The 245 bp segment 38. The rate of nucleotide incorporation at each growing
of the orlC locus is highly conserved among Gram- replication fork would be roughly:
negative bacteria. Even though the doubling time of this
bacterium can range from less than 20 minutes to about A. 225 nucleotides per second.
10 hours (depending upon the growth medium), it takes B. 425 nucleotides per second.
about 40 minutes for the chromosome to replicate itself C. 850 nucleotides per second.
completely. D. 1700 nucleotides per second.

Copyright @ by The Berkeley Review 34t The Berkeley Review


Specializing in MCAT Preparation
Biology DNA Calculations Passage VII

39. Unlike the circular chromosome of a prokaryotic 42. During DNA replication, the base sequence 5'-
organism like E. coli, the linear chromosomes of pApTpApGpApC-3' would give rise to which of
eukaryotic organisms have many origins of these complementary base sequences?
replication. These sites of replication are roughly
spaced 30 to 100 kilobases apart from one another. A. 3'-UpApUpCpUpGp-5'
Suppose that a given tissue culture of mammalian B. 5'-pTpApTpCpTpG-3'
DNA has L2 meters of duplex DNA in every single C. 5'-pGpTpCpTpApT-3'
cell and that the synthesis phase (i.e., that period in D. 3'-TpApTpGpUpGp-5'
the cell when DNA is synthesized) for these cells is
5 hours. How many replication forks are there, if the
growth rate in these cells is l6 pm/minute?

A. 125 replication forks


B. 250 replication forks 43, Which of the following statements about the action
C. 375 replication forks of DNA polymerases must be true?
D. 500 replication forks
L In addition to its 5'to 3'polymerizing activitl'.
DNA polymerase I also has a 5' to 3'
exonuclease activity that excises RNA
primers.
il. DNA polymerase III can add nucleotides at the
3' end of a growing polynucleotide chain.
40. In a rich growth medium, bacterial cells can divide while DNA polymerase I can add nucleotides
every 20 minutes. Which of the following BEST at the 5' end of a growing polynucleotide
describes the cell's bidirectional replication in this chain.
case?
nI. DNA polymerase III adds new deoxyribo-
nucleotides to a growing polynucleotide chain
I. At division, each daughter cell receives a
roughly 100 times faster than DNA polymrase
chromosome that is half replicated.
I.
U. At division, each daughter cell receives a
chromosome that is one-quarter replicated.
III. There is a total of four replication forks.
A. II only
IV. There is a total of six relication forks.
B. II and III only
A. III only C. I and III only
B. I and III only D. I and II only
C. II and IV only
D. I and IV only

41- During the replication of an E. coli chromosome,


roughly how many Okazaki fragments are
synthesized?

A. 100 to 200
B. 1000 to 2000
C. 2000 to 4000
D. 4000 to 6000

lr

Copyright @ by The Berkeley Review 342 The Berkeley Reviev C


Specializing in MCAT Preparatiom
Biology Semiconservative DNA Replication Passage VtrI

Passage VIII (Questions 44-50) 44. Watson and Crick's proposal for the structure of
DNA does not allow for which of the following base
After Watson and Crick announced their discovery of pairings?
the structure of DNA in the early 1950s, they proposed
that DNA could reproduce itself autocatalytically. They I. Cylosine-guanine
hypothesized that the two parental strands of a DNA I I. Adenine-guan ine
duplex would separate, and each would serve as a ilI. Cytosine-thymine
template for the synthesis of a new daughter strand of
DNA. Once DNA replication was complete, each new A. I only
duplex would contain one parental strand and one B. II only
daughter strand, linked by hydrogen bonds between C. II and III only
complementary purine (adenine and guanine) and D. III only
pyrimidine (thymine and cytosine) bases. Watson and
Crick called this type of replication semiconservative.
Conservative replication, in contrast, would conserve the
integrity of the parental strands in the DNA duplex after
replication. 45. After Watson and Crick proposed the structure of
DNA, it was postulated that DNA could replicate in
In the late 1950s, Meselson and Stahl were able to a conservative fashion. After two rounds of
confirm Watson and Crick's hypothesis that DNA conservative DNA replication, the original duplex
replicated semiconservatively by performing a density- parental strands would give rise to:
gradient equilibrium sedimentation experiment. In this
experiment, a 6M cesium chloride (CsCl) solution with a A. two duplexes of DNA, each containing one
density of about 1.7 glcm3 is placed in a test tube, which daughter strand and one parental strand.
is spun at high speed in an ultracentrifuge. A smoothly B. four duplexes of DNA, each containing one
changing concentration gradient is thus established in the daughter strand and one parental strand.
test tube (Figure 1). C. two duplexes of DNA, one duplex containing
two daughter strands and one duplex
Increasing density
containing two parental strands.
r_____
of solution r) D. four duplexes of DNA, two duplexes each
containing two daughter strands and two
duplexes each containing two parental strands.

CsCl spinning in uitracentrifuge


- 46. The structural component of DNA is called
Figure I nucleotide (see below):
E. coli were grown in a complete medium containing
l5NH+C1 for numerous generations, ensuring that the
DNA in these cells i.vould contain the heavy, stable
o.R Jt''^
isotope of nitrogen, 15N. An excess of l4NH+Cl was then A nucleotide n=J-n
";; ,'Y
trA"A**,
added to the culture of l5N-labeled bacterial cells.
Immediately after this addition of laNHaCl, the DNA
from a sample of the cultured cells was extracted and
sedimented to equilibrium in the CsCl density gradient.
Ultraviolet absorption photographs (at 260 nm) indicated
containing
the base
guanine
'[J'
I","'^

o-R'
the presence of just one band of DNA. This band
corresponded to the all heavy lsN-labeled DNA. The stable, heavy isotope of nitrogen, l5N, can be
found in all of the following components of DNA
The cells that remained in culture were allowed to EXCEPT:
double and complete one generation of growth. Analysis
of the DNA from a sample of these cells indicated the A. the pyrimidine rings.
presence of a new band, that o1 u 1411-15p hybrid. On the B. the purine rings.
basis of this density-gradient experiment, it was C. the ribose rings.
concluded that DNA replicated in a semiconservative D. the acetal linkages.
fashion.

Copyright @ by The Berkeley Review 345 The Berkeley Review


Speciatizing in MCAT Preparation
Biology Semiconservative DNA Keplication Passage Vtrl B

47. Ultraviolet absorption photographs (at 260 nm) of 50. Macromolecules that are not bound to ions have
DNA in a CsCl density-gradient solution indicate a densities close to 1.3 g/ml. Which of the following
particular DNA banding pattern. The distribution of single-stranded nucleic acid polymers would show a
l4N and l5N in a CsCl density gradient after two greater density in a CsCl density gradient?
rounds of semiconservative DNA replication is
represented by which of the following DNA banding 5'-pGpCpApApCpCpGpGpCpC-3' DNA
patterns?
5'-pCpCpGpGpCpCpApApCpG-3' RNA
A. B.

l-l
ll
tt
FF
-tt
tt
(C = cytosine; G = guanine; p = phosphate)

A. DNA, because the Cso ions can bind to the


negatively charged phosphate groups and the
tt
-
I ".:.1
Htt free hydroxyl groups of DNA.
B. RNA, because the Cs@ ions can bind to the
@jj m
r,i.1,'.:$

\.ry negatively charged phosphate groups and the


free hydroxyl groups of RNA.
Control Control
C. DNA, because the Cle ions can bind to the
C. D. positively charged nitrogenous bases of DNA.
D. RNA, because the Cle ions can bind to the
positively charged nitrogenous bases of RNA.

)f'\B*.
t)i):::i

Control Control

48. After two rounds of semiconservative DNA


replication, the ratio of laN to l5N will be:
Ir:
A. 3:l :u
B. 2:l trx;

C. l:2 [5
D. l:3 llll,

tr,"

T,,I,

49. DNA synthesized from the 1sN isotope is denser f;;


than DNA synthesized from the l4N isotope, illf

because of the additional:

A. neutron in l5N.
B. proton in 15N. mi

C. electron in l5N. j
D. water molecules surrounding the 15N DNA. rim
m

u
,ilE

Copyright @ by The Berkeley Review 344 The Berkeley Revier I


Specializing in MCAT Preparatiol
Biology DNA Structure Passage IX

Passage IX (Questions 51-58) Replication of the DNA helix requires local


denaturation (melting) of hydrogen bonds between base
The molecules that store and transmit information in pairs. Each single-stranded segment of old (parental)
cells are deoxyribonucleic acid (DNA) and ribonucleic DNA then serves as a template to which new (daughter)
acid (RNA). The fundamental unit of each of these DNA is attached. After one round of replication is
nucleic acids is the nucleotide. Each nucleotide contains a complete, two new DNA duplexes are formed, each
nitrogenous base, a five-carbon sugar called ribose, and a containing one strand of parental DNA and one strand of
phosphate. The bases adenine (A), guanine (G), and daughter DNA. This is referred to as semiconservative
cytosine (C) are found in both DNA and RNA. Thymine replication.
(T) is found in DNA and uracil (U) is found in RNA.
Figure 1 shows the structural components of these two Three other forms of DNA replication have been
nucleic acids (Note: dR = deoxyribose; R = ribose): examined. If the DNA helix replicates conservatively,
O NH. o then after one round of replication one of the DNA
A duplexes still contains both parental strands, while the

""4at"' *)" HN} other DNA duplex contains two new daughter strands. If
replication is dispersive, then after one round of

"1,,tr
rt
dR

Thymine
"I) dR
Cytosine
.L*) I
R
replication the DNA in each strand of each duplex is
composed of alternating segments of parental and
daughter DNA. Finally, if replication is end-to-end
Uracil conservative, then after one round of DNA replication
NH" o each strand in each duplex contains one end that is all
I il parental DNA and one end that is all daughter DNA.

r3) I
dR
"*a-"
"."1_-L) dR
Adenine Guanine 51. Which of the following bases is LEAST likely to
o_
tt
exist in the enol form?
o 5
o- P- o- cH.
I A. Adenine
n-o
B. Cytosine
C. Guanine
Ribose phosphate OH (or H) D. Uracil

Figure I

DNA and RNA can exist in a double-stranded (duplex) 52. DNA can be classified by all of the following
or a single-stranded form, and they can be linear or characteristics EXCEPT:
circular. Both nucleic acids can also hybridize to each
other. In the nucleus of eukaryotic cells, DNA is A. it exhibits a polynucleotide arrangement.
associated with positively charged proteins called B. it is negatively charged at neutral pH.
histones. C. it is deoxy at the C-3' position.
D. it contains acid anhydride linkages.
The DNA double helix consists of two polynucleotide
strands that wind about a common axis, making a
complete turn every 3.4 nm. Each strand is joined by
hydrogen bonding between the nitrogenous bases. The 53. The following sequence of a short segment of a
base pairs are partially stacked on each other and are single strand of DNA is given:
separated by 0.34 nm.
5'-A-T-G-C-C-G-A-T-3'
Each strand in the DNA double helix has a specific
chemical orientation. Attached to the 5' carbon of the The complementary single strand of DNA is:
ribose ring is a hydroxyl gr6up that bears a phosphate; the
3' carbon atom of the ribose ring has a free hydroxyl A. 3'-pTpApCpGpGpCpTpA-3'
group. Polynucleotide sequences are conventionally B. 3'-pTpApCpGpGpCpTpAp-5'
written in the 5' -+ 3' direction, and in DNA which is C. 5'-ApTpCpGpGpCpApT-3'
double-stranded the two polynucleotide sequences are D. 5'-pApTpCpGpGpCpApT-3'
antiparallel to one another.

Copyright @ by The Berkeley Review 345 The Berkeley Review


Specializing in MCAT Preparation
Biotogy DNA Structure Passage H

54. On average, how many bases are there per turn of 58. An unknown organism is incubated in a complelt
the DNA double helix? growth medium containing l5NH+Cl for manl
generations. The organism is next transferred to I
A.5 new growth medium containing l4NH+Cl. After tr*:
B. 10 generations in the new growth medium, its DNA ri
c. 15 analyzed using CsCl equilibrium density-gradien:
D. 20 ultracentrifugation. Next, ultraviolet absorpticr,
photographs of the ultracentrifuge test tubes ar;
taken, and a microdensitometer trace is made of tle
55. Which of the following base pairs is NOT usually results.
associated with DNA?

A. Uracil hydrogen bonding to adenine o


B. Adenine hydrogen bonding to thymine !-
C. Guanine hydrogen bonding to cytosine J:E
D. Thymine hydrogen bonding to uracil qN
€€
.Ed

/.
50. DNA is the carrier of genetic information during cell
growth and division. Which of the following
of DNA is NOT essential for the
characteristics Direction of sedimentation --------->
accurate transmission of this information?
Two peaks arre observed, and the amount of DNA :r
A. A genetic code that is degenerate each peak is recorded. Based on the information n
B. A mechanism for self-replication the graph presenting these data, we can say thc
C. A low mutation rate DNA replication in this organism is:
D. A conformationally variable molecule
A. conservative.
B. semiconservative.
5/. A typical DNA melting curve is shown below. C. dispersive.
Denatured DNA shows an increase in ultraviolet D. end-to-endconservative.
absorbance. The temperature at which one-half the
maximum absorbance is reached is called the
melting temperature, T 6.

1.4
C)
o
X r.r

€8r.2
o c]
.Ed
fi& r.r

1.0
50 60 '70 80 90
Temperature ("C)

All of the following factors will lead to an increase


in the Tm of DNA EXCEPT:
n.-

A. a lower mole fraction of A.T base pairs.


B. a higher concentration of Mg2o.
C. a lower concentration of histones.
D. a higher mole fraction of G'C base pairs.

Copyright @ by The Berkeley Review 346, The Berkeley Revier


Specializing in MCAT Preparatiou
Biologv Mitochondrial DNA (mtDNA) Passage X

Passage X (Questions 59-65) 61. The codon AUG is used as a start codon in the
genetic code of the mitochondria. This codon codes
The biosynthesis of mitochondria involves the for which of the following amino acids?
contribution of two separate genetic systems. Most of the
proteins are encoded by nuclear DNA and imported into A. Leucine
the organelle. However, some proteins are encoded by B. Histidine
organelle DNA and synthesized on ribosomes within the C. Methionine
organelle. The protein traffic between the cytosol and the D. N-formyl methionine
mitochondrion appears to be unidirectional.

Mitochondrial DNA (mtDNA) molecules are small 62. As stated in the passage, every nucleotide in the
All mitochondria contain multiple copies of
and circular. mitochondrial genome is part of a coding sequence.
their DNA molecules, and the genome is likely to A probable result of this is thar compared to the
resemble bacterial chromatin. The human mitochondrial nuclear genome, there would be:
genome has several surprising characteristics. One is that
every nucleotide appears to be part of a coding sequence A. fewer mRNA molecules ffanscribed.
for a protein, rRNA, or tRNA. Second, only 22 tRNAs are B. more mRNA molecules transcribed.
needed for mitochondrial protein synthesis versus the 31 C. more room for regulatory DNA sequences.
tRNAs required in the cytosol. Third, the genetic code D. less room for regulatory DNA sequences.
used in the mitochondrion differs from the universal code
used in other genomes. It has been shown that four out of
the sixty-four codons in mtDNA have different meanings. 63. Which of the following statements offers a
Further investigation has shown that the mitochondrial reasonable explanation for the different genetic code
genetic code is different in different organisms. For found in mitochondria?
example, the codon UGA codes for tryptophan in the
mitochondria of mammals, fungi, and protozoans. But in A. The drift responsible for changes in the genetic
plant mitochondria, UGA is a stop codon. code occurs only in mitochondria.
B. The small number of proteins encoded in the
Both strands of human mitochondrial DNA are mitochondrial genome makes changes in the
transcribed from a single promoter region on each strand. meaning of the code tolerable.
Two giant RNA molecules are thus created. The two C. The mitochondria has unique tRNAs, which
molecules are known as the heavy and light strands. The correct for the different genetic code.
heavy strand is processed to produce many RNAs, D. In mitochondria, the primary structure of a
including ten polyadenine-containing RNAs. Roughly protein has no effect on the function of the
90Vo of the light strand contains no discernibly useful molecule.
information.

64. The ten polyadenine-containing RNAs noted in the


passage are MOST likely to:
59. In a human cell,* protein transport does NOT move
from cytosol to: A. make their way to the cytosol to be translated.
B. have an attachment site for amino acids.
A. mitochondria. C. be involved with the structure of ribosomes.
B. peroxisome. D. code for proteins.
C. nucleus.
D. chloroplast.
65. The DNA that codes for the light strand is also
known as the:

60. Which of the following is LEAST likely to be A. antisense strand, because the light chain is
associated with the mitochondrial genome? complementary.
B. antisense strand, because the light chain is
A. Distinct prombter regions nearly identical.
B. RNA polymerase protein C. sense strand, because the heavy chain is
C. DNA polymerase protein complementary.
D. Histone protein complex D. sense strand, because the heavy chain is nearly
identical.

-opyright @ by The Berkeley Review 347 The Berkeley Review


Specializing in MCAT Preparation
Biotogy Viral Complementation Passage )il

Passage XI (Questions 66-72) 66. The DNA of the bacteriophage T4 encodes all of the
components necessary for the replication of its viral
The bacteriophage T4 is a DNA virus which infects E. genome. Which of the following might NOT be
coli cells. When a wild-type T4 phage is placed on a encoded?
lawn, or thin layer, of E. coli cells growing on an agar
plate, a local clearing develops where bacterial cells have A. tRNA
been lysed by the original phage and its progeny. This B. Primase
clear region on the otherwise opaque lawn is termed a C. Ligase
plaque. D. DNA polymerase

Two mutant forms of phage T4 have been isolated and


named rllA and rlIB, respectively. Both mutants cause
larger-than-normal plaques on plates containing a strain
of bacteria known as E. coli B. However, both of these T4
mutants fail to form plaques at all on a second bacterial
strain, E. coli K. Wild-type T4, however, is capable of
lysing cells of E. coli K.

An experiment is conducted in which rllA and rIIB 67. E. coli K cells are coinfected with rIIA and a neu T4
mutants are allowed to infect E. coli K cells mutant called X1. No plaques form. What can be
simultaneously. Individual bacterial cells are therefore concluded from this experiment?
coinfected by both mutant forms of T4. While infection
by either mutant alone causes no plaque formation, the A. The rIIA and Xl mutations each aff*tn
simultaneous coinfection by both mutants causes cell lysis different genes.
and the formation of small plaques. This phenomenon is B. The rIIA and X I mutations both affect rtu
an example of genetic complementation. same gene.
C. The rIIA and XI mutations complement e,rin
In this example, complementation occurs when the other.
protein that is defective in rIIA is provided by rIIB, and D. The X1 mutant phage could lyseE. coli K ceilin
vice versa. This complementation results in the restoration if it infected them alone.
of the wild-type, lysis phenotype (Figure 1). In order for
two mutants to complement each other, they must each
affect different genes. If they both were to affect the same
gene, they would be incapable of complementation,
because neither mutant could provide a functional protein
product.

(+) rIlA C) rIIB


Genes 68. Based on information given in the passage, whi;h
(phage rIlB)

Pathway (l
I
)r------) (2)r----) Lysis
the following would explain why rII mutanl:
incapable of forming plaques on lawns of E. cal;

t cells?

I. r11 mutant DNA has sequences that


,o,,lr"Jii,o, recognized and cleaved by endonucle
produced by E. coli K.
Figure 1. Genetic complementation in phage T4. U. r11 mutants lack functional gene pro*X
necessary for lysis of E. coli K cells.
IIII. E. coli K lacks specific surface receptor$
which r11 mutant phages bind.

A. I only
B. II only
C. I and III only
D. I.II. and III

Copyright @ by The Berkeley Review 34A The Berkeley


Specializing in MCAT
Biology Viral Complementation Passage Xf

69. A and B each cause


In Drosophila, recessive mutants 72. The T4 bacteriophage can BEST be described as:
a curly-wing phenotype when homozygous.
However, transheterozygous flies (i.e., flies that are A. an obligate parasite.
NB)have curly wings, even though neither mutation B. an obligate heterotroph.
is homozygous. Which of the following conclusions C. auxotrophic.
is MOST consistent with these results? D. prototrophic.

A. Mutations A and B complement each other;


therefore they affect different genes.
B. Mutations A and B fail to complement each
other; therefore they affect different genes.
C. Mutations A and B complement each other;
therefore they affect the same gene.
D. Mutations A and B fail to complement each
other; therefore they affect the same gene.

70. A large number of the T4 mutants rIIA and rIIB are


added separately to two plates of E. coli K cells. The
plate infected with rIIA forms one revertant plaque,
while the plate infected with rIIB forms no plaques
at all. Based on these results, which of the following
assumptions may be valid?

A. rlIA is caused by a deletion of DNA, while


rIIB is caused by a silent mutation.
B. rIIA is caused by an deletion of DNA, while
rlIB is caused by a point mutation.
C. rlIA is caused by a point mutation, while rIIB
is caused by a deletion of DNA.
D. Both rIIA and rllB are caused by frameshift
mutations.

71. Which of the following would be incapable of


exhibiting the type of complementation described in
the passage?

A. Recessive mutations
B. Sex-linked recessive mutations
C. Dominant mu-tations
D. Deletion mutations

Copyright @ by The Berkeley Review 349 The Berkeley Review


Specializing in MCAT Preparation
Biology Atrial Natriuretic Peptide Experiment Passage XII

Passage XII (Questions 73-79) 74. Atrial tissue from the animals is isolated and
homogenized. The amount of ANP (pg/mg wet
About lOVo of the US population has sodium-sensitive tissue) in the atria is then measured. The results are
hypertension. As dietary sodium increases, blood pressure given in the following table:
increases in these people. Blood pressure is regulated by
blood volume, peripheral resistance, and the cardiac rate.
Animal Right atria Left atria
One molecular mechanism of control involves a 28-
amino acid peptide produced primarily in the atria, called Homozygous Undetected Undetected
atrial natriuretic peptide (ANP). It is stored in dense mutant
granules in the cytoplasm of atrial cells as a larger
precursor, pro-ANP. In response to atrial distention or Heterozygous 54.3 t'.t.1 53.7 !7.1
stretch, pro-ANP is processed, and ANP is secreted into mutant
the bloodstream. It promotes sodium excretion into the
Wild-type 114.7 + 5.9 112.2 r7.5
urine and the movement of fluid out of the intravascular
space.

Researchers use a genetically engineered mouse model What would be the BEST type of measuremenr r:
to study the interrelationships between ANP, dietary detect and quantify ANP, which is a small pepride?
sodium, and blood pressure. Mouse embryos are treated
so that the gene coding for ANP is mutated and A. Lowry assay
nonfunctional. The mice are then interbred to produce B. Peptide digestion
homozygous mutants and heterozygous mutants. Control, C. Southern blot
wild-type animals are also used. All mice are fed one of D. Radioimmunoassay
three diets: either standard chow at 0.5Vo sodium, an
intermediate diet at 2Vo sodltm, or a high-sodium diet at
87o sodium.
t5. Based on the observed effects of ANP om
Diet
intravascular compartments, explain the follourrnE
Homorygous Heterozygous Wild-type
mutants
hematocrit values taken from the study animals:
7o sodium animals animals

0.5 124 + 3.7* 115 r 1.8 116 + 2.6


(standard Animal Hematocrit
chow)
Homozygous 45.80/o
2 156 + 9.0* 132 ! 6.7 t34 + 6.1 mutant
8 145 + 7.8* 11816.2
Heterozygous 48.6Vo
* p < 0.05 versus wild-type.
mutant

Table 1. Blood pressure (mmHg) in experimental animals Wild-type 49.9Vo


on various sodium-containing diets.

A. The homozygous mutants have a loqtn-


73. Which of the following statements is TRUE about hematocrit due to more fluid in inc
intravascular space.
the data in Table 1?
B. The wild-type animals have a highcu"
A. Homozygous mutant animals were normo- hematocrit due to more fluid in ftc
intravascular space.
tensive on the standard chow diet.
B. The 27o sodium diet increased blood pressure C. ANP directly affects the blood cells, so
in all groups. more of them are produced in the wild-rlpcr
animals, raising the hematocrit.
C. Heterozygous animals mimicked the responses
D. ANP directly affects the blood cells,
of wild-type animals as dietary sodium
fewer of them are produced
so
increased. in
D. Blood pressure increases in the heterozygous homozygous mutant animals, lowering
hematocrit.
animals were inversely proportional to dietary
sodium.

Copyright @ by The Berkeley Review 550 The Berkeley


Specializing in MCAT
Biology Atrial Natriuretic Peptide Experiment Passage XII

76. Individuals processing a newly discovered human 79. Applying these ANP gene data to human beings, if a
mutation are found to be lacking the protease that child has one parent who is not salt-sensitive (AA)
liberates ANP from the pro-ANP precursor. What and one parent who is heterozygous (Aa) and salt-
would be the probable fate of pro-ANP in these sensitive, what is the likelihood that the child is also
people? salt-sensitive (either Aa or aa), assuming Mendelian
inheritance?
A. It would probably be degraded inside the cell
by lysosomal mechanisms. A. O%
B. It would probably be released as pro-ANP and B. 257o
activated by proteases in the bloodstream. C. 50Vo
C. It would probably be activated by lysosomal D. 757o
mechanisms and released as ANP.
D. It would probably be released as pro-ANP and
excreted by the kidney.

77. Which other theoretical experimental approaches


could provide information similar to what was found
in the experiment described in the passage?

I. Administer to wild-type animals the same anti-


ANP antibodies and compare to untreated
animals.
II. Administer to wild-type animals a compound
that irreversibly binds the ANP receptor and
compare to untreated animals.
ilI. Administer to wild-type animals a compound
that binds and inhibits processing of pro-ANP
and compare to untreated animals.

A. I only
B. I and II only
C. II and III only
D. I, II, and III

78. Based on information in the passage, what is the


postulated role of ANP?

A. To lower blood pressure


B. To raise arterial blood pressure
C. To raise atrial piessure
D. To lower sodium output in the urine

Jopyright @ by The Berkeley Review 351 The Berkeley Review


Specializing in MCAT Preparation
Biology Griffith's Pneumococcus Passage tr(il

Passage XIII (Questions 80-87) 81. Based on information in the passage, which of fu
following conclusions is NOT valid?
In 1928, Fred Griffith carried out the following set of
experiments using mice and the bacterium Streptococcus A. The DNA of pathogenic strains codes for rte
pneumoniae, a Gram-positive, facultative anaerobe. polysaccharide coat seen on transforrnod
strains.
Experiment I B. R mutants lack enzymes needed for thc
synthesis of the polysaccharide coat.
A live, encapsulated strain of virulent pneumococcus C. Injection of only the polysaccharide coat inmo
is injected into a mouse. The mouse dies. mice will result in their death.
D. The transformation of the R-strain to the S
Experiment 2 strain is permanent.
A live, nonencapsulated strain of nonvirulent
pneumococcus is injected into a mouse. The mouse lives.

Experiment 3
A heat-killed strain of virulent pneumococcus is
injected into a mouse. The mouse lives. 82. Pneumococcus strains used in the Griffifrr
experiments that were treated with heat were killlsd,
Experiment 4 because of damage to:
A mixture of hearkilled virulent and live nonvirulent
pneumococcus is injected into a mouse. The mouse dies A. lipids.
and is found to contain the live pathogenic B. proteins.
pneumococcus. C. carbohydrates.
D. nucleic acids.

Griffith concluded that the heat-killed virulent cells


contained a transforming factor that converted the live,
nonvirulent cells, to live virulent cells.

The pathogenicity of pneumococcus can be 83. All of the following statements support Ave4r'b
categorized by the presence or absence of an outer conclusion EXCEPT:
polysaccharide capsule. Strains with the capsule are
pathogenic and are denoted as the S-form of the bacteria. A. there was no loss of transforming activity
Nonpathogenic strains are denoted as the R-form. the addition of trypsin and chymotrypsin.
B. the addition of ribonuclease had no effect
In 1944, a series of experiments by Avery and his the transforming principle.
coworkers characterized this transforming principle as a C. there was no loss of transforming activiq'
nucleic acid of the ddbxyribose type. the extraction of protein or lipid.
D. there was no loss of transforming activiq
the addition of deoxyribonuclease.

80. The stain used to distinguish between Gram-positive


(+) and Gram-negative (-) bacteria takes advantage
of differences in which of the following structures? 84. Which of the following BEST describes
metabolic requirements of pneumococcus?
A. Plasma membrane
B. Nuclear region A. The organism requires the presence of oxy-g
C. Ribosomes B. The organism may function in the presence
D. Cell wall absence of oxygen.
C. The organism requires the absence of oxygur.
D. The organism requires the absence of ni

Copyright @ by The Berkeley Review 352 The Berkeley


Specializing in MCAT
Biology Griffi th's Pneumococcus Passage XIII

85. All of the following statements are FALSE except:

A. pneumococcus translation takes place in the


mitochondria.
B. pneumococcus does not contain organelles.
C. pneumococcus cells are larger in diameter than
most eukaryotic cells.
D. the pneumococcus nucleus is a double
membrane-bound structure.

86. In an experiment that provided further evidence for


Avery's conclusion, radiolabeled elements were used
to distinguish between protein and nucleic acids.
Which of the following elements were MOST likely
used?

A. Oxygen and nitrogen


B. Phosphorus and nitrogen
C. Sulfur and oxygen
D. Phosphorus and sulfur

87. Pathogenic pneumococcus coated with antibodies


can be rendered avirulent. Therefore, it can MOST
likely be concluded that the polysaccharide capsule
normally inhibits:

A. pinocytosis of the bacterium by B-cells.


B. phagocytosis ofthe bacterium by p-cells.
C. pinocytosis of the bacterium by macrophage.
D. phagocytosis of the bacterium by macrophage.

Copyright @ by The Berkeley Review ccc The Berkeley Review


Specializing in MCAT Preparation
Biology Drosophila eyeless Gene Passage XIV

Passage XIV (Questions 88-94) 88. Assume that ey is X-linked. If ey/+ females are
crossed to wild-type males, what are the phenotypes
The eyeless (ey) mutation in Drosophila melanogaster of the progeny?
was first described in 1915 on the basis of its distinct
phenotype, the partial or complete absence ofthe insect's A. 50Vo of males have small eyes, 50Vo of females
compound eyes. Known alleles of the e)
gene are have small eyes; all others are normal.
recessive hypomorphs, meaning that the mutant gene B. 5OVo of males have no eyes, 507a of females
product has reduced function relative to wild-type. have small eyes; all others are normal.
Heterozygous (ey/+) flies have smaller eyes, while C. 50Vo of males have no eyes, 50Vo of males are
homozygotes (ey/ey) have almost no eyes at all. The normal, l00%o of females are normal.
eyeless gene has been cloned and appears to be a D. IOO% of males have no eyes, l00vo of females
transcription factor. It shares sequence homology to a are normal.
gene in mice called small eyes.In situ hybridization
shows that ey is normally expressed only in the
embryonic tissues that eventually form the eyes. Mice and
flies are separated evolutionarily by 500 million years.
In Drosophila, a technique has recently been
developed in which a gene of interest can be ectopically
expressed in specitic tissues in which it is normally not
active. GAL4, a transcriptional activator protein originally
isolated in yeast, can bind to an upstream activating 89. In Experiment 1, some fertilized eggs died as
sequence (UAS) on DNA, triggering transcription of the embryos. In situ hybridization, a technique wherebl
gene directly downstream from the UAS. "Downstream" fluorescently-labeled antisense ey mRNA is allowed
means in the 3'direction, while "upstream" means in the to hybridize with sense mRNA in vivo, showed e-r
5' direction. mRNA was expressed throughout the embryo. Whar
would MOST likely have caused this?
ft,-*'#\ ftf!) p,ot.in
A. A substitution mutation at the UAS binding
Jf, era site

-"':T:::::*E-
ennancer
Wing specific expression
uAS
Transcription of ey in
B.
C.
D.
A constitutive mutation in the GAL4 promoter
Expression of the normal genomic ey mRNA
An amber mutation in the ey coding sequence
ofGAL1 wing tissue only

Figure I

By ligating the cDNA sequence encoding the CAL4


protein to a weak promoter and a Drosophila tissue-
specific enhancer (a sequence that boosts transcription of
nearby genes), GAL4 will be transcribed only in the
specific tissue where the enhancer is active, thereby
activating only the UAS-controlled gene in that particular 90. Which of the following theories CANNOT be
tissue. supported by evidence given in the passage?
Experiment I
I. 'The ey gene is necessary and sufficient fcr
A transgenic line of flies is engineered with a DNA normal eye development in D rosophila.
construct consisting of wild-type ey cDNA directly II. The ey protein acts as a developmente,
downstream from the UAS site. These flies are crossed to "master switch," activating transcription r:
another line that has a wing tissue-specific enhancer genes needed for eye formation.
(active only in wings) adjacent to a GAL4 sequence. III. Evolutionarily, eyesoriginated independenril
Remarkably, most of the resulting progeny have complete in insects and mammals.
compound eyes growing from their wings.
Experiment 2 A. I only
The wing-enhancer/dAl4 line from Experiment 1 is
B. II only
crossed to flies containing a construct that links the mouse
C. I and III only
small eyes gene sequence to the UAS site. The results are
D. I, II, and III
identical to those in Experiment 1.

Copyright @ by The Berkeley Review 354 The Berkeley Keviex (--


Specializing in MCAT Preparation
Biology Drosophila eyeless Gene Passage XIV

91. Which of the following is MOST likely be the cause


of the mutant phenotype shown in the ey alleles
mentioned in the passage?

A. A point mutation in a non-conserved region of


the ey sequence
B. A two base-pair deletion near the beginning of
the ey sequence
C. An inversion of a segment of the ey sequence
D. A point mutation in a highly conserved region
of the ey sequence

92. Which of the following findings would be


inconsistent with the results of Experiment 2?

A. Humans have a gene, Aniridja, which is 9OVo


homologous to ey.
B. Ectopic expression of ey in the mouse results
in ectopic compound eyes.
C. ey mRNA forms hybrids with small eyes
antisense mRNA.
D. Several other organisms with visual organs
have genes homologous to ey.

93. A population of Drosophila is in Hardy-Weinberg


equilibrium with respect to the eyeles.r gene. Which
of the following conditions must apply?

A. There can be no selective pressure on the


population.
B. Genetic drift must be present in the population.
C. Mutati-ons must occur in the population.
D. There -can be no random mating between
members of the population.

94. A group of 1000 flies is in Hardy-Weinberg


equilibrium. If the frequency of an eyeless mutant
allele is l07o,how many flies in the population will
have small eyes?

A.0r
B. I
c. 180
D. 810

Copyright @ by The Berkeley Review 5bb The Berkeley Review


Specializing in MCAT Preparation
Biology Meiotic Nondisjunction Passage XY

Passage XV (Questions 95-100) 96. The commonest form of aneuploidy in liveborr


infants is Down's syndrome (trisomy 21). It follo$.
Maintaining of the normal chromosome number in the an incidence curve that is indicated in the diagrar:
gametes requires the separation of the paired shown below:
chromosomes at the first meiotic division and the
separation of the sister chromatids at the second meiotic
division. The failure to disjoin properly at either division
will produce gametes with abnormal numbers of
chromosomes and offspring with an abnormal number of
chromosomes in each cell.
l0
8
Aneuploidy (having an abnormal number of 6
chromosomes) is the most frequently observed -o 4
chromosomal abnormality in human infants and 2
miscarried fetuses (abortuses). The following table a)
tr.
indicates the incidence of this condition in both live and
O 1
miscarried fetuses: 0.8
c) 0.6
0.4
Incidence in Incidence in
0.2
Aneuploidy Live Births Abortuses
Trisomy 21 1/700 - l/1000 8/100 - 9/100 0.1
18 1/6000 - 1/9000 5/100 25 30 3-5 40
13 Ul2,O00 , 1/24,000 6/100 Matemal age
XXX U975 Rare
XXY 1t930 Rare What can be determined from this graph regardiig
XYY U975 Rare the incidence of trisomy 21?
Monosomy X 1/2500 - 1/5000 20/r00
Thble l. Incidence of aneuplody in livebom children and I. The incidence is lowest when a womrtt
in recovered abortuses. conceives a child in the latter half of tire
second decade of her life.
II. The incidence is approximately the same :l
1 3-year-olds and 35-year-olds.
III. The incidence is highest in the age r3rgt
closest to menarche.

A. I only
B. II only
C. I and II only
D. I, II, and III

95. Karyotypes are pictures of chromosomes that are


prepared by arresting cell development with 97. How does the karyotype of a person with D;'mn
colchicine. Paired chromosomes are spread out on a syndrome differ from the karyotype of a perscn
slide and photographed to produce a karyotype. affected by this genetic disorder?
What information CANNOT be learned from the
karyotype of fetal chromosomes obtained from an A. The Down individual has three copiei.
amniotic fluid sample of a fetus? chromosome 21.
B. The Down individual has three copiei
A. The sex cif the individual chromosome 18.
B. The incidence of trisomy C. The Down individual has one
C. The presence ofdevelopmental defects chromosome 21.
D. The incidence of monosomy D. The Down individual has one
chromosome 18.

Copyright @ by The Berkeley Review 556 The Berkeley Rer


Specializing in MCAT
Biology Meiotic Nondisjunction Passage XV

98. Does an individual with trisomy XXY (Kleinfelter's


syndrome) have a definite genotypic sex?

A. Yes, the person is a female, based on


genotype.
B. Yes, the person is a male, based on genotype.
C. No, the person has clearly differentiated
genitalia of both sexes.
D. No, the XX chromosomes cancel out the
effects of the Y chromosome, producing a
person with genitalia not clearly differentiated
for either sex.

99. What is the commonest form of aneuploidy among


abortuses?

A. Trisomy l8
B. Trisomy 21
C. Trisomy X
D. Monosomy X

100. It is possible through an examination of restriction


fragment length polymorphisms (RFLPs) to identify
the parent from whom a nondisjoined chromosome
originated. Given the following gel electrophoresis
pattern for a child with trisomy 21, which parent or
parents contributed the extra chromosomal material?

Mother Child Father

"9
EE
- - -
-Il- -l-
| !o
- -
A. Both the mother and the father
B. The mother only
C. The father only
D. Neither the mother nor the father

Copyright @ by The Berkeley Review 357 The Berkeley Review


Speciatizing in MCAT Preparation
Biology Genetic Information Section IX Answers

1. D is correct. In order to find the percentage of iron in hemoglobin, we must first find the mass of iron in
hemoglobin and then divide that value by the mass of hemoglobin itself. Information needed to do this is given in
the passage. The quotient is multiplied by 1007o to give the desired answer. Since each globin protein has one iron
atom, and each iron atom has a molecular weight of 55.8 amu, we multiply 4 iron atoms by 55.8 amu to get an
overall iron mass of about 223 amu. We next divide this value by the molecular weight of hemoglobin, which is
about 65,000 amu. This gives a value of 0.0034. Multiplying this value by IOOVo gives an overall percentage of iror
in hemoglobin of 0.347o. The correct choice is D.
2- B is correct. A zwitterion is a molecule (e.g., amino acid or protein) that has an overall net charge of zero. Er e n
though the overall net chatge is zero, the molecule can still have a charge. A change from HbA to HbS involves the
substitution of a negatively charged amino acid (Glu) for a neutral amino acid (Val). HbS is now a protein with one
less negative charge. But having one less negative charge on HbS does not mean that the molecule no longer has "
zwitterionic form. It still does, but the zwitterionic form of HbA is slightly different from the zwitterionic form r:
HbS. This allows us to eliminate choice D.

Since we are changing the zwitterionic form of the protein when we go from HbA to HbS, we must also be changin_.
the isoelectric point (pI). Remember, the pI is defined as the pH at which the zwitterion is at its maximun,
concentration. This allows us to eliminate choice A.
What happens to the pI if we change HbA to HbS? It's the same as what happens to the pI of a protein when one ::
its negative charges is removed. The pI increases (becomes a larger value). As you have learned in gener",
chemistry, we can determine the value of an isoelectric point through titration. Suppose we start off with tu:
beakers, one beaker having a fully protonated HbA protein, the other beaker having a fully protonated HbS protei:.
We begin to add a strong base (e.g., NaOH) to each of these beakers. As the base is added, we begin to titrate off tht
hydrogen atoms on those amino acids in each protein that have ionizable side chains. Note that HbA and HbS .rr:
identical, except for the presence of valine at the p6 position in HbS. In other words, HbS has one less ionizable si.i*
chain. Valine does not have a side chain pKn value and therefbre has no ionizable hydrogens on its side chain. n:
HbS has one less ionizable side chain, then the pI (i.e., the zwitterionic species) for HbS will be reached sligh:-;
later than the pI for HbA. The pI for HbS is a little higher than the pI for HbA. The correct choice is B.

3. C is correct. In the second paragraph of the passage it states the pI of HbA is 6.9. A buffered solution of pH 9 ".
about 2 pKa units above the pI of6.9. This tells us that HbA is negatively charged (and behaves as an anion) at pi
values greater than its pI. This can be verified by using he Henderson-Hasselbalch equation. Negatively charll;
anions migrate toward a positively charged electrode. Thus, the positively charged electrode is the anode. This mu;r
information eliminates choices A and D.

Because HbS has a valine residue instead of a glutamate residue at the p6 position, it is missing one negative char::
We know that the pI for HbS is slightly less than the pI for HbA; the exact pI for HbS is irrelevant. Since r: :;
slightly less than the pI for HbA, we find that HbS, at a buffered pH of 9, is also negatively charged and migra::r
toward the positively charged anode. This is exactly what we see in choices B and C. Since we cannot pick b:il
answers, there must be something we have overlooked.

What we have overlooked is that both HbA and HbS, at a buffered pH of 9, contain a different amount of nege:,, *
charge. HbA has one more negative charge than HbS. Since HbS has one less negative charge, it moves more slo'n l
toward the anode. There is less force pulling on HbS than on HbA. This is why we are able to see a separs:--m
between HbS and HbA in the two lanes in the gel. Since HbA moves toward the anode at a faster rate than HbS. r: *
f'arther down in the gel. In other words, Lane 2 represents HbA, while Lane 1 represents HbS. The correct choice h
C.

4. A is correct. The passage clearly states that the difference between HbA and HbS is one amino acid. In the quesr::,rr-
we read that each amino acid is coded for by three bases in DNA. Those three bases represent a reading frame .r**
we will see later, an enzyme "reads" the bases in a particular strand of the DNA duplex and then makes a : -,li
calledmessengbr RNA (mRNA). Then ribosomes next "reads" those copied bases three at a time. Each set of ti::m
bases is referred to as a codon, and each codon codes for a particular amino acid. As the ribosome joins :lnn
individual amino acids together in sequence, a protein is formed.
A base-substitution mutation is the substitution of one base in a DNA sequence for another base. Recall that D\*r
contains the bases adenine (A), thymine (T), guanine (G), and cytosine (C). For example, suppose we examin: :'nu

Copyright @ by The Berkeley Review 558 The BerkeleY Kelicmrr -,1r.

Specializing in MCAT Preparatitm


Biology Genetic Information Section IX Answers

base sequence in the DNA coding strand for the sixth position in both HbA and HbS. In HbA, that sequence is -
XXX-CTC-XXX-, where "X" is a base we are not interested in at the moment. In HbS, that sequence is -XXX-
CAC-XXX. If we were to look at a table of the genetic code, we would find that CTC codes for glutamate, while
CAC codes for valine. In this case, there has been a base substitution (A for T) that distinguishes HbA from HbS.
Note that this single base substitution does not affect the rest of the DNA sequence (i.e., it does not alter the reading
frame) and therefore does not alter the rest of the amino acids in the HbS protein.

Let's consider what happens in a base-addition mutation. These types of mutations do alter the reading frame. For
example, suppose we add an "A" between the "CT" bases in -XXX-CTC-XXX. The new sequence, read three bases
at a time, becomes -XXX-CAT-CXX-X-. It turns out that CAT codes for valine. However, the rest of the reading
frame beyond that point has changed as well. This type of mutation does not generate the HbS molecule. A base-
deletion mutation has similar consequences. If we were to remove one of the bases in the "CTC" portion of -XXX-
CTC-XXX-, then we would again alter the reading frame. This type of mutation also does not generate the HbS
molecule. The correct choice is A.

5. A is correct. If the father has sickle-cell trait, then his hemoglobin composition is HbA and HbS. The mother, who
is heterozygous for HbC, has the hemoglobin composition HbA and HbC. We can set up a Punnett square as shown
below. We find that one quarter (0.25) of the children should not have any hemoglobin disorder.
Father
AS

^[;T;_I
Mother.ffi Only l/4 of the children
are free of any
hemoglobin disorder.

HbC turns out to be the second hemoglobin disorder identified. This disorder is caused by a change in amino acids
glutamate to lysine at the p6 position. Individuals who carry the gene for HbC have a milder form of the disease,
compared to those who carry the gene for HbS. The correct choice is A.

6. B is correct. This was the first part of Mendel's experiment, by which he was able to assure himself that the forms
of the traits he was studying were indeed constant, transmitted regularly from generation to generation. With this
constancy in mind, he felt safe to carry out the experimental crosses. In hindsight, we must consider the entire
' experiment to appreciate the significance of choice B, the correct answer. From this step, Mendel could go on to
carry out crosses to get the Ft generation. If the P generation had not been true breeders (if they were
heterozygotes), he might have seen the recessive trait in the Fr generation and not realize that it is indeed recessive.
Remember, he saw only one trait in the F1 generation of true-breeding varieties of peas, and this is considered the
dominant forrn of the trait. The correct choice is B.
7. D is correct. lf ll2 of the individuals in the Fz generation are not true breeders, they are heterozygotes. If
heterozygotes are allowed to self-pollinate, they produce progeny that exhibit a dominant trait in a ratio of 3:l over a
" recessive trait. Let's take white-flowered versus purple-flowered individuals. A heterozygous individual is Ww. If
we cross Ww x Ww, we get WW, 2Ww, and ww. This shows that the expression of the dominant to recessive trait is
in a ratio of 3:1. The correct choice is D.

8. A is correct. The second filial F2 generation comes from the self-pollination of the F1 generation. Recall that all Ft
individuals are heterozygotes, because they are the product of two different, but true-breeding parents. Therefore, if
we cross Ww x Ww, we get WW, 2Ww, and ww. From this cross, it becomes clear that the ratio of homozygotes to
heterozygotes is l:1. This is different from asking for the ratio ofphenotypic expression. Since the question is
asking about the genotypic makeup, 1: I is correct. The correct choice is A.

9. B is correct. A human cell that is in prophase of meiosis I has a replicated set of chromosomes, but homologous
pairs have not yet separated. They still have 46 chromosomes. Even though the amount of genetic material in the
cell has been doubled, the number of chromosomes remains the same. One could call the chromatin ploidy number
4, but still only two sets of chromosomes remain in the cell. The choice becomes diploid, with n = 2. The correct
choice is B.

Copyright @ by The Berkeley Review 559 The Berkeley Review


Specializing in MCAT Preparation
Biology Genetic Information Section IX Answers

10. B is correct. We are performing a test cross to determine the genotype of the flower. We can do this by crossing it
with a plant of known genotype (the white flower must have the genotype ww). If the plant in questioriis
heterozygous, we have the following cross: Ww x ww. The possibilities are 2**, and 2 ww. Therefbre, if the plant
in question is heterozygous, we should expect to see that 1/2 of the plants have purple flowers. The correct choice
is ts.

11. C is correct. The pods themselves do not segregate and contain uniform types of peas as shown. Each pea within a
pod is an individual case. Let us consider the other choices. The ratio of wrinkled to round seeds is ionect; but
again, not all the wrinkled seeds would be fbund in one pocl. The number of progeny in the Fz generation is correct.
as only two of the pods from F1 are used to create the F2 generation. Finally, we have no reason to believe that the p
generation are not true breeders. In fact, we have evidence to the contrary. That evi<lence is that the peas in the F1
generation are all uniform. Again, each pea, not an entire pod, is an individual case where segregation of alleles is
occurring. The correct choice is C.

12. B is correct. The following Punnett square shows why. Each pink parent is Rr. This gives t homozygous red (RRr.
2 homozygous whites (rr), and 2 heterozygous pinks (Rr). The correct choice is B.

Rr
[;-EI
H
l*l"l
13. D is correct. Homologous pairs are chromosomes paired with each other; they are not specific gene loci. Thar
means choice A is incorrect. Pheromones are substances secreted by one member of a species that affect
other
members of the same species, so choice B is incorrect. Choice C is a nonsense answer and is incorrect. Alleles fit the
detlnition given. The correct choice is D.
14. A is correct. A monohybrid cross involves individuals that differ only with respect to the alleles at a single locus
The flower example assumes only one trait, color, distinguishes the flowers. Choice A is correct. A test cross
involves a homozygous parent and an unknown parent. By examining the offspring, one can determine the genolpe
of the unknown parent. Choice B is incorrect. A dihybrid cross involves inaivlduits that differ at two alleles. Thr:
would yield a 16-box Punnett square, not a 4-box Punnett square, so choice C is incorrect. Inbreeding mear,
crossing closelyrelated individuals. The parents ofthe Ft generation are notclosely related. Choice D is als.
incorrect. The correct choice is A.
15. A is correct To get a prnk tlower with curly leaves, the genotype must be Rr 11. The Rr yields a pink color due i_
incomplete dominance. The ll yields a curly leaf due to classical dominance. Make a l6-box Punnett square to prc.,:
the answer to yourself. Only 4 out of the 16 offspring would be pink flowers with curly leaves. This is a l_{i
probability, or 0.25 since probabilities are expressed as a decimal fiaction of l. The correct choice is A.
16. C is correct. The passage tells us that the A and B proteins are codominant, and that O is recessive to A and B. T:t:
text of the question tells us that the Rh factor is classically dominant/recessive, with the + being dominant. So rh:
queen contributes A and B alleles, and the king contributes B and X (either B or O) alleles. The offspring of tl,i
king and queen must be either AB, AO, BB, or BO. Since the presumed heir is A, this is compatible witlithe _t-
choice. So far, so good. However, the answer lies in the Rh-factor part. Both the king and the queen a.e homozysi;;
recessive, or negative, for the Rh factor. Since the presumed heir is Rh positive, this allele hadto come from anorhr
f'ather. Neither the queen nor the king could have provided it. The presumed heir is not the child of the king. Choi-;,
A and B are incorrect. The passage and the text ofthe question did provide what is needed to answer the qiestion. r
choice D is also incorrect. The correct choice is C.
17. A is correct. If neither the A nor B proteins are expressed on the RBC of an individual, then that person is ty,pe -
Type O blood is recessive in the ABO blood group system. The person with type O blood is homozygous recessr. :
A child born of two people who are homozygous recessive for the same traiiwill also be homozygous recessr,:
Since neither parent expresses A or B, the child cannot express them, either. Statements II and III are incorr:r
Incidentally, the same reasoning holds true for the Rh factor. The correct choice is A.

Copyright O by The Berkeley Review 360 The Berkeley Revim


Specializing in MCAT Preparatim
Biology Genetic lnformation Section IX Answers

18. C is correct. This can be calculated from the information given inthe passage.The genome of E. coli is said to
contain 4.7 x 106 nucleotide base pairs. It is also stated in the passage that each replication fork polymerizes at a rate
of 500 nucleotides per second. Since there are two replication forks (the replication is bidirectional), the total rate is
about 1000 nucleotides per second or 60,000 nucleotides per minute. Simple division gives one an answer closest to
choice C, 80 minutes. If one did not take into account the two replication forks, one would come up with 160
minutes, or choice A. Choices B and D can be eliminated with the above information. The comect choice is C.

19. A is correct. Remember, the eukaryotic chromosome has so much DNA that multiple origins of replication are
needed to have any sort of timely, complete replication. Let us consider the other possibilities: Choice B is incorrect,
because the chromosome shown is circular. Circular chromosomes are found in prokaryotes; the question specifically
asks for a eukaryotic chromosome. Consider choice C. While this chromosomes is linear, there are no replication
bubbles, and the question asks for what a chromosome will most likely look like during DNA replication. Consider
choice D. This answer can be tempting, but one must remember that eukaryotic chromosomes have multiple
transcription bubbles, and this shows only one. Yes, this may happen and does happen; but the question asks for the
best representation, which is the chromosome with multiple replication bubbles. The correct choice is A.

20. C is correct. Chromatin is DNA wrapped around histone proteins. The appearance is that of beads on a string. One
speculative model claims that the histone is split in half while the DNA is replicating. The two halves stay on one
parent strand as the new DNA is being synthesized. After the DNA is completely replicated, the two halves
recombine to form the functional histone.

Consider ttp other possibilities: Choice A is incorrect, because we are talking about the replication of a single
chromosome, and the presence of other chromosomes will have no effect on replication. Choice B is incorrect,
because the enzymes involved in polymerizing are not prejudiced towards whether the bases are coding (exons) or
are noncoding (introns). They read the template and insert the correct base pair in the growing strand. Choice D is
incorrect, because the mere location of the DNA does not affect the rate of polymerization. We need to be looking at
the actual structure of eukaryotic DNA compared to prokaryotic DNA. When we do this, we remember that
eukaryotic DNA is packaged with proteins to make chromatin. While choice D is not the correct answer, the
principle behind the claim is extremely important and represents one of the fundamental differences between
-ukaryotes
and prokaryotes. Eukaryotes have their DNA enclosed in a nuclear envelope, while prokaryotes do not
have a nuclear envelope. The correct choice is C.

21. B is correct. This question is very straightforward and calls upon your knowledge of the cell cycle. The replication
forks will become activated when DNA is undergoing replication. This occurs in the S division of the cell cycle. Gt,
S, and Gz are known collectively as interphase. Interphase usually takes up about9OVo of the total cell cycle time.
Gt and G2 are stages where biosynthetic activities of the cell take place, preparing the cell for DNA replication and
division. S, as already stated, is where the DNA replication takes place. Finally, M is the mitotic stage where we see
both a nuclear and a cellular division. Based on this information, we are most likely to see the replication forks
activated during the S phase ofthe cell cycle. The correct choice is B.
3H-uridine, which will be incorporated not into DNA, but
22. B is correct. The question tells us we have the addition of
into RNA. RNA ia initially made in the nucleus, but will eventually be removed from the nucleus and enter the
cytoplasm. In the cytoplasm, it will undergo translation to make a functional protein. Therefore, we are most likely
to sei the label in both the nucleus and the cytoplasm. Based on this information, we can easily eliminate choices A
and C. They are both true but incomplete, and therefore not the best answers. Choice D is incorrect, because the
membrane will not have nucleotides incorporated into its structure. The nuclear membrane is composed of primarily
phospholipids and large protein granules making up the nuclear pores. The correct choice is B.
tl C is correct. Statement I is correct, because we see the presence of multiple replication bubbles, which should
immediately suggest a eukaryotic organism. Statement II is incorrect, because there is no evidence for this claim.
Yes, we see more label in B, but half of the label is less dense, indicating some sort of change in the level of
radioactivity. The level of radioactivity is not an indication of the rate of polymerization. If we look carefully, the
dense regions a/6 the same in both A and B, indicating that the polymerases are not different, but,exactly the same.
Therefoie, statement II is false. Statement III is correct, because in B we see more silver grains from the
radioactivity. That means that more incubation occurred. However, we can definitely recognize the low level of
radioactivity. That must mean that there was a pool of unlabeled nucleotides added to the medium. When this
happens, the labeled and unlabeled are competing for the polymerase, and we should see a smaller level of
radibactivity. This is what we see by the lesser density of silver grains. The correct choice is C.

J,rpyright @ by The Berkeley Review 361 The Berkeley Review


Specializing in MCAT Preparation
Biology Genetic Information Section IX Answers

24. D is correct. Gene duplication involves copying the gene or genes in the DNA that are already there. If gene
duplication occurs, then replication of those genes into new duplexes of DNA will give the same DNA sequences
Similarly, the transcription of DNA into mRNA will be similar, as will the translation of the mRNA into a
potypepiiAe chain (composed of amino acids). If similar mRNA sequences can be synthesized, then similar tRl{A
tt.u*tir RNA) and even similar rRNA (ribosomal RNA) sequences can be synthesized. Therefore, choices A and B
both occur. We can eliminate them. Duplication of genes results in more of the same types of genes. And this means
that more (of the same) genes can begin to diverge. Each of the newly duplicated genes can experience its own forn
of mutation, independerit from the oiher duplicated genes. This is how natural selection begins to take hold. Sincc
choice C is correct, we can eliminate it. tf we duplicate a gene, we will have two genes of equal size and not om
gene which is twice as long. Therefore, when a transcript of mRNA is made, it will still be as long as the gene
hfl
was duplicated and not any longer. The correct choice is D.

25. D is correct. A genetic locus refers to a given gene location on a chromosome. Males have one X chromosome
(arl
one Y chromosome), while females have two X chromosomes. The frequency for color blindness in the male foi

given as 8% (0.08) in the passage. In order for females to be color-blind, they must be homozygous for th
Iondition. The third pu.ug.upi of the passage says that red-green color blindness is sex-linked and recessive. Whn b
rhe chance of a female U"eing cotor-Uiind [.e., ito*orygooi for the allele)? It would just be q2, or (0.08) x
(0
which is 0.0064 or 0.64Vo. The correct choice is D.
26. C is correct. Paragraph 3 of the passage says that about 67o of all males who are color-blind have a deutan defo
which results in abnoimal syntheais of the green-sensitive pigment. The percentage of females that would be derr
color-blindis(0.06)x(0.06)=0.0036 =0.36Ta.Notethatthepercentageof femaleswhoareprotan(red-sensi
pigmenr defect) color-blind is (0.02) x (0.02) = 0.0004 = O.04Vo.If we add 0.36Vo and 0.04Vo, we get 0.40%"
inIia"n." of red-green color blindness among females in the United States. The correct choice is C.
27. B is correct. In the passage we learn that about 6Vo of all males who are color-blind have the deutan defect while
of all males who are coloi-blind have the protan defect. In a heterozygote, the dominant allele is p and the reces
allele is q. We see that q = 27o = 0.02 and that p = 987o = 0.98. From the Hardy-Weinberg equation, we find thm
heterozygote women u'ho
= 2(0.98j(0.02) = 0.O3gZ x lOOTo = 3.927o or about 4Vo. Note that the percentage of
.u.ri.r, of deutan color blindness is 2(0.94X0.06) = 0. 1128 x 1007o = ll.287o or about ll%o. lf we add 1l7c +
then we get 157o, which is roughly the percentage of women who carry the color blindness defect. The
choice is B.

28. be represented by D, then the recessive


D is correct. If we let the dominant allele for deutan color blindness
can be represented by d. If the husband is deutan color-blind,
he has the recessive allele, d, on his one and
chromosome. His wife has normal color vision. She has two X chromosomes, and they both carry a dorn
allele
Father Mother
blind)
(color (normal)

II
xtxd XDY
Daughter Son
(carrier) (normal)

If they have a daughter, then she will receive one X chromosome with the D allele from her mother and
chromosome:with the d allele from her father. The daughter will be heterozygous for the deutan trait. In other
she will be a carrier. If they have a son, the mother donates an X chromosome with the D allele but the falhu
donates a Y chromosome, which has no defect for color blindness. The son will thus have normal color visim-
correct choice is D.
29. C is correct.The easiest way to follow the alleles is to draw a pedigree as shown below. The symbol D rep
the dominant form of the allele for the deutan trait, while the allele d represents the recessive form. In order

Copyright @ by The Berkeley Review 362 The Berkeley


Specializing in MCAT
Biology Genetic lnformation Section IX Answers

trait to be expressed, the genotype must be dd in the female or d- in the male. The symbol C represents the dominant
autosomal allele required for proper development of the cones. If the genotype cc is present, the individual will have
complete color blindness.

Father I Mother
(does not have deutan trait) (will express deutan trait)
(does not have color vision) (will have color vision)

It
Daughter Son
(carrier tbr deutan trait) XDXd xdv (will express deutan trait)
(will have color vision) .
a
I I "ll' (will have color vision)

By examining the pedigree, we see that both children will have color vision. However, only the son will show red-
green color blindness by expressing the deutan trait. The correct choice is C.

30. B is correct. The last paragraph of the passage says that if a hamster is homozygous for the a allele (i.e., a/a),then
its coat will be white. In other words, the hamster will be albino. The same paragraph also states that the normal
(wild-type) allele is usually designated as +. If a hamster has a + allele at one of the two loci, then the coat color will
be normal. This must mean that the + allele is dominant over the a allele, or that the a allele is recessive to the +
allele. If the a allele and the + allele were codominant, we would see both types of coat color. If the + allele showed
incomplete dominance, we would also expect to see both types of coat color. Also, the a allele, as written in the
answer choices, is written in the lower case. This is the nomenclature that has been agreed upon by geneticists to
indicate a recessive allele. The correct choice is B.
31. C is correct. A true-breeding strain of black hamsters will always produce black hamsters (unless there is a mutation
of some type). In order for progeny hamsters to be purely black, they must always receive a B and a + allele from
each parent. Therefore, each parent and the progeny hamsters must have the genotype B/B +/+. The correct choice
is C.
7'' C is correct. In the third paragraph of the passage, we see that the recessive allele Z' results in brown coat color
when it is present in the homozygous (b/b) condition. The fourth paragraph says that if the hamster has the allele a
present in the homozygous (a/a) condition, then the coat color will be white. The genotype a/ameans that these
hamsters do not have any pigmentation (that is why they are white). The correct choice is C.

33. A is correct. We can use the outline shown below to help us determine the coat colors of the first filial (Fr)
generatbn progeny.

Black coat
Parents (P1)
color

Gamete Gamete
lormation formation

ba
ffi
X
Cross

0
Firstfilial(Ft) re Blackcoar
generation a=:r---:-: :--Onr'---.-T: color

Copyright @ by The Berkeley Review 363 The Berkeley Review


Specializing in MCAT Preparation
Biology Genetic Information Section IX Answers

We see that in the F1 offspring, the genotype is B/b +/a. Since B is dominant over b, the coat color will be black
Since the wild-type allele + is dominant over the a allele, the coat color will be normal, which in this case is black. If
there had been two a alleles (e.g., a/a), the coat color would have been white. The correct choice is A.

34. B is correct. In order to estimate the genetic map distance between the two genetic loci, we must use the equation
given in the passage, which is:

Number of Crossover Indidivuals in F2


Vo CO = x lOOTo
Total Number of Individuals in F2

We need to determine the number of crossover individuals in Fz, and we need to determine the total number of
individuals in Fz before we can estimate the genetic map distance. Refer to Experiment II.

In Experiment II, we learned that one hamster from the Ft generation of Experiment I was back-crossed to the pareil
with the double recessive genotype (b/b a/a). We determined that the genotype of the hamster from the Fl
generation is B/b +/a. The result of this cross will give the Fz progeny, namely 66 black hamsters, 34 broc,r
hamsters, and 100 white hamsters. The total number of hamsters is 200. At this point we know the total number d
individuals in Fz. Our equation now becomes:

vo CO =
Number of Crossover Indidivuals in F, x loovo
200

This alone does not tell us the number of crossover individuals we have. Let us next assume that the two loci
linked, and let us also suppose that no crossover events took place. What would we expect? Let's look at this
cross as outlined by Experiment II. We can set up the chromosomes as shown below and generate the Fz progeny'.

First filial (Fr)


generatlon

ffi
B+ ba
ttlr:rl-*irirCl Whitecoat
Blackcoat
color <==-a.-"llr:l.-:rlr--;=- (...=f........ color
-T-.. a
ba
Gamete Gamete
formation formation

ba
@

ba
ffifir,'fir.rrre
X
Cross

--_--\-
\
\ reba Gamete

\
Gamete
B+
(1.:rrlx'|:=l*.E1 rTr @ Black coat
(:ir:l_lriTl*.Iillll'l!: color
!
b
ba White coat
Gamete @ color

F2 progeny from test cross

Copyright @ by The Berkeley Review 36.4 The Berkeley


Specializing in MCAT
Biology Genetic lnformation Section [X Answers

If no crossover took place, we would expect to find half of the progeny B/b +/a and half the progeny b/b a/a. In
other words, we would expect that 507o of the hamsters would be black and 5OVo would be white. We would not
expect to find any brown hamsters. But this is not what is observed in the results of Experiment II, which was 34
brown hamsters. This tells us that the brown hamsters are the crossover or recombinant hamsters. If we use this
value in our equation, we get:

7o CO = 34 * l00Vo = lTVo
200

Since one genetic map unit (m.u.) gives a recombinant frequency (RF) of I percent, we have a recombination
frequency of l7% or 17 m.u., which is choice A. But choice A is not the correct answer.

Let's see what happens if a crossover event does take place. In paragraph 4 of the passage, we read: "If there is
crossing over between any two non-sister chromatids, intrachromosomal recombination results." Crossing over must
take place between non-sister chromatids. One possibility of how this could happen is shown below in Crossover 1;

<-:-r:;.=:-:.o-lrl;;r=- Blackcoat
<=.--lr:'-.-. cr. :;r==' color
Crossover I
b
Crossover
a ba
b a ba
<rl--r---:;r;Ct :: Whitecoat
@ re color
ba
There is also another possible crossover event, as shown below in Crossover 2. Again, this crossover is taking place
between non-sister chromatids :

ba <:l:r.Tl:ll..--r::.
@ Whitecoat
ffi::i:-:tr-ll.n color
B + Ba
Crossover 2 Crossover
b a b+f
(::lirl-:]..:..TrJii:l==rrili,, Browncoat
rcba .re color

In Crossover l, we produced black and white hamsters in equal numbers. In Crossover 2, we now produce brown
and white hamsters in equal numbers. We can clearly tell from the phenotype which are the hamsters are brown, and
we know that these brown hamsters can arise only from crossing over. What about the white hamsters? We can get
white hanrsters from non-crossover events (see above). How can we tell which white hamsters came from non-
crossover events and which came from crossover events? We can't! Here is where the insight comes in. In order to
get a more accurate number of recombinants, we must assume that there are also 34 white hamster recombinants.
This is because when we produce one brown hamster recombinant, we also produce one white hamster recombinant.
And since we have 34 brown hamster recombinants, we must add an additional 34 white hamster recombinants to
the total number of crossover individuals in the Fz generation. (Why don't we need to do this for Crossover I ?)

We can now use our equation to calculate the genetic map distance between the two loci:

ToCO = 2 x 34 x IOOVo = 347o


200
In other words, there are 34 m.u. between the genetic loci. fNote: Since there can also be double crossover events,
the actual grhetic distance between the loci is probably greater than 34 map units. What we have calculated is the
minimum map distance.l The correct choice is B.
35. A is correct. If the genetic loci were not linked, we would expect to see a recombination frequency in the Fz
generation of 5O7o (from the third paragraph of the passage), which means that there was interchromosomal
recombination (i.e., between the chromosomes). If the recombination frequency is less than 5OVo, it means that there

Copyright @ by The Berkeley Review 365 The Berkeley Review


Specializing in MCAT Preparation
Biology Genetic lnformation Section IX Answers

was intrachromosomal recombination (i.e., within the chromosome). If there is intrachromosomal recombination.
then genetic loci must be linked.

There is another way to look at this: We would expect half of the hamster offspring to have white coats, which thel'
do. What about the other half of the hamster population? If the two genetic loci were not linked, we would expect to
see this half of the hamster population having 507o black coats and 50% brown coats. But when we look at the
values given in Experiment II we do not see this. Instead, we see that there are almost twice as many hamsters with
black coats as there are with brown coats. In order for this to happen, the genetic loci must be linked. If they were
not linked, they could be called unlinked (i.e., they assort independently). The genetic loci are not entirely epistatic"
because we do see black and brown hamster coats. This means that the a allele is not homozygous (a/a) but rather
heterozygous (a/+).The a allele has to be homozygous before it can influence a different genetic locus. Besides, the
B and b genetic loci are not epistatic to the a allele. The comect choice is A.
36. B is correct. In the sixth paragraph of the passage, we see that neither the black nor the brown coat color of a
hamster will be expressed, if the hamster is homozygous for the a allele. For example, consider these chromosomes:

B
€.-TIr1Ir'T:T.Efl::=t.-'It)
rerF
If the a allele were not present, the coat color of the hamster would be black. This is because the B allele is domin*m
over the b allele. However, the presence of the a allele, in the homozygous form, suppresses the activity of the alleh
for the black coat color. Because the a allele is recessive (note that it is written in a lower case), we can cali rhrn
recessive epistasis . As stated in paragraph 2 of the passage, incomplete dominance and codominance are concerned
with heterozygotes. Both of these characteristics involve expression of either one of the same allele (e.g., B or b). [o
the case of epistasis, allele a influences some other allele, such as allele B or b. The correct choice is B.

37. D is correct. Since the average molecular mass of an amino acid residue is I l0 D, the protein has 40,000/110 = 360
residues. This is specified by 3 x 360 nucleotides. Each base pair has an average molecular mass of 660 D. Hera,;c-
the molecular mass of the DNA is 660 x 1080 = 700,000 D (or 7 x 105^ D;. The length of this molecule, considening
that B-DNA has a rise along its helix of 3.4 A per base pair, is 3.4 A x 1080 = IIOO it = 0.37 pm. The
choice is D. "o-"i
38. C is correct. There are 4.0 x 106 bp in the E coli chromosome, and it takes 40 minutes to replicate the chromosom
Thus, (4.0 x 106 bp)/(40 min) x (1 min/60sec) = 1,666 bp/sec replicated. But since there aie two replication fodq
there are about (1,666 bplsec)l2 or about 833 nucleotides per second added. The closest answer is 850 nucleoridse"
The correct choice is C.

39. B is correct. At 16 pm/min, each replication fork travels 4800 pm or 4.8 x l0-3 m in 5 hours (300 minutest
replicate'theentirecontentof DNA(1.2m)inthisinterval,theremustbe(1.2m)/(4.8x 10-3m/replicationfoikr*
250 replication forks. The correct choice is B.

40. D is correct. In a rich medium, a second round of bidirectional replication begins at the origin when the first
is only half-completed. This second initiation results in four new replication forks, making a total of six.

6 replication forks

Thus, one round of replication is competed every twenty minutes, and each daughter cell at division receives I
chromosome that is already half-replicated. The correct choice is D.

Copyright @ by The Berkeley Review 366, The Berkeley Reviev


Specializing in MCAT Preparation
Biology Genetic lnformation Section IX Answers

41. C is correct. okazaki fragments are about 1000 to 2000 nucleotides in length. The E coli
chromosome consists of
about 4 x 106 bp. Thus,E coll must produce about 2000 to 4000 okazaki fragments. The correct choice is C.
42. C is correct. Since DNA is antiparallel, the complementary strand must be 5'-pGpTpCpTpApT-3'.
The correct
choice is C.
43. C is correct. In the next-to-last paragraph in the passage, it we read that "...all known DNA polymerases
require a
primer before new DNA can be synthesized...." And in the last paragraph of the passag",
*" u'r" told that .,Once the
primer has been established,_DNA polymerase III begins to syntheiize DNA in the 5'
to 3' direction.,' From this
information, we can assume that DNA polymerase I must also iaa itr nucleotides in the 5' to the
3' direction (i.e., at
the 3' end) of a growing polypeptide chain and not in the 3'to the 5' direction (i.e., at ttre
s; ena;. This allows us to
eliminate statement II, which in turn allows us to eliminate choices A, B, and D. The correct choice is
C.

44. C is correct. There were two ways to answer this question. First, you could have remembered that the
base-pairing
arrangement in a DNA double helix (a duplex) follows the rules that adenine (A) base pairs
with thymine 1rj
through 2 hydrogen bonds and that guanine (G) base pairs with cytosine (C) through : nydrogen
bonds. fne onty
base-pairing arrangement in the answers that shows this is the bonding between cytosine
and guanine. The adenine-
guanine and cytosine-thymine base-pairing is not allowed. This allowi
us to pick choice C u, ih, unr".r ff you Jia
not remember this' then.you needed to rely on information in the first paragraph
of the purrul". It was stated that the
purine bases were adenine and guanine, while the pyrimidine bases
were-thy-in" uni .i;;.. Ir was also stated
that.complementary purine and pyrimidine bases ai" tint"o together through
hydrogen bonding. In the answer we
need to find that base pair that depicts a purine bonding with a-pyrimloine.
itre onry""rroi.. ir the cytosine-guanine
pair' The adenine-guanine pair representi two purines,lhile thi cytosine-thymine
pair represents two pyrimidines.
The correct choice is C.
45. D is correct' The second paragraph of the passage says that "conservative replication
would conserve the integrity
of the parental strands in the DNA duplex urt"r ..ptl"ition." In other words, uitr.
on" round oIDNA replication, we
would see one DNA duplex that contained both parental strands, and another DNA
duplex that contained the two
new daughter strands. This is shown schematically below:

When the second round of replication begins,


One round of both duplexes are now treated as parjntai
replication
.--_----_------./ Effi^,,Fg] strands each giving rise to duplex <iaughter

3R
6 strands of DNA. Thus, in the'end we"haue
o four duplexes of DNA. Two of the duplexes
contain all parental DNA, and two bf the
duplexes contain all daughter DNA.

Dupld{ DNA
RlJ
As the second round of DNA replication begins, we now treat both duplexes as being parental
strands. After the
,second round of replication, we would find two DNA duplex that contained all pa"rental
DNA and two DNA
duplexes that contained all new daughter DNA. The correct choice is D.

46. c is correct. This question is designed to get you to think about the fundamental components of DNA. The
answer
cannot be obtained from the passage. Instead, it must be deduced from your fundamental
knowledge of DNA. As
mentioned in the question,-DNA is composed of nucleotides. Each nucleotide contains
a base (either thymine or
cytosine, which are pyrimidines; or adenine or guanine, which are purines). Eactr
nlt-genoni, uur. is attached to a
ribose ring,(i."', u pentose sugar). Each ribose is attached to a phosphate. Each
oithose ring systems could
incorporate 15N. The ribose ring does not contain any nitrogen, und it seems like
a good answer choice. But what
about the acetal dnkages? The linkage between the sugar
bond'rn particular, it is.an N-glycosidic bond becauie of -a tn. nitrogenous base is referred to as a glycosidic
its attachment to a nit.olen uio- in the base. This
particular type oflinkage is also considered an acetal linkage. And sincethe
acetal linkate contains a nitrogen atom
that could be replaced by l5N, we can eliminate it as a possilble answer. The correct
choiie is C.
47. C is corect. The passage mentions that the CsCl solution is less dense near the top
of the test tube and denser near
the bottom of the test tube. If we were to centrifuge the DNA rhar was labeled
.^.iuriu"iy *iir.llN, *";;;;il;

Copyright @ by The Berkeley Review 367 The Berkeley Review


Specializing in MCAT preparation
Biology Genetic Information Section IX Answers

that its band would appear at a lower position in the test tube compared to DNA that was exclusively labeled with
t4N. This is what is represented by the control test tube (see below).

AII 14N DNA


fr
\'l-_ uuuna l+N7tsp pp4
AII15N DNA

Control

If we analyze the DNA afterone generation following the incorporation of "14N into the growth medium, then that
DNA would be neirher exclusivel/all heavy 11sN) nor exclusively all light 1laN;. Instead, the DNA would represent
a hybrid of laN and 15N DNA. The banding pattern (after analysis with ultraviolet absorption) would be
intermediate between the two bands shown in the control. Note that we do not see this pattern in any of the answer
choices.
15N; and duplexes
After two rounds of DNA replication, we find duplexes that contain the hybrid DNA (14N and
that contain exclusively tigntbNe (llN). W" wouid expect to find two bands: One intermediate between the all-l4N
[""J""J irr" all-lsN buni l"hu.u"teristic of the hybrid DNA), and one that is the same as the all-laN band on the
control. There is only one answer choice that gives this pattern. The correct choice is C.

48. A is correct. As described in the passage, one round of semiconservative replication would give two duplexes of
l4N. After the second round of replication' each
DNA, each containing one strand with-l5N and one strand with
hybrid duplex would-generate.two duplexes. One duplex would still be a hybrid 1t+p-tstri;, but the other duplex
would coniain DNA that is all 14N. This is outlined below.

l4N
I

15N
rN@\@\@
lt {
I
l''t
u@\@\@ -tlttN
Ir
I
l5N
Parental
duplex DNA
LN@\@\@
'oir
i N@\@tu4 lsN
1st Generation 2nd Generation

In the second generation, note that there are four duplexes of DNA, for a total of eight strand,s of DNA. Out of these
l5N of 3:1. The correct
eight DNA striands we see that two are l5N and six are l4N. This gives a ratio of laN to
choice is A.

49- A is correct. The characteristic that distinguishes one atom from the next is the number of protons contained within
an atom of a particular element. The atomic mass unit (amu) of a proton is about 1.0073. If the number of protons
changes, the atom changes, and therefore its elemental name changes as well. Any mass difference between atoms
of the same element is due to the difference in the number of neutrons. These atoms are referred to as isotopes. The
amu of a neutron is about 1.0087. The natural isotopes found in the nitrogenous bases are hydrogen (atomic mass of
1.01), 12C (12.00), t+N 1t+.Ot;, and 160 (16.00). The corresponding heavy isotopes would be deuterium (2.01), l3C
(13.01), tslt (IS.OO), and 18O (18.00). Heavy isotopes have a greater density, because they have one or more
neutrons in their nuclei. Electrons have an amu of about 0.0006 and do not significantly effect the mass of an atorru
The same number of water molecules would surround l5N DNA as would surround laN DNA. The correct choic
is A.

Copyright O by The Berkeley Review 368 The Berkeley Review


Specializing in MCAT Preparation
Biology Genetic Information Section IX Answers

50. B is correct. First, note that the polymers of DNA and RNA in the question have the same base composition.
Therefore, we must look elsewhere to find differences in the density of these two nucleic acids. It is important to
know the basic differences between DNA and RNA. We know that DNA contains adenine (A), guanine (G),
cytosine (C), and thymine (T). RNA contains the same bases, except it replaces thymine with uracil (U). The
question avoided the differences in the structures of thymine and uracil by leaving them out of the question. One
other important difference is the presence of a hydroxyl (OH) group at the C-2' position of the ribose ring of RNA.
DNA does not have this C-2' hydroxyl group (hence the name "deoxyribonucleic acid" for DNA). This tells us that
each nucleotide of the DNA polymer is missing an oxygen atom. In other words, the RNA polymer has ten more
oxygen atoms than the DNA polymer. The RNA polymer is heavier and shows a greater density. This allows us to
eliminate choices A and C. Consider choice D for a moment. Nucleic acids do not have positively (or negatively)
charged nitrogenous bases at physiological pH. The only charge they show is on their negatively phosphate groups.
We can eliminate choice D. The positively charged cesium ion (Cs@) can bind to the negatively charged phosphates
of both DNA and RNA. It can also bind to those free hydroxyl groups at the C-2' position of RNA as well. The
correct choice is B.

51. A is correct. Examine the bases shown in Figure I of the passage. The purine and pyrimidine bases can exist in
alternate forms called tautomers. Keto groups can be converted to enol groups, and enol groups can be converted to
keto groups. Similarly, amino groups can be converted to imino groups, and imino groups can be converted to amino
groups. Selected examples are shown below:

f^o
l-A tfoi+ "-A'-
OH H
OH

",ffup dR
r\-ti {\-)
I
R
*"/ n.4.
.-)
l
(^Ai
dR
*
dR

Cuanine (keto) Guanine (enol) Uracil (keto) Uracil (enol) Adenine (amino) Adenine (imino)

In order to form the enol in one of the bases, we must first have a keto group in the molecule. Not only that, but we
must have a hydrogen atom on a neighboring nitrogen atom that can leave and participate in the formation of the
enol. All of the bases except adenine have keto groups. Even though cytosine has a keto group, there is no hydrogen
atom on a neighboring nitrogen atom that can participate in enol formation. Therefore, the bases adenine and
cytosine cannot form enol derivatives. The only bases that can form enol derivatives are guanine, thymine, and
uracil. What about adenine? This base has an amino group at the C-6 position of the purine ring. Tautomerism leads
to the imino form. The correct choice is A.
52. C is correct. DNA and RNA are polynucleotides that contain repeating nucleotide units linked together through
phosphodiester linkages. Remember, a nucleotide is composed of a base, a ribose sugar, and a phosphate group. The
phosphodiester linkages are formed between the 5' carbon of one ribose ring and the 3' carbon of the adiacent ribose
ring (see below). In order for the acidic phosphate group to be positioned between the two ribose rings, there was a
loss of water during the reaction. Anhydrous refers to the loss of water. Therefore, this phosphodiester linkage can
also be called an acid-anhydride linkage. At neutral pH, each nucleotide contains a negatively charged phosphate
group.
\o
-\2"v
I
cH^ ^ Base

-'y+
OH
Phosphodiester
Ol
o-P=o
linkage I
o

'hil^"
I

3' end of the polymer +HO H

Copyright @ by The Berkeley Review 569 The Berkeley Review


Specializing in MCAT Preparation
Biology Genetic lnformation Section IX Answers

If the nucleotides were deoxy at the C-3' position, the DNA polymer would not be able to
form, because the oxygen
atom of the hydroxyl group at the C-3' position attacks the nexi incoming nucleotide's phosphate
group in order ro
form phosphodiester linkage. The correct choice is C.
53. D is correct. Selection of the correct answer depends on things. First, we must obtain the correct sequence ot
.two
the desired DNA strand in the 3' --> 5' direction. Following the fatson and Crick base-pairing rules, we get rh.
following sequence:
3'-T-A-C_c_c_C_T_A_5'
Notice that in all four answer choices the DNA bases read the same in the 3' -+ 5' direction. Be
aware of rhe
sequence from both ends of the polymer. Just because we are looking for a sequence that reads
from the 3' -+ _i
direction does not necessarily mean that it will be written with the 3' eni on the leti and the 5'end
on the right.
Once we have determined the sequence, the second thing we neecl to consider are the 3' and
5' ends. The 3' end of
DNA polymer represents the C-3'iydroxyl group on the ribose ring. The 5' end of the DNA polymer "
represents rhr
p\osp\ate group attac\red t9 r\e a-5 \r'ydrox'y\ group o\ t\e rrbosi rrng.\l e are \ookng
ior' oo ooa*., chcl,:: -:r,irii
beans ap\r.ospha\e (P) at.the 5' end, anil ahydroxy\
!o\\) ar the 3 end.'when writrngiout u DNA sequen;e. .r mu
so-metimes customary to include the phosphate group at the 5' ends of the
bases. rn, iTnyiri-yl groupsare lerr --,1r
of the picture. They are understood to be at the 3' end. In choices A and B, we find a phosphate at the 3, posiri .
which is not allowed" Therefore, we can eliminate the first two answers. In choice C, tie phosphate at the 5' en: ;
missing. We can eliminate this answer, too. In choice D, we see that the phosphate group ii at the 5, end. The
3' e r:
shows no phosphate group. It does not show a hydroxyl group either, but the hydioxyi group is understood to b.r
there. The correct choice is D.
54. D is correct. The distance between base pairs is 0.34 nm. The helix undergoes one complete turn (twist) even i
-r
nm' This tells us that there are l0 base pairs (3.4 nml0.34 nm = 10) per turn of the DNA double heljx. Wl mi_sh: :,:
inclined to pick choice B This would be wrong. Remember, it is 10 base pairs per turn. A base pair is compoieJ :,
two bases held together by hydrogen bonds. The question specifically aski for ihe number of baies (not base p: :.
per turn of the DNA double helix. The correct choice is D.

D is correct.The classical hydrogen bonding between base pairs in DNA occurs between adenine and thymine :.. :
hydrogen bonds) and between guanine and cytosine (three hydrogen bonds) as shown in the following drawing. T:,.r
allows us to immediately eliminate choices B and C.

Cytosine H Thymine
, Hydrogen
CH,

aY"' ,,,,,,,('ono I

aYu"',,,,,,,
Riuo,e-
N"',,,,,,,
)fo,,,,. "'il Nf N'

I jr\
,
H _
niuo..t
",,,,,,,,,,,"
X] *..
..T ./
H. ---"- -\ (.JL J
H f,,0o..
Guanine Adenine

What about choices A and D? Both answers contain bases (adenine and thymine) found in the DNA double h.';r,
Both answers also contain uracil. a base found in RNA.

Transcription Bubble
A-G-T-6-A-G_^
5'

J t_r*tr. (Y^-Mt 3'


i;;-;-'l DNA
mRNA
Hybrid
+
DNA-RNA

Copyright @ by The Berkeley Review 37o The Berkeley Revier


Specializing in MCAT preparation
Biology Genetic Information Section IX Answers

Consider choice A. Can uracil hydrogen bond to adenine? If it can, that base pair associated with DNA? The answer
to both of these questions is, yes. As the DNA double helix unwinds and forms a transcription bubble, two single
strands of DNA are exposed. One of those strands is used as a template to make
-".r"ng.. RNA (mRNA;. As ihe
mRNA is synthesized, it is hydrogen-bonded to the DNA. During transcription, a ONe-RXa hybrid is temporarily
formed (see above).

If these is a cytosine (C) in the DNA template, then in the mRNA a guanine (G) is incorporated. If there is a thymine
(T) in the DNA template, then in the mRNA an adenine (A) is incorporated. If there is an A in the DNA template,
then a uracil (U) is called for in the mRNA.

Uracil

Z\ao",,,.
tt
.N- ,N- H-N-H
Ribose- - Ht,,,,,,t,,,,i,I)
T
t*,0o."

Adenine

Remember, in RNA the base U replaces the base T. A hydrogen-bonding arrangement between adenine (in DNA)
and uracil (in mRNA) is shown above. The correct choice is D.

56. D is correct. In order to transmit information accurately from one cell to the next, there must be an alphabet that
makes the communication possible. In DNA, this alphabet is composed of the bases adenine (A), guanine (G),
cytosine (C), and thymine (T). A triplet of these bases represents a codon, and a codon specifies a singlJ amino acid.
There are 64 different codons (from 43 = 64), and among these 64 coclons there is reduniancy. In othlr words, more
than one codon can code for the same amino acid" Because of this, the genetic code is referre<l to as being
degenerate. If there were no mechanism that would allow DNA to replicate itself, then the information contained in
these codons would not be passed to the next generation. Thus, a means of self-replication is crucial. As the
information is replicated, the fidelity must be maintained. If a mutation were to be incorporated into the next
generation of DNA, then it could have deleterious consequences. Therefore, a very low mutation rate is essential.
DNA can exist as either a single strand or a double strand. In the doubled-stranded form, it can exist in a number of
different states (e.9., A-DNA, B-DNA, or Z-DNA). DNA is said to have variable conformations. The information
passed on to the next generation is still contained in these different forms of DNA. It is just a matter of having the
appropriate mechanism to transmit that information. DNA, as a conformationally variable molecule, woulcl not-pose
a problem for the transmission of genetic information. The correct choice is D.

57. C is correct. If there is an increase in Tm, it means that it takes a higher temperature to melt (denature) the DNA.
This must gean that the DNA is stabler. What would make the DNA stabler? If there were more hydrogen bonds
between the base pairs, then the stability of the double helix would increase slightly. Since there are three hydrogen
bonds between G'C base pairs and two hydrogen bonds between A.T base pairs, we would want an increaie in the
amount of G'C base pairs and/or a decrease in the amount of A.T base pairs. Both of these factors lead to an
increased stability of the DNA duplex. We can eliminate choices A and D.

What about the divalent Mg2+ ion? DNA is quite negatively charged. If these negative charges were not shielcled
from one another, they would tend to blow the duplex apart (i.e., make it less stable). The result is a decrease in the
Tm. However, if there is a higher concentration of Mg2+ ions in the medium, there is also a greater chance of these
magnesium ions associating with the negatively charged phosphate groups and shielding those negative charges
from one another. The DNA double helix becomes slightly stabler. We can eliminate choice B.
Histones are proteins that bind to DNA. These proteins have a large proportion of the amino acid residues arginine
(Arg) and lysine (Lys). Both of these amino acids have side chains that are positively charged at physiologi.it pH.
These positively charged residues ionically bind DNA's negatively charged phosphates and heip stabilize the
molecule. Histones can be dissociated from their interaction with DNA by using a sufficiently concentrated salt
solution that interferes with these ionic interactions. The DNA becomes slightly less stable (due to the deshiel<ling of
the negatively charged phosphates) and easier to melt. The correct choice is C.

Copyright @ by The Berkeley Review 371 The Berkeley Review


Specializing in MCAT Preparation
Biology Genetic Information Section IX Answers

58. A is correct. The question is based on a modest understanding of the Meselson-Stahl experiment performecl in 1957.
Recall that Meselson and Stahl were able-to prove that DNA replicated semiconservatively. They grew the bacteria
E. coli in a growth medium containing 15NH+CI for many generations. At a particulur *o*.ni in time, they then
transferred the bacteria to a growth medium containing laNH+Cl. At the time of transfer, they analyzed the bacterial
DNA and found it to contain the heavy isotop_e of nitrogen, l5N. After one generation, they founcl that the DNA in
one strand of the double helix contained l5N, while the DNA in the other strand containe<l laN. After two
generations, the DNA was diluted even more with laN.

We would therefore expect lo fild two peaks in the graph. One peak would correspond to two duplexes of DNA,
both of which were hybrid 155- lay DNA. The other peak would Corresponcl to two d-uplexes of DNA, both of which
-50Vo
were all l+N pNA. ln other words, each peak would correspond to of a part'icular type of DNA. We can
eliminate semiconservative DNA replication as_ a possible answer choice. Note that in semiconservative replication,
the DNA strands that contain the l5N and 14N strands become increasingly rare as the number of bacterial
generations increase.

15y 15y o layTlay 1sp71451


-- 50Va
'-TiJ,"j"X."7 5O7o

15111 la1g
[1 a, 1411 l5p
€\O
oN
cg
o
&
c*:|:n""y'/
ls1..1 la11
lIf lay 14111 l4ir1
/Wr
l4p lary 15y
Direction ol Sedimentation rrl

o"#r,." l[ [[ [[ ill
What would the replication process look like for conservative replic_ation? In conservative replication, the original
parental strands of the DNA double helix (which are labeled with 15N) serve as templates for new daughter DNA.
However, after the first round of replication, we find that the parental strands recombine. Two duplexes result. One
is completely labeled with lsN, and the other is completely labeled with laN.

This type ofreplication continues_through each succeeding generation. Note that after two generations, we have one
duplex completely labeled with l5N, and three duplexes completely labeled withlaN. The co.."rponding graph
shows two peaks. One peak represents that DNA which is completely labeled with lsN (25%). The other peak
represents that DNA which is completely labeled with laN (157o).

o
o

LF

d\O
l5y 151..1 14p 14 N 9:

llf
'd
e.
G"":::n",
lsy 1511
y'/
lay 14111 l4p l4p
/an \lay l4y Direcl ion ol' Sedimentation r)

[[[[ [[
""#1,'*'ll
What about dispersive and end-to-end DNA replication? After one generation, we see that both types of replication
followthesemiconservativemodeinthateach duplexishalfheavyll5N)andhalf light(l4N).

Copyright @ by The Berkeley Review 372 The Berkeley Review


Specializing in MCAT Preparation
Biology Genetic Information Section IX Answers

l51g 15y 15111 l5p

'-'TiJ,"ilKo',/
llX ll
DNA

rst Generatio"
II IIare
Both
it
Both are
tt
Both are
Both are
lspTlay 1s5714P 1sP7l411 1sp7141g

Dispersive End-to-end
conservative

However, if we were to denature these duplexes after the first generation, we would find that every single strand
would be composed of half heavy (lsN) and half tigtrt 1l4N; DNA. This is quite different from what we would find
if we denatured a DNA duplex after the first round of replication of either semiconservative or conservative DNA.
What do you think the graphs would look like for these two proposed types of replication? The correct choice is A.

59. D is correct. Human cells do not have chloroplasts as organelles. Therefore, there should be no protein transport
from the cytosol into the chloroplast. There is protein transport from the cytosol to the mitochondria, nucleus, and
peroxisome. A peroxisome is an organelle that carries out oxidative reactions. It has no genome and is surrounded
only by a single membrane. Human beings are not sessile organisms. Therefore, we do not need to photosynthesize
in order to obtain energy. The correct choice is D.
60. D is correct. From the passage, we learn that the mitochondrial genome is very similar to that of the bacterial
genome. Therefore, one must realize that a bacterial genome does not have its DNA packaged with histone proteins,
as is the case with human DNA. The mitochondrial genome has distinct promoters and must have both DNA
polymerase and RNA polymerase in order to replicate and transcribe its genome. Thus, the mitochondrial genome
lacks histone proteins. The correct choice is D.
61. D is correct. This problem requires the we know which amino acid is called for by a start codon. It is neither
histidine or leucine. Therefore, choices A and B can easily be eliminated. Now the question becomes, is it
methionine or N-formylmethionine, the modified amino acid used in prokaryotes? There is our big clue. The
mitochondrial genome is very similar to the prokaryotic genome. Thus, mitochondria use N-formylmethionine
instead of methionine. The nuclear genome uses methionine. The correct choice is D.

62. D is correct. If every nucleotide is used for coding purposes, this leaves little room for any regulatory
sequences in
the mitochortdrial genome. Let's consider the other possibilities. Choice A is a true statement. There will be fewer
RNA molecules coming from the mitochondrial genome. However, the reason is not because every nucleotide in the
mitochondrial genome is a coding nucleotide. The reason is simply the size of the respective genomes. Therefore,
choice A can be eliminated. Choice B is a false statement and can be eliminated. It now becomes only a matter of
discriminating between choices C and D. The correct choice is D.

63. B is correct. The question is asking why we see a different genetic code in the mitochondria. The answer lies in the
volume of proteins that are produced by the genome. Relative to the nuclear genome, there is a small number of
proteins produced by the mitochondrial genome. Thus, a change in the genetic code is not very far-reaching. In other
words, the change is tolerable because more likely than not, a small number of proteins would be affected.
Furthermore, the changes that do occur in the protein as a result of the change in genetic code may be harmless (they
may not affect function). Because of the increased number of proteins that are coded for by the nuclear genome,
there is a higha probability that a change in the genetic code would be very far-reaching, affecting a good many
proteins. Therefore, the understanding comes from thinking about the probabilities and how that relates to the size of
the genome.

Consider the other choices: There is no evidence that genetic drift occurs only in mitochondria. Furthermore, it
would be difficult to rationalize how such a force would affect only one particular genome. The reason we do not

Copyright @ by The Berkeley Review 373 The Berkeley Review


Specializing in MCAT Preparation
Biology Genetic lnformation Section IX Answers

see the drift in other genomes probably has to do with the elaborate proofreading systems that maintain their fidelity.
Therefore, we can eliminate choice A. For choice C, we have no reason to believe that the tRNAs in the
mitochondria have any special correcting function. They are bound by the same physical laws that all other
molecules abide by. In other words, we have codon-to-anticodon base-pairings, and the mitochondrial tRNAs do not
change those base-pairing rules to accommodate a different genetic code. Finally, like proteins anywhere else, the
primiry structure definitely affects the function by dictating the final shape of the molecule. Therefore, we can
eliminate choice D. The correct choice is B.

64. D is correct. The question is asking us to identify the function of the ten polyadenine-containing RNAs noted in the
passage. The polyadenine tail is the clue to this question. Recall that mRNA has a polyadenine tail. We can assume
iUr nNn ntoleCute to be 6RNA. Choice A assumes that the mRNA is translated in the cytosol, but this is
happening in the mitochondria. The mRNA that is created is translated in the mitochondria and not in the cytosol.
EliminatJchoice A. Choice B indicates that the RNA is a transfer RNA. This is not the case, because IRNA does not
have a polyadenine tail. Eliminate choice B. Choice C clearly indicates that the RNA is rRNA. Again, the
polyadenine tail indicates that the RNA is mRNA, not rRNA. The mRNA is functionally responsible for coding for
ribosomal proteins. The correct choice is D.
65. D is correct. We are told from the passage that the heavy strand of RNA is responsible for encoding many proteins.
while the light chain of RNA is over 907o nonsense. One should realize that the heavy chain of RNA is nearll'
identical to ihe strand of DNA that gave rise to the light chain of RNA. The only difference is that the thymines in
the DNA have been replaced by uracils. Since the heavy chain of RNA contains all of these functional codons, the
strand that is nearly identical to it is called the sense strand. The piece of DNA that actually gave rise to the hear'1'
strand of DNA is called the antisense strand. The correct choice is D.

66. A is correct. The question tells us that the T4 phage codes for all of the proteins necessary for its own DNA
replication. Therefore, we can approach this problem by choosing the answer that has nothing to do with DNA
replication. Transfer RNA, or tRNA, is a ribonucleic acid compound that is important during translation, rather than
DNA replication. Therefore, the phage genome need not contain instructions for making tRNA; the virus exploits
the hostiell's tRNA. This problem can also be answered by eliminating choices that actually do play a role in DNA
replication. Recall that primase is an enzyme which lays down RNA primers during DNA replication. Eliminate
choice B. Ligase seals nicks introduced on the lagging strand during DNA replication; rule out choice C. And
finally, DNA polymerase is the enzyme responsible for extending newly replicated DNA. Eliminate choice D. All of
these choices affect replication of DNA. The correct choice is A.

67. B is correct. The experiment described in the question is basically identical to the complementation experiment
detailed in the passage. In this case, however, the results are different; no plaques form, suggesting that the tu'o
coinfecting mutants could not complement each other. Recall that complementation occurs when one mutanl
provides Jfunctional protein product that the other one lacks, and vice versa. Therefore. the net effect would be a
wild-type phenotype, as all of the protein components of the wild-type pathway would be present (see Figure I in
ttre pashgit. In the case of the experiment described in the passage, however, there are no wild-type plaques formed
as tlhe reiult of coinfection of the rIIA and X1 phages. A logical conclusion to be drawn from this failure ot
complementation would be that both mutants affect the same gene. In other words, neither mutant could provide the
othei with a functional copy of the defective protein. As a result, a wild-type phenotype can't be achieved. With this
knowledge, we can eliminaie the other answer choices. Choice A is inconect; if the two mutations affected different
genes, th;y would complement each other by each providing a good copy of the protein that the other lacks. This
ivould result in wild-type plaque formation, which is not observed. By the same reasoning, we can eliminate choict
C; the two mutationJ do ,rt-"o*plement each other, which is why they are incapable of lysing E. coli K cells-
Choice D can be eliminated, because the experiment does not support this statement in any way (even though it ma3
not directly contradict i0. Recall, the question asks, "What can be concluded from this experiment?" Definitely ntrt
answer choice D! The correct choice is B.

68. B is correct. The question essentially asks us why r11 mutants of phage T4 are incapable of lysing E. coli K cell-q"
Recall that wild-type T4 is capable of lysing these cells (resulting in plaque formation). Additionally, the rll mutants
are capable of lyJing E. coli B cells. Why can't they form plaques on lawns of E. coli K? Let's approach this
problem first by eliminating unlikely answers: Statement I postulates that the mutant viral DNA sequences contaim
new restriction sites that are recognized and cleaved by bacterial endonucleases (or restriction enzymes). Recall thar
such enzymes recognize palindromic sequences and cleave them. Normally, this would be a perfectly plausible

Copyright @ by The Berkeley Review 374 The Berkeley Kevieu


Specializing in MCAT Preparatio'n
Biology Genetic Information Section IX Answers

explanation for why r11 mutants can't lyse E. coli K cells. But we must remember that the question is prefaced,
"Based on information given in the passage...". From the passage we learn that when a single E. cotik cell is
coinfected with both mutants at once, the result is lysis of the bacterial cell and the formation ofi plaque. Could this
occur if both mutants' DNA were chopped up by restriction enzymes? Most likely not, so we must eliminate
statement I. Statement III hypothesizes that since E coliK cells lack specific receptors for mutant phage T4, the
phages could never bind or inject their DNA in the first place (recall the sequence of viral infection: adJorption to
the cell surface, injection of DNA, etc.). Could this be the case, considering that the successful complementation
experiment described in the passage required the injection of mutant DNA into E. coli K cells? Probably not, so
eliminate statement III. This leaves us with statement II, which states that r11 mutants lack functional gene products
(i.e., enzymes) necessary for the lysis of E. coli K cells. Referring to Figure I in the passage, we see i pathway for
virally-induced cell lysis, which directly supports statement II's hypothesis. Statement II is our winner. ihe coirect
choice is B.

69. D is correct. This question basically proposes an experiment similar to the one described in the passage. In this case,
however, we are using a diploid organism, Drosophila melanogaster (the fruit fly). The question states that mutants
A and B are each recessive, meaning that they normally express their curly wing phenotype only when they are
homozygous (i.e., A/A or B/B). However, transheterozygous flies were made by crossing the two mutant lines to
each other, resulting in some progeny which were A/8. Each individual mutant is still heterozygous, but for some
reason A/B flies have the curly wing phenotype. What is going on? It turns out that thii experiment is a
complementation test. We are trying to learn whether mutant A and mutant B affect the same gene. Hypothetically, if
they affected different genes (as in Figure I from the passage), each would provide a functi,onal profein which-the
other lacks, resulting in completion of a wild-type pathway leading to normal wing formation. On the other hand, if
the two mutants both affected the same gene, flies that are AiB would lack a furctional protein encoded by the
mutually-affected gene. Therefore, wild-type wing formation would not occur and a curly wing phenotype *ouid b"
observed. This is the case in the experiment described in the question. Therefore, mutanis A ind B fail to
complement each other and must therefore affect the same gene. The correct choice is D.
70. C is correct. The experiment described in the question is basically testing whether each of the T4 mutants can revert
to the wild type and form plaques on a lawn of E. coli K cells. Recall that normally, rllA and, rIIB can'tform plaques
(i.e., lyse cells) on this bacterial strain. The question informs us that r11A forms one single revertant ptuqu.,
however. The mutation that prevents rlIA from lysing E. coli K cells has spontaneously revertid. For example, if the
mutation were caused by a single base-pair substitution, another spontaneous mutation that switches the suLstitution
back to the wild type would cause a reversion of the mutant phenotype (i.e., plaques could now form). This is most
likely what has happened to rIIA. What about r11B? No revertant plaques form at all. We can infer from this that the
type of mutation that causes the rIIB defect is difficult or impossible to revert. The bottom line is that this question
tests your understanding of the different types of mutations that can occur in DNA. Let's go over the ones mentioned
in the answer choices: A point mutation occurs when a single base pair is altered (i.e., G is changed to A, etc.). point
mutations revert relatively easily, because it only takes another mutation in that base pair to switch it back to the
wild type. Since r11A is capable of reverting, we can fairly assume thatIIIA is caused by a point mutation. A deletion
mutation occurs when a segment of DNA is removed. Deletions hardly ever revert, mainly because it is nearly
impossible to?eplace a segment of DNA spontaneously (eliminate choices A and B). Since r11B doesn't revert, wL
can fairly assume that it may be caused by a deletion mutation. Therefore, choice C is the correct answer. Frameshift
mutations result from deletion or insertion of base pairs, resulting in the altering of the normal reading frame of a
t !eno. Frameshift mutations are difficult or impossible to revert (eliminate choice D). The correct choice is C.
71. C is correct. Answering this question requires a solid.understanding of what complementation is all about.
Complementation in diploid organisms occurs when two mutants which affect different genes each provide a
functional protein which the other lacks. A dominant mutation is one which shows a mutant phenotype even when
heterozygous, or present in one copy. In other words, there is one wild-type chromosome present in organisms that
are heterozygous for a dominant mutation. This wild-type chromosome, although providing a full complement of
wild-type proteins, does not complement the dominant mutation; in other words, the mutant phenotype is still
expressed. This problem can be approached by eliminating answers that are incorrect. For example, choices A and
B are both recessive mutations. Recessive mutations don't express their phenotype when they are heterozygous,
because the homologous wild-type chromosome provides enough functional protein to make up for that *irictt ls
defective in the mutant chromosome. In other words, the wild-type chromosome complements t6e defect present in
the mutant chromosome. We can eliminate choices A and B. Choice D can likewise be ruled out; a deletion mutation
would most probably be a recessive mutation, because the wild-type chromosome could provide a normal protein
product. The correct choice is C.

Copyright @ by The Berkeley Review 5/b The Berkeley Review


Specializing in MCAT Preparation
Biology Genetic lnformation Section IX Answers

72. A is correct. The key to this question is knowing our terms. An obligate parasite is an organism that must rely on
other organisms solely to survive. A virus can't reproduce without a host cell; therefore, it is an obligate parasite.
This BEST describes the T4 bacteriophage. Let's eliminate the other answer choices: An obligate heterotroph is an
organism that must feed on others to survive. More specifically, it must consume complex carbon molecules.
Animals are obligate heterotrophs, while plants are autotrophs; they can synthesis their own complex carbon
molecules (i.e., carbohydrates, proteins, etc.). Therefore, we can eliminate choice B, because the virus does not feed
on anything directly. It simply uses its host to reproduce. Prototrophic organisms, such as certain bacteria, can
survive on a minimal media, while auxotrophs need supplementation of certain nutrients to survive. These are terms
most often used to describe bacteria, and therefore they wouldn't apply to bacteriophageT4. Eliminate choices C
and D. The correct choice is A.

73. B is correct. The homozygous mutant animals were hypertensive on the standard chow diet. Eliminate choice A.
The response of the heterozygous animals differed from that of the wild-type animals at the 8Vo dietary sodium
level. Eliminate choice C. Blood pressure increases in the heterozygous animals were directly proportional to dietary
sodium. Choice D is incorrect. The correct choice is B.
74. D is correct. The Lowry assay is for protein concentration, and it cannot discriminate among proteins. Choice A is
incorrect. To digest the peptides in a tissue would create a bunch of amino acids, but no real information on ANP as
an intact molecule. Choice B is incorrect. Southern blotting is for DNA fragments, not proteins. Radioimmunoassay
uses antibodies to the compound of interest to separate it from a mix of compounds. Also, it is sensitive to at very
low levels (picograms). The correct choice is D.
75. A is correct. They do not have any detectable ANP, so the action of ANP to direct fluid out of the blood is not
available. The intravascular fluid level tends to be higher in these animals, as shown in the hematocrit. The
hematocrit measures the Vo of the blood that is RBCs. A lower hematocrit indicates more fluid in the blood relative
to the RBCs. Choice B is incorrect. ANP does not directly affect the RBCs, and this is not srared in the passage.
Choices C and D are incorrect. The correct choice is A.
76. A is correct. The pro-ANP is stored in granules, cleaved by a specific protease, and then released from the cell as
ANP. A person missing the enzyme would not secrete ANP. The question is whether pro-ANP is released intact.
Compare this to what you know about glycogen. Glycogen is made of glucose polymers stored in granules, but only
the subunit, glucose, is released from the cell. It is unlikely that the larger pro-ANP precursor is released directly.
Also, there are no proteases in the bloodstream to activate the pro-ANP, since that would be disastrous to cells.
Choice B is incorrect. Choice D is incorrect. The correct choice is A.
77. D is correct. Instead of genetically altering animals, other approaches are to somehow remove ANP from regular
wild-type animals. These are all theoretical answers. If all the ANP is bound by antibody in the blood, then the effect
is that no ANP is present. Statement I is correct. If the receptor is bound by something else and cannot interact with
ANP, then the effect is the same. Statement II is correct. If pro-ANP were bound and not processed to ANP, the
effect is the same. Statement III is correct. The correct choice is D.
78. A is correct. The atria would be stretched in the case of too much pressure. ANP lowers fluid volume in the blood
and promotes sodium excretion (and water follows sodium). The lowering of fluid volume promotes a lower blood
volume and lower pressure. No pressures are raised, therefore choices B and C are incorrect. Sodium output in the
urine is increased by ANP ("natriuretic" means promotes sodium in the urine). Choice D is incorrect. The correct
choice is A.
79. C is correct. Use a Punnett square to figure this out. We cross AA x Aa. AA means having 2 good copies of the
ANP gene, and Aa is the heterozygote. The offspring will be 5OVo AA and 507o Aa. The Aa are salt-sensitive, just
like the Aa parent. The correct choice is C.

80. D is correit. We must come up with this answer based on previous knowledge, and not something stated in the
passage. The gram stain takes advantage of differences in the cell walls of bacteria. The cell wall of a Gram-positive
bacteria contains peptidoglycan, assorted polysaccharides, and teichoic acids. The cell wall of Gram-negative
bacteria contains peptidoglycan, phospholipids, lipopolysaccharides, and assorted proteins. The cell wall of a Gram-
positive cell does not have an outer membrane, and the peptidoglycan layer is very thick when compared to the layer
in the Gram-negative bacterium. The correct choice is D.

Copyright @ by The Berkeley Review 376 The Berkeley Review


Specializing in MCAT Preparation
Biology Genetic lnformation Section IX Answers

81. C is correct. This is not the case, and can be determined from one of Griffith's experiments. Let's look at the
injection of a heat-killed, pathogenic bacteria. The mice lived. If the polysaccharide itself was the cause of death,
then this would not have been the case. The mice would have died. The death must result from something a live
bacterium can produce; and furthermore, the polysaccharide coat must serve another function besides causing death.
The correct choice is C.

82. B is correct. Recall that proteins are very sensitive to temperature. When the temperature becomes too high or too
low, the protein shape is altered. That protein shape is the key to protein function, and helps a protein carry out a
cell's everyday function. If the proteins are non-functional, then nothing can continue. There can be no synthesis or
metabolic processes. If no processes can be carried out (even the replication of nucleic acids), the cell will not be
able to produce energy and it will die. The correct choice is B.

83. D is correct. This question is very straightforward. Avery's conclusion was that the transforming principle was
DNA. Recall that deoxyribonuclease destroys DNA. If Avery's conclusion held true, the addition of
deoxyribonuclease should eliminate transforming activity. We are looking for a statement that does not support
Avery's conclusion. The correct choice is D.

84. B is correct. This question is straightforward. We are told from the passage that the bacterium is a facultative
anaerobe. This means that the bacteria can function either in the presence or absence of oxygen. An obligate
anaerobe cannot function in the presence ofoxygen, while aerobes can function only in the presence ofoxygen. the
correct choice is B.
85. B is correct. Remember that ribosomes are considered to be organelles. They are just not membrane-bound
organelles. The bacterium certainly requires the presence of ribosomes to carry out the translation of mRNA to make
protein. The bacterium does contain organelles. The correct choice is B.
86. D is correct. We are looking to use two elements that will enable us to discriminate between proteins and nucleic
acids. This question is based on the experiments of Hershey and Chase. The backbone of nucleic acids contains
phosphate groups. Nucleic acids do not contain sulfur. Proteins contain sulfur, but do not contain phosphorous.
Therefore, phosphorus and sulfur are the two elements we can use that will enable us to discriminate between
nucleic acids and proteins. The correct choice D.

87. D is correct. This question comes in two parts. The first requires one to think whether a cell taken up by another cell
is pinocytosis or phagocytosis. The answer is phagocytosis. Recall that solid particles endocytosed by a cell is
referred to as phagocytosis, while endocytosis of dissolved particles is termed pinocytosis. The bacterium can be
classified as a solid particle, enabling one to eliminate two of the answers. Next, is phagocytosis carried out by B-
cells or macrophage? p-cells produce antibodies, while macrophage are considered the body's professional
phagocytes. The correct choice is D.

88. B is correct. Since we are assuming that eyeless is on the X chromosome (X-linked), we can infer that the
genotypes usedin the cross are: ey/+ females x +,/Y males. Fruit flies have the same sex chromosome setup as
humans (two X chromosomes for females and an X and Y for males). The real trick to this question is remembering
from the passage that flies heterozygous for ey have small eyes, while those homozygous for it have no eyes. After
that, it's just a matter for a Punnett square. The correct choice is B.
89. B is correct. As you recall, a promoter is an untranscribed sequence of DNA upstream (towards the 5' end) of the
DNA sequence that is actually transcribed (and that codes for the gene's protein product). The relative "strength" of
a promoter (i.e., the degree to which it promotes transcription) is dependent on the promoter's sequence. In
Experiment I, GAL4 has a very weak promoter. The is the reason that GAL4 is not normally transcribed or
expressed. The wing-specific enhancer can "enhance" transcription of GAL4 only in wing tissue, overcoming the
weakness of the promoter. In this question, this system has failed and GAL4 is transcribed everywhere, thereby
activating the UAS-controlled eyeless gene everywhere. This is why the in situ hybridization experiment showed
thatey mRNA wqs present throughout the embryo when we should expect it to be present only in certain cells. A
constitutive (meaning "always active") mutation in the GAL4 weak promoter could have made it "stronger," thereby
increasing transcription. Answer choice A is wrong, because a mutation in the UAS site could only prevent GAL4
from binding, thereby reducing ey expression. Answer choice C is wrong, because the normal genomic copy of ey is
expressed only in cells that will become eyes (from the passage). Answer choice D is wrong, because an amber
mutation (a stop codon) would cause an abnormally short ey protein but would not affect where it is transcribed.
The correct choice is B.

Copyright @ by The Berkeley Review cl I The Berkeley Review


Specializing in MCAT Preparation
Biology Genetic Information Section IX Answers

90. C is correct. Statement I is an unsupported statement, because from the passage we know that while the ey gene
may be necessary, it is not sfficient for normal eye development. We know this because in heterozygotes, there is
one good copy of the gene, but the phenotype is still abnormal (small eyes). Moreover, no evidence in the passage
ruppo.tr theiiaim tharey is sufficient for normal eye development (there are in fact hundreds ofother genes that are
n.i-"srury to make an eye). We can infer only that ey is a developmental "master switch," possibly acting to turn on
other genes needed for eye formation. From the passage, we see that ey is a transcription factor, which serves to
back this claim. Transcription factors bind to DNA and promote transcription of certain genes. This means statement
II can be supported by the passage. Statement III says that eyes arose independently in insects and mammals. This
means they ivolved r"pu.uGly from different ancestors and have nothing in common evolutionarily. The results of
Experiment 2 as well is the homology between Drosophila eyeless and mouse small eyes shows this to be false.
This evidence suggests that eyes evolved from a common ancestor. The correct choice is C.

9t. D is correct. The mutant eyeless allele is a recessive hypomorph, meaning that its function is reduced, or weaker. A
point mutation in a nonconserved region of the sequence would probably not have much effect on protein function,
t".uur" nonconserved sequences vary throughout evolution without altering protein function to any great degree. A
two base-pair deletion near the beginning of the sequence would cause a frameshift mutation that would completely
wipe out protein function, and this can't be the case. An inversion would cause the same effect, so that leaves choice
D.'A muiation in a sequence of eyeless that has been highly conserved throughout evolution would cause a reduction
in the function of the protein. These sequences are conserved, because they are crucial to the proper functioning of
the protein. The correct choice is D.

92. B is correct. Ectopic means "not in the normal place." Since Experiment 2 showed that the mouse small eyes gene
made normal fruit fly eyes in Drosophila, a finding that the reciprocal experiment failed to make normal mice eyes
-Such
would be inconsistent. a finding is shown in choice B. Answer choices A and D both confirm that eyeless is
evolutionarily conserved, which agreis with the results of Experiment 2. Answer choice C confirms that eyeless is
highly homoiogous to small eye,s, a case that supports the results of Experiment 2. The correct choice is B.

93. A is correct. In order for a population to be in Hardy-Weinberg equilibrium, answer choices B, C' and D must not
be true, while choice A musi be true. This is a memorization question; but if you remember that at equilibrium
there
pool, you could come up with the right answer' The
is no net change in the frequency of alleles in the gene then
correct choice is A.

94. C is correct. This type of question was given o^n a previous MCAT. You must know how to use the Hardl-
Weinberg equarion, *ti.h ,tut., thar p2 + ipq + g2 = l.In this equation, p is the frequency of the wild-type allele-
while q ii thi frequency of the recessive alleie in the population. The questions asks for the number of flies that
\\'ill
have small eyes, and irom the passage, we know that this means those heterozygous for eyeless. The 2pq term
represents heierozygotes. Thus, Zpq=Z1O.S1(0. 1) = 0.18. This is the frequency of heterozygotes in the
population
ilh"n *" multiplythis by the populition of 1000, we come up with 180 flies. The correct choice is C.

C is correct. The karyotype gives a picture of the chromosomes, which means that the sex of the fetus
is
95.
immediately discernibli. itoi.e A is thus not the best answer. The incidence of extra or missing chromosomes-m
defects may or may not be
also immediately visible, so choices B and D are poor selections as well. Developmental
false, making it the best answer' The
due to geneti. .rtorr. Some have environmental causes. Choice C is therefore
correct choice is C.
C is correct. The second decade of one's life spans the years fiom 10- 19. we finish the
first decade when we turnalx
96.
Down syndrome ir lowest for women in the years from 15-19' or the later porti;m'
ten. The chance of bearing a
"ilta in both 13-1'ear-
of the second decade, so itatement I is correct. Reading from the graph, we see that the incidence
iI is too. Menarche is the beginnine cf
olds and 35-year-olds is about 0.9 per 1000 births, sJsmtement "orre.t, of Down births is higher in \r'o{TEa
menstruation, while menopause is the conclusion of menstruation. The incidence
near melopause (age 40-50+) than in women close to menarche (9-15+). Statement
III is therefore inconect. 11:
correct ihoice is C.

97. A is correct. Down syndrome is also called trisomy 21. Tri means "three," somy refers to body
(in this case" fu
bodies of the chromosomes), and 2 I refers to chromosome 21. Choices B and D, which
refer to chromosome I 8' m
both incorrect. Also, choice C (monosomy 2l) is incorrect. The correct choice is A'

by The BerkeleY Review 374 The Berkeley Revirr


Copyright @
Specializing in MCAT
Biology Genetic Information Section IX Answers

98' B is correct. The presence of the Y chromosome means the person is a genotypic male, so
choice A is incorrect.
This question is about th,e genotype' not the phenotype. Choices C andb are'thus incorrect,
since they refer to
phenotype. The correct choice is B.

99' D is correct. Table I provides us with the_answer: Monosomy X is the highest, at 20/100. There is no trisomy X
listed in the table, so choice C is incorrect. Both trisomy 21 and trisomy 18 Jccur less frequently
than monosomy X,
so choices A and B are incorrect. The correct choice isD.

100' B is correct' RFLPS can be used for this kind of work because restriction enzymes cleave different
DNA strands
into fragments of different lengths, based on the location of specific restriction sites. The child
has ttr"" of
chromosome 21. One clearly came from the father and two from the mother. The mother "opi"s
is the source of the
nondisjunction in this instance. Choices A, C, and D are incorrect. The correct choice is B.

Copyright @ by The Berkeley Review 379 The Berkeley Review


Specializing in MCAT preparation
sgction'
rcr6$y
x A.:,, I;INA Pfoc€ssing ,

'[. RhJA,Polym€rase & Promoters


,', 2,. "RNA$linthesis &Modifiiationsl
, 1 ', f;; ' ProteinSynthesis '
Expr€$$idn, rOf ' 4.. The Lactose Operon ' .

Genetic 5.,,
llutations
The Tryptophan Operon

Information 8,,,,
l. Mutations & Ploofreading
lst ' .i. .2rd.Position . . r'
, 2. ' The Ames Test
'
UCA C.,Genetic: Engineering
:

;3rd,
Psiqltlon
{5'.End} G {3l.rEndl
,, l. Cenetic Enginqerin$,lsequencing,
r
Plrq i
.' Ser: . Tyr . . -: Cys
Phe ..Ser. 'tyr
, U
C
..&Cloning..
U
.rCys..
Leu , Ser, STOP ' ST&p A
Leu .. : .Ser,,.,, ..S.TOP. r ..:,...TrP ., G Practice Passages & Ansryers
Leu Pro His fu'g U
Leu Pro' His r 'A{g C

C Leu
Leu
Pro
Pro
Cln
Cln
Arg
Arg
A
G

Ile Thi A,!n Ssr U

A Ile
IIe
Mff
Thr
Thr
Thr
Asn
Lvq
Lys
Ser
Arg
Arg
C
A
G

val , Ala r A$F, Gly: U

G Val * Ala
val
Val
Ala
Ala
Asp
Glu . ,
Clu
Cll
Gly
Gly
C
A
G

Specialiiing in MCAT Preparation


Expression of Genetic Information
Top 1O Section Goals

Understand the differences between RNA & DNA polvmerase.


o? RNA polymerase is used to transcribe DNA inio RNA language. DNA polymerase replicates DNA
before cellular division. Understand these functions and how they apply to the cell.

Be familiar with the concept of a transcription promoter.


RNA polvmerase does not beqin transcription just anyplace on the DNA helix. There are specific
-Understand
addresse's that fcl/ the polyderase where to bind. the basics of these funitions.

Iftrow the differences between eukaryotic & prokarJrotic BNA Processing.


Eukaryotic RNA must undergo extensive processing in the nucleus before it can be used in the
cytoscil for protein synthesG. Prokaryotic RNA ii used immediately and is not processed.

Be able to read the genetic code.


@? DNA codons on ihe coding strand are the same as the mRNA codons. DNA codons on the template
strand are the same as theinticodons on IRNA (except for replacement of Us and Ts)

tlave a feel for transcription & translation in eukaryotes &- prokaryotes.


In prokaryotes, transcription and translation are tightly coupled. Il eukaryotes, transcription and
translation are separat-ed both in space and in time. Understand this important distinction.

t? Be familiar with the actual process of transcription.


You should have a firm grasp of IRNA activation and stibsequent binding of the activated IRNA.
to the P-site and A-sitd on the ribosome. Understand translocation-and protein synthesis.

Understand the lactose operon and how regulation is achieved.


The lactose operon ties in all of these aspects of transcription and translation disc-ussed so_far. It
is an exceilent example of how a bactbrial cell regulafes its need for a particular metabolite.
W

Understand the basic tools involved in genetic engineering-


-:
Probably the most important genetic engineering to_ols are the restriction endonucleases. Do.nc:
memo.ire their recdgnitiorisequencds. Instead, know the basics of how they function Jltl

flt
al
Be able to read autoradiographs from polyacrylamide gels. fl'
Understand the process of gel eiectrophoresis_and why it-is-that bands are read f1om. the bottom oi pr
the gel to the top of the gellBe famili'ar with the Sang6r dideoxy sequencing method. um

'a- rmd
r-'

Be familiar with tfte pto."tt of .lo"it T


This is ihe core of genetic engineering. Understand how to remove a seqment of forergn DNA, ope: m
]fi
up a plasmid, inse"rt that forEign DN"A into the plasmid, and then express the gene oFinterest. ,iun

fmr

ffm
Biology Expression of Genetic Information RNA Polymerase & Promoters

||Fol# a$Hliffi1 o't-brs


RNA Polymerase
The enzyme RNA polymerase transcribes DNA into RNA. Let's compare the
differences between replication and DNA-directed DNA polymerase with
transcription and DNA-directed RNA polymerase.

'1,. Both polymerases proceed in the 5' -+ 3' direction.


2. DNA polymerase uses dNTP's (releasing PP1) while RNA polymerase
uses NTP's (also releasing PPi).
3. Both polymerases require a DNA bubble for operation.
4. With DNA polymerase we find that DNA replication is semiconservative
while in the case of RNA polymerase the two DNA strands will
eventually rewind, thus conserving the process.
5. DNA polymerase requires a primer with a free 3'-OH group to initiate
chain elongation. RNA polymerase does not require a primer.
6. The DNA directed DNA polymerase is almost always involved in
synthesizing DNA bidirectionally. Both DNA strands are being copied.
In the case of DNA directed RNA polymerase only one strand of the
DNA is being copied.

one way to demonstrate that RNA polymerase does not need a primer is to use a
labeled substrate such as ATP labeled at the B andyphosphates (Figure 10-1).

ooo Adenine
ll* ll* ll
-o-P-o-P-O-P-O-CFI?
trt\
o_o_o4' I'
3'2'T H

Ho oH
Adenosine triphosphate
(A ribonucleoqile triphosphate)

Figure lO-l

Are any of the p or yphosphates incorporated into the nucleic acid? If we try this
using dATP and DNA polymerase, the answer would be no. Why? Because we
are starting with a primer and when we add dATP to the free 3'-OH group, the p
and y phosphates of dATP are lost in the reaction mechanism (as pyro-
phosphate). However, with RNA polymerase we do get some p and y labeled
phosphates in the RNA polymer. This is because the first nucleotide (ATP)
incorporated into the growing RNA polymer has at its 5'end a labeled B and y
phosphate. r.

The time course of incorporation of the B-y labels and also of cr labels in a
growing RNA polymer can be measured. It takes about 1 or 2 minutes to make
an average mRNA molecule in a test-tube using RNA polymerase and the DNA
from a phage such as T2 or T 4. If we measure the incorporation of the p-y label,

Copyright @ by The Berkeley Review 343 The Berkeley Review


Specializing in MCAT Preparation
Biology Expression of Genetic Information BNA Polymerase & promoters

we will find that within a few seconds that label is in the RNA polymer and. then
no more is incorporated. However, if we measure the incorporation of the a
label, we will find that the time course is more even up to about two minutes
when RNA synthesis stops. This can be seen in Figure ro-2. rnis graph tells us
that the p-y goes in first while the a label goes in second.

o
Promoters
rn E. coli there is only one RNA polymerase and this RNA polymerase has to
make transfer RNA linrvA;, ribosornal RNA (rRNA), and miny different types
N
of messenger RNA (mRNA). It prefers to start transcription at a particular siie c,n
the DNA duplex. what tells an RNA polymerase whele to staritranscription? I:
0 Time (minutes)
turns out that there are very specific starting sites for transcription *hi.h utt
Figure to.2 different in both prokaryotic and eukaryotic organisms. consider the doub;e
stranded DNA in Figure 10-3.

Upstream Downstream
Start Site
-35 -10 +1
5' 3'
Coding Stiand
Duplex
TTGACA TATAAT
Template Strand DNA
3 5'

mRNA

Figure IO.3

Note the 5'and 3' ends of the duplex. Let's suppose that this small segment cd
DNA is about a thousand or so base pairs and that we wish to copy the lowm
strand. The direction of RNA synthesis will be in the 5'-+ 3' dlirection as
previously mentioned. The very first base pair of the transcription process rr
referred to as the +1 base pair. Immediately to the left (upstream) of +i we hane
the -1 base pair, -2 base pafu, -3 base pair, etc. Immediatery to the rigfit
(downstream) of +1 we have the +2 base pair, +3 base pair, etc. There is no base
designated as "0".

The signals that tell the RNA polymerase where to begin transcription a_re
upstream from the +1 site. In a prokaryotic organism these areas are usua_Ilr
centered around the -10 region (TATAAT) and the -35 region (TTGACA). The
-10 region is referred to as the Pribnow box while the -35 region is simply the -3F
region. As we will see, there is some flexibility in these numbers from gene !o
gene or organism to organism. Therefore, we can speak of these regions as be,rng
consensus sequences. As the RNA polymerase moves along the DNA dupler, d
is looking for the proper signals that will tell it where to promote transcripticm-
Hence, these regions are called promoter sites.

what are the consensus sequences for eukaryotic organisms? In the case d'
eukaryotic organisms the promoter areas are centered around a -25 regi,m
(TATA box) and a -75 region (CAAT box), upstream from the +1 site. This m
shown in Figure 10-4. The -25 region is referred to as the Hogness box. The
CAAT box may or may not sometimes be present. It might even be moved
further upstream.

Copyright O by The Berkeley Review 3A4 The Berkeley Revien {


Specializing in MCAT Preparatfun
Biology Expression of Genetic Information RNA Potymerase & promoters

Upstream Downstream
Staft Site
-75 -25 +1
5' 3'
Coding Strand
GGNCAATCT TATA Duplex
Template Strand DNA
3 5'

RNA

Figure lO-4

There are also signals in the DNA that tell the RNA polymerase when to stop
transcription. These occur in specific areas downstream from the +1 site.

Elongation of the RNA chain occurs by a nucleophilic attack of the 3'-oH group
at the end of the growing RNA chain with the alpha phosphate of the neit
incoming ribonucleotide triphosphate. A phosphodiester bond is formed. This
reaction is similar to that of DNA svnthesis.

DNA Template
I

v
3'-GCGI TCGGGCGI GATTACTI CGCCCGA
IAAAAAAACTTGTTT-s'
s'-CCC 4GCCCGC CTAATGA TGCGGGCT TTTTTTTGAACAAA_3'
I

k- GC-Rich region
---*l* AT-Rich region
I
5'-CCCAGCCCGCCUAAUGAGCGGGCUUUUUUUU-OH-3'
RNA Transcript
CA
C-G
Figure lO.5 G.C
C.G
C-G
The RNA polymerase will proceed down the DNA template until it runs into the
C-G
terminator sequences. These stop signals are composed of a GC-rich region
G-C
followed by an AT-rich region on the DNA template (Figure 10-5). 5'-CCCA UUUUUUUU-OH.3'
Termination Hairpin
rn E. coli the terminator sequence for the end of protein synthesis is a based-
paired hairpin sequence on the newly synthesized RNA strand (Figure 10-6). Figure lO.6

The hairpin structure is formed by the complementary base-pairing of a region of


the RNA sequence that is rich in the bases G and C. Following this hairpin
structure is a sequence of four or more u residues (referred to as the poly-U tail).
Once this hairpin pairing occurs in the RNA molecule the RNA polymerase
pauses (stalls). The polyribonucleotide uracil and polydeoxyribonucleotide
adenine that are still annealed to each other (the RNA-DNA hybrid) are rather
unstable. The result is that the RNA chain will dissociate from RNA polymerase
and the DNA dupleN This process is referred to as rho-independent termination.

The second way in which termination can be accomplished involves the rho (p)
protein and is referred to as rho-dependent termination. The rho complex is a
hexameric protein consisting of 46 kd subunits and has an ATPase activity that
allows it to specifically bind newly synthesized single-stranded RNA and pull
Copyright @ by The Berkeley Review 345 The Berkeley Review
Specializing in MCAT Preparation
Biology Expression of Genetic Information BNA Polymerase 6( promoters

itself
towards the replication bubble where it wilt dislodge RNA polymerase
from the DNA template. The end result is a fully transcribed"transcript
of RNA, a
free rho protein, and a Jree RNA polymerase enzyme. In higher
organisms it is
not entirely clear what the termination signals are.

The overall process which we have just described happens at a variety


of places
on the DNA duplex and the result is that a lot of specific RNA molecules are
synthesized. The three phases of RNA synthesis are called initintion, elongation,
and terminstion.

Copyright @ by The Berkeley Review 386 The Berkeley Review


Specializing in MCAT preparation
Biology Expression of Genetic Information RNA Synthesis & Modifications

l$. tffiCi'lffi .iffidiffidAtf'UHS


RNA, like DNA, is synthesized in the 5' -+ 3' direction. one immediate difference
between DNA synthesis and RNA synthesis is that the enzyme RNA polymerase
incorporates ribonucleoside triphosphates into the growing RNA chain. In DNA
synthesis deoxyribonucleotide triphosphates were used. Another difference is
that RNA polymerase does not need a primer in order to start transcription of
the RNA polymer.

Y-1:" the RNA polymer is synthesized during transcription, only one of the
DNA template strands is utilized. This is shown in Figure 10-7. As RNA
polymerase moves along the DNA duplex it unwinds about i 17 base pair section
of the DNA duplex and reads the template strand in order to synthesiie the RNA
transcript. Local unwinding of the DNA duplex occurs ahead of the polymerase
while local rewinding takes place to the rear of the polymerase. Note that
towards the 3' end of the RNA transcript we have an RNA-DNA hybrid.

\
l-/
5',

Figure lO-7

Types of RNA
Recall that there are three major classes of RNA. They are messenger RNA
(mRNA), transfer RNA (IRNA), and ribosomar RNA (rRNA). In the case of
prokaryotic bacterial cells there is one type of RNA polymerase that is able to
transcribe all three of these types of RNA. How"rr"t, i.r-the case of eukaryotic
cells there are three separate types of RNA polymerases. RNA polymerase i will
transcribe rRNA, RNA polymerase II will transcribe mRNA, and RNA
polymerase III will transcribe tRNA.

Iranscription Promoter Sites


lvithin the DNA molecule are regions called promoter sites that bind RNA
polymerase. These promoter sites determine where transcription is to begin. In
bacteria such as E. coli there are two sequences on the DNA template which are
rmportant to the RNA polymerase, the Pribnow box and the - 35 region. The
Pribnow box is located upstream (towards the 5' end) by 10 nucleotides (i.e., at
-10) from the regiory of the DNA template where the first nucleotide is
lranscribed (denoted as +1). The -35 region is located upstream by 35 nucleotides
i.e., at -35). RNA polymerase will move along the DNA template and
:ranscription of one strand will take place until a signal to stop (a terminator
region) is reached. This can be seen in Figure 10-8.

Copyright @ by The Berkeley Review 347 The Berkeley Review


Specializing in MCAT preparation
BioIo sv Expression of Genetic Information RNA Synthesis & Modifications

Promoter
+l
r0

TTGACA TATAAT Gene DNA

-35 Region Pribnow


Box
DNA .-/--.,-/ RNA
Temolate ^--
Start of End of
RNA RNA

Figure lO-B

Note that the -10 region and the -35 regions of the promoter are not palindrome.
They are not like the CATC box that we will discuss when we look at t:.=
mismatch repair enzyme system or even like the EcoRI sites that we will discu-"=
when we look at restriction enzymes. If the promoter region were palindrorn .
RNA polymerase would not know what to do.
Promoter Strengths Vary
The promoters for eukaryotic rRNA genes are simply not recognized by R\ d
polymerase II or RNA polymerase III. Within a class of genes the promoi.:
strengths can also vary. Consider the gene for a glycolytic enzyme. Since enei:-,,
metabolism is a major activity of the cell, these enzymes need to be transcribe; ::
a high rate. Thus, the enzymes for glycolysis are always quite prominent in ce_..
For example, there might be in excess of 100 transcripts per cell for any of i:*
glycolytic enzymes. If you were to assume that these transcripts were degraJe-
at the same rate as their synthesis, then there must be a lot of transcriptior. :r
glycolytic genes. \A/hat this means is that glycolytic genes have strong promoters

In contrast, consider the coenzymes that we have been discussing. Coenzr-r:.s


are present in cells in catalytic amounts. In other words, the enzymes for ::r*
coenzyme biosynthetic pathways are present in very small amounts. You mi*'L:
only need one copy of an enzyme molecule per cell for each step in the pathr,, =,
Thus, the number of transcripts in the cell might be on the order of one. \\hr
this means is that the genes in the coenzyme biosynthetic pathway have weall
promoters.

This is telling us that you will not find the same promoter sequence in front of a-
genes. The -10 region (TATAAT) and the-35 region (TTGACA) of the promote:
sequence are simply two common motifs which are consensus sequences base;
on the analyses of literally hundreds of promoter regions.

Viral Promoters
The promoter region in front of the gene that codes for the RNA polymerase i'i
the T7 viral bacteriophage is very strong. Once this phage's DNA is injected int:
the host cell, its strong promoter region attracts the RNA polymerase of the hcs;
cell and forces it to transcribe the viral RNA polymerase. Once the viral RN-{
polymerase is synthesized it will recognize only its own genes and not the genc:
of the bacterial host cell.

Copyright @ by The Berkeley Review 3aa The Berkeley Revieu


Specializing in MCAT Preparation
Biology Expression of Genetic Information RNA Synthesis & Modifications

BNA Polymerase lloloenzyrme


E. coli RNA polymerase is composed of 4 subunits, rt2,
B, B', and o. Together
these four subunits are-calred the RNA porymerase horoenzyme.
The sigira (o)
lbun{_1s responsible. for.locating the piomoter site that iniiiates transcription.
once RNA synthesis begins, the;igm; subunit wilr dissociate from ttre
nNe
polymerase holoenzyme. RNA poly-etuse, which now has three
su units
(cxz00'), is called the core enzyme. The core enzyme can
still initiate transcription
even though it has lost its sigma subunit, but it does so poorly and
anywirere.
P:"uyr: the core enzyme has rost one of its subunits lsrgm;;, it is an apoenzyme.
The holoenzyrne and the apoenzyme are represented"it, rigrrru 10-9.
Note that
we are giving directionality to this enzyme.

crrFFb 0zF0'
.. Holoenzyme
enzyme)
(A complete
Apoenzyme
(An incompleie enzyme)

Figure lO-9

sigma.factors are quite important in bacterial transcription. For example,


they
have the ability to (1) Iower the affinity of the_RNA polymerase for gur,"rul
regions in the DNA duplex by a factor of about 104, and
til tt",uy raise the t'ir,airrg
constant (affinity) for one class of promoters. Thus, sigma would be
an examplE
of a regulatory subunit. It regulites the specificity-of the RNA polymerase
enzyme.

How does RfA polyqerase find its promoter region? The RNA polymerase
will
bind to the DNA duplex and then sftde along it" looking for the -38 region
and
the -1.0 region. In this search, transient hydiogen bonJs are formed with
the
hydrogen acceptor and donor groups of base puir, in the grooves of the
DNA duplex' This one dimensionil search for the "*po."d
promoter site is much faster
than a three dimensional search in which the RNA polymerase would have
to
repeatedly bind and dissociate from the DNA duple" (se" rig.r.e 10-10).

--+ Promoter

-35
+l
-10

TTGACA TATAAT Gene z

Polymerase Box
it
RNA Polymerase slides along the DNA duplex
looking for the promoter site. It does not repeatedly
bind and then dissociate in its qwst for this site.

Figure tO-lO

Copyright @ by The Berkeley Review 349 The Berkeley Review


Specializing in MCAT preparation
Biology Dxpression of Genetic Information RNA Synthesis & Modifications

Tfanscription
RNA polymerase will synthesize RNA from a DNA duplex in three stages,
initiation, elongation, and termination, as indicated in the sequence of diagrims
shown in Figure 10-11.

Initiation

Promoter
Region

l'---*o
Elongation t

-35

l-- I Rho
Termination

i
i
i

f*g.h"." gr
it'

|n
R
b,i
itr€

I NE
sr

@RNA
h
Figure lo-l I 1r:
:1-
when the RNA polymerase binds to the promoter region of the DNA duplex ar ;g
initiation complex is formed. As soon as the initiation complex is formld, the ca
sigma subunit dissociates from the polymerase leaving the core enzyme bound 5€{
ar€
Copyright @ by The Berkeley Review 39O The Berkeley Keview Cq
Specializing in MCAT preparation
Biology Expression of Genetic lnformation RNA Synthesis & Modifications

to the DNA duplex at the promoter site. with sigma missing, the core enzyme is
able to bind more strongly to the DNA template. As the DNA tempiate is
unwound and read in the 3'-+ 5'direction by the RNA polymerase core enzyrrre,
newly synthesized RNA is made in the 5' -+ 3' direction. This region of activity is
referred to as a transcription bubble. Nascent RNA is synthesized until a
termination signal is reached.

Accuracy of Replication and Tfanscription


We can consider the accuracy of replication and transcription by looking at Table
10-1. Recall that during DNA replication there were two process that ensured the
accuracy of the final product. These were (1) editing by the DNA polymerase
enzymes and (2) the repair enzymes. As a result, the probability of an error
occurring during DNA replication is quite low. Roughly 1010 bases will be
replicated before an error is made that is not caught by either the editing aspect
of DNA polymerase or the repair enzymes. Therefore, the mutation rate is very
1ow.

In contrast, the RNA polymerase do not have a polymerase editing function and
they do not have an enzyme repair system. As a result, the probability of an error
occurring is about one in every 104 or 105 transcribed bases.

Coding Polymerase Repair probability


Process Editing Enzymes of Enor
Replication + + 10-10

Transcription 10-4.10-5

Table I O. I

why, then, does DNA replication begin with an RNA primer? The editing system
of the DNA polymerase system will not work when replication is just starting
from the first base. The DNA polymerase enzymes will only work properly i}
there is a complementary double stranded structure to start working on. 'it
RNA primer that begins the okazaki fragments has a high probability of error--"
and they will eventually need to be removed. since there is now a double
stranded structure to work with, the DNA polymerase enzyme can use its
editing function to remove the RNA primer (and filt in the gap with DNA
nucleotides).

Inhibitors of RNA Polymerase


RNA polymerase can be inhibited. For example, the antibiotic actinomycin D
binds specifically to double stranded DNA and prevents RNA polymerase from
using it as a template for transcription. Actinomycin D has a phenoxazone ring
(similar to the anthracene ring) which can act as an intercalator and slip between
neighboring G-C base pairs in the DNA duplex.

Promoters and Dnh4ncers


we mentioned that in the prokaryote E. coli, the promoter sites can be found at
about the -10 and -35 regions. In eukaryotic DNA the promoter regions are
centered around -25 (called the TATA box or Hogness box) and around -75
(called the CAAT box and ranges between -40 and -110). There are also enhancer
sequences which also help to stimulate transcription. These enhancer sequences
are usually quite far away from the promoter region (e.g., several kilobases

Copyright @ by The Berkeley Review 391 The Berkeley Review


Specializing in MCAT Preparation
Biology Expression of Genetic Information RNA Synthesis & Modifications

away) and can be either upstream or downstream from the promoter region.
They can even be within the transcribed gene itself.

The process of whether or not a gene gets transcribed not only depends on the
correct promoter region but also on the interaction with specific enhancers. ThL.
can be seen in Figure 10-12. For example, consider the gene that codes for the
synthesis of hemoglobin. Hemoglobin will only be expressed in one cell type,
even though you have a great variety of different cell types in your body with the
same hemoglobin gene in every one of them. It might be that cell or tissue
specific proteins act as passwords which are capable of binding specific enhancer
regions. This interaction, in turn, will allow RNA polymerase to sit down on a
given promoter region and begin transcription of a specific gene.

Promoter
Several Resion
kilobases
aw -75 -25

Gtr}Str
CAAT TATA

DNA loops so distant


sites are juxtaposed

Multiprotein
Complex

Figure lO-12

The use of enhancers can also be seen in the hormonal action of glucocorticoidr,
Glucocorticoids can bind to a soluble receptor protein that is specific for a certaiun
enhancer region. Once this hormone-receptor complex binds to glucocorticri
enhancer region, transcription is stimulated.

For example, you will not be able to transcribe the genes that are important
mammary function unless you have the hormone as well. Males and
both have mammary glands. However, females have certain hormones which
bind specific proteins. Interaction of this hormone-protein complex with
enhancer regions allows for the transcription of mammary genes speci-tic
females. Since males do not have enough of this hormone, transcription of
mammary genes does not take place.

Modification of the Transcripts


A variety of modifications to RNA transcripts can occur. There can be
trimming and splicing (in which the ends of the RNA transcript can be excised or

Copyright @ by The Berkeley Review 392 The Berkeley Revier


Specializing in MCAT Preparatior
Biology Expression of Genetic Information KNA Synthesis Er Modifications

certain intervening sequences removed). There can be (2) addition of bases to the
primary transcript (such as the addition of the sequence CCA to the 3' end of
IRNA molecules).
A
There can also be (3) modification of bases, a feature that is particularly evident
with IRNA molecules. (Processes (2) and (3) are quite common in prokaryotic
HO- CH:n/ "1..x
fi"
cells.) For example, once you have a IRNA that has already been synthesized, its
bases can be modified (post-transcriptionally). Once such case would a
H
modification from uridine to pseudouridine as shown in Figure 10-13. Note the
difference in attachment of the glycosidic bond. HO OH

The most extreme modifications are those seen in eukaryotic transcripts. Recall Uridine
that within the nucleus of a eukaryotic cell we find DNA replication,
transcription, and processing taking place before the transcripts are sent out into
the cytosol. Once the transcript is in the cytosol, translation occurs at the
ribosomes.

Consider the processing of the primary RNA transcript. This primary RNA
transcript can be modified by (1) capping at its 5' end, (2) addition of a poly-
adenine (poly-A) tail at its 3'end, and (3) splicing which involves the excision of
introns.

5' Capping the of the Nascent RNA HO OH

If the first residue at the 5' end of our nascent RNA were not modified, then you Pseudouridine
would expect to find three phosphate groups attached to the ribose ring. These
"bare" 5'ends can easily be degraded by phosphatases and nucleases lurking Figure lO-15
within the cell. However, the 5' end is modified in a reaction with GTP.

First, the gamma phosphate is hydrolyzed at the 5' end of the RNA. Next, the
beta phosphate at the 5' end of the RNA attacks the alpha phosphate of GTP,
releasing pyrophosphate in the process (which is subsequently hydrolyzed to
two molecules of orthophosphate). An unusual triphosphoanhydride linkage is
formed between the 5' end of the RNA and the 5'end of the guanylate residue as
shown in Figure 10-14,

Ji .*,':
"'* -"ffc-o-i;'-i;'-i;:H"
",.1-.r)^ o-r- "- r- "- i
H,c-

HO OH o ocHl
\/ Ol
o-P=o
I

cap

"H
n- o ocHj

Figure lO-14

i]opyright O by The Berkeley Review 393 The Berkeley Review


Specializing in MCAT Preparation
Biology Expression of Genetic Information RNA Synthesis & Modifications

s-adenosylmethionine (sAM) is the methyl donor that next adds a methyl


group
to the N-7 position of the purine ring system of guanine. Also, the z'-hyd"roxyl
groups of the adjacent ribose rings can be methylaled by sAM as well. Capping
protects the 5' end of the polymer from degradation and thereby
tn!
stability of the RNA molecule. "rrnur,.",

3' Polyadenylation of the Nascent RNA


As eukaryotic RNA polymerase nears the end of its transcription process, a
sequence of bases is recognized in the primary transcrip t by 'a specific
endonuclease. The primary transcript is cleaved and about 250 adenine ,"iidrru,
(from ATP) are added to the free 3' end of the RNA polymer to form the
polyadenylated tail as shown in Figure 10-15. The poly-A tait hetps to proteci
5' 3' the 3' end of the RNA polymer from nucleases and phorphuturur.
cAPr A-A-A-(A)n-OH
Nascent Spticing (an overview)
RNA within many genes in eukaryotic DNA are sequences which are untranslatei.
These untranslated sequences, called intervening sequences (introns),
Figure IO-lE lie
between regions of the gene that are translated orlre expressed (exonsj,
The
introns are spliced out-of the primary RNA in a process .uil"d splicing
and th:
exons are joined together. The exons are then used as the coding dorirains for
proteins. The 5' non-coding region of the RNA is important for ribosome
recognition. This is shown in Figure 10-16.

Primary
RNA
-3'
t l- Inrron]rntron
Non-c.oding
I f
Non-coding
reglon I reglon
I Processrng Events:
Spticing: Capping;
I
I Polyadenylotion

mRNA
t
t'.* poly-A 3,

Non-coding Non-coding
region feglon

Figure I O- l6
It is still a mystery as to why the exons are separated by the introns. one
possibility might deal with recombination. A relaled possibility is that it might
allow enhancers to come and go. This would allow for genetic variation within
and among species with regards to the signals that say juit how turned on a
gene
should be. Recall that we mentioned that enhanc"r ,"qtr"r-r."s can often lie within
a gene--in particular, the enhancers reside within these introns.

snRNPs can form Spliceosomes


Within the cytosol and nucleus of many eukaryotic cells are RNA molecules that
are associated with specific proteins. When this situation exists in the nucleus
these complexes are referred to as small nuclear ribonucleoprotein particles
(snRNPs), or more fondly referred to as "snurps". snRNps are iivolved with
the

Copyright @ by The Berkeley Review 394 The Berkeley Review


Specializing in MCAT preparation
Biology Expression of Qenetic Information RNA Synthesis & Modifications

base pairing reactions at the spliced junctions of introns and exons. snRNPs can
recognize the G-U and A-G. If this processing is not done accurately, then a
frame shift error might result.

Self'Splicing RNA
The details of how snRNPs react are still not clear. However, it is quite possible
that the actual chemistry is done by the RNAs and not the associated proteins.
This came into consideration because of the discovery in ciliated protozoans that
the self-splicing reaction involves rRNA (not mRNA) and not the associated
proteins. In this case the reaction can be catalyzed by the attack of a 3'-hydroxyl
of a guanosine cofactor residue on the phosphodiester bridge between exon A
and the intron. This is shown in Figure 10-17.

Exon A ends up with a free 3'-hydroxyl that can next attack the phosphodiester
bridge between exon B and the intron. The result is the splicing of exon A and
exon B and the excision of the intron with the guanosine residue at its 5' end and
a free 3'-hydroxyl. Further reactions (not shown) will ultimately release the
guanosine residue, meaning that guanosine has a catalytic function. This type of
intron self-splicing is referred to as being in Group I.

G-OH
I enack
uo-ltiTl-s'
f=ft--""
I t-J
o1-.'

lrn,ron
t
L.@'''->
5'- Exon AlExon B

:3'

no-lp-oills'

Figure lO-17

This was the first clear example in which RNA could be involved in its own
chemistry without the help of any enzymes. Up until a few years ago it was one
of the holy laws of bioc\emistry that only proteins and their coenzymes could be
catalysts. The clearest case in which RNA can calalyze reactions by itself are from
the protozoan rRNAs.

Copyright @ by The Berkeley Review 395 The Berkeley Review


Specializing in MCAT Preparation
Biol ogy Expression of Genetic Information BNA Synthesis & Modifications

so, under the category of self-splicing there can be Group I introns or Group II
introns. Group I introns are spliced out by guanosine at the 3lhydroxyl. Group
II introns are spliced out by adenosine at the 3'-hydroxyl.

There can be mutations in DNA which will affect splicing. For example, you
could mutate the 5'splice site so that the G-u sequence is changed to some other
sequence that is not recognized by the snRNP. Similarly, you could mutate the 3'
splice site to something that is not recognizedby the snRNp. Also, the region
that contains the adenosine moiety involved in the 2', S'-phosphodiester bond
could also be mutated as well.

New splice signals could also be created. If a new A-G splice sequence came too
soon in the intron, the exon at the 5' end could attack the wrong site. The
resulting functional message would have a piece of the 3rend of the intron which
would not be a coding sequence. This would lead to a defective protein.
Mutations like these can turn up in the hemoglobin genes, and one of the more
common types of mutations are those that cause thalassemia (defective synthesis
of a hemoglobin chain(s)).

Copyright @ by The Berkeley Review 396 The Berkeley Review


Specializing in MCAT Preparation
Biology Expression of Genetic Information Protein Synthesis

ffiftefifiilii:ffiffi r$
Let's take a look at protein synthesis. Protein synthesis occurs on the ribosomes
in three steps: initiation, elongation, and termination. However, before we can
begin to consider these processes, we first need to consider how it is that amino
acids are brought to the site of protein synthesis.

Amino acids are brought to the ribosome by an adapter molecule called transfer
RNA (tRNA). This molecule contains a template recognition site for the mRNA ! eo.nin.
codon called the anticodon and an amino acid attachment site. An amino acid o= o_.g":.Zo:.1
(like Phe) would be attached to the 3'-OH function at the 3' end of the IRNA OO i_
molecule. This can be seen in Figure 10-18.

The base sequence at the 3' end of the tRNAs is 5'-CCA-3'. The 5' end of the
OJOH
I
TJ
IRNA has a free phosphate group. Hydrogen bonds between U-A and G-C o=c
I
ribonucleotides help to hold the tRNA together at the stems. About half of the H-C_R
base pairs in IRNA can be found in these stems. Certain groups of bases within t 1",
the tRNA molecule are not base-paired. Some of these groups are found in the
An amino
loops of the molecule. For example, there is the dihydrouracil (DHU) loop, the acid
ribothymine-pseudouracil-cytosine (TyC) loop, and the anticodon loop. These
Figure rO-18
features define the secondary structure of IRNA.

The Genetic Code: How was the genetic code deciphered? In 1961 Marshall
Nirenberg took a solution of E. coli, lysed the cells, and removed the cell-
membrane and cell-wall fragments. He was left with an extract that contained
various other constituents of the cell, including DNA, mRNA, IRNA, enzymes,
and ribosomes. When amino acids and an energy source, such as ATP or GTP,
was added to this extract and then incubated at 37 'C for a brief period of time,
protein synthesis occurred on the ribosomes. Nirenberg was able to stop this
protein synthesis by degrading the DNA template used for nRNA synthesis. It
turned out that mRNA had a rather short half life (because it is degraded by
specific cellular enzymes). Protein synthesis could be started once again if mRNA
was added back to the extract.

Nirenberg had access to an enz)rmatic protein called ribonucleotide phosphory- nNDP + (RNA)n
lase. This enzyme was an undirected enzyme that allowed for the synthesis of
synthetic chains of ribomrcleic acids as long as the right ribonucleotides were
added to the medium. R&aI that DNA polyherase at d RNA polymerase were 1l
both directed enzymes. Ribonucleotide phosphorylase uses ribonucleoside (RNA)'*1 + nP1
diphosphates (NDPs) and catalyzes the synthesis of polyribonucleotides (RNA)
with the subsequent release or orthophosphate (P). This can be seen in Figure Figure lO-19
10-19.

Note that in the case of RNA polymerase, pyrophosphate (PP) is formed which
can then be hydrolyzed by inorganic pyrophosphatase in order to help drive the
reaction (of RNA synthesis) to completion. \A/hat this means is that in vivo the
reaction involving ribonucleoside phosphorylase cannot be driven to the right
(completion) by the subsequent hydrolysis of pyrophosphate. Why? Because
there is no pyrophosphate produced. Why? Because in this case we are using
ribonucleoside diphosphates (NDPs) instead of ribonucleoside triphosphates
(NTPs). In other words, the equilibrium for the reaction shown in Figure 10-19 is
to the left (degradation of RNA). In fact, ribonucleotide phosphorylase actually
takes partially degraded mRNA and phosphorylates it to form ribonucleoside
diphosphate substrates. So, how does ribonucleotide phosphorylase catalyze the

Copyright @ by The Berkeley Review 397 The Berkeley Review


Specializing in MCAT Preparation
Biology Expression of Genetic Information Protein Synthesis

synthesis of mRNA? It depends on the concentration


of the ribonucleotides
added to the reaction.mixture. If high concentrations of a
given ribonucleotide
are incubated with ribonucleotide phosphorylase, then th!
reaction shown in
Figure 10-19 will be driven to the right.

lst 2nd Position


3rd
Position
(5'End) U C A G (3'End)

Phe Ser Tyr Cys U


Phe Ser Tyr Cys C
Leu Ser STOP STOP A
Leu Ser STOP Trp G

Leu Pro His Arg U


Leu Pro His Arg C
Leu Pro Gln Arg A
Leu Pro Gln Arg G

Ile Thr Asn Ser U


Ile Thr Asn Ser c
Ile Thr Lyt Arg A
Met Thr Lys A.g G

Val Ala Arp Gly U


Val Ala Asp Gly C
Val Ala Glu Gly A
Val Ala Glu Gly G

Table lO-2
The Genetic Code.

when Nirenberg used high concentrations of the ribonucleotide uDp in the


presence of ribonucleotide phosphorylase, the polyribonucleotide
uuuuuL-
(i.e., poly-u) was synthesized. It turns out that iiboro*", are
not designed to
take substrates like poly-U' In order to get protein synthesis, artificial
conditions
had to be employed. For example, 0.01 M Mg2e ions were used, which is
at leas;
ten-fold more magnesium than normally found in cells. once pory-u .w,as
translated on the ribosome, the protein that was synthesized contained all phe
(i.e., poly-Phe). Nirenberg was able to postulate that the nucleotide
triptet UUL
coded for the amino acid phe.

How does the ribosome read the mRNA in order to make proteins? we knoir
that proteins are composed of amino acids. In 1961 various ieatures of the
base
sequence in DNA and the amino acid sequence in proteins led to the
establishment of a genetic code. There are foui different Lases in DNA. If
the
codervere a simple single-base code, then only 4 different types of amino
acids
could be specified. And since we know that we have at teasi zo different aminc
acid this does not seem too likely. If we had a two-base code, then 16 differerri
types of amino acids could be specified. Again, this is still less than the 20
different amino acids that we know we have. F{owever, if we had a three_base

Copyright @ by The Berkeley Review 59a The Berkeley Review


Specializing in MCAT preparation
Biology Expression of Genetic Information Protein Synthesis

code, then we could specify 64 amino acids. Indeed, the 20 different amino acids
that we have been exposed to are coded by a three-base code on the DNA
molecule. These groups of three bases are referred to as codons (see Table 10-2)

which way are the proteins synthesized? Is it from the amino terminal to the
carboxyl terminal or is it from the carboxyl terminal to the amino terminal? For
proteins it was postulated that they were synthesized from the amino end to the
carboxyl end.

In order to determine this, hemoglobin synthesis from the red blood cells (RBCs)
of rabbits was analyzed. There are immature RBCs that make almost nothing but
hemoglobin. If you inject a rabbit with hydrazine, it will destroy the rabbit's
RBCs. To compensate for this the rabbit will begin to synthesize new RBCs. If
you had a start site for hemoglobin at one end of the protein or the other, then 5'-AAAAAAAACAAA-3'
you would make a longer and longer chain until you reached the end. It should
be possible to label chains that are almost complete just by adding radioactively vfl Ribon.r"l""."
labeled amino acids to your reaction mixture for just a few seconds. The ends of 5'-AAAAAAAAC-3'
the polypeptide chains that was synthesized last would be labeled. + 5'-AAA-3'

The rabbit RBCs were lysed and amino acids with a label were added to the $Transtation
solution for a few seconds. The mature hemoglobin was isolated in order to find
N-Lys-Lys-Asn-C
out where the label was. The label was found to be at the carboxyl end of the
polypeptide. Therefore, the carboxyl end was synthesized last while the amino fl Carboxypeptidase
1'
end was synthesized first. You must also rcalize that as the protein is being
synthesized from the N-terminus to the C-terminus, that region near the N-
N-Lys-Lys-C + N-Asn-C
terminus can begin to fold up on itself to give tertiary structure even before the Figure lO-2O
rest of the protein has been synthesized.

In what direction is the mRNA read? Polynucleotide phosphorylase was


incubated with a lot of ADP and a little bit of cDP in order to make a mixed
polymer. This polymer mostly consisted of A residues. Occasionally a C residue
would be incorporated. The polymer was then treated with pancreatic
ribonuclease which cleaves on the 3' side of C residues. What is observed are
polymers with long runs of As ending in a C residue at the 3'end. This polymer
can then be used as a translation system. What you will get is a polymer with a
lot of Lys in it and a very little bit of Asn. When this polymer is treated with
carboxypeptidase, Asn will be released from the poly Lys polymer. This is shown
in Figure 10-20.

Carboxypeptidase is a digestive enzyme that hydrolyzes the carboxyl-terminal


peptide bond in polypeptide chains, particularly if that residue has a bulky
aliphatic side chain or an aromatic ring. Thus, Asn must be at the carboxyl end of
the polymer (the end that is synthesized last). This means that the coding triplet
for Asn is AAC. In other words, the coding triplet for Asn is at the 3'end of the
polymer and also at the carboxyl end of the protein. The direction of translation
is from 5'-+ 3'. When RNA polymerase is transcribing DNA in order to
synthesize RNA, it makes RNA in the same direction in which you get
translation.
{
About half of the codons were determined by a triplet binding assay. Suppose
that ribosomes were able to bind to trinucleotides. The example that we will use
is 5'-AUU-3'. Once this binds to the ribosomes, then IRNA can bind. In this case
the tRNA for Ile would be the only IRNA that would bind to the ribosomes.

Copyright @ by The Berkeley Review 399 The Berkeley Review


Specializing in MCAT Preparation
Biology Expression of Genetic lnformation Protein S5rnthesis

Thus, AUU was said to be the coding unit for lle. In this way various other
codons could be worked out.

The remainder of the genetic code was filled out by Gobind Khorana. Khorana
was able to synthesize polyribonucleotides with a repeating sequence that was
well defined from one of the two strands of a double strarided DNA molecule
that had the repeating deoxyribonucleotide residues 5,-TAc-3' and 5'-GTA-3'. In
order to be selective about which of the two DNA strands that were to be
transcribed, only three of the four complementary ribonucleoside triphosphates
were added to the reaction mixture. This technique allowed one strand of the
duplex DNA to be transcribed but not the other itrand. when Khorana added
ATP, urP, and CTP to the mixture containing the double stranded DNA, he got
the repeating RNA sequence 5'-UACUACUACUACUACUAC-3'. when he
added GTP, urP, and ATP to the mixture containing the double stranded DNA,
he got the repeating RNA sequence S'-CATCATCAT-ATCATCAT_3'.

once the defined RNA polymers were synthesized they were used as templates
for protein synthesis on the ribosome. For example, if ihe mRNA polymer were
poly -UAC, there would be three ways to read it, we could either rlad it as poly-
UAC, poly-ACU, or poly-CUA. This will give us three different polymers. t? ltis
read as poly-UAC, then we will get poly-Tyr. If it is read as poty-lcu, then we
will get poly-Thr. If it is read as poly-CUA, then we will geipoiy-Leu. This can
be seen in Figure 10-2'1,a.

5'-U-A-C-U-A-C-U.A-C-U-A-C-3' 5'-G-U-A-G-U-A-G-U-A-G-U-A-3'
gatJHl- t---Jt-'_r

Tyr Tyr Tyr Tyr Val Val Val Val

5'-U-A-C-U-A-C-U-A-C-U-A-C-3' 5'-G-U-A-G-U-A-G-U-A-G-U.A-3'
\---
--J
Thr Thr Thr
-/J
Stop Stop Stop

5'-U-A-C-U-A-C-U-A-C-U-A-C-3' 5'-G-U-A-G-U-A-G-U-A-G-U-A-3'
\--l
-r#
Leu Leu Leu Ser Ser Ser
(a) (b)

Figure IO.2l

However, if we read the complement of the poly-uAC strand, which is poir--


GUA, then we will get only two polymers. Why? We could read poly-GUA a_.
either poly-GUA, poly-UAG, or poly-AGU. If we read it as poly-GUA, then u-e
will get poly-Val. If we read it as poly-AGU, we will get poly-Ser. However, jj
we read it as poly-uAG, then nothing ever appears. It turns out that 5'-uAG'-3' L.
a stop codon. The other termination codons are 5'-UAA-3'and St-uGA-3'. ThL.
can be seen in Figure 10-21b.

Not only are there stop signals but there are also start signals. The start signaX-
could not be determined by using Nirenberg's system because his translati.on
system would start anywhere (due to the high magnesium levels). In order t;
Copyright @ by The Berkeley Review 40o The Berkeley Reviel
Specializing in MCAT Preparation
Biology Expression of Genetic Information Protein Synthesis

find the start signals, a natural mRNA molecule was needed (not a synthetic
mRNA).

The RNA from RNA bacteriophages can be recognized from regular DNA
because RNA is denser than DNA (due to the presence of uracil and the extra
oxygen atom at the 2'position of the ribose ring). The RNA from these phages,
when first isolated, was thought to have about 3,000 nucleotides (or about 3
genes). A start site on the RNA from one of these RNA phages was determined in
the following manner.

Sticking out of this length of phage RNA is a small protrusion that is the start
signal for the coat protein of the phage. Ribosomes can be denatured into the 30S
and 50S subunits in low magnesium concentrations. The 30S subunits of the
ribosome will bind to the start signal for the coat protein. If ribonuclease is added
to the solution, then the rest of the RNA will be degraded. The only RNA that is
left will be that stretch of RNA "protected" by the 30S subunit of the ribosome. In
other words, the start signal is not denatured. Within this protected piece of RNA
is the sequence 5'-AUG-3' followed by a few more coding residues. This triplet
codes for the amino acid Met. If you now take the phage coat protein and
analyze the amino terminus, you will find that it starts with Met, followed by the
corresponding coding residues.

The start codons for the maturation protein (only one copy at the vertex of the
phage) and the replicase protein (copies RNA to make more RNA) were
discovered in a similar fashion. The RNA was sheared in a Waring Blendor to
obtain smaller fragments. The same experiment was repeated by binding the 30S
subunit of the ribosome to all the fragments and then looking for protected
segments. Both of these genes also start with 5'-AUG-3'.

It turns out that the most common start codon is 5'-AUG-3'which codes for Met.
There are some rare cases in which the start codon is 5'-GUG-3'. This triplet
codes for Val.
CH.
Eukaryotic cells also start their proteins by using the codon 5'-AUG-3'. Instead of I

fMet as thefirst amino acid we find that it is simply Met-but with a slight Gr A-A-A-(A)n-OH
variation. Eukaryotic messages get processed before they even reach the 5' Nascent 3'
ribosome. For example, if the first residue at the 5'end of our nascent RNA were RNA
not modified, then you-would expect to find three phosphate groups attached to
Figure lO-22
the ribose ring, These "bare" 5' ends can easily be degraded by phosphatases and
nucleases lurking within the cell.

However, the 5' end is modified in a reaction with GTP. First, the gamma
phosphate is hydrolyzed at the 5' end of the RNA. Next, the beta phosphate at
the 5' end of the RNA attacks the alpha phosphate of GTP, releasing
pyrophosphate in the process (which is subsequently hydrolyzed to two
molecules of orthophosphate). An unusual triphosphoanhydride linkage is
formed between the 5'end of the RNA and the 5' end of the guanylate residue.

A methyl donor (S-adenosylmethionine (SAM)) adds a methyl group to the N-7


position of the purine ring system of guanine. This process is called capping'
Also, the 2'-hydroxyl groups of the adjacent ribose rings can be methylated by
SAM as well. Remember, capping not only protects the 5' end of the polymer
from degradation but it also enhances the stability of the RNA molecule as well.
This can be shown in Figure 10-22.

Copyright @ by The Berkeley Review 40t The Berkeley Review


Specializing in MCAT Preparation
Biology Expression of Genetic Information Protein Synthesis

As eukaryotic RNA porymerase nears the end of its transcription process,


a
sequenceof bases is recognized in the primary transcripi by i specific
endonuclease. The primary transcript is cleaved and about 250 adenine
residues
(from ATP) are added to the free 3' end of the RNA polymer to
form the
poiyadenylated tail as shown in Figure 10-22. The poly-a tait hetps
to protect
the 3' end of the RNA polymer from nucleases and phorihutur"r.

within many genes in eukaryotic DNA are sequences which are untranslated.
These untranslated sequences, called intervening sequences (introns),
rie
between regions of the gene that are translated or-are (exorrsj. The
introns are spliced out-of the primary RNA in a process ""pr"rrud
splicing and the
"ull"d
exons are joined together. The exons are then uied as the coding doirains for
proteins. The 5' non-coding region of the RNA is important lor ribosome
recognition. This is shown in Figure 10-23.

Primary
RNA
- --^--r1 "*on t ^.r.r.nffi6 -.1

t Trntron=rntron? t
Non-c.oding Non-coding
reglon regibn
I o.o..rr'n, Evenrs:
I Spticing; Capping;
I Polyadenylation

mRNA
I
-'-a" poly-A 3,
it23t
Non-c.oding Non-c.oding
regron reglon

Figure 1O.23

It is still a mystery as to why the exons are separated by the introns. one
possibility might deal with recombination. A relaied possibility is that it might
allow enhancers to come and go. This would allow for genetic variation within
and among species with regards to the signals that say just how turned on a gene
should be.

Amino Acid Activation


The activation of amino acids is similar to the process in which fatty acids are
activated and to the process in which DNA ligase joins to segments or brua.

Peptide bond formation between two amino acids is thermodynamicallr-


unfavorable. This unfavorable situation is overcome by activation of tire carbox;,I
group of incoming amino acids. In the first step of the reaction ATp reacts with
an amino acid to form an activated mixed anhydride linkage in the form of
aminoacyl-AMP. In the second step the aminoaryl group of a-minoacyl-AMp is
transferred to a IRNA molecule to form aminoacyl-tRNA. rnis transfer can take

Copyright @ by The Berkeley Review 402 The Berkeley Review


Specializing in MCAT preparation
Biology Expression of Genetic Information Protein Synthesis

place at either the 2'-hydroxyl or the 3'-hydroxyl of the ribose moiety at the 3' end
of the tRNA molecule. The amino group, however, can migrate between either of
these two hydroxyl functions. Aminoacyl-tRNA is the activated intermediate in
protein synthesis and the enzyme that catalyzes this reaction is an activating
enzyme (i.e., a specific aminoacyl-tRNA synthetase). This is shown in Figure 10-
24.

Step 1

HO HOO
O I ll O o
t il ll
H1N-C-C-O
'l + ATP : H3N- C-C- O- P- O- Ribose + ppi
lrl
R
R oo Adenine

Aminoacyl-AMP

Transfer RNA
I

o
ol
O-P=O
Step 2 I

O - CH2 ,., Adenine O Adenine


oll
Aminoacyl-AMP + IRNA + )/"v
H-\J-" + O- P-O-Ribose
lo
I

/\ o
HN OH
AMP
I

o=c
I

H_C_R
@t
NH:

Aminoacyl-tRNA

Figure 10.24

You shouid be aware that there are many different IRNA molecules within the
cell, and they might average about 80 bases long. This means that there has to be
great specificity as to which amino acid is placed on which IRNA molecule.
There is a minimum of 20 different activating enzymes, one for each amino acid.
Each activating enzyme is quite selective. For example, it will only place valine
on the IRNA specifyilg valine and not on the tRNA specifying isoleucine. Valine
and isoleucine are rather similar to one another and differ only by the fact that
:soleucine carries an extra methylene GCHz-) group. How does the activating
.nzyme distinguish between the two amino acids?

.{minoacyl-tKNA Synthetase Specificity


'\'e need to consider the accuracy of the aminoacyl-tRNA synthetase enzyme
:.e., the activating enzyme). Roughly 1 mistake is made for every 3000 catalytic
-r-ents. One way to assure this accuracy is to have a proofreading system
aaalogous to that in DNA replication). In many IRNA synthetases there are two
:atalytic sites--a hydrolytic site and an acylation site. The hydrolytic site is
:cncerned with the aminoacyl-AMP molecule while the acylation site is
, rncerned with the aminoacyl-tRNA.

:uppose we want to attach isoleucine to a IRNA molecule but in the initial step
. e mistakenly activate a valine molecule (i.e., valine-AMP). Since valine is

,naller than isoleucine it can easily enter the hydrolytic site where it is

, rpyright O by The Berkeley Review 403 The Berkeley Review


Specializing in MCAT Preparation
Biology Expression of Genetic lnformation Protein Synthesis

zubsequently hydtolyzed. Since isoleucine is larger than valine it cannot fit


into
the hydrolytic site and therefore is not hydrolyid. However, isoleucine can fit
into the acylation site where it activated and attached to an isoleucine specific
IRNA.

During replication we mentioned that the error rate is about 1'l101o. During
transcription the error rate is about 1/104 or 1/105. Why is transcription more
error prone comp_ared to replication? Most transcripts are rather short (less than
104 or 105 baseJlong). if ybu have a probability oi ot tt111'a or t/10u,;h""
"rro,
most of the molecules that are made that are less than 104 or 105 units long will
not have any errors. In other words, you do not need to have an error ra"te of
Ll10tu if you are making a molecule that is only 11104 or 1/105 nucleotides lons.
The same holds true for protein synthesis. Error rates of lllr1a are fine b".a.rr""
you are not making proteins that are 104 amino acids long.

tRNA Design
Atl IRNA molecules are capable of forming a cloverleaf structure that is about g0
ribonucleotides long. An amino acid like phenylalanine would be attached to
the
terminal adenosine residue at the 3'end of the IRNA. The 5' end of the tRNA has
a free phosphate group. Hydrogen bonds between U-A and G-C ribonucleotides
help to hold the IRNA together at the stems. About half of the base pairs in IRNA
can be found in the stems. One will occasionally see G-U base pairing (we will
come back to this point later). Certain groups of bases within tne tnNCmolecule
are not base paired. Some of these groups are found in the loops of the molecule.
mRNA

**;X
ffi 3' There is the dihydrouracil (DHU) loop, the ribothymine-pseldouracil-cytosine
(TyC) loop, and.the anticodon.loop. The featurei that we have just mention
define the secondary structure of tRNA.

The IRNA molecule is not flat and planar but rather has tertiary structure ir-
which the molecule is L-shaped and contains two segments of doutle helix, eaC
containing about 10 base pairs. Base pairing in the nonhelical regions of thE
molecule are rather unusual in that there can be base-base, base-b-ackbone, ol
3',
backbone-backbone interactions. The backbone interactions involve the 2'-
Figure lO-25 hydroxyl function of the ribose ring.

Codon-Anticodon Interactions
A triplet of three bases in an mRNA polymer represent a codon that calls for l
particular amino acid. The amino acid that is called for should be attached ic :" :
tRNA molecule that has an anticodon that is complementary to the codon of -,:.: JI

Codon Codon
nRNA. This can be seen in Figure 10-2s. Note that the codon-anticod.i_
relationship is anti-parallel.
YVU\./ UUC The rules for base pairing of the codon-anticodon junction are like those ,fi::
I I I 'T;'J"' operate in DNA. In this case A will pair with U and G will pair with C. Hou.e-,,e:
there is an additional possibility in which G can pair with U. Recall that there .::
AAG AAG only 20 different u*ino acids but there are o+- triplets (3 stop codons anc. dl.
Anticodon Anticodon informational codons). on the average there are 3 codons per amino acid. Thr:r
Figure 1o'.26 are not 61 tRNA molecules, which means that a given IRNA can recognize r:,::*
than one codon. For example, 5'-uUU-3'and 5'-uuc-3'are codons tiat spe_-:,
phenylalanine. Both of these codons are recognized by the same IRNA which" :ro
the anticodon sequence 5'-GAA-3'. Codon-anticodon pairing can be see:. rm
Figure 10-26. How can one IRNA base pair with two different codons? The b::.r*i
for this phenomenon has to do with wobble in the base pairing.
;n

Copyright @ by The Berkeley Review 4o4 The Berkeley Rericm {m,r


Specializing in MCAT preparatiun
Biology Expression of Genetic lnformation Protein Synthesis

These two codons, 5'-UUU-3' and 5'-UUC-3', are both recognizedby the same
tRNA as shown in Figure 10-27. The IRNA that is recognized by the activating
enzyme is tRNAPhe, and in particular it is recognizedby
a specific activating
enzyme for Phe called phenylalanyl-tRNA synthetase. whenthis specific
activating enzyme transfers the aminoacyl group of Phe to tRNAPhe, we can
designate that activated aminoacyl-tRNA as Phe-tRN4Phe. Note that a charged
IRNA would have the amino acid attached to it while the uncharged tRNA does
not have the amino acid attached to it.

3'-Phe 3'-Phe

5' 5'

'*^1^L
5'-y-g 3' mRNA 5'-
A-A
U-U _C-
L 3'

(a) (b)

Figure lO-27

Note that in Figure l0-27a we have a non-standard base pair between U and G
while in Figure l0-27b we have a standard base pair between C and G. We can
represent these pairings as shown in Figure 10-28. We can line the structures up
such that positions 2,3, and 4 in the pyrimidines and positions2,l, and 6 in the
purines are facing each other.

ln the case of tlte non-standardbase pairing between U and G, the pyrimidine


base is displaced with respect to the purine base. This is shown in Figure 10-28a.
If this base pairing were to occur in a DNA double helix, it would distort the
backbone. However, in IRNA this does not matter too much because we are not
making that large of a double helical structure. We simply have three bases of
nRNA pairing with three bases of IRNA. In other words, there is a great deal
more flexibility in this type of interaction in IRNA (compared to DNA). Standard
hydrogen bonding between the C and G base pairs will occur as shown in Figure
10-28b.

)
N
Cvtosine Guanine
I
Backbone

Guanine
(a) (b)

Figure lO-28

Copyright @ by The BerkeleY Review 405 The BerkeleY Review


Specializing in MCAT Preparation
Biology Expression of Genetic Information Protein Synthesis

Wobble Rules in Base Pairing


Wobble is simply steric freedom in the pairing of the third base of the codon. The
wobble rules apply at the first base of the anticodon and the third base of the
codon. These rules are:

1. U can pair with A (standard) or G (non-standard).


2. G can pair with C (standard) or U (non-standard).
3. Inosine (I) can pair with either A, U, or C"

We could have a four-fold degenerate codon such as 5'-GGG-3', GGA, GGU,


GGC, where they would all specify glycine. Three of these codons could be read
by one IRNA if the IRNA had hypoxanthine at the first position of its anticodon.
The other codon (GGG) would need to be read by a different tRNA. Recall that
hypoxanthine is the purine base of inosinate (IMP), an intermediate in nucleotide
biosyrrthesis.

Ribosome Structure
About two-thirds of the weight of a ribosome is IRNA. Ribosomes in E. coli are
Iarge molecules, about 200 A in diameter and having a mass of about 2700 kd.
The ribosome is described in terms of a sedimentation coefficient called the
Svedberg unit (S). A ribosome (70S) can be dissociated into a small subunit (30S)
and a large subunit (50S). This is shown in Figure 10-29. Note that it is a non-
linear relationship between size and S-value. The dissociation of the 70S
ribosome is controlled by the concentration of Mg2o. Low Mg2e causes
dissociation of the ribosome.

TOSA
[ ..',,,,,.
Ribosome l

\
/\

50s
subunit
qry 30s
subunit

-+? I
Yo
I
Y
23S rRNA 55 rRNA 165 rRNA

+ 34 proteins 21 proteins

Figure lO-29

The 50S and the 30S subunits can be further dissociated in the presence of urea.
Dissociation of the 50S subunit yields about 34 proteins and 23S rRNA and 55

Copyright @ by The Berkeley Review 406 The Berkeley Review


Specializing in MCAT Preparation
Biology Expression of Genetic lnformation Protein Synthesis

rRNA. Dissociation of the 30s subunit yields about 21 proteins and 165 rRNA.
rRNA has a well defined folding pattern containing many short duplex regions.

Ingredients for Protein Synthesis


In order to make a protein we would need the following items:

1, 20 aminoacyl-tRNA's, each attached to the correct tRNA (more may be


needed).
2. nRNA that contains the codons that specify the protein sequence that is
desired.
3. Ribosomes.
4. Additional proteins called initiation factors that are not part of the
ribosome. There are initiation, elongation, and termination factors.
5. GTP as an energy source.

Initiation of Protein Synthesis


Consider a fairly late stage in initiation when the 50S and the 30S subunits have
joined together to form the 70S ribosome. The nRNA is appropriately placed
with its 5'end in the 30S subunit. The mRNA sequence which has recently been
synthesized from our DNA duplex has signals which are important. For
example, the mRNA molecule has polarity-it has a 5'end as well as a 3' end. The
initiating codon in mRNA is the base triplet 5'-AUG-3'. About 10 nucleotides
upstream from the initiating codon is a purine-rich sequence of bases (e.g.,
AGGAGGU) called the Shine-Dalgarno sequence. The Shine-Dalgarno
sequence is responsible for binding the mRNA to the 165 subunit--an
architectural constituent of the 30S ribosomal subunit. Binding of the Shine-
Dalgarno sequence to the 3' end of the 165 ribosomal subunit forms the initiation
complex as shown in Figure 10-30.

G
3'end of 165
3'-OH A +- ribosomal RNA
mRNA AU
I UC
UCCUCCA
s'-GAUUCCU AGGAGGU UUGACCU AUG CGAGCU UUU AGU-3'
fMet-Arg-Ala-Phe-Ser- -

Shine-Dalgarno Polypeptide
Sequence Sequence

Figure f O-3O

The codon in mRNA that starts protein synthesis is 5'-AUG-3'. This codon is
:a1ling for a special modified amino acid called formylmethionine (fMet). How is
:his modified amino acid brought to the ribosome for protein synthesis? It is
rrought to the ribosome by an adapter molecule called IRNA. We can abbreviate
--nis IRNA adapter molecule with fMet on it as fMet-tRNAf. Remember, a IRNA
rolecule contains a template recognition site (for the nRNA codon) called the
anticodon and an amino acid attachment site.

,-.rpyright @ by The Berkeley Review


speciarizin" ffir3i'f Ft:L }.:HH
Biology Expression of Genetic Information Protein Synthesis

After the 30S subunit has formed a complex with the initiation factors that we
mentioned as part of the ingredients in protein synthesis, GTP will bind to a
particular initiation factor called IF2. This allows the mRNA and the fMet-
flVlet
tRNAf initiator signal to join the 30S subunit as well. The subsequent release of
these initiation factors allows the 50S complex to attach to the 30S complex,
fM", ,RNA.l i'
thus forming the 70S initiation complex. The 70S initiation complex is now

l"l ,,'f,i",,
mRNA
primed for protein synthesis. This can be seen in Figure 10-31.

It turns out that there are two tRNAs for methionine. One form of IRNA, called
tRNAf, has the initiation role we just mentioned, while the other IRNA, we'll
call tRNAm, will simply bind to AUG codons elsewhere in the mRNA.
30S Initiation Complex

Let's consider the process by which methionine is attached to its respective


tRNA. We would expect that an activating enzyme will attach methionine to
tRNAf and form Met-tRNAf. We would also expect the same activating
A-site \ enzyme to attach methionine to tRNAm.
3',
fMet

li n
In the next step a formyl group attached to THF will react with Met-tRNAf to
form formyl-methionine-tRNAf (abbreviated as fMet-tRNAf) and THF
without the formyl group. The formyl group has been attached to the
methionine residue as shown in Figure 10-32. Because the amino group on
methionine is formylated, it is called N-formylmethionine-tRNA.

Elongation
70S Initiation Complex Once the 70S initiation complex is primed for synthesis, what happens?
Figure I O-3 t Suppose the next codon in our mRNA is 5'-CGA-3' as shown in Figure 10-30
and Figure 10-31. Looking at our genetic code we find that this codon codes for
the amino acid arginine (Arg).

Arginine is attached to the 3' end a specific IRNA by a specific aminoacyl-


tRNA synthetase and is then carried by this arginine-specific aminoacyl-tRNA
oHo to the empty A-site on the 70S complex. Next to the A-site is the P-site. Note
ll t il that the P-site is already occupied by fMet-tRNAf. This is shown in Figure 10-33.
H-C-N-C-C- O-rRNAi
tt
H CH]
t-
CH,
I P-site A-site
S
3',
I

cHr fMet

Formylmethioninyl-tRNA 1
(fMet-rRNAf)

Figure lO-32
JI
70S Initiation Complex

Figure lO-33
Ti

Copyright @ by The Berkeley Review 4oa The Berkeley Revieu C,.


Specializing in MCAT Preparation
Biology Expression of Genetic Information Protein Synthesis

once we have amino acids in both the p-site and A-site, the enzyme peptidyl
transferase wlrl catalyze the formation of a peptide bond. The amino nltrogen tr
Arg attacks (nucleophilic) the carbonyl carbon of the ester linkage in itvtet-
tRNAf. The result is cleavage of that ester bond and the formation Jf a peptide
bond between fMet and Arg. In other words, the fMet moiety of the p-site will
be transferred to the amino group of the Arg residue of the A-site. The formation
of the dipeptidyl-tRNA at the A-site is a favorable reaction.

The empty tRNAf will leave the p-site and the mRNA will move a distance of
three nucleotides thus bringing the dipeptidyl-tRNA (5'-fMet-Arg-3') to the p-
site. This is called translocation and can be seen in Figure 10-34. we are now left
rylth
tfe A-site empty and ready for the next incoming amino acid (specified by
the mRNA codon) during the second round of elongation.

5' - AUG
J
70S
ff:'

I A-site

Initiation Complex
mRNA

Figure lO-34

Energetics of Protein Synthesis


Consider the ATP equivalence involved in protein synthesis. Recall that each of
the amino acids has to be activated by binding to tRirlA. This process involves 2
ATP equivalences (one for the hydrolysis to get pyrophosphate and one for the
hydrolysis of pyrophosphate itself). rhe blnding"of in aminoacyi-tRNA to its
respective codon costs an ATP equivalence. In the translocation step we find that
no ATP equivalendOs are required for the first amino acid because i1 is already at
the P-site. Flowever, each additional amino acid will bind at the A-site first and
then translocate over to the P-site. This process will require l ATp equivalence
eachl If we add up the sum in terms of ATp equivalences, we will find that g are
required for the first amino acid and then 4 for each subsequent amino acid. This
is summarized in Table 10-3.

fMet aa2 aa3 Aan

Activation )))',
Binding 111t
Translocation 0111
ATP Equivalence

Table lO-3

Copyright @ by The Berkeley Review 409 The Berkeley Review


Specializing in MCAT preparation
Biology Expression of Genetic Information Protein Synthesis

Termination
suppose we are close. to the end of the 3' end of the mRNA. In Figure 10-35 is our
mRNA with some nth codon. Attached to the nth codon is a po-lypeptide chain
with some nth amino acid. This is all at the p-site. \Arhat we will find is that a
termination signal (e.g., UAG, uAA, or UGA) will come into view. Let's use 5'-
UAG-3'as our stop signal. These stop signals are not recognized by a IRNA but
instead are recognized by a protein release factor that will bind to the signal
when it comes into view. This release factor has an effect on peptidyl transferase,
the enzyme that forms peptide bonds between two amino acids. peptidyl
transferase will now use water instead of an amino group to attack the ester
linkage of the nth amino acid at the P-site. This wilirelease the polypeptide
chain.

Y''
Arg
P-site A-site

I
aa- aa-aa-aa

5' - XXX XXX_ UAG mRNA

70S Initiation Complex


P
Figure lO-35 :
gl
Puromycin ;
HN OH
You should be aware of the fact that inhibitors were useful (and continue to be)
I
o=c in working out the pathways of protein synthesis. one inhibitor that we can
;-r-.",/\o..n,
| \_,/
mention is puromycin, which is shown in Figure 10-36. Puromycin binds at the
A-site and acts as an analog of an aminoacyl-tRNA, thus preventing other
NH^
aminoacyl-tRNAs from entering the A-site. The q-amino group of puromycin
forms a peptide bond with the carboxyl group of the amino acid at the p-site.
Figure f 0-36
Puromycin
once this happens peptidyl puromycin leaves the 70s ribosome, thus causing
premature termination of protein synthesis.

Ft
iorl
::
b".r

:-
:c:

ra
_:l

.imc
:i1:

Copyright @ by The Berkeley Review 4to The Berkeley Revieu


Specializing in MCAT Preparatiom
Biology Dxpression of Genetic Information The Lactose Operon

The Lactose Operon


In bacteria the majority of regulation occurs at the level of transcription (i.e., to
make or not to make a particular mRNA). we will now consider the lactose
operon. what exactly is an operon? An operon is simply a transcription unit
which is involved in the expression of multiple genes. Another way to word this
would be that an operon is an arrangement of genes on the DNA which are
cotranscribed.

rn 1967 Francois jacob and jacques Monod proposed the operon model for the
regulation of protein synthesis from work they were doing on the metabolism of
the disaccharide lactose (glucose and galactose). Even though the bacterium E.
coli pteferc glucose as a substrate it can survive on alternate carbohydrates such
as lactose, galactose, and even arabinose. However, in order for E. coli to utilize
these other sugars it must synthesize a whole new series of enzymes. E. coli d,oes
not want to be synthesizing these enzymes unless a particular substrate, like
lactose, is in the medium.

Thus, E. coli can induce the expression of certain genes only when they are
needed. A priori one can imagine an inducible system working in two ways: (1)
If you have a system that is always on and then you tum it off or (2) if you have a
system that is always off and then you turn it on. This "turning on" and "tuming
off" can be mediated by a variety of different mechanisms.

We mentioned that an operon is simply a transcription unit which is involved in


the expression of multiple genes. For example, the lactose operon includes a
promoter (P), an operator (O), and structural genes (2,y, and a). Upstream from
the lactose promoter (on the 5' side of the lactose promoter) is the regulatory
gene (i) with its own promoter (P1). These elements of the lactose operon model
are shown in Figure 1,0-37. we will modify this diagram as the discussion
continues.

Regulatory Contlol
I gene I sltes r

# StructuratGenesJ

z Pr I P o z v a

+ Lactoseoperon
---+l
Figule tO.57
When the enzyme RNA polymerase binds to the lactose promoter it can begin to
transcribe the structural genes of the lactose operon. Structural gene z codes for
b-galactosidase whose major function is to hydrolyze lactose to glucose and
galactose (and whose minor function is to convert lactose to allolactose).
Structural gene y codes for galactoside permease, a carrier molecule that
transports lactose into the cell. Structural gene a codes for thiogalactoside
transacetylase whosdtole is still uncertain.

The regulatory gene (i) codes for a repressor protein that can bind to the
operator and prevent the transcription of genes z, y, and a when lactose (the
inducer) is not present in the cell's medium. In other words, the products of
those genes act on lactose in order to metabolize the disaccharide. If lactose is not

Copyright @ by The Berkeley Review 4tt The Berkeley Review


Specializing in MCAT Preparation
Biology Expression of Genetic Information The Lactose Operon

present,
-operator
why waste the cell's energy to synthesize those gene products? [An
is simply a DNA sequence that binds a repressor.]

The conclusion that Jacob and Monod came to was that there is a diffusible
component in the cell that is involved in turning down the exp,ression of a whole
series of genes which are all involved in the metabolism of lactose. The set of
genes thal was need.ed to metabolize lactose would only be turned on and then
Jynthesized when lactose was present in the medium. Those particular genes are
the structural genes (mentioned above).

Let's consider what happens when lactose is not present in the medium' When
lactose is not present, you do not want RNA polymerase to transcribe the
structural gurr", y, uttd a. The expression of these three genes are controlled by
",
the regulaiory gene (i). The regulatory gene codes for a repressor protein as
showti in Figure L0-38. This gene is transcribed separately from the structural
genes of the lac operon.

z Pr I P o z v a

mRNA .-A
s
o
Repressor
protein
ffi
Figure lO-34

Once the repressor protein is synthesized it can do one of two things' If lactose, is
not in the medium, the repressor protein will bind to a region in the DNA
sequence called the operatoi (O) and prevent RNA polymerase from transcribing
the structural genes. Th" t"ptestor protein binds tightly to the operatol region
as

shown in Figure L0-39.

RNA polymerase cannot Repressor protein binds


transcribe the structural genes to the operatir

I
Figure lO-59 I
g
Even though the RNA polymerase can bind to the plomoter site while the tr
repressor piotein is bound to the operator site, it seems that the rePressor protein
pievents transcription by RNA-polymerase because it interferes with the a
formation of the transcripiion bubbie. It turns out that there is an overlap
in the
u

The BerkeleY Review C


Copyright @ by The BerkeleY Review 412
Specializing in MCAT PreParation
Biology Expression of Genetic Information The Lactose Operon

binding of RNA polymerase and the repressor protein to the duplex DNA
molecule.

However, if lactose is present in the medium, then a small amount of it will be


converted to allolactose by a few molecules of B-galactosidase that are present in
the cell. This is shown in Figure 10-40.

HO-H2C HO_H)C

Ho-H2c "1- oyo-'L


^htrf' o o,
"9/-o, "^ry -p-""r"""'id"\Y
B-caractosidase

q"
au.,o..o" Arolactose
Q"" OH

Figure lO-4O

Allolactose will bind to the repressor and decrease the repressor's affinity for the
operator site. Thus, allolactose is the inducer of the lactose operon. once the
inducer binds to the repressor, the repressor (and inducer) dissociates from the
operator and RNA polymerase can begin to transcribe the lactose operon genes.
The polycistronic mRNA that is transcribed (mRNA which codei for two or
more polypeptide chains) will eventually be translated into the desired enzymes.
This is shown in Figure 10-41.

z Pi I P o z a
o v
+5efl\
/\
/ 1 lnducer-Repressor complex no longer
,/ @ has affiniiy Ibr rhe operaror regio-n
Allolactose
RNA Polymerase can transcribe
the lactose structural genes E:+

Pr I P o z v a
rnn\-t n \5)z

-ilNA
I U I transcrirrion
ruonc./
(Polycistonic)
gn n
+ ,[nT Translation
p-calactosidme
ffi
f{F.h
\iiii,t,
ffi ,,**
Q!y'acetyrre

Figure I O-4 I

It turns out that transcription of the E. coli lactose operon also depends on the
relative concentrations of glucose within the cell's medium. E. coli will utilize
glucose as the preferred eneigy source over lactose. Therefore, when glucose is
present in the medium there is reduction in the synthesis of the enzymes needed
-
to utilize lactose. This is called catabolite repression. However, when glucose is
absent from the medium, the rate of transcription of the enzymes needed to
utilize lactose is increased dramatically.

Copyright @ by The Berkeley Review 413 The Berkeley Review


Specializing in MCAT Preparation
Biology Expression of Genetic Information The Lactose Operon

i
zo R I CAP-
cAMP
P o z v a crp
gene
I
(

t
A decrease in 0
@
I

[Glucosel i
means an increase in i
s t
[cAMP]. i

CAP
e F
i
L

Figure 10.42 tl
T
A considerable distance downstream from the structural genes of the lactose C

operon is a gene [you do not need to know the name but if you are interested it is
the crp gene) that codes for a protein called the catabolite activating protein R
(cAP). CAP is sometimes called the cAMP receptor protein (cRp) as well. CAp -l
L]
is a DNA binding protein that has the ability to mediate catabolite repression of SN

many inducible operons, such as the lactose operon. CAp can only bind to DNA c;
when it is complexed with cyclic adenosine monophosphate (cAMp). cAMp is it
synthesized from ATP and increases in concentration when glucose levels are e)
low. when cAMP binds to cAP it forms the cAMP-cAP complex which can then fc
bind to the DNA at the CAP site, located right next to the lactose promoter on
the 5' side (upstream). This is shown in Figure 10-42.

RNA Polymerase can transcribe


AMP
C, P
.,\, the laciose structural genes €
\ s ,<<\\\\
zo Pr I CAP.
cAMP P o z v a crp
gene

n n :\\5>z
U + $ Trrnscription

(r-orycrstronrc)
+ O
ffi@@
$Translation
tl(

p-Galactosidase Permease Trans- Su


acetylase t'2
bir
Figure lO-43 tfc
gii
When the cAMP-CAP complex is formed it acts as an activator and binds to the rril
CAP-cAMP site. Transcription of the lactose operon is enhanced by somehow opf
b"
converting a weak lactose promoter -into a stronger lactose promoter. This is hai
shown in Figure 10-43.
ifr
Mutants chr
A normal wild type E. colibacterium would have a genotype i+z+y+a+. One of dip
the mutants that Jacob and Monod characterized had the ability to synthesize the diP
three proteins shown in Figure \0-43 at normal levels in the absence of the gaL
inducer. This type of mutation was called a constitutive mutant (meaning that dor
the genes {or those three proteins are always expressed and unregulated). The rep
mutant cells that they examined had a regulatory gene (the i gene) that had been chr,
altered. Instead of this regulatory gene having an i+ genotype it now had an i- ind

Copyright 3 by The Berkeley Review 414 The Berkeley Review cop


Specializing in MCAT Preparation
Biology Expression of Genetic Information The Lactos'e Operon

genotype. In other words, this mutant E. colibacterium would have the genotype
i'z+y+a+. These i-mutants can synthesize the three proteins either inthe presence
or the absence of the inducer whereas the i+ cells can only synthesize the three
proteins rn the presence of the inducer. Jacob and Monod were able to show that
the i gene mapped close to the structural genes.

A normal i+z+y+a+ cell can make the repressor protein. If lactose is present, then
the repressor protein will bind with the inducer. This repressor-inducer complex
will not bind to the operator and thus RNA polymerase will be able to transcribe
the structural genes. A cell which is i-z+y+a+ cannot make a functional repressor
protein. Therefore, whether the inducer is present or not does not matter because
there is no functional repressor to bind to the operator to prevent transcription.
Thus, the normal levels of the structural genes are always expressed. Hence, the
constitutive mutant.

Recall that we mentioned that bacterial cells such as E. coli have a large circular
chromosome. Besides this large circular chromosome they can also have a
smaller circular chromosome called the fertility factor (or F factor). The F factor
can exist in two states. It can either be free in the cytoplasm of the bacterial cell or
it can be integrated into the larger circular chromosome. Since this F factor can
exist in either of these two states it is referred to as an episome (a word coined
for precisely this characteristic). See Figure 10-44.

S--*
F
Host factor
chromosome Integrated F factor

E. coli

Figure lO-44

Suppose we have a situation on the host chromosome in which the genotype is


i+)n+u+.In this situation a normal replessor would be produced which would
bind inducer. The operator would no longer be occupied and the structural genes
would be transcribed. However, because there is a mutation in the lac z gene,
giving th€ z- genotype, the enzyme b-galactosidase will not be synthesized (i.e., it
will be defective). Suppose we have a situation on the F factor in which the
genotype is i-z+y+a+. In this case the repressor protein is non-functional, as we
have mentioned, the structural genes are continuously transcribed.

If we have both of these genotypes in the cell simultaneously (one on the host
chromosome and one on the F factor), then the cell acts as if it were a partial
diploid. This is indicated as i+z-/i-z+ as shown in Figure 10-45' When this partial
aiplola was analyzed it rras found to be inducible. In other words, the protein b-
galactosidase was being synthesized. This meant that the normal i+ allele is
dominant to the recessive i- allele. If there is no inducer in the cell, then the
repressor protein from the i+ gene binds to the operator regions of both
chromosomes. Synthesis of p-galactosidase does not occur. However, if the
inducer is present, then the repressor protein produced by the i+ gene and

Copyright @ by The BerkeleY Review 415 The BerkeleY Review


Specializing in MCAT Preparation
Biology Expression of Qenetic Information The Lactose Operon

inducer bind and dissociate from the operator. In this case p-galactosidase can be
made. It was this type of genetic analysis that led ]acob and Monod to
hypothesize the presence of a repressor protein.

l- z+
i+ .$\\
z
\-$
F
Host factor
chromosome

Figure lO-45
Partial diploid

Mutations were also discovered in the operator region of the lac operon. These
mutants were referred to as operator constitutive mutants (or Oc mutants) and
they allow for the continual transcription of the lac structural genes. Why? If
there is no inducer present, then the repressor would be free to bind to the
operator region. However, since there is a mutation in the operator region the
repressor cannot bind. Therefore, RNA polymerase can transcribe the structurai
genes.

Consider a partial diploid which has the genotype i+O%+/i+O+z+ (which we can
make by using an F factor). V\4:rat happens in the absence of the inducer? In the
absence of inducer the repressor protein that is synthesized cannot bind io the Oc
site of the first chromosome because it has been mutated. Thus, RNA polymerase
can transcribe the structural genes. However, the repressor protein can bind the
O+ site of the other chromosome and thus prevent transcription. This says that
the Oc mutation is dominant over the wild type O+ situation. In other words, in
the absence of the inducer there will be a constitutive synthesis of the gene
products in a cell which is Oc/O+.

The Oc mutants are said to be cis-acting mutants while the i- mutants are said tc
be trans-acting mutants. A trans-acting situation refers to a product of a gene
that is diffusible. For example, in the case of the i- mutants we saw that the gene
product from the i+ gene of the partial diploid i+z- /i'z+ was able to diffuse or-er
to the mutant chromosome and bind to the operator in the absence of inducer-
The structural genes were not synthesized. A cis-acting situation is characteristic
of a gene that affects only the genes juxtaposed to it. In other words, the Oc
mutation is felt only on that chromosome and not on the chromosome with the
wild type O+ gene. This says that for a cis-acting situation there is no diffusible
product. This makes sense because the operator region of the lac operon does nst
produce any mRNA or protein that can diffuse anywhere.

Copyright @ by The Berkeley Review 416 The Berkeley Revier


Specializing in MCAT Preparatiom
Biology Expression of Genetic Information The Tryptophan Operon

ffi*ii fi 6.$$ffih
How can transcription be regulated at the level of termination? what are
terminators? There are two types of termination that occur tn E. coti. The first is a
spontaneous termination, referred to as rho (r) independent termination; the
second is a factor dependent termination, called rho dependent termination.

Anti-Sense
f I
I coo;ng Strand
l
I
DNA I
=:!
OUDlexl ===
l===
3' .N -N-N .N-N-N-N-N.N

f
+
s"n." I
il:iP
Hairoin Loon
j ft\
lr.-o,"lir,.o,o fottowed by a
series of U's.
Nis tl
any nucleotide NC
specific to DNA or RNA (\

Figure 1o,46
Rho Independent Termination.

Rho independent termination usually occurs when RNA polymerase recognizes


a looped hairpin structure followed by a run of U's (Figuri ro-ee;. Note th! high
GC content in the stem of the loop. When this hairpin structure forms it appeirs
to jam the transcriptional process and causes the RNA polymerase to staillor a
few seconds. The result is dissociation between the deoxy-A residues in the
templaie strand of the DNA and the ribo,u residues in the mRNA. These two
bases form a relatively weak hybrid and are therefore not too stable.

Rho dependent termination occurs in the presence of a hexameric protein


referred to as the rho;factor. This enzyme is actually a helicase than can iatalyze
the unwinding of RNA-RNA and RNA-DNA double helices. This enzyme
recognizes an 80 to 100 nucleotide sequence on the nascent nRNA thit is
upstrea;n from the termination site. Rho factor binds to this recognition site and
moves along the mRNA (5'-+ 3') until it finds an RNA polymerase that has
paused. It then begins to unwind the RNA-DNA duplex and terminates
transcription.

During the early 1950's Charles Yanofsky began to study the regulation of
tryptophan synthesis by looking at the genes of the tryptophan operon
(abbreviated as trp operon). He began by isolating two types of mutanti. one
type of mutant involved structural gene mutations. These types of mutants were
auxotrophic for tryptophan. These mutants required trypiophan for growth as
they were unable to synthesize this amino acid. yanofsky developed a map for
these genes as shown in Figure 10-47. The trpE, trpD, trpC, trpB, and trpA genes
coded for a polycistronic message which gave rise to the enzymes needed to
convert the precursor molecule chorismate to tryptophan.

Copyright @ by The Berkeley Review 417 The Berkeley Keview


Specializing in MCAT Preparation
Biology Expression of Genetic Information The Tryptophan Operon

Attenuator
I
z
n rrpR trpPrO rrpl ffi trpE trpD trpC trpB trpA

z'*
0 Transcription n
\z
<t
J

u
@
Repressor
Translation
I //
Anthranilate Phosphoribosyl 3-glycerol
Anthranilate phosphoribosyl anthranilate phosphate Tryptophan

Chorismut. J%-%
Figure lO-47
The other type of mutant Yanofsky obtained was a regulatory mutant. These
mutants were able to constitutively synthesize the enzymes necessary for the
biosynthesis of tryptophan. somehow they were altered in their ability to
regulate the expression of the tryptophan structural genes.

In other words, the frpR gene, which codes for the tryptophan repressor, is not
effective in regulating the synthesis of tryptophan. The gene for frpR mapped in
another quadrant of the E. coll chromosome, at about 100 map minutes, while the
trp operon mapped at about 28 minutes. Yanofsky purified the dimeric frp
rePressor protein and discovered that it does not function alone. In order to
regulate the synthesis of tryptophan the repressor must bind the end product of
that metabolic pathway. The end product is tryptophan.

Therefore, tryptophan acts as a corepressor for its own biosynthesis. When the
concentrations of tryptophan are high, the repressor binds to tryptophan and this
repressor-tryptophan complex binds to the operator region of the trp operon.
When this complex is bound to the operator it prevents RNA polymerase from
initiating transcription of the structural genes. This is an example of feedback
repression at the transcriptional level.

In contrast, if the concentration of tryptophan is low in the cell, the repressor-


tryptophan complex wili not form and therefore will not bind to the operator.
RNA polymerase is able to transcribe the structural genes and tryptophan will be
synthesized.

Around 1975 Yanofsky isolated more mutants in the trp operon and came up
with some results that could not immediately be explained. Upstream from the
start of the structural genes and downstream from the operator is a 15 2
nucleotide sequence of DNA called the leader sequence. This sequence is coded
for by the trpL gene. Yanofsky had isolated trp deletion mutants in the leader
region that resulted in a 8 to 10'fold increase in the expression of the structura-
genes of the try operon. How could this be explained?

Towards the 5' end of the 162 nucleotide leader sequence is a region of 14 amino
acids called the leader peptide. The 1Oth and 11th amino acids of this leade:

Copyright @ by The Berkeley Review 4ta The Berkeley Reviex'


Specializing in MCAT Preparation
Biology Dxpression of Genetic Information The ftyptophan Operon

peptide are tryptophan residues. These two amino acids are also part
of a
nucleotide sequence that is GC rich. There are four GC rich regions in the
leader
sequence. They are sometimes called region 1, region 2, regio"n 3, and
region 4.
This is shown in Figure 10-48. Becaur" th"r" GC iegions ar"e complemenlary
to
each other, we find that two alternative ,".orlury structures resembiing
hairpins can be formed.

one_secondary structure involves base pairing between region 1 and region


2
and between region 3 and region 4. Note that ihere ur" u r"iiu, of ribo-u'i
after
the hairpin formed regions 3 and 4. This represents a rho-independent
_by
termination site. The other secondary structure involves base pairing just
between region 2 and region 3. This base pairing sequence will only oc"cur
if
region 1 is not available for base pairing with region 2. Note that G=C base
pairing is rather stable (as opposed to A=T base pairing).

we mentioned that when the concentration of tryptophan is high the repressor


protein binds to tryptophan and forms the represior-tryptopha-n compl&. This
complex binds to the operator and prevents trinscriptio.t. f in" concentration
of
tryptophan is low, then the repressor-tryptophan comprex does not form, the
operator is left open, RNA polymerase binds to the promoter, and transcription
of the structural genes occurs.

Leaderpeptide //

,,

U.U-U.U-3'

U-U-U-U-3'

Figure lO.4A

Let's examine transcription of the leader region in a little more detail. Suppose
that the cell has a sufficient level of tryptophan. As RNA porymerase transiiibes
region 1, and region 2 these two regions begin to form a hairpin structure that
causes the polymerase to pause. By this time the shine-Dilgarno sequence
(towards the 5' end oJ the mRNA) has been mad.e and a riboJome binds and
begins translation (which we will discuss in more detail in a future lecture). As
the ribosome translates the leader peptide region it disrupts the hairpin created
between regions '1, and 2"

RNA polymerase then begins to transcribe region 3. Meanwhile, the ribosome


reaches the tryptophan codons and inserts the required tryptophan amino acid.s

Copyright @ by The Berkeley Review 419 The Berkeley Review


Specializing in MCAT preparation
Biology Expression of Genetic Information The Tryptophan Operon

(supplied by a charged tryptophanyl-tRNATTP molecule) into the growing


polypeptide chain. As the ribosome moves downstream a few more codons it
encounters a termination codon and stops translation. The ribosome dissociates
and the hairpin between regions 1 and 2 reform. As RNA polymerase finishes
transcription of region 4 a hairpin structure forms between region 3 and region 4.
This halrpin is a termination signal because of the ribo-Us that immediately
follow region 4. Once this termination signal is formed the transcription complex
dissociates from the duplex DNA and the structural genes are not transcribed.
Tryptophan will not be synthesized.

suppose that the levels of tryptophan in the cell are quite low. It should be
obvious that the cell needs to synthesize more tryptophan. Initially the RNA
polymerase and the ribosome follow the same sequence of events we just
mentioned but with one important difference. When the ribosome reaches the
tandem tryptophan codons it pauses because the concentrations of tryptophan in
the cell ui" q"it" low. There are not enough charged tryptophanyl-tRNArrP
molecules to bring tryptophan to the site of protein synthesis. This results in the
ribosome sitting on region '1, and coaering it up. Meanwhile, RNA polymerase has
transcribed region 2 and region 3. Since region l is not available for hairpin
formation with region 2 it turns out that region 2 will form a hairpin structure
with region 3. By this time RNA polymerase has transcribed region 4 and the
string of ribo-U's.

Region 3 will not base pair with region 4 because the hairpin structure formed
beiareen region 2 and region 3 is more stable than the hairpin structure that could
be formed between region 3 and region 4. This means that the termination
hairpin is not formed. The transcription complex is not disturbed and RNA
polymerase transcribes the structural genes of the trp opeton. This type of
iegulation of the synthesis of mRNA is referred to as attenuation and the control
element which is responsible for this phenomenon is called an attenuator.

With this information in mind we can return to the question we posed earlier.
Why would mutants in the leader region have 8 to L0 fold levels increased
tryptophan biosynthesis? The deletion mutants that were made had a deleted
terminator sequence. If this occurs, tryptophan will be synthesized.

\n/hy bother with two regulatory systems (i,e., the repressor-tryptophan complex
and attenuation)? The repressor system controls the level of the tryptophan
enzymes some 70 fold. Attenuation gives an 8 to 1,0 fold range of expression. The
gives a 600 to 700 fold range of control. In other words, the control is
tongnty"TIie
Uroia. tryptophan operon began to explain other data that researchers could
-f
not interpr"f. oi example, a repressor could not be found for the histidine
operon, the phenylalanine operon, the leucine, threonine or valine operons. After
the discovery of ine tryptophan operon and its regulation the leader regions of
these other operons *ete s"q,tenced. It was discovered that all these leader
regions were rich in the amino acids which they controlled'

The histidine operon had a leader region that contained 7 tandem histidine
codons. The phenylalanine oPeron had a leader region that contained 7
phenylalanine codons. These leader regions also contained alternative hairpin
ittrr.in."t which could act as termination sequences. In some cases there was
multivalent repression. There were several end products from some of these
pathways und *or" than one amino acid could feedback to regulate
transcription.

Copyright @ by The BerkeleY Review 420 The BerkeleY Review


Speciatizing in MCAT PreParation
Biology Expression of Genetic lnformation Mutations & Proofteading

Mutations & Proofreading :: ::: , , ,

The most common type of mutation is where there is substitution of one base
pair for another base pair. A transition mutation occurs when one purine is
replaced by another purine or one pyrimidine is replaced by another pyti-iai.,".
A transversion mutation is simply the replacement of a pyrimidin" ty a purine
A-T T-A
or a purine by a pyrimidine. These mutations are summarized in Figure 10-49.

Tautomerism
How can transition mutations occur? Watson and Crick noticed that certain
hydrogen atoms on each of the four bases can change their positions thus giving
c-G c-c
Transitions
rise to tautomeric structures. For example, consider the pyrimidine base thlmini
(in the lactam form). Under normal conditions thymine will bond with adenine
as shown in Figure 10-50.
A-T <-> T-A
H
A
\N rH tttttttttttttrtttO. I

N
/q \\\
\t c
(, rtrrrrrrrrrrrrfl -N-l C-c<-> c-c
N.
Backbone
O
Fr, Backbone
Transversions
Figure 1o,49
Adenine Thymine
(lactam form)

Figure lO-5O

However, if the hydrogen atom on the nitrogen at position 3 in the thymine ring
moves to the oxygen atom at position 4, then we have a rare form of thymine (ttre
lactim form) that is capable of hydrogen bonding with guanine as shown in
Figure 10-51. The fraction of thymine in this rare form is about 10-4.

N
(z
\r
Bac kbone

Gu
.t
anrne H Thymine
(lactim form)

Figure lO-51

Suppose we have a duplex of parental DNA, as shown in Figure \0-52a, in which


we focus on a particulaf A-T base pair. Each strand is labeled '1, and2so we can
follow them during replication. The DNA duplex goes through the first round of
replication as shown in Figure 10-52b. Parental strand 1, successfully gives rise to
a daughter strand (labeled 3). However, as parental strand 2 is being replicated
our thymine base tautomerizes into the rare lactim form of thymine (indicated by
T*). In the nascent daughter strand (labeled 4) a guanine will now be inserted
instead of an adenine.

Copyright @ by The Berkeley Review 42t The Berkeley Review


Specializing in MCAT Preparation
Biotogy Expression of Qenetic lnformation Mutations & Proofteading

During the second round of replication, as shown in Figure '1,0-52c, we will get
two normal DNA duplex polymers from the duplex with strands 1 and 3. Since

(a)llA-T our rare tautomeric form of thymine does not last that long we find that during
the second round of replication it returns to the normal lactam form. After the
DNA duplex with strands 4 and 2 go through the second round of replication,
we will get one normal DNA duplex polymer, indicaied by strands 2 and 8,

I12 lm* and one mutant DNA duplex polymer, indicated by strands 4 and7. What we
have done is gone from a normal situation in which the A-T base pairs in
strands '1. and 2 have spontaneously mutated (transition mutation) to a G-C

(b),-# base pair as shown in strands 4 and7.

lt
A=T
I
G:T *
Proofreading via 5'-+ 5'Exonuclease Activity
Since the fidelity of the genetic message needs to be preserved in order to
proliferate the organism, there are methods of detecting mutations like the
ones that we have just mentioned. Both DNA polymerase t and D N A

il
13
il
42
polymerase III possess a 3' -+ 5' exonuclease activity.

Suppose we have a hypothetical template strand composed of adenine bases as


shown in Figure 10-53. During our replication process we are incorporating
c,AReericatio{ thymine bases into the growing nascent daughter strand. A mistake occurs and

lrl I il
a rare tautomeric form of cytosine is brought in and becomes part of the nacent
DNA chain (because it looks like thymine).

DNA polymerase is continually checking the segment of new DNA that it has

llll I I
A-T A-T G=C A=T just synthesized. If it detects an error, such as a cytosine where there should be
a thymine, then that error will be removed by the 3'-> 5' exonuclease activity
of the enzyme. After the correction is made the polymerase will continue with
1563 47 82 the replication of the DNA template. This proofreading mechanism reduces the
tt tt eror rate by about a thousand times. Cells with high mutation rates have a
Normal Mutation Normal defective DNA polymerase (III) exonuclease activity.

Figure 1o,52
Cytosine will be removed
with 3'to 5' exonuclease
activity.

5' -T-T-T1 T.T. {",


il il il( ilil
A. A. A-
ilt
A -A-A-
+
3' -A-A-AJ 5'

DNA Polymerase III

Figure f O-53

Deletion
Suppose in our hypothetical template strand composed of adenine bases one of
those bases slips out of register as shown in Figure L0-54. As the nascent DNA
strand is being synthesized a thymine base will not be incorporated at the poini
of slippage (or tooping out) of the adenine base. The result is that there will be
one less thymine base in the nascent DNA strand. in other words, there has
been a deletion of a base.

Copyright @ by The Berkeley Review 422 The Berkeley Review


Specializing in MCAT Preparation
Biology Expression of Genetic Information Mutations & proofreading

Deletion of base

.,
s'-T-r-T+T-T-T_3, t^ \
3'
il il il (il A-.A,-
- A-A- A\
il il A- )*
./{: A-,r5',
Stippage--| { \_/
of base DNA polymerase III

Figure l0-la'4

Insertion
suppose, once agaia, in our hypothetical template strand of adenine bases there
a slippage of a thymine on the nascent DNA strand as shown in Figure
tt_!"9
10-55. This will result in an additional thymine base being added to the gro#ing
DNA strand. In other words, we have an insertior oiu base. sequerices liki
these, where you have a run of the same base, are hot spots for length mutations
(e.g., either deletions or insertions). H,N

^, \
Insertion of base
tshl
O Backbone

5' Cytosine

---+ o.u'inution
3'
l|
DNA Polymerase III

Figure lO-55 "_:h


\ r/
l- *,
Deamination O Backbone

Recall that the bases cytosine, adenine and guanine have amino groups on them Uracil
that can be hydrolyzed. Roughly 5,000 amino groups are lost fiom ihese bases
per cell per day. The example that we will consider is cytosine being deaminated Figure 10.56
to form uracil as shown in Figure 10-56.

uracil is an analog of thymine. It base pairs just like thymine. If a cytosine were
to be deaminated to form uracil in the template strand of DNA, then the
polymerase would put in an adenine at the corresponding position on the
nascent DNA strand instead of a guanine. This is another way in which
transition mutations can arise.

Howevet, the cell has a repair system that recognizes these uracils and removes
them. Consider the DN*{ duplex shown in Figure 10-57 and the cytosine that has
spontaneously deaminated to form uracil. The enzyme uracil-DNA glycosidase
hydrolyzes the N-glycosidic bond between the deoxyribose ring und tn" uracil
base. The uracil base is removed.

Copyright @ by The Berkeley Review 423 The Berkeley Review


Specializing in MCAT preparation
Biology Expression of Genetic Information Mutations & Proofteading

.,H
--) 5'

Ucrt G

Figure lO'57

This site on the DNA duplex is called an AP site (either apurinic or apyrimidinic)
because it is without either purine base or a pyrimidine base. This baie defect is
recognized by an enzyme called AP endonuclease which cleaves the bond on the
3' side of the phosphodiester bond of the nucleotide with the missing base. DNA
polymerase I cleaves the phosphodiester bond at the 3'end on the next
nucleotide unit via its 5'-+ 3' exonuclease activity. This is shown in Figure 10-5g.

-5'

CutbyAP CutbyDNA
endonuclease Polymerasel

Figure lO-54

Once the defective ribose-phosphate unit is removed, DNA polymerase I will


read the complementary strand and find the base that corresponds to the AP site.
In this case it turns out to be a guanine. This is how DNA polymerase I knows to
insert a cytosine at the AP site on the damaged DNA strand. DNA ligase will
then seal the inserted cytosine into the damaged strand in the usual fashion.

Mismatch Repair
The most common types of mutations are caused by tautomerism and slippage
during replication. suppose we already have a DNA duplex that has already
been replicated. During this replication we had a tautomerism in which a
thymine momentarily looked like a cytosine. The result was that a guanine was
incorporated into the duplex rather than an adenine. Let's also say that our
proofreading system failed to catch this error. DNA polymerase might leave
about one mistake per 108 replicated base pairs. Because or trur impeifect base

Copyright @ by The Berkeley Review 424 The Berkeley Review


Specializing in MCAT Preparation
Biology Expression of Genetic Information Mutations & Proofreading

pairing (G-T) there is a slight bulge in the DNA duplex at that point. This is
shown in Figure 10-59.

Mismatch Repair Enzyme


finds the bulge and then
Newly synthesi zed searches for the GATC box
DNA strand

,l a CTAG_
GATC-
t
Template DNA
I
CH:

Strand

Figure lO-59

DNA duplex feeling for bulges like


The mismatch repair enzyme slides along the
the one we just mentioned. Once this enzyme finds one of these bulges it does
not know which base was incorporated into the replicating DNA in error. Is it
the guanine or is it the thymine?

The enzyme needs to make a decision. It scans the duplex DNA for help. What
the enzyme is searching for is the palindromic sequence of bases G-A-T-C. This
is referred to as a GATC box. This box will tell the er.zyrr.e which is the newly
synthesized DNA strand. How?

The template strand (the parent strand) turns out to have the adenine base
methylated in each of these GATC boxes. The methyl groups are attached to
adenine by SAM (S-Adenosylmethionine). This process of methylation is a signal
that identifies the old DNA strand from the new DNA strand. The newly
synthesized DNA strand is not methylated until a few seconds or minutes after
the replication process. This gives the mismatch repair enzyme plenty of time to
locate these errors.

Once the mismatchJepair enzyme finds a bulge in the DNA duplex, how far
does it have to travel before it will find a GATC box? The probability is (ll4)4.
The probability that a particular base at a particular position is, say, a guanine is
1/4. The probability that a guanine will have an adenine right next to it is
(1,14\(1.14). Therefore, the probability that there will be a sequence GATC is
(114)(1t4)('t 14)(1/4) or (114)a or 't 1256.

This means that the enzyme will need to more along the DNA duplex about 256
residues before it will randomly encounter a GATC box. (Note that this is not
quite correct. What we have done is to calculate how far apart the GATC boxes
are from one another on a duplex of DNA.)

We now have a *uy oi,"lllr-rg which is the correct base and which is the incorrect
base. Once the incorrect base is located, the mismatch repair enzyme binds to the
unmodified GATC box (with the non-methylated adenine) and to the incorrectly
inserted base as shown in Figure 10-60a.

Copyright O by The Berkeley Review 425 The Berkeley Review


Specializing in MCAT Preparation
Biology Expression of Qenetic Information Mutations & proofreading

(b)

CTAG-
GATC-
I I
CH: CH:
Figure l0.60

The mismatch repair enzyme then removes the mispaired base and the
intervening sequence of DNA up to the point of the unmeihylated GATC box as
shown in Figure 10-60b.

DNA polymerase (I) will then resynthesize the DNA that has been removed by
the mismatch repair enzyme and will replace the mispaired base with the correct
base, which in this case would be a adenine. DNA ligase will then catalyze
the
formation of a phosphodiester bond between the r"triuini.,g 3'-hydroxyf at one
end,of the replaced DNA strand and the 5'-phosphate"of the previously
synthesized DNA strand. This is shown in figurJfO-ei.

Template DNA DNA


Strand

t
--+
Itil
DNA Ligase will lorm
phosphodiester bond
CTACF__.
GAT C:--'
I
CH:

Figure lO.6l

Mutagens
Certain types of external agents can not only cause mutations but can also
increase the possibility of mutations. These agents are called mutagens. Three
types of mutagens that we will briefly consider are base analog, chemical, and
ultraviolet mutagens.

Base Analog
A base analog mutagen can easily substitute for a naturally occurring base in
DNA. For example, the compound S-bromouracil is a base analog of thymine.

Copyright @ by The Berkeley Review 426 The Berkeley Review


Specializing in MCAT preparation
Biology Expression of Genetic Information Mutations & Proofreading

This can be seen in Figure '1.0-62. Even though the bromine group of 5-
bromouracil is more electronegative than the methyl group of thymine, the
methyl group and the bromine group have about the same rran d"r waals radius.
One of the possible base pairs that results after replication from the substitution
of S-bromouracil in DNA is 5-bromouracil-adenine. After a subsequent round of
replication occurs, a transition mutation would result.

O CHi OBr

H
)<
-Nt-t ) "_.H
O
l-*, '
Backbone O
>'.( Beckbone

Thymine 5-Bromouracil

Figure 1o,62

Chemical (Nitrous Acid (HNO2))


Roughly 5,000 amino groups are lost from the bases cytosine, adenine and
guanine per day. Nitrous acid can cause the deamination of cytosine to uracil as
shown in Figure 10-63. Cytosine wants to base pair with guanine. However, if
cytosine is deaminated to uracil, then uracil will have a tendency to base pair
with adenine. The result is that a transition mutation would occur.

Cytosine Uracil

Figure 10-63

Ultraviolet (UV) Radiation


If two pyrimidine ltsidues such as thymine are adjacent to one another, then the
energy stored in uv radiation (at a wavelength of about 260 nm) can link them
together as shown in Figure 10-64. Thymine dimers inhibit DNA replication.

\!r
o

Sugar
- N ii

\>
Phosphate

Sugar
-
O

N
Thymine Dimer

Figure 10-64

Copyright @ by The Berkeley Review 427 The Berkeley Review


Specializing in MCAT Preparation
x
Biology Expression of Genetic Information Mutations & proofreading

Pyrimidine dimers can be corrected by a repair mechanism. A specific


endonuclease enzyme (called uvrABC) wlil cut on both sides of the dimer and
remove a segment of about 12 nucleotides as shown in Figure 10-65.

Thymine

I oamageo ow A is removed by enzyme


+

rttw ll
--TRT-

DNA Polymerase I fills in the


missing nucleotides and is
sealed by DNA Ligase

Figure l0-65

The resulting gap is filled in by DNA polymerase I and eventually sealed by


DNA ligase.

Intercalators
The tricyclic benzenoid hydrocarbon called anthracene, which is shown in Figure
'l'0-66, is the basic planar structure for many
ring systems which can intercalate
between adjacent base pairs in the DNA molecule. These mutagens are about the
Anthracene
size of a base pair, making them quite capable of slipping in between ad.jacent
Figure 10-66 base pairs in DNA and causing frame shift mutations (e.g., either the insertion
or deletion of one or more base pairs) during the replication process. If it were
not for the repair systems that we have been discussing, an orginism would soon
die due to the various lesions that can be introduced into DNA.

Copyright O by The Berkeley Review 42a The Berkeley Review


Specializing in MCAT Preparation
Biol ogy Expression of Genetic Information The Ames Test

The biosynthetic pathway for histidine requires ten enzymatic steps. Even
though histidine is an essential amino acid for humans, bacteria like E. coli and
Salmonella typhimurium are quite capable of synthesizing it. In these bacteria each
of these ten enzymes is encoded for by a gene and all of these genes are clustered
together in an operon, which is essentially a set of functionally related genes. we
can simplify the histidine pathway as shown in Figure 10-67.

Possible point
of mutation

Enz I Enz 2 Enz 3 Enz4 Enz Enz


A r----) B ----1C ----1D r---) E r---1 5
F -aG
6

n
nnz z
ll
Enz I0 Enzg EnzS U
Histidine (::- J €::- I €==H
Figure 1o,.67

If there is a mutation in any one of these genes, then these bacteria will no longer
be able to make histidine. If they cannot make histidine, their growth will stop
because of the requirement of histidine in many metabolic pathways. For
example, histidine is a five-carbon amino acid which can be degraded to o,-
ketoglutarate which can then enter into the Krebs cycle. If histidine is required
for'growth, then those organisms are referred to as being His-. Such an orginism
is auxotrophic (it needs something for its growth). If the organism has no
problem in synthesizing histidine, it is referred to as being His+. Such an
organism is prototrophic (it can make what it needs).

Ames started with a gram negative bacterium that was a histidine auxotroph
(His-). These mutations are very often transition point mutations. The transitibn
mutation not only has the possibility of being created from a normal gene, but it
also has the possibility of being reverted from a mutant form back to a normal or
wild type form (e.g., Sir*). The reversion of a point mutation back to a normal
base pair can occur by way of a transition mutation. A transition mutation is
simply the replacement of one pyrimidine by another pyrimidine or one purine
by another purine (e.g., the base pair A-T for G-C). A transversion mutation,
however, involves the replacement of a pyrimidine by a purine or a purine by a
pyrimidine (e.g., the base pair A-T for C-G). This is the basis of the Ames Test. It
examines the efficiency of conversion of a His- cell (an auxotroph) to a His+ cell
(a prototroph) that can grow on a minimal plate without histidine.

A lawn of bacteria that are His- are spread out to confluence on a petri plate that
lacks histidine. The His- bacteria settle down and may go through one or two cell
divisions before they s{op growing. They need histidine to grow. A test spot
containing a chemical that you suspect is a mutagen is then applied in the middle
of the plate. If this chemical causes mutations, then with some frequency it will
revert the His- auxotrophic mutations back to His+ prototrophs. If this happens,
then colonies of bacteria will emerge in the vicinity of the test spot. This can be
seen in Figure 10-68. The potency of the mutagen is roughly proportional to the

Copyright @ by The Berkeley Review 429 The Berkeley Review


Specializing in MCAT Preparation
Biology Expression of Genetic tnformation The Ames Test

size of the ring of colonies that forms around the test spot. Ames has been able to
show that chemical mutagens have a high probability of also being carcinogenic.

Test Spot
(suspected mutagen) Revertant colonies
in vicinity of
mutagen

$@ f
His'

Plate with minimal medium


(no Histidine)

Figure l0-64

If we are providing so much mutagen that there might be a chance that new
mutations will be introduced into the same bacterium that has sustained a
reversion event, then we might lose that bacterium if the new mutations also
inactivate one of the 10 enzymes required for, say, histidine biosynthesis. This
capacity is diminished by the fact that (1) mutations that are used to measure
reversion frequencies are particularly easily mutable and revertible. In other
words, they are hot spots (i.e., the context in which the mutation is located
readily recruits mutagens). The effective dose of mutagen detected by this test is
such that (2) the reversion event is one of very few mutations that would be
caused by a mutagen. We don't want to place so much mutagen in your test spot
such that we create lots of mutations in each bacterium. We only use a little
mutagen so that we are detecting relatively small numbers of mutations in each
bacterium.

Dilute region Area of high mutagenic

I
Q.: His'
il
,::ttra:t .::

:o',

Plate with minimal medium


(no Histidine)

Figure fO-69

If we were to use a high concentration of the mutagen, then that area


immediately around the test spot would be void of any revertant colonies. This is
because we are creating other mutations that prevent the revertants from
growing. As the mutagenic agent moves out and away from the point of
application, its concentration decreases (i.e., it becomes more dilute) as shown in
Figure 10-59. If we compare Figure 10-68 with Figure 10-69, we will find that the
concentration of the mutagenic agent in Figure 10-58 is not as concentrated as the
mutagenic agent in Figure 10'69.

Copyright O by The Berkeley Review 43o The Berkeley Review


Specializing in MCAT Preparation
Biology Dxpression of Genetic Information

" Efigiildb Cloning


Let's consider some of the technology involved in analyzing and
altering genes.
Recombinant DNA technology (or genetic engineering)not"o'ty dependi
In the
restriction enzymes and the modificatiorl ryrL*r fou"na in bacteria but
also on
the enzymatic processes in specific proteins that are designed to
-inherent
synthesize and process the DNA and RNA found in both eukaryofes and
prokaryotes.

Recall that we have mentioned that proteins could be cut into smaller
fragments
by using enzymes such as or chymotrypsin. It wasn't until the earti, tozos
that this application could-trypsin
be appried to nucleic acids. Restriction enzymes
restriction endonucleases) recognize specific sequences in a DNA polymer 1or
and Phage particles create
then cleaves that DNA at those sites. For example, bacterial cells
have a variety of a clear plaque in lawn
restriction enzymes which can degrade the foreign DNA of an attacking
virus.
The bacteria's restriction enzymes do not degrade-its own DNA
because its DNA
h1.b::. methylated at specific locations b"y modification enzymes. when the
viral DNA is injected into the bacterial cell its DNA has not been methylated :.ii:::.iiiii...,i

yet
and so the bacteria's restriction enzymes recognize that DNA as "
being foreign.

Ba.cteria can be Petri dish with agar and a


grown on a sorid surface of nutrient agar in a petri dish. This
solid medium is called a plate. If you add about 108 E. colibacteria confluent lawn of E. coli
to your plate,
they will eventually form a "lawn" of bacteria that will cover the Figure lO.7O
surface of the
nutrient agar._ if a phage infects a bacteria in this assay, then that infected
bacterium will lyse and release more phage progeny which in turn will
infect
other bacterial cells. After several hoursLf ^rep"eated infection by the
phage
progeny/ a clear, circular region will form in the lawn of bacteria. ThiJ
cleir
region, as shown in Figure 10-70, is called a plaque. The efficiency
of plating
(EoP) for most bacterial viruses is about 1. The Eop i, that fraction
of pnagl
progeny that can form a plaque.

l,-Phage

RI Plasm id

a-
+
e-{::,,\-_/
Lawn of E. coli Lawn of E coli with
RI Plasmid

Figure lO-71

In 1953 it was discoveled that if you try to grow a bacterial virus on a new strain
of bacteria, that virus will run into trouble. The bacteria virus called lambda (l)
p-hage had been grown on a standard E. coli strain. The Eop on these plates was
about 1. If you took phage from a plaque and tried to grow it on an E. coli strain
carrying an RI plasmid that was known to have drug resistance genes (the "R"
stands for drug resistance), then the Eop decreased to 2 x 10-3. In other words,

Copyright @ by The Berkeley Review 431 The Berkeley Review


Specializing in MCAT preparation
Biology Expression of Genetic Information Genetic Engineering & Cloning

1bo,ut
1in 500 virus particles would make a praque. This is shown in Figure l0-
71. It could be shown that most of the l-phage DNA had been degraded.

If_you take a phage from one of those 1in 500 plaques that could be formed and
plate it on the same plate, then the Eop would be 1. If you plated a phage from
that plaque back on the original strain of E. coli, then the Ebp would alio be 1.
Frowever, if you now take one oJ these plaques and replate it on the plate with
the E. coll stain carrying the RI plasmid, you will once again get an Eop of about
2x'!,0-r. This can be seen in Figure 10-72.

l.-Phage EOp = I

Lawn of E. coli
EOP=1in500

Figure 1o,72

Initially the phage were restricted in their growth but later became modified in
order to grow on the E. coli strain with the RI plasmid. If these modified phage
grow for a number of generations on the E. coli strain without the RI plasmii,
EcoRI they will eventually lose their modification and once again become restricted.
Thus, it was thought that the RI plasmid codes for rortre gerr"s that have the
Cut
ability to restrict and modify phage DNA.
o
5'-pG-pA-pA-pT-pT-pC-- 3' Herbert Boyer (at UCSF) purified the RI plasmid restriction endonuclease. The
3'--Cp-Tp-Tp-Ap-Ap-Gp--5' sequence of DNA that is recognized by the Eco RI restriction endonuclease is
shown in Figure 10-73. This enzyme cuts the DNA at the sites indicated by the
o
Cut
arrows. once the phosphodiester bonds in the duplex DNA are nicked, ii is a
simple task to denature the hydrogen bonds holding that portion of the duplex
DNA together. Note that this particular cleavage site his twofold rotational
symmetry. It is palindromic. A palindrome is a word (e.g., radar and rotator), and,
Sticky End expression (e.g., nurses run), or a sentence (e.g., a man, a plan, a canal, panama) that
reads the same from right to left as it does from left to right. once the hydrogen
pA-pA-pT-pT-pC--3' bonds break we will be left with two ends which are cohesive (i.e., "sticky end"s,'),
-.!.r-a meaning that they can join back together again given the right conditions.
Gp-5'

5'-pG How often does this enzyme cut DNA? There are 6 nucleotides in the Eco RI
sequence. There are also 4 possible nucleotides at the first position in the Eco RI
3'--Cp-Tp-Tp-Ap-Ap
sequence/ 4 possible nucleotides at the second position, and so on. If we multiply
these possibilities together (4x4x4x4x4x4), it turns out that (on the average) every
Sticky End 4,096 nucleotides (or about every 4 kb) there will be an Eco RI restriction site.
Figure 10.73 Remember, though, that this is only in theory.

when you look for restriction sites in DNA you want to look for two-fold
rotational symmetry. For example, consider the DNA sequence shown in Figure
'l'0-74. only one strand has been shown because the
complementary strand is
quite easy to write. However, you should still be able to find regions of two-fold

Copyright @ by The Berkeley Review 432 The Berkeley Keview


Specializing in MCAT Preparation
Biology Expression of Qenetic Information Genetic Engineering & Cloning

rotational symmetry if presented with only one strand of DNA. In this example
there are two possible regions of two-fold rotational symmetry.
Regions of twofold
In Figure 10-74 we have one region of two-fold rotational symmetry that is 6 rotational symmetry.
nucleotides in length. we mentioned that (on average) we would expect to find
one cut made by such a restriction enzyme every 4,096 base pairs. This particular
-A-A-G-A-T-C-T-G-G-C-C-
restriction enzyme is called Bgl II (Irom Bacillus globigii). Also shown in Figure --t
'10'74 is a region of two-fold rotational symmetry that is 4 nucleotides
in length. Figure lO-74
We would expect this restriction enzyme to make a cut in the DNA (on average)
every 256 base pairs. This particular restriction enzyme is Hae III (from
Ha emophilus a e gy p tiu s).

II and Hae III in a little more detail. The restriction sites


Let's take a look at Bgl
for Bgl II are shown in Figure 10-75a. Note that if we make our cuts at the
arrows/ we will have a 4-base overhang that we call the sticky end(s). These
overhangs are also referred to as being S'-overhangs because of the 5' phosphate (a)
group that is still attached to that particular end. This is similar to the restriction Cut
enzyme Eco RI. Hae III does not give sticky ends when it makes its cut but
rather gives blunt ends as shown in Figure l0-75b.
o
5'-pA-pG-pA-pT-pC-pT- 3'
3'-Tp-Cp-Tp-Ap-Gp-Ap-5'
There are also restriction endonucleases which will give 3'-overhangs when they
make their cut in the DNA. An example of such an enzyme is Hae IL This is
shown in Figure 10-76.In this particular example, "Pu" stands for a purine while Bgr rr tl o
cur
"Py" stands for a pyrimidine. {,|
Cut

5'-Pu-pG-pC-pG-pC-pPy-3'
o S'-pG-pA-pr nc-p[n1s,

3'- Pyp{R-Gp-Cp-Gp-Pup-5' 5'-pA


U 3'-Tp-Cp-Tp-Ap-Gp-5'
Cur I I Haerr
\Z
(b)
5'-Pu-pG-pC-pG-pC-3'
3'-Pyp pPy-3' Cut
3' - Cp -Gp -Cp- Gp - Pup- 5' s
5'-pG-pG-pC-pC-3'
3'-cn-cn7flp-Gp-5'
Figure lo-76

I
Cut
The modification enzyme for EcoRI (a DNA methylase enzyme) recognizes the
same sequence as the restriction enzyme and transfers a methyl group from a Hae III
molecule called S-Adenosylmethionine (SAM) to the C-6 position of the purine
adenine to make 6-methyladenine. This process is a signal that identifies this
DNA as being the DNA of the host cell. The l,-phage that survived the transfer to 5'-pG-pG-3' 5'-pC-pC-3'
the E. coli plate with the RI plasmid did so because that phage's DNA was 3'-Cp-Cp-5' 3'-Gp-Gp-5'
methylated before the restriction enzymes had a chance to degrade it. Figure lO-75
Let's consider how to make a map of restriction sites on a duplex of DNA. In
order to separate large fragments of DNA an agarose gel is used whereas small
fragments are separated on an acrylamide gel. The smaller fragments (in both
gels) have an easier time of getting through the gel matrix and therefore migrate
faster. The larger fragments migrate slower. If ),-phage is digested with the

Copyright @ by The Berkeley Review 433 The Berkeley Review


Specializing in MCAT Preparation
Biology Expression of Genetic Information Genetic Engineering & Cloning

restriction endonuclease Eco RI, then you should (in theory) get about
10
fragments. These fragments can be separated on the basis of size.

DNA is digested wirh


restriction enzyme(s) Add fragments
at top

Nitrocellulose
\ Paper
\ Paper towels

I -+lF:l
r@Gr

7.(J -r-T
cel ntltin.
,-,. (
) solution
Restriction fragments Electrophoresis Nitrocellulose
are separated by gel paper
electrophoresis

Figure 10.77

How would you know the order in which these fragments went together? one
method to determine the order of the fragments wou'id be by south"L utottir,g.
The DNA sample is digested with a particular restriction endonuclearu ur",-d
separated into fragments using gel electrophoresis. The DNA is then denatured
(because later you will want to hybridize to tne ONa; with alkali and
transferred
from the gel onto a piece of nitrocelrulose paper. paper towels can be used to
absorb the buffer solution. The DNA binds to the nitrbceilulose paper and
gives
you a permanent record of the DNA fragments that you had on the get.inis
procedure is shown in Figure 10-77.

suppose we were to cut our DNA sample with Eco RI and Hind III. what you
would like to know are the restriction sites for Eco RI and for Hind III. Fiow
would the Hind III fragments relate to the Eco RI fragments? The chances are
the Hind III fragments and the Eco RI fragments wilioverlap. A possibility is
shown in Figure 10-78.

Cut
Eco RI Eco RI
Fragment Fragment

DNA
Hind Itr
Fragment

Figure lO.7A

How do we analyze this possibility? The DNA to be analyzed is cleaved with


each restriction enzyme in separate testtubes and each set of restriction fragments
(in each testtube) is separated by gel electrophoresis. we will have a gellor the
Eco RI restriction enzyme and a gel for the Hind III restriction enzyme. we
follow the procedure outlined in Figure 10-76 for making a nitrocelluloie record
9f th" fragments from our two restriction enzymes. we ian pick a fragment cut
by the Hind III enzyme and radioactively label it and use that ridioactive
sequence as a probe. If we apply this probe to the nitrocellulose paper with the
fragments cut by Eco RI, then it will hybridize to those sequences oinNA ir-, the

Copyright @ by The Berkeley Review 434 The Berkeley Review


Specializing in MCAT preparation
Biology Expression of Genetic Information Genetic Engineering & Cloning

Eco RI fragments which are complementary. If the labeled Hind III fragment
overlaps two Eco RI fragments (as shown in Figure 10-78), then we should see
two dark bands on our developed x-ray film indicating which Eco RI fragments
contain the complementary sequences to the Hind III fragment. Repeating this
procedure with other fragments will eventually lead to a map of your DNA
sample.

FIow can the sequence of a piece of DNA be determined? Two techniques which
have been used successfully for sequencing DNA are the Maxam-Gilbert
method, involving specific chemical cleavages, and the sanger dideoxy method,
involving a controlled interruption of DNA replication. Since the Maxam-Gilbert
method is not used that much anymore, we will turn our attention to the Sanger
dideoxy method.

Sanger Dideoxy Sequencing


The sanger dideoxy technique makes use of the fact that DNA polymerase I can
be inhibited by a 2',3'-dideox;mucleoside triphosphate (ddNTp) analog of one of
the four bases A, T, G, or C. The example shown in Figure 10-29 is ddATp.

-o-P
-o -P -o -P -O -CHr
ttt
_O _O _O 4'
3',2'
TBO

Figure 1o,79

1. Select the DNA strand to be sequenced. Attach a short radioactively labeled


section of primer to the DNA strand being analyzed. The label of choice is 35S
instead of 32P begruse 35S has a longer friU tUe than 32p, Also, 35S has such a
weak emission that it doesn't require shielding. The 35s atom replaces the oxygen
atom that is double bonded to the o-phosphate of dNTP.

To each of four solutions add DNA polymerase I, the four deoxynucleotides


triphosphates (dNTP)-which are dATP, dTTP, dGTP, and dCTP-and one of the
ddNTP's (ddATP, ddTTP, ddGTP, or ddCTP)in a controlled ratio.

2. DNA polymerase I will take the dNTPs and add them to the primer. The
2',3'-dideoxy analog can be incorporated into the growing DNA chain by reaction
with its 5' triphosphate function. However, when the next deoxynucleotide
triphosphate is use{ in the reaction sequence it cannot be added to the 2',3r-
dideoxy analog because the analog does not contain a 3'-OH function to form a
phosphodiester bond. Termination of the DNA chain occurs specifically where
the nucleotide analog was incorporated. This yields DNA fragments of different
length containing the analog at the 3' end.

Copyright @ by The Berkeley Review 435 The Berkeley Review


Specializing in MCAT Preparation
Biology Expression of Genetic Information Genetic Engineering & Cloning

DNA backbone

TTAGACCCGATAA CCCGCA
GCGT
Labeled 5t
Primer
DNA backbone

TTAGACCCGATAAGb.iG6i
ATTCGGGCGT

H ATCTGGG 9T4TTCGGGCGT
H

H
++TgTggggT4TTCGGGCGr
H

Figure lO-aO

3. The labeled fragments.shown_in Figure 10-g0 are then separated by


. size on a
polyacrylamide gel by electrophoresis. The pattern of fiuj*e'ts
from the
autoradiogram shown in Figure 10-g1 yieldi tt u pNa ,"irr"r,."
that was
synthesized using the radioactively rabeled complementary DN,i
primer strand__
not the original DNA strand you wished to anaiyze. since this "primer,,strand
is
complementary to the DNA sequence of the original strand, there
should be no
problem in obtaining the base sequence of that or:iginal strand.

ddATP ddTTP ddCTP


I AT
AT
L II TA
a. II CG
!)_ I TA
qa
Ebo
- II GC
q
AF
'9
I-r
r CG
I TA
II - AT
oc) I TA
TA
boc
CC
- DNA sequence of
- : :
cc
E-
-I cc
complementarv
'./ strand

-l TA ,/
-
-
Figure lO-81

Copyright @ by The Berkeley Review 436 The Berkeley Review


Specializing in MCAT preparation
Biology Expression of Genetic Information Genetic Engineering & Cloning

Cloning
Recall that we mentioned that there were restriction enzymes that cut DNA at
specific sequences. Those sequences have a unique characteristic in that they
have twofold rotational symmetry. Some restriction enzymes leave a 5' overhang
while others leave a 3' overhang. There are also restriction enzymes that leave
blunt ends.

There are several methods that will allow you to make a map of restriction sites
and restriction fragments in your DNA. bne method that we mentioned was
Southern blotting. If you wanted to determine the sequence of the DNA
fragment, then you could use the Sanger dideoxy technique (or the Maxam-
Gilbert technique).

VVhen the DNA that you are going to analyze is labeled, you want to make sure
that it is labeled on the end of your fragment of DNA. why? one reason involves
sensitivity. In the case of the Maxam-Gilbert technique, labeled 32P at the end of
the DNA fragment might give you about 1,09 radioactive counts per minute.
Another reason to use end labeling involves the simplicity of the pattern that you
see on your gel.

with the use of restriction enzymes, one can use recombinant DNA technology to
create new combinations of genes which can then be cloned. For example, a piece
of foreign DNA cut with a specific restriction enzyme, such as Eco RI, can be
inserted into a DNA vector which has also been cut with Eco RI. Replication of
this recombinant DNA molecule takes place in the host cell. A suitable vector for
E. coli would either be l,-phage or a plasmid.

It turns out that there are a few problems involved in the cloning procedure. one
problem concerns the copy number. In the case of the first plasmid used, there
were only three per E. coli chromosome. Another problem is the size of the
plasmid. The plasmid that was first used in a cloning experiment was about 100
kb, which is a lot of DNA to have to work with.

There is also a problem with reclosure of the DNA. For example, if you cut your
DNA with Eco RI, then that same piece of DNA can close right back up again
without ever taking in the foreign piece of DNA that you want to clone. One way
to get around this problem is to add an excess of the foreign DNA so that you get
more collisions between"the foreign DNA that you want to insert and the
plasmid.

Another way to get around this reclosure problem is to use phosphatase on the
plasmid DNA. If you want to join a nick between two nucleotide residues in a
segment of double stranded DNA, then the enzyme DNA ligase is used. The
energy source for this procedure comes from either ATP or NADo, depending
on the organism in use. (In bacterial cells the energy source is NADo while in
animal cells the energy source is ATP. NAD is an abbreviation for nicotinamide
adenine dinucleotide.)

Figure 10-82 we see a seginent of double stranded DNA with a nick. Note the
Ii-r
iree 3' hydroxyl and the free 5' phosphate at the nick site. DNA ligase will
eventually form a phosphodiester bond at that position in order to seal the nick.

The 5'phosphate on the DNA duplex is absolutely required in order for DNA
-rgase to work. If we were to treat the DNA segment shown in Figure 10-82 with

Copyright @ by The Berkeley Review 457 The Berkeley Keview


Specializing in MCAT Preparation
Biology Expression of Genetic Information Genetic Dngineering & Cloning

phosphatase and remove that 5' phosphate residue, we would not be able to close
the nick with DNA ligase.

3'_A_T_G_T_5,
*Holll
5
ll ll V,rC ll
A- 3'
OHP
-T-A.. 3' 5'

Figure lO-82

Suppose we have 5'phosphates hanging out at the ends of our cleaved DNA
plasmid. If we remove those 5' phosphates with phosphatase, then we will not be
able to join these ends of the plasmid. This is shown in Figure 10-83.

Phosphatase
3'
OH

A restriction enzyme The ends will


cuts the duplex DNA not rejoin

Figure lo-a3

If a foreign piece of DNA that has 5' phosphates at two of its ends is now inserted
into our plasmid (from Figure 10-83), then only two strands of the DNA can be
joined. The other two strands of DNA will be left with nicks as shown in Figure
10-84. However, there will be some integrity to this plasmid.

3'HO
J
OH
5'P P5'

Foreign DNA to be inserted

DNA without 5' DNA with two nicks


phosphates

Figure rO-84

Suppose we have cut our plasmid with the restriction endonuclease Eco RI and
we would like to insert a piece of foreign DNA that has also been cut with Eco

Copyright @ by The Berkeley Review 434 The Berkeley Review


Specializing in MCAT Preparation
Biology Expression of Genetic lnformation Genetic Dngineering & Cloning

RI. One way this could be done is shown in Figure 10-85. Note the arrow above
the sequence 5'-TAC-3' in the fragment of foreign DNA. Let this represent a
direction for the insertion of the foreign piece of DNA. This direction is "in
phase" with the direction of the arrow for the plasmid. The "8" in our example
simply represents any base.

5' B_G A_A-T -T { -B DNA from plasmid


lil lil cur with EcoRI
1 B-C-T-T_A-A G+
+

5'A-A-T -T-C-T -A-C-T -G-G-C


lllllt
3' G_A-T -G-A-C.C-C-T
cur with EcoRI

-T -A-A
Annealing fragments with
DNA ligase

r ,) ----)
5 B -C-A-A-T-T-C-T -A-C-T -c-c-CFA-A-T -T -C -B
J
llr ill
B -C-T -T -A*TG -A-T-G-A-C-C-C-T -T
lil
-A-TG -B
lll
Figure lO-85

Our foreign piece of DNA could also be inserted into our plasmid in the opposite
direction as shown in Figure 10-86. The two arrows are "out of phase." This is an
important aspect of cloning because if you are inserting a gene that you want to
clone into a plasmid, then in order for that gene to be properly expressed it must
go into the plasmid in the right orientation. Why? Because if the foreign DNA
goes into the plasmid in the wrong orientation, then when RNA polymerase
transcribes it, the protein that will eventually be produced (if it is produced at all)
most likely will be non-functional.

-G-A_A_T -T -C -C-C -A_G_T _A-C -A_A-T_T{-B


5'
B

3',
il ilt
B _C _T -T-A*A -G .G.G-T -C-A-T -C_T -T -A-A.C +
ilt
ilt

Figure f O-Ao

Let's consider the plasmid pBR322, which has a high copy number of about 30
per chromosome. This plasmid has an ampicillin resistance gene (Amp) and a
tetracycline resistance"gene (Tetr). Both of these genes have promoters and the
direction of transcription in each case is indicated by the arrows. Within the
tetracycline gene is a restriction site for the restriction endonuclease Bam HI. This
is shown in Figure 10-87.

Copyright O by The Berkeley Review 439 The Berkeley Review


Specializing in MCAT Preparation
Biology Expression of Genetic Information Genetic Dngineering & Cloning

Promoter

Tet'

pBR322
Promoter

A*p'

Figure 10.87

If we open the plasmid with the restriction endonuclease Bam HI, then we can
insert a foreign DNA fragment in two possible orientations. If this fragment
contains a gene without a promoter, then it is important as to which orientation
the foreign DNA is inserted. If the gene is inserted in the direction given by the
arrow for the gene, then the tetracycline promoter will not work with that gene.
This is shown in Figure L0-88a. However, if the gene is oriented in the same
direction as transcription, then the tetracycline promoter will work with the
gene. This is shown in Figure 10-88b.

Td'
Promotel

Foreign
DNA DNA

FR322 FRr2

A*p' Atttpt

(a) (b)

Figure f O'aa

How would you know which way the foreign piece of DNA has inserted itself
into the plasmid? Suppose we have an Eco RI site in our plasmid as shown in
Figure L0-89. Also suppose we have an Ssp site in our piece of foreign DNA. If
our piece of foreign DNA inserts as shown in Figure L0-89a, and we cut at the
Eco RI and Ssp sites, then we will get two fragments--one large and one oI
moderate length.

Copyright @ by The Berkeley Review 440 The Berkeley Review


Specializing in MCAT Preparation
Biology Expression of Genetic Information Genetic Engineering & Cloning

FRe2 frR322

A*p' Amp'

Figure lO.ag

Flowever, if our piece of foreign DNA inserts in the


opposite direction as shown
in Figure 't'0-89b, and we cut it the Eco RI and the ssp sites,
then we *iiig"i ;
large fragment and a smalr fragment. If you take your
ilor,", and do a restriction
digest, you will see different pattems. you can just save one sample of each
fw9
until you are sure which one you want.

A plasmid carrying genes_for drug resistance is usefur because


insert a piece of foreign DNA into one of those genes
if you were to
that codes for a drug
resistance, you would lose the activity of that
particrilar gene. If a piece of foreigf;
DNA has inserted into, say, the tetracycline resistan." gIr,"
of pBk322,then you
would get a host cell that has tetracyciine sensitivity. If"the
did not properly insert into the tetricycline gene, then the
piel of roreign dla
host cell would still be
resistant to tetracycline. This would. t"ll yo.tlhut your
insertion reaction failed.
To remove the problem of orientation of the insertion
fragment, two restriction
endonucleases were used that gave different cleaved
pBR322 there is an Eco RI site and a Hind III site
ui-rdr. tro, exampre, in
not too far away.If you cut the
o
5'-A-C-G-C-G_T_3'
pBR322 plasmid with these two enzymes and purify
the large fragment, you will 3'-T-G-C-G-C.A-5'
have two diff"l"", truis..1"aqs.
-Now,
cut with Eco RI and Hind IlI, then
if you have a piece oT foreign DNA that is
this insertion fiagment will insert into the
o
pBR322 plasmid in only one orientation. Mla I
The pB_R322 plasmid also has a pvu rr (proteus aulgaris)
site which gives blunt
ends. These blunt ends can anneal with the blunt Jnds produced b|
any other
o
5'-G-C-G-C-G-C-3'
restriction enz).rne that gives that type of cut.
3'-C-G-C-G-C-G-5'
You could also join together two half restriction sites that have
the same cohesive
o
ends but different recognitions. Two examples are Mla I and
Bss HII. The
Bss HII
sequences that these endonucleases recognize are shown in
Figure 10-90. This is Figure lO-9O
good for insertion of the ioreign pNa but you cannot remove that
DNA once it
has been inserted (because you lose the restiiction site).

T"t"_11" certain types of plasmids that have been engineered at UC Davis calred
the puC plasmids. These plasmids are about 2 kb ind have a very high
copy
number, about 90 per chromosome. These plasmids have a gene for a.ipicitiii-,

Copyright @ by The Berkeley Review 44t The Berkeley Review


Specializing in MCAT preparation
Biology Expression of Genetic Information Genetic Engineering & Ctoning

resistance and in one small region, called a polytinker, there are about 17
restriction sites.

Another conunon vector is lambda (1,) phage. This phage infects the bacterium E.
coli and injects its linear DNA into the host. The viral DNA soon becomes circular
and is quickly presented with two choices--to become part of the host (the
lysogenic pathway) or to produce more progeny and destroy the host in the
process (the lytic pathway). It turns out that the genome of this phage is about 4g
kb, and not all of it is essential for the phage's life cycle. What this means is that
portions of the phage's genome can be removed and foreign DNA inserted. once
the bacterium has been infected, the fragments of foreign DNA can be amplified
many fold, thus giving rise to a genomic library. This library is then analyzed to
find the gene of interest. A particular gene of interest can be located by using a
radioactively labeled probe.

One useful application of cloning is the production of proinsulin, a precursor to


insulin. within a given plasmid is a region of inserted DNA that is
complementary (called cDNA) to the mRNA for pancreatic proinsulin. In order
to make this cDNA from the mRNA, the enzyme reverse transcriptase was
utilized. In order to ensure maximum transcription of the cDNA, it is inserted
into the plasmid in the correct reading frame and next to a strong promoter.

In the late 1970s the complete sequence was determined for the filamentous
phage M13. This 6.4kb phage has nine known genes. The area between genes
vII and IX is capable of coding for a small peptide. Mutants were made for this
region. A mismatched oligonucleotide was made in which a C residue was made
to be opposite a T residue. This is shown in Figure 10-91. when this region is
transcribed there will be a 5'-uAG-3' stop codon in the middle of the gene. This
inactivates the gene (which is necessary for growth of the phage), thus causing a
mutation. However, the mutation rate was only about lo/o because within the E.
coli bacterium is a mismatch repair enz)rrne that is constantly checking the DNA
and correcting errors in the base pairing.

3'-T-A-c-T-c-T -A-T-T-C-T-T-T-C-G-C-C _5'


DNA
5'-C at
J

3'-G-A-U-5' mRNA

Figure lO-91

Copyright @ by The Berkeley Review 442 The Berkeley Review


Specializing in MCAT Preparation
Expression of Genetic
Information

15 Passages

100 Questions

Passage Tifles
Questions
I. Southern Blotting
II. Restriction Endo*nucleases 1-5
ilI. Ribozymes and HIV 6-11
IV. The Genetic Cocle 12-11
t8-23
_y. Polymerase
VI.
Chain Reacrion (pCR)
Lactose Operon 24-29
VII. Enhancers^andGeneExpression 30-36
VI1f. Replication Block 31 -43
C;li cycte 44-50
IX. Northern and Southern
""J;fi;
Blottins
X. Restriction Enzvmes ---o 51 -58
59-65
XI. Arabinose Opeion 66-72
_XII. Gene Therapy Strategies 73-19
{.!U, Tryprophan'Operon'
Xry. Translation 80-87
XV. Cancerous Gene 88-94
95 - 100

speci altzing in MCAT Preparation


Suggestions
The passages that follow are designed to get you to think in a conceptual manner about the processes
of molecular biology at the organismal level. If you already have a solid foundation in molecular biology,
many of the questions you read here will seem to be very straight forward and easy to answer. But if you
are new to the subject or if you have not had a pleasant experience with molecular biology in the past,
some of them might appear to come from the void that spreads out beyond the Oort field at the edges of
our solar system.

Pick a few passage topics at random. For these initial few passages, do not worry about the time. Just
focus on what is expected of you. First, read the passage. Second, look at any diagrams, charts, or graphs
in it. Third, read each question and the accompanying answers carefully. Fourth, answer the questions
the best you can. Check the solutions and see how you did. \Alhether you got the answers right or wrong,
it is important to read the explanations and see if you understand (and agree with) what is being
explained. Keep a record of your results.

After you feel comfortable with the format of those initial few passages, pick another block of
passages and try to do them in one sitting. Be aware that time is going to become important. On average,
you have about 1 minute and 15 seconds to complete a question. Be creative in how you approach this
next group. If you feel comfortable with the outline presented above, fine. If not, then try different
approaches to a passage. For example, you might feel well versed enough to read the questions first and
then try to answer some of them, without ever having read the passage. Maybe you can answer some of
the questions by just looking at the diagrams, charts, or graphs that are presented in a particular passage.
Remember, there are many effective learning styles. You need to begin to develop a format that works
best for you. Keep a record ofyour results.

The last block of passages might contain at least a few topics that are unfamiliar even to those who
know a good deal about molecular biology. Find a place where the level of distraction is at a minimum.
Get out your watch and time yourself on these passages, either individually or as a group. It is important
to have a feel for time, and an awareness of how much is passing as you try to answer each question.
Never let a question get you flustered. If you cannot figure out what the answer is from information
given to you in the passage, or from your own knowledge base, dump it and move on to the next
question. As you do this, make a note of that pesky question and come back to it when you have more
time. When you are finished, check your answers and make sure you understand the solutions. Be
inquisitive. If you do not know the answer to something, look it up. The solution tends to stay with you
longer that way. (For example, what ls the Oort field, anyway?)

The estiniated score conversions for 100 questions are shown below. At best, these are rough
approximations and should be used only to give one a feel for which ballpark they are sitting in.

Section X
Estimated Score Conversions
Scaled Score Raw Score
>13 80- 100
rt-12 10-19
9- 10 60-69
1-8 50-59
5-6 40-49
<4 0-39
Biology Southern Blotting Passage I

Passage I (Questions l-5)


3. Gel electrophoresis is_ commonly used to
separate
molecules and molecular fragments. What
From a mixture of DNA sequences, a particular does the
separation of DNA into fragmints depend
fragment may be isolated with the Southern transfer on when a
Southern blot is done?
technique, also known as Southern btotting. The
basics of
this technique are:
A. Charge
l) DNA is fragmented into lengths that are appropriate
B. Size
for.the.agarose gel. This miy be accomptisteO
C. Radioactivity
sonication, mechanical shearing, or enzymatic action.
Uy D. Linking number
Convenrionally, abort 100,000 base pairs (bp) is the
upper limit on size for agarose gel ellctrophoresis
of
DNA.

2) The DNA on rhe gel is denatured with NaOH. A 4. If you were probing for
sheet of nitrocellulose paper is placed over the DNA fragment:
the gel,
99ver9d with paper towels, and pressed down. T:his 5'CGATTACCCG3,
blots liquid and any single_stranded DNA from the
gel. which of the following probes would you
use?
3) The nitrocellulose paper is dried under vacuum
with A. 5'GCTAATGGGC3,
heat, permanently fixing the adhered DNA. B. 3'CGGGTAATCGs'
C. 3'GCTAATGGGCs'
4) A DNA probe, labeled either radioactively or D. 5'CGATTACCCG3'
enzymatically, that is complementary to the sequence
of interest is applied to the nitrocelluiose paper.

5) Heat is applied over several hours to allow


renaturation. Then the paper is washed to remove
only unbound probe. 5. Huntington's chorea (HC) is a fatal, degenerative
brain disease that is transmitted in an autosomal
6) The paper is read either by autoradiography on
x_ray dominant fashion. In the results shown below
film or by quantifying for a
the bnzymatic riaciion. gel electrophoresis done totest for a DNA
fragment
cortaining a region common to people who
have
., Southern
the
blotting is particularly useful for identifying
presence of a specific_ DNA fragment ln
HC, assume that A and B are the pu."ntr, and
C and
a largE D are rhe. offspring. Which of tn" following
mixture of fragments. It can be used to iientify carriers statements is TRUE?
of
certain inherited diseases. for example.
Parents Offspring

AB''CDf
1. Which types of enzymes are commonly used
(- (- (-
fragment DNA for Southern blotting? EII
0)
rD (I) (-
F" rrr (D (- (-
A. Resiriction enzymes
B. Proteases
fiIV (- r- (- rD
C. Nucleotidesynthases (- rD (-
D. DNA gyrases
A. If Fragment II is the HC fragment, then both
children are affected.
2. When DNA is denarured by NaOH in Srep 2, what B. B^oth parents carry the HC Fragment, V.
structural change takes place?
C. If Fragment III is the HC fralment. then both
children are affected.
A. The phosphates of DNA become positively D. The children show identical patterns of
charged.
inheritance.
B. DNA strandsjoin to form a quad structure.
C. The base-pairing of DNA is altered.
D. DNA is separated into single strands.

Copyright @ by The Berkeley Review


44s The Berkeley Review
Specializing in MCAT preparation
Biology Kestriction Endonucleases Passage tr

Passage II (Questions 6-11) Restriction endonucleases can cut DNA into fragments
of varying length, which can then be separated from one
When viral DNA enters a bacterial cell, a defense another by gel electrophoresis and identified using
mechanism in the bacterial cell degrades the DNA of the autoradiography. Larger fragments are located near the
virus. The activity of the viral DNA is restricted from top of the gel, while smaller fragments are located near
operating within the bacterial cell by the action of the bottom.
bacterial enzymes called re st ric tion endonucle as e s. These
restriction enzymes recognize a particular sequence of If a single restriction endonuclease is used to cut a
bases in double-stranded DNA and then cleave the DNA sample of DNA, the procedure is called a single digest. A
at specific sites within those sequences. double digest involves the use of two different restriction
endonucleases. DNA fragments are described in terms of
Recognition sequences are written in the 5' -+ 3' length. One kilobase (kb) is equal to 1000 bases (or base
direction and, by convention, only one DNA strand is pairs). A restriction map can be established by comparing
given. A vertical arrow that represents the cleavage site is the lengths of the DNA fragments from different digests.
placed within the written DNA sequence to indicate
where the restriction endonuclease cuts the DNA. For
example, the DNA sequence AJ AGCTT is an
abbreviation for: 6. Which of the following restriction endonucleases
would produce 5' sticky ends?
Cut
+ L PstI
5'-AAGCTT-3' il. SalI
3'-TTCGAA.5'
i III. TaqI
Cut
A. I only
The restriction endonuclease Hindlll recognizes this B. II only
sequence and cleaves the DNA at the cut sites. Once the C. III only
two phosphodiester bonds holding the DNA backbone D. II and III only
together are cleaved, the hydrogen bonds between
complementary bases break apart. If the cut sites are
staggered, then complementary sticky ends are produced. 7. The plasmid pBR322 is a circular duplex of DNA
If the cut sites are not staggered, then blunt ends are that can replicate autonomously in the bacterium
produced. Most restriction endonucleases recognize DNA Escherichia col/. This plasmid contains 4.3 kb of
sequences that are palindromic (the sequence is the same DNA. In a random sequence of DNA, how many
if read forwards or backwards). Examples of some cuts would be expected, if the restriction enzyme
restriction endonucleases and their recognition sequences A/u I is used?
are shown in Table 1.
A.8
B. ll
Recognition Recognition
c. t4
Sequence :Enzyme Sequence Enzyme D. t',l

AGJCT Alul GAGCTJC Sac I


GJGATCC BamHI GJTcGAC Sal I
8. A polylinker is a segment of DNA that contains
several restriction endonuclease sites. Based on the
AJGATCT BgtIl CCCJGGG Smal information in Table l, how many unique restriction
GJAATTC EcoRI TJCGA TaqI endonucleases can cut the following segment of
duplex DNA?
CTGCAJG PsT I CCCJGGG Xmal
5'-CGGATCCCGGGTCGACG-3'
Table 1. Restriction Endonucleases 3'.GCCTAGGGCCCAGCTGC-5'

A.2
Bacterial DNA il protected from the cell's own B.3
restriction enzymes by the action of modification c.4
endonucleases. These enzymes catalyze the methylation D.5
of either an adenine or a cytosine base within a particular
recognition sequence.

Copyright @ by The Berkeley Review 446 The Berkeley Review


Specializing in MCAT Preparation
Biology Restriction Endonucleases Passage II

9. A segmenr of bacrenal D\{ is cut by the restriction


11. Various combinations of restriction endonucleases
endonuclease BamHI Tuo complementary sticky are used to digest a length of bacterial DNA.
ends are formed. These sricky ends are uUt. The
to fragments from these single and double
estab.lish hvdrogen bonds at the sticky ends digests are
formed analyzed-by agarose gel electrophoresis and
by which of the tbllou ine restriction endonucleases,l
compared to a known standard (S), as ihown
below:
A. HindlII
Eco Rl pst I
J-
Eco Rl
B. BglIl A8l II 6co Rt psr I Bgt U Bgl lt psr I
C. Psr I
D. Sac I
7kb II
6kb I r r
10. Two linear DNA molecules are digested with the 5kb I I
same restriction enzyme (,EcoRIJ in different
experiments, and their-fragments ui" ,"pu.u,"O Uy
4kb I I I
size using agarose gel electrophoresis (see below;:

DNA
Lane 1
DNA
Lane 2
3kb

2kh
I

II
I r
II II
I r A
lkb I III
I r Based on the DNA fragment patterns in
the gel
C I I C
shown above, which of the following ,estriction
maps BEST represents the length of bacierial
I DNA?
D I
A.
Y Psr I
E Bgl Il Cco Rl Psr I
- DNA
- -
- - B.
Basedo_n the separation of fragments in the gel, pst I
the
two DNA molecules seeri to be similar. Ttie only Pst I Eco Rl Bgl lI
d_ifference is that Fragment D in Lane I does DNA
not
show up at the same position in Lane 2. Instead,
two new fragments, X and y, have appeared. The
presence of Fragments X and y is due tb the: c.
Bsl Il
Pst I Eco RI Pst I
A. addition of DNA that contains at Eco Rl site DNA
to Fragment D.
B. deletion of DNA from Fragment D.
C. addition of DNA to Fragment D. D.
D. deletion of DNA from"Fragment D, resulting Eco Rl
in the subsequent formation of a new Eco RI Pst I Bgl tl Pst I
site. DNA

lopyright @ by The Berkeley Review


447 The Berkeley Review
Specializing in MCAT preparation
Biology Ribozymes and IIIV Passage III

Passage III (Questions 12-17) 13. HIV is notorious for mutating in response to
treatment. For example, giving a particular
Although it is well known that enzymes are catalysts experimental drug led to a two-week decline in the
(molecules having the ability to increase reaction rates), it number of HIV particles, followed by a dramatic
is less well known that RNA molecules can have catalytic mutation resulting in a95Vo new population of HIV
activity as well. The term ribozyme refers to an RNA particles. How could the ribozyme approach BEST
molecule with catalytic activity. Thomas Cech of the fight the mutation problem?
University of Colorado and Sidney Altman of Yale
University won the 1989 Nobel Prize in chemistry for A. Increase the concentration of the ribozyme by
their work that indicated RNA was more than a passive a more efficient transfection process
carrier of genetic information. RNA can act to catalyze B. Follow one ribozyme with another in two
itself or other molecules of RNA. weeks
C. Create a specific ribozyme for the strain of
Ribozymes were probably the first catalytic agents HIV the patient has
formed during the evolution of life. Later, RNAs began to D. Link multiple ribozymes to attack HIV mRNA
code for protein synthesis, and proteins are superior at several locations at once
enzymes. Proteins became the dominant form of catalysts.

Cech's work in 1982 indicated that some of the RNA


found in Tetrahymena is autocatalytic. It has the ability to
splice out introns and edit its own mRNA sequence. One
year later, Altman discovered some RNA molecules that
could cleave other molecules as well. 14. Why do Hampel and Wong-Staal target a highly
conserved region of the HIV mRNA that is common
Ribozymes are currently being studied to combat among many different strains of HIV?
human immunodeficiency virus (HIV). Researchers
Arnold Hampel and Flossie Wong-Staal are modifying a A. The highly conserved regions are known for
tobacco virus ribozyme to recognize and cleave a highly their wide range of variability, and this
conserved region common among HIV strains. A approach targets more mutants.
specially designed ribozyme can chop up the HIV mRNA B. Usually, highly conserved regions code for
in test-tube experiments. In culture, this modified well-designed functions critical for survival,
ribozyme reduced HIV transcripts by 10,000-fold. and this approach targets more mutants.
C. The HIV particle resists attack by ribozymes
Cech and Bruce Sullenger are focusing on packaging except at these highly conserved sites.
the ribozyme gene so it co-locates with the virus into the D. The highly conserved regions attract
virus packaging center of the cell. This means that the ribozymes for more efficient cleavage of
ribozyme against HIV will be present inside the "HIV mRNA.
factory" and will destroy copies of the viral mRNA. One
clinical approach is to remove T cells from the patient,
infect them with a ribozyme that cleaves HIV mRNA, and
return it to the patieot. Although this scheme will not be
tested ln vivo for several years, these are the big plans
some researchers are exploring right now.
15. What is the immediate effect (prior to mutarional
changes) of decreasing with a ribozyme the number
of available mRNAs for HIV in a T cell?
12. If the right ribozymes attacked an animal cell, which
structures could be damaged? A. The number of virus particles does not change.
because each mRNA becomes more efficient
I. Ribosome at translation.
II. Transcript copy of DNA B. The number of virus particles decreases, due to
ilI. Amino acid carrier for protein translation fewer copies of the transcript.
: C. The number of virus particles increases, due to
A. I only more copies of the transcript.
B. I and II only D. The number of virus particles does not change.
C. II and III only due to competing HIV mRNAs.
D. I, II, and III

Copyright @ by The Berkeley Review 444 The Berkeley Keview C"


Specializing in MCAT Preparation
Biology Riborymes and HIV Passage III

16. In the course of evolutionary history, RNA_based


catalysts were replaced by protein-bised catalysts.
whv?

I wider variety of subunits provides greater


flexibility in the structure and function of
protein-based catalysts.
IL Fnzymes outcompeted ribozymes and chopped
them up.
III. Enzymes are more advantageous, because they
change the equilibria of reactions.

A. I only
B. I and Ii only
c. II and III only
D. I, II, and III

17. Which of the following statements would be TRUE


about catalysts?

I. Catalysts remain unchanged after completing a


reaction.
il. Catalysts increase the rate of a reaction.
III. The body does not synthesize catalysts.

A. I only
B. I and II only
C. II and III only
D. I,II. and III

Copyright @ by The Berkeley Review 449 The Berkeley Review


Specializing in MCAT preparation
Biotogy The Genetic Code Passage IV

Passage IV (Questions 18-23) 18. The difference in the number of possible kinds of
amino acids between a single-base code and a three-
In a pioneering biological experiment, Marshall base code is:
Nirenberg showed how to produce polyphenylalanine
from the addition of polyuridylate to a cell-free protein A. 4
synthesizing system. To begin, E. coli bacteria were B. 16
broken by agitation and subsequently centrifuged. The C. 60
resulting supernatant contained DNA, tRNA, mRNA, D. 64
ribosomes, enzymes, and other cytoplasmic components.
Protein synthesis could be obtained by the addition of
GTP, ATP, and amino acids. The mixture was incubated
at physiological temperature for some time, after which
trichloroacetic acid was added. The protein containing 19. The following base sequence is given:
precipitate was washed, and the number of l4C counts per
minute (cpm) was measured. t23456789
ATCGGTATA
An essential component of this system is the addition
of deoxyribonuclease (DNAse). The mRNA present when If the genetic code is overlapping,
a mutation in the
DNAse is added is short-lived, and protein synthesis soon base cytosine (C) may alter how many amino acids?
ceases. But synthesis resumes with the introduction of
additional mRNA, making possible a protein-synthesizing A. I
system that is entirely cell-free. 8.2
c.3
The synthetic polyribonucleotide that is added D.4
(polyuridylate) is created using polynucleotide
phosphorylase, an enzyme that catalyzes the following
reaction:

(RNA)n + Ribonucleoside diphosphsls = (RNA)1.,.1 + Pi 20. The following base sequence is given:

This enzyme does not use a template, and the relative


1 2 3 4 5 6 7 8 9 l0ll 12

composition of ribonucleotides present determines the


ACTGTTACATTG
RNA to be synthesized. A polynucleotide consisting of
Insertion of the base thymine (T) between the 7rh
two different bases is created by using two types of
and 8th base may aft-ect how many amino acids?
ribonucleoside diphosphates in the presence of
polynucleotide phosphorylase. Table I shows the amino
acid incorporation created by a random copolymer of l67o
A.0
uracil (radiolabeled) and 24Vo guanine.
B. I
c.2
D.3
Relative amount
AminoAcid incorporated

Phenylalanine r00 21. To obtain measurable results in this experiment,


which of the following cellular components MUST
Valine 3t
be used?
Leucine 36
Tryptophan t4 A. Radiolabeled mRNA
Glycine t2 B. Radiolabeled amino acids
C. Radiolabeled ATP
Tatrle l. Amino acid incorporation resulting D. Radiolabled ribosomes
from a random copolymer of U* (76Vo) and
G (247o). (Not& *Radiolabeled)

Copyright @ by The Berkeley Review 450 The Berkeley Review C.


Specializing in MCAT Preparation
Biology The Genetic Code Passage IV

22. Which of the following graphs BEST represents


protein synthesis as measured throughout the
experiment?

A. B.

a*l
lgl / / *=\
lEl \
E*I' Time
;EI \ Time

C. D.

tr
a) o

V
!o
o.N b.s
Oo !6
ao

Zq Y4
\
Time Time

23. Based on the information in Table l, the triplet that


codes for glycine MOST likely contains:

A. more guanine than uracil.


B. more uracil than guanine.
C. equal amounts of guanine and uracil.
D. all guanine and no uracil.

Copyright @ by The Berkeley Review 45t The Berkeley Review


Specializing in MCAT preparation
Biotogy Polymerase Chain Reaction (PCR) Passage V

Passage V (Questions 24-29\ A DNA polymerase called Taq, from the thermophilic
bacterium Thermus aquaticus, is added to the mixture.
The polymerase chain reaction is a process that The polymerase extends the primers and completes the
amplifies DNA within a certain region of a chromosome. first cycle of DNA amplification. The result is two new
The first step (Figure 1) is to denature the targeted DNA DNA double helices where there was once just one.
by heating it. Next, an excess of two DNA primers, about
15 to 20 oligonucleotides in length, is added to the Each cycle, which takes approximately five minutes,
denatured DNA. For proper hybridization, the two doubles the number of DNA molecules present. The cycle
oligonucleotide primers must be complementary to the is usually repeated 20 to 40 more times. The DNA
opposite strands of DNA. synthesized in subsequent cycles extends to just the ends
of the primers.

dsDNA rr One application of PCR involves the screening of


Targeted sequence \,2 particular steps in cloning. For example, one must screen
transformants after the step of inserting a DNA fragment
into a plasmid. The reason is to check to see whether the
I ll
. Denature transformants contain the inserted fragment and to verify
cYcle .enneut primers that the clone is suitable for further work. In this
$ screening process, transformant colonies are picked and
placed into tubes used for PCR. The cells are lysed, and
primers (indicated by I and 2 in Figure 2) are used that
flank the site into which the fragment is to be cloned. The
product of the PCR amplification is run on a gel.
. *,-., extension
[l
\7 Cloning site Inserted DNA
tMttlttltt

O O
+

[l . Denature
Cvcle
' 2 II l.AnnealDnmers
I 27
(7
v
rrrrIrrrrrr i r r r r r r r r r I I I I I I ll=!]lt I I | | I I I
Products of PCR

rr r rrll tffit t r rr rrrrlrrr rr rr rrr

Figure 2

T'1
tl .Primerextension 24. The MOST likely reason that the DNA polymerase
ll
I fromThermus aquaticus is used preferentially over
\.7
the polymerase from E. coli is that the:
tttrtttt
+ A. polymerase from Thermus aquaticus has a
@
ttttlttttttltlffi much faster rate of polymerization.
B. heat used to denature the DNA would destroy
the polymerase from E. coli.
rrlri,tttttttttt rttrlttt
C. polymerase from Thermus aquaticus is known
a-* to be much easier to isolate.
+
D. polymerase from E coli is not functional in
cell-free systems.

Figure 1

Copyright @ by The Berkeley Review 452 The Berkeley Review


Specializing in MCAT Preparation
a*
Biology Pol5rmerase ehain Reaction (pCR)
Passage V

25. A brief heat treatment is used to separate the double_


stranded DNA into single_strandedDNA. The
28. After seven cycles of the pCR (as depicted in
Figure
result would expect to see how many strands of
of this heat treatment should result in: }-gi"
DNA?
A. increased absorbance of light at 260 nm, an A. 64
effect krown as hyperchroiism.
B. increased absorbance of light at
B. 128
effect known as hyperanneaTing.
260 nm, an c. 256
C. decreased absorbance of lighl at
D. 612
260 nm, an
effect known as hypochromiim.
-D. decreased absorbance of light at 260 nm, an
effect known as hypoannealing.
29, The primers used in the polymerase chain
reaction:

4. are complementary to each other.


26. Separation of the pCR products in Figure 2 by gel
B. contain the same nucleotide sequence.
filtration chromatography would Ue U6Sf tit"ry C. should contain uracil.
result in the product of the:
to D. are designed using portions of the sequence
surrounding the region of DNA to be
A. amplified.
suitable clone emerging first, based on its
size.
B. suitable clone emerging first, based on its
charge.
C. unsuitable clone emerging first, based on its
slze.
D. unsuitable clone emerging first, based on its
charge.

7. Shown in the diagram below are four different DNA


melting curves for four different molecules of
duplex DNA:

\o
al
o
c!
-o

!6
o

C)
&

Temperature (.C) rv

Which of the four curves would be expected to have


an abundance of A-T base pairs?

A. I
B. II
C. n
D. IV

ryright @ by The Berkeley Review


The Berkeley Review
Speciatizing in MCAT preparation
Biology Lactose Operon Passage VI

Passage VI (Questions 30-36) Wild-type (normal) structural genes of the lac operon
are written as Z+Y+A+. This designation indicates that
Lactose is synthesized in the mammary gland by normal levels of the three proteins coded for by these
lactose synthase, an enzyme composed of two subunits, genes are synthesized. A mutation in any one of these
one with catalytic activity (galactosyl transferase) and one genes (e.g., Z-Y+ A+) could lead to an inactive form of the
without catalytic activity (a-lactalbumin). In human corresponding protein.
infants, this milk sugar is hydrolyzed by lactase, an
enzyme located in the epithelial cells of the small Independent mutations can also occur in the trans-
intestine. In some adult humans, lactase synthesis is acting (capable of activity on another chromosome)
inhibited, leading to a condition known as lactose regulatory gene and in the cis-acting (capable of activity
intolerance. only on the same chromosome) operator gene. Regulatory
gene mutations are designated as I- and are called
The bacterium E. coli car also utilize lactose, constitutive mutants. This type of mutation leads to an
especially if
glucose, the preferred carbon source, is not increase in the synthesis of the lac operon proteins in the
available. Metabolism of lactose requires galactoside absence of an inducer. In contrast, normal inducible
permease, a cotransporter of protons and lactose across bacteria synthesize the proteins of the lac operon only in
the bacterial plasma membrane, and p-galactosidase, an the presence of lactose. A constitutive mutation in the
enzyme that catalyzes the hydrolysis of lactose into operator gene is designated as Oc.
glucose and galactose.

The enzymes required for the catabolism of lactose in


E. coli are derived from genes organized into an operon.
The lactose (lac) operon contains three structural genes
designated as lac Z (B-galactosidase), lac Y (galactoside 30. Lactose intolerance can lead to all of the following
permease), and lac A (thiogalactoside transacetylase). The EXCEPT:
control region of the lac operon is adjacent to the lac Z
gene and contains a promoter (P) that binds RNA A. diarrhea.
polymerase, and an operator (O) that binds a repressor B. CO2 production.
protein transcribed from the (regulatory) Iac I gene. A C. epithelial cell uptake of water from the
genetic map of these sites is shown in Figure 1: intestinal lumen.
D. intestinalbacterial fermentation.

DNA I PlOllacZllacYllacA
c \c$c
CRP-cAMP

+s+
.
mRNA
site v
---\-r'--\-
c 31.
otla
Which of the following transport systems is MOST
Protein
likely to be associated with galactoside permease?

A. Active
Figure I B. Uniport
C. Symport
If lactose is absent from the medium, the lac repressor D. Antiport
binds to the operator, and RNA polymerase is unable to
transcribe the structural genes. When lactose is present in
the medium, it enters the cell and is converted to the
inducer, allolactose. The inducer binds to the repressor
and reduces the repressor's affinity for the operator. RNA
polymerase can now initiate transcription. 32. The process of synthesizing mRNA from DN-\
occurs with the use of a:
When glucose is in the medium, cAMP concentrations
in the cell are low and transcription of the lac operon is A. RNA-directed DNA polymerase.
reduced by 50-fold. F*owever, in the absence of glucose B. DNA-directed DNA polymerase.
the cAMP levels in the cell are high, allowing cAMP to C. RNA-directed RNA polymerase.
bind to the cAMP receptor protein (CRP). The CRP- D. DNA-directed RNA polymerase.
cAMP complex then binds upstream from the lac
promoter and increases the rate of transcription of the lac
structural genes by RNA polymerase.

Copyright @ by The Berkeley Review 454 The Berkeley Revien


Specializing in MCAT Preparatio'm
Biotogy Lactose Operon Passage VI

33. Which of the followin_g arrangements is necessary


36. Wild-type donor bacteria can be crossed with mutant
!9f9re transcription of the strictural genes can be recipient bacteria to produce partial diploid cells
initiated?
called m.erozygotes. The genotype of one
such
crossing is shown below:
A. Lactose must bind to the repressor before RNA
polymerase can bind to the operator.
B. DNA polymerase must bind to the repressor f ocz+y-A-
before the inducer can bind to the promoter. l-O+Z-y+A+
C. RNA polymerase must bind to the promoter,
Based on this genotype, all of the following genes
and the inducer must bind to the repressor.
code for enzymes that are produced constiiuiively
D. Allolactose must bind to the repressor, and the EXCEPT:
inducer-repressor complex must bind the
promoter. I. Lac Z
il. Lac Y
III. Lac A
34. A culture of E. coli is transferred from a growth
medium containing lactose to a growth medium A. I only
conraining glucose. Which of the f6llowing will
observed after the transfer is made?
be B. II only
C. III onty
A. The repressor
D. II and III only
will become inactive, cAMp
levels will increase, CAp will become active,
and B-galactosidase will be synthesized.
B. The repressor will become active. cAMp
levels will decrease. CAp will become
inactive, and p-galactosidase will not be
synthesized.
C. The repressor will become active, cAMp
levels will increase, CAp will become active,
and B-galactosidase will not be synthesized.
-'
D. The repressor will become inactive, cAMp
levels will decrease, CAp will become
inactive, and p-galactosidase will not be
synthesized.

35. The following haploid g€nome is found jn E. coli:


Lac I-O+Z+y+A+. which of the following enzyme
levels BEST represents this genome, boih in- the
presence and in the absence ofan inducer?

A.
lacZ lacy lac A
low low low
B.
lacZ lacy lac A
high low high
C.
lacZ lacY lacA
high low low
D.
lacZ lacY lac A
high high high

Copyright @ by The Berkeley Review


455 The Berkeley Review
Specializing in MCAT preparation
Biology Enhancers and Gene Expression Passage VII

Passage VII (Questions 37-43) The effbct of each mutation on transcription is tested
(Figure 3):
Experiments have shown DNA sequences near the

@o&
RNA start site are required for gene transcription. These
sequences are known as promoters and usually contain
the consensus sequence TATA. Interestingly, sequences
further upstream known as enhancer elements arre
required to give the same level of transcription as a
normal gene.

The activity of each enhancer and promoter can


0 ''T'"liii;T'"
mutant enhancer
generally be focused on a segment of DNA 100 to 200
nucleotide pairs long. The function of both regulatory
elements requires binding of specific gene-regulatory
proteins, which are often tissue-specific. In other words,
most regulatory elements function best in the particular t hour incubation
cell types that express the gene with which they are (3'7 "C) Production of CAT
normally associated. One example is the p-globin gene in activity is measured
chickens.
Figure 3
To identify gene-regulatory proteins that bind to the p-
globin enhancer, one must first determine the exact Results of the mutant enhancer activity (Figure 3) are
nucleotide sequence necessary for function (Figure l): then correlated with data showing the location of protein
binding sites for proteins in the normal enhancer. These
t
binding sites have been determined by gel-mobility shift
o z
3 assays and DNA footprinting. The results of the
4 comelation are shown in Figure 4:
o 5
6 Regions in enhancer DNA covered by proteins
-o
7
bo
*a
24
cd 25
26
27 l--- E 100
k-- 108 Nucleotide pairs ----*-+l o
o{n
L JV

Figure I tr
l!o
A series of mutated versions of the enhancer sequence
10 15 20 25
Mutant Number
can be joined (Figure 2) to a reporter gene whose product
is easily measured. Jhis reporter gene is a bacterial
enzyme called chloramphenicol acetyl transferase (CAT): Figure 4

These and other studies have led to the following


B-globin promoter general conclusions regarding enhancers and promoter
mid elements:

l. Each regulatory element consists of a specific


nucleotide sequence, which binds a corresponding
series of gene-regulatory proteins. Some of these
proteins are found only in specific tissues, while

Poly-A addition
signal
u, B-globin enhancer
(mutant)
2.
some are found in all cell types.

Some gene-regulatory proteins activate transcription


when they bind, while others inhibit transcription.

-). Enhancer and promoter elements seem to bind to


many of the same proteins.
Figure 2

Copyright @ by The Berkeley Review 4s6 The Berkeley Review


Specializing in MCAT Preparation
Biology Enhancers and Qene Expression Passage VII

37, The chicken p-globin enhancer lies downstream 41. A newly discovered regulatory element from a
from the p-globin transcription unit. It forms a vertebrate gene is analyzed, and it is found that
nuclease-hypersensitive site only in erythrocytes and many of the proteins that bind to the region have
no other cells. The MOST likely explanation for this been previously found regulating other genes. This
statement is: finding suggests:
A. the enhancer sequence is found only in A. promoters and enhancers affect transcription in
erythrocytes. similar manners.
B. gene-regulatory proteins found only in B. the net effect of a regulatory element depends
erythrocytes bind to the enhancer, displacing on the combination of regulatory proteins
nucleosome complexes. bound.
C. red blood cells are nuclease-hypersensitive, C. a relatively small number
of gene-regulatory
because loss of their nucleus is inevitable. proteins control transcription in higher
D. in other cell types, gene regulatory proteins eukaryotes.
bind only to promoters and not to enhancers. D. promoters and enhancers often act
synergistically in promoting transcription.

38. Histone octamers serve as a protein core around 42. Which of the following molecular complexes is
which double-stranded DNA can loop twice. The MOST likely to bind to a gene-regulatory element?
histone proteins are MOST likely to have a high
proportion of which of the following amino acids? A. Steroid hormone
B. Steroidhormone-receptorcomplex
C. Protein hormone
A. Valine and serine D. Proteinhormone-receptorcomplex
B. Glutamine and cysteine
C. Lysine and arginine
D. Histidine and isoleucine

43. According to Figure 4, which gene-regulatory


proteins make the largest contribution to the
enhancer's stimulation of transcription?
39. According to the protocol in the passage, how many
nucleotide pairs are changed in each enhancer A. AP2-like, NFl-like. CACCC factor.
mutant? B. Eryfl, CACCC factor, NF1-like.
C. API-like, AP2-tike, Eryfl.
A.2 D. AP2-like, CACCC factor, Eryft.
8.4
c. 21
D. 108

40. Figure 3 illustrates the testing of mutant enhancers


for their effects on transcription. In this experiment,
transcription is measured:

A. directly by quantifyirlg levels of RNA.


B. directly by recording rates of nucleotide
incorporation into RNA.
C. indirectly by moniroring levels of B-globin
protein.
D. indirectly by monitoring CAT enzyme activity.

Copyright @ by The Berkeley Review 457 The Berkeley Review


Specializing in MCAT preparation
Biology Rereplication Block and the Cell Cycle Passage VIII

Passage VIII (Questions 44-50) 44. When an S phase cell is fused with a Gl phase cell,
DNA synthesis is induced in the Gl phase nucleus.
In the normal S phase of a cell cycle, the genome must This information suggesrs rhar MOST likely the
be replicated exactly once and no more. However, in the transition from:
middle of a eukaryotic S phase, some parts of a
chromosome will not yet have started replication, while A. S to Gl is mediated by a diffusible acrivaror of
other regions will be completed. This presents an DNA synthesis.
enormous problem during the middle and late stages of B. S to Gl is mediated by a nondiffusible
the S phase. Some replication origins and their respective activator of DNA synthesis.
DNA sequences already used and duplicated are C. Gl to S phase is mediared by a diffusible
presumably identical to other replication origins not yet activator of DNA synthesis.
used. Since each replication origin may be used only once D. Gl to S phase is mediated by a nondiffusible
in each S phase, there must exist some form of record- activator of DNA synthesis.
keeping and regulation.

lnitiator proteins
45. The S phase of a eukaryotic cell can BEST be
DNA described as:

A. synchronous, with one origin of replication.


B. synchronous, with multiple origins of
Ji replication.
C. asynchronous, with one origin of replication.

urry _l L Iniriaror protein


D. asynchronous,
replication.
with multiple origins of

\.7 (inactivated) Fragments of bacterial DNA that replicate when


G2 injected into a fertilized frog egg can be shown to be
affected by the rereplication block. This
information:
G2
A. indicates the mechanism providing for a
rereplication block requires a highly specific
0 origin of replication.
indicates the mechanism providing
B. for a
rereplication block does not require a highly
specific origin of replication.
c. provides evidence for the activator-removal
model.
DNA can leplicate once again D. provides evidence against the activator-
removal model.
Figure 1

Cell fusion experiments have provided an important


clue to the answer to this problem. When an S phase cell 47. It can be inferred from information in the passage
is fused with a Gl phase cell, DNA synthesis is induced that a G2 nucleus fused with a S phase nucleus is:
in the Gl phase nucleus. In contrast, when the S phase
cell is fused with a G2 phase cell (a cell that has A. stimulated to begin DNA replication by some
completed the S phase), the G2 nucleus is not stimulated nondiffusible molecule bound to its DNA.
to replicate DNA, while the S phase cell continues DNA B. stimulated to begin DNA replication by some
synthesis. One model which could explain this diffusible molecule found in the nucleoplasm.
rereplication block is b'hsed on tightly bound initiator C. prevented from DNA replication by some
proteins, which are inactivated by the passage of a nondiffusible molecule bound to its DNA.
replication fork. This model, known as the activator- D. prevented from DNA replication by some
removal model, is illustrated in Figure 1.
diffusible molecule found in the nucleoplasm.

Copyright @ by The Berkeley Review 4sa The Berkeley Review


Specializing in MCAT Preparation
Biology Rereplication Block and the eell Cycle Passage VIII

48. According to the activator-removal model, the


addition of new initiator molecules occurs during:

A. GI.
B. S.
C. G2.
D. mitosis.

49. In several types of secretory cells found in fly


larvae, all the homologous chromosome copiei
remain side by side, creating a giant polytene
chromosome. In the salivary glands of Drosophila
larvae, each of the four chromosomes has been
replicated through ten cycles without separation of
the daughter chromosomes. How many identical
strands of chromatin are lined up side by side?

A. 128
B. 256
c. 5t2
D. 1024

50. The initiator proteins used in the activator removal


model:

A. are translated in the nucleoplasm and


transported into the cytosol.
B. are translated in the cytosol and diffuse across
the nuclear membrane.
C. contain a significant hydrophobic region of
amino acids.
D. contain a nuclear import signal.

opyright @ by The Berkeley


Biology Northern and Southern Blotting Passage IX

Passage IX (Questions 51-58) 53. Based on the information in the passage, detergents
are MOST likely used to inactivate:
To determine the nature of the defect in mutant mice
that produce abnormally low amounts of the protein A. proteases.
albumin, the following protocol can be followed: First, B. nucleases.
one collects tissue samples from defective and normal C. lipases.
mice. Using strong detergents, the cells are disrupted to D. cellulases.
inactivate cellular molecules. After removing proteins by
phenol extraction, one can precipitate the nucleic acids
with alcohol. To separate the DNA from the RNA further, 54. The RNA harvested from the normal mouse cells is
one can make use of their different solubilities in alcohol. MOST likely used to:

A technique known as Northern blotting can be used A. test the equipment.


to analyze the albumin-encoding RNAs. Intact RNA B. establish an experimental control.
molecules from defective and normal liver cells are C. establish the size of the mutant RNA
fractionated by gel electrophoresis. A replica of the gel is molecules.
blotted onto a sheet of nitrocellulose paper. Then, D. establish the amount of mutant RNA
radiolabeled DNA probes are incubated along with the molecules present.
paper so that the RNA encoding albumin hybridizes with
the correct DNA probe. The sheet is then washed
thoroughly afterward, and the hybridized probe is 55. Washing the nitrocellulose paper after incubation
detected by autoradiography. Using this technique, one with the probes removes all:
can gather information about the size of the albumin RNA
in both the defective and normal mice. A. radiolabelbeforeautoradiography.
B. radiolabelafterautoradiography.
By an analogous method, DNA instead of RNA can be C. background radiolabel before autoradiography.
examined. This technique is known as Southern blotting. D. background radiolabel afterautoradiography.
DNA fragments are separated according to their size by
gel electrophoresis, and complementary probes are
identified by blotting and hybridization. By repeating this 56. Shown below is a Northern blot indicating both
procedure with several different restriction endonucleases, defective and normal RNAs encoding for albumin.
a restriction map of a particular portion of the genome can Band A corresponds to the wild-type RNA, while
be constructed. band B corresponds to the defective RNA.

Band A
51. The protein albumin is MOST likely to be found in:
Band B
A.
B.
the cytosol-of white blood cells.
cerebral spinal fluid.
-
C. blood plasma. -
D. the cytosol ofred blood cells. These results indicate that the:

A. defective protein is smaller, and the


differences can be explained by a mutation in
the defective cell's RNA.
B. defective protein is larger, and the differences
52. The cells that produce albumin can be found in the: can be explained by a mutation in the defective
cell's DNA.
A. spleen. C. wild-type protein is smaller, and the
B. gall bladder. - differences can be explained by a mutation in
C. pancreas. the defective cell's RNA.
D. liver. D. wild-type protein is larger, and the differences
can be explained by a mutation in the defective
cell's DNA.

Copyright @ by The Berkeley Review 460^ The Berkeley Review


Specializing in MCAT Preparation
Biology Northern and Southern Blotting Passage IX

57. In the following experiment, two different restriction 58. In sickle-cell anemia, the exact nucleotide change in
endonucleases, ResEnd A and ResEnd B, are used to the mutant gene is known. DNA probes can be
create a restriction map of a lOkb DNA molecule: constructed for both the normal and defective genes
and used in prenatal diagnosis. A child's mother is
homozygous dominant, while the child's father is
10 kB DNA Molecule heterozygous for the sickle-cell defective gene.
What is the chance that both normal and defJctive
DNA probes will react with the child's genome?

A. 25Vo
B. 50Vo
C. 75Vo
D. IOOVo

Which of the following conclusions is consistent


with the data? [Note: The numbers indicated the
length of the DNA in kb, while the capital letters
indicate the restriction endonuclease cut sites.l

A.

rcAB
B.
BA
ffi

C.
. AB
ffi
21 7

D.
BA
ffi21
7

Copyright @ by The Berkeley Review


Biology Restriction EnzSrmes Passage X

Passage X (Questions 59-65) 60. Which of the following starements is TRUE of DNA
conservation?
About 99Vo of human DNA is identical between any
two individuals. The other lVo contains regions of A. DNA is highly conserved among human
hypervariability that differ dramatically from person to beings.
person. By examining these differences through DNA B. DNA is not highly conserved among human
fragmentation, it is possible to produce a unique DNA beings.
identification for each individual. C. DNA is completely conserved among human
beings.
One of the most important biochemical tools for DNA D. DNA differs dramatically from person to
manipulation, Typ" II restriction enzymes are often used person.
in producing fragments for analysis of DNA, to generate a
DNA "fingerprint." These enzymes recognize and cleave
double-stranded DNA at specific sequences of base pairs, 61. Although bacteria are the source of restriction
usually 4-8 bases long and palindromic (reading the same enzymes, their own DNA is protected against
forwards and backwards). cleavage by specific methylations. What is the
probable purpose of the bacterial restriction
By treating DNA with a series of restriction enzymes, enzymes?
a researcher produces precisely defined DNA fragments.
Since genetic differences between individuals create or A. To activate foreign DNA
eliminate restrictions sites, each person has a unique B. To inactivate foreign DNA
pattern of fragments. Differences also exist between C. To incorporate foreign DNA into the bacterial
corresponding segments of homologous chromosomes in genome
the same person. The fragments from chromosomes D. To promote cell lysis in response to foreign
treated with restriction enzymes are of different lengths DNA
and exhibit "restriction fragment length polymorphisms"
(RFLPS). The fragments can be separated according to
size by gel electrophoresis. 62. This diagram of gel electrophoresis results indicates
DNA fragments corresponding to the gene for the
To screen for an inherited disease, probes can be used beta chain of hemoglobin. The DNA fragments were
to check the fragmented DNA for the warning marker, produced by the restriction enzyme MstII. The
indicated by a unique, predicted cleavage pattern. parents are A and B, and their children are C and D.
Screening of fetal tissue for sickle-cell anemia and other Hb A refers to normal adult hemoglobin, and Hb S
inherited diseases can now be carried out. In the case of refers to mutated hemoglobin that causes the
sickle-cell anemia, the substitution of valine for glutamate sickling ofred blood cells.
at one site in the beta chain leads to the elimination of a
restriction site. This changes the pattern of fragments and
can be used as an identifier of inheritance of mutated
ABCD
genes.

In some crimiaal investigations, small samples of


tissue taken from the crime scene can be amplified with Hbs
the polymerase chain reaction (PCR), fragmented with
HbA
--I
restriction enzymes, and compared with samples taken --I
from suspects.

Figure I
59. Assuming DNA has an equal proportion of all bases,
Which of the following statemenrs are FALSE,
how many fragments would a restriction enzyme
based on the fragmentation pattern depicted in the
that recognized 4 base pairs produce from a DNA
gel shown above?
fragment that is.10,000 base pairs long?

A. 625
A. Child C does not have sickle-cell anemia.
B. 63
B. Both parents are heterozygotes.
c. 390
C. Child D does have sickle-cell anemia.
D. 39
D. The parents have a 50/50 chance of having a
child with sickle-cell anemia.

Copyright @ by The Berkeley Review 462 The Berkeley Review


Specializing in MCAT Preparation
Biotogy Kestriction EnzSrmes Passage X

63. In the gel clepicted


in Figure l, why do the children
have only one band, instead of two like
their
parents?

I. Each of the children carries only one copy


of
the gene for the beta chain of hemoglobin.
II. The children are homozygotes.
III. The two fragments mfurated to the same
location on the cell.

A. I only
B. I and II only
C. Ii and III only
D. I, II, and III

64. How can fetal tissue be collected in utero for


analysis ofrisk of inherited genetic diseases,/

A. Collect fetal cells in amniotic fluid by


amniocentesis.
B. Collect fetal cells from the maternal blood
supply.
C. Perform a therapeutic abortion.
D. Perlorm a Cesarean section.

65. For which of the following projects would


resrriction
enzymes be used advantageously?

Isolating and sequencing DNA


L Solving a paternity dispite
I!
III. Genetic counseling foipotential parents

A. I only
B. I and II only
C. Il and Ill only
D. I, II, and III

)opyright @ by The Berkeley Review


463 The Berkeley Review
Speciatizing in MCAT freparation
Biology Arabinose Operon Passage Xl

Passage XI (Questions 66-72) 68. The following reacrion is carried out by rhe product
of the araA gene:
The arabinose operon is found in bacteria and is
responsible for the bacteria's ability to use arabinose as a CHO CH,OH
source of energy. The operon (Figure 1) consists of a
I t-
H- C- OH c=o
regulator gene, three control sites, and three structural I
.-
I

genes. Production of the C protein (from the araC gene) is


dU-L-n HO-C- H
I I

under control of the araOq operator. Transcription of the HO- C- H HO_C- H


I I
araC gene occurs when the level of C protein and cAMP- CH,OH cHroH
CAP complex is low. CAP is the catabolite gene-activator
protein found in many inducible operon systems. If the The enzyme that catalyzes this reaction is:
level of cAMP-CAP complex is low while the level of C
protein is high, a C protein binds to the araOl site and A. an epimerase.
inhibits transcription of the araC gene. B. an isomerase.
C. a dehydrogenase.
The transcription of the structural genes B, A, and D is D. a kinase.
dependent on the formation of a DNA loop. If cAMP
levels are low and C protein levels are high, a C protein 69. Which of the following conceprs is NOT illustrated
binds to both the araO2 and ara I
sites by forming a by the arabinose operon?
hairpin turn within the DNA. AraI is adjacenr to the
promoter for the structural genes, and a bound C protein A. A protein can alter its own production by
blocks transcription of those genes. In the situation where altering the transcription of its gene.
both arabinose and cAMP are abundant, the hairpin loop B. Structural genes are regulated only by DNA
is not formed. The reason is that the C protein with bound sites found in contact with those genes.
arabinose has an altered conformation and binds to both C. Changes in transcription brought about by a
araOl and aral, but not to araO2. In addition, the cAMP- signal molecule are reversible.
CAP complex binds between the araOl and aral sites. The D. Proteins can act as inhibitors or activators of
result of these changes is a polymerase that is now able to transcription, depending on the presence of a
bind to the araBAD promoter site and promote signal molecule.
of these genes. The products of these
transcription
structural genes promotes the uptake of arabinose and 70. High levels of cellular cAMP are associated with:
converts it to a usable form ofenergy.
A. high levels of arabinose.
B. low levels of arabinose.
araC I arao2 | araOl I aral I araB I araA I araD C. high levels of glucose.
Li-_ D. low levels of glucose.

Regulator
l-L___f__j
Control Structural 71. The polymerase responsible for the transcription of
gene sites genes the structural genes in the arabinose operon is most
likely:

66. The synthesis of the C protein is BEST described as: A. RNA polymerase.
B. RNA polymerase L
A. unregulated. C. RNA polymerase II.
B. deregulated. D. RNA polymerase IIL
C. regulated by arabinose.
D. autoregulated. 72. It has been discovered that the genes responsible for
the uptake of arabinose are not in the operon shown
67. An organism lacking the araC gene would exhibit: in Figure 1. This may have selective value, because:

A. increased use of glucose as a source of cellular A. there is an increase in the amount of arabinose
energy. transported into the cell.
B. a lack of cAIvF production. B. there is a preferential use of glucose over
C. inefficient transcription of structural genes ara arabinose as a cellular energy source.
B, A, and D. C. the design leads to a more efficient metabolism
D. increased use of arabinose as a source of of arabinose.
cellular energy. D. a mutation in one area would not affect the
integrity of the other region.

Copyright @ by The Berkeley Review 46,4 The Berkeley Review


Specializing in MCAT Preparation
Biology Gene Therapy Strategies Passage XII

Passage XII (Questions 73-79) 74. A plasmid is transfected into a cell line with the aim
of blocking the expression of a wild-type
Our modern understanding of the molecular nature of chromosomal gene called Xl. In order for this to be
genetic expression has paved the way for potential accomplished, the plasmid would have to contain:
breakthroughs in the field of gene therapy. It is now
possible for us, at least in vitro, to prevent the expression A. a mutant Xl gene sequence with a strong
of disease-causing genes. Additionally, the same promoter.
techniques can prevent the expression of viral genes, B. a mutant Xl gene sequence with a weak
thereby preventing cellular production of new viruses. promoter.
Two major approaches have been at the forefront of gene C. a wild-type Xl gene sequence that is inverted
therapy research in recent years. with respect to its promoter.
D. a wild-type Xl gene sequence that is separated
The first takes advantage of the mRNA molecule's from its promoter.
ability to form double-stranded RNA-DNA hybrids.
Artificial DNA can be made that is complementary to the
"sense" mRNA of a disease-causing or viral gene. This 75. There is much debate over the therapeutic potential
complementary sequence is termed antisense DNA. The of ribozymal versus antisense gene therapy. Which
antisense and sense strands can hybridize to each other, of the following would represent advantages of the
preventing translation of the disease-causing or viral ribozymal approach?
protein product. DNA, rather than RNA, is used to make
antisense strands, because it is inherently stabler. I. Base-paired RNA is less easily degraded than
Additionally, to protect it from enzymatic degradation, synthetic single-stranded RNA.
the DNA's phosphate backbone is replaced with a II. Ribozymes act catalytically.
synthetic compound called methylphos phonate. III. Individual ribozymes are very broad in their
substrate specificity.
The second approach relies on the ability of certain
RNA molecules to act enzymatically and cleave mRNAs A. I only
at specific sequences. These RNA enzymes are referred to B. II only
as ribozymes. Ribozymes, like IRNA molecules and C. I and II only
ribosomes, have a tertiary structure resulting from base- D. I and III only
pairing of part of the RNA strand to itself. Artificial
ribozymes can be designed to cleave HIV viral RNA at
specific sites, preventing it from being translated and 76. Which of the following would NOT affect the Ku of
thereby slowing the reproduction of the virus. a ribozyme in vitro?

A. A change in the concentration of substrate


B. A large change in the pH of the medium
C. A decrease in the salt concentration of the
medium
D. An increase in the temperature of the medium

73. Which of the following could NOT be treated 77. Of the two gene therapy techniques described in the
effecrively by either of the gene therapy techniques passage, which would be the MOST successful in
discussed in the passage? combating a rapidly mutating strain of the HIV
virus?
A. Tumors caused by overexpression of an
oncogene A. The antisense approach, because mutation in
B. Sickle-cell anemia caused by a homozygous the viral DNA would not affect antisense-
mutation in the hemoglobin gene sense binding.
C. Liver disease caused by the hepatitis B virus B. The antisense approach, because DNA-RNA
D. Herpes caused by the herpes simplex retrovirus hybrids would preferentially form with HIV
mutants.
C. The ribozyme approach, because ribozymes
cleave all mRNA.
D. The ribozyme approach, because ribozymes
can target conserved regions of HIV mRNA.

Copyright @ by The Berkeley Review 46s The Berkeley Review


Specializing in MCAT Preparation
Biology Gene Therapy Strategies Passage XII

78. Which of the following could be a problem with


antisense gene therapy that would occur in human
patients, but not in human tissue culture?

A. Lysosomal digestion of synthetic antisense


DNA molecules
B. Immune rejection of synthetic antisense DNA
molecules
C. Failure of DNA-RNA hybridization due to
cellular pH levels
D. Degradation of antisense DNA by restriction
enzymes

79. What might be a normal function of naturally-


occurring ribozymes?

A. Splicing of unprocessed mRNA


B. Proofreading of single-stranded DNA
C. Catalysis of protein reacrions
D. Cleavage of viral DNA

Copyright @ by The Berkeley Review 466 The Berkeley Keview


Specializing in MCAT Preparation
Biology Tryptophan Operon Passage XItr

Passage XIII (Questions 80-87) 80. The fact that the mRNA in the tryptophan operon is
polycistronic indicates one:
The tryptophan operon, including the promoter (p),
operator (o), attenuator (a) control sites, genes for the A. protein is coded for by multiple genes.
leader sequence (L), and the DNA encoding the enzymes B. protein is coded for by a single gene.
(A-E) needed to convert chorismate into tryptophan (trp), C. mRNA molecule codes for more than one
is shown in Figure l. These five enzymes are synthesized protein.
by the translation of polycistronic trp mRNA. D. mRNA molecule codes for only one protein.

P o la trpE trpD trpC trpB trpA

L Structural genes 81. As it interacts with the trp operon, tryptophan is


BEST described as:
Figure 1
A. an inducible repressor.
One way to regulate the Trp operon is through a B. an inducer.
specific repressor protein. The repressor protein is C. a corepressor.
encoded by the trpR gene, and it binds to the operator D. an attenuator.
region of the trp operon only when tryptophan itself is
bound to the repressor molecule. The repre ssor-
tryptophan complex binds specifically to a region on the
operator that overlaps with the promoter region. The
result is that no mRNA is produced. 82. In an experiment with the Trp operon, deletions
were made between the operator and the gene for the
It was discovered that a 162-nucleotide leader first enzyme (trp$, resulting in a significant
sequence preceded the initiation codon of trpE, the first increase in the production of trp mRNA. This
enzyme encoded in the operon. In nonmutants with high experiment provides evidence for:
tryptophan levels, a transcript of only 130 nucleotides is
found. In nonmutants with scarce trp levels, the mRNA A. signaltransduction.
transcript product is 7000 nucleotides long. Investigators B. attenuation.
concluded that the transcription of the trp operon is C. tryptophan-repressorcomplex repression.
regulated by a controlled termination site called an D. tryptophaninduction.
attenuator. The site is located between the operator and
the gene for the first enzyme. The terminator site has a
GC-rich sbquence followed by an AT-rich sequence.
Each region exhibits a twofold axis of symmetry.
83. The binding of the tryptophan-repressor protein to
The attenuator site works with the generator site in the operator prevents interaction between:
regulating the transcription of trp genes. When tryptophan
is plentiful, initiation of transcription is blocked by A. DNA polymerase and DNA.
binding of the trp-repressor complex to the operator. B. RNA polymerase and DNA.
When trp levels fall, some polymerase molecules C. RNA polymerase II and DNA.
dissociate from the template at the attenuation site, while D. RNA polymerase III and DNA.
others proceed to synthesize the entire trp message.

The presence of tryptophan residues at positions l0


and 1 1 of the leader polypeptide is significanr in the
attenuator site's ability to deieCt the level of tryptophan in 84. According to the passage, it is important for
the cell. When tryptophan is abundant, this complete attenuation that:
polypeptide is synthesized, which will terminate
transcription. When tryptoplan is scarce, the ribosome A. replication be coupled to transcription.
stalls at tandem UGG codons because of a lack of B. replication be coupled to translation.
tryptophanyl tRNA. The stalled ribosome alters the C. transcription be coupled to replication.
structure of the mRNA so that the polymerase D. transcription be coupled to translation.
transcribing it proceeds beyond the attenuator site.

Copyright @ by The Berkeley Review 467 The Berkeley Review


Specializing in MCAT Preparation
Biology Tryptophan Operon Passage XItr

85. The melting of the trp attenuator site on DNA could


be described as:

A. continuous, as the nucleotide population is not


varied.
B. continuous, as the nucleotide population is
unimportant.
C. discontinuous, with the A-T rich region
melting after the G-C rich region.
D. discontinuous, with the A-T rich region
melting before the G-C rich region.

86. Several other operons for the biosynthesis of amino


acids in E. coli are known to have an attenuator site.
In each case, the leader peptide should contain:

A. tandem pairs of the codon UGG.


B. a relatively small number of the codon UGG.
C. an abundance of amino acid residues of the
kind synthesizedby the operon.
D. a relatively small number of residues of the
kind synthesized by the operon.

87. Increasing the number of residues in the leader


peptide of the kind synthesized by the operon
MOST likely:

A. decreases the sensitivity of the detection


system.
B. increases the sensitivity of the detection
system.
C. eliminates the need for attenuation-
D. serves only to alter the primary structure of the
protein.

Copyright @ by The Berkeley Review 46a The Berkeley Review


Specializing in MCAT Preparation
Biology Ttanslation Passage XIV

Passage XIV (Questions 88-94) 90. The BEST experimental method to separate
polyribosomes from single ribosomes is through:
The expression of genetic information often results in
the production of protein molecules from the translation A. addition of ribonucleases.
of messenger RNA (mRNA). To initiate protein synthesis, B. centrifugation.
initiating factors (IF) bind ro the small ribosomal unit, C. acidic wash.
along with a special initiator transfer RNA molecule. D. addition of radiolabeled amino acids.
This complex recognizes the mRNA start codon. The
recognition of the start codon causes the release of the 91. In a prokaryotic cell, the special initiator aminoacyl
initiating factors. This is followed by the binding of the transfer RNA molecule that binds to the start codon
large ribosomal subunit and results in a functional transports which amino acid?
ribosome.
A. Glycine
Multiple initiating events, involving multiple B. Glutamine
ribosomes, can occur on a single mRNA transcript. The C. Asparagine
resulting polyribosome/polysome structure gives multiple, D. Formylmethionine
simultaneous translations of a single mRNA transcript.
An important eukaryotic IF is eIF-2, which is 92. A polycistronic mRNA molecule is MOST likely to
instrumental in binding the initiator transfer RNA onto the be found in a cell that is:
small ribosomal subunit during the initiating phase. In
reticulocytes, eIF-2 is believed to be involved in A. eukaryotic and contains many start codons,
controlling the overall rate of protein synthesis, with translated separately into multiple proteins.
phosphorylation of one of its subunits reducing eIF-2 B. .eukaryotic ard gives rise to one polypeptide,
activity. cleaved into many functional proteins.
C. prokaryotic and contains many start codons,
Translation is not an error-proofprocess. Roughly 1 in translated separately into multiple proteins.
every 104 amino acids that are incoiporated into proteins D. prokaryotic and gives rise to one polypeptide,
is inserted incorrectly. The fidelity of this process is cleaved into multiple functional proteins.
contingent on several processes, one being the correct
pairing of bases in the codons in mRNA and anticodons in 93. The delay in elongation, caused by the elongation
the transfer RNA. Cells have evolved a proofreading factor, acts as a proofreading mechanism, because:
mechanism for this pairing process involving the
elongation factor, a protein that binds to an incoming A. the delay allows the time needed to insert the
aminoacyl-transfer RNA and to a molecule of GTp. This correct amino acid.
factor has the ability to separate in time codon/anticodon B. the delay allows for hydrolysis of GTp, which
base-pairing and peptide elongation. This short delay attaches the correct amino acid to the correct
between the two events allows for a bound-transfer RNA transfer RNA.
molecule to be discharged from the ribosome. C. incorrectly bound transfer RNA molecules
form weaker codon/anticodon interactions and
88. In a hypothetical elperiment, eukaryotic cells are are more likely to dissociate from the
starved. Based on information in the passage, one ribosome.
would expect increased activity from: D. the delay allows for hydrolysis of GTp, which
drives the peptide-forming reaction at the
A. 'a mutase. ribosome.
B. a kinase.
C. a dehydrogenase. 94. Which of the following sratements is FALSE, based
D. an epimerase. on information in the passage?

A. The initiator transfer RNA binds to the start


89. Given that the average size of a protein molecule is codon in the P site of the ribosome.
400 amino acids, one can conclude that there should B. Initiation factors are not needed for secreted
be a translation error in one out of how many protein proteins, because these proteins are translated
molecules? r and transported in the endoplasmic reticulum.
C. The anticodon found on the initiator transfer
A. 10 RNA is CAU.
B. 25 D. The elongation factor is reversibly bound to
c. 50 the aminoacyl transfer RNA molecule.
D. 100

Copyright @ by The Berkeley Review 469 The Berkeley Review


Specializing in MCAT Preparation
Biology Cancerous Gene Passage XV

Passage XV (Questions 95-100) 96. The tag sequence used in the above experiment
should:
In theory, one could compare the base sequence of all
chromosomes in a normal cell to those in a cancerous cell, A. normally be fbund in the mouse cell line.
in order isolate a mutation suspected of causing cancer. B. be an unknown, random region of DNA.
However, such a method is entirely impractical. To isolate C. not normally be found in E. coli.
the mutation more efficiently, one could exploit the D. not normally be found in human DNA.
process of expressing genetic information. This is
accomplished by using a particular line of mouse cells,
which stop proliferation when the population of cells on
the medium is one cell-layer thick. This is a normal 97. The Alu sequence is MOST likely used as a tag
process, as these cells are sensitive to their population becauseit is:
density. However, the cells can be induced, through the
addition of DNA, to continue multiplying and to enter a A. rare within the human genome.
state of uncontrolled growth. B. easily mutated.
C. likely to be near a cancer-causing, mutated
The added DNA responsible for inducing the gene.
cancerous state is extracted from human tumors and is D. easily inserted into the mouse cell line.
tagged before it is added to the mouse cell line. Individual
cells transformed by human DNA into cancerous cells
form a pile of cells known as a focus. Once a focus has
appeared, DNA from these focus cells is extracted and 98. Mutated gene A has been shown to induce
cloned in E. coli. The colonies of E. coli can then be uncontrolled growth. In vivo studies of the gene
screened for the presence of the tag. The screen involves have demonstrated normal levels of mRNA and
the addition of radiolabeled DNA complementary to the protein product from gene A. From this information,
tag sequence. it can be concluded that:

One particular method used to tag the human tumor DNA


takes advantage of the A/a sequence found throughout the A. the mutation in gene A has resulted in a
entire human genome. Human DNA is cleaved using A/r; depressed RNA polymerase binding
restriction sites and is added to the mouse cell line. After frequency.
the appearance of a focus and cloning in E. coli, a B. protein product A is necessary for entering
radiolabeled probe complementary to the restriction site is mitosis.
used to isolate a DNA sequence containing not only the C. the mutation in gene A has resulted in a
tag, but the mutated gene responsible fbr inducing a significant structural change in protein A.
cancerous state. D. the cell line used possesses a retarded cellular
ribosome level.

99. Hybridization between complementary DNA


sequences takes advantage of:

A. covalent bonds.
B. hydrogen bonds.
95. Comparing the entire genome of a wild-type cell to a C. hydrophobic base stacking interactions.
cancerous cell in order to isolate a mutation is D. ionic bonds.
impractical, because:

A. it is theoretically impossible to sequence an


entire genome. 100. To screen for the presence of the DNA tag, which of
B. it is more important to characterize the faulty the following experimental methods should be used?
proteins and nbt the DNA mutations.
C. naturally occurring mutations would mask A. Southern transfer
cancer-causing mutations. B. Northern transfer
D. the human genome varies among individuals. C. Western transfer
D. Gel electrophoresis

Copyright @ by The Berkeley Review 470 The Berkeley Review


Specializing in MCAT Preparation
Biology Expression of Genetic Information Section X Answers

A is correct. Even if
we don't know the answer, we can eliminate the three wrong answers in this question.
Proteases degrade proteins, not DNA. Choice B is incorrect. Nucleotide synthases are involved in the synihesis of
nucleotides. Choice C is incorrect. DNA gyrases are involved in nicking and reattaching DNA so it does not tangle.
Choice D is inconect. Restriction enzymes cleave DNA at specific, often palindromic-, sequences. Their cleavige
points are predictable, and they therefore are often used to cleave DNA for further study. The correct choice is A.
, D is correct. DNA does not form a quad structure. Choice B is incorrect. Phosphates bear many negative charges,
and they do not become positively charged. Choice A is incorrect. Denaturing means that somehow the DNA
structure is changed. The base-pairing is not altered, but the hydrogen bonds holding the base pairs together break
and the two DNA strands separate. choice c is incorrect. The correct choice is D.

3. B is correct. In many types of gels, the electrophoretic mobility of a fragment is inversely proportional to the
number of base pairs in it (i.e., its size). The buffers used often produce a similarity in charge between molecules, so
choice A is incorrect. The radioactive molecule used in the Southern blot is employed afterihe gel, in Step 4. Choice
C is thus incorrect. The linking number is the number of times one strand of DNA *rupt *ouid anothei in a right-
handed direction, so choice D is also incorrect. The correct choice is B.

4. C is correct. Choices A and B are the same answer, just reversed. Both are incorrect. Choice D is a repeat of the
original strand and is incorrect. A probe is the complementary base sequence to the fragment of intirest. The
correct choice is C.

5. A is correct. This is really an easy, read-the-figure type of question. Although you are not told what the fragments
mean, you can still test each statement against the figure. One of the fragments corresponds to a region that iniicates
the inheritance of the HC gene. Neither parent cirries Fragment V, so choice B is incorrect. Neither child carries
Fragment III, so choice C is incorrect. The children have different fragmentation patterns, indicating different
patterns of inheritance. Choice D is incorrect. Finally, both children have FragmentII, so they would be affected if
this is the HC fragment. The correct choice is A.

6. D is correct. The three restriction sequences for the three restriction endonucleases are given in Table l. Let's place
each of those restriction sites in a section of DNA. Consider the Psr I site first. Aiter the enzyme cuts ai the
appropriate site, we are left with two sticky ends. Both of those sticky ends show a piece of single-stranded DNA,
which bears a 3' end. This is sometimes referred to as a 3'-overhang. Based on this observation, we can eliminate
choice A.
J
5 [----rcTGCAGr------t 3' 5'r------r 5'-Gr-----r
DNA
r--------l GACGTC t-----t _\ CTGCA-3'
-----l c-s'
3'
3'
-t 5''--------v' 3'.1- 3'-ACGTC t-----r 5'

Psr I site Sticky end

Consider the Sal I site next. After the endonuclease makes its cut we are again left with two sticky ends. However,
this time the sticky ends can be found at the 5' of each DNA strand. Choice B is a correct answer.

J
DNA
5 5'r-----r c-3' s'-TCGAC t-------t 3'
3' t-----_1 CACCTC I-----__-l 5' 3'r------l CAGCT-5' t-----r
t \-a-J
3'-G 5'

Sal I site Sticky end

However, before wetiush into picking choice B, let's take a look at choice C. If we use the same type of reasoning
for the Taq I restriction endonuclease, we also find 5' sticky ends. Therefore, both Sal I and Taq I pioduce 5' stick!
ends. The correct choice is D.

7. D is correct. If we assume that the DNA sequence is random, then any given base (A, T, G, or C) has a l/4
probability of occurring at any position in the polynucleotide chain. Table I tells us that Alu I contains 4 bases. We

Copyright @ by The Berkeley Review


Biology Dxpression of Genetic Information Section X Answers

would expect to find an Alu I restriction site every 256 bases along the length of the DNA. This is calculated as
follows: (l/4)n=0/4)4=l/256,wherenisthelengthoftherecognitionsite(i.e.,thenumberofbasepairs).The
fength of pBR322 is 4.3 kb or 4,300 base pairs. Remember, I kb is 1,000 base pairs. Dividing 4,300 by 256 gives the
expected number of sites that Alu I would cut in pBR322. The correct choice is D.

8. D is correct. We are asked to determine the number of unique restriction endonucleases that can cut a particular
segment of DNA. All we need to do is compare those restriction sites in Table I with the DNA sequence in the
question.
SmaI
X*or sorl
5'-CGGATCCCGGGTCGACG-3'
3'-GCCTAGGGCCCAGCTGC-5'
BamHl TaqI

Be aware of restriction sites that can be cut by two different enzymes, and be aware of smaller restriction sites
within larger restriction sites. In the segment of DNA shown above, the heavy lines indicate the location of the
various restriction sites. The correct choice is D.

9. B is correct. First, let's consider the sticky ends created by the restriction endonuclease BamHL This is shown
below:
J
DNA
5 5'r------rG-3' 5'.-GATCC r------t 3',
3'l---__-lccTAGG t----t 5' 3'r------___lccTAG-s' 3'-cr--------l 5'
1 \-a-J
BamHI site - Sticky end

We want to use a restriction endonuclease that can cut a segment of DNA and give us sticky ends complementary to
those created by Bam HI. Let's immediately examine the answer. The sticky ends created by BgtII are sho*n below.

t
DNA
W 5 AGATCTW! 5'WA-3' s'-GATCT W3'
3'wrcr;;;*;,+ 1
3'WTCTAG-s'
\-.--' 3'-AW5'
Bgi II site Sticlry end

Next, we can combine one of the sticky ends from BamHI with one of the sticky ends from Bgl II as shown below.
An interesting thing happens when we do this: After hydrogen bonding has taken place and the phosphodiester
backbone hasJeen reestablished, we lose the restriction sites for both Bam HI and Bgl IL

Bgl II sticky end

DNA
5 r-----rc-3' 5'.GATCTW3' L
\ 5't-----.l GGATCT M 3',

3',t-------__l ccTAG-5' 3',-AWs', " 3':-ICCTAGA W 5'

-.,J
Bamlil, sticky end uoE#ur,r,
sites are lost

The other three choices (IlindlIl, PstI, and SacI) do not give DNA with sticky ends that are complementary to
those given by B: HI. The correct choice is B.

10. A is correct. We are digesting two linear DNA molecules. Let's call those DNA molecules DNA #1 (for Lane 1) and
DNA #2 (for Lane 2). Note that if we cut DNA #1 with EcoP.I (see below), we get six fragmenrs, each a differenr
size. This must mean that we have five Eco RI restriction sites in DNA #1.

Copyright @ by The Berkeley Review 472 The Berkeley Review


Specializing in MCAT Preparation
Biology Expression of Genetic lnformation Section X Answers

EcoRl Eco RI EcoRl


DNA#1
...... E

DNA#2 Y'.. F'


Eco RI Eco RI EcoRI New acd Rr EcoRl
EcoRl
If we cut DNA #2.with
.Eco P.r, we get seven fragments, each a different size (see above). This means that there are
six Eco RI restriction sites in DNA #2. Note thaiFragment X from DNA
#2 ii a little larger than Fragment D from
DNA #l' Also note that Fragment Y is a little smalGr than Fragment D, but a
little larger than Fragment E from
DNA #l' Fragments c and E in DNA #2.are the same as Fragments C and E in
DNA #1. we can conclude from this
that more DNA and an additional resffiction site were added it Fragment
D of DNA #1.
If we were to delete DNA from Fragment D of DNA #1 (without adding a new restriction
site), we would still get
six fragments' This is not what we see in DNA #2. Eliminate choice g. *Ttre
same would be true if we were to add
DNA only to Fragment D of DNA #l (without adding a new restriction site).
Btiminate ctroice c.
If we delete DNA from Fragment D of DNA #1, and as a result a new
Eco RI site forms, then we get seven
fragments' This is what we are looking for. However, once we delete
DNA fr;. p*g.*n, D, the new fragment that
forms (call it x'Y') is smaller than thJoriginal Fragment D. Afrer electrophoresit,
irii, n"* Fragmenr X,y, is below
the level of Fragment D on the gel. Furthirmor", i? *" now cut at the
new rco Rt restriction site within Fragment
x'Y" the resulting two fragments still end up below the level of Fragment D ufd"i;;;.ophoresis.
we observe in Lane 2 of the gel for the fragmintation of DNA #2. Thecorrect This is not what
choice is A.
11' B is correct' The first thing to do when creating a restriction map is to determine
the size of the DNA. If we add the
fragment sizes in the first lane (created by a sin-gle digest with Bgilr),
we get 7 kb + 2 kb = g kb. If we follow this
same procedure for the remaining fragmentation tanei, we get
s ku for of ttem. rnis tells us our fragmented
bacterial DNA was originally o tlu in tength. Look at all of"the restriction"uihmaps in rhe answer
fragment sizes add up to 9 kb? yes, they do. choices. Do all the

The next step is to look at the fragmentation sizes in each of


the six digest lanes of the gel to see whether they match
up with the answer choices..A single digest with_ just BglII gives
frag-ment ri"", oii [u una 2 kb. we observe rhis
fragmentation pattem in choices B and C. A single aigeJt witln just
Eco RI gives fragment sizes of 5 kb and 4 kb. we
observe this fragmentation pattern in choices i unin. At this poinr
in ;r.;;;6*, oi"" B looks like a good
answer. But let's make sure. "t
A single digest with just Ps/ I gives fragment sizes of 6 kb, 2 kb, and I kb.
we observe this fragmentation pattern in
choices A' B' and D. A double digest uiing bo$ Ec9 RI and ngi[gives
fragment sizes of 4 kb, 3 kb, and 2 kb. we
observe this fragmentation pattern in choices B and D. A douEle
dlgest usii'g il#;; i and BguI gives fragmenr
sizes of 6kb,2 kb, and I kb. we observe this fragmentation pattern
in cholces A and B. Finally, a double digest
using both EcoP.r h'nd PslI_gives fragment sizes oi3 kb (there are two
of them, since the band is twice as thick), 2
kb' and 1 kb' we observe this fragmintation pattern in ciroices A, B, and c.
find that each digest lane gives thehagmentatibn pattern shown in choice B.
irt.r.t"tidering all the choices, we
The correct choice is B.

t2. D is correct' This is a question designed to make sure we understand the roles
of the three RNA molecules in the
cell' A ribosome is composed of amino acids and rRNA, and is essentially
a surface for assembling proteins.
statement I is correct' .lN{ is the transcript or copy of DNA. statement
II is correct. IRNA is the amino acid
carrier for protein translation. Statement III is correct. The correct choice
is D.
13. D is correct' The focus of this question is the mutation of HIV. To combat
this research problem most effectively,
one should try attac.ks- simultaneously by linking several ribozymes together.
-multiplb This would cause the most
damage in the first strike and also would attack any mutaiions that
survived and tried to reproduce. Following one
enzyme with another two weeks later could still permit mutations
to occur, so choice B is inconect. Increasing the
concentration of the original ribozyme does noi combat mutation, because
the survivors would still be able to
recolonize the patient' choice A is incorrect. A specific ribozyme for each patient
would not aif'ect further mutation,
so choice C is incorrect. The correct choice is D.

Copyright @ by The Berkeley Review 473 The Berkeley Review


Specializing in MCAT preparation
Biology Expression of Genetic lnformation Section X Answers

14. B is correct. Choice A is incorrect, because highly conserved regions do not have much variability from strain to
strain. HIV can be attacked by ribozymes at any number of sites, not just these conserved regions, meaning that
choice C is also incorrect. And eliminate choice D: These regions cannot attract ribozymes. The action leading to
catalysis is a chance meeting of two molecules that fit together. The correct choice is B.

15. B is correct. Prior to mutation, there are fewer copies of a virus if its mRNA is decreased. Each mRNA does not
become more efficient at translation. Choice A is incorrect. The number of transcripts is not increased, so viral
particles cannot increase. Choice C is incorrect. Choice D refers to mutant (competing) HIV mRNAs, and the
question specifically asks about changes prior to mutation, so choice D is incorrect. The correct choice is B.

16. A is correct. There are only four different nucleic acids used to build every RNA molecule. An enzyme has access
to twenty different amino acids. This increase in the number of potential subunits gives much more variety in
structure and function to protein-based catalysts. Statement I is correct. Enzymes did not outcompete ribozymes by
chopping them up. They merely worked better and were therefore selected by evolutionary pressure. Statement II is
incorrect. Enzymes do not change the equilibria of reactions, so statement III is incorrect. The correct choice is A.

17. B is correct. Catalysts can participate in many reactions, because they remain unchanged themselves while effecting
change in other molecules. Statement I is correct. Catalysts do increase the rate of reactions, but not the equilibrium,
meaning that statement II is correct. And since we know that the body does synthesize catalysts, statement III is
incorrect. The correct choice is B.

18. C is correct. A single-base code can code for four different amino acids. One can arrive at this answer by knowing
that there are four different bases used in DNA and that ifthe code had only one base, then only four different amino
acids could be coded. Accordingly, a three-base system can code for a total of sixty-four amino acids (4 x 4 x 4 =
64). The question asks for the difference between the two, making 64 - 4 = 60 the correct answer. The correct
choice is C.
19. C is correct. To answer this question, one has to know what is meant by an overlapping genetic code. As in the
question, let us assume the base sequence is ATCGGTATA. If this genetic code is overlapping, the first three amino
acids come from ATC, TCC, and CGG respectively. If the code is nonoverlapping, the first three amino acids come
from ATC, GGT, and ATA respectively. Since the question asks about the products of an overlapping code (the
code is, of course, non-overlapping), as many as three amino acids may be affected. The correct choice is C.
20. C is correct.Without the extra base inserted, the base sequence can generate four amino acids. Remember, the code
is nonoverlapping. Let us look at the amino acids produced with the extra base inserted. The first two amino acids
are not affected by the insertion. However, the third amino acid is. Without the insertion, the triplet is ACA. With
the insertion, the code is ATC. The same logic applies to the fourth amino acid. Thus two amino acids are affected.
The correct choice is C.
21. B is correct. The passage states that part of the protocol of the experiment is to wash the protein-containing
precipihteand measure the level of carbon 14. One can assume we are measuring the radiolabel in the protein. If
this is the case, we must use radiolabeled amino acids as part of the experiment, because this is the only way we can
incorporate radiolabel into the protein. Do not lose track of the big picture. We are controlling the mRNA added and
asking what amino acids are called for. This means we need to keep track of amino acids, so radiolabeling them is
essential. The correct choice is B.
)) C is correct. The passage states that a component of the experiment is the addition of DNAse, which results in a
cessation of protein synthesis. This implies that protein synthesis was occurring and then ceased, and it should be
indicated on a graph by a line with a positive slope followed by a line with zero slope. This alone should eliminate
all choices except C. Looking at the graph in C, the zero-slope line is followed by another rise in protein synthesis.
This occurs because we added our own synthesized mRNA, which created the new protein. Remember, that is the
point of the experiment--the ability to synthesize protein in a cell-free system. The correct choice is C.
23. A is correctl Let us look at the data in Table 1. With phenylalanine, we see a l00Vo relative amount incorporated.
Remember, the label is on the uracil, and therefore we can conclude that phenylalanine is coded by a triplet
consisting entirely of uracil. The question asks about glycine. Glycine has the lowest count, according to the data.
The fact that it has some count invalidates the idea that the triplet is all guanine. Furthermore, there cannot be an
equal amount of the two bases in a triplet design. Since the count is low, the most logical conclusion is that there is
more guanine than uracil in the code for glycine. The correct choice is A.

Copyright @ by The Berkeley Review 474 The Berkeley Review


Specializing in MCAT Preparation
Biotogy Dxpression of Genetic Information Section X Answers

24. B is^correct' Although DNA polymerase I from E coli could be used in the pcR reaction, the scheme would be
inefficient, because each round of denaturation of the double-stranded DNA
helix to form the individual single
strands would result in destruction of the polym erase. Thermus aquaticus
is a thermophl", ana because of its normal
living conditions, it contains a temperature-resistant polymerase. This polymerar.
*ithrtund, temperatures of 95" c,
which is necessary to denature the DNA. The use oi thir polymerase aliows for
increased specificity between the
hybridized primers and the template' This is so because a temperature of 45"
cit n.""rru.y ror the hybridization. At
this temperature, the polymerase is inactive. Therefore, *" ,uir" the temperature
to 75' C, at wtrich the polymerase
becomes active. By this point, those primers that are not hybridized correctly
will dissociate from the incorrect sites.
Fven though some of the correctly hybridized primers fali off, a much gr.ut..
fr";;i;;;i;.;mers on incorrecr sires
fall off' The reaction becomes more specific. Again, the polymerase fr6m E. coli.unnotiuk"
the heat used in the
denaturation process. The correct choice is B.

25. A is correct' The melting of DNA is readily monitored by measuring its absorbance of
light at 260 nm. The
unstacking of base pairs.th.at occurs during the melting pro."rr results in
an increased absorbance of light at this
wavelength' This effect is known as hypeichromism. tie spontaneous reassociation
of separated complementary
strands of DNA is known as annealing. Therefore, this eliminates choices
B and D. To discriminate between choices
A and c, one must draw on their knowledge of biochemistry. As stated above, the
increased absorption due to base-
pair unstacking is known as hyperchromism. The correct choice is
A.
26. A is correct' First, we must realize that a suitable clone would contain the inserted
fragment. This is why we are
using PCR' we are screening clones to see if a piece of DNA was in-serted
into the plasmid. The larger piece of
DNA thus be represents the suitable clone, becauie the foreign piece of DNA was
draw on our knowledge of gel filtration chromatograptry.The apparatus used td"dyl;r"rred. Next, we must
in tlis setup includes a column
consisting of porous beads made of an insolubte uut trig-trty hydraied polymer.
Small moiecules can enter these
beads, but larger ones cannot. In the beads, the movement of the small moiecul"r
ir tt"i;;d. The result of this is
that large molecules flow more rapidly through this column and emerge first (because
a smaller volume is accessible
to them)' Therefore, the product of the suitible clone, alarge mole"cule, emerges
first because of its size, not its
charge. The correct choice is A.

27. A is correct' we are looking for the DNA molecule that is expected to have an
abundance of A-T base pairs. The
melting temperature ofa.DNA molecule depends heavily on its base composition.
DNA molecules rich in G-C base
pairs have a higher melting temperature (defined as hali of helical structure
lost) relative to those DNA molecules
with a high A-T base pair content. The reason for this lies in the number of hydrogen
bonds between the bases, with
three between G-c and only two between A-T. In addition, adjacent G-C
base paiis interact more strongly with one
another than do adjacent A-T base pairs. The DNA melting Ju.u", show
absorbance as a function of temperature.
Remember that the absorbance riie is indicative of a d'ouble-stranded molecule
becoming a single-stranded
molecule' We are looking for the molecule that is going_to have the highest A-T
bur;-p;; In other words, it
is going to have the lowest melting point temperature. ihis correspond"s to the curve
on the"oni"",.
far left, as its absorbance
rise occurs at the l6west te-pe.utuie. The correct choice is A.

28. C is coffect. According to Figure l, one cycle of PCR uses one molecule of DNA to
make two molecules of DNA.
Another round of PCR uses two molecules of DNA to make four molecules of DNA.
In other words, the number of
molecules of DNA made during n cycles is 2n. Therefore, after seven cycles of the pCR
reaction, we should see l2g
molecules of DNA. However, the question asks how many strands of DNA exist
after seven cycles. Each DNA
molecule is made up of two strands. So 128 x 2 256. The correct choice is c.
=
29. D is correct' The primers used in the PCR scheme are complementary to the template
DNA, which contains the
region of DNA to be amplified. They are not complementaryio each other. This
eliminates choice A. Furthermore,
th-ese primers probably are not going to contain the same nucleotides.
Their sequen".r d;;;J;n the sequence of the
DNA region with-which they are hybridizing. This eliminates choice g. Rs
for choice c, the primers are DNA
oligonucleotides. Thdrefore, they do not contain uracil. only RNA contains uracil
in place #ttry.in". choice c can
be.elimjnated. The only logical choice is D. The primers aie complementary
to the iegions of DNA that flank both
sides of the piece of DNA to be amplified. one p.i*". is complementary to
one side,-while the other DNA primer
hybridizes with the other. only then do we get ihe amplification of DNA that
we are interested in. Therefore, the
primers are designed to conform to the regioni of DNA ihat surround the DNA
to be amplified. The correct choice
is D.

Copyright @ by The Berkeley Review 475 The Berkeley Review


Specializing in MCAT preparation
Biology Expression of Genetic Information Section X Answers

30. C is correct. If lactose cannot be utilized by the epithelial cells, then stays in the intestinal lumen. Lactose acts as a
solute in the lumen and begins to draw water out of the lumenal epithelial cells (the reverse of choice C). Too much
water in the lumen can leid to diarrhea. Also, bacterial cells have the ability to take up lactose and metabolize it
anaerobically (fermentation). One of the products of fermentation is COz, which is a gas. The correct choice is C.

31. C is correct. The second paragraph of the passage says, "Metabolism of lactose requires galactoside permease, a
cotransporter of protons and lactose...." If protons and lactose are cotransported, it means that they are allowed to
pass thiough the iell's membrane together. The transport of lactose into the cell is coupled to the energy stored in the
irunr*.1nd.une potential that residei across the plasma membrane. Recall from metabolism that protons are pumped
out of the cell is the electrons move down the electron transport chain. Not only is a large membrane potential
established, but a high concentration of protons now resides in the medium outside the cell. These gradients of
stored energy can beirsed to import sugars like lactose into the cytosol. When molecules are transported into a cell
together (oi-out of a cell together), the process is referred to as a symport.If molecules move in opposite directions
as"they are transported through the ceil's membrane, the process is referred to as an antiport.If one molecule is
transpbrted through the cell's membrane, the process is referred to as a uniport. Active transport requires the
hydrolysis of ATP. There was no mention of ATP hydrolysis in the passage. The correct choice is C.
32. D is correct. In the third paragraph of the passage, we learn that RNA polymerase binds to the promoter (P). The
promoter, as shown in Figure 1 of the passage, is a section of DNA. In the fourth paragraph, we learn that RNA
polymerase can transcribe the structural genes (i.e., can make mRNA). The structural genes also involve a section of
bNA. Sin"" we know that RNA polymerase synthesizes mRNA, we can eliminate choices A and B. Is the RNA
polymerase directed to synthesize mRNA by RNA or DNA? The polymerase is binding DNA. Therefore, it is the
bNA that is directing the RNA polymerase as it synthesizes the mRNA. The correct choice is D.
33. C is correct. In the fourth paragraph of the passage, we learn that "if lactose is absent from the medium, the lac
repressor binds to the operaior, and RNA polymerase is unable to transcribe the structural genes." This tells us that
in order to make a tranicript, we must remove the repressor from the operator. How do we remove the repressor?
We need an inducer (see the fourth paragraph again). Lactose can enter the cell and be converted into allolactose,
which is the inducer. Lactose itself is not the inducer. It is allolactose (an isomer of lactose) that is the true inducer.
Once the inducer binds to the repressor, the inducer-repressor complex is no longer active, and it no longer binds to
the operator. With this informaiion alone, we can eliminate choices A and B. RNA polymerase can bind to the
p.o-bt... The inducer-repressor complex (choice D) does not bind to the promoter. If it could, RNA polymerase
would not have a place to occupy in order to begin transcription. Once RNA polymerase binds to the promoter and
the repressor has been removed from the operator (by complexing with the inducer), the DNA is free of
obstructions, and transcription can begin. The correct choice is C.
34. B is correct. We learned what happens to lactose in the first four paragraphs of the passage' What happens to
glucose is contained in the fifth paiagraph. When glucose is in the cell, the cAMP concentrations start to decrease.
F.ight u*uy, we can eliminate choices A and C. We next learn that transcription of the lac operon is reduced some
SO-tota. Inother words, the synthesis of B-galactosidase is greatly reduced. This is what choices B and D state. We
need something else. If the levels of p-galactosidase decrease, it must mean that there is a repressor on the operator.
The repressor-must be active in order to be occupying the operator. This allows us to eliminate choice D. Let's
continue (even though we now know the answer). If the cAMP levels in the cell are low, there is not enough cAMP
to bind the CRp. Tlie cAMp-CRp complex does not form. Therefore, the CRP cannot be activated. It will remain
inactive until the levels of cAMP once again begin to increase. The correct choice is B.

35. D is correct. The key to this question involves the I- mutation of the FO+Z+Y+A+ genome. In the last paragraph of
the passage, we find ihat th"re types of mutations are called constitutive. They lead to an increase in the synthesis of
lac ope.oi proteins in the absenie of inducer. If there is no inducer around, we would expect the repressor to bind to
the operator. However, the repressor is defective due to a mutation. It cannot bind to the operator. Thus, in the
absenie of inducer, we would expect to find a lot of the lac Z, lac Y, and lac A proteins being synthesized.

lacZ lacA lacY


high high high
We would also expect to see high levels of these gene-product proteins being synthesized in the presence of inducer
as well. The induier does not bind to the operator, so in this case it does not matter whether it is present. The
correct choice is D.

Copyright @ by The Berkeley Review 476 The BerkeleY Keview


Specializing in MCAT Preparation
Biology Expression of Genetic Information Section X Answers

36, D is correct. Based on the genotype, which enzymes are produced all the time? Let's break the partial diploid into
two haploid segments. Consider the top genotype first (I+OCZ+Y-A-). The I+ gene produces a normal ."p..rro.,
which in turn binds to the operator, O. In the absence of an inducer, the repreisor bind, to the operator, and
the
structural genes are not transcribed. In the presence of an inducer, the represior does not bind the operatol
and the
structural genes are transcribed. However, there is a mutation in the operitor, Oc, that allows the structural genes
to
be transcribed all the time., The lac Z+ gene is normal and produces a good protein. The lac y and lac A- glnes
are
mutants. The proteins produced from these genes are defective. Therefore, lhe top genotype permits onlyihe
lac Z
gene to be constitutively produced.

Consider the bottom genotype next (I-O+Z-Y*A*). The f gene does not produce a normal repressor. The mutant
repressor is not able to bind to the normal operator. In the presence of an inducer, this is not a problem. The inducer
does not bind to the operator anyway. Since the repressor is a mutant, and since nothing is binding the operatoi,
the
structural genes can be transcribed. However, the lac Z- gene bears a mutation, and the protein is defective. The lac
Y+ and lac A+ genes are normal and give good proteins. In the absence of an inducer, the operator region is still
wide open. we get a defective lac z- protein and good lac y+ and lac A+ proteins.

Based on this, we might conclude that all three gene products are made constitutively. However, we are dealing
with
a partial diploid genotype and not individual haploid genotypes. Let's take a look at the combination
of th-e two
genotypes in the partial diploid:

lo+z-Y+A+
The last paragraph of the passage states that "independent mutations can also occur in the trans-acting (capable
of
activity on another chromosome) regulatory gene and in the cis-acting (capable of activity only -on the same
chromosome) operator gene." This tells us that the product of the lac I gJne is Aiffusible. In oiher words, once
the
repressor is synthesized by. the normal lac I+ gene, it can diffuse thoughihe cytosol and bind to the
operator, which
is O+. In the absence of an inducer,, the repressor remains bound to thaioperator, and transcription cannot proceed.
an inducer becomes available, it binds to the repressor and removes it from the operator. This is refirred
If
to as
induction' and this is what we see occurring in the case of the lac Y+ and lac A+ genes. The only gene that is
synthesized constitutively is the lac Z+ gene. The correct choice is D.

37. B correct. Some regions of DNA that lack a nucleosome (histone protein complex) can be detected by treating
-is
cell nuclei with trace amounts of DNase I. At low concentrations, the nuclease digeits long stretches of nucleosome-
free DNA, but not the short stretches of linker DNA between nucleosomes. Manf of the SNase-hypersensitive
sites
in cell chromatin are located in the regulatory regions of a gene, and more of these sites are present in cells where
the gene is active than in other cells. Most such sites are believed to represent regions from which a nucleosome
has
be_en displaced bgrr sequence-specific DNA-binding proteins involved ln eukary6tic gene regulation.
Knowing this
information, it is most likely that gene-regulatory proteins found only in erythrocytet-UinA toitr" enhancer andiause
displacement of the nucleosome.

Cbnsider the other answers. Alt cells have the same genome, so the argument that only the RBC has the enhancer
does not work. Therefore, we can eliminate choice A. Choice C is simply a false statement. Just because the cell will
eventually shed its nucleus does not mean that the biology of an intact nucleus can be ignored. Finally, it is not true
that gene regulatory proteins in other cells bind only to promoters and not to enhancers.-Most likely, ih.." *" g"n"-
regulatory proteins that bind to both regulatory elements. The point here is that only in red blood cells do we fin-d the
particular gene-regulatory protein that binds to the enhancer and causes displacement of the nucleosome. The
correct choice is B.

38. C is correct. If welare looking for proteins that can bind to DNA, the amino acids that make up those proteins
should carry a positive charge to create an electrostatic attraction. We are therefore looking for baiic aminb acids,
because they carry positive charges at physiological pH. The two basic amino acids in ou-r choice of answers are
lysine and arginine. Therefore, we should see a high proportion of these amino acids in histone proteins. The
correct choice is C.

Copyright @ by The Berkeley Review 477 The Berkeley Review


Specializing in MCAT Preparation
Biology Dxpression of Genetic Information Section X Answers

39. B is correct. This problem requires a proper interpretation of Figure 1. We are looking at a sequence of 108
nucleotide pairs. The figure tells us that we make 2J mutant B-globin enhancers. Therefore, a block of four
nucleotide pairs is changed in each mutant. The correct choice is B.

40. D is correct. Looking at the figures, one should be able to deduce the following: We are inserting a mutant B-globin
enhancer in a test plasmid. The oligonucleotide and the cloning site are joined by the enzyme DNA ligase. The
enzyme produced after placing the plasmid inside a leaky erythrocyte is the bacterial enzyme chloramphenicol
acetyl transferase, which is easily assayed. One should realize that we are not measuring the rates of RNA synthesis
directly. We are measuring transcription in an indirect way. Therefore, we can eliminate choices A and B. Also, we
are not producing the p-globin protein; we are using the mutated p-globin enhancer and producing another protein
(CAT), which is assayed. We can thus eliminate choice C. We are indirectly testing the mutant enhancers for their
effect on RNA transcription by measuring the amount of protein that the recombinant gene produces (as CAT
enzyme activity). The correct choice is D.

41. C is correct. We are told that a newly defined regulatory element from a vertebrate gene is analyzed, and that many
of the proteins that bind to it (gene-regulatory proteins) turn out to be previously described. This can suggest only
that there is a relatively small number of gene-regulatory proteins, and that they may control transcription in higher
eukaryotes. Some of these gene-regulatory proteins are AP2, ATF, SPl, and NFl. None of the other conclusions can
be derived from the claim made in the question. All of the other choices are general conclusions believed to be true.
Nonetheless, the information in the question would support only the inference drawn in choice C. The correct
choice is C.

42. B is correct. The major difference between protein and steroid hormones is that protein hormones bind to receptors
on the surface of the cell and bring about cellular change via secondary messengers. Therefore, we can eliminate
choices C and D. Steroid hormones, which are quite lipid-soluble, have the ability to traverse the lipid bilayer. When
they do, they bind to a receptor molecule located in the cell. The receptor molecule has a region for binding the
steroid itself and a region for binding a DNA molecule. When the steroid binds to the receptor to make a steroid
receptor complex, the complex becomes active and binds to a region of the DNA known as a regulatory element.
The steroid itself cannot bind DNA, because the DNA-binding region is on the receptor. The correct choice is B.
43. C is correct. One must look at Figure 4 and interpret the data carefully. The y-axis is percent enhancer activity. Let's
look at the protein NFl-like. If we mutate the region where NF1-like proteins bind, we see 100 percent enhancer
activity. This means that NF1-like proteins do not play a part in this enhancer's activity. In other words, we took
away NF1-like activity and nothing changed. We can conclude that this protein does not make a large contribution
to the enhancer's activity. Therefore, we are looking for mutations that resulted in a low percentage of enhancer
activity. This would indicate that these proteins make a large contribution to the enhancer's activity. From Figure 4,
we see that AP1-like, AP2-like, and Eryfl proteins do make a large contribution. We know this because when we
mutate the region where these proteins normally bind, the enhancer activity goes way down. The correct choice is
C.

44. C is correct. We fused the two cells together, and we found that the Gl phase cell nucleus began to replicate DNA.
Therefore, we are talking about a transition from G1 to S, and not from S to Gl. The cell cycle goes from Gl to S.
Based on this information, we can eliminate two of the choices. Next, we ask whether this activator is going to be
diffusible or not. If the molecule is not diffusible, it would most likely be bound to the DNA of the S phase cell. In
that case, it could not act upon the DNA of the Gl nucleus. The activator must be diffusible, because it can set the
G2 nucleus into an S-phase activity, namely DNA replication. The correct choice is C.
45. D is correct. Let us first decide whether the replication is synchronous or not. The passage tells us that during the S
phase, some parts of a chromosome will not yet have been replicated, while others are finished. This describes
something asynchronous. Furthermore, we are talking about a eukaryotic cell that is known to contain multiple
origins of repl,ication because of the volume of genetic material found in its chromosomes. The correct choice is D.
46. B is correct. One can assume that the replication origin for a frog looks different than one for a bacterium. However,
the bacterial DNA is still subject to a rereplication block. We know nothing of how this block is achieved, so
choices C and D can be eliminated. Nevertheless, since the rereplication block affects the DNA of the bacterium,
one can assume that the mechanism does not require a specific origin of replication. The correct choice is B.

Copyright @ by The Berkeley Review 47A The Berkeley Keview


Specializing in MCAT Preparation
Biology Dxpression of Genetic lnformation Section X Answers

47. C is correct. First, we have to decide whether the G2 nucleus is stimulated or prevented from performing DNA
replication. From the passage, we are told that this fusion does not result in the rereplication of the G2 nucleus.
Therefore, we can eliminate answers claiming that the G2 cell is stimulated to begin DNA replication. The next
question becomes whether the molecule responsible for the prevention is bound or diffusible. If the molecule were
diffusible, it would seem logical that the S phase cell would cease DNA replication. The passage tells us differently,
however, so we can infer that the molecule is bound to the DNA. The correct choice is C.
48. D is correct.Look at Figure l. In the G2 phase, the proteins are not on the genetic material. In the Gl phase, they
are. The only phase in between those two is mitosis. This makes sense. We want to be able to initiate DNA synthesis
again only after mitosis is complete. The two new daughter cells will go on to divide. The rereplication block must
be removed at or near the time of mitosis. The correct choice is D.

49. D is correct. This question is very straightforward. We are told that one chromosome has been replicated through
ten cycles without separation, so how many strands of chromatin are now side by side? One chromosome replicating
makes two strands of chromatin. Two strands of chromatin replicating makes four strands, and so forth. Two to the
tenth power is 1024. The correct choice is D.

50. D is correct. The protein bound to the DNA in the model must get there somehow. Recall that in eukaryotic cells,
transcription and translation are separated by a nuclear membrane. The mRNA transcript is translated in the cytosol.
In order for the protein to reenter the nucleus, it must cross back over the nuclear membrane. This is not
accomplished by simple diffusion. Recall that the nuclear membrane contains nuclear pores that allow proteins to
enter the nucleus. The pore recognizes the protein as belonging in the nucleus by means of the protein's nuclear
import signal, and the protein is then transported into the nucleus. The correct choice is D.

51. C is correct. Albumin is a plasma protein found in an average concentration of 4 gm/dl. Albumin provides the
critical osmotic pressure that regulates the passage of water and diffusible solutes through the capillaries. When the
concentration of albumin is severely reduced, excess extracellular fluid may accumulate in the extravascular tissues.
This condition is known as edema. Albumin can also act as a carrier protein for things like bilirubin and fatty acids.
Based on this information, one can eliminate the other possible answers. Albumin is not likely to be found in the
cytosol of white or red blood cells. Furthermore, there is no evidence that the protein albumin would not be found in
the cerebral spinal fluid. The correct choice is C.
<) D is correct. The cells that produce albumin are the hepatic parenchymal cells. Thus, the cells that produce albumin
are found inside the liver. This is a very straightforward question that tests previously acquired knowledge not
found in the passage. The correct choice is D.

53. B is correct. This answer comes from an understanding of the goal of our experiment. We want to amplify the
nucleic acids of these liver cells so we can do further experimentation. Therefore, it is important for us to protect
those molecules we want to use later. We can protect the nucleic acids by inactivating the nucleases. Consider the
other answers. Would we want to inactivate the proteases? It probably does not matter. We are going to do phenol
extractions to rerffive protein. Furthermore, we are not interested in the protein, just the nucleic aiids.lherefore, we
can eliminate choice A. Again, we are not interested in inactivating lipases, as long as the nucleic acids are not
threatened. Therefore, we can eliminate choice C. Finally, human cells do not have cellulases (enzymes able to
ddgrade the molecule cellulose), so choice D cannot be right. Therefore, the protection of the nucleic acids is central
to our experiment, and thus inactivation of the nucleases is fundamental. The correct choice is B.

54. B is correct. The RNA from the normal mouse cells is used as an experimental control. For example, when we run a
gel of the two RNAs (one from defective cells and one from wild-type cells) we have a means of comparison, or a
control. Let's consider the other possibilities. Choice A is silly. We do not need the RNA to test the equipment. Both
choices C and D hinge on the word "directly." We cannot use this normal RNA to measure the size of the mutant
RNA directly. We will be able to say whether one is bigger or smaller from the results of the gel, but the actual size
determination comes from running standards (pieces of RNA whose size is known already) on the gel as well. The
same holds true for ihe amount. The normal amount cannot be used to measure the amount of mutant RNA directly.
(Although, if we so desired,.we could measure the amount of mutant RNA directly.) The correct choice is B.

55. C is correct. From the passage, it can be determined that the wash is done before the autoradiography. From this
alone, one can eliminate choices B and D, because they claim the wash is done after autoradiography. This ofcourse

Copyright @ by The Berkeley Review 479 The Berkeley Review


Specializing in MCAT Preparation
Biology Expression of Genetic Information Section X Answers

47. C is correct. First, we have to decide whether the G2 nucleus is stimulated or prevented from performing DNA
replication. From the passage, we are told that this fusion does not result in the rereplication of the G2 nucleus.
Therefore, we can eliminate answers claiming that the G2 cell is stimulated to begin DNA replication. The next
question becomes whether the molecule responsible for the prevention is bound or difTusible. If the molecule were
diffusible, it would seem logical that the S phase cell would cease DNA replication. The passage tells us differently,
however, so we can infer that the molecule is bound to the DNA. The correct choice is C.
48. D is correct.Look at Figure 1. In the G2 phase, the proteins are not on the genetic material. In the Gl phase, they
are. The only phase in between those two is mitosis. This makes sense. We want to be able to initiate DNA synthesis
again only after mitosis is complete. The two new daughter cells will go on to divide. The rereplication block must
be removed at or near the time of mitosis. The correct choice is D.

49. D is correct. This question is very straightforward. We are told that one chromosome has been replicated through
ten cycles without separation, so how many strands of chromatin are now side by side? One chromosome replicating
makes two strands of chromatin. Two strands of chromatin replicating makes four strands, and so forth. Two to the
tenth power is 1024. The correct choice is D.

50. D is correct. The protein bound to the DNA in the model must get there somehow. Recall that in eukaryotic cells,
transcription and translation are separated by a nuclear membrane. The mRNA transcript is translated in the cytosol.
In order for the protein to reenter the nucleus, it must cross back over the nuclear membrane. This is not
accomplished by simple diffusion. Recall that the nuclear membrane contains nuclear pores that allow proteins to
enter the nucleus. The pore recognizes the protein as belonging in the nucleus by means of the protein's nuclear
import signal, and the protein is then transported into the nucleus. The correct choice is D.

51. C is correct. Albumin is a plasma protein found in an average concentration of 4 gm/dl. Albumin provides the
critical osmotic pressure that regulates the passage of water and diffusible solutes through the capillaries. When the
concentration of albumin is severely reduced, excess extracellular fluid may accumulate in the extravascular tissues.
This condition is known as edema. Albumin can also act as a carrier protein for things like bilirubin and fatty acids.
Based on this information, one can eliminate the other possible answers. Albumin is not likely to be found in the
cytosol of white or red blood cells. Furthermore, there is no evidence that the protein albumin would not be found in
the cerebral spinal fluid. The correct choice is C.

<) D is correct. The cells that produce albumin are the hepatic parenchymal cells. Thus, the cells that produce albumin
are found inside the liver. This is a very straightforward question that tests previously acquired knowledge not
found in the passage. The correct choice is D.

53. B is correct. This answer comes from an understanding of the goal of our experiment. We want to amplify the
nucleic acids of these liver cells so we can do further experimentation. Therefore, it is important for us to protect
those molecules we want to use later. We can protect the nucleic acids by inactivating the nucleases. Consider the
other answers. Would we want to inactivate the proteases? It probably does not matter. We are going to do phenol
extractions to ren,6ve protein. Furthermore, we are not interested in the protein, just the nucleic aiids.lherefore, we
can eliminate choice A. Again, we are not interested in inactivating lipases, as long as the nucleic acids are not
threatened. Therefore, we can eliminate choice C. Finally, human cells do not have cellulases (enzymes able to
ddlrade the molecule cellulose), so choice D cannot be right. Therefore, the protection of the nucleic acids is central
to our experiment, and thus inactivation of the nucleases is fundamental. The correct choice is B.

54. B is correct. The RNA from the normal mouse cells is used as an experimental control. For example, when we run a
gel of the two RNAs (one from defective cells and one from wild-type cells) we have a means of comparison, or a
control. Let's consider the other possibilities. Choice A is silly. We do not need the RNA to test rhe equipmenr. Both
choices C and D hinge on the word "directly." We cannot use this normal RNA to measure the size of the mutant
RNA directly. We will be able to say whether one is bigger or smaller from the results of the gel, but the actual size
determination comes from running standards (pieces of RNA whose size is known already) on the gel as well. The
same holds true for ihe amount. The normal amount cannot be used to measure the amount of mutant RNA directly.
(Although, if we so desired, we could measure the amount of mutant RNA directly.) The correct choice is B.

55. C is comect. From the passage, it can be determined that the wash is done before the autoradiography. From this
alone, one can eliminate choices B and D, because they claim the wash is done after autoradiography. This of course

Copyright @ by The Berkeley Review 479 The Berkeley Review


Specializing in MCAT Preparation
Biology Dxpression of Genetic Information Section X Answers

would make no sense, because it is the radiolabel that makes the autoradiography process possible. If we washed
away all the radiolabel after the autoradiography, then we would see no autoradiography results. If we washed the
background away after the autoradiography, our results would not be as accurate as they could be. This leaves us
with choices A and C. Do we wash all the radiolabel away? Of course not. This would give us no results. We wash
the background away only to get better results. Those radiolabeled pieces of DNA that do not really belong (i.e.,
they are not truly hybridizing to the RNA) but they still manage to bind to something anyway should be washed
away. The correct choice is C.

56. D is correct. From the picture, one can see that the RNA from the defective cell ran farther down on the gel. This
indicates that the protein is indeed smaller. Therefore, based on this information alone, one can eliminate choices B
and C, because they claim the opposite case. In choice A, we are told the defective protein is smaller, but the defect
is explained as arising from a mutation in the RNA. This does not hold. Mutations in RNA come from sloppy
transcription and should not be permanent. It is not right to say that mutant DNA gave rise to mutant RNA. The
faulty RNA is just the transcript of the faulty DNA. In other words, the mutation is in the DNA, the information
molecule. The correct choice is D.

57. B is correct. The map should show that the restriction nuclease B cuts 2kb to the right of the left end of the
molecule. In addition, the restriction nuclease A cuts 3kb to the left of the right end of the molecule. This is the only
combination that fits with the given data. If we switch the sites (meaning A is where B is), the data would not fit,
because cutting with A gives a piece of DNA that is 2kb long and one that is 8kb long. The correct choice is B.

58. B is correct. In order for both probes to react, the child must be a heterozygous. The heterozygous child will be in
possession of a normal and a defective copy of the gene. So understanding this, we are really only looking for the
probability of the child to be heterozygous for the trait. In running the cross, one would find that there is a 50Vo
chance that this is true. The correct choice is B.

59. D is correct. Restriction fragments, in general, are 44 = 256 base pairs long. Divide 10,000 by 256 to get
approximately 39. Choice C is off by a factor of 10 and is incorrect. Choices A and B are other math mistakes, and
they are incorrect. The correct choice is D.

60. A is correct. Approximately 997o of human DNA is identical among all human individuals. This is highly conserved
DNA, but not completely conserved. Choices B and C are thus incorrect. Only l%o of DNA differs dramatically from
person to person, so choice D is incorrect, too. The correct choice is A.

61. B is correct. The probable function of restriction enzymes is defense. The bacterial cell can inactivate foreign,
invading DNA (perhaps from viruses) before the invader has a chance to grow and lyse the bacterial cell or to
incorporate itself into the bacterial genome. (Neither of these possibilities is particularly desirable for the host.)
Choices A, C, and D are incorrect. The correct choice is B.

62. D is corred?. Child C has two copies of normal HbA, so this child does not have sickle-cell anemia. It is a
homozygote for HbA, thus choice A is true. Both parents carry one copy of the gene for HbA and one copy of the
gene for HbS. This means they areheterczygotes, so choice B is also true. Child D has 2 copies of the sickle cell
anemia gene, and therefore has sickle-cell anemia: Choice C is correct. Confirming that choice D is the false (and
therefore, the best) answer requires applying our knowledge of standard genetics to the heterozygous parents. AS x
AS yields 25Vo AA, 50Vo AS, and 25Vo SS. This gives a 25Va chance of these parents having a child with sickle-cell
anemia. Choice D is false. The correct choice is D.

63. C is correct. The children have to carry two copies each of the gene, one on each of their homologous
chromosomes. Statement I is thus incorrect. Unlike the parents, the children are homozygotes, having two identical
copies of the gene. C is homozygous for HbA, and D is homozygous for HbS. Therefore, statement II is correct.
There is only one spot in the lane on the gel, since the two identical copies migrated to the same spot. On a real gel,
the spots for the homozygous child should be twice as dense as those single ones for the homozygous parents.
Statement III is correct. The correct choice is C.

64. A is correct. Amniocentesis is a common procedure performed on some pregnant women. A very long needle is
inserted through the abdomen and the uterus to collect samples of amniotic fluid. Cells from the fetus slough off into
the fluid, and their DNA can be studied after cleavage with restriction enzymes. Fetal cells are not routinely moving

Copyright O by The Berkeley Review 4AO The Berkeley Review


Specializing in MCAT Preparation
Biology Expression of Genetic lnformation Section X Answers

around in the mother's blood supply, and it would be difficult to separate the two for analysis if this were the case,
so choice B is incorrect. An abortion would not help an in utero diagnosis. Choice C is incorrect. A Cesarean section
is used for some deliveries, not fbr tissue collection. Choice D is incorrect. The correct choice is A.

65. D is correct. Since DNA is long and fragile, it is usually chopped into small fragments for study and sequencing
with restriction enzymes, so statement I is correct. Although parents and offspring have differeni DNA, tirere arJ
plenty of similarities in the variable regions that could be used to establish piternlty along with other techniques.
That means statement II is also corect. So is statement III: Parents who havi a familial history of genetic disease
can be screened for major, identified genetic diseases to provide a better understanding of what -problems their
children may face. The correct choice is D.

66. D is correct. The passage tells us that the production of the C protein is under the control of the araOt operator.
Furthermore, the passage states that if the level of C protein is high, the protein binds to the araOt operator site and
inhibits transcription of the ara C gene. In other words, protein C is regulating its own production. Therefore, the
synthesis is autoregulated. The correct choice is D.

67. C is correct. According to the passage, transcription of the structural genes takes place when the hairpin loop is not
formed. The passage also explains that the loop is not formed because arabinose bound to the C protein aliers its
conformation and the C protein binds to both the aral and araOl site. This, along with changes in the cAMp-CAp
complex, brings about transcription of structural genes. It is clear from this information that the C protein is
necessary for transcription. Therefore, a organi.sm lack the araC gene would exhibit inefficient transciiption of
structural genes ara B, A, and D. The correct choice is C.
68. B is correct. One can see from the drawings that accompany the question that the enzyme catalyzed a reaction in
which the structure but not the atomic composition of the substrate has been altered by moving a group from one
position to another within the same molecule. This is describing the work of an isomerase. ttre correct choice is B.

69. B is correct. We know from the passage that a hairpin loop of DNA can be formed that prevents the transcription of
the structural genes. The hairpin loop intimately involved the ara Oz site, which is certainly not touching the
structural genes on either side of it. Therefore, DNA-regulatory sites need not be directly in contact with the gines
that they regulate. The correct choice is B.

70. D is correct. This answer must come from one's previous knowledge of operons. The key is to realize that bacteria
prefer to use glucose as their fuel source, but they can use a whole host of other energy molecules when glucose is
not around. Therefore, the bacteria need some signal to denote when glucose is abundant or scarce. Thai signal is
cAMP. Glucose lowers the concentration of cAMP in E. coli. Therefore, we can assume that high levels of J"llulu.
cAMP are associated with low levels of glucose. The correct choice is D.
71. A is correct. The passage tells us that the arabinose operon is found
in bacterial systems. Bacteria have only one
molecule introlved in transcription, RNA polymerase. Eukaryotic organisms, on the other hand, have three types of
RNA polymerases: RNA pol I (rRNA), RNA pol II (mRNA and hnRNA), and RNA pol III (tRNA ana inNn;.
Therefore, the polymerase is RNA polymerase. The correct choice is A.
72. C is correct. If the genes for the uptake and usage of arabinose were all part of the same operon, the entire operon
would have to be transcribed before the uptake could occur. However, if the genes were separate, transcription bf the
uptake and structural genes could occur simultaneously. In this way, as soon as the arabinose entered thi organism,
there would be enzymes already present to begin its metabolism. This is certainly a more efficient means of
metabolism. The comect choice is C.

73. B is correct. The question asks which problem could not be treated using either of the two gene therapy strategies
described in the passage. The first paragraph ofthe passage provides us with an important clue: Gene therapy is u"sed
to prevent the expression of both disease-causing human genes and viral genes. The aim of both the gene therapy
strategies is somehow to inactivate a dangerous gene. These approaches would be successful in preventing t^G
overexpression of a tumor-causing oncogene (eliminating choice A). They would also be effective at preventin! the
translation of viral RNAs into DNA (thereby eliminating choices C and D). We are left with answer choice B. In

Copyright O by The Berkeley Review 4at The Berkeley Review


Specializing in MCAT Preparation
Biology Dxpression of Genetic Information Section X Answers

sickle-cell anemia, both copies of the vital gene coding for a hemoglobin subunit are mutated. The result is a defect
in erythrocyte function. The types of gene therapy mentioned in the passage could not help this condition, because
inactivation of the mutant hemoglobin gene would only make matters worse. Instead of defbctive hemoglobin,
there
would be no hemoglobin! This would not bode well for a sickle-cell patient. The correct choice is B.

74. C is correct. The hardest part about this question is understanding what it is really asking. If a plasmid is introduced
into a cell line, how could it inactivate expression of a gene that is present in the cellis chromosomes? From the
passage, we learn about the antisense approach to inactivating genes. This question, in essence, is
asking how one
might design a plasmid that would encode an antisense RNR that could bind to the sense RNA prJduced by
transcription of the chromosomal gene. Answer choices A and B both mention plasmids that contain mutant gene
sequences. This would be detrimental if we were trying to knock out the expreision of a wild-type chromosJmal
gene, because mutant RNA does not form hybrids with the wild-type transcripi. Answer choice D iin
be eliminated,
because a gene sequence separated from its promoter is not transcribed. Recall, a promoter sequence lies just
upstream of the transcribed portion of the gene. It attracts RNA polymerase and helpi initiate tranicription.
Ifthe
gene sequence is separated from the promoter.sequence, the gene might not be tranicribed. Even
if the wild-type
g_ene sequence on the plasmid were transcribed, this would not knock out the
chromosomal gene's RNA transciipt.
We are left with answer choice C. Remember that RNA polymerase can transcribe only in the S'
-+ 3'direction. If
we inveit the gene sequence on the plasmid relative to iti promoter sequence, the RNi polymerase can
no longer
read the correct strand of DNA, because this strand is now facing in the 3' -+5'direciion. Instead,
it reads ite
opposite strand, resulting in the production of antisense mRNA. ThiJantisense transcript can then
hybridize with the
sense transcript made by the chromosomal gene, thereby preventing its expression. The correct
choice is C.
75. B is correct. The question asks which statement would not represent an advantage that ribozyme therapy
would hold
over antisense therapy. We are asked to compare the two techniques. Statement i may be a tiue statem;nt
in general,
but it does not represent an advantage of ribozyme therapy, because the antisense approach doesn,t use
sinthetic
RNA--it uses synthetic DNA. Ribozymes are base-paired RNA, but the statement ^db".n't say that they are any
stabler than the synthetic single-stranded DNA used by the antisense approach. Statement IiI is also
in.or.""i
because if individual ribozymes were very broad in their substrate specificity, they would cleave a lot
of mRNAs in
the cell' not just the disease-causing ones. This could be harmful ind certiinly ioesn't represent an
advantage of
ribozyme therapy. Statement II is correct. Ribozymes act catalytically. We know this because the passage
statei that
ribozymes act "enzymatically." Enzymes are catalysts; they Can be-recycled after speeding up a reaction.
In what
regard is this an advantage that ribozymes hold over antisense DNA?Antisense DNe is-not a
catalyst. Once it
hybridizes to its target, it is stuck there for good. It might take many antisense DNA molecules to inactivate
a viral
gene, for example, while it might take only a few catalytic ribozymes to accomplish the
same task. This is an
advantage for ribozymal therapy. The correct choice is B.

76. A is correct. The Kv of an enzyme is a measure of that enzyme's affinity for its substrate. It is equal to the amount
of substrate necessary for the catalyzed reaction to proceed at half of its maximal rate, or l/2 ymax. The question
asks for the condition that would not affect the Krt. Answer choice A mentions altering the concentration
of
substrate. Recall from the Michaelis-Menten curve that changing the substrate concentration has no affect
on the Ku
of an etzyme. Therefore answer choice A is correct. What factors would affect the affinity of an enzyme for its
substrate? One possibility is the partial or total denaturation of the enzyme. An enzyme can be denatured when its
tertiary (folded) structure is disrupted. In this question, the enzyme we are dealing with is a ribozyme. The tertiary
structure of a ribozyme is dictated fundamentally by the way it base-pairs with itsell much in the same *uy u, u
protein's tertiary structure is dictated by interactions between amino acid residues and the water environment. What
factors could affect base-pairing in the RNA ribozyme? A drastic change in pH could alter hydrogen-bonding
between base-pairs. A decrease in the salt concentration of the medium *ould mean that the phosphate -backbone
oT
the base-paired RNA would be less stabilized by ionic interactions, leading to denatuiation. An increase in
temperature could melt the RNA hybrid, thereby denaturing it. These possibilities prompt us to eliminate choices B,
C, and D. The correct choice is A.

77. D is correct. From the passage, we learn that ribozymes cleave mRNAs at specific sequences. If a ribozyme could
cleave an HIV mRNA at a specific, conserved_sequence, such a ribozyme would be capable of chopping up most
mutant forms of the viral RNA as well. This is because a conserved sequence by its definition doesn'i ctra-nge much
from mutant to mutant. It may encode a very essential part of a protein product ihut .an't be altered. TherefJre, even
if the HIV virus mutates, the ribozyme could still recognize the-conserved sequence and cleave the *RNA, thereby
preventing production of mature viral particles. Antisense DNA, on the othei hand, must be complementary to
the
mRNA to which it hybridizes. If HIV mutates rapidly, a predesigned antisense strand would not be able to hybridize

Copyright @ by The Berkeley Review 4A2 The Berkeley Review


Specializing in MCAT Preparation
Biology Expression of Genetic lnformation Section X Answers

*irh nerl murant mRNA. This eliminates choices A and B. Choice C is incorrect, because if ribozymes cleaved all
mRli-{s. thel' u ouldn't be much use as a therapy at all. The correct choice is D.

78. B is correcl This is the only answer that mentions a problem that could occur only in human patients and not in a
tissue culture of human cells. Human immune response to the synthetic DNA (containing methylphosphonate) could
be a tatal side effect to some kinds of antisense gene therapy. Answer choice A is incorrect, because it describes
something that could occur both in the human patient and in the tissue culture. Answer choices C and D are incorrect
for the siune reason. The correct choice is B.

79. A is correcL This question can be answered through the process of elimination. We must bear in mind what exactly
ribozymes are capable of, and with this knowledge we can rule out functions that ribozymes could never have in
normal cells. Answer choice B is such an example. From the passage we learn that ribozymes cleave RNA at
specific sites. Nowhere are we told that it can proofread DNA. This is the function of DNA polymerase. Answer
choice C is likewise incorrect: Ribozymes do not catalyze protein reactions. Answer choice D is wrong, because
ribozymes can't cleave DNA at all; they can only cleave RNA. This leaves us with answer choice A. In order to
splice mRNA and remove introns, the mRNA strand must be cleaved at a specific site. Ribozymes are ideal for this
function, and in fact, this is how they were initially discovered. The correct choice is A.

80. C is correct. Answering this question correctly just requires one to be familiar with the concept of a mRNA that is
polycistronic. The answer cannot be gathered from information in the passage. An mRNA molecule coding for more
than one protein is known as a polycistronic transcripl. Let's consider the other possibilities: Since one protein is not
encoded for by multiple genes, choice A can easily be eliminated. Choice B wants us to believe that one protein is
coded for by a single gene. This statement is true, but it is does not pertain to polycistronic transcripts. The
definition is the relationship between mRNA and proteins, not DNA and proteins. Therefore, choice B can be
eliminated. Finally, choice D tells us that one mRNA molecule codes for only one protein. While this is not the
definition of polycistronic, this method of translation occurs in all eukaryotic organisms. The polycistronic transcript
is not seen in eukaryotic organisms. The correct choice is C.

81. C is correct. From the passage, we learn that the repressor protein alone does not cause the repression of the
tryptophan operon. The presence of tryptophan in a complex with the repressor protein brings about the true
repression. Therefore, tryptophan is best described as a corepressor. Let's consider the other possibilities: The term
inducible repressor is nonsense. It certainly has no application to the tryptophan operon and should be easily
recognized as an incorrect answer. The only example that comes to mind that resembles an inducible repressor is the
situation where glucose in a medium causes a reduction in the synthesis of the enzymes needed to utilize lactose.
This occurs in the lactose operon, and the process is termed "catabolite repression." Nevertheless, choice A does not
best describe tryptophan, and therefore choice A can be eliminated. Eliminating choice B should be easy, because it
is very clear that the presence of tryptophan causes a reduction in the expression of the operon. Therefore,
tryptophan is not acting as an inducer. In considering choice D, one needs only to consider what an attenuator is. It
is a seque-nce of DNA that offers a means of regulation through differential transcription. This is not the role of
tryptophan, and therefore choice D can be eliminated. The correct choice is C.

82. B is correct. This answer can be arrived at by looking at the diagram. It is clear from the question that the
attenuator site along with the leader sequence has been deleted from the operon. The passage tells us that these DNA
sites are involved in attenuation. Attenuation (along with the repressor-tryptophan complex) regulates tryptophan
synthesis in such a way that the operon is expressed only when tryptophan is needed. If one sees a dramatic increase
in the production of tryptophan mRNA, one can assume that this experiment provides evidence for the role of
attenuation in regulating the synthesis of tryptophan mRNA. Let's consider the other possibilities: The experiment
tells us nothing about signal transduction (the process of extracellular signals resulting in intracellular changes).
Therefore, we can eliminate choice A. Furthermore, this experiment provides no evidence for the tryptophan-
repressor complex repression. The operator/promoter is still intact, as nothing is stated about the repressor protein.
Therefore, otroice C can be eliminated. Finally, tryptophan induction is not a term defined in the passage, so the
experiment probably does not have anything to do with it. In fact, the term has no set definition, and this answer
choice can be easily eliminated. The correct choice is B.

83. B is correct. The tryptophan-repressor protein binds to the operator and prevents transcription. Therefore, it
prevents the interaction not between DNA polymerase and DNA (choice A), but between RNA polymerase and

Copyright @ by The Berkeley Review 4A3 The Berkeley Review


Specializing in MCAT Preparation
Biology Dxpression of Genetic Infonnation Section X Answers

DNA. So the question becomes: Which RNA polymerase? The tryptophan operon is found in E. coli, a bacterium.
Bacteria have only one type of RNA polymerase, known simply as RNA polymerase. Choice B is the best answer.
Eukaryotes, on the other hand, have more than one type of RNA polymerase. In fact, they have three types. The
correct choice is B.

84. D is correct. During attenuation, it is important for the leader peptide to be translated. That is how the system
detects the level of tryptophan in the cell. If the codon UGG is there, but there is a lack of tryptophanyl IRNA, this
causes a pause and a conformational change in the RNA polymerase. That change results in the RNA polymerase
continuing to transcribe the operon. If there is no pause, the RNA polymerase falls off the DNA. Therefore, one can
see that it is important that tianscription and translation be closely coupled. Based on this information, the other
answers can be easily eliminated. One can see that replication of DNA has little to do with attenuation, but
attenuation regulates the cellular levels of tryptophan. The correct choice is D.

85. D is correct. The attenuator site, according to the passage, contains a G-C rich region and an A-T rich region.
Therefore, melting the piece of DNA (turning double-stranded DNA into single-stranded DNA) does not occur
abruptly at one ternperature, because there is a varied nucleotide composition. The melting can be described as
discontinuous. This eliminates choices A and B. The next decision comes from knowing that it takes less energy to
melt an A-T rich region when compared to a G-C rich region. This, of course, stems from knowing that there are
two hydrogen bonds between adenine and thymine, and three hydrogen bonds between guanine and cytosine. The
correct choice is D.

86. C is correct. Let us look at the example in the passage. The leader peptide of the tryptophan operon contains
tryptophan residues, because that is how the system is designed to measure the level of the amino acid. Therefore,
thile;der peptide of any operon should contain a relatively large number of residues of the kind synthesized by the
operon. The operon should not contain tandem pairs of the codon UGG, unless it is the tryptophan operon.
Tierefore, choiie A is incorrect. If the operon has a relatively small number of the UGG codons, it is most likely by
chance and not design. In other words, a leader peptide does not have to contain a small number of the UGG codons.
Therefore, choice B can be eliminated. Again, to detect the level of a particular amino acid in the cell better, the
leader peptide of an operon should contain a relatively large number of the residues of the kind synthesized by the
operon. The conect choice is C.

87. B is correct. Remember, the residues in the leader peptide are the detection system of the operon. Increasing the
number of those residues (the kind synthesized by the operon) would detect the amino acid level in the cell better.
This automatically eliminates choice A. There is no evidence for either choices C or D. Attenuation is clearly not
eliminated by incieasing the number of residues in the leader peptide. In addition, the change in residues results in
more thanjust a change in structure, but also a change in function. The correct choice is B.

88. B is corqgct. The passage informs us that in reticulocytes (eukaryotic cell), eIF-2 is involved in controlling the
overall rate of prot.in synthesis. In addition, we are told that phosphorylation of this protein reduces its activity and
thus most likeiy reduces the overall rate of protein synthesis. Under starving conditions, one can conclude that
protein synthesis in the cell decreases. Therefore, phosphorylation of eIF-2 would occur. This requires the work of a
kinut". One assumption should be made here: Yes, the passage does specifically state "reticulocytes," while the
question asks abouichanges in the more general term "eukaryotic cell." However, we must use what information the
purrug" gives us, as thi; type of questioning can appear on the MCAT. Therefore, we would expect increased
activity from a kinase. The correct choice is B.

89. B is correct. We are told in the passage that roughly I out of every 10,000 amino acids incorporated into proteins is
inserted incorrectly. Then the question says that the average size of a protein is 400 amino acids. Therefore,
10,000/400 = 25. One out of every 25 protein molecules should contain an error. The correct choice is B.

90. B is correcL According to the passage, a polyribosome is a single mRNA transcript that has multiple ribosomes
attached. Comparing a single ribosome to a polyribosome, we can clearly see a difference in size, with the
polyribosome beinglarger. We know that centrifugation takes advantage of this situation, based on the fact that
iarge molecules move faster than smaller ones in a gravitational field. Therefore, centrifugation would allow us to
."udily separate these two kinds of ribosomes in solution readily. The correct choice is B.

Copyright @ by The Berkeley Review 4A4 The Berkeley Review


Specializing in MCAT Preparation
Biology Expression of Genetic Information Section X Answers

91. D is correct. This question must be answered from a previous knowledge base. It is asking what amino acid is called
for by the start codon in a prokaryotic cell. The answer is a modified methionine. In particular, a formyl group is
attached to make a formylmethionine. In a eukaryotic cell, the start codon calls for methionine. Since the question
asks for the prokaryotic cell, formylmethionine is correct. The correct choice is D.
ot C is correct. This question requires previous knowledge. A polycistronic transcript contains multiple start codons.
Each start codon results in the translation of a particular protein, each with a unique structure and function. This is in
contrast to a monocistronic transcript, which contains only one recognized start codon, which gives rise to only one
peptide chain. Polycistronic transcripts are often found in prokaryotic cells, while monocistronic transcripts are often
found in eukaryotic cells. The correct choice is C.

93. C is correct. A proofreading mechanism implies preventing and/or removing an incorrect unit so that a correct unit
can be inserted. Therefore, our answer should address this issue. The only choice that satisfies this criterion is
choice C. Choice A is not the best answer, as it implies that time alone assures that the correct aminoacyl transfer
RNA eventually will be inserted. There is no explanation about any proofreading function. Indeed, there are
hydrogen-bond interactions between the bases in the codon and the anticodon. If there is an incorrect bonding, the
dllay makes it very difficult for the incorrect pairing to hold, as such a situation is inherently unstable. The delay
allows for the diisociation of this unstable complex, and the correct aminoacyl transfer RNA can then be
incorporated. The correct choice is C.

94. B is correct. Choice B claims that no initiating factors are needed for secreted proteins. Recall that under the signal
peptide hypothesis, a protein that is to be secreted begins its translation in the cytosol and then is attached to the
endoplasmic reticulum to continue and finish its translation. The key point here is that translation begins in the
cytosol, just like a protein that is not to be secreted. This requires formation of a competent ribosome, which
according to the passage, involves initiating factors. Therefore, we cannot make the claim that initiating factors are
not needed, because they clearly are. The correct choice is B.

9s. C is correct. The passage states that in theory, one could compare the base sequences of a normal cell and a
cancerous cell. In thiory, the differences one would encounter could be attributed to cancer. However, this is not the
case. Even if it were practical to sequence both genomes, we could not ignore the fact that mutations naturally occur.
Therefore, there would be more than just one difference, and one probably could not determine which mutation was
the cause of cancer. Therefore, naturally occurring mutations would mask the cancer-causing gene. The correct
choice is C.

96. C is correct. In the passage, it is clearly stated that the screening process involves using a sequence of DNA
complementary to the tagging sequence. We want this probe to hybridize only with the tagging region (which is
tropetutly uttuih"d to our mutated cancer gene), and not with other pieces of DNA. If we used a tag sequence that
was normally found in E. coti, the probe would hybridize to regions of DNA other than the one we are interested in.
This would iho* up as many false positives, and our isolation technique would be unsuccessful. The correct choice
is C.

9-7. C is correct. The tagging sequence is hopefully near the cancer-causing mutated gene we are interested in isolating.
From the passage, *e i." told that the Alu site is found throughout the entire human genome. This is a very
attractive tiait as a tag, because we are betting that because the site is so widespread throughout the chromosomes, it
has a solid chance of Ueing near our mutated gene of interest. Therefore, when we clone and screen, we will
hopefully include this mutated gene. The correct choice is C.

98. C is correct. This question is straightforward. We know from the question that gene A is known to induce cancer.
Also, from the information in the question, we can conclude that there is nothing wrong with the transcription of the
gene or the efficiency of translating the messenger RNA. Therefore, we should conclude that even though the
frotein is syhthesized, it is useless. Recall that protein function is tied into its structure. Therefore, a nonfunctional
protein probably suffers from a significant structural change that resulted from the gene mutation. The correct
choice is C.

Copyright O by The Berkeley Review 4a5 The BerkeleY Review


Specializing in MCAT Preparation
Biology Expression of Genetic Information Section X Answers

99. B is correct. Two strands of DNA hybridize because they have complementary bases. Recall, that bases bind to each
other because of hydrogen bonds. Adenine and thymine share two hydrogen bonds, while guanine and cytosine
share three hydrogen bonds. Therefore, the hybridization occurs because base pairs do form hydrogen bonds with
each other and create a stable structure. Without the hydrogen-bond interaction, the two strands would not anneal.
The correct choice is B.

100. A is correct. This question must be answered from a previous knowledge base. A Southern transfer is as follows:
Using electrophoresis, DNA fragments are distinguished on the basis of their size. The fragments are transferred to
some sort of membrane and are immobilized. Under hybridizing conditions, labeled oligonucleotides are added and
complementary bases form pairs. The label can then be found with autoradiography. Northern transfers involve
RNA, while Western transfers involve proteins. Although the molecules are different for these experimental
methods, the concepts are very much the same. The correct choice is A.

Copyright @ by The Berkeley Review 4a6 The Berkeley Review


Specializing in MCAT Preparation
Biotogy Glossary MCAT Words and Phrases

Glossary
Most of the items listed in this glossary are important words or phrases that have appeared in one of the
following publications of the Association of American Medical Colleges (AAMC):

1,. MCAT Practice Test I


2. MCAT Practice Test II
3. MCAT Practice Test III
4. MCAT Practice Test IV
5. MCAT Practice Items (Biological Sciences)

These words or phrases are specific to the Biological Sciences portion of the MCAT. We have chosen them based
on student response, and on how important we feel they are in building a good conceptual understanding of the
information basic to all biological sciences.

A Alanine (Ala)
o-ketoglutaric acid Albumin
ABO blood system Alcohol
Absorption Alcohol dehydrogenase
Acetaldehyde Aldehyde
Acetic acid Aldosterone
Acetylcholine Allele
Acetylcholinesterase Allergy, allergic reaction
Acetyl-CoA Alveolus, alveoli (p1.)
Acid, acidic Ames test
Acidophil, acidophilic Amino acid
Acquired immune deficiency syndrome (AIDS) Ammonia
Acrosome Amoeba, amoebae or amoebas (pl.)
Actin Amphoteric
Action potential Ampicillin
Active transport Anabolism, anabolic
Adenine Anabolize
Adenosine Anaerobe, anaerobic
Adenosine diphosphate (ADP) Androgen
Adenosine triphosphate (ATP) Aneuploidy
Adenylate cyclase Angiotensin I
Adipose cells Angiotensin iI
Adrenal cortex Angiotensin-converting en4,"rne (ACE)
Adrenal gland Angiotensinogen
Adrenaline Aniline
Adrenocorticotropic hormone (ACTH) Antagonist
Adsorption Antibiotic
Aerobe, aerobic Antibody
Afferent fiber !' Antidiuretic hormone (ADH)
Afferent vessel Antigen, antigenic
African sleeping sickness Antigenic shift
Agar Antihistamine
Agglutination Antitoxin
Agonist Aorta

Copyright @ by The Berkeley Review 4A7 The Berkeley Review


Specializing in MCAT Preparation
Biology GIossar5r MCAT Words and Phrases

Apocarboxylase Camphoric acid


Aqueous base Capillary
Arginine (Arg) Capillary bed
Artery, arterial Capsule
Asparagine (Asn) Carbaminohemoglobin
Aspartic acid (Asp) Carbohydrate
ATP synthetase Carbonic anhydrase
ATPase Carboxyl
Atrium, atria (pl.) Carboxylase
Atrophy Carboxylate, carboxylation
Atropine Carboxylic acid
Autoimmune disorder Carcinogen, carcinogenic
Autonomic nervous system Cardiac output
Autosomal Cardiac stroke volume
Autosomal dominant Cardiopulmonary circulation
Autosomal recessive Carrier
Auxotroph Cartilaginous Haversian canal
Axial Casein
Axon Catabolism, catabolic
Azidothymidine (AZT) Catabolize
Catalyze
B Cell membrane
p-galactosidase Cell wall
B lymphocyte Cellulose
Back-mutate Central nervous system (CNS)
Bacterium, bacteria (p1.) Centrifuge
Base Centriole
Base pair Cerebellum
Basement membrane Cervix, cervical
Basic Channel blocker
Basophili, basophilic Cholesterol
Cholic acid
Bicarbonate ion (HCO3-)
Chordate
Binary fission Chromatid
Biocytin Chromatin
Biotin Chromatography
Biotinidase Chromosome
Blood pH Chylomicron
Blood platelet Chymotrypsin
Bone deposition Chymotrypsin mechan ism
Bone marrow Cilium, cilia (pl.)
Bone remodeling Circular DNA
Bone resorption Circular muscle
Brain Cisterna, cisternae (p1.)
Bronchiole Citric acid cycle
Buffer Class
Clostridium tetani
ci! Codominant
Calcitonin Color blindness
Calcium carbonate Committed stem cell
Calcium caseinate Congenital
Calmodulin Conjugate, conjugation
Calvin cycle Connective tissue

Copyright @ by The BerkeleY Review The BerkeleY Review


Specializing in MCAT PreParation
Biology Glossary MCAT Words and Phrases

Constitutive D r osophila mel ano gaster


Contractile Ductus arteriosus
Conus arteriosis Duodenum
Coprophagy
Cortex, cortical E
Cortisol Early gene
Creatine phosphate Ectopic pregnancy
Cross-bridge Efferent fiber
Crossover Efferent vessel
Cryoprotection Electrical potential
Cutaneous Electron transport
Cyclic AMP (cAMP) Electron-withdrawing group
Cysteine (Cys) Electrophile, electrophilic
Cystic fibrosis (CF) Elution
Cytochrome b Embryo
Cytochrome oxidase Embryogenesis
Cytokine Embryonic
Cytoplasm Encode
Cytosine Endocrine system
Cytoskeleton Endocytosis
Cytosol Endocytotic vesicle
Endogenous
D Endoplasmic reticulum
Dalton Endothelium, endothelia (p1.)
Dark reactions (Calvin cycle) Endotoxin
Daughter cell Enol
Daughter strand En4zme
Decarboxylation Epimerize
Dehydration Epinephrine
Dehydrogenase Epithelial
Deletion mutation Equatorial
Dendrite Erythrocyte
Deoxygenation Escherichia coli
Deoxyribonucleic acid (DNA) Essential amino acid
Deoxyribonucleoside Ester
Dephosphorylation Esterification
Depolarization * Estradiol
Depolymerization Estrogen
Deprotonation Ethanol
Deterministic Ether
Deuterium Eukaryote
Diastolic blood pressure Excision
Differentiation Excitatory
Digestion Excrete
Dilate Exocrine system
Diploid Exocytosis
Distal tubule Exogenous
DNA recombination Exon
DNA repair Exonuclease
Dominant External respiration
Dopamine Extracellular medium
Dorsal root ganglion
Down syndrome (trisomY 21)

Copyright O by The BerkeleY Review 449 The BerkeleY Review


Specializing in MCAT PreParation
Biotogy Glossary MCAT llllords and phrases

F Glycoside
Facilitated diffusion Golgi apparatus
Facultative anaerobe Golgi body
FAD Gonorrhea
FADHz Gradient
Fallopian tube Gramicidin
Familial hypercholesterolemia Gram-negative
Family Gram-positive
Fatty acid Growthhormone (GH)
Feedback-inhibited Guanine
Feminization Guanosine
Fermentation Guanosine diphosphate (GDp)
Fibrin Guanosine triphosphate (GTp)
Fibrinogen
Fibroblast H
Fightor-fl ight response Habituation
First filial (Fr) generation Haldane effect
Follicle-stimulating hormone (FSH) Haploid
Formaldehyde Heart contraction
Frameshift mutation Heart rate
Fungus, fungi (pl.) Heatstroke
Helper T cell (helper T lyrnphocyte)
G Hematopoiesis, hemopoiesis
G protein Hematopoietic stem cell (HSC)
Galactose Heme
Ganglion, ganglia (pl.) Hemoglobin (Hb)
Gastric enzyrne Hemolysis
Gastrointestinal tract Hemophilia
Gastrulation Hemophilia factor VIII
Gel chromatography Hemorrhage
Gel electrophoresis Hepatic
Gene Hepatitis
Genetic engineering Heterotropic
Genital, genitata (pl.) Heterotrophic
Genome Heterozygous
Genotype Hibernation
Genus, genera (plJ High-density lipoprotein (HDL)
Germ cell Hippuric acid
Germ cell layer Histamine
Glopin Histidine (His)
Glomerular filtrate Histidine-deficient
Glomerular filtration Histone
Glomerulus, glomeruli (p1.) Holocarboxylase slmthetase
Gluconeogenesis Homeostasis
Glucose Homogenized
Glucose-6-phosphate Homozygous
Glutamic acid (Glu) Hormone
Glutamine (Gln) -.* Host cell
Glycerol Human chorionic gonadotropin (hCG)
Glycine (Gly) Huntington's chorea
Glycogen Hybrid
Glycolysis Hydration
Glycoprotein Hydroboration

Copyright @ by The Berkeley Review 49o The Berkeley Review


Specializing in MCAT Preparation
Biology Glossary MCAT Words and Phrases

Hydrolysis K
Hydrophilic Kanamycin
Hydrophilic amino acid Ketone
Hydrophobic Killer T cell (killer T lymphocyte)
Hydrophobic amino acid Kinetochore microtubule
Hydroxyl Kingdom
Hydroxylation Krebs cycle
Hydroxyprogesterone
Hyperglycemia L
Hyperosmotic Lactalbumin
Hyperplasia Lactase
Hyperpolarization Lactation
Hypertension Lacteals
Hypoglycemia Lactic acid
Hypotension Lactoglobulin
Hypothalamus, hYPothalamic Lactose
Hypovolemic shock Late gene
Left ventricle
I Lethal mutation
Imidazole ring Leucine (Leu)
Immune reaction Leukocyte
Immune response Lewis acid
ImmunitY Lewis base
Immunization Ligament
immunoglobulin Ligand
Imprinting Light reactions
Inhibitory Lipid
Innervation Lipid bilayer
Insulin Lipid-soluble
Integration Lipophilic
Intermolecular Lipophobic
Internal resPiration Lipopolysaccharide
Interstitial fluid Lipoprotein
Intestinal goblet cell Liver
Intestinal wall Locus,loci (pl.)
Intracellular medium Logarithmic microbial growth
Tntramoleculat Loop of Henle
Intrauterine Low-densitY liPoProtein (LDL)
Intravenous infusion Lumbar
lntron Luteinizing hormone (LH)
Invariant Lymphatic sYstem
Ionic LymphocYte
Ionic imbalance Lysine (Lys)
Isoleucine (Ile) Lysis
Isomeric Lysogen, lYsogenic
Isotonic Lysosome

M
I Macrophage
Juxtaglomerular aPParatus
Mammal
]uxtaglomerular cells
Medulla oblongata
Meiosis, meiotic

491 The BerkeleY Review


The BerkeleY Review
Copyright @ bY
Specializing in MCAT PreParation
Biology Glossary MCAT Words and phrases

Membrane-bound enzyme Nonessential amino acid


Membrane-bound receptor Nonkinetochore microtubule
Menopause Nonsense mutation
Menstruation Norepinephrine
Messenger RNA (mRNA) Nuclear membrane
Metabolism, metabolic Nucleic acid
Metabolize Nucleophile, nucleophilic
Methanol Nucleoside
Methionine (Met) Nucleotide
Microbe Nutrient
Microcirculation Nutrition
Microfilament
Microtubule o
Mitochondrial DNA (mtDNA)
Obligate aerobe
Mitochondrion, mitochondria (pl.)
Obligate anaerobe
Mitosis, mitotic
Oleic acid
Molec u la r hyb rid i zation
Oncogene
Monogastric
Opiate
Monosaccharide
Optical antipole
Morphine
Order
Morphology
Organelle
Mosaic hypothesis
Osmoregulatory
Motiiity Osmosis, osmotic
Motor neuron
Osteoblast
Mucosa
Osteoclast
Mucus (n.), mucous (adj.)
Osteocyte
Multiple genes Osteoporosis
Mutagenic
Ovary
Mutation Oviduct
Mr1 c ob a c t erium tub eruli sis Ovulation
Myelin Ovum, ova (p1.)
Myoglobin Oxidation
Myosin Oxidative phosphorylation
Myosin light chain
Oxidizing condition
Myosin light chain kinase (MLCK) Oxygenation
Myxoairus influenzae
Oxytocin

N P
Na+/6+ -ATPase Pso
Na+/K+ pump Pancreas, pancreatic
NAD+ Parasympathetic nervous system
NADH Parathyroid hormone (PTH)
Natural killer cell Parent cell
Negative feedback Parent strand
Nephron Parental (P) generation
Neuroeffector Parkinson's disease
Neuron Partial pressure
Neurosecretory : Passive diffusion
Neurotransmitter Pathogen, pathogenic
Neutrophil Penicillin
Nitrogenous waste Pepsin
Nitrous oxide Peptidase
Peptide

Copyright O by The Berkeley Review 492 The Berkeley Review


Specializing in MCAT preparation
Biology Glossary MCAT Words and Phrases

Peptide bond Prototroph


Peptidoglycan Protozoon, protozoa (pl.)
Peripheral nervous system (PNS) Proximal tubule
Peritoneum Pulmonary artery
pH optimum Pupillary light reflex
Phage progeny Purine
Phenol Pyridine
Phenotype Pyrimidine
Phenylalanine (Phe) Pyrrolidine
Phosphatase Pyruvic acid
Phosphatidylcholine
Phosphoprotein a
Phosphorylation Quinone
Photophosphorylation
Photosvnthesis R
Phylum, piryla (pl.)
Racemize
Physostigmine
Radial muscle
Pigment
Radioimmunoassay (RIA)
Pilomotor
Rana sylaatica
Pilus, pili (p1.)
Rate-determining step
Pineal gland
Reabsorption
Pinealectomr-
Receptor
Piperichne
Recessive
Pituitarv gland Recombination
PKo Red blood cell (RBC)
Placebo effect
Reduction
Plasma
Regulative hypothesis
Plasma clearance
Renal
Plasma mernbrale
Renin-angiotensin system
Plasma osmolaritv
Replication
Plasmid
Repolarization
Plateau
I
Reproductive tract
Pluripotent stem cell Resonance
Point n'Lutatior-r
Respiratory tract
Polar
Reticulum, reticula (pi.)
Polarized Retina
Polvmerize, polvmerization Retrovirus
Pol;,'peptide Reverse transcriptase
Polvsaccharide Rhesus factor
Positir-e ieedback Rh-negative
Postmenopausal Rh-positive
Postsvnaptic membrane Ribonucleic acid (RNA)
Premature stop codon Ribonucleoside
Preprotein Ribose
Presvnaptic membrane Ribosomal RNA (IRNA)
Primarv structure Ribosomes, eukaryotic (40S, 50S, 80S)
Probe
Ribosomes, prokaryotic (30S, 50S, 70S)
Progesterone . Ribozyme
Prokarvote Right ventricle
Proline (Pro) RNAase
Prophage Rough endoplasmic reticulum (RER)
Proteolytic Rumen
Protonation

Copyright @ by The Berkeley Review 493, The Berkeley Revieu'


Specializing in MCAT Preparation
Biology Glossary MCAT llllords and Phrases

S Stereochemistry
S acchar omy ces c er eaisiae
Stereoselective
Sacral Stochastic
Salivary gland Stomach acidity
S almonelln typhimur ium
Stop codon
Sarcomere Streptomycin
Sarcoplasmic reticulum Striated muscle
Saturated hydrocarbon Strict aerobe
Second filial (Fz) generation Strict anaerobe
Secondary structure Subspecies
Secrete Succinic acid
Sucrose
Secretory protein
Semiconservative replication Support cell
Seminal vesicle Sweat gland
Seminiferous tubule Symbiotic
Sensory neuron Sympathetic nervous system
Sepsis Synapse
Septic shock Synapsin
Septicemia Synaptic transmission
Serine (Ser) Synthesis, synthetic
Serotonin Synthesize
Serum, sera (pl.) Systemic artery
Sex pilus Systemic circulation
Sex-linked trait Systemic vein
Sickle-ceil anemia Systolic blood pressure
Signal hypothesis
Signal peptidase T
Signal peptide T cell
Signal recognition particle (SRP) Tautomerism
Simple diffusion Tendon
Sinus venosus Tertiary strucfure
Skeletal muscle Testis, testes (pl.)
Smallpox Testosterone
Smooth endoplasmic reticulum (SER) Tetanospasmin
Smooth muscle Tetanus
Sodium-potassjum pump (Na+/K+ pump) Thermogenesis
Solubility Thermoregulation
Species Thorax, thoracic
Threonine (Thr)
$pectrophotometry
Spectroscopy Thrombin
Sperm Thymine
Spermatid Thymus
Spermatocyte, primary and secondary Thyroid gland
Spermatogonium, spermatogonia (p1.) Thyroid-stimulating hormone (TSH)
Spermatozoon, spermatozoa (P1.) Thyroxine
Spinal cord Totipotent stem cell
Spleen -.:
Toxin, toxic
Splicing Trans
Spontaneous mutation Transcription
Spore Transcriptional regulation
Start codon Transfer RNA (IRNA)
Stearic acid Translation
Stem cell Triglyceride

Copyright @ by The BerkeleY Review 49,4 The BerkeleY Review


Specializing in MCAT PreParation
Biology Glossary MCAT Words and Phrases

True-breeding variety X
Trypanosome X chromosome
Trypanosomiasis
Tryptophan (Trp) Y
Tubal pregnancy Y chromosome
Tubular load
Tubular transport maximum (T6) z
Tubulin
Zygote
Tumer's syndrome
Zymogen
Tyrosine (Tyr)

U
Ubiquinone
Uracil
Urea
lJrease
Uric acid
Uridine triphosphate (UTP)
Urine
Uterus, uterine

V
Vaccine
Vaccinia r-irus
Vagus nen-e
Valine (Val)
ValinomJ-cin
Variable surface glr-coproteins
Vascular smooth m'.r-.cle
Vasoconstriction
Vasodilation
Vasopression
Vein, venous
Vena cava
Ventricle
Vertebra, vertebrae ipJ.. r

Vertebrate
Very low-densiti- lipoprotein (VLDL)
Villus, villi (p1.)
Viral envelope
Viral genome
Virion
Virulence
Virus
Viscosity
Visual centers
Vitamin D

w
Water-soluble
White blood cell (\\BCl
Wild type

Copyright @ by The Berkelel Revie,.r 495 The BerkeleY Review


Specializing in MCAT Preparation
Biology Section Answers

Biology Bubble Sheet


(Make five copies, one for each section of the book.)

l. @@o@l 37. @@ €>@l zs.oo@@


@@ .i@@l
l
,,
@@@@ 38. 74.'@ @ @ @
3. @@o@ l
l
39. @@ (o@
/-\ /,F\ l

7s. @ @ @ @
4. @: @o@] 40. @@ @@ 76. @ @ @ @
5. @G)(oq_l l
41. @@ @@ 77. @ @ @ @
6. @@@@ 42. @@ @@ 78.@@@@
7. @@@@ 43. @@ o@ 7e.@@@@
8. @@@@ 44. @ @@@ 80.@@@@
9. @@@@ 45. @ @@@ 8t.@@@@
10. @@@@ 46. @ @@@ 82. @ @ @ @
11. @@@@ 47. @ @@@ ffi.I@@o@
12. @@@@ 48. @ @@@ 84.@@@@
13. @@@@ 49. @ @@@ 8s.@@@@
14. @@@@ 50. @ @@@ 86.@@o@
15. @@@@ 51". @ @@@ 87.@@@@
16. @@@@ 52. @ @o@ 88. @@@@
L7. @@@@ 53. @ @@@ 89. @@@@
18. @@ @ @ 54. @ @@@ 90. @@@@
19. @@ @ @ @ @o@ 9r. @@@@
20. @@ @ @ 56. @ @@@ 92. @@@@
2r. @@ @ @ 57. @ @@@ 93. @@@@
)) @@ @ @ 58. @ @@@ 94. @@@@
23. @@ @ @ 59. @ @@@ 95. @@@@
24. @. @ @ @ 60. @ @@@ 96. @@@@
25. @@ @ @ 61. @ @@@ 97. @@@@
26. @@ @ @ 62. @ @@@ 98. @@@@
27. @@ @ @ 63. @ @@@ 99. @@@@
28. @@ o @ 64. @ @@@ 100. @@@@
29. @@ @ @ 65. @ @@@
30. @@ @ @ 66. @ @@(Et
31. @@ @ @ 67. @ @@@]
a) @@ @ @ 68. @ @@@l
33. @@ @ @ 69. @ @o@ Raw
34. @@ @ @ 70. @ G>(e)@i [] Score
1( @@ @ @ 7L. @ @@@l I Estimated
36. @@ @ @ 72. @ @(o@l I Scaled Score

Copyright @ The Berkeley Review 496 The Berkeley Review


Specializing in MCAT Preparation
EKI{ELEY
R.D.V.I.E.W'"

PERIODIC TABLE OF TTIE ELEMENTS

2
He
-1.0

3 5 6 7 8 910
lri Be B c N o F\C
i 19.0
I

6.9 9.Cr 10.8 12.O 14.0 16.0 20.i


1l i i: 13 14 15 16 t7 18
Na | \tg AI Si P S Cl]r.
23.0 : l-1 ,i 27.0 28. I 31.0 32.1 :s.s I re q
lQ : lir a5t)6 27 28 29 30 31 32 JJ 34 35 ,i6
TI i c; lt'i;; Co Ni Cu Zn Ga Ge As Se Br Itu
n-i -<t, 5J.9i55.8 s8.9 58.7 63.5 65.4 69.7 72.6 74.9 '79.0 79.9 8-r,S

37irs -r_ ': -1,i i 44 45 46 47 48 49 50 5l 52 53 51


Rb Sr \h \Io Tc i Ru i
Rh Pd Ag Cd In Sn sb Te IXe
85.5 I S- 6 ;:; r:" 9_li 1101_l 102.9 106.4 r0't.9 112.4 114.8 118.7 121.8 t2'7.6 126.9 1_I l
_<6
--t -,5 : ,16 77 78 79 80 81 82 83 84 85 86
Ba Tr SReOs lr Pt Au Hg TI Pb Bi Po At Rn
l l- _l i,i, : , ir i 155: 190.2 192.2 I 95.1 t97.0 200.6 204.4 207.2 209.O (209) (2t0) (222t
87is. - ": , "r I -r- ltrS 109 110 lll 112
Fr Rr Eho' _(E Bh tls llt Uun Uuu Uub
(223r lit - --: -:- -f':l (266) (269) (2't2) (2'77)

:i ;.- :. :. 6l 63 64 65 66 67 68 69 70 7l
tfc Pr \d Pm Sm Eu Gd Tb Dy Ho Er Tm Yb Lu
.l{" I ' 1< I<
4: .l,1: l -i1,0 r57.3 158.9 t62.5 164.9 t67.3 168.9 173.0 175.0
"".",...----'-Tt-
t: i-l 95 96 97 98 99 100 101 l02 103
ll'n p\1 [- \p Pu -{m Cm BK Cf Es Fm Md No Lr
:,: - --:-1 r 'Jf r (217) (247) (2st) (2s2) (2s7) (2s8) (2se) (260)

Specializing b !rc"[T FlurymMdixum

Você também pode gostar